Sie sind auf Seite 1von 558

1

L
L
i
i
s
s
t
t
a
a
s
s
2
2
0
0
1
1
3
3

3
3

A
A
n
n
o
o

E
E
n
n
s
s
i
i
n
n
o
o
M
M

d
d
i
i
o
o


Conhea:


Av. Liberdade nr. 1060, Conjunto Riviera Goinia GO
Fone: (62) 3273-8084

M Ma ai is s q qu ue e u um ma a e es sc co ol la a. . U Um m j je ei it to o d de e s se er r. .

Colgio Metropolitano
Rua X-13, n 60, Jardim Brasil Goinia GO
Fone: (62) 3542-2746
Email: metropolitano@metropolitano-go.com.br
Site: www.metropolitano-go.com.br

2







PROF.: LUCIANA SEBA CAPUZZO






C
C
o
o
n
n
t
t
e
e

d
d
o
o
s
s
:
:

c
c
a
a
p
p

t
t
u
u
l
l
o
o
1
1

E
E
s
s
p
p
a
a

o
o
e
e
r
r
e
e
p
p
r
r
e
e
s
s
e
e
n
n
t
t
a
a

o
o
:
:
a
a
l
l
i
i
n
n
g
g
u
u
a
a
g
g
e
e
m
m

d
d
a
a
G
G
e
e
o
o
g
g
r
r
a
a
f
f
i
i
a
a

c
c
a
a
p
p

t
t
u
u
l
l
o
o
2
2

A
A
s
s
t
t
e
e
r
r
r
r
a
a
s
s
e
e
m
m
e
e
r
r
s
s
a
a
s
s
e
e
a
a
s
s
i
i
m
m
p
p
l
l
a
a
n
n
t
t
a
a

e
e
s
s

h
h
u
u
m
m
a
a
n
n
a
a
s
s

c
c
a
a
p
p

t
t
u
u
l
l
o
o
3
3

O
O
s
s
c
c
l
l
i
i
m
m
a
a
s
s
e
e
a
a
s
s
s
s
o
o
c
c
i
i
e
e
d
d
a
a
d
d
e
e
s
s


M
M
i
i
n
n
i
i
s
s
t
t
r
r
a
a
d
d
o
o
s
s
d
d
u
u
r
r
a
a
n
n
t
t
e
e
o
o
m
m

s
s
d
d
e
e
f
f
e
e
v
v
e
e
r
r
e
e
i
i
r
r
o
o














D Da at ta a d da a E En nt tr re eg ga a : : _ __ __ __ __ __ __ __ _/ /_ __ __ __ __ __ __ __ __ __ __ _/ /2 20 01 13 3
LISTA 1 GEOGRAFIA

3

1. (FUVEST/2011)



Um viajante saiu de Araripe, no Cear, percorreu, inicialmente, 1000 km para o sul, depois 1000 km para o oeste e, por
fim, mais 750 km para o sul. Com base nesse trajeto e no mapa acima, pode-se afirmar que, durante seu percurso, o
viajante passou pelos estados do Cear,
a) Rio Grande do Norte, Bahia, Minas Gerais, Gois e Rio de Janeiro, tendo visitado os ecossistemas da Caatinga,
Mata Atlntica e Pantanal. Encerrou sua viagem a cerca de 250 km da cidade de So Paulo.
b) Rio Grande do Norte, Bahia, Minas Gerais, Gois e Rio de Janeiro, tendo visitado os ecossistemas da Caatinga,
Mata Atlntica e Cerrado. Encerrou sua viagem a cerca de 750 km da cidade de So Paulo.
c) Pernambuco, Bahia, Minas Gerais, Gois e So Paulo, tendo visitado os ecossistemas da Caatinga, Mata Atlntica e
Pantanal. Encerrou sua viagem a cerca de 250 km da cidade de So Paulo.
d) Pernambuco, Bahia, Minas Gerais, Gois e So Paulo, tendo visitado os ecossistemas da Caatinga, Mata Atlntica e
Cerrado. Encerrou sua viagem a cerca de 750 km da cidade de So Paulo.
e) Pernambuco, Bahia, Minas Gerais, Gois e So Paulo, tendo visitado os ecossistemas da Caatinga, Mata Atlntica e
Cerrado. Encerrou sua viagem a cerca de 250 km da cidade de So Paulo.

2. (UFG-2012/1) Analise a figura e o texto apresentados a seguir.

Atualmente existem trs categorias de equipamentos GPS em uso: o recreacional (ou navegador), o topogrfico e o
geodsico. Para os dois ltimos, necessrio processar as informaes antes de us-las.
Considerando-se o exposto a respeito desse recurso tecnolgico,
a) caracterize o funcionamento do sistema GPS (Global Positioning System);
b) indique duas informaes que podem ser obtidas por meio de um aparelho GPS.

Caderno de Atividades

Disciplina:
G Ge eo og gr ra af fi ia a

Professor(a):
Luciana Capuzzo
Aluno:
3 ano
Ensino Mdio
Data de Recebimento:
_____/_____/_____
Lista 01
Data Entrega:
_____/_____/_____

4
3. (UFG/08) Para atingir o objetivo de ler e interpretar mapas, o leitor necessita de identificar e analisar os elementos
de representao cartogrfica. Dentre esses, a escala cumpre um papel importante, visto que a partir dela que se
tem
(A) a localizao de um fenmeno na superfcie terrestre.
(B) a apresentao da superfcie esfrica no plano.
(C) os diferentes fusos horrios no globo.
(D) a identificao dos diferentes hemisfrios terrestres.
(E) o nvel de detalhe das informaes representadas.
(UEG/2010-1) Observe o grfico abaixo. Considerando que o eixo X corresponde Linha do Equador e o eixo Y
corresponde ao Meridiano de Greenwich, responda s questes 4 e 5.

4. Identifique as coordenadas geogrficas correspondentes, respectivamente, aos pontos B e A:
a) 30 de Lat. Sul e 45 de Long. Leste; 90 de Lat. Sul e 60 de Long. Leste
b) 45 de Lat. Norte e 30 de Long. Oeste; 90 de Lat. Sul e 60 de Long. Leste
c) 30 de Lat Norte e 45 de Long. Oeste; 60 de Lat. Sul e 90 de Long. Leste
d) 30 de Lat. Sul e 45 de Long. Leste; 60 de Lat. Norte e 90 de Long. Leste

5. Considerando que no ponto A so 14 horas, calcule o horrio local do Ponto B. Em sua resposta, desconsidere a
possibilidade da existncia de horrio de vero e de horas cifradas:
a) 20 horas
b) 18 horas
c) 17 horas
d) 8 horas

6. (UEG/2010-1) No mapa do municpio de Anicuns, as distncias em linha reta entre a sede do municpio e Choupana
e entre Anicuns e Capelinha so, respectivamente, de 5,0 cm e 4,5 cm. J a distncia entre Choupana e Capelinha
corresponde a 6,5 cm. Sabendo-se que a escala do mapa de 1: 400.000, a distncia real entre as localidades de
aproximadamente:
a) 18 km, 20 km e 26 km
b) 20 km, 18 km e 26 km
c) 20 km, 26 km e 18 km
d) 26 km, 18 km e 20 km



5
7. (UNICAMP/08-modificada) A ltima Olimpada ocorreu em 2008 na China, tendo como sede a cidade de Pequim
(Beijing).

a) Tomando por base o mapa acima apresentado, qual era a diferena horria total entre a realizao das
competies e seu acompanhamento televisivo ao vivo no Brasil? Supondo que a cerimnia de abertura fosse
realizada a partir das dezoito horas (18h00min), no dia 8 de agosto de 2008, qual a data e o horrio
correspondente no horrio oficial do Brasil?
b) Sabendo-se que a diferena de horrio entre as cidades de Braslia e Pequim decorre da existncia de diferentes
fusos horrios, explique como so delimitados os fusos horrios e indique qual a sua extenso padro em graus de
longitude.

8. (UEG/2010-1) A crosta terrestre formada por trs tipos de estruturas geolgicas, caracterizadas pelos tipos
predominantes de rochas, pelo processo de formao e pela idade geolgica. Essas estruturas so os macios
cristalinos, as bacias sedimentares e os dobramentos modernos. Sobre esse assunto, CORRETO afirmar:
a) os macios antigos ou escudos cristalinos datam da era pr-cambriana, so constitudos por rochas sedimentares e
so ricos em jazidas de minerais no metlicos.
b) as bacias sedimentares so formaes muito recentes, datando da era quaternria, ricas em minerais energticos e
com intenso processo erosivo; constituem 64% do territrio brasileiro.
c) os dobramentos modernos, resultantes de movimentos epirogenticos, so constitudos por rochas magmticas,
datam do perodo tercirio e so ricos em carvo e petrleo, como os Andes, os Alpes e o Himalaia.
d) as principais reservas petrolferas e carbonferas do mundo encontram-se nas bacias sedimentares, enquanto
minerais como ferro, nquel, mangans, ouro, bauxita etc. so encontrados nos macios cristalinos; os dobramentos
modernos constituem reas de intenso vulcanismo.

9. (UEG-2010/2)


6
A figura acima apresenta as principais placas tectnicas que formam a crosta terrestre, destacando seus limites e as
zonas de terremotos. Com base na anlise da figura e em seus conhecimentos, CORRETO afirmar:
a) a placa Sul-Americana possui limite transformante com a placa Nazca, o que proporciona a presena de vulces,
cadeias de montanhas e intensos terremotos, a exemplo do ocorrido no inicio deste ano no Chile.
b) em 1960, ocorreu o mais violento sismo no Chile, atingindo a magnitude de 9,5 Mw, fato que produziu um tsunami
que afetou o Hava e o Japo; consequncia do movimento das placas Nazca e Sul-Americana.
c) os tremores de terra no Haiti, em janeiro de 2010, provocaram centenas de mortes e foram ocasionados pelos
movimentos divergentes das placas Africana e Norte-americana.
d) as placas tectnicas tm espessura de 80 km e movimentam-se impulsionadas pelas correntes martimas dos
oceanos e pela presso atmosfrica.

10. (UFG-2011/2) Leia o fragmento apresentado a seguir.

Os traos em relevo da me Terra so feitos por muitos escultores ao mesmo tempo. H alguns escondidos debaixo
da crosta terrestre, so justamente esses que nunca aparecem que constroem as grandes montanhas e tambm as
maiores depresses: as vezes cospem fogo como vulces, ou danam como no baile das placas tectnicas.
H outros que so mais superficiais como o vento, o mar, os rios, todos fazendo parte do mesmo movimento: traz,
leva, deposita, torna a retirar, mexe pra c e depois pra l, e assim vo pintando o sete nas telas da natureza.
[] A natureza [] constri coisas assim como as chapadas, os pes de acar, baas [...] e vez por outra d na
cabea de fazer arranha-cus, tais como, as cadeias montanhosas.
(FERNANDES, Manoel. Aula de geografia e algumas crnicas. Campina Grande: Bagagem, 2003. p. 52. [Adaptado].)

O fragmento faz referncia s formas e processos de formao do relevo. Com base em sua leitura e nos seus
conhecimentos sobre o relevo,
a) identifique duas das formas de relevo mencionadas no texto e d exemplos dos nomes das feies encontradas no
territrio nacional ou internacional e os locais onde elas ocorrem.
b) identifique e explique os dois conjuntos de processos de formao do relevo, mencionados no texto, considerando a
origem dos processos.

(UFG/07) Observe as figuras a seguir.

As figuras acima apresentam dois tipos de representao do relevo. A anlise dessa representao orienta o uso e
a ocupao do espao. Tendo-as como referncia,
a) identifique o tipo de representao do relevo utilizado em cada uma das figuras;
b) identifique, entre as reas A, B e C destacadas nas figuras, a rea propcia realizao da agricultura mecanizada
e explique por que a mais adequada para a atividade e como esse aspecto pode ser observado nas figuras
apresentadas.

11. (UFG/2009-1) O relevo terrestre dinmico. Ele se forma e modifica-se em virtude de fatores internos e externos.
Considerando a dinmica do Planalto Central brasileiro, constata-se que a transformao desse relevo ocorre por
causa
(A) do soerguimento da crosta terrestre produzido pelo tectonismo.
(B) dos processos erosivos mais intensos do que os de deposio de materiais provocados pelo intemperismo.
(C) do rebaixamento do relevo em virtude de movimento das placas tectnicas.
(D) da sedimentao de materiais em detrimento dos processos de degradao oriundos de atividade fluvial.
(E) dos falhamentos formados em decorrncia de um sistema de fraturas ocorrido na crosta terrestre.

12. (UFU-2011/2) A crosta terrestre formada por blocos rochosos continentais sobrepostos s placas tectnicas que
flutuam sobre o manto de lavas fluidas e viscosas.
Cite dois eventos derivados da movimentao da crosta terrestre e explique as consequncias destes para a vida e
sociedade humana.


7
13. (UFG-2010/2) Observe dois tipos de projees cartogrficas aplicadas aos mapas.

As projees cartogrficas cilndricas permitem mostrar a esfera terrestre com alguns tipos de distores geomtricas,
que afetam as aparncias das reas e das formas continentais. Os mapas apresentados foram elaborados de acordo
com as projees de Mercator e de Peters. A partir destas projees, analise os dois mapas quanto:
a) manuteno ou alterao das reas dos continentes;
b) s distores maiores ou menores nas representaes das formas dos continentes em baixas,
mdias e altas latitudes.

15. (UNICAMP/06) O aquecimento global assunto polmico e tem sido associado intensificao do efeito estufa.
Diversos pesquisadores relacionam a intensificao desse efeito a vrias atividades humanas, entre elas a queima de
combustveis fsseis pelos meios de transporte nos grandes centros urbanos.
a) Explique que relao existe entre as figuras A e B e como elas estariam relacionadas com a intensificao do efeito
estufa.
b) Por que a intensificao do efeito estufa considerada prejudicial para a Terra?
c) Indique outra atividade humana que tambm pode contribuir para a intensificao do efeito estufa. Justifique.


16. (UEG-2013/1)Durante a noite, o esfriamento da atmosfera, decorrente da perda de calor da superfcie, forma uma
camada de ar frio prxima ao solo; por ser mais pesada, essa camada no sobe [...]. Nas grandes cidades, esse
fenmeno agrava o problema da poluio atmosfrica, pois, no havendo movimentao ascendente do ar, no h
disperso dos poluentes.
LUCCI, E. A. Territrio e sociedade no mundo globalizado. So Paulo: Saraiva, 2005. p. 522.

a) O texto faz referncia a qual fenmeno climtico?
b) Discorra sobre as caractersticas desse fenmeno e os impactos que causa na sociedade.

17. (UEG/06) Ciclones so [...] grandes massas de ar e vapor dgua que giram ao redor de uma rea de baixa
presso na atmosfera e sobre o oceano. (FELCIO, Ricardo Augusto. O vento que leva tudo. Discutindo Geografia, ano
2, n.7, 2005. p. 39.)
a) Explique a diferena entre tufo e furaco.
b) Defina ciclone extratropical.

18. (UEG-2009/1-modificada) Observe o mapa abaixo.

As reas hachuradas no mapa apresentam quais tipos climticos?







8
19. "O navio de salvamento deve atingir o ponto com a ______________ de 12 de _________ S e 30 de ________
O, para efetuar o resgate dos nufragos."
Complete a frase de modo correto:
a) posio geogrfica; latitude; longitude
b) coordenadas geogrficas; latitude; longitude
c) latitude; longitude; posio geogrfica
d) longitude; latitude; posio geogrfica
e) latitude; coordenadas geogrficas; longitude

20. Os climogramas referem-se a trs localidades de diferentes partes do mundo. Identifique nessas localidades as
seguintes informaes:

a) Os dois meses mais chuvosos e os dois mais secos.
b) Os dois meses mais quentes e os dois mais frios.


21. Utilize o mapa a seguir para responder as questes.
a) Calcule a amplitude trmica de A,B e C. (diferena entre a maior e a menor temperatura)
b) Cite os tipos climticos das regies A, B e C.


22. (FUVEST) No sul do Paran, Santa Catarina e Rio grande do Sul, durante o inverno, temos a ocorrncia de
chuvas. Ao contrrio, na maior parte do Sudeste, o inverno corresponde a um perodo de estiagem. Explique esta
diferena climtica.

23. (FUVEST/07 As geleiras da foto acima podem ser utilizadas como indicadores da tendncia de aumento das
temperaturas globais, pois

a) o maior aporte de sedimentos nas partes baixas das
geleiras representa aumento da precipitao pluvial
em detrimento da precipitao nival (niveal).

b) o maior aporte de gua doce no mar interfere nas
temperaturas e pode ser calculado a partir da retrao
dos lagos glaciais.

c) a rea de recuo do gelo indica aumento de
temperatura e pode ser identificada pela maior
exposio dos depsitos glaciais tpicos.

d) a maior precipitao nival (niveal) representa
desequilbrio nas temperaturas globais e pode ser
identificada pelo aumento dos icebergs.

e) a ampliao de escavao dos vales glaciais pode
ser precisamente medida, indicando desequilbrio nas
temperaturas globais.

9
24. (FUVEST/09) Considere os mapas e as afirmaes abaixo.

I. Atualmente, observam-se significativos graus de degradao de ecossistemas marinhos e costeiros, sendo o
mangue um dos mais afetados, especialmente, em decorrncia de sua posio
prxima s principais fontes de poluio.
II. Dentre as mais graves ameaas s regies coralferas do planeta, esto os efluentes lanados por acidentes com
embarcaes, o aquecimento global, a explorao predatria e a poluio em zonas costeiras.
III. Os mangues constituem-se em ecossistemas resistentes e adaptveis a mudanas fsicas, o que se comprova por
sua distribuio independente de zonas trmicas.
IV. As regies coralferas, embora permanentemente ameaadas de degradao, esto entre os ecossistemas mais
resistentes do planeta, tal como ocorre com os mangues.
Est correto apenas o que se afirma em
a) I.
b) II.
c) I e II.
d) II e III.
e) I, III e IV.

25. (FUVEST/09) Segundo a CETESB, depois de cinco anos de melhora, a qualidade do ar na metrpole de So
Paulo voltou a piorar nos ltimos dois anos. O nmero de vezes em que a qualidade do ar ficou inadequada ou m foi
54% maior em 2007, se comparada de 2006. Dentre possveis causas e conseqncias, correto afirmar que a
gravidade do problema da poluio, a partir de 2006,
a) aumentou, em funo do forte crescimento das taxas de
industrializao na capital e no litoral e em razo da
desobedincia legal das indstrias dessas reas.
b) teve desdobramentos, como a expanso da rea mais poluda,
em funo do aumento da emisso de poluentes por veculos
automotores e outras fontes.
c) aumentou, em virtude de um novo fenmeno, o da emisso de
gs oznio pela frota de automveis bicombustveis, concentrada
na regio metropolitana.
d) teve desdobramentos sobre a formao das ilhas de calor, cujos
efeitos de aquecimento foram atenuados no centro da regio
metropolitana.
e) aumentou, em funo do crescimento econmico do interior do
Estado e em virtude da ausncia de legislao sobre emisso de
poluentes nessa regio.




10











PROF.: LUCIANA SEBA CAPUZZO




C
C
o
o
n
n
t
t
e
e

d
d
o
o
s
s
:
:

c
c
a
a
p
p

t
t
u
u
l
l
o
o
4
4

O
O
s
s
d
d
o
o
m
m

n
n
i
i
o
o
s
s
d
d
e
e
n
n
a
a
t
t
u
u
r
r
e
e
z
z
a
a
e
e
o
o

p
p
a
a
t
t
r
r
i
i
m
m

n
n
i
i
o
o
a
a
m
m
b
b
i
i
e
e
n
n
t
t
a
a
l
l

c
c
a
a
p
p

t
t
u
u
l
l
o
o
5
5

D
D
a
a
n
n
a
a
t
t
u
u
r
r
e
e
z
z
a
a
a
a
o
o
s
s
r
r
e
e
c
c
u
u
r
r
s
s
o
o
s
s
n
n
a
a
t
t
u
u
r
r
a
a
i
i
s
s

c
c
a
a
p
p

t
t
u
u
l
l
o
o
6
6

T
T
e
e
c
c
n
n
o
o
l
l
o
o
g
g
i
i
a
a
s
s
e
e
e
e
s
s
p
p
a
a

o
o
g
g
e
e
o
o
g
g
r
r

f
f
i
i
c
c
o
o



M
M
i
i
n
n
i
i
s
s
t
t
r
r
a
a
d
d
o
o
s
s
d
d
u
u
r
r
a
a
n
n
t
t
e
e
o
o
m
m

s
s
d
d
e
e
m
m
a
a
r
r

o
o















D Da at ta a d da a E En nt tr re eg ga a : : _ __ __ __ __ __ __ __ _/ /_ __ __ __ __ __ __ __ __ __ __ _/ /2 20 01 13 3

LISTA 2 GEOGRAFIA

11

1. (FUVEST/08) Com referncia biodiversidade, existem no mundo 17 pases classificados como megadiversos.
Dentre eles, destacam-se: Tailndia, Indonsia, Gabo, Congo, Colmbia e Brasil. Considerando as relaes entre
biodiversidade, economia e geopoltica,
a) explique, utilizando-se de dois argumentos, por que a biodiversidade tornou-se um elemento importante, do ponto de
vista econmico, no mundo atual.
b) esclarea, utilizando-se de dois argumentos, a importncia geopoltica da Amaznia.

2. (FUVEST/08)
a) Identifique as obras 1 e 2, representadas no mapa, considerando
o contexto do atual planejamento brasileiro de infra-estrutura.
b) Cite e explique dois possveis impactos causados pela presena
dessas obras, sendo um ambiental, em relao ao rio Madeira, e
outro socioeconmico, referente ao ncleo urbano de Porto Velho.
c) Cite um motivo pelo qual a Bolvia solicitou, do governo brasileiro,
maiores informaes sobre tais obras.

3. (FUVEST/08) Um dos problemas enfrentados por muitos
brasileiros so os escorregamentos de grandes volumes de solo
e rocha, cujas conseqncias podem incluir, no raras vezes,
perda de vidas humanas.
a) Cite dois fatores fsico-naturais envolvidos em processos de
escorregamento como o apresentado na foto ao lado.
Explique-os.
b) Analise, criticamente, dois tipos de ao humana, que, em
geral, colaboram para a ocorrncia de episdios dessa
natureza.

4. (FUVEST/2011) Em 1988, o eclogo ingls Norman Myers
props a criao do conceito de hotspot com o objetivo de resolver um dos dilemas dos conservacionistas: quais so
as reas mais importantes onde se deve preservar a biodiversidade na Terra? Conforme Myers, um hotspot deve
conter pelo menos 1.500 espcies endmicas de plantas e haver perdido mais de 3/4 da vegetao natural existente
na rea.
Observe o mapa a seguir.


Sobre os dois hotspots em terras emersas, assinalados no mapa:
a) Identifique e explique as causas da existncia do hotspot em territrio brasileiro.
b) Explique as causas da existncia do hotspot na sia equatorial.
Caderno de Atividades

Disciplina:
G Ge eo og gr ra af fi ia a

Professor(a):
Luciana Capuzzo
Aluno:
3 ano
Ensino Mdio
Data de Recebimento:
_____/_____/_____
Lista 01
Data Entrega:
_____/_____/_____

12
5. (UNICAMP) Um processo erosivo pode ser causado pela gua das chuvas que escoa sobre uma superfcie. Nas
regies tropicais, onde os totais pluviomtricos so mais elevados que em outras regies do planeta, o processo
erosivo, associado ao desmatamento para a produo agrcola, tende a ser mais intenso, colocando em risco tal
produo e as infra-estruturas do territrio, como, por exemplo, as rodovias. (Adap. GUERRA, Antonio Jos Teixeira. O
incio do processo erosivo. RJ,1999.)
a) O que um processo erosivo?
b) Por que o escoamento superficial pluvial ocorre nas encostas?
c) Que relao pode ser estabelecida entre o comprimento da encosta e a sua declividade na gerao de um processo
erosivo?

6. (ENEM) Apesar da riqueza das florestas tropicais, elas esto geralmente baseadas em solos infrteis e
improdutivos. Grande parte dos nutrientes armazenada nas folhas que caem sobre o solo, no no solo propriamente
dito. Quando esse ambiente intensamente modificado pelo ser humano, a vegetao desaparece, o ciclo dos
nutrientes alterado e a terra se torna rapidamente infrtil. (Manual global de ecologia)
No texto acima, pode parecer uma contradio a existncia de florestas tropicais exuberantes sobre solos pobres.
No entanto, este fato explicado pela:
a) Profundidade do solo, pois, embora pobre, sua espessura garante a disponibilidade de nutrientes para a sustentao
dos vegetais da regio.
b) Boa iluminao das regies tropicais, uma vez que a durao regular do dia e da noite garante os ciclos dos
nutrientes nas folhas dos vegetais da regio.
c) Existncia de grande diversidade animal, com nmero expressivo de populaes que, com seus dejetos, fertilizam o
solo.
d) Capacidade de produo abundante de oxignio pelas plantas das florestas tropicais, consideradas os pulmes do
mundo.
e) Rpida reciclagem dos nutrientes, potencializada pelo calor e umidade das florestas tropicais, o que favorece a vida
dos decompositores.

7. Explique como o capital acumulado pelos donos dos meios de produo possibilitou:
a) Os avanos tecnolgicos:
b) As mudanas nas formas de trabalho, nos hbitos e nos costumes das populaes:
c) As transformaes no espao e nas paisagens do mundo:

8. (ENEM/05)

Com base na tabela e no texto acima, analise os possveis motivos para a liderana do Brasil no tempo de uso
da internet.

I.O pas tem uma estrutura populacional com maior percentual de jovens do que os pases da Europa e os EUA.
II.O uso de internet em casa se distribui igualmente entre as classes A, B e C, o que demonstra iniciativas de incluso
digital.
III.A adeso ao sistema de internet por banda larga ocorre, porque essa tecnologia promove a mudana de
comportamento dos usurios.

Est correto apenas o que se afirma em

(A) I. (B) II. (C) III. (D) I e II. (E) II e III.





13
9. (ENEM/05) A gua um dos fatores determinantes para todos os seres vivos, mas a precipitao varia muito nos
continentes, como podemos observar no mapa abaixo.

Ao examinar a tabela da temperatura mdia anual em algumas latitudes, podemos concluir que as chuvas so
mais abundantes nas maiores latitudes prximas do Equador, porque

(A) as grandes extenses de terra fria das latitudes extremas impedem precipitaes mais abundantes.
(B) a gua superficial mais quente nos trpicos do que nas regies temperadas, causando maior precipitao.
(C) o ar mais quente tropical retm mais vapor de gua na atmosfera, aumentando as precipitaes.
(D) o ar mais frio das regies temperadas retm mais vapor de gua, impedindo as precipitaes.
(E) a gua superficial fria e menos abundante nas latitudes extremas, causando menor precipitao.

10. (UNICAMP/08) O mapa abaixo destaca as reas ridas da Terra. Responda:

a) Quais os fatores ambientais que determinam a existncia desses grandes desertos?
b) Apesar da escassez de gua, alguns desertos so povoados. Quais as intervenes que possibilitam a uma
sociedade viver nessas reas?

11. Considerando a figura anterior reconhea os desertos e suas respectivas localizaes.

12. (UEG/2009-1) A associao das condies climticas, da latitude e da altitude concorre para propiciar a
composio diversificada da cobertura vegetal do planeta. Levando em considerao a interao de fatores abiticos e
biticos, CORRETO afirmar:

a) nos desertos, os vegetais so tipicamente halfitos adaptados s perdas de gua por apresentarem estruturas
morfolgicas como espinhos e bainha no mesfilo foliar, que garantem a sobrevivncia das espcies.
b) nas savanas, o predomnio de arbustos e rvores de pequeno porte impede a ocorrncia de matas ciliares, que
mantm o equilbrio hidrolgico, alm de possuir recursos-chave.
c) nas Florestas Temperadas, localizadas em regies de mdia latitude, a cobertura vegetal tipicamente composta
por rvores caduciflias sendo adaptadas ao rigor do inverno local.
d) nas Florestas Tropicais, tpicas de regies de alta latitude, a grande diversidade biolgica da sua cobertura vegetal
est relacionada aos processos de adaptao a ambientes midos, caractersticos de perodos de interglaciaes.


14
13. (UNICAMP/09-modificada) O Pantanal j teve 17% de sua paisagem natural devastados, mas o drama da plancie
alagada, assim como o de outras reas midas do Brasil, praticamente ignorado pelos governos estadual e federal,
afirmaram cientistas reunidos em Cuiab para discutir o futuro dessas regies. Segundo Walfrido Toms, especialista
em gesto de biodiversidade da Embrapa Pantanal, a pecuria intensiva est se difundindo no Pantanal e tem
desmatado muito mais do que a tradicional pecuria pantaneira. (Adaptado de BBC Brasil:
www.viagem.uol.com.br/ultnot/bbc/2008/07/25/ult454u209.htm?action=print)
Considerando o Domnio Morfoclimtico do Pantanal, quais as caractersticas naturais que favorecem a atividade
pecuria nessa rea?

14. (UFF RJ) Os solos representados abaixo encontram-se em uma mesma regio brasileira de clima tropical mido e
com semelhante composio mineralgica. A diferena entre ambos est na maior ou menor presena de cobertura
vegetal.

Com base em sua observao e conhecimento sobre o estudo dos solos, compare-os, tendo em vista a ao
das gotas de chuva, infiltrao da gua e eroso.

15. (UEG/2010-1) A poca atual caracterizada pela soma avassaladora de informaes, em volume muito superior
capacidade de armazenamento do crebro humano. Em nenhuma poca da histria da humanidade tantos dados
estiveram disposio. Mas, paradoxalmente, lembra Eco, talvez nunca tenhamos sido to mal informados. Ou ainda:
to malformados. Temos grande quantidade de informao, mas pouca qualidade para process-la [...]. H a solido
do internauta, que o isola da comunidade cultural dos homens, e a quantidade quase ilimitada de informao. Mas,
alm disso, aumentam as possibilidades de as pessoas se perderem nessa selva ao saltar de um site para outro em
busca de alguma informao especfica.
Com base no texto acima, CORRETO afirmar:
a) a disseminao dos meios de comunicao eletrnicos e a acelerao da difuso das informaes em tempo real
promoveram o esprito de solidariedade e cooperao entre os povos.
b) o acesso internet trouxe uma maior integrao no ambiente familiar, visto que os seus membros compartilham e
discutem resultados de pesquisas divulgadas.
c) as tecnologias de comunicao promoveram a integrao cada vez maior entre os homens, ao unir os pases
ricos e desenvolvidos aos pobres e perifricos.
d) a internet criou um novo espao para o pensamento e para a comunicao, esse espao no existe fisicamente,
mas virtualmente

*Leia o texto a seguir e responda as questes 16, 17 e 18.

A chegada dos automveis
[...] quando os tradicionais fabricantes de carruagens abriram em So Paulo e no Rio de Janeiro as primeiras
montadoras de automveis, os habitantes dessas duas grandes cidades comearam a conviver com esse engenho que
na mesma poca invadia as ruas das cidades mais modernas no mundo ocidental, alterando para sempre o estilo de
vida pacato que nelas se desfrutava. Seus moradores passaram desde ento a conviver com um fenmeno
essencialmente urbano: a velocidade. O automvel, tanto quanto o cinema, apressou o ritmo de vida, atropelou o
tempo, encurtou distncias. Criaram, ambos, um novo modo de ver as coisas, simples imagens observadas na tela ou
no espelho retrovisor. A cidade moderna o mundo do movimento [...].
Superinteressante Especial
16. O automvel mudou as paisagens das cidades e a vida das pessoas. De acordo com o texto e seus
conhecimentos, como isso ocorreu?

17. Cite alguns elementos que compem as paisagens urbanas e que so decorrentes da incorporao dos
automveis ao cotidiano das cidades.

18. Que relaes diretas podemos estabelecer entre a transformao do espao da cidade provocada pelos
automveis, a indstria automotiva e a explorao de recursos fsseis como o petrleo?

19. Duas novas formas de organizao produtiva foram colocadas em prtica no sculo XX, gerando um processo de
alienao do trabalho: o fordismo e o taylorismo. Explique-as relacionando com o desenvolvimento das atividades
industriais.

15
20. (ENEM/09) At o sculo XVII, as paisagens rurais eram marcadas por atividades rudimentares e de baixa
produtividade. A partir da Revoluo Industrial, porm, sobretudo com o advento da revoluo tecnolgica, houve
um desenvolvimento contnuo do setor agropecurio. So, portanto, observadas consequncias econmicas,
sociais e ambientais inter-relacionadas no perodo posterior Revoluo Industrial, as quais incluem
A) a erradicao da fome no mundo.
B) o aumento das reas rurais e a diminuio das reas urbanas.
C) a maior demanda por recursos naturais, entre os quais os recursos energticos.
D) a menor necessidade de utilizao de adubos e corretivos na agricultura.
E) o contnuo aumento da oferta de emprego no setor primrio da economia, em face da mecanizao.

21. (ENEM/09) Alm dos inmeros eletrodomsticos e bens eletrnicos, o automvel produzido pela indstria fordista
promoveu, a partir dos anos 50, mudanas significativas no modo de vida dos consumidores e tambm na habitao e
nas cidades. Com a massificao do consumo dos bens modernos, dos eletroeletrnicos e tambm do automvel,
mudaram radicalmente o modo de vida, os valores, a cultura e o conjunto do ambiente construdo. Da ocupao do
solo urbano at o interior da moradia, a transformao foi profunda. MARICATO, E. Urbanismo na periferia do
mundo globalizado: metrpoles brasileiras. Disponvel em: http://www.scielo.br. Acesso em: 12 ago. 2009
(adaptado).

Uma das consequncias das inovaes tecnolgicas das ltimas dcadas, que determinaram diferentes formas de uso
e ocupao do espao geogrfico, a instituio das chamadas cidades globais, que se caracterizam por
A) possurem o mesmo nvel de influncia no cenrio mundial.
B) fortalecerem os laos de cidadania e solidariedade entre os membros das diversas comunidades.
C) constiturem um passo importante para a diminuio das desigualdades sociais causadas pela polarizao social e
pela segregao urbana.
D) terem sido diretamente impactadas pelo processo de internacionalizao da economia, desencadeado a partir do
final dos anos 1970.
E) terem sua origem diretamente relacionadas ao processo de colonizao ocidental do sculo XIX.

22. (ENEM/09) Com a perspectiva do desaparecimento das geleiras no Polo Norte, grandes reservas de petrleo e
minrios, hoje inacessveis, podero ser exploradas. E j atiam a cobia das potncias. KOPP, D. Guerra Fria sobre o
rtico. Le monde diplomatique Brasil. Setembro, n. 2, 2007 (adaptado).
No cenrio de que trata o texto, a explorao de jazidas de petrleo, bem como de minrios diamante, ouro, prata,
cobre, chumbo, zinco torna-se atraente no s em funo de seu formidvel potencial, mas tambm por
A) situar-se em uma zona geopoltica mais estvel que o Oriente Mdio.
B) possibilitar o povoamento de uma regio pouco habitada, alm de promover seu desenvolvimento econmico.
C) garantir, aos pases em desenvolvimento, acesso a matrias-primas e energia, necessrias ao crescimento
econmico.
D) contribuir para a reduo da poluio em reas ambientalmente j degradadas devido ao grande volume da
produo industrial, como ocorreu na Europa.
E) promover a participao dos combustveis fsseis na matriz energtica mundial, dominada, majoritariamente, pelas
fontes renovveis, de maior custo.

23. (FUVEST/2011) Viver numa grande cidade implica o reconhecimento de mltiplos sinais. Trata-se de uma
atividade do olhar, de uma identificao visual, de um saber adquirido, portanto. Se o olhar do transeunte, que fixa
fortuitamente uma mulher bonita e viva ou um grupo de moas voltando do trabalho, pressupe um conhecimento da
cor do luto e das vestimentas operrias, tambm o olhar do assaltante ou o do policial, buscando ambos a sua presa,
implica um conhecimento especfico da cidade. Maria Stella Bresciani, Londres e Paris no sculo XIX: o espetculo
da pobreza. So Paulo: Brasiliense, 1982, p.16. Adaptado.
O texto mostra como o forte crescimento territorial e demogrfico de algumas cidades europeias, no sculo XIX,
redefiniu formas de convivncia e sociabilidade de seus habitantes as quais, em alguns casos, persistem at hoje.
a) Cite e explique dois motivos do crescimento de cidades como Londres e Paris, no sculo XIX.
b) Indique e analise uma caracterstica, dentre as mencionadas no texto, que se faa presente em grandes cidades
atuais.

24. (UEG/2009-2) Estudo da ONU revela que, desde a dcada de 1970, os homicdios em So Paulo
quadruplicaram [...].
A violncia e a criminalidade urbanas aumentam em todo o planeta, incitando ao medo generalizado e afastando os
investimentos em muitas cidades, disse o secretrio geral da ONU, Ban Ki-Moon, na introduo do estudo. Cerca de
60% dos moradores das cidades nos pases em desenvolvimento foram afetados pela criminalidade, e mais da metade
deles, em pases ricos e pobres, se preocupa com a criminalidade o tempo todo ou com muita frequncia. Disponvel
em: <http://www.abin.gov.br/modules/articles/article. php?id=1119>. Acesso em: 7 mar. 2009. (Adaptado).

Os moradores dos bairros pobres, considerados como produtores da violncia humana, segundo a crena que a
misria tornaria o homem violento, so, em realidade, suas maiores vtimas. PEDRAZZINI, Yves. A violncia das
cidades. Petrpolis: Vozes, 2006. p. 19.

16

A violncia urbana no um fenmeno isolado: a urbanizao catica, a densificao ou a privatizao dos espaos
pblicos, a segregao social e racial levam a considerar as atividades informais e ilegais, violentas ou no, como
indicadores de uma transformao mundial da civilizao urbana. A informalizao da urbanizao uma resposta das
populaes carentes globalizao e s polticas de segurana, na medida de seus meios. PEDRAZZINI, Yves. A
violncia das cidades. Petrpolis: Vozes, 2006. p. 23.

Com base na leitura dos dados apresentados acima e dos trechos citados da obra do socilogo Yves Pedrazzini, e
considerando a contribuio da Geografia e da Sociologia para o estudo da violncia, CORRETO afirmar que o
aumento da violncia urbana est relacionado com
a) a globalizao, com todas as suas consequncias sociais, tais como informalizao da urbanizao, privatizao
dos espaos pblicos, segregao racial e social, aumento da pobreza.
b) a natureza humana, j que o homem o lobo do homem, ou seja, egosta por natureza e por isso a violncia no
tem relao com a pobreza ou com as mudanas sociais e espaciais.
c) a pobreza, j que os moradores dos bairros pobres so, naturalmente, produtores de violncia, tendo em vista que a
misria cria o homem violento, o que significa que aumento da pobreza produz automaticamente o aumento da
violncia e da criminalidade.
d) o Estado, que realiza um processo de informalizao da urbanizao, agindo de forma catica, informal e ilegal.

25. (UEG/2009-2) Historicamente, os rios tiveram um papel importante na formao de centros urbanos, tanto pelo
abastecimento de gua como por ser um meio de transporte que favorecia as trocas de mercadorias, sobretudo de
produtos agrcolas.
Entretanto, ao longo do tempo, os rios passaram a ser utilizados como rede de esgoto. Discorra sobre dois problemas
socioambientais decorrentes do uso inadequado dos recursos hdricos em reas urbanas no Brasil




17







PROF.: LUCIANA SEBA CAPUZZO




C
C
o
o
n
n
t
t
e
e

d
d
o
o
s
s
:
:

c
c
a
a
p
p

t
t
u
u
l
l
o
o
7
7

A
A
e
e
n
n
e
e
r
r
g
g
i
i
a
a
e
e
o
o
a
a
q
q
u
u
e
e
c
c
i
i
m
m
e
e
n
n
t
t
o
o
g
g
l
l
o
o
b
b
a
a
l
l

c
c
a
a
p
p

t
t
u
u
l
l
o
o
8
8

A
A
g
g
r
r
i
i
c
c
u
u
l
l
t
t
u
u
r
r
a
a
e
e
s
s
e
e
g
g
u
u
r
r
a
a
n
n

a
a
a
a
l
l
i
i
m
m
e
e
n
n
t
t
a
a
r
r

c
c
a
a
p
p

t
t
u
u
l
l
o
o
9
9

S
S
o
o
c
c
i
i
e
e
d
d
a
a
d
d
e
e
e
e
E
E
s
s
t
t
a
a
d
d
o
o




M
M
i
i
n
n
i
i
s
s
t
t
r
r
a
a
d
d
o
o
s
s
d
d
u
u
r
r
a
a
n
n
t
t
e
e
o
o
s
s
m
m
e
e
s
s
e
e
s
s
d
d
e
e
a
a
b
b
r
r
i
i
l
l
e
e
m
m
a
a
i
i
o
o















D Da at ta a d da a E En nt tr re eg ga a : : _ __ __ __ __ __ __ __ _/ /_ __ __ __ __ __ __ __ __ __ __ _/ /2 20 01 13 3

LISTA 3 GEOGRAFIA

18

1. (FUVEST-2010) Grande parte da produo de petrleo, no Brasil, provm de bacias localizadas na plataforma
continental (off shore). Todavia, a produo de petrleo, em rea terrestre (on shore), tem significativa importncia
econmica.
a) Identifique duas reas produtoras de petrleo on shore no Brasil e explique as causas da existncia de petrleo
nessas reas.
b) No Brasil, nos ltimos anos, a exportao de petrleo tem superado, em volume, a importao. Apesar disso,
persiste um deficit comercial relativo a esse produto. Explique o porqu desse deficit.

Texto para as questes 2 e 3.
Aumento de produtividade

Nos ltimos 60 anos, verificou-se grande aumento da produtividade agrcola nos Estados Unidos da Amrica (EUA).
Isso se deveu a diversos fatores, tais como expanso do uso de fertilizantes e pesticidas, biotecnologia e maquinrio
especializado. O grfico abaixo apresenta dados referentes agricultura desse pas, no perodo compreendido entre
1948 e 2004.

2. (FUVEST-2010) Observe o grfico abaixo.

a) Analise a matriz brasileira dos transportes, em 2005,
considerando aspectos histricos e polticos.
b) Explique a previso da matriz brasileira dos transportes,
para o ano de 2025, considerando
aspectos ambientais implcitos.

3. (ENEM/07) A respeito da agricultura estadunidense no
perodo de 1948 a 2004, observa-se que
(A) o aumento da produtividade foi acompanhado da
reduo de mais de 70% dos custos de mo-de-obra.
(B) o valor mnimo dos custos de material ocorreu entre as
dcadas de 70 e 80.
(C) a produtividade total da agricultura dos EUA
apresentou crescimento superior a 200%.
(D) a taxa de crescimento das despesas de capital
manteve-se constante entre as dcadas de 70 e 90.
(E) o aumento da produtividade foi diretamente
proporcional reduo das despesas de capital.





Caderno de Atividades

Disciplina:
G Ge eo og gr ra af fi ia a

Professor(a):
Luciana Capuzzo
Aluno:
3 ano
Ensino Mdio
Data de Recebimento:
_____/_____/_____
Lista 03
Data Entrega:
_____/_____/_____

19
4. (FUVEST/09) O debate atual em torno dos biocombustveis, como o lcool de cana-de-acar e o biodiesel, inclui o
efeito estufa. Tal efeito garante temperaturas adequadas vida na Terra, mas seu aumento indiscriminado danoso.
Com relao a esse aumento, os biocombustveis so alternativas preferveis aos combustveis fsseis porque
a) so renovveis e sua queima impede o aquecimento global.
b) retiram da atmosfera o CO2 gerado em outras eras.
c) abrem o mercado para o lcool, cuja produo diminuiu o desmatamento.
d) so combustveis de maior octanagem e de menores taxas de liberao de carbono.
e) contribuem para a diminuio da liberao de carbono, presente nos combustveis fsseis

Texto para as questes 5 e 6.

As presses ambientais pela reduo na emisso de gs estufa, somadas ao anseio pela diminuio da
dependncia do petrleo, fizeram os olhos do mundo se voltar para os combustveis renovveis, principalmente
para o etanol. Lderes na produo e no consumo de etanol, Brasil e Estados Unidos da Amrica (EUA) produziram,
juntos, cerca de 35 bilhes de litros do produto em 2006. Os EUA utilizam o milho como matria-prima para a
produo desse lcool, ao passo que o Brasil utiliza a cana-de-acar. O quadro abaixo apresenta alguns ndices
relativos ao processo de obteno de lcool nesses dois pases.

5. (ENEM/07) Se comparado com o uso do milho como matria-prima na obteno do etanol, o uso da cana-de-acar

(A) mais eficiente, pois a produtividade do canavial maior que a do milharal, superando-a em mais do dobro de litros
de lcool produzido por hectare.
(B) mais eficiente, pois gasta-se menos energia fssil para se produzir 1 litro de lcool a partir do milho do que para
produzi-lo a partir da cana.
(C) igualmente eficiente, pois, nas duas situaes, as diferenas entre o preo de venda do litro do lcool e o custo de
sua produo se equiparam.
(D) menos eficiente, pois o balano energtico para se produzir o etanol a partir da cana menor que o balano
energtico para produzi-lo a partir do milho.
(E) menos eficiente, pois o custo de produo do litro de lcool a partir da cana menor que o custo de produo a
partir do milho.

6. (ENEM/07) Considerando-se as informaes do texto, correto afirmar que
(A) o cultivo de milho ou de cana-de-acar favorece o aumento da biodiversidade.
(B) o impacto ambiental da produo estadunidense de etanol o mesmo da produo brasileira.
(C) a substituio da gasolina pelo etanol em veculos automotores pode atenuar a tendncia atual de aumento do
efeito estufa.
(D) a economia obtida com o uso de etanol como combustvel, especialmente nos EUA, vem sendo utilizada para a
conservao do meio ambiente.
(E) a utilizao de milho e de cana-de-acar para a produo de combustveis renovveis favorece a preservao das
caractersticas originais do solo.

7. (FUVEST/2010) Pela primeira vez na histria da humanidade, mais de um bilho de pessoas, concretamente 1,02
bilho, sofrero de subnutrio em todo o mundo. O aumento da insegurana alimentar que aconteceu em 2009
mostra a urgncia de encarar as causas profundas da fome com rapidez e eficcia. Relatrio da Organizao das
Naes Unidas para a Agricultura e Alimentao [FAO], primeiro semestre de 2009.
Tendo em vista as questes levantadas pelo texto, correto afirmar que
a) a principal causa da fome e da subnutrio a falta de terra agricultvel para a produo de alimentos necessrios
para toda a populao mundial.
b) a proporo de subnutridos e famintos, de acordo com os dados do texto, inferior a 10% da populao mundial.
c) as principais causas da fome e da subnutrio so disparidades econmicas, pobreza extrema, guerras e conflitos.
d) as consequncias da subnutrio severa em crianas so revertidas com alimentao adequada na vida adulta.
e) o uso de organismos geneticamente modificados na agricultura tem reduzido a subnutrio nas regies mais pobres
do planeta.

20
8. (FUVEST/2010) Considere os mapas do Estado de So Paulo, seus conhecimentos e as afirmativas abaixo.

I. A expanso desse cultivo tem ocorrido, principalmente, com vistas ao aumento da produo de etanol para o
abastecimento dos mercados interno e externo.
II. O cultivo desse produto agrcola tem ocupado pores do Oeste Paulista que, tradicionalmente, eram ocupadas com
pasto.
III. A expanso desse cultivo tem acarretado a diminuio da produo de gneros alimentcios em algumas regies do
estado.
Est correto o que se afirma em:
a) I e II, apenas.
b) II, apenas.
c) II e III, apenas.
d) III, apenas.
e) I, II e III.

9. (UEG/2007-2) A regio Centro-Oeste, com destaque para o Estado de Gois, passou por profundas transformaes
em sua dinmica socioeconmica a partir de 1970, principalmente no que se refere introduo da agricultura
moderna, tendo a soja como principal produto agrcola cultivado na regio. Porm, no incio do sculo XXI, em funo
de fatores de ordem interna e externa, vem se intensificando o plantio da cana-de-acar, voltado principalmente para
a produo de acar, lcool e outros derivados. Sobre esse assunto, INCORRETO afirmar:
a) A cana-de-acar apresenta uma estreita ligao com o setor agroindustrial, principalmente com as usinas de lcool
e acar, contribuindo para o crescimento de instalaes ligadas ao setor no Estado de Gois.
b) O cultivo da soja, bem como o da cana-de-acar, vem contribuindo significativamente para a melhoria das
condies de alimentao da populao brasileira, sobretudo da parcela mais carente, tendo em vista o elevado valor
protico que apresentam e a destinao de maior parte da produo ao mercado interno.
c) Em Gois, a expanso sucroalcooleira contribui para que haja uma supervalorizao do preo da terra e um
aumento considervel nos preos de locao e venda de equipamentos agrcolas.
d) O incremento na produo de cana-de-acar vem ganhando destaque em virtude da elevao dos preos das
commodities de acar e lcool no mercado internacional, combinado com a crescente produo de automveis
bicombustveis.

10. (UEG/2008-1)

Com base na anlise da tabela, CORRETO concluir:
a) Em decorrncia do agravamento dos problemas ambientais brasileiros, de 1940 a 2003 aumentou a oferta de fontes
de energia renovveis e diminuiu a de fontes no-renovveis.
b) Entre as dcadas de 1940 e 1960, a elevada oferta de lenha e de carvo vegetal como fontes de
energia estava relacionada ao intenso processo de xodo rural e o conseqente processo de urbanizao ocorrido no
perodo.
c) Por causa de crises geradas pelo esgotamento das fontes de energia no-renovveis vem aumentando no Brasil a
oferta de fontes alternativas, como a oriunda de produtos derivados da cana.
d) Como forma de superar as sucessivas crises energticas (apages), houve nos ltimos anos o crescimento na
oferta de energia eltrica em detrimento das demais fontes de energia.

21
11. O modelo agrcola implantado no perodo colonial era fundamentado no trinmio: grande propriedade, monocultura
destinada ao mercado externo e mo-de-obra escrava. Outra caracterstica marcante dessa poca foi o fato de as
terras serem tratadas como produto descartvel. Que conseqncias o modelo agrcola de exportao dos pases
subdesenvolvidos, como o Brasil, traz para as suas populaes?

12. A biotecnologia aumentar mais ainda as diferenas entre agricultores de pases ricos e de pases pobres, porque
suas patentes esto nas mos de grandes transnacionais. Justifique a afirmativa explicando o papel da biotecnologia
no aumento da produo agrcola mundial.

13. Se voc tem um computador em casa ou no trabalho, h mais de 90% de chance de que ele funcione com o
sistema Windows. Voc vive na pele o caso mais conhecido de monoplio da atualidade. O transgnico (OGM -
organismo geneticamente modificado) um instrumento para fazer o mesmo na rea agrcola: garantir s empresas o
controle de todas as sementes. Explique quais as implicaes econmicas e ambientais a respeito da utilizao de
sementes transgnicas.

14. O agronegcio responde por um tero do PIB, 42% das exportaes e 37% dos empregos. Com clima
privilegiado, solo frtil, disponibilidade de gua, rica biodiversidade e mo-de-obra qualificada, o Brasil capaz de
colher at duas safras anuais de gros. As palavras so do Ministrio da Agricultura e correspondem aos fatos. (Carta
Capital, 01/09/2004). O texto faz referncia ao agronegcio e empresa agrcola, uma das caractersticas do mundo
contemporneo nos pases desenvolvidos e em algumas regies de pases subdesenvolvidos.
a) O que e o que caracteriza uma empresa agrcola?
b) Cite trs caractersticas da agricultura tradicional.

15. (VUNESP) Pesquisas recentes revelam que, nas ltimas dcadas, o meio rural brasileiro vem ganhando novas
funes agrcolas e no-agrcolas, reduzindo cada vez mais os limites entre o rural e o urbano.
Indique trs causas que explicam a procura por atividades no-agrcolas pela mo-de-obra residente na zona rural.

16. Analisando a situao atual, que tipo de problema podemos detectar na delimitao das terras indgenas se
comparada ao incio da colonizao?

17. Relacione as caractersticas da sociedade e da lavoura da cana-de-acar nos sculos XVI e XVII no nordeste
brasileiro.
*Leia o trecho a seguir e responda as questes 18,19 e 20.
De todos os produtos coloniais Isto , aqueles plantados para render um excedente exportvel para a metrpole
o mais valioso e vivel era a cana-de-acar. Cultivada h sculos na ndia e plantada em todo o Mediterrneo, foi um
importante produto de exportao dos portugueses para os mercados do norte da Europa. (Extrado de: Warren Dean.
A ferro e fogo: a histria e a devastao da mata atlntica brasileira. So Paulo, 2002.)

18. Caracterize o tipo de clima e solo que favoreceram o cultivo da cana-de-acar na Zona da Mata nordestina.

19. Comente os impactos ambientais causados pelo plantio da cana do perodo colonial aos dias atuais.

20. Espanha e Portugal foram os pioneiros na expanso martimo-comercial iniciada no sculo XV, resultando na
conquista de novas terras para os dois pases. O que significa afirmar que o Tratado de Tordesilhas constitui-se num
dos componentes cruciais da formao histrico-geogrfica brasileira?

21. As atividades econmicas brasileiras at o desenvolvimento da economia cafeeira no sculo XIX eram regionais,
isoladas umas das outras. Explique como funcionava a economia dos arquiplagos coloniais.

22. A instalao de bases militares ao norte dos vales dos Rios Solimes e Amazonas, tem objetivos claros de controle
da regio e defesa das fronteiras. Quais so os pases que fazem fronteira com o Brasil nessa regio e quais os
problemas a serem combatidos pelas foras armadas?

23. (UEG-2010/1) Uma diviso regional fruto de teorias e mtodos utilizados para a regionalizao. Ela apresenta
uma espcie de fotografia do estgio da organizao do espao geogrfico nacional feita a partir das lentes dessas
teorias e desses mtodos.
Com base nesse fragmento e nos mapas abaixo, apresente as diferenas entre as regionalizaes estabelecidas para
o Brasil por Milton Santos (1999), IBGE (1988) e Pinchas Geiger (1964), identificando os critrios (naturais,
econmicos e/ou sociais) utilizados pelos autores.

22

24. (FUVEST/08) Uma das caractersticas do setor agropecurio, na atualidade, a alta especializao produtiva, que
refora a necessidade de circulao de alimentos pelo planeta. Como, todavia, os custos de produo so muito
distintos nas diferentes pores do globo, polticas de subsdios agrcolas e de barreiras protecionistas foram e
continuam sendo adotadas por alguns Estados, no sentido de proteger seus produtores rurais.
Sobre polticas de subsdios agrcolas e barreiras protecionistas,

a) cite dois pases que as utilizam de forma sistemtica e, ao menos, um produto por pas citado.
b) analise criticamente as aes recentes do Estado Brasileiro junto OMC e os resultados alcanados.

25.(UEG/2011-1) A noo de regio est associada a uma poro do territrio caracterizada por alguns aspectos que
lhe conferem individualidade em relao aos demais espaos. Vrios so, portanto, os critrios que podem determinar
uma regionalizao. Sobre esse tema, responda:

a) Como a diviso regional do continente americano pode ser feita do ponto de vista geogrfico e socioeconmico?
b) Considerando o critrio aspectos naturais para caracterizar uma regio, cite dois exemplos de regies naturais em
Gois.






23










PROF.: LUCIANA SEBA CAPUZZO




C
C
o
o
n
n
t
t
e
e

d
d
o
o
s
s
:
:

c
c
a
a
p
p

t
t
u
u
l
l
o
o
1
1
0
0

A
A
n
n
a
a
t
t
u
u
r
r
e
e
z
z
a
a
d
d
o
o
t
t
e
e
r
r
r
r
i
i
t
t

r
r
i
i
o
o

c
c
a
a
p
p

t
t
u
u
l
l
o
o
1
1
1
1

C
C
o
o
n
n
s
s
t
t
r
r
u
u

o
o
d
d
o
o
t
t
e
e
r
r
r
r
i
i
t
t

r
r
i
i
o
o
n
n
a
a
c
c
i
i
o
o
n
n
a
a
l
l

c
c
a
a
p
p

t
t
u
u
l
l
o
o
1
1
2
2

A
A
s
s
c
c
i
i
d
d
a
a
d
d
e
e
s
s
e
e
o
o
e
e
s
s
p
p
a
a

o
o
b
b
r
r
a
a
s
s
i
i
l
l
e
e
i
i
r
r
o
o

c
c
a
a
p
p

t
t
u
u
l
l
o
o
1
1
3
3

A
A
i
i
n
n
d
d

s
s
t
t
r
r
i
i
a
a
e
e
o
o
t
t
e
e
r
r
r
r
i
i
t
t

r
r
i
i
o
o
b
b
r
r
a
a
s
s
i
i
l
l
e
e
i
i
r
r
o
o




M
M
i
i
n
n
i
i
s
s
t
t
r
r
a
a
d
d
o
o
s
s
d
d
u
u
r
r
a
a
n
n
t
t
e
e
o
o
s
s
m
m
e
e
s
s
e
e
s
s
d
d
e
e
m
m
a
a
i
i
o
o
e
e
j
j
u
u
n
n
h
h
o
o














D Da at ta a d da a E En nt tr re eg ga a : : _ __ __ __ __ __ __ __ _/ /_ __ __ __ __ __ __ __ __ __ __ _/ /2 20 01 13 3
LISTA 4 GEOGRAFIA

24


1. (FUVEST/09)

Considere a figura e seus conhecimentos para responder.
a) Anote os nmeros de 1 a 5 correspondentes a cada unidade de relevo ou de estrutura geolgica.
b) Compare as reas A e B quanto s atividades agrrias espacialmente predominantes, relacionando essas atividades
a caractersticas do relevo

2. A minerao de pedras preciosas, metais pesados, rochas carbonticas, fosfatos, minerais radioativos e minerais
cermicos, entre outros, desempenha importante papel na economia industrial. No Brasil, que apresenta uma variada
formao geolgica e muito rico na diversidade de recursos minerais, h significativa explorao de ferro, bauxita
(alumnio), cassiterita (chumbo), ouro, diamante, areias, urnio entre outros.(Geografia do Brasil)
a) D trs exemplos de produtos que utilizam minerais em sua composio.
b) Destaque a importncia do minrio de ferro para a economia brasileira.

3. (FUVEST/08) Considere este trecho do texto de Euclides da Cunha: ... ele levava os sertanistas, sem uma remada,
para o rio Grande e da ao Paran e ao Paranaba. Era a penetrao em Minas, em Gois, em Santa Catarina, no Rio
Grande do Sul, no Mato Grosso, no Brasil inteiro. A seguir, observe o mapa.

Identifique o rio e a bacia hidrogrfica a que o autor se refere na primeira parte do texto. Qual a denominao da
bacia hidrogrfica internacional que permite que estas guas brasileiras cheguem ao Oceano Atlntico?

4. (FUVEST/2011) Os ventos alsios fazem parte da circulao atmosfrica global, soprando das zonas tropicais, de
alta presso, para a zona equatorial, de baixa presso, sendo responsveis, por exemplo, pelo transporte de umidade
ocenica para o nordeste brasileiro. Esse tipo de vento aparece no poema de Joo Cabral de Melo Neto A escola das
facas, publicado em 1980 no livro de mesmo nome, a seguir.
Caderno de Atividades

Disciplina:
G Ge eo og gr ra af fi ia a

Professor(a):
Luciana Capuzzo
Aluno:
3 ano
Ensino Mdio
Data de Recebimento:
_____/_____/_____
Lista 04
Data Entrega:
_____/_____/_____

25
O alsio ao chegar ao Nordeste baixa em coqueirais, canaviais; cursando as folhas laminadas, se afia em peixeiras,
punhais. Por isso, sobrevoada a Mata, suas mos, antes fmeas, redondas, ganham a fome e o dente da faca com que
sobrevoa outras zonas. O coqueiro e a cana lhe ensinam, sem pedra-m, mas faca a faca como voar o Agreste e o
Serto: mo cortante e desembainhada.
a) Existe relao entre o que ocorre com o alsio, ao chegar ao Nordeste, e a palavra escola, presente no ttulo do
poema de Joo Cabral de Melo Neto? Explique.
b) A umidade do ar, trazida pelos ventos alsios, diminui ao entrar no continente. Descreva e explique duas adaptaes
evolutivas, relacionadas a esse fato, que diferenciam a vegetao da Zona da Mata da vegetao do Serto.

5. Com base no mapa, reconhea as massas de ar que atuam no Brasil.


6. (UNESP/08) s margens das represas Billings e Guarapiranga, que abastecem a Grande So Paulo, vivem,
irregularmente, cerca de 1.800.000 pessoas. O impacto desta populao nas guas represadas intenso e contnuo.
Assinale a alternativa que contm trs impactos ambientais que alteram a qualidade das referidas guas.
(A) Baixo ndice pluviomtrico; escassez de gua potvel; elevao do nvel do aqfero.
(B) Baixo consumo de gua; poluio do solo; estiagem prolongada.
(C) Contaminao da gua; alta insolao; chuvas torrenciais.
(D) Baixa insolao; enchentes; deslizamentos.
(E) Desmatamento; despejo de esgoto; depsitos de lixo.

7. (FUVEST/09)

Os ciclones tropicais formam-se sobre os oceanos, em regio onde a gua quente e o vapor dgua, abundante. Eles
nem sempre evoluem para um furaco, mas suas trajetrias no Atlntico Norte favorecem essa evoluo.
a) Caracterize os furaces quanto s latitudes e s presses atmosfricas das reas em que se originam.
b) Identifique as regies onde os furaces ficam enfraquecidos em suas trajetrias.
c) Caracterize os impactos sociais e infra-estruturais dos furaces sobre pases insulares na rea representada acima.
Cite, ao menos, um desses pases como exemplo.

8. (FUVEST/09)


26
Considere os mapas ao lado e seus conhecimentos para responder.
a) Compare o uso da gua em relao ao total disponvel do Brasil ao de pases de clima temperado ocenico na
Europa, considerando o Mapa A. Justifique sua resposta.
b) Analise o Mapa B, considerando o uso da gua em relao ao total disponvel, para os EUA. Explique, identificando,
ao menos, duas razes.
9. Estabelea uma relao entre o Planalto da Borborema formao de relevo nordestina e a formao da mancha
semirida do serto.

10. (UNCAMP/2010) O mapa abaixo representa a rea abrangida pelo projeto de transposio do rio So Francisco.

a) Qual o principal bioma a ser atingido pela transposio do So Francisco? D duas caractersticas desse bioma.
b) Indique um impacto positivo e outro negativo esperados no projeto de transposio do So Francisco.

11. (UNESP/08) Observe a tabela.

Assinale a alternativa que contm o nome atribudo variao verificada entre as duas sries de dados e as
localidades que apresentam a maior e a menor variao.
(A) Variao climtica; Liubliana e Atenas.
(B) Amplitude trmica; Kiev e Dublin.
(C) Mudana climtica; Bucareste e Copenhague.
(D) Amplitude trmica; Berlim e Reikjavik.
(E) Variao climtica; Madri e Atenas.



27
12. (UNICAMP/09) Nos primeiros dias do outono subitamente entrado, quando o escurecer toma uma evidncia de
qualquer coisa prematura, e parece que tardamos muito no que fazemos de dia, gozo, mesmo entre o trabalho
quotidiano, essa antecipao de no trabalhar... (Fernando Pessoa, Livro do Desassossego. Campinas: Editora da
Unicamp, 1994, vol II, p. 55).
a) Compare as caractersticas do outono em Portugal (terra natal de Fernando Pessoa) com o outono da regio
nordeste do Brasil.
b) Diferencie solstcio de equincio.

13. (UNICAMP/2010) O El Nio um fenmeno atmosfrico-ocenico que ocorre no oceano Pacfico Tropical, e que
pode afetar o clima regional e global, porque altera padres de vento em nvel mundial. Desse modo, afeta regimes de
chuva em regies tropicais e de latitudes mdias. Com o auxlio da figura abaixo, responda s questes:

a) O que acontece com a temperatura das guas do Oceano Pacfico quando ocorre o El Nio? Qual a razo para esse
fenmeno ser denominado El Nio?
b) Nos anos em que esse fenmeno ocorre, qual a consequncia para a atividade pesqueira do Peru? Qual a alterao
do tempo no Nordeste Brasileiro?

14. (FUVEST/08) Em setembro de 2007, aconteceram passeatas, em diversas cidades do Pas, como forma de
protesto contra a privatizao da Vale (Companhia Vale do Rio Doce, antiga CVRD).
a) Caracterize o contexto poltico-econmico mundial e nacional em que se deu a privatizao da Vale.
b) Outros movimentos pr-reestatizao de empresas pblicas que foram privatizadas tm ocorrido na Amrica Latina.
Identifique um pas em que isto aconteceu recentemente e explique o fato.

15. (UNICAMP/2010) Leia abaixo o trecho da msica Tropiclia, de Caetano Veloso (1968). A seguir responda s
questes:
Sobre a cabea os avies
Sob os meus ps os caminhes
Aponta contra os chapades
Meu nariz.
Eu organizo o movimento
Eu oriento o carnaval
Eu inauguro o monumento no planalto central do pas.

a) O movimento tropicalista, do qual Caetano Veloso foi um representante, traa um retrato cantado do Brasil.
Segundo algumas interpretaes, na msica Tropiclia o autor contesta a ideologia que dominava o pensamento
poltico do Brasil, principalmente entre as dcadas de 1930 e 1960, mostrando as contradies da modernizao
subdesenvolvida do Brasil. A que fatos se referem os versos segundo e stimo do trecho da msica Tropiclia acima
reproduzida?
b) Braslia, inaugurada em 1960, completa 50 anos em 2010. A sua construo no Planalto Central era um velho sonho
do Estado brasileiro desde o Imprio. Aponte duas justificativas para a construo de Braslia.

16. (UEG/06) No fim da dcada de 1980, j eram ntidos os sinais das disperses industriais no Brasil. Esse processo
passou a ocorrer em duas escalas no territrio brasileiro, uma entre as regies do pas, outra dentro da regio Sudeste.
a) Discorra acerca de fatores que proporcionaram a descentralizao industrial da regio Sudeste.
b) Os estados de Gois e do Cear so reas receptoras de indstrias. Explique os fatores atrativos desse processo
para essas localidades.

17. (UNESP/08) O nmero de casos de dengue no Brasil aumentou 25,64% nos dois primeiros meses de 2007, em
comparao com o mesmo perodo do ano anterior. Observe o mapa, que contm a distribuio espacial da doena
por regio e estados brasileiros, segundo os casos registrados entre janeiro e fevereiro de 2007.

28

Assinale a alternativa que contm, em ordem decrescente, as regies que concentravam o maior nmero de casos e
aquela onde o grau de ocorrncia da doena foi uniforme em todos os estados.
(A) Centro-Oeste, Nordeste e Sul; Regio Norte.
(B) Sudeste, Nordeste e Norte; Regio Centro-Oeste.
(C) Centro-Oeste, Sudeste e Nordeste; Regio Sudeste.
(D) Sudeste, Nordeste e Sul; Regio Nordeste.
(E) Nordeste, Sudeste e Centro-Oeste; Regio Sul.
18. A idia de interiorizao da capital foi debatida durante todo o sculo XIX. Polticos e intelectuais defendiam sua
localizao no centro geogrfico do pas. Naquela poca, sugeriu-se Braslia como nome da capital.
a) Que motivos levaram o governo construo de Braslia?
b) Qual o significado geogrfico de cidade planejada?

19. (UFRRJ RJ/05) A raiz da concentrao industrial na regio Sudeste, inicialmente no interior do estado e depois, na
capital, a economia cafeeira no estado de So Paulo. Muitos fatores representavam atrativos para as novas
indstrias, que formaram na regio o maior parque industrial da Amrica Latina.
a) Indique quatro desses fatores atrativos.
b) Nas ltimas dcadas, a indstria paulista sofreu uma descentralizao industrial. Apresente 2 (dois) fatores
responsveis por esse fato.

20. (UEG-2013/1) A figura a seguir representa a constituio espacial da Grande Goinia.


Todos os municpios retratados no mapa mantm, embora de forma diferenciada, uma relao funcional com a cidade
de Goinia, que exerce sua polarizao, sendo a imprensa escrita ou televisiva um exemplo disso. Nos jornais dirios,
por exemplo, os moradores locais aproveitam para fazer suas queixas e reclamaes sobre os problemas que os
afetam diariamente.
Considerando-se estas informaes, cite e explique trs problemas que afetam a populao da Grande Goinia.


29
21. (UFPB PB/2005) No incio do sculo XX, o Brasil ainda ocupava papel destacado no cenrio internacional como
produtor de produtos primrios. Nas ltimas dcadas, este papel modificou-se estando o Brasil, atualmente, situado
entre os pases industrializados. Nesse contexto, considere o mapa a seguir.

A partir dessas informaes e de conhecimentos geogrficos, localize espacialmente e explique dois problemas,
sendo um econmico e um socioambiental, que contriburam para a atual distribuio regional da indstria brasileira.

22. (FUVEST/09) No perodo colonial, a escravido africana e a pecuria bovina interligaram, de algum modo, as
vrias ilhas regionais daquele antigo arquiplago econmico, conforme definido por alguns autores.
Com base nessas informaes e em seus conhecimentos, identifique
a) as duas principais portas de entrada e respectivas rotas de penetrao do gado nordestino serto adentro, durante a
fase colonial;
b) a frente pioneira da expanso pecuria bovina no Brasil de hoje.

23. (FUVEST/09)

Observe os dois mapas acima. Ambos representam diferentes regionalizaes do espao brasileiro. O de n 1 apia-se
nos conceitos de plo e hierarquia urbana e o de n 2, no de eixo de integrao e desenvolvimento.
a) Apresente, ao menos, dois critrios para a considerao da rea destacada no Mapa 1 como uma regio.
b) Justifique a delimitao da rea de influncia de nmero 3 do Mapa 2, destacando as principais
infra-estruturas de transporte a presentes.

24. (UFG/2009-2) As regies metropolitanas so estruturas urbanas complexas marcadas por intensas relaes entre
os municpios que a compem. Em 1999, foi criada a Regio Metropolitana de Goinia.
a) Apresente um motivo para a sua criao.
b) Apresente duas causas da migrao pendular nessa regio.

25. (UFG/2009-1) A regio Sul do Brasil, em decorrncia de seu processo de colonizao e de seu tipo de clima,
apresenta caractersticas culturais peculiares. Tendo por referncia essa regio:
a) apresente um hbito cultural de sua populao relacionado ao clima;
b) apresente um fator climtico e relacione-o s mdias trmicas anuais de aproximadamente 18C.





30










PROF.: LUCIANA SEBA CAPUZZO



C
C
o
o
n
n
t
t
e
e

d
d
o
o
s
s
:
:

c
c
a
a
p
p

t
t
u
u
l
l
o
o
1
1
4
4

A
A
a
a
g
g
r
r
i
i
c
c
u
u
l
l
t
t
u
u
r
r
a
a
e
e
a
a
q
q
u
u
e
e
s
s
t
t

o
o
a
a
g
g
r
r

r
r
i
i
a
a

c
c
a
a
p
p

t
t
u
u
l
l
o
o
1
1
5
5

P
P
o
o
l
l

t
t
i
i
c
c
a
a
s
s
e
e
n
n
e
e
r
r
g
g

t
t
i
i
c
c
a
a
s
s

c
c
a
a
p
p

t
t
u
u
l
l
o
o
1
1
6
6

P
P
o
o
p
p
u
u
l
l
a
a

o
o
e
e
t
t
r
r
a
a
b
b
a
a
l
l
h
h
o
o

c
c
a
a
p
p

t
t
u
u
l
l
o
o
1
1
7
7

O
O
e
e
s
s
p
p
a
a

o
o
d
d
a
a
s
s
m
m
e
e
t
t
r
r

p
p
o
o
l
l
e
e
s
s




M
M
i
i
n
n
i
i
s
s
t
t
r
r
a
a
d
d
o
o
s
s
d
d
u
u
r
r
a
a
n
n
t
t
e
e
o
o
s
s
m
m
e
e
s
s
e
e
s
s
d
d
e
e
a
a
g
g
o
o
s
s
t
t
o
o
e
e

s
s
e
e
t
t
e
e
m
m
b
b
r
r
o
o












D Da at ta a d da a E En nt tr re eg ga a : : _ __ __ __ __ __ __ __ _/ /_ __ __ __ __ __ __ __ __ __ __ _/ /2 20 01 13 3


LISTA 5 GEOGRAFIA

31

1. (UFMS MS/06-modificada) O Brasil possui os mais expressivos ndices de produtividade mdia na produo de soja
do mundo. Segundo o INCRA (2004/2005), os estados brasileiros com maior produtividade so: Mato Grosso (2,9),
Gois (2,7) e o Paran (2,6 toneladas por hectare), superiores aos 2,4 dos EUA; aos 1,7 da China e aos 1,5 do Japo.
Quais os fatores fsicos (climticos e de solos) que favorecem essa elevada produtividade nas regies citadas?

2. (UFRN RN/06) Um estudante do ensino mdio, consultando um dos fascculos do Atlas do Rio Grande do Norte,
tomou conhecimento de que [...] a economia potiguar vem se diversificando por meio de vrias alternativas
modernizantes. Nos ltimos anos, a apicultura vem se constituindo numa dessas alternativas. (Adaptado do Atlas do
RN, Dirio de Natal, 2005).
A partir dessa informao, atenda s solicitaes que seguem.

a) Cite dois motivos que justifiquem a importncia da apicultura para o Rio Grande do Norte, como alternativa de
reproduo social, sobretudo para produtores familiares rurais.
b) Explique por que cada um desses motivos justifica a importncia da apicultura para o Rio Grande do Norte.

3. (UFRRJ RJ/2006) A agricultura brasileira vem se transformando, nas ltimas dcadas, como resultado de sua
articulao cada vez maior com o setor industrial. A expanso das culturas de produtos agrcolas de exportao,
marcada pelo consumo de tratores, mquinas e insumos industriais, seria a expresso dessa modernizao.
A partir do texto:
a) cite duas razes para a modernizao da agricultura na Regio Centro-Sul;
b) apresente dois exemplos que mostrem como a aplicao de produtos agroqumicos vem provocando graves
desequilbrios ambientais;
c) apresente dois exemplos que mostrem como a industrializao da agricultura vem alterando as relaes de
produo na agricultura brasileira.

4. (UFG/08) Um fator determinante do processo de uso e ocupao do territrio goiano a demanda do mercado por
produtos agropecurios e seus derivados. Atualmente, verifica-se o aumento da instalao de unidades agroindustriais
e da rea cultivada de
(A) pastagens para criao de gado destinado produo de leite e derivados.
(B) soja para a produo do biodiesel.
(C) algodo para produo de sementes e fibras.
(D) pastagens para criao de gado destinado produo de carne e derivados.
(E) cana-de-acar para a produo do etanol.

5. (UEG/2010-1) Um sistema agrrio ou agrossistema um modelo de produo agropecuria em que se observam
quais cultivos ou criaes so praticados, quais tcnicas so utilizadas, o destino da produo, entre outras coisas.
Levando-se em conta vrios critrios, os sistemas agrrios podem se classificar em agrossistemas tradicionais,
agrossistemas modernos e agrossistemas alternativos ou orgnicos. Tendo em vista estas consideraes, responda:
a) Em qual dos agrossistemas se encaixam as plantations e a agricultura de jardinagem?
b) Cite quatro caractersticas do agrossistema moderno

6. (UFG/2009-1) A agricultura brasileira marcada, atualmente, por uma diferenciao espacial relativa produo e
ao destino dos produtos oriundos dessa atividade. O agronegcio o processo que envolve uma extensa cadeia desde
a produo at a comercializao de produtos agropecurios.
Conforme esta informao, identifique duas caractersticas espaciais do agronegcio.

7. Uma nova safra agrcola recorde no pas, considerada certa no fim de 2003, pode estar comprometida por
problemas climticos em importantes regies produtoras de soja. Produto responsvel por cerca de 45% da safra total
do pas, a soja est sendo a principal prejudicada. (O Estado de S. Paulo, mar.2004) Especifique quais so os
principais elementos climticos que podem influenciar a agricultura.

8. (UFCG PB/06) Os recursos naturais renovveis aparecem como uma soluo alternativa para a produo de energia
e com vantagens para o meio ambiente. Nessa perspectiva, novas pesquisas em espcies vegetais e alguns
condicionantes naturais indicam que a regio Nordeste apresenta grande potencial para a produo desse tipo de
energia, possibilitando uma ampliao da oferta de empregos e melhoramento de renda na regio.
Considerando a informao acima, responda questo abaixo.
a) Quais as fontes de energia alternativas que podem ser produzidas na Regio Nordeste?
Caderno de Atividades

Disciplina:
G Ge eo og gr ra af fi ia a

Professor(a):
Luciana Capuzzo
Aluno:
3 ano
Ensino Mdio
Data de Recebimento:
_____/_____/_____
Lista 05
Data Entrega:
_____/_____/_____

32
b) Justifique sua resposta, apresentando as implicaes ambientais desse tipo de energia.

9. (UFRJ/08) A proposta brasileira de estimular a produo de etanol, com tecnologia nacional, para os mercados
interno e externo tem sido tratada com destaque no cenrio mundial.
a) Apresente dois fatores que despertam o interesse atual pelo desenvolvimento da produo de biocombustveis.
b) Apresente dois riscos da expanso da produo de etanol no Brasil.

10. O consumo de gs natural cresceu 36% em novembro em So Paulo, em relao a igual perodo de 2001. S
a indstria paulista registrou um crescimento no consumo do gs natural de 32,5%. (O Estado de S. Paulo, dez.2002)
a) D as principais utilizaes do gs natural como fonte de energia.
b) Destaque a posio do gs natural entre as fontes de energia mais utilizadas no Brasil e suas vantagens em relao
ao petrleo.

11. (ENEM/2010)

12. (Puc SP/06-modificada) Leia com ateno:
A expectativa de vida da populao brasileira aumentou em mais de trs anos entre 1991 e 2000, segundo o IBGE. A
esperana de vida das mulheres passou de 70,9 anos para 74,1 anos no perodo. J para os homens aumentou de
63,1 para 66,7 anos. Em 1991, 7,8% da populao era de mulheres com 60 anos ou mais. Esse percentual subiu para
9,3% em 2000. Em 2000, 49% da populao era do sexo masculino e 51%, feminino. (Adaptado de Folha Online.
Expectativa de vida do brasileiro aumenta em mais de trs anos. 22/05/2006)
Explique quais os motivos que levam a essas diferenas entre a expectativa de vida dos homens e das mulheres.

13. (UFF RJ/06-modificada) Um estudo realizado no Hospital da Universidade de Braslia revela:
Mulheres brasileiras esto sendo laqueadas na faixa dos 20 anos, sem serem informadas das implicaes do mtodo
que, dependendo da tcnica utilizada, pode ser irreversvel e muitas vezes por vontade do parceiro e no delas
prprias. A Federao das Sociedades de Ginecologia e Obstetrcia defende uma maior oferta e promoo de
procedimentos menos agressivos como a plula, o DIU, os anticoncepcionais injetveis e os implantes.(...) (Adaptado
de Jornal Folha de So Paulo, p. C1, 20 junho de 2005.)
Quais as implicaes socioeconmicas dos procedimentos de esterilizao em massa das mulheres brasileiras?

14. (UFG GO/02) No Censo Demogrfico de 1991, o Governo do Brasil, representado pelo IBGE, considerou
alfabetizada a pessoa capaz de escrever um bilhete simples. Mesmo com este baixo nvel de exigncia, a taxa de
analfabetismo no Brasil alta e a situao do ensino, em geral, e dos investimentos em pesquisa cientfica crtica,
conforme atestam os movimentos grevistas de professores das escolas pblicas e os dados abaixo:
TAXA DE ANALFABETISMO E NDICE DE DESENVOLVIMENTO
HUMANO (IDH) EM ALGUNS PASES DO SUL

Considerando os dados acima, faa o que se pede:

33
a) estabelea uma relao entre o nvel de escolaridade da populao brasileira e a situao do Brasil na economia
mundial.
b) aponte a importncia do ensino e da escolaridade da mo-de-obra no contexto da Terceira Revoluo Industrial.

15. (FGV/07) O Brasil realizou, na segunda metade do sculo XX, a transio demogrfica e mudou a distribuio
espacial da sua populao. A partir dessa afirmativa:
a) indique duas conseqncias para a economia (uma positiva e outra negativa) da transio demogrfica;
b) relacione o processo de urbanizao com a organizao socioespacial das metrpoles.

16. (Unifesp SP/07-modificada) Os mapas indicam o IDH no Brasil, por estado, em dois momentos.

Identifique as causas do aumento do IDH nas regies norte e centro-oeste a partir de1991.
17. (UEPB PB/2006-modificada) A manchete abaixo vem se constituindo um pano de fundo nos principais
telejornais do pas.

Segundo a Constituio Federal, o salrio mnimo o mais elementar dos direitos garantidos aos trabalhadores
urbanos e rurais, alm de outros que visem melhoria de sua condio social. Faa um comentrio sobre a frase
retirada da Constituio Brasileira, estabelecendo uma relao entre o valor do salrio mnimo e o seu poder de
compra.

18. (Ufjf MG/06) O Brasil apresenta uma das piores concentraes de renda do mundo, s sendo superado por
poucos pases, tais como Serra Leoa, Repblica Centro-Africana e Suazilndia. [...] Atualmente, no combate pobreza
e como poltica de transferncia de renda, o governo federal conta, entre outros, com o Programa Bolsa Famlia. Este
programa est sob a gide do Ministrio do Desenvolvimento Social e Combate Fome. Em junho de 2004, o governo
federal registrava que 4.103.016 famlias eram beneficirias do Programa Bolsa Famlia, absorvendo, naquele ms,
recursos da ordem de R$ 288,2 milhes. [...] O Bolsa Famlia constitui, hoje, o maior programa de transferncia de
renda na rea assistencial. Estima-se em 16 milhes e 512 mil brasileiros o nmero de beneficirios desse programa.
(MARQUES, Rosa Maria (coord.). A importncia do Bolsa Famlia nos municpios brasileiros. Braslia: Ministrio do
Desenvolvimento Social e Combate Fome, 2004.)
Explique o impacto do Bolsa Famlia para a economia dos municpios cujos moradores sejam beneficirios desse
programa.

19. (UEM PR/06) O IDH (ndice de Desenvolvimento Humano) um ndice criado pela ONU (Organizao das
Naes Unidas) para medir a qualidade de vida das populaes dos pases.
a) Quais os indicadores sociais considerados para a obteno do IDH?
b) Quais os fatores que explicam a elevao do IDH brasileiro nas ltimas dcadas?

20. (UFRJ RJ/06) Doenas rurais invadem cidades! Nos ltimos cinco anos, as cidades brasileiras registram
aumentos na incidncia de vrias doenas antes associadas s reas rurais, como a leishmaniose e a doena de
Chagas, entre outras.

Apresente duas razes para a difuso de doenas rurais nas cidades.

34
21. (UFG/2009-1) O mercado consumidor brasileiro, exemplificado pela venda de automveis, geladeiras e
aparelhos de tev, caracteriza a existncia de diferenciaes regionais. Nesse contexto, a geografia do consumo no
Brasil pode ser explicada pela desigualdade
(A) da densidade demogrfica.
(B) cultural entre as regies.
(C) da renda entre as regies.
(D) na oferta do sistema de crdito.
(E) no sistema rodovirio.

22. (UFG/2009-2) As regies metropolitanas so estruturas urbanas complexas marcadas por intensas relaes
entre os municpios que a compem. Em 1999, foi criada a Regio Metropolitana de Goinia.
a) Apresente um motivo para a sua criao.
b) Apresente duas causas da migrao pendular nessa regio.

23. (UFG/2010-2) Os dados dos ltimos censos demogrficos do Brasil indicam aumento da migrao urbano-
urbano e da pendular. Com base nesta afirmao,
a) apresente dois fatores que explicam a relevncia atual da migrao urbano-urbano;
b) explique uma causa para o aumento atual da migrao pendular.

24. (UFG/08) Os movimentos sociais contemporneos so complexos, por confrontarem a estrutura social vigente. Por
isso, necessitam compor foras organizando-se em rede. Nesse contexto, a rede atua como
(A) instrumento de solidariedade poltica entre grupos que questionam as desigualdades da globalizao.
(B) elemento de anlise dos grupos que sugere os caminhos para atingir as mudanas.
(C) meio de fortalecer uma ao questionadora organizada para formar uma conscincia de cidadania.
(D) forma de criao de parcerias internacionais para potencializar a interveno poltica.
(E) mecanismo de suporte financeiro de organizaes que controlam as polticas dos lugares.

25. (UFG/2011-2) Observe o mapa e leia o fragmento apresentados a seguir.
Esquecemos da importncia da vida urbana colonial. A histria do Brasil tem sido contada como a de um pas rural:
plantadores de cana, criadores de gado e plantadores de caf. Mas houve outra histria, que permaneceu em grande
parte esquecida: a histria do Brasil urbano colonial. [].
Esquecemos que, nas vrias regies das minas, a populao era predominantemente urbana, o que s voltaria a
acontecer na segunda metade do sculo XX.

Com base na leitura do mapa e do
fragmento,

a) apresente e explique um fator
econmico-social que influenciou, no
sculo XVIII, a expanso da marcha
de povoamento nas reas que,
atualmente, correspondem regio
Norte.

b) apresente e explique uma
caracterstica que diferencie o papel
das vilas e cidades nas reas de
minerao em relao s reas de
plantation, na vida urbana no sculo
XVIII.











35






PROF.: LUCIANA SEBA CAPUZZO




C
C
o
o
n
n
t
t
e
e

d
d
o
o
s
s
:
:

c
c
a
a
p
p

t
t
u
u
l
l
o
o
1
1
8
8

P
P
o
o
l
l

t
t
i
i
c
c
a
a
s
s
t
t
e
e
r
r
r
r
i
i
t
t
o
o
r
r
i
i
a
a
i
i
s
s
:
:
n
n
o
o
r
r
d
d
e
e
s
s
t
t
e
e
e
e

a
a
m
m
a
a
z
z

n
n
i
i
a
a

c
c
a
a
p
p

t
t
u
u
l
l
o
o
1
1
9
9

O
O
s
s
i
i
s
s
t
t
e
e
m
m
a
a
m
m
u
u
n
n
d
d
i
i
a
a
l
l

c
c
a
a
p
p

t
t
u
u
l
l
o
o
2
2
0
0

A
A
e
e
c
c
o
o
n
n
o
o
m
m
i
i
a
a
d
d
a
a
g
g
l
l
o
o
b
b
a
a
l
l
i
i
z
z
a
a

o
o




M
M
i
i
n
n
i
i
s
s
t
t
r
r
a
a
d
d
o
o
s
s
d
d
u
u
r
r
a
a
n
n
t
t
e
e
o
o
m
m

s
s
d
d
e
e
o
o
u
u
t
t
u
u
b
b
r
r
o
o




* *O OB BS S: : O OS S C CA AP P T TU UL LO OS S 2 21 1 A A 3 30 0 S SE ER R O O T TR RA AB BA AL LH HA AD DO OS S N NA A M MA AR RA AT TO ON NA A. .








D Da at ta a d da a E En nt tr re eg ga a : : _ __ __ __ __ __ __ __ _/ /_ __ __ __ __ __ __ __ __ __ __ _/ /2 20 01 13 3

LISTA 6 GEOGRAFIA

36

1. (Ufba BA/06)

Com base na anlise do mapa, identifique as sub-regies I, II, III e IV e destaque as principais caractersticas
socioambientais e econmicas da sub-regio representada por II.

2. O rio So Francisco possui um grande potencial hidrulico, o que gerou a construo de vrias usinas no decorrer do
seu curso. Quais os problemas ambientais decorrentes da construo de hidreltricas nesse rio?

3. (UFCE) Dentre as estratgias de aproveitamento dos recursos hdricos superficiais do Nordeste brasileiro, admite-se
a transposio de guas entre bacias hidrogrficas. Quais os possveis impactos ambientais nos rios nordestinos com
a transposio de suas guas? Cite um ponto positivo caso os projetos venham a ser executados.

4. (UFG-2 FASE/05) A rea de 950 mil km2 que abrange mais da metade da Regio Nordeste e parte do territrio de
Minas Gerais denominada de Polgono das Secas. Nessa rea conjugam-se longos perodos de seca, as piores
condies de vida e os mais baixos ndices de Desenvolvimento Humano (IDH). Considerando essas caractersticas,
apresente e explique
a) dois elementos do meio fsico que caracterizam o Polgono das Secas;
b) dois fatores socioeconmicos que se relacionam com a problemtica da seca.

5. (UFG/2010-1) Na anlise do desenvolvimento regional do Brasil, vrios estados do Nordeste sofreram menores
impactos diante da atual crise mundial, indicando crescimento superior mdia nacional. Esse crescimento explicado
(A) pelo aumento da produo de bens exportados para os Estados Unidos.
(B) pela importncia do setor petroqumico, que ampliou o nmero de empregos.
(C) pelo aumento do consumo interno facilitado pela influncia dos programas sociais.
(D) pelo aumento da produo nas indstrias de celulose causado pelo barateamento do dlar.
(E) pelo estreitamento de negcios com a regio Centro- Oeste, formando uma nova rede de relaes.

6. Apesar da devastao, algumas iniciativas parecem apontar no sentido da explorao mais racional das riquezas
amaznicas com a criao de reservas extrativistas e reas protegidas como unidades de conservao. Explique o
papel dos seringueiros no processo de preservao dos recursos da floresta.

7. A instalao de bases militares ao norte dos vales dos Rios Solimes e Amazonas, tem objetivos claros de controle
da regio e defesa das fronteiras. Quais so os pases que fazem fronteira com o Brasil nessa regio e quais os
problemas a serem combatidos pelas foras armadas?

8. (UEG/2007-2) No processo de reestruturao da rede urbana brasileira a partir da dcada de 1970, cabe reconhecer
que, nos novos papis e valores desempenhados pelas cidades e suas respectivas regies, identificam-se novas
funes urbanas e novas interaes espaciais nas escalas intra e interurbana. Assim, a rede urbana brasileira pode ser
caracterizada pelas diversas relaes que assume (relao cidade-campo, relao entre cidades, redes de
comunicao e transporte, entre outras) e que possibilitam a sua integrao, marcada, no entanto, por disparidades
regionais. Em relao a esse processo, julgue a validade das proposies abaixo.
I. O Sudeste possui as maiores cidades, as quais esto bem articuladas nas escalas regional, nacional e internacional,
inclusive abrigando uma metrpole global.
II. A regio Norte encontra-se parcialmente desarticulada do territrio nacional, destacando-se, no entanto, algumas
metrpoles com influncia regional, como, por exemplo, Manaus e Belm.
III. A regio Nordeste possui a segunda rede urbana mais importante do pas, destacando-se a rea litornea onde
esto localizadas as regies metropolitanas de Salvador, Recife e Fortaleza.
Caderno de Atividades

Disciplina:
G Ge eo og gr ra af fi ia a

Professor(a):
Luciana Capuzzo
Aluno:
3 ano
Ensino Mdio
Data de Recebimento:
_____/_____/_____
Lista 06
Data Entrega:
_____/_____/_____

37
IV. O Centro-Oeste revela uma nova dinmica urbana por causa da expanso do setor agropecurio, que possibilita a
atrao populacional, favorecendo o surgimento de novos ncleos urbanos, como Chapado do Cu (GO), Alta
Floresta (MT) e Chapado do Sul (MS).
V. A regio Sul apresenta tambm as mesmas caractersticas da regio Centro-Oeste, com poucas cidades
importantes e um sistema de comunicao e transporte precrio.
Assinale a alternativa CORRETA:
a) Apenas as alternativas I, III e V so verdadeiras.
b) Apenas as alternativas IV e V so verdadeiras.
c) Apenas as alternativas I e II so verdadeiras.
d) Apenas as alternativas I, II, III e IV so verdadeiras.

9. (PUC/2011-1) As duas grandes marcas do sculo XX foram as guerras mundiais e o socialismo, ocasies que
geraram um terceiro grande fenmeno: a Guerra Fria, em que a moldura de uma ordem mundial bipolar se baseava na
rivalidade entre os EUA e a Unio Sovitica. Analise as proposies seguintes sobre as grandes transformaes do
sculo XX:
I - A partir de 1945, o mundo esteve dividido, predominantemente, em blocos de pases sob influncia dos EUA e da
Unio Sovitica, que entraram em confronto de forma direta, o que levou o mundo a temer o deflagrar de uma guerra
nuclear iminente.
II - No Plano Marshall encontra-se a origem da Guerra Fria. Esse Plano representou a resposta americana crise
europeia, por meio do financiamento americano da reconstruo da Europa.III - O znite da Guerra Fria aconteceu no
momento em que duas graves crises colocaram prova a resoluo das duas superpotncias e comprovaram o perigo
de uma guerra total. Trata-se da crise de Berlim, em 1961, e a crise dos msseis em Cuba, em 1962.
IV - Por consequncia do fim da Guerra Fria e da queda do muro de Berlim, o socialismo definitivamente deixou de
existir e de orientar a poltica de diversos pases.
V - Pode-se concluir que, para o quadro histrico do final do sculo XX e incio deste sculo, tanto o socialismo quanto
o capitalismo conseguiram consolidar diretrizes para os graves problemas socioeconmicos e polticos que afligem a
humanidade.
Aps a anlise das proposies, assinale a alternativa verdadeira:
A ( ) Apenas o item III correto.
B ( ) Os itens II e III esto errados.
C ( ) Apenas o item V correto.
D ( ) Os itens II e III esto corretos.

10. Equivalente a 35% das reas florestais do planeta, a Amaznia abrange terras de diferentes pases, alm do
Brasil, como Bolvia, Colmbia, Equador, Guiana, Peru, Suriname, Guiana Francesa e Venezuela. Justifique a
necessidade da preservao e da explorao racional da floresta Amaznica.

11. (PUC/MG) A Amaznia apresenta um quadro econmico e ambiental com particularidades que a diferenciam das
demais regies geoeconmicas brasileiras, sendo INCORRETO afirmar:
a) A estrutura geolgica das reas serranas rica em recursos minerais e estimula a atrao de investimentos para o
setor.
b) O extrativismo vegetal, principal atividade regional, a base da economia e gera intensos desmatamentos.
c) A ao antrpica predatria acentua a poluio, empobrece os solos e assoreia os cursos dgua, agravando os
impactos ambientais.
d) A formao de pastagens e reas de culturas substitui parcialmente os trechos desmatados da floresta e altera o
modo de sobrevivncia dos caboclos locais.

12. Leia os textos a seguir e responda as questes.

Texto 1: Tratado do Atlntico Norte 1949
Artigo 5. As partes concordam que um ataque armado contra uma ou vrias dentre elas, sobrevindo na
Europa ou na Amrica do Norte, ser considerado como um ataque dirigido contra todas as partes [...]

Texto 2: Tratado de Varsvia 1955
Artigo 4. No caso de agresso armada na Europa contra um ou vrios dos Estados signatrios [...] cada Estado
conceder s vtimas de tal agresso uma assistncia imediata...

1. Aponte os pases que integravam as referidas organizaes durante o perodo da Guerra Fria.
2. Determine o destino dessas organizaes aps o fim da Guerra Fria.
3. Compare os dois textos e comente as semelhanas existentes entre eles.

13. O ano de 2009 foi marcado pelas discusses a respeito da chegada do homem a Lua, visto que completaram 40
anos do incio da primeira aventura espacial. De l pra c a tecnologia avanou de forma assustadora e privilegiou a
produo de bens de consumo que facilitaram a vida da populao.

38
Descreva a importncia da corrida espacial para a evoluo tecnolgica citando alguns produtos lanados no
mercado baseados nessas descobertas.

14. Descreva os objetivos das seguintes organizaes criadas durante a Guerra Fria:
a) ONU:
b) FMI:
c) GATT (atual OMC):
d) CIA:

15. (FUVEST/2010) O poder do cidado, o poder de cada um de ns, limita-se, na esfera poltica, a tirar um governo
de que no gosta e a pr outro de que talvez venha a se gostar. Nada mais. Mas as grandes decises so tomadas em
uma grande esfera e todos sabemos qual . As grandes organizaes financeiras internacionais, os FMIs, a
Organizao Mundial do Comrcio, os bancos mundiais, tudo isso. Nenhum desses organismos democrtico. E,
portanto, como que podemos falar em democracia, se aqueles que efetivamente governam o mundo no so eleitos
democraticamente pelo povo? Discurso de Jos Saramago, disponvel em www.revistaforum.com.br. Acessado em 11/09/2009.

Na charge acima, o cidado sentado representa o presidente de um pas emergente. Considerando a referida charge,
o texto e seus conhecimentos,
a) caracterize a Nova Ordem Econmica Mundial.
b) analise a relao entre regime poltico democrtico e neoliberalismo, no mundo atual.

16. (FUVEST/08)

O mapa acima representa a distribuio, pelo mundo, das principais barreiras entre pases.
Identifique as barreiras 1 e 2 e analise criticamente os problemas fronteirios correspondentes, considerando os pases
envolvidos.


39
17. (FUVEST/08)


Segundo estimativas, aps 11 de setembro de 2001, quando ocorreram os atentados s Torres Gmeas e ao
Pentgono, os EUA aumentaram em cerca de 20% sua presena militar ao redor do mundo. Estima-se tambm que
por volta de 300 mil soldados estejam em bases militares americanas em mais de 140 pases.
Considerando o mapa acima,
a) identifique os locais indicados pelos nmeros 1 e 2.
b) cite e explique ao menos um motivo para a manuteno, pelos EUA, de bases militares em cada um dos locais
citados no item a.

19. Por mais de 28 anos, o Muro de Berlim foi o smbolo da diviso das duas Alemanhas e do mundo bipolar. Em 2009
comemoraram-se os 20 anos de sua queda. De que maneira a construo do Muro de Berlim representou um dos
maiores smbolos da Guerra Fria?

20. (FUVEST/09) As bombas atmicas, lanadas contra Hiroshima e Nagasaki em 1945, resultaram na morte de
aproximadamente 300.000 pessoas, vtimas imediatas das exploses ou de doenas causadas pela exposio
radiao. Esses eventos marcaram o incio de uma nova etapa histrica na corrida armamentista entre as naes,
caracterizada pelo desenvolvimento de programas nucleares com finalidades blicas. Considerando essa etapa e os
efeitos das bombas atmicas, analise as afirmaes abaixo.
I. As bombas atmicas que atingiram Hiroshima e Nagasaki foram lanadas pelos Estados Unidos, nico pas que
possua esse tipo de armamento ao fim da Segunda Guerra Mundial.
II. As radiaes liberadas numa exploso atmica podem produzir mutaes no material gentico humano, que causam
doenas como o cncer ou so transmitidas para a gerao seguinte, caso tenham ocorrido nas clulas germinativas.
III. Desde o fim da Segunda Guerra Mundial, vrias naes desenvolveram armas atmicas e, atualmente, entre as
que possuem esse tipo de armamento, tm se China, Estados Unidos, Frana, ndia, Israel, Paquisto, Reino Unido e
Rssia.
Est correto o que se afirma em
a) I, somente.
b) II, somente.
c) I e II, somente.
d) II e III, somente.
e) I, II e III.

21. Explique como o liberalismo econmico dos EUA pode ser considerado uma das causas da atual crise financeira
norte-americana?

22. Descreva os interesses estabelecidos na Conferncia de Bretton Woods ao implantar o chamado padro-dlar,
ou seja, o dlar como a moeda de comercializao internacional.

23. O G-20 um grupo de pases em desenvolvimento criado em 20 de agosto de 2003, na fase final da preparao
para a V Conferncia Ministerial da OMC, realizada em Cancun, entre 10 e 14 de setembro de 2003. O Grupo
concentra sua atuao em agricultura, o tema central da Agenda de Desenvolvimento de Doha.
a) Quais os pases integrantes do G-20?
b) Pesquise sobre a Rodada de Doha.


40
24. (UEG/2008-1) As rivalidades entre povos e o temor em relao aos imigrantes se materializam em barreiras fsicas,
os chamados muros da vergonha. So exemplos dessas barreiras defensivas: o muro de Berlim, o muro que divide
EUA e Mxico, a cerca metlica que separa a Unio Europia da frica e o muro que divide Israel da Palestina. Apesar
dos protestos e da condenao de entidades de direitos civis, como a Corte de Haia e a Organizao das Naes
Unidas (ONU), tais barreiras fsicas, com exceo do muro de Berlim, so mantidas e at reforadas. Acerca da
temtica, CORRETO afirmar:
a) O muro de Berlim teve como objetivo separar a Europa Ocidental (socialista) dos pases capitalistas. Sua queda em
1991 significou o fim da Guerra Fria e da rivalidade Leste-Oeste.
b) A consolidao da fronteira entre Mxico e EUA, via construo de um muro, assim como a existncia de uma cerca
separando a Europa da frica, tm como objetivo conter a entrada de trabalhadores ilegais nos Estados Unidos e na
Europa.
c) O muro que separa Israel dos territrios palestinos visto pela populao de ambos os pases como uma barreira de
segurana que impede atentados de extremistas.
d) Apesar da existncia de um muro separando fisicamente EUA e Mxico, por fora do Tratado de Livre- Comrcio da
Amrica do Norte (NAFTA), os mexicanos podem circular livremente pelos pases membros do referido tratado.

25. Criada em outubro de 1952, a Petrobras (Petrleo Brasileiro S. A.) a maior empresa genuinamente brasileira e
uma das 15 maiores empresas petrolferas do mundo.
Explique porque a Petrobras pode ser chamada de transnacional brasileira.







































41









PROF.: ELIEL DE QUEIROZ







C
C
o
o
n
n
t
t
e
e

d
d
o
o
:
:

S
S
I
I
N
N
T
T
A
A
X
X
E
E
D
D
O
O
P
P
O
O
R
R
T
T
U
U
G
G
U
U

S
S
.
.





M
M
i
i
n
n
i
i
s
s
t
t
r
r
a
a
d
d
o
o
d
d
u
u
r
r
a
a
n
n
t
t
e
e
o
o
m
m

s
s
d
d
e
e
f
f
e
e
v
v
e
e
r
r
e
e
i
i
r
r
o
o














D Da at ta a d da a E En nt tr re eg ga a : : _ __ __ __ __ __ __ __ _/ /_ __ __ __ __ __ __ __ __ __ __ _/ /2 20 01 13 3

LISTA 1 GRAMTICA

42

01) Sintaxe representa uma das divises da gramtica, sendo responsvel pelo estudo das relaes que se
estabelecem entre as palavras em um dado enunciado lingustico. Dessa forma, o exerccio a seguir, retirado de uma
questo de vestibular, aborda acerca dos tipos de sujeito, parte essa pertencente a tal diviso. Observe o que ele
prope para que assim tenha a habilidade necessria para resolv-lo. A seguir, comente sua resposta.
Assinale a orao na qual o sujeito oculto:
a) Encontramos homens e mulheres famintos.
b) Durante a noite, picharam a parede.
c) Existem razes para incrimin-lo.
d) Entraram o ministro e seus assessores.
e) Haviam sido realizadas todas as provas do concurso.

_ __ __ __ __ __ __ __ __ __ __ __ __ __ __ __ __ __ __ __ __ __ __ __ __ __ __ __ __ __ __ __ __ __ __ __ __ __ __ __ __ __ __ __ __ __ __ __ __ __ __ __ __ __ __ __ __ __ __ __ __ __ __ __ __ __ __ __ __ __ __ __ __ __ __ __ __ __ __ __ __ __ __ __ __ __ __ __ __ __ __ __ __ __ __ _
_ __ __ __ __ __ __ __ __ __ __ __ __ __ __ __ __ __ __ __ __ __ __ __ __ __ __ __ __ __ __ __ __ __ __ __ __ __ __ __ __ __ __ __ __ __ __ __ __ __ __ __ __ __ __ __ __ __ __ __ __ __ __ __ __ __ __ __ __ __ __ __ __ __ __ __ __ __ __ __ __ __ __ __ __ __ __ __ __ __ __ __ __ __ __ _
_ __ __ __ __ __ __ __ __ __ __ __ __ __ __ __ __ __ __ __ __ __ __ __ __ __ __ __ __ __ __ __ __ __ __ __ __ __ __ __ __ __ __ __ __ __ __ __ __ __ __ __ __ __ __ __ __ __ __ __ __ __ __ __ __ __ __ __ __ __ __ __ __ __ __ __ __ __ __ __ __ __ __ __ __ __ __ __ __ __ __ __ __ __ __ _
_ __ __ __ __ __ __ __ __ __ __ __ __ __ __ __ __ __ __ __ __ __ __ __ __ __ __ __ __ __ __ __ __ __ __ __ __ __ __ __ __ __ __ __ __ __ __ __ __ __ __ __ __ __ __ __ __ __ __ __ __ __ __ __ __ __ __ __ __ __ __ __ __ __ __ __ __ __ __ __ __ __ __ __ __ __ __ __ __ __ __ __ __ __ __ _
_ __ __ __ __ __ __ __ __ __ __ __ __ __ __ __ __ __ __ __ __ __ __ __ __ __ __ __ __ __ __ __ __ __ __ __ __ __ __ __ __ __ __ __ __ __ __ __ __ __ __ __ __ __ __ __ __ __ __ __ __ __ __ __ __ __ __ __ __ __ __ __ __ __ __ __ __ __ __ __ __ __ __ __ __ __ __ __ __ __ __ __ __ __ __ _
_ __ __ __ __ __ __ __ __ __ __ __ __ __ __ __ __ __ __ __ __ __ __ __ __ __ __ __ __ __ __ __ __ __ __ __ __ __ __ __ __ __ __ __ __ __ __ __ __ __ __ __ __ __ __ __ __ __ __ __ __ __ __ __ __ __ __ __ __ __ __ __ __ __ __ __ __ __ __ __ __ __ __ __ __ __ __ __ __ __ __ __ __ __ __ _
02) Verifique o cdigo em evidncia, empregando-o corretamente de acordo com os casos expressos pelas oraes a
seguir:
A coordenada aditiva
B coordenada adversativa
C coordenada alternativa
D coordenada explicativa
E coordenada conclusiva

a- No fomos ao aniversrio, porm trouxemos o presente ( ).
b Ou tentas se qualificar melhor, ou sers demitido ( ).
c Conseguimos obter um timo resultado, pois nos esforamos bastante ( ).
d- A garota no compareceu aula porque estava doente ( ).
e Viajamos muito e chegamos exaustos.
f No vejo importncia neste tema, portanto encerraremos a reunio.
g No gosto de sua atitude, todavia no lhe trato mal.
A Ag go or ra a, , d d o os s s se eg gu ui in nt te es s c co on nc ce ei it to os s: : C CO OO OR RD DE EN NA A O O E E S SU UB BO OR RD DI IN NA A O O. .
_ __ __ __ __ __ __ __ __ __ __ __ __ __ __ __ __ __ __ __ __ __ __ __ __ __ __ __ __ __ __ __ __ __ __ __ __ __ __ __ __ __ __ __ __ __ __ __ __ __ __ __ __ __ __ __ __ __ __ __ __ __ __ __ __ __ __ __ __ __ __ __ __ __ __ __ __ __ __ __ __ __ __ __ __ __ __ __ __ __ __ __ __ __ __ _
_ __ __ __ __ __ __ __ __ __ __ __ __ __ __ __ __ __ __ __ __ __ __ __ __ __ __ __ __ __ __ __ __ __ __ __ __ __ __ __ __ __ __ __ __ __ __ __ __ __ __ __ __ __ __ __ __ __ __ __ __ __ __ __ __ __ __ __ __ __ __ __ __ __ __ __ __ __ __ __ __ __ __ __ __ __ __ __ __ __ __ __ __ __ __ _
_ __ __ __ __ __ __ __ __ __ __ __ __ __ __ __ __ __ __ __ __ __ __ __ __ __ __ __ __ __ __ __ __ __ __ __ __ __ __ __ __ __ __ __ __ __ __ __ __ __ __ __ __ __ __ __ __ __ __ __ __ __ __ __ __ __ __ __ __ __ __ __ __ __ __ __ __ __ __ __ __ __ __ __ __ __ __ __ __ __ __ __ __ __ __ _
_ __ __ __ __ __ __ __ __ __ __ __ __ __ __ __ __ __ __ __ __ __ __ __ __ __ __ __ __ __ __ __ __ __ __ __ __ __ __ __ __ __ __ __ __ __ __ __ __ __ __ __ __ __ __ __ __ __ __ __ __ __ __ __ __ __ __ __ __ __ __ __ __ __ __ __ __ __ __ __ __ __ __ __ __ __ __ __ __ __ __ __ __ __ __ _
_ __ __ __ __ __ __ __ __ __ __ __ __ __ __ __ __ __ __ __ __ __ __ __ __ __ __ __ __ __ __ __ __ __ __ __ __ __ __ __ __ __ __ __ __ __ __ __ __ __ __ __ __ __ __ __ __ __ __ __ __ __ __ __ __ __ __ __ __ __ __ __ __ __ __ __ __ __ __ __ __ __ __ __ __ __ __ __ __ __ __ __ __ __ __ _
_ __ __ __ __ __ __ __ __ __ __ __ __ __ __ __ __ __ __ __ __ __ __ __ __ __ __ __ __ __ __ __ __ __ __ __ __ __ __ __ __ __ __ __ __ __ __ __ __ __ __ __ __ __ __ __ __ __ __ __ __ __ __ __ __ __ __ __ __ __ __ __ __ __ __ __ __ __ __ __ __ __ __ __ __ __ __ __ __ __ __ __ __ __ __ _
0 03 3) ) O perodo composto por coordenao retratado por oraes que no mantm dependncia sinttica entre elas,
isto , somente so ligadas pelo uso da conjuno.Assim sendo, demonstre seu conhecimento elaborando uma
orao para cada modalidade solicitada:
a- aditiva
b- adversativa

c- alternativa
d- explicativa
e- conclusiva
04) Observe o seguinte excerto potico e em seguida atente-se para as questes que a ele se referem:
As horas passam, os homens caem, a poesia fica (Emlio Moura)
a Estamos diante de um perodo, pois o mesmo formado por vrias oraes. Como ele se denomina? E por qu?
_ __ __ __ __ __ __ __ __ __ __ __ __ __ __ __ __ __ __ __ __ __ __ __ __ __ __ __ __ __ __ __ __ __ __ __ __ __ __ __ __ __ __ __ __ __ __ __ __ __ __ __ __ __ __ __ __ __ __ __ __ __ __ __ __ __ __ __ __ __ __ __ __ __ __ __ __ __ __ __ __ __ __ __ __ __ __ __ __ __ __ __ __ __ __ _
_ __ __ __ __ __ __ __ __ __ __ __ __ __ __ __ __ __ __ __ __ __ __ __ __ __ __ __ __ __ __ __ __ __ __ __ __ __ __ __ __ __ __ __ __ __ __ __ __ __ __ __ __ __ __ __ __ __ __ __ __ __ __ __ __ __ __ __ __ __ __ __ __ __ __ __ __ __ __ __ __ __ __ __ __ __ __ __ __ __ __ __ __ __ __ _
_ __ __ __ __ __ __ __ __ __ __ __ __ __ __ __ __ __ __ __ __ __ __ __ __ __ __ __ __ __ __ __ __ __ __ __ __ __ __ __ __ __ __ __ __ __ __ __ __ __ __ __ __ __ __ __ __ __ __ __ __ __ __ __ __ __ __ __ __ __ __ __ __ __ __ __ __ __ __ __ __ __ __ __ __ __ __ __ __ __ __ __ __ __ __ _
_ __ __ __ __ __ __ __ __ __ __ __ __ __ __ __ __ __ __ __ __ __ __ __ __ __ __ __ __ __ __ __ __ __ __ __ __ __ __ __ __ __ __ __ __ __ __ __ __ __ __ __ __ __ __ __ __ __ __ __ __ __ __ __ __ __ __ __ __ __ __ __ __ __ __ __ __ __ __ __ __ __ __ __ __ __ __ __ __ __ __ __ __ __ __ _
_ __ __ __ __ __ __ __ __ __ __ __ __ __ __ __ __ __ __ __ __ __ __ __ __ __ __ __ __ __ __ __ __ __ __ __ __ __ __ __ __ __ __ __ __ __ __ __ __ __ __ __ __ __ __ __ __ __ __ __ __ __ __ __ __ __ __ __ __ __ __ __ __ __ __ __ __ __ __ __ __ __ __ __ __ __ __ __ __ __ __ __ __ __ _
Caderno de Atividades

Disciplina:
G Gr ra am m t ti ic ca a

Professor(a):
Eliel
Aluno:
3 ano
Ensino Mdio
Data de Recebimento:
_____/_____/_____
Lista 01
Data Entrega:
_____/_____/_____

43
b As oraes que o compem so coordenadas sindticas ou assindticas? Justifique.
_ __ __ __ __ __ __ __ __ __ __ __ __ __ __ __ __ __ __ __ __ __ __ __ __ __ __ __ __ __ __ __ __ __ __ __ __ __ __ __ __ __ __ __ __ __ __ __ __ __ __ __ __ __ __ __ __ __ __ __ __ __ __ __ __ __ __ __ __ __ __ __ __ __ __ __ __ __ __ __ __ __ __ __ __ __ __ __ __ __ __ __ __ __ __ _
_ __ __ __ __ __ __ __ __ __ __ __ __ __ __ __ __ __ __ __ __ __ __ __ __ __ __ __ __ __ __ __ __ __ __ __ __ __ __ __ __ __ __ __ __ __ __ __ __ __ __ __ __ __ __ __ __ __ __ __ __ __ __ __ __ __ __ __ __ __ __ __ __ __ __ __ __ __ __ __ __ __ __ __ __ __ __ __ __ __ __ __ __ __ __ _
_ __ __ __ __ __ __ __ __ __ __ __ __ __ __ __ __ __ __ __ __ __ __ __ __ __ __ __ __ __ __ __ __ __ __ __ __ __ __ __ __ __ __ __ __ __ __ __ __ __ __ __ __ __ __ __ __ __ __ __ __ __ __ __ __ __ __ __ __ __ __ __ __ __ __ __ __ __ __ __ __ __ __ __ __ __ __ __ __ __ __ __ __ __ __ _
_ __ __ __ __ __ __ __ __ __ __ __ __ __ __ __ __ __ __ __ __ __ __ __ __ __ __ __ __ __ __ __ __ __ __ __ __ __ __ __ __ __ __ __ __ __ __ __ __ __ __ __ __ __ __ __ __ __ __ __ __ __ __ __ __ __ __ __ __ __ __ __ __ __ __ __ __ __ __ __ __ __ __ __ __ __ __ __ __ __ __ __ __ __ __ _
_ __ __ __ __ __ __ __ __ __ __ __ __ __ __ __ __ __ __ __ __ __ __ __ __ __ __ __ __ __ __ __ __ __ __ __ __ __ __ __ __ __ __ __ __ __ __ __ __ __ __ __ __ __ __ __ __ __ __ __ __ __ __ __ __ __ __ __ __ __ __ __ __ __ __ __ __ __ __ __ __ __ __ __ __ __ __ __ __ __ __ __ __ __ __ _
_ __ __ __ __ __ __ __ __ __ __ __ __ __ __ __ __ __ __ __ __ __ __ __ __ __ __ __ __ __ __ __ __ __ __ __ __ __ __ __ __ __ __ __ __ __ __ __ __ __ __ __ __ __ __ __ __ __ __ __ __ __ __ __ __ __ __ __ __ __ __ __ __ __ __ __ __ __ __ __ __ __ __ __ __ __ __ __ __ __ __ __ __ __ __ _
c Reescreva os versos introduzindo as conjunes coordenadas que melhor se adequarem ideia expressa.
_ __ __ __ __ __ __ __ __ __ __ __ __ __ __ __ __ __ __ __ __ __ __ __ __ __ __ __ __ __ __ __ __ __ __ __ __ __ __ __ __ __ __ __ __ __ __ __ __ __ __ __ __ __ __ __ __ __ __ __ __ __ __ __ __ __ __ __ __ __ __ __ __ __ __ __ __ __ __ __ __ __ __ __ __ __ __ __ __ __ __ __ __ __ __ _
_ __ __ __ __ __ __ __ __ __ __ __ __ __ __ __ __ __ __ __ __ __ __ __ __ __ __ __ __ __ __ __ __ __ __ __ __ __ __ __ __ __ __ __ __ __ __ __ __ __ __ __ __ __ __ __ __ __ __ __ __ __ __ __ __ __ __ __ __ __ __ __ __ __ __ __ __ __ __ __ __ __ __ __ __ __ __ __ __ __ __ __ __ __ __ _
_ __ __ __ __ __ __ __ __ __ __ __ __ __ __ __ __ __ __ __ __ __ __ __ __ __ __ __ __ __ __ __ __ __ __ __ __ __ __ __ __ __ __ __ __ __ __ __ __ __ __ __ __ __ __ __ __ __ __ __ __ __ __ __ __ __ __ __ __ __ __ __ __ __ __ __ __ __ __ __ __ __ __ __ __ __ __ __ __ __ __ __ __ __ __ _
_ __ __ __ __ __ __ __ __ __ __ __ __ __ __ __ __ __ __ __ __ __ __ __ __ __ __ __ __ __ __ __ __ __ __ __ __ __ __ __ __ __ __ __ __ __ __ __ __ __ __ __ __ __ __ __ __ __ __ __ __ __ __ __ __ __ __ __ __ __ __ __ __ __ __ __ __ __ __ __ __ __ __ __ __ __ __ __ __ __ __ __ __ __ __ _
_ __ __ __ __ __ __ __ __ __ __ __ __ __ __ __ __ __ __ __ __ __ __ __ __ __ __ __ __ __ __ __ __ __ __ __ __ __ __ __ __ __ __ __ __ __ __ __ __ __ __ __ __ __ __ __ __ __ __ __ __ __ __ __ __ __ __ __ __ __ __ __ __ __ __ __ __ __ __ __ __ __ __ __ __ __ __ __ __ __ __ __ __ __ __ _
_ __ __ __ __ __ __ __ __ __ __ __ __ __ __ __ __ __ __ __ __ __ __ __ __ __ __ __ __ __ __ __ __ __ __ __ __ __ __ __ __ __ __ __ __ __ __ __ __ __ __ __ __ __ __ __ __ __ __ __ __ __ __ __ __ __ __ __ __ __ __ __ __ __ __ __ __ __ __ __ __ __ __ __ __ __ __ __ __ __ __ __ __ __ __ _

05) (Marlia) Assinale a alternativa que contm uma coordenativa conclusiva:

a Srgio foi bom filho; logo ser um bom pai.
b Os meninos ora brigavam, ora brincavam.
c Jaime trabalha depressa, contudo produz pouco.
d Os ces mordem, no por maldade, mas por precisarem viver.
e Ado comeu a ma, e nossos dentes at hoje doem.

06) Analise as oraes expostas e procure construir perodos compostos por coordenao utilizando-se de conectivos
apropriados. Atenha-se para as alteraes que se fizerem necessrias:
a- No me esforcei muito. Obtive um bom resultado.
b- Precisamos nos apressar. O voo j est quase partindo.
c- Ora tens uma opinio. Ora outra.
d- No comparecemos estreia do filme. Estvamos trabalhando.
e- O acidente foi terrvel. No houve vtimas fatais.
f - Mariana estuda. Mariana toca no coral de sua igreja.
_ __ __ __ __ __ __ __ __ __ __ __ __ __ __ __ __ __ __ __ __ __ __ __ __ __ __ __ __ __ __ __ __ __ __ __ __ __ __ __ __ __ __ __ __ __ __ __ __ __ __ __ __ __ __ __ __ __ __ __ __ __ __ __ __ __ __ __ __ __ __ __ __ __ __ __ __ __ __ __ __ __ __ __ __ __ __ __ __ __ __ __ __ __ __ _
_ __ __ __ __ __ __ __ __ __ __ __ __ __ __ __ __ __ __ __ __ __ __ __ __ __ __ __ __ __ __ __ __ __ __ __ __ __ __ __ __ __ __ __ __ __ __ __ __ __ __ __ __ __ __ __ __ __ __ __ __ __ __ __ __ __ __ __ __ __ __ __ __ __ __ __ __ __ __ __ __ __ __ __ __ __ __ __ __ __ __ __ __ __ __ _
_ __ __ __ __ __ __ __ __ __ __ __ __ __ __ __ __ __ __ __ __ __ __ __ __ __ __ __ __ __ __ __ __ __ __ __ __ __ __ __ __ __ __ __ __ __ __ __ __ __ __ __ __ __ __ __ __ __ __ __ __ __ __ __ __ __ __ __ __ __ __ __ __ __ __ __ __ __ __ __ __ __ __ __ __ __ __ __ __ __ __ __ __ __ __ _
_ __ __ __ __ __ __ __ __ __ __ __ __ __ __ __ __ __ __ __ __ __ __ __ __ __ __ __ __ __ __ __ __ __ __ __ __ __ __ __ __ __ __ __ __ __ __ __ __ __ __ __ __ __ __ __ __ __ __ __ __ __ __ __ __ __ __ __ __ __ __ __ __ __ __ __ __ __ __ __ __ __ __ __ __ __ __ __ __ __ __ __ __ __ __ _
_ __ __ __ __ __ __ __ __ __ __ __ __ __ __ __ __ __ __ __ __ __ __ __ __ __ __ __ __ __ __ __ __ __ __ __ __ __ __ __ __ __ __ __ __ __ __ __ __ __ __ __ __ __ __ __ __ __ __ __ __ __ __ __ __ __ __ __ __ __ __ __ __ __ __ __ __ __ __ __ __ __ __ __ __ __ __ __ __ __ __ __ __ __ __ _
_ __ __ __ __ __ __ __ __ __ __ __ __ __ __ __ __ __ __ __ __ __ __ __ __ __ __ __ __ __ __ __ __ __ __ __ __ __ __ __ __ __ __ __ __ __ __ __ __ __ __ __ __ __ __ __ __ __ __ __ __ __ __ __ __ __ __ __ __ __ __ __ __ __ __ __ __ __ __ __ __ __ __ __ __ __ __ __ __ __ __ __ __ __ __ _
07) H um poema no qual o discurso, de uma forma interessante e ldica, aborda questes relacionadas morfologia
e sintaxe. Analise-o e atente para o que se pede:
O assassino era o escriba
Meu professor de anlise sinttica era o tipo do sujeito inexistente.
Um pleonasmo, o principal predicado de sua vida,
regular como um paradigma da 1 conjuno.
Entre uma orao subordinada e um adjunto adverbial,
ele no tinha dvidas: sempre achava um jeito
assindtico de nos torturar com um aposto.
Casou com uma regncia.
Foi infeliz.
Era possessivo como um pronome.
E ela era bitransitiva.
Tentou ir para os EUA.
No deu.
Acharam um artigo indefinido na sua bagagem.
A interjeio do bigode declinava partculas expletivas,
conectivos e agentes da passiva o tempo todo.
Um dia, matei-o com um objeto direto na cabea.
Paulo Leminski


44
a D a funo sinttica dos termos que se encontram em destaque.
_ __ __ __ __ __ __ __ __ __ __ __ __ __ __ __ __ __ __ __ __ __ __ __ __ __ __ __ __ __ __ __ __ __ __ __ __ __ __ __ __ __ __ __ __ __ __ __ __ __ __ __ __ __ __ __ __ __ __ __ __ __ __ __ __ __ __ __ __ __ __ __ __ __ __ __ __ __ __ __ __ __ __ __ __ __ __ __ __ __ __ __ __ __ __ _
_ __ __ __ __ __ __ __ __ __ __ __ __ __ __ __ __ __ __ __ __ __ __ __ __ __ __ __ __ __ __ __ __ __ __ __ __ __ __ __ __ __ __ __ __ __ __ __ __ __ __ __ __ __ __ __ __ __ __ __ __ __ __ __ __ __ __ __ __ __ __ __ __ __ __ __ __ __ __ __ __ __ __ __ __ __ __ __ __ __ __ __ __ __ __ _
_ __ __ __ __ __ __ __ __ __ __ __ __ __ __ __ __ __ __ __ __ __ __ __ __ __ __ __ __ __ __ __ __ __ __ __ __ __ __ __ __ __ __ __ __ __ __ __ __ __ __ __ __ __ __ __ __ __ __ __ __ __ __ __ __ __ __ __ __ __ __ __ __ __ __ __ __ __ __ __ __ __ __ __ __ __ __ __ __ __ __ __ __ __ __ _
_ __ __ __ __ __ __ __ __ __ __ __ __ __ __ __ __ __ __ __ __ __ __ __ __ __ __ __ __ __ __ __ __ __ __ __ __ __ __ __ __ __ __ __ __ __ __ __ __ __ __ __ __ __ __ __ __ __ __ __ __ __ __ __ __ __ __ __ __ __ __ __ __ __ __ __ __ __ __ __ __ __ __ __ __ __ __ __ __ __ __ __ __ __ __ _
_ __ __ __ __ __ __ __ __ __ __ __ __ __ __ __ __ __ __ __ __ __ __ __ __ __ __ __ __ __ __ __ __ __ __ __ __ __ __ __ __ __ __ __ __ __ __ __ __ __ __ __ __ __ __ __ __ __ __ __ __ __ __ __ __ __ __ __ __ __ __ __ __ __ __ __ __ __ __ __ __ __ __ __ __ __ __ __ __ __ __ __ __ __ __ _
_ __ __ __ __ __ __ __ __ __ __ __ __ __ __ __ __ __ __ __ __ __ __ __ __ __ __ __ __ __ __ __ __ __ __ __ __ __ __ __ __ __ __ __ __ __ __ __ __ __ __ __ __ __ __ __ __ __ __ __ __ __ __ __ __ __ __ __ __ __ __ __ __ __ __ __ __ __ __ __ __ __ __ __ __ __ __ __ __ __ __ __ __ __ __ _
_ __ __ __ __ __ __ __ __ __ __ __ __ __ __ __ __ __ __ __ __ __ __ __ __ __ __ __ __ __ __ __ __ __ __ __ __ __ __ __ __ __ __ __ __ __ __ __ __ __ __ __ __ __ __ __ __ __ __ __ __ __ __ __ __ __ __ __ __ __ __ __ __ __ __ __ __ __ __ __ __ __ __ __ __ __ __ __ __ __ __ __ __ __ __ _
_ __ __ __ __ __ __ __ __ __ __ __ __ __ __ __ __ __ __ __ __ __ __ __ __ __ __ __ __ __ __ __ __ __ __ __ __ __ __ __ __ __ __ __ __ __ __ __ __ __ __ __ __ __ __ __ __ __ __ __ __ __ __ __ __ __ __ __ __ __ __ __ __ __ __ __ __ __ __ __ __ __ __ __ __ __ __ __ __ __ __ __ __ __ __ _
_ __ __ __ __ __ __ __ __ __ __ __ __ __ __ __ __ __ __ __ __ __ __ __ __ __ __ __ __ __ __ __ __ __ __ __ __ __ __ __ __ __ __ __ __ __ __ __ __ __ __ __ __ __ __ __ __ __ __ __ __ __ __ __ __ __ __ __ __ __ __ __ __ __ __ __ __ __ __ __ __ __ __ __ __ __ __ __ __ __ __ __ __ __ __ _
08) Leia as expresses destacadas na seguinte passagem: E comecei a sentir falta das pequenas brigas por causa
do tempero na salada o meu jeito de querer bem.
Tais expresses exercem, respectivamente, a funo sinttica de:
a ( ) objeto indireto e aposto
b ( ) objeto indireto e predicativo do sujeito
c ( ) complemento nominal e adjunto adverbial de modo
d ( ) complemento nominal e aposto
e ( ) adjunto adnominal e adjunto adverbial de modo
09) Analise os seguintes enunciados e aponte a classe morfolgica referente aos termos em destaque:
Fogo
namata
Mata!
Acendi um cigarro e
vocdois.
_ __ __ __ __ __ __ __ __ __ __ __ __ __ __ __ __ __ __ __ __ __ __ __ __ __ __ __ __ __ __ __ __ __ __ __ __ __ __ __ __ __ __ __ __ __ __ __ __ __ __ __ __ __ __ __ __ __ __ __ __ __ __ __ __ __ __ __ __ __ __ __ __ __ __ __ __ __ __ __ __ __ __ __ __ __ __ __ __ __ __ __ __ __ __ _
_ __ __ __ __ __ __ __ __ __ __ __ __ __ __ __ __ __ __ __ __ __ __ __ __ __ __ __ __ __ __ __ __ __ __ __ __ __ __ __ __ __ __ __ __ __ __ __ __ __ __ __ __ __ __ __ __ __ __ __ __ __ __ __ __ __ __ __ __ __ __ __ __ __ __ __ __ __ __ __ __ __ __ __ __ __ __ __ __ __ __ __ __ __ __ _
_ __ __ __ __ __ __ __ __ __ __ __ __ __ __ __ __ __ __ __ __ __ __ __ __ __ __ __ __ __ __ __ __ __ __ __ __ __ __ __ __ __ __ __ __ __ __ __ __ __ __ __ __ __ __ __ __ __ __ __ __ __ __ __ __ __ __ __ __ __ __ __ __ __ __ __ __ __ __ __ __ __ __ __ __ __ __ __ __ __ __ __ __ __ __ _
_ __ __ __ __ __ __ __ __ __ __ __ __ __ __ __ __ __ __ __ __ __ __ __ __ __ __ __ __ __ __ __ __ __ __ __ __ __ __ __ __ __ __ __ __ __ __ __ __ __ __ __ __ __ __ __ __ __ __ __ __ __ __ __ __ __ __ __ __ __ __ __ __ __ __ __ __ __ __ __ __ __ __ __ __ __ __ __ __ __ __ __ __ __ __ _
_ __ __ __ __ __ __ __ __ __ __ __ __ __ __ __ __ __ __ __ __ __ __ __ __ __ __ __ __ __ __ __ __ __ __ __ __ __ __ __ __ __ __ __ __ __ __ __ __ __ __ __ __ __ __ __ __ __ __ __ __ __ __ __ __ __ __ __ __ __ __ __ __ __ __ __ __ __ __ __ __ __ __ __ __ __ __ __ __ __ __ __ __ __ __ _
_ __ __ __ __ __ __ __ __ __ __ __ __ __ __ __ __ __ __ __ __ __ __ __ __ __ __ __ __ __ __ __ __ __ __ __ __ __ __ __ __ __ __ __ __ __ __ __ __ __ __ __ __ __ __ __ __ __ __ __ __ __ __ __ __ __ __ __ __ __ __ __ __ __ __ __ __ __ __ __ __ __ __ __ __ __ __ __ __ __ __ __ __ __ __ _

10) Analise sintaticamente os termos em destaque das oraes em pauta, de acordo com a funo desempenhada
pelos mesmos:
a Deus,obrigada por proteger-me sempre!
_ __ __ __ __ __ __ __ __ __ __ __ __ __ __ __ __ __ __ __ __ __ __ __ __ __ __ __ __ __ __ __ __ __ __ __ __ __ __ __ __ __ __ __ __ __ __ __ __ __ __ __ __ __ __ __ __ __ __ __ __ __ __ __ __ __ __ __ __ __ __ __ __ __ __ __ __ __ __ __ __ __ __ __ __ __ __ __ __ __ __ __ __ __ __ _
_ __ __ __ __ __ __ __ __ __ __ __ __ __ __ __ __ __ __ __ __ __ __ __ __ __ __ __ __ __ __ __ __ __ __ __ __ __ __ __ __ __ __ __ __ __ __ __ __ __ __ __ __ __ __ __ __ __ __ __ __ __ __ __ __ __ __ __ __ __ __ __ __ __ __ __ __ __ __ __ __ __ __ __ __ __ __ __ __ __ __ __ __ __ __ _
_ __ __ __ __ __ __ __ __ __ __ __ __ __ __ __ __ __ __ __ __ __ __ __ __ __ __ __ __ __ __ __ __ __ __ __ __ __ __ __ __ __ __ __ __ __ __ __ __ __ __ __ __ __ __ __ __ __ __ __ __ __ __ __ __ __ __ __ __ __ __ __ __ __ __ __ __ __ __ __ __ __ __ __ __ __ __ __ __ __ __ __ __ __ __ _

b - A garota pareceu tranquila durante a apresentao.
_ __ __ __ __ __ __ __ __ __ __ __ __ __ __ __ __ __ __ __ __ __ __ __ __ __ __ __ __ __ __ __ __ __ __ __ __ __ __ __ __ __ __ __ __ __ __ __ __ __ __ __ __ __ __ __ __ __ __ __ __ __ __ __ __ __ __ __ __ __ __ __ __ __ __ __ __ __ __ __ __ __ __ __ __ __ __ __ __ __ __ __ __ __ __ _
_ __ __ __ __ __ __ __ __ __ __ __ __ __ __ __ __ __ __ __ __ __ __ __ __ __ __ __ __ __ __ __ __ __ __ __ __ __ __ __ __ __ __ __ __ __ __ __ __ __ __ __ __ __ __ __ __ __ __ __ __ __ __ __ __ __ __ __ __ __ __ __ __ __ __ __ __ __ __ __ __ __ __ __ __ __ __ __ __ __ __ __ __ __ __ _
_ __ __ __ __ __ __ __ __ __ __ __ __ __ __ __ __ __ __ __ __ __ __ __ __ __ __ __ __ __ __ __ __ __ __ __ __ __ __ __ __ __ __ __ __ __ __ __ __ __ __ __ __ __ __ __ __ __ __ __ __ __ __ __ __ __ __ __ __ __ __ __ __ __ __ __ __ __ __ __ __ __ __ __ __ __ __ __ __ __ __ __ __ __ __ _

c De toda aquela convivncia apenas restou algo: uma mgoa intensa.
_ __ __ __ __ __ __ __ __ __ __ __ __ __ __ __ __ __ __ __ __ __ __ __ __ __ __ __ __ __ __ __ __ __ __ __ __ __ __ __ __ __ __ __ __ __ __ __ __ __ __ __ __ __ __ __ __ __ __ __ __ __ __ __ __ __ __ __ __ __ __ __ __ __ __ __ __ __ __ __ __ __ __ __ __ __ __ __ __ __ __ __ __ __ __ _
_ __ __ __ __ __ __ __ __ __ __ __ __ __ __ __ __ __ __ __ __ __ __ __ __ __ __ __ __ __ __ __ __ __ __ __ __ __ __ __ __ __ __ __ __ __ __ __ __ __ __ __ __ __ __ __ __ __ __ __ __ __ __ __ __ __ __ __ __ __ __ __ __ __ __ __ __ __ __ __ __ __ __ __ __ __ __ __ __ __ __ __ __ __ __ _
_ __ __ __ __ __ __ __ __ __ __ __ __ __ __ __ __ __ __ __ __ __ __ __ __ __ __ __ __ __ __ __ __ __ __ __ __ __ __ __ __ __ __ __ __ __ __ __ __ __ __ __ __ __ __ __ __ __ __ __ __ __ __ __ __ __ __ __ __ __ __ __ __ __ __ __ __ __ __ __ __ __ __ __ __ __ __ __ __ __ __ __ __ __ __ _

d Eu necessito de seu carinho para continuar seguindo em frente.
_ __ __ __ __ __ __ __ __ __ __ __ __ __ __ __ __ __ __ __ __ __ __ __ __ __ __ __ __ __ __ __ __ __ __ __ __ __ __ __ __ __ __ __ __ __ __ __ __ __ __ __ __ __ __ __ __ __ __ __ __ __ __ __ __ __ __ __ __ __ __ __ __ __ __ __ __ __ __ __ __ __ __ __ __ __ __ __ __ __ __ __ __ __ __ _
_ __ __ __ __ __ __ __ __ __ __ __ __ __ __ __ __ __ __ __ __ __ __ __ __ __ __ __ __ __ __ __ __ __ __ __ __ __ __ __ __ __ __ __ __ __ __ __ __ __ __ __ __ __ __ __ __ __ __ __ __ __ __ __ __ __ __ __ __ __ __ __ __ __ __ __ __ __ __ __ __ __ __ __ __ __ __ __ __ __ __ __ __ __ __ _
_ __ __ __ __ __ __ __ __ __ __ __ __ __ __ __ __ __ __ __ __ __ __ __ __ __ __ __ __ __ __ __ __ __ __ __ __ __ __ __ __ __ __ __ __ __ __ __ __ __ __ __ __ __ __ __ __ __ __ __ __ __ __ __ __ __ __ __ __ __ __ __ __ __ __ __ __ __ __ __ __ __ __ __ __ __ __ __ __ __ __ __ __ __ __ _

e As lembranas da infncia atormentavam-lhe constantemente.
_ __ __ __ __ __ __ __ __ __ __ __ __ __ __ __ __ __ __ __ __ __ __ __ __ __ __ __ __ __ __ __ __ __ __ __ __ __ __ __ __ __ __ __ __ __ __ __ __ __ __ __ __ __ __ __ __ __ __ __ __ __ __ __ __ __ __ __ __ __ __ __ __ __ __ __ __ __ __ __ __ __ __ __ __ __ __ __ __ __ __ __ __ __ __ _
_ __ __ __ __ __ __ __ __ __ __ __ __ __ __ __ __ __ __ __ __ __ __ __ __ __ __ __ __ __ __ __ __ __ __ __ __ __ __ __ __ __ __ __ __ __ __ __ __ __ __ __ __ __ __ __ __ __ __ __ __ __ __ __ __ __ __ __ __ __ __ __ __ __ __ __ __ __ __ __ __ __ __ __ __ __ __ __ __ __ __ __ __ __ __ _
_ __ __ __ __ __ __ __ __ __ __ __ __ __ __ __ __ __ __ __ __ __ __ __ __ __ __ __ __ __ __ __ __ __ __ __ __ __ __ __ __ __ __ __ __ __ __ __ __ __ __ __ __ __ __ __ __ __ __ __ __ __ __ __ __ __ __ __ __ __ __ __ __ __ __ __ __ __ __ __ __ __ __ __ __ __ __ __ __ __ __ __ __ __ __ _


45
11) Idem ao exerccio anterior, aponte o vocativo nos casos abaixo:

a Tenha esperana, meu filho! No desanime!
b Tome cuidado, garoto! Poders se machucar.
c Pai, obrigada por proteger-me.
d Amigo, posso pedir-lhe um favor?
e - Participem todos da comemorao, caros colegas!
12) Atente-se para o poema exposto, procurando responder s questes referentes ao mesmo:
Dois vocativos
A maravilha d de trs cores:
branca, lils e amarela,
seu outro nome bonina.
Eu sou de trs jeitos:
alegre, triste e mofina,
mas meu outro nome eu no sei.
mistrio profundo!
amor!
Adlia Prado
(In: O corao disparado. Rio de J aneiro: Record, 2006, p.19)

a O eu lrico representado pelo poema, explicita informaes tendo como referncia dois elementos: a flor e uma
pessoa, cuja identificao no retratada.Como se trata de um texto potico, sabemos que o mesmo nos revela algo a
mais daquilo que aparentemente parece bvio. Partindo desse pressuposto, qual a pretenso da autora em se referir
s caractersticas deste outro ser, ou seja, o humano?
_ __ __ __ __ __ __ __ __ __ __ __ __ __ __ __ __ __ __ __ __ __ __ __ __ __ __ __ __ __ __ __ __ __ __ __ __ __ __ __ __ __ __ __ __ __ __ __ __ __ __ __ __ __ __ __ __ __ __ __ __ __ __ __ __ __ __ __ __ __ __ __ __ __ __ __ __ __ __ __ __ __ __ __ __ __ __ __ __ __ __ __ __ __ __ _
_ __ __ __ __ __ __ __ __ __ __ __ __ __ __ __ __ __ __ __ __ __ __ __ __ __ __ __ __ __ __ __ __ __ __ __ __ __ __ __ __ __ __ __ __ __ __ __ __ __ __ __ __ __ __ __ __ __ __ __ __ __ __ __ __ __ __ __ __ __ __ __ __ __ __ __ __ __ __ __ __ __ __ __ __ __ __ __ __ __ __ __ __ __ __ _
_ __ __ __ __ __ __ __ __ __ __ __ __ __ __ __ __ __ __ __ __ __ __ __ __ __ __ __ __ __ __ __ __ __ __ __ __ __ __ __ __ __ __ __ __ __ __ __ __ __ __ __ __ __ __ __ __ __ __ __ __ __ __ __ __ __ __ __ __ __ __ __ __ __ __ __ __ __ __ __ __ __ __ __ __ __ __ __ __ __ __ __ __ __ __ _
_ __ __ __ __ __ __ __ __ __ __ __ __ __ __ __ __ __ __ __ __ __ __ __ __ __ __ __ __ __ __ __ __ __ __ __ __ __ __ __ __ __ __ __ __ __ __ __ __ __ __ __ __ __ __ __ __ __ __ __ __ __ __ __ __ __ __ __ __ __ __ __ __ __ __ __ __ __ __ __ __ __ __ __ __ __ __ __ __ __ __ __ __ __ __ _
_ __ __ __ __ __ __ __ __ __ __ __ __ __ __ __ __ __ __ __ __ __ __ __ __ __ __ __ __ __ __ __ __ __ __ __ __ __ __ __ __ __ __ __ __ __ __ __ __ __ __ __ __ __ __ __ __ __ __ __ __ __ __ __ __ __ __ __ __ __ __ __ __ __ __ __ __ __ __ __ __ __ __ __ __ __ __ __ __ __ __ __ __ __ __ _
_ __ __ __ __ __ __ __ __ __ __ __ __ __ __ __ __ __ __ __ __ __ __ __ __ __ __ __ __ __ __ __ __ __ __ __ __ __ __ __ __ __ __ __ __ __ __ __ __ __ __ __ __ __ __ __ __ __ __ __ __ __ __ __ __ __ __ __ __ __ __ __ __ __ __ __ __ __ __ __ __ __ __ __ __ __ __ __ __ __ __ __ __ __ __ _
b - No poema h dois versos que tm como funo esclarecer sobre um termo especificado anteriormente, logo
remetemo-nos ideia do aposto. Identifique-os.
_ __ __ __ __ __ __ __ __ __ __ __ __ __ __ __ __ __ __ __ __ __ __ __ __ __ __ __ __ __ __ __ __ __ __ __ __ __ __ __ __ __ __ __ __ __ __ __ __ __ __ __ __ __ __ __ __ __ __ __ __ __ __ __ __ __ __ __ __ __ __ __ __ __ __ __ __ __ __ __ __ __ __ __ __ __ __ __ __ __ __ __ __ __ __ _
_ __ __ __ __ __ __ __ __ __ __ __ __ __ __ __ __ __ __ __ __ __ __ __ __ __ __ __ __ __ __ __ __ __ __ __ __ __ __ __ __ __ __ __ __ __ __ __ __ __ __ __ __ __ __ __ __ __ __ __ __ __ __ __ __ __ __ __ __ __ __ __ __ __ __ __ __ __ __ __ __ __ __ __ __ __ __ __ __ __ __ __ __ __ __ _
_ __ __ __ __ __ __ __ __ __ __ __ __ __ __ __ __ __ __ __ __ __ __ __ __ __ __ __ __ __ __ __ __ __ __ __ __ __ __ __ __ __ __ __ __ __ __ __ __ __ __ __ __ __ __ __ __ __ __ __ __ __ __ __ __ __ __ __ __ __ __ __ __ __ __ __ __ __ __ __ __ __ __ __ __ __ __ __ __ __ __ __ __ __ __ _
_ __ __ __ __ __ __ __ __ __ __ __ __ __ __ __ __ __ __ __ __ __ __ __ __ __ __ __ __ __ __ __ __ __ __ __ __ __ __ __ __ __ __ __ __ __ __ __ __ __ __ __ __ __ __ __ __ __ __ __ __ __ __ __ __ __ __ __ __ __ __ __ __ __ __ __ __ __ __ __ __ __ __ __ __ __ __ __ __ __ __ __ __ __ __ _
_ __ __ __ __ __ __ __ __ __ __ __ __ __ __ __ __ __ __ __ __ __ __ __ __ __ __ __ __ __ __ __ __ __ __ __ __ __ __ __ __ __ __ __ __ __ __ __ __ __ __ __ __ __ __ __ __ __ __ __ __ __ __ __ __ __ __ __ __ __ __ __ __ __ __ __ __ __ __ __ __ __ __ __ __ __ __ __ __ __ __ __ __ __ __ _
_ __ __ __ __ __ __ __ __ __ __ __ __ __ __ __ __ __ __ __ __ __ __ __ __ __ __ __ __ __ __ __ __ __ __ __ __ __ __ __ __ __ __ __ __ __ __ __ __ __ __ __ __ __ __ __ __ __ __ __ __ __ __ __ __ __ __ __ __ __ __ __ __ __ __ __ __ __ __ __ __ __ __ __ __ __ __ __ __ __ __ __ __ __ __ _
c Caso identifique a presena do vocativo, destaque-o (s).
_ __ __ __ __ __ __ __ __ __ __ __ __ __ __ __ __ __ __ __ __ __ __ __ __ __ __ __ __ __ __ __ __ __ __ __ __ __ __ __ __ __ __ __ __ __ __ __ __ __ __ __ __ __ __ __ __ __ __ __ __ __ __ __ __ __ __ __ __ __ __ __ __ __ __ __ __ __ __ __ __ __ __ __ __ __ __ __ __ __ __ __ __ __ __ _
_ __ __ __ __ __ __ __ __ __ __ __ __ __ __ __ __ __ __ __ __ __ __ __ __ __ __ __ __ __ __ __ __ __ __ __ __ __ __ __ __ __ __ __ __ __ __ __ __ __ __ __ __ __ __ __ __ __ __ __ __ __ __ __ __ __ __ __ __ __ __ __ __ __ __ __ __ __ __ __ __ __ __ __ __ __ __ __ __ __ __ __ __ __ __ _
_ __ __ __ __ __ __ __ __ __ __ __ __ __ __ __ __ __ __ __ __ __ __ __ __ __ __ __ __ __ __ __ __ __ __ __ __ __ __ __ __ __ __ __ __ __ __ __ __ __ __ __ __ __ __ __ __ __ __ __ __ __ __ __ __ __ __ __ __ __ __ __ __ __ __ __ __ __ __ __ __ __ __ __ __ __ __ __ __ __ __ __ __ __ __ _
_ __ __ __ __ __ __ __ __ __ __ __ __ __ __ __ __ __ __ __ __ __ __ __ __ __ __ __ __ __ __ __ __ __ __ __ __ __ __ __ __ __ __ __ __ __ __ __ __ __ __ __ __ __ __ __ __ __ __ __ __ __ __ __ __ __ __ __ __ __ __ __ __ __ __ __ __ __ __ __ __ __ __ __ __ __ __ __ __ __ __ __ __ __ __ _
_ __ __ __ __ __ __ __ __ __ __ __ __ __ __ __ __ __ __ __ __ __ __ __ __ __ __ __ __ __ __ __ __ __ __ __ __ __ __ __ __ __ __ __ __ __ __ __ __ __ __ __ __ __ __ __ __ __ __ __ __ __ __ __ __ __ __ __ __ __ __ __ __ __ __ __ __ __ __ __ __ __ __ __ __ __ __ __ __ __ __ __ __ __ __ _
_ __ __ __ __ __ __ __ __ __ __ __ __ __ __ __ __ __ __ __ __ __ __ __ __ __ __ __ __ __ __ __ __ __ __ __ __ __ __ __ __ __ __ __ __ __ __ __ __ __ __ __ __ __ __ __ __ __ __ __ __ __ __ __ __ __ __ __ __ __ __ __ __ __ __ __ __ __ __ __ __ __ __ __ __ __ __ __ __ __ __ __ __ __ __ _

13) Destaque o aposto referente s seguintes oraes:

a Viajamos por trs pases: Frana, Alemanha e Itlia
b- Ceclia Meireles, uma importante escritora da literatura, pertenceu era moderna.
c Santos Dumont, o inventor do 14 Bis, tornou-se um cnone na histria da humanidade.
d- So Paulo, a maior metrpole brasileira, enfrenta vrios problemas sociais.
e Dia 21 de abril: Dia de Tiradentes


46
14) (UFMG) A propsito do trecho que segue, aponte a classificao correta referente ao termo em destaque:

Minha bela Marlia, tudo passa,
A sorte deste mundo mal segura
Se vem depois dos males a ventura
Vem depois dos prazeres a desgraa.
Toms Antnio Gonzaga.

a - ( ) vocativo
b - ( ) sujeito
c - ( ) aposto
d - ( ) adjunto adnominal

15) Univ. de Passo Fundo RS:

Em No choremos, amigo, a mocidade!, o termo amigo tem a mesma funo sinttica que o segmento destacado
na alternativa:

a) Que educao essa, meu filho, que autoriza troar assim das pessoas de idade?
b) As crianas, um menino e uma menina, foram resgatadas a tempo.
c) Para no ser roubado o dinheiro, o menino chegou a escond-lo entre a palma do p e as meias.
d) Dona Bibiana, professorinha respeitada, percorria a cavalo aqueles rinces.
e) O irmo do Nogueira, o Zeferino, viveu muito pouco em nossa companhia
16) UFGO
Leia:

Num dos trechos de sua carta a D. Manuel, Pero Vaz de Caminha descreve como foi o contato entre os
portugueses e tupiniquins, que aconteceu em 24 de abril de 1500.

A respeito desse fragmento de texto, aponte a(s) afirmao(es) correta(s):

- O sujeito da orao principal classifica-se como simples: Pero Vaz de caminha.
- O pronome relativo que exerce a funo sinttica de sujeito.
- A palavra entre uma conjuno.
- A expresso em 24 de abril de 1500 tem a funo sinttica de adjunto adverbial de lugar.
- Em O contato entre os portugueses e os tupiniquins foi descrito por Pero Vaz de Caminha, a orao est na voz
passiva.
17) Fuvest SP
Considere esta frase: A matana dos marginais escandalizou a populao.
Explique os dois sentidos que podem ser atribudos a ela.
_ __ __ __ __ __ __ __ __ __ __ __ __ __ __ __ __ __ __ __ __ __ __ __ __ __ __ __ __ __ __ __ __ __ __ __ __ __ __ __ __ __ __ __ __ __ __ __ __ __ __ __ __ __ __ __ __ __ __ __ __ __ __ __ __ __ __ __ __ __ __ __ __ __ __ __ __ __ __ __ __ __ __ __ __ __ __ __ __ __ __ __ __ __ __ _
_ __ __ __ __ __ __ __ __ __ __ __ __ __ __ __ __ __ __ __ __ __ __ __ __ __ __ __ __ __ __ __ __ __ __ __ __ __ __ __ __ __ __ __ __ __ __ __ __ __ __ __ __ __ __ __ __ __ __ __ __ __ __ __ __ __ __ __ __ __ __ __ __ __ __ __ __ __ __ __ __ __ __ __ __ __ __ __ __ __ __ __ __ __ __ _
_ __ __ __ __ __ __ __ __ __ __ __ __ __ __ __ __ __ __ __ __ __ __ __ __ __ __ __ __ __ __ __ __ __ __ __ __ __ __ __ __ __ __ __ __ __ __ __ __ __ __ __ __ __ __ __ __ __ __ __ __ __ __ __ __ __ __ __ __ __ __ __ __ __ __ __ __ __ __ __ __ __ __ __ __ __ __ __ __ __ __ __ __ __ __ _
_ __ __ __ __ __ __ __ __ __ __ __ __ __ __ __ __ __ __ __ __ __ __ __ __ __ __ __ __ __ __ __ __ __ __ __ __ __ __ __ __ __ __ __ __ __ __ __ __ __ __ __ __ __ __ __ __ __ __ __ __ __ __ __ __ __ __ __ __ __ __ __ __ __ __ __ __ __ __ __ __ __ __ __ __ __ __ __ __ __ __ __ __ __ __ _
_ __ __ __ __ __ __ __ __ __ __ __ __ __ __ __ __ __ __ __ __ __ __ __ __ __ __ __ __ __ __ __ __ __ __ __ __ __ __ __ __ __ __ __ __ __ __ __ __ __ __ __ __ __ __ __ __ __ __ __ __ __ __ __ __ __ __ __ __ __ __ __ __ __ __ __ __ __ __ __ __ __ __ __ __ __ __ __ __ __ __ __ __ __ __ _
_ __ __ __ __ __ __ __ __ __ __ __ __ __ __ __ __ __ __ __ __ __ __ __ __ __ __ __ __ __ __ __ __ __ __ __ __ __ __ __ __ __ __ __ __ __ __ __ __ __ __ __ __ __ __ __ __ __ __ __ __ __ __ __ __ __ __ __ __ __ __ __ __ __ __ __ __ __ __ __ __ __ __ __ __ __ __ __ __ __ __ __ __ __ __ _
18) Dado o enunciado em questo, analise-o, com vistas a responder as questes que a ele fazem referncia:

Os professores julgaram os alunos incapacitados.
a- Podemos afirmar que a construo sinttica se encontra bem elaborada? Caso sua resposta seja negativa, procure
elucidar acerca das razes que o (a) levaram a chegar a tal concluso.
_ __ __ __ __ __ __ __ __ __ __ __ __ __ __ __ __ __ __ __ __ __ __ __ __ __ __ __ __ __ __ __ __ __ __ __ __ __ __ __ __ __ __ __ __ __ __ __ __ __ __ __ __ __ __ __ __ __ __ __ __ __ __ __ __ __ __ __ __ __ __ __ __ __ __ __ __ __ __ __ __ __ __ __ __ __ __ __ __ __ __ __ __ __ __ _
_ __ __ __ __ __ __ __ __ __ __ __ __ __ __ __ __ __ __ __ __ __ __ __ __ __ __ __ __ __ __ __ __ __ __ __ __ __ __ __ __ __ __ __ __ __ __ __ __ __ __ __ __ __ __ __ __ __ __ __ __ __ __ __ __ __ __ __ __ __ __ __ __ __ __ __ __ __ __ __ __ __ __ __ __ __ __ __ __ __ __ __ __ __ __ _
_ __ __ __ __ __ __ __ __ __ __ __ __ __ __ __ __ __ __ __ __ __ __ __ __ __ __ __ __ __ __ __ __ __ __ __ __ __ __ __ __ __ __ __ __ __ __ __ __ __ __ __ __ __ __ __ __ __ __ __ __ __ __ __ __ __ __ __ __ __ __ __ __ __ __ __ __ __ __ __ __ __ __ __ __ __ __ __ __ __ __ __ __ __ __ _
_ __ __ __ __ __ __ __ __ __ __ __ __ __ __ __ __ __ __ __ __ __ __ __ __ __ __ __ __ __ __ __ __ __ __ __ __ __ __ __ __ __ __ __ __ __ __ __ __ __ __ __ __ __ __ __ __ __ __ __ __ __ __ __ __ __ __ __ __ __ __ __ __ __ __ __ __ __ __ __ __ __ __ __ __ __ __ __ __ __ __ __ __ __ __ _

47
b Ainda supondo que se trate de uma questo que necessite ser reformulada, retifique-a, de modo a tornar clara a
mensagem.
_ __ __ __ __ __ __ __ __ __ __ __ __ __ __ __ __ __ __ __ __ __ __ __ __ __ __ __ __ __ __ __ __ __ __ __ __ __ __ __ __ __ __ __ __ __ __ __ __ __ __ __ __ __ __ __ __ __ __ __ __ __ __ __ __ __ __ __ __ __ __ __ __ __ __ __ __ __ __ __ __ __ __ __ __ __ __ __ __ __ __ __ __ __ __ _
_ __ __ __ __ __ __ __ __ __ __ __ __ __ __ __ __ __ __ __ __ __ __ __ __ __ __ __ __ __ __ __ __ __ __ __ __ __ __ __ __ __ __ __ __ __ __ __ __ __ __ __ __ __ __ __ __ __ __ __ __ __ __ __ __ __ __ __ __ __ __ __ __ __ __ __ __ __ __ __ __ __ __ __ __ __ __ __ __ __ __ __ __ __ __ _
_ __ __ __ __ __ __ __ __ __ __ __ __ __ __ __ __ __ __ __ __ __ __ __ __ __ __ __ __ __ __ __ __ __ __ __ __ __ __ __ __ __ __ __ __ __ __ __ __ __ __ __ __ __ __ __ __ __ __ __ __ __ __ __ __ __ __ __ __ __ __ __ __ __ __ __ __ __ __ __ __ __ __ __ __ __ __ __ __ __ __ __ __ __ __ _
_ __ __ __ __ __ __ __ __ __ __ __ __ __ __ __ __ __ __ __ __ __ __ __ __ __ __ __ __ __ __ __ __ __ __ __ __ __ __ __ __ __ __ __ __ __ __ __ __ __ __ __ __ __ __ __ __ __ __ __ __ __ __ __ __ __ __ __ __ __ __ __ __ __ __ __ __ __ __ __ __ __ __ __ __ __ __ __ __ __ __ __ __ __ __ _
_ __ __ __ __ __ __ __ __ __ __ __ __ __ __ __ __ __ __ __ __ __ __ __ __ __ __ __ __ __ __ __ __ __ __ __ __ __ __ __ __ __ __ __ __ __ __ __ __ __ __ __ __ __ __ __ __ __ __ __ __ __ __ __ __ __ __ __ __ __ __ __ __ __ __ __ __ __ __ __ __ __ __ __ __ __ __ __ __ __ __ __ __ __ __ _
_ __ __ __ __ __ __ __ __ __ __ __ __ __ __ __ __ __ __ __ __ __ __ __ __ __ __ __ __ __ __ __ __ __ __ __ __ __ __ __ __ __ __ __ __ __ __ __ __ __ __ __ __ __ __ __ __ __ __ __ __ __ __ __ __ __ __ __ __ __ __ __ __ __ __ __ __ __ __ __ __ __ __ __ __ __ __ __ __ __ __ __ __ __ __ _

19) (Essa para os feras!) Sabe-se que a Sintaxe se apresenta como uma das partes das quais se constitui a
gramtica, cuja incumbncia diz respeito s relaes que se estabelecem entre os elementos de um discurso.
Partindo desse princpio, discorra acerca da afinidade estabelecida entre tais conhecimentos e as habilidades da
escrita.
_ __ __ __ __ __ __ __ __ __ __ __ __ __ __ __ __ __ __ __ __ __ __ __ __ __ __ __ __ __ __ __ __ __ __ __ __ __ __ __ __ __ __ __ __ __ __ __ __ __ __ __ __ __ __ __ __ __ __ __ __ __ __ __ __ __ __ __ __ __ __ __ __ __ __ __ __ __ __ __ __ __ __ __ __ __ __ __ __ __ __ __ __ __ __ _
_ __ __ __ __ __ __ __ __ __ __ __ __ __ __ __ __ __ __ __ __ __ __ __ __ __ __ __ __ __ __ __ __ __ __ __ __ __ __ __ __ __ __ __ __ __ __ __ __ __ __ __ __ __ __ __ __ __ __ __ __ __ __ __ __ __ __ __ __ __ __ __ __ __ __ __ __ __ __ __ __ __ __ __ __ __ __ __ __ __ __ __ __ __ __ _
_ __ __ __ __ __ __ __ __ __ __ __ __ __ __ __ __ __ __ __ __ __ __ __ __ __ __ __ __ __ __ __ __ __ __ __ __ __ __ __ __ __ __ __ __ __ __ __ __ __ __ __ __ __ __ __ __ __ __ __ __ __ __ __ __ __ __ __ __ __ __ __ __ __ __ __ __ __ __ __ __ __ __ __ __ __ __ __ __ __ __ __ __ __ __ _
_ __ __ __ __ __ __ __ __ __ __ __ __ __ __ __ __ __ __ __ __ __ __ __ __ __ __ __ __ __ __ __ __ __ __ __ __ __ __ __ __ __ __ __ __ __ __ __ __ __ __ __ __ __ __ __ __ __ __ __ __ __ __ __ __ __ __ __ __ __ __ __ __ __ __ __ __ __ __ __ __ __ __ __ __ __ __ __ __ __ __ __ __ __ __ _
_ __ __ __ __ __ __ __ __ __ __ __ __ __ __ __ __ __ __ __ __ __ __ __ __ __ __ __ __ __ __ __ __ __ __ __ __ __ __ __ __ __ __ __ __ __ __ __ __ __ __ __ __ __ __ __ __ __ __ __ __ __ __ __ __ __ __ __ __ __ __ __ __ __ __ __ __ __ __ __ __ __ __ __ __ __ __ __ __ __ __ __ __ __ __ _
_ __ __ __ __ __ __ __ __ __ __ __ __ __ __ __ __ __ __ __ __ __ __ __ __ __ __ __ __ __ __ __ __ __ __ __ __ __ __ __ __ __ __ __ __ __ __ __ __ __ __ __ __ __ __ __ __ __ __ __ __ __ __ __ __ __ __ __ __ __ __ __ __ __ __ __ __ __ __ __ __ __ __ __ __ __ __ __ __ __ __ __ __ __ __ _
_ __ __ __ __ __ __ __ __ __ __ __ __ __ __ __ __ __ __ __ __ __ __ __ __ __ __ __ __ __ __ __ __ __ __ __ __ __ __ __ __ __ __ __ __ __ __ __ __ __ __ __ __ __ __ __ __ __ __ __ __ __ __ __ __ __ __ __ __ __ __ __ __ __ __ __ __ __ __ __ __ __ __ __ __ __ __ __ __ __ __ __ __ __ __ _
_ __ __ __ __ __ __ __ __ __ __ __ __ __ __ __ __ __ __ __ __ __ __ __ __ __ __ __ __ __ __ __ __ __ __ __ __ __ __ __ __ __ __ __ __ __ __ __ __ __ __ __ __ __ __ __ __ __ __ __ __ __ __ __ __ __ __ __ __ __ __ __ __ __ __ __ __ __ __ __ __ __ __ __ __ __ __ __ __ __ __ __ __ __ __ _
_ __ __ __ __ __ __ __ __ __ __ __ __ __ __ __ __ __ __ __ __ __ __ __ __ __ __ __ __ __ __ __ __ __ __ __ __ __ __ __ __ __ __ __ __ __ __ __ __ __ __ __ __ __ __ __ __ __ __ __ __ __ __ __ __ __ __ __ __ __ __ __ __ __ __ __ __ __ __ __ __ __ __ __ __ __ __ __ __ __ __ __ __ __ __ _
_ __ __ __ __ __ __ __ __ __ __ __ __ __ __ __ __ __ __ __ __ __ __ __ __ __ __ __ __ __ __ __ __ __ __ __ __ __ __ __ __ __ __ __ __ __ __ __ __ __ __ __ __ __ __ __ __ __ __ __ __ __ __ __ __ __ __ __ __ __ __ __ __ __ __ __ __ __ __ __ __ __ __ __ __ __ __ __ __ __ __ __ __ __ __ _
_ __ __ __ __ __ __ __ __ __ __ __ __ __ __ __ __ __ __ __ __ __ __ __ __ __ __ __ __ __ __ __ __ __ __ __ __ __ __ __ __ __ __ __ __ __ __ __ __ __ __ __ __ __ __ __ __ __ __ __ __ __ __ __ __ __ __ __ __ __ __ __ __ __ __ __ __ __ __ __ __ __ __ __ __ __ __ __ __ __ __ __ __ __ __ _
_ __ __ __ __ __ __ __ __ __ __ __ __ __ __ __ __ __ __ __ __ __ __ __ __ __ __ __ __ __ __ __ __ __ __ __ __ __ __ __ __ __ __ __ __ __ __ __ __ __ __ __ __ __ __ __ __ __ __ __ __ __ __ __ __ __ __ __ __ __ __ __ __ __ __ __ __ __ __ __ __ __ __ __ __ __ __ __ __ __ __ __ __ __ __ _

20) (IBGE) Assinale a opo que apresenta a regncia verbal incorreta, de acordo com a norma culta da lngua:

a) Os sertanejos aspiram a uma vida mais confortvel.
b) Obedeceu rigorosamente ao horrio de trabalho do corte da cana.
c) O rapaz presenciou o trabalho dos canavieiros.
d) O fazendeiro agrediu-lhe sem necessidades.
e) Ao assinar o contrato, o usineiro visou, apenas, ao lucro pretendido.


21) ENEM 2007
A figura abaixo parte de uma campanha publicitria.

















Essa campanha publicitria relaciona-se diretamente com a seguinte afirmativa:
a) O comrcio ilcito da fauna silvestre, atividade de grande impacto, uma ameaa para a biodiversidade nacional.
b) A manuteno do mico-leo-dourado em jaula a medida que garante a preservao dessa espcie animal.
c) O Brasil, primeiro pas a eliminar o trfico do mico-leo-dourado, garantiu a preservao dessa espcie.
d) O aumento da biodiversidade em outros pases depende do comrcio ilegal da fauna silvestre brasileira.
e) O trfico de animais silvestres benfico para a preservao das espcies, pois garante-lhes a sobrevivncia.






48
22) (Enem 2007)

Antigamente
Acontecia o indivduo apanhar constipao; ficando perrengue, mandava o prprio chamar o doutor e, depois, ir
botica para aviar a receita, de cpsulas ou plulas fedorentas. Doena nefasta era a phtsica, feia era o glico.
Antigamente, os sobrados tinham assombraes, os meninos, lombrigas (...)
Carlos Drummond de Andrade. Poesia completa e prosa. Rio de Janeiro: Companhia Jos Aguilar, p. 1.184.

O texto acima est escrito em linguagem de uma poca passada. Observe uma outra verso, em linguagem atual.

Antigamente
Acontecia o indivduo apanhar um resfriado; ficando mal, mandava o prprio chamar o doutor e, depois, ir farmcia
para aviar a receita, de cpsulas ou plulas fedorentas. Doena nefasta era a tuberculose, feia era a sfilis.
Antigamente, os sobrados tinham assombraes, os meninos, vermes (...)

Comparando-se esses dois textos, verifica-se que, na segunda verso, houve mudanas relativas a
a) vocabulrio.
b) construes sintticas.
c) pontuao.
d) fontica.
e) regncia verbal.

23) (Enem 2009) Quanto s variantes lingusticas presentes no texto, a norma padro da lngua portuguesa
rigorosamente obedecida por meio


















a) do emprego do pronome demonstrativo "esse" em "Por que o senhor publicou esse livro?".
b) do emprego do pronome pessoal oblquo em "Meu filho, um escritor publica um livro para parar de escrev-lo!".
c) do emprego do pronome possessivo "sua" em "Qual foi sua maior motivao?".
d) do emprego do vocativo "Meu filho", que confere fala distanciamento do interlocutor.
e) da necessria repetio do conectivo no ltimo quadrinho.

24) ENEM 2007
Se a explorao descontrolada e predatria verificada atualmente continuar por mais alguns anos, pode-se antecipar a
extino do mogno. Essa madeira j desapareceu de extensas reas do Par, de Mato Grosso, de Rondnia, e h
indcios de que a diversidade e o nmero de indivduos existentes podem no ser suficientes para garantir a
sobrevivncia da espcie a longo prazo. A diversidade um elemento fundamental na sobrevivncia de qualquer ser
vivo. Sem ela, perde-se a capacidade de adaptao ao ambiente, que muda tanto por interferncia humana como por
causas naturais.
(Internet: www.greenpeace.org.br, com adaptaes)
Com relao ao problema descrito no texto, correto afirmar que
a) a baixa adaptao do mogno ao ambiente amaznico causa da extino dessa madeira.

49
b) a extrao predatria do mogno pode reduzir o nmero de indivduos dessa espcie e prejudicar sua diversidade
gentica.
c) as causas naturais decorrentes das mudanas climticas globais contribuem mais para a extino do mogno que a
interferncia humana.
d) a reduo do nmero de rvores de mogno ocorre na mesma medida em que aumenta a diversidade biolgica
dessa madeira na regio amaznica.
e) o desinteresse do mercado madeireiro internacional pelo mogno contribuiu para a reduo da explorao predatria
dessa espcie.


25) Enem 2004


Da minha aldeia vejo quanto da terra se pode ver no Universo...
Por isso minha aldeia grande como outra qualquer
Porque sou do tamanho do que vejo
E no do tamanho da minha altura... (Alberto Caeiro)

A tira Hagar e o poema de Alberto Caeiro (um dos heternimos de Fernando Pessoa) expressam, com linguagens
diferentes, uma mesma ideia: a de que a compreenso que temos do mundo condicionada, essencialmente,
a) pelo alcance de cada cultura.
b) pela capacidade visual do observador.
c) pelo senso de humor de cada um.
d) pela idade do observador.
e) pela altura do ponto de observao.



















50











PROF.: ELIEL DE QUEIROZ





C
C
o
o
n
n
t
t
e
e

d
d
o
o
:
:

M
M
O
O
R
R
F
F
O
O
L
L
O
O
G
G
I
I
A
A
D
D
O
O
P
P
O
O
R
R
T
T
U
U
G
G
U
U

S
S
.
.


M
M
i
i
n
n
i
i
s
s
t
t
r
r
a
a
d
d
o
o
d
d
u
u
r
r
a
a
n
n
t
t
e
e
o
o
m
m

s
s
d
d
e
e
m
m
a
a
r
r

o
o






















D Da at ta a d da a E En nt tr re eg ga a : : _ __ __ __ __ __ __ __ _/ /_ __ __ __ __ __ __ __ __ __ __ _/ /2 20 01 13 3
LISTA 2 GRAMTICA

51


01) O estudo dos aspectos morfolgicos, pertencente a uma das divises da Gramtica (Morfologia), abarca, entre
outros aspectos, acerca da formao, estrutura e classificao das palavras, o que nos permite estabelecer
familiaridade com os substantivos, adjetivos, advrbios, verbos, entre outras classes gramaticais. Assim, com
base nessa premissa, o exerccio a seguir, extrado de uma questo de vestibular, aponta:

Indique a alternativa em que s aparecem substantivos abstratos:
a) tempo, angstia, saudade, ausncia, esperana, imagem
b) angstia, choro, sol, presena, esperana, amizade
c) amigo, dor, claridade, esperana, luz, tempo
d) angstia, saudade, presena, esperana, amizade
e) espao, mos, claridade, rosto, ausncia, esperana

2) Eis abaixo listados alguns vocbulos. Ao analis-los, identifique os morfemas ora requisitados:
CANTVAMOS
Radical -
Vogal temtica -
Desinncia modo-temporal -
Desinncia nmero-pessoal -

FALARIAM
Radical -
Vogal temtica -
Desinncia modo-temporal -
Desinncia nmero-pessoal -

GAROTO
Radical
Desinncia nominal de gnero -

3) (UFSCar- SP) Assinale a alternativa em que o elemento mrfico em destaque est corretamente analisado.

a) menina (-a) = desinncia nominal de gnero
b) vendeste (-e)= desinncia vogal de ligao
c) gasmetro (-)= vogal temtica de segunda conjugao
d) amassem (-sse)= desinncia de segunda pessoa do plural
e) cantareis (-is)= desinncia do imperfeito do subjuntivo

4) (UFRJ) Assinale a alternativa cujo prefixo sub tem sentido de posterioridade.
a) sublinhar
b) subsequente
c) subdesenvolvido
d) subjacente
e) submisso
5) Partindo do pressuposto de que os morfemas se definem como as menores unidades significativas que se
combinam para formar as palavras, cite os casos representativos que demarcam tal ocorrncia lingustica.
_______________________________________________________________________________
_______________________________________________________________________________
_______________________________________________________________________________
_______________________________________________________________________________
_______________________________________________________________________________


Caderno de Atividades

Disciplina:
G Gr ra am m t ti ic ca a

Professor(a):
Eliel
Aluno:
3 ano
Ensino Mdio
Data de Recebimento:
_____/_____/_____
Lista 02
Data Entrega:
_____/_____/_____

52
6) Levando em conta seus conhecimentos acerca das diferenas que demarcam a classe e a funo, leia, analise e
aponte a que classe e a que funo os elementos dos quais se constituem os enunciados lingusticos em questo
pertencem:
a Meu amigo, traga-me o livro amanh.
b As crianas, sozinhas, brincavam no ptio.
c As questes foram realizadas pelos alunos.
d Naquela empresa precisa-se de funcionrios experientes.
_______________________________________________________________________________
_______________________________________________________________________________
_______________________________________________________________________________
_______________________________________________________________________________
_______________________________________________________________________________
_______________________________________________________________________________
_______________________________________________________________________________
7) Analise os enunciados que seguem e, em seguida, responda ao que se pede:
I Os alunos deixaram destruda a quadra de esportes.
II Os alunos deixaram a quadra de esportes destruda.
a Pode-se afirmar que ambos os enunciados se encontram harmoniosamente construdos no que tange aos aspectos
sintticos?
_______________________________________________________________________________
_______________________________________________________________________________
_______________________________________________________________________________
_______________________________________________________________________________
_______________________________________________________________________________
b Em qual classe morfolgica poderamos enquadrar o termo destruda, em destaque?
_______________________________________________________________________________
_______________________________________________________________________________
_______________________________________________________________________________
_______________________________________________________________________________
_______________________________________________________________________________
c Ainda fazendo referncia a esse mesmo termo, sobretudo no que diz respeito funo por ele desempenhada,
pode-se afirmar que se trata da mesma nos dois casos?
_______________________________________________________________________________
_______________________________________________________________________________
_______________________________________________________________________________
_______________________________________________________________________________
_______________________________________________________________________________
8) Analise atentamente o poema a seguir, atentando-se para as questes pertinentes ao mesmo:
F o r m a
R e f o r m a
D i s f o r m a
T r a n s f o r m a
C o n f o r m a
I n f o r m a
F o r m a

53
a Estamos diante de um poema concreto de autoria de Jos Lino Grnewald. De acordo com o processo ligado
formao de palavras, conceitue-o, levando em considerao o radical.
_______________________________________________________________________________
_______________________________________________________________________________
_______________________________________________________________________________
_______________________________________________________________________________
_______________________________________________________________________________
b Indique a classe gramatical a que pertence o radical e as demais palavras que dele se originaram.
_______________________________________________________________________________
_______________________________________________________________________________
_______________________________________________________________________________
_______________________________________________________________________________
_______________________________________________________________________________
9) Analisando os versos de Guimares Rosa: Eu juro que nunca vi moa to bonitonazinha como a senhora (...) / E
bala um pedacinhozinho de metal.
a Descreva a referida ocorrncia levando em considerao o processo que constitui a formao das palavras.
_______________________________________________________________________________
_______________________________________________________________________________
_______________________________________________________________________________
_______________________________________________________________________________
_______________________________________________________________________________
10) Relacione a 2 coluna de acordo com o seguinte cdigo:
( I) Derivao prefixal ( ) pombo-correio
( II) Derivao sufixal ( ) os guerrilheiros efetuaram ataques relmpagos
(III) Derivao parassinttica ( ) retroagir
(IV) Derivao imprpria ( ) deslocamento
(V) Aglutinao ( ) planalto
(VI) Justaposio ( ) entristecer
11) Jos Paulo Paes utiliza tambm de um recurso lingustico relacionado estrutura das palavras. Veja:
Seu metalxico
economiopia
desenvolvimentir
utopiada
consumidoidos
patriotrios
suicidados
(Os melhores poemas de Jos Paulo Paes)

54
a O poeta, usufruindo-se de sua habilidade artstica, cria um jogo de palavras a partir de outras j existentes na
lngua. Analise cada verso indicando as palavras de origem.
_______________________________________________________________________________
_______________________________________________________________________________
_______________________________________________________________________________
_______________________________________________________________________________
_______________________________________________________________________________
b Explicite seus conhecimentos relatando qual foi o processo a que se deve a tal ocorrncia.
_______________________________________________________________________________
_______________________________________________________________________________
_______________________________________________________________________________
_______________________________________________________________________________
_______________________________________________________________________________
12) (Ita SP) Considere as seguintes significaes:

Escolha a alternativa cujas palavras traduzem os significados apresentados acima:
a) pentgono, plutocracia, eufonia, mialgia
b) enegono, oligarquia, eufonia, cefalalgia
c) nonangular, democracia, cacofonia, dispneia
d) enegono, aristocracia, sinfonia, cefalalgia
e) hendecgono, monarquia, sonoplastia, cefaleia
12) Baseando-se nas formas verbais em evidncia, retrate acerca dos elementos que as integram, tendo em vista o
processo de estruturao das palavras.
a) ESTUDVAMOS
b) ESTUDSSEIS
c) ESTUDARIAM
13) Atribua a devida definio aos elementos abaixo descritos:
a) radical
b) afixos
c) desinncias
d) vogal temtica
e) vogal de ligao
f) sufixos


55
14) (FGV-RJ) Assinale o item em que h erro quanto anlise da forma verbal cantvamos:
a) cant- radical
b) -- vogal temtica
c) canta- tema
d) -va- desinncia de pretrito imperfeito do subjuntivo
e) -mos desinncia de 1 pessoa do plural
15) Eis que abaixo se evidencia o refro da msica Cabelo, de Jorge Ben Jor e Arnaldo Antunes. Com base na
anlise deste, procure responder ao que se pede:
Cabelo, cabeleira
Cabeluda, descabela
Cabelo, cabeleira
Cabeluda, descabelada...
Disponvel em: http://letras.terra.com.br/jorge-ben-jor/86143/
a) Entre os elementos que formam a estrutura de uma palavra, h um elemento comum a vrios vocbulos,
denominado de radical. Identifique-o no fragmento em questo.
_______________________________________________________________________________
_______________________________________________________________________________
_______________________________________________________________________________
_______________________________________________________________________________

b) Percebemos que a partir desse mesmo radical alguns elementos a ele se juntaram, formando novas unidades de
significao. Com base nesse pressuposto, retrate-os.
_______________________________________________________________________________
_______________________________________________________________________________
_______________________________________________________________________________
_______________________________________________________________________________
16) (UFSCar-SP - adaptada) Assinale a alternativa em que o elemento mrfico em destaque est corretamente
analisado:
a) menina (-a) desinncia nominal de gnero
b) vendeste (-e) vogal de ligao
c) gasmetro (--) vogal temtica de segunda
conjugao
d) amassem (-sse-) desinncia de segunda pessoa
do plural
e) cantareis (-is) desinncia do imperfeito do
subjuntivo
17) Enem 2006 Os textos a seguir foram extrados de duas crnicas publicadas no ano em que a seleo brasileira
conquistou o tricampeonato mundial de futebol.

O General Mdici falou em consistncia moral. Sem isso, talvez a vitria nos escapasse, pois a disciplina consciente,
livremente aceita, vital na preparao espartana para o rude teste do campeonato. Os brasileiros portaram-se no
apenas como tcnicos ou profissionais, mas como brasileiros, como cidados deste grande pas, cnscios de seu
papel de representantes de seu povo. Foi a prpria afirmao do valor do homem brasileiro, como salientou bem o
presidente da Repblica. Que o chefe do governo aproveite essa pausa, esse minuto de euforia e de efuso patritica,
para meditar sobre a situao do pas. (...) A realidade do Brasil a exploso patritica do povo ante a vitria na Copa.
(Danton Jobim. ltima Hora, 23/6/1970 - com adaptaes).

O que explodiu mesmo foi a alma, foi a paixo do povo: uma exploso incomparvel de alegria, de entusiasmo, de

56
orgulho. (...) Debruado em minha varanda de Ipanema, [um velho amigo] perguntava: Ser que algum terrorista se
aproveitou do delrio coletivo para adiantar um plano seu qualquer, agindo com frieza e preciso? Ser que, de outro
lado, algum carrasco policial teve nimo para voltar a torturar sua vtima logo que o alemo apitou o fim do jogo?
(Rubem Braga. ltima Hora, 25/6/1970 - com adaptaes).

Avalie as seguintes afirmaes a respeito dos dois textos e do perodo histrico em que foram escritos.

I - Para os dois autores, a conquista do tricampeonato mundial de futebol provocou uma exploso de alegria popular.
II - Os dois textos salientam o momento poltico que o pas atravessava ao mesmo tempo em que conquistava o
tricampeonato.
III - poca da conquista do tricampeonato mundial de futebol, o Brasil vivia sob regime militar, que, embora
politicamente autoritrio, no chegou a fazer uso de mtodos violentos contra seus opositores.

correto apenas o que se afirma em
a) I.
b) II.
c) III.
d) I e II.
e) II e III.

18) ENEM Um jornal de circulao nacional publicou a seguinte notcia:
"Choveu torrencialmente na madrugada de ontem em Roraima, horas depois de os pajs caiaps Mantii e Kucrit,
levados de Mato Grosso pela Funai, terem participado do ritual da dana da chuva, em Boa Vista. A chuva durou trs
horas em todo o estado e as previses indicam que continuar pelo menos at amanh. Com isso, ser possvel
acabar de vez com o incndio que ontem completou 63 dias e devastou parte das florestas do estado".
Jornal do Brasil, abr./1998 (com adaptaes).

Considerando a situao descrita, avalie as afirmativas seguintes.
I. No ritual indgena, a dana da chuva, mais que constituir uma manifestao artstica, tem a funo de intervir no ciclo
da gua.
II. A existncia da dana da chuva em algumas culturas est relacionada importncia do ciclo da gua para a vida.
III. Uma das informaes do texto pode ser expressa em linguagem cientfica da seguinte forma: a dana da chuva
seria efetiva se provocasse a precipitao das gotculas de gua das nuvens.

correto o que se afirma em
a) I, apenas.
b) III, apenas.
c) I e II, apenas.
d) II e III, apenas.
e) I, II e III.


19) ENEM 2007 H cerca de dez anos, estimava-se que 11,2% da populao brasileira poderiam ser considerados
dependentes de lcool. Esse ndice, dividido por gnero, apontava que 17,1% da populao masculina e 5,7% da
populao feminina eram consumidores da bebida. Quando analisada a distribuio etria desse consumo, outro
choque: a pesquisa evidenciou que 41,2% de estudantes da educao bsica da rede pblica brasileira j haviam feito
uso de lcool.
Dados atuais apontam que a porcentagem de dependentes de lcool subiu para 15%. Estima-se que o pas gaste 7,3%
do PIB por ano para tratar de problemas relacionados ao alcoolismo, desde o tratamento de pacientes at a perda da
produtividade no trabalho. A indstria do lcool no Brasil, que produz do acar ao lcool combustvel, movimenta
3,5% do PIB.
Revista Brasileira de Psiquiatria, v. 28, n. 4, dez/2006 e Internet: www.alcoolismo.com.br (com adaptaes)

A partir dos dados acima, conclui-se que
a) o pas, para tratar pessoas com problemas provocados pelo alcoolismo, gasta o dobro do que movimenta para
produzir bebida alcolica.
b) o aumento do nmero de brasileiros dependentes de lcool acarreta decrscimo no percentual do PIB gasto no
tratamento dessas pessoas.
c) o elevado percentual de estudantes que j consumiram bebida alcolica indicativo de que o consumo do lcool
problema que deve ser enfrentado pela sociedade.
d) as mulheres representam metade da populao brasileira dependente de lcool.

57
e) o aumento na porcentagem de brasileiros dependentes de lcool deveu-se, basicamente, ao crescimento da
indstria do lcool.

20) Enem 2008 Assinale o trecho do dilogo que apresenta um registro informal, ou coloquial, da
linguagem.



a) "T legal, espertinho! Onde que voc esteve?!?
b) "E lembre-se: se voc disser uma mentira, os seus chifres cairo!"
c) "Estou atrasado porque ajudei uma velhinha a atravessar a rua..."
d) "...e ela me deu um anel mgico que me levou a um tesouro."
e) "mas bandidos o roubaram e os persegui at a Etipia, onde um drago..."

21) Enem 2004


Da minha aldeia vejo quanto da terra se pode ver no Universo...
Por isso minha aldeia grande como outra qualquer
Porque sou do tamanho do que vejo
E no do tamanho da minha altura... (Alberto Caeiro)

A tira Hagar e o poema de Alberto Caeiro (um dos heternimos de Fernando Pessoa) expressam, com
linguagens diferentes, uma mesma ideia: a de que a compreenso que temos do mundo condicionada,
essencialmente,
a) pelo alcance de cada cultura.
b) pela capacidade visual do observador.

58
c) pelo senso de humor de cada um.
d) pela idade do observador.
e) pela altura do ponto de observao.


22) ENEM 2008
Calcula-se que 78% do desmatamento na Amaznia tenha sido motivado pela pecuria cerca de 35% do rebanho
nacional est na regio e que pelo menos 50 milhes de hectares de pastos so pouco produtivos. Enquanto o custo
mdio para aumentar a produtividade de 1 hectare de pastagem de 2 mil reais, o custo para derrubar igual rea de
floresta estimado em 800 reais, o que estimula novos desmatamentos. Adicionalmente, madeireiras retiram as
rvores de valor comercial que foram abatidas para a criao de pastagens. Os pecuaristas sabem que problemas
ambientais como esses podem provocar restries pecuria nessas reas, a exemplo do que ocorreu em 2006 com o
plantio da soja, o qual, posteriormente, foi proibido em reas de floresta.
poca, 3/3/2008 e 9/6/2008 (com adaptaes).
A partir da situao-problema descrita, conclui-se que
a) o desmatamento na Amaznia decorre principalmente da explorao ilegal de rvores de valor comercial.
b) um dos problemas que os pecuaristas vm enfrentando na Amaznia a proibio do plantio de soja.
c) a mobilizao de mquinas e de fora humana torna o desmatamento mais caro que o aumento da produtividade de
pastagens.
d) o supervit comercial decorrente da exportao de carne produzida na Amaznia compensa a possvel degradao
ambiental.
e) a recuperao de reas desmatadas e o aumento de produtividade das pastagens podem contribuir para a reduo
do desmatamento na Amaznia.



































59













PROF.: ELIEL DE QUEIROZ






C
C
o
o
n
n
t
t
e
e

d
d
o
o
:
:
L
L
E
E
X
X
I
I
C
C
O
O
L
L
O
O
G
G
I
I
A
A
D
D
O
O
P
P
O
O
R
R
T
T
U
U
G
G
U
U

S
S
e
e

E
E
S
S
T
T
U
U
D
D
O
O
D
D
A
A
F
F
R
R
A
A
S
S
E
E
N
N
O
O
T
T
E
E
X
X
T
T
O
O
.
.


M
M
i
i
n
n
i
i
s
s
t
t
r
r
a
a
d
d
o
o
d
d
u
u
r
r
a
a
n
n
t
t
e
e
o
o
m
m

s
s
d
d
e
e
a
a
b
b
r
r
i
i
l
l






















D Da at ta a d da a E En nt tr re eg ga a : : _ __ __ __ __ __ __ __ _/ /_ __ __ __ __ __ __ __ __ __ __ _/ /2 20 01 13 3

LISTA 3 GRAMTICA

60


01-(UFG-2002) Examine as capas de revista abaixo:


Escombros do
World Trade Center,
na tera-feira 11
18 set. 2001 19 set. 2001 17 set. 2001

As capas das trs revistas semanais de maior circulao no pas tratam dos atentados ocorridos nos EUA, em 11 de
setembro de 2001.
As trs questes a que voc responder, em seguida, referem-se edio de texto e de imagem desse material
jornalstico.

- Diante do impacto dos incidentes e de suas conseqncias, a imprensa construiu a notcia de diferentes maneiras.
Tendo em vista essa possibilidade de abordar o mesmo fato de modos distintos, considere os itens abaixo:

01- Explique a relao existente entre a imagem e o ttulo O imprio vulnervel, presentes na capa da revista Veja.
_______________________________________________________________________________________________
_______________________________________________________________________________________________
_______________________________________________________________________________________________
_______________________________________________________________________________________________
______________________________________________________________________________________________

02- Nas expresses nominais, as posies de ncleo (substantivo) e de modificador (adjetivo) tm relao com aquilo
que se quer focalizar. Reescreva a manchete da revista Veja invertendo essas posies sintticas, ou seja, transforme
o ncleo em modificador e o modificador em ncleo.

_______________________________________________________________________________________________
_______________________________________________________________________________________________
_______________________________________________________________________________________________
_______________________________________________________________________________________________
______________________________________________________________________________________________
- Leia as frases abaixo, presentes, respectivamente, no subttulo e na legenda da capa de ISTO:

1. Saddam Hussein suspeito de ter fornecido apoio logstico.
2. Saddam Hussein, o pai de todas as encrencas de Tio Sam.




Caderno de Atividades

Disciplina:
G Gr ra am m t ti ic ca a

Professor(a):
Eliel
Aluno:
3 ano
Ensino Mdio
Data de Recebimento:
_____/_____/_____
Lista 03
Data Entrega:
_____/_____/_____

61
03- Comparando as duas frases acima, possvel perceber uma contradio gerada pelo uso de alguns vocbulos.
Explique-a.

_______________________________________________________________________________________________
_______________________________________________________________________________________________
_______________________________________________________________________________________________
_______________________________________________________________________________________________
______________________________________________________________________________________________

04- Reescreva as frases em um s perodo de modo a eliminar a contradio.

_______________________________________________________________________________________________
_______________________________________________________________________________________________
_______________________________________________________________________________________________
_______________________________________________________________________________________________
______________________________________________________________________________________________

A manchete estampada na capa da revista poca indica um propsito militar.

De acordo com o sentido gerado pela interao que se estabelece entre o verbal (palavra) e o no-verbal (imagem),
nesse tipo de publicao, amplie a manchete Guerra contra o terror, acrescentando o que se pede:

05- Na primeira verso, uma expresso nominal que possa estar subentendida.
_______________________________________________________________________________________________
_______________________________________________________________________________________________
_______________________________________________________________________________________________
_______________________________________________________________________________________________
______________________________________________________________________________________________

06- Na segunda verso, uma expresso nominal e uma expresso verbal que possam estar subentendidas.

_______________________________________________________________________________________________
_______________________________________________________________________________________________
_______________________________________________________________________________________________
_______________________________________________________________________________________________
______________________________________________________________________________________________

07- Explicao de como possvel recuperar os elementos lingsticos que se encontram subentendidos.

_______________________________________________________________________________________________
_______________________________________________________________________________________________
_______________________________________________________________________________________________
_______________________________________________________________________________________________
______________________________________________________________________________________________

-(UNESP) Na morte dos rios
Desde que no Alto Serto um rio seca,
a vegetao em volta, embora de unhas,
embora sabres, intratvel e agressiva,
faz alto beira daquele leito tumba.
Faz alto agresso nata: jamais ocupa
O rio de ossos de areia, de areia mmia.
(Joo Cabral de Melo Neto)

Joo Cabral de Melo Neto pretendeu criar uma linguagem para seus poemas que se afastasse um pouco da linguagem
usual, por meio de pequenos desvios. Para isso, empregou, s vezes, palavras fora das classes morfolgicas a que
pertencem.

08- Transcreva os fragmentos em que isso acontece.
_______________________________________________________________________________________________
_______________________________________________________________________________________________
_______________________________________________________________________________________________
_______________________________________________________________________________________________
______________________________________________________________________________________________

62
09- Identifique a classe original das palavras e a classe em que Joo Cabral as utilizou em seu poema.
_______________________________________________________________________________________________
_______________________________________________________________________________________________
_______________________________________________________________________________________________
_______________________________________________________________________________________________
______________________________________________________________________________________________

10- Na viso de Joo Cabral, como era a vegetao no Alto Serto? Fundamente sua resposta.

_______________________________________________________________________________________________
_______________________________________________________________________________________________
_______________________________________________________________________________________________
_______________________________________________________________________________________________
______________________________________________________________________________________________

- (UFPE 2000) A vida CURTA. CURTA !

Num jogo de linguagem, os dois termos destacados tm sentidos diferentes.

11- Indique um sinnimo para cada um dos termos.

_______________________________________________________________________________________________
_______________________________________________________________________________________________
_______________________________________________________________________________________________
_______________________________________________________________________________________________
______________________________________________________________________________________________

12- Reescreva os perodos, explicitando, por meio de um conectivo, a relao semntica estabelecida entre os dois.

_______________________________________________________________________________________________
_______________________________________________________________________________________________
_______________________________________________________________________________________________
_______________________________________________________________________________________________
______________________________________________________________________________________________


13- A que classes gramaticais pertencem as expresses destacadas no perodo?

_______________________________________________________________________________________________
_______________________________________________________________________________________________
_______________________________________________________________________________________________
_______________________________________________________________________________________________
______________________________________________________________________________________________

- (ITA 2001)

Leia o texto seguinte:

A aposentada A. S., 68, tomou na semana passada uma deciso macabra em relao ao seu futuro. Ela pegou o
dinheiro de sua aposentadoria (um salrio mnimo) e comprou um caixo.
A. mora com a irm, M. F., 70, que tambm aposentada. Elas no tm parentes. A. diz que est investindo no futuro.
Sua irm a apia. A. tambm comprou a mortalha roupa que quer usar quando morrer. O caixo fica guardado na
sala da casa. (Aposentada compra caixo para o futuro. Folha de S. Paulo, 22/8/1992, adapatado.)

14- Localize um trecho que revela ironia.
_______________________________________________________________________________________________
_______________________________________________________________________________________________
_______________________________________________________________________________________________
_______________________________________________________________________________________________
______________________________________________________________________________________________





63
15- Explique como se d esse efeito de ironia.

_______________________________________________________________________________________________
_______________________________________________________________________________________________
_______________________________________________________________________________________________
_______________________________________________________________________________________________
______________________________________________________________________________________________

- (UFJF-MG) Leia com ateno, o fragmento de texto abaixo, produzido por uma aluna de ensino mdio (F. G.), para
um concurso de redao, com a temtica: Redes do futuro: inteligncia, ignorncia ou loucura.


(...) A Internet est muito presente na vida das pessoas; desde crianas brincando com joguinhos, adolescentes
nos Chat de bate-papo, at adultos checando suas contas bancrias. Esse um fenmeno que, a cada ano que
passa, atinge as pessoas cada vez mais cedo. Muitos de ns criticam as pessoas que dependem de algo, como
drogas, chocolates. Porm, devemos parar para pensar at que ponto nossa relao com as mquinas sadia; afinal
impossvel haver uma desvinculao, somos dependentes destes.
Contudo, traz inmeras vantagens; praticidade e maior rapidez na maneira de se corresponder com as
pessoas, os e-mails facilitam a vida de inmeros indivduos, as compras, pesquisas, compartilhamento de informaes,
ajuda no diagnstico de doenas, bancos de sangue, doao de rgos, so feitos facilmente, em segundos, de um
lado ao outro do mundo. Est trazendo algumas mudanas de hbitos no benficas, como acabar com a prtica da
leitura. Os livros foram postos de lado; os resumos esto sobrepondo os clssicos literrio(...).
Reproduzido exatamente como no site www.conhecimento,usp.br

16- Identifique e explique o problema de referncia na forma destacada destes.

_______________________________________________________________________________________________
_______________________________________________________________________________________________
_______________________________________________________________________________________________
_______________________________________________________________________________________________
______________________________________________________________________________________________

17- Leia o segundo pargrafo do fragmento de texto e complete as lacunas abaixo com os recursos coesivos mais
adequados:
Contudo,.......traz inmeras vantagens (...) .......est trazendo algumas mudanas(...)

_______________________________________________________________________________________________
_______________________________________________________________________________________________
_______________________________________________________________________________________________
_______________________________________________________________________________________________
______________________________________________________________________________________________

18- Justifique as escolhas feitas na questo anterior.

_______________________________________________________________________________________________
_______________________________________________________________________________________________
_______________________________________________________________________________________________
_______________________________________________________________________________________________
______________________________________________________________________________________________

19- (FCC-BA) O trao todo da vida para muitos um desenho de criana esquecido pelo homem, e ao qual este ter
sempre de se cingir sem o saber...
A qual palavra do texto se refere diretamente o pronome este?

_______________________________________________________________________________________________
_______________________________________________________________________________________________
_______________________________________________________________________________________________
_______________________________________________________________________________________________
______________________________________________________________________________________________





64
20- (FGV-SP) A frase abaixo foi extrada de recente anncio para a venda de um imvel. Comente o uso que nela se
faz do pronome demonstrativo isso.
- Isso aqui o Paraso.
_______________________________________________________________________________________________
_______________________________________________________________________________________________
_______________________________________________________________________________________________
_______________________________________________________________________________________________
______________________________________________________________________________________________

21- Reescreva duas vezes a passagem abaixo, sem alterar-lhe o sentido, de modo que sejam substitudos os termos
em destaque, evitando-se a repetio.

Diz o historiador haver no pas um convvio pacfico entre explorados e exploradores. Ocorre porm que, enquanto os
explorados vivem em condies de pobreza, s vezes at de misria total, os exploradores passeiam sorridentes
pelos shopping centers, procura de suprfluos.

_______________________________________________________________________________________________
_______________________________________________________________________________________________
_______________________________________________________________________________________________
_______________________________________________________________________________________________
______________________________________________________________________________________________

- Matte a vontade. Matte Leo. (UNICAMP-2007-2 FASE).

Este enunciado faz parte de uma propaganda afixada nos lugares nos quais se vende ch Matte Leo. Observe as
construes abaixo, feitas a partir do enunciado em questo:

Matte vontade.
Mate a vontade.
Mate vontade.

22- Complete cada uma das construes acima com palavras ou expresses que explicitem as leituras possveis
relacionadas propaganda.

_______________________________________________________________________________________________
_______________________________________________________________________________________________
_______________________________________________________________________________________________
_______________________________________________________________________________________________
______________________________________________________________________________________________

23- Retome a propaganda e explique o seu funcionamento, explicitando as relaes morfolgicas, sintticas e
semnticas envolvidas.

_______________________________________________________________________________________________
_______________________________________________________________________________________________
_______________________________________________________________________________________________
_______________________________________________________________________________________________
______________________________________________________________________________________________

- (FUVEST adaptada) Um jornal era isso, o sobressalto da novidade e a garantia de que a nossa rotina continuava.
Simultaneamente um espalhafato - um espalha fatos e um repetidor das nossas confortveis banalidades municipais.
(L.F, Verssimo, O ESTADO DE S. PAULO, 18/10/98, D7)

24- Interprete o jogo de palavras entre ESPALHAFATO E ESPALHA FATOS, considerando-o no contexto
do trecho anterior. Depois d a casse gramatical dos termos espalhafato e espalha de acordo com o
texto.
_______________________________________________________________________________
_______________________________________________________________________________
_______________________________________________________________________________
_______________________________________________________________________________
_______________________________________________________________________________
_______________________________________________________________________________

65
25- A qual dos termos do primeiro perodo se refere a expresso confortveis banalidades municipais?

_______________________________________________________________________________
_______________________________________________________________________________
_______________________________________________________________________________
_______________________________________________________________________________
_______________________________________________________________________________
_______________________________________________________________________________












































66










PROF.: ELIEL DE QUEIROZ






C
C
o
o
n
n
t
t
e
e

d
d
o
o
:
:

I
I
N
N
T
T
E
E
L
L
E
E
C
C

O
O
(
(
F
F
U
U
V
V
E
E
S
S
T
T
E
E
U
U
E
E
G
G
)
)
.
.





M
M
i
i
n
n
i
i
s
s
t
t
r
r
a
a
d
d
o
o
d
d
u
u
r
r
a
a
n
n
t
t
e
e
o
o
m
m

s
s
d
d
e
e
m
m
a
a
i
i
o
o















D Da at ta a d da a E En nt tr re eg ga a : : _ __ __ __ __ __ __ __ _/ /_ __ __ __ __ __ __ __ __ __ __ _/ /2 20 01 13 3
LISTA 4 GRAMTICA

67

01- Leia o texto a seguir e responda:
(UFG-2 FASE)
Menina mal amada

[...] Minha me, muito viva, isolava-se em seu mundo de frustraes, ligada maternalmente caula do seu
terceiro casamento. Eu, perna mole, pandorga, moleirona, vencendo sozinha as etapas destes primeiros
tempos. Afinal, paramos no dtraqu. Tudo isso, aumentava minha solido e eu me fechava, circunscrita no
meu mundo de faz-de-conta... [...]
CORALINA, Cora. Melhores poemas de Cora Coralina. Seleo de Darci F. Denfrio. So Paulo: Global,
2004. p. 117.
A partir do cenrio idealizado no fragmento do poema, explique o sentido que a palavra muito adquire no
texto.
_______________________________________________________________________________
_______________________________________________________________________________
_______________________________________________________________________________
_______________________________________________________________________________
_______________________________________________________________________________
_______________________________________________________________________________
_______________________________________________________________________________

- Leia o texto seguinte e responda s questes 02 e 03.

Desculpe-nos pela demora em responder a sua reclamao sobre a sua TV de plasma. Precisvamos ter a
certeza de que a nossa matriz aqui no Brasil estaria nos enviando a referida pea. Na prxima semana,
estaremos fazendo uma reviso geral no aparelho e vamos estar enviando ele para o senhor.
Atenciosamente...
(Texto do e-mail de uma empresa, justificando o atraso em consertar um aparelho eletrnico.)

Observa-se, nesse texto, um problema de estilo comum nas correspondncias comerciais e nas
comunicaes de tele-marketing e tambm um desvio da norma padro do portugus do Brasil.

02- Identifique o problema de estilo e redija o texto em que ele ocorre, corrigido.
_______________________________________________________________________________
_______________________________________________________________________________
_______________________________________________________________________________
_______________________________________________________________________________
_______________________________________________________________________________
_______________________________________________________________________________
_______________________________________________________________________________

03- Argumente sobre esse problema de estilo.
_______________________________________________________________________________
_______________________________________________________________________________
_______________________________________________________________________________
_______________________________________________________________________________
_______________________________________________________________________________
_______________________________________________________________________________
_______________________________________________________________________________
Caderno de Atividades

Disciplina:
G Gr ra am m t ti ic ca a

Professor(a):
Eliel
Aluno:
3 ano
Ensino Mdio
Data de Recebimento:
_____/_____/_____
Lista 03
Data Entrega:
_____/_____/_____

68
- Leia a tira abaixo e responda s questes de 04 a 07.

Tirinha #01


04- A partir do conceito de lngua, o uso da expresso VC rompe com as propostas gramaticais da lngua
portuguesa? Fundamente sua resposta.
_______________________________________________________________________________
_______________________________________________________________________________
_______________________________________________________________________________
_______________________________________________________________________________
_______________________________________________________________________________
_______________________________________________________________________________
_______________________________________________________________________________

05- A expresso VC adequada a que tipo de comunicao? Explique a sua resposta.
_______________________________________________________________________________
_______________________________________________________________________________
_______________________________________________________________________________
_______________________________________________________________________________
_______________________________________________________________________________
_______________________________________________________________________________
_______________________________________________________________________________

06- Identifique duas marcas de informalidade presentes no texto e exemplifique-as.
_______________________________________________________________________________
_______________________________________________________________________________
_______________________________________________________________________________
_______________________________________________________________________________
_______________________________________________________________________________
_______________________________________________________________________________
_______________________________________________________________________________

07- Identifique duas marcas de formalidade presentes na tira e exemplifique-as.
_______________________________________________________________________________
_______________________________________________________________________________
_______________________________________________________________________________
_______________________________________________________________________________
_______________________________________________________________________________
_______________________________________________________________________________
_______________________________________________________________________________





69
08- Leia o texto abaixo:

Horscopo
ASTECA
Seu bom humor e espontaneidade a fazem popular entre os amigos. Est sempre alegre, divertida e
adora ouvir o que eles tm para contar. Faz qualquer sacrifcio em nome da amizade e conquista as
pessoas ao se tornar confidente delas. O trabalho no prioridade na sua vida. Voc no se deixa
escravizar pela carreira, mas alcana o sucesso profissional mesmo assim. Embora no deixe transparecer,
tem problemas como todo mundo e nem sempre consegue resolv-los.
Para ser feliz: decida impasses com mais rapidez. Sua hesitao pode emperrar projetos.

Identifique trs marcas de formalidade presentes no texto e exemplifique-as.
______________________________________________________________________________________
_______________________________________________________________________________
_______________________________________________________________________________
_______________________________________________________________________________
_______________________________________________________________________________
_______________________________________________________________________________
_______________________________________________________________________________

- Leia atentamente o texto dado e procure identificar:


09- Que tipo de texto observa-se?
________________________________________
________________________________________
________________________________________
________________________________________
________________________________________
________________________________________
________________________________________
________________________________________
________________________________________
________________________________________
_________________________________________________________________________

10- Caracterize a estrutura desse texto.
_______________________________________________________________________________
_______________________________________________________________________________
_______________________________________________________________________________
_______________________________________________________________________________
_______________________________________________________________________________
_______________________________________________________________________________

11- Leia o texto seguir e responda:
L1: No bem comunicao, transporte.
L2: Pra mim , ainda...
L1: Transporte, no, acho comunicao...
L2: Voc comunica diferentes pontos da cidade quando voc...sabe? faz com que pessoas que antes
teriam acesso ou mais difcil, ou no teriam...de um ponto para outro...
L1: No...Mas vem da conotao de comunicao, hein?
L2: Ahn ahn.
L1: Isso a seria um...
L2: mercrio. (ri)
L1: ... diferente...certo?

70
L2: Mas em suma, acho que...sabe, est ligado a todo um contexto de...que...
L1: Tira tira tira o contexto de humano essa comunicao... Comunicao de transporte comunicao no
humana n?... Por exemplo voc est em guerra o importante voc acabar com as comunicaes...n?
Ento...voc destri uma ponte e fica isolado assim da...

Projeto NURC/SP

O texto acima reproduz a conversao entre dois interlocutores L1 e L2 e, como voc percebeu, a
comunicao estabelece porque ambos esto situados num mesmo contexto.
Levante do texto trs caractersticas pertinentes s marcas de oralidade presentes, exemplificando-as.

_______________________________________________________________________________
_______________________________________________________________________________
_______________________________________________________________________________
_______________________________________________________________________________
_______________________________________________________________________________
_______________________________________________________________________________
_______________________________________________________________________________

12- Leia este trecho da letra de uma msica do folclore mineiro:

Cuitelinho
Cheguei na beira do porto
Onde as onda se espaia
As gara d meia volta
E senta na beira da praia
E o cuitelinho no gosta
Que o boto de rosa caia.[...]
Milton Nascimento e Wagner Tiso
In A arte de Milton Nascimento. Rio de Janeiro, 1988.

A linguagem do texto favorece sua expressividade potica. Justifique.
_______________________________________________________________________________
_______________________________________________________________________________
_______________________________________________________________________________
_______________________________________________________________________________
_______________________________________________________________________________
_______________________________________________________________________________
_______________________________________________________________________________

-(FUVEST) Um jornal era isso, o sobressalto da novidade e garantia de que a nossa rotina continuava.
Simultaneamente um espalhafato- um espalha fatos- e um repetidor das nossas confortveis banalidades
municipais.
(L.F. Verssimo, O ESTADO DE SO PAULO, 18/10/98, D7)

13- Interprete o jogo de palavras entre ESPALHAFATO e ESPALHA FATOS, considerando-o no contexto do
trecho anterior.
_______________________________________________________________________________
_______________________________________________________________________________
_______________________________________________________________________________
_______________________________________________________________________________
_______________________________________________________________________________
_______________________________________________________________________________
_______________________________________________________________________________



71
14- A qual dos termos do primeiro perodo se refere a expresso confortveis banalidades municipais?

_______________________________________________________________________________
_______________________________________________________________________________
_______________________________________________________________________________
_______________________________________________________________________________
_______________________________________________________________________________
_______________________________________________________________________________
_______________________________________________________________________________

- (FUVEST 2004) Compare o provrbio Por fora bela viola, por dentro po bolorento com a seguinte
mensagem publicitria de um empreendimento imobilirio:


Por fora as mais belas rvores. Por dentro a melhor planta.

15- Os recursos sonoros utilizados no provrbio mantm-se na mensagem publicitria? Justifique a sua
resposta.
_______________________________________________________________________________
_______________________________________________________________________________
_______________________________________________________________________________
_______________________________________________________________________________
_______________________________________________________________________________
_______________________________________________________________________________
______________________________________________________________________________

16- Aponte o jogo de palavras que ocorre no texto publicitrio, mas no no provrbio.
_______________________________________________________________________________
_______________________________________________________________________________
_______________________________________________________________________________
_______________________________________________________________________________
_______________________________________________________________________________
_______________________________________________________________________________
_______________________________________________________________________________

17- (UEG-2002/1).

Os provrbios tm origem annima e, apesar de sua aparente simplicidade, exprimem,
de forma prtica e condensada, certas crenas e vises de mundo que fazem parte do imaginrio de um
povo ou de certo grupo social.
No trecho abaixo, Joelmir Betting faz um comentrio sobre a situao crtica do Brasil,
provocada pela alta do petrleo da OPEP: Estamos perdendo nas duas pontas: na alta dos barris e na alta
dos juros. Mxico, Venezuela, Equador e Indonsia, exportadores de petrleo,recuperam nos barris e
perdem nos juros da dvida.
FOLHA DE S. PAULO. 20 set. 1990. p.15.

Marque a alternativa cujo provrbio sintetiza a situao crtica do Brasil, apontada no
trecho acima.
a) Uns plantam, outros colhem.
b) O bom-bocado no para quem faz, para quem come.
c) Mais vale um pssaro na mo do que dois voando.
d) Quando a pedra cai no vaso, o vaso quebra, azar do vaso; quando o vaso cai na pedra, o vaso quebra,
azar do vaso.
e) Quando um no quer, dois no brigam.



72
As questes 18 e 19 referem-se ao texto que segue (UFG-2004).

A qumica do amor
Esquea a velha mxima que diz que os opostos se
atraem. O conceito, afirmam cientistas, s vale para a
fsica e no passa de mito em matria de relacionamentos.
Para os bilogos Peter Buston e Stephen Emlen, da
Universidade de Cornell, em Nova York, a escolha de um
parceiro baseada na preferncia por pessoas que se
assemelham a ns mesmos. Quem busca um companheiro
com valores parecidos com os seus acaba enfrentando
menos conflitos no relacionamento. Por isso, tem
mais chances de estabelecer laos duradouros e criar
filhos com sucesso, explica Emlen. O estudo contradiz
algumas noes que temos sobre as diferentes estratgias
de acasalamento praticadas por machos e fmeas,
derivadas da teoria do naturalista ingls Charles Darwin
(1809-1882) e defendidas, hoje, pela psicologia evolutiva.
Hoje, a cincia j interpreta a formao de casais luz
dos elementos culturais e comea a abrir espao para
contestaes. Afinal, existe a frmula do amor? Os especialistas afirmam que no.
ARTONI, Camila. A qumica do amor. Galileu. Rio de Janeiro: Globo, n. 146,
set. 2003, p. 63. [Adaptado].

18- Destaca-se no texto o recurso argumentativo de
(A) desqualificao do oponente.
(B) formao de consenso.
(C) reunio de provas concretas.
(D) exposio de vocabulrio tcnico.
(E) citao de autoridade.

19- A expresso os opostos se atraem retomada, de forma
mais ampla e abstrata, por
(A) estratgias de acasalamento.
(B) frmula do amor.
(C) conflitos no relacionamento.
(D) laos duradouros.
(E) elementos culturais.

20- Reconhea os paralelismos empregados nestes textos:
a- No perodo de um ano, cresceu no s o nmero de assaltos e roubos na cidade mas, mas tambm o
de assassinatos por armas de fogos.
_______________________________________________________________________________
_______________________________________________________________________________
_______________________________________________________________________________
_______________________________________________________________________________
_______________________________________________________________________________
_______________________________________________________________________________
_______________________________________________________________________________

b- Hoje em dia , quanto mais uma pessoa estuda, tanto mais ela tem condies de competir no mercado de
trabalho.
_______________________________________________________________________________
_______________________________________________________________________________
_______________________________________________________________________________
_______________________________________________________________________________
_______________________________________________________________________________

73
c- Afirma-se que a pobreza do Brasil erradicvel. Para faz-lo so necessrias duas coisas: primeiro,
decidir que isso que se quer; segundo, dar apoios institucionais a quem j trabalha com a pobreza.

_______________________________________________________________________________
_______________________________________________________________________________
_______________________________________________________________________________
_______________________________________________________________________________
_______________________________________________________________________________
_______________________________________________________________________________
_______________________________________________________________________________

d- No pudemos viajar no carnaval passado, nem nas frias de julho, nem nas de janeiro.
_______________________________________________________________________________
_______________________________________________________________________________
_______________________________________________________________________________
_______________________________________________________________________________
_______________________________________________________________________________
_______________________________________________________________________________
_______________________________________________________________________________

e- Se todos colaborassem, tudo seria mais fcil.
_______________________________________________________________________________
_______________________________________________________________________________
_______________________________________________________________________________
_______________________________________________________________________________
_______________________________________________________________________________
_______________________________________________________________________________
_______________________________________________________________________________

- Leia a charge a seguir:
Quando os eruditos descobriram a lngua, ela j estava completamente pronta pelo povo. Os
eruditos tiveram apenas que proibir o povo de falar errado



74
21- A qual acontecimento social a tira faz referncia? Fundamente sua resposta com base na charge.
_______________________________________________________________________________
_______________________________________________________________________________
_______________________________________________________________________________
_______________________________________________________________________________
_______________________________________________________________________________
_______________________________________________________________________________
_______________________________________________________________________________

- Observe a charge seguinte:




22- A charge faz referncia a qual acontecimento poltico-social? Fundamente sua resposta.

_______________________________________________________________________________
_______________________________________________________________________________
_______________________________________________________________________________
_______________________________________________________________________________
_______________________________________________________________________________
_______________________________________________________________________________
_______________________________________________________________________________
23- Que sentido semntico podemos retirar da expresso lcool, claro!?

_______________________________________________________________________________
_______________________________________________________________________________
_______________________________________________________________________________
_______________________________________________________________________________
_______________________________________________________________________________
_______________________________________________________________________________
_______________________________________________________________________________

- Leia atentamente o fragmento abaixo:
O que vinha dizendo que o projeto: o que a gente v de projeto que, dentro do nosso Brasil, maior parte
da riqueza todinha projeto. E cada projeto que aparece e no fim !!! Os pobres ...quem dana nos
projetinho, n? quem dana porque...a gente v que seu Fulano chegou fazendo um trabalho desse,
nem um carro para andar tem, no tem nada na vida, que quando com um ano que passa mais a
gente, dois anos,no chega mais dois anos, no...A vai logo com um carrinho do ano de uma banda
e a gente dancemo.
Antnio T. Montenegro. Histria oral e memria. Contexto: S. Paulo, p.25.

75
24- Transcreva trs caractersticas especficas da linguagem oral, do trecho transcrito em destaque.

_______________________________________________________________________________
_______________________________________________________________________________
_______________________________________________________________________________
_______________________________________________________________________________
_______________________________________________________________________________
_______________________________________________________________________________
_______________________________________________________________________________

25- Compare essas trs caractersticas com outras trs da linguagem escrita, presente no texto, explicando
suas diferenas.
_______________________________________________________________________________
_______________________________________________________________________________
_______________________________________________________________________________
_______________________________________________________________________________
_______________________________________________________________________________
_______________________________________________________________________________
_______________________________________________________________________________




76







PROF.: ELIEL DE QUEIROZ





C
C
o
o
n
n
t
t
e
e

d
d
o
o
:
:

N
N
O
O
R
R
M
M
A
A
P
P
A
A
D
D
R
R

O
O
E
E
S
S
I
I
N
N
T
T
A
A
X
X
E
E
D
D
O
O

P
P
O
O
R
R
T
T
U
U
G
G
U
U

S
S
I
I
I
I
I
I
(
(
I
I
T
T
A
A
E
E
U
U
F
F
G
G
)
)
.
.





M
M
i
i
n
n
i
i
s
s
t
t
r
r
a
a
d
d
o
o
d
d
u
u
r
r
a
a
n
n
t
t
e
e
o
o
m
m

s
s
d
d
e
e
j
j
u
u
n
n
h
h
o
o















D Da at ta a d da a E En nt tr re eg ga a : : _ __ __ __ __ __ __ __ _/ /_ __ __ __ __ __ __ __ __ __ __ _/ /2 20 01 13 3
LISTA 5 GRAMTICA

77



01- Leia o trecho a seguir e responda;

A princesa Diana j passou por poucas e boas. Tipo quando
o ex-marido Charles tece um love affair com Lady Camille
revelado para Deus e o mundo.
(Folha de S. Paulo, 5/11/93

No texto acima, h expresses que fogem ao padro culto da lngua escrita.

a) Identifique-as.
_______________________________________________________________________________________________
_______________________________________________________________________________________________
_______________________________________________________________________________________________
_______________________________________________________________________________________________
______________________________________________________________________________________________

b) Reescreva conforme o padro culto.
_______________________________________________________________________________________________
_______________________________________________________________________________________________
_______________________________________________________________________________________________
_______________________________________________________________________________________________
______________________________________________________________________________________________

02- Observe este texto, criado para propaganda de embalagens:

Ao final do processo de reciclagem, aquele lixo de lata vira lata de
luxo, embalando as bebidas que todo mundo gosta, das marcas que
todo mundo pode confiar.

Reescreva, corrigindo-os, os seguimentos do texto que apresentem algum desvio em relao norma gramatical.
_______________________________________________________________________________________________
_______________________________________________________________________________________________
_______________________________________________________________________________________________
_______________________________________________________________________________________________
______________________________________________________________________________________________

03- Reelabore o dilogo abaixo, usando o nvel formal:

_ O meu, v se no me deixa numa furada. Essa de pagar mico toda hora j ta me azucrinando todo e mais, no
arrasta-p das minas l no morro, no vai aprontar pra cima de mim.
_ Podes crer, irmo! No vou deixar a peteca cair e nem dar mancada. O lance o seguinte: a amizade aqui vai sacar
uma mina que um estouro e voc vai ficar babando!
_______________________________________________________________________________________________
_______________________________________________________________________________________________
_______________________________________________________________________________________________
_______________________________________________________________________________________________
______________________________________________________________________________________________

- Aponte, nos textos a seguir, os casos de regncia verbal e nominal; mostre tambm os termos regente e regido.

04-

_________________________
_________________________
_________________________
_________________________
_________________________
_________________________
Caderno de Atividades

Disciplina:
G Gr ra am m t ti ic ca a

Professor(a):
Eliel
Aluno:
3 ano
Ensino Mdio
Data de Recebimento:
_____/_____/_____
Lista 05
Data Entrega:
_____/_____/_____

78
_____________________________________________________________________________________________
______________________________________________________________________________________________


____________________________________________
____________________________________________
____________________________________________
____________________________________________
____________________________________________
____________________________________________
____________________________________________
____________________________________________
____________________________________________
____________________________________________
____________________________________________
____________________________________________

07- H, evidentemente, um jogo de palavras no anncio no exerccio 6. Qual ?
_______________________________________________________________________________________________
_______________________________________________________________________________________________
_______________________________________________________________________________________________
_______________________________________________________________________________________________
______________________________________________________________________________________________

08- Que mudana de sentido foi promovida pela mudana de regncia do verbo falar?
_______________________________________________________________________________________________
_______________________________________________________________________________________________
_______________________________________________________________________________________________
_______________________________________________________________________________________________
______________________________________________________________________________________________

09- Leia com ateno a seguinte frase de um texto publicitrio:

Esta a escola que os pais confiam.
Identifique a preposio exigida pelo verbo e refaa a construo, obedecendo norma gramatical. Justifique a
correo.
_______________________________________________________________________________________________
_______________________________________________________________________________________________
_______________________________________________________________________________________________
_______________________________________________________________________________________________
______________________________________________________________________________________________

- Numa cidade, porta de uma escola, est escrito:

Estacionamento exclusivo para alunos e pais da escola.
Sobre a frase acima responda:

10- A que interpretao absurda se pode chegar?
_______________________________________________________________________________________________
_______________________________________________________________________________________________
_______________________________________________________________________________________________

79
_______________________________________________________________________________________________
______________________________________________________________________________________________
11- O que causou essa interpretao?
_______________________________________________________________________________________________
_______________________________________________________________________________________________
_______________________________________________________________________________________________
_______________________________________________________________________________________________
______________________________________________________________________________________________

12- Reescreva a frase evitando o entendimento indesejado.

_______________________________________________________________________________________________
_______________________________________________________________________________________________
_______________________________________________________________________________________________
_______________________________________________________________________________________________
______________________________________________________________________________________________

13- Assinale a alternativa em que a regncia do verbo implicar est incorreta:
a) Ele nos implicou mais ainda
b) O estudo implica disposio e disciplina
c) Ele sempre implicou comigo
d) Fomos implicados no movimento grevista
e) Seu talento implicou numa promoo.

- Nas reprodues a seguir, identifique os possveis erros de regncia. Corrija-os e explique as correes feitas

14-

15-









16-
______________________________
______________________________
______________________________
______________________________
______________________________
______________________________
______________________________
______________________________
______________________________
______________________________
______________________________
______________________________
______________________________
______________________________
______________________________
______________________________
______________________________
______________________________
______________________________
______________________________
______________________________
______________________________
______________________________
______________________________
______________________________
______________________________
______________________________
______________________________
______________________________
______________________________
______________________________
______________________________
______________________________
______________________________
______________________________
______________________________

80

17-


Turistas de toda parte gostam de ver e participar das lindas festas folclricas brasileiras.

18- (UFV) Reescreva o texto abaixo, passando MUDANA para o plural e substituindo PROBLEMA por
IRREGULARIDADES. Faa as alteraes estritamente necessrias para que o texto continue adequado ao uso culto
da lngua. Uma vez que nesta semana dever ser providenciada a mudana que se fizer necessria na portaria de
prdio, pedimos aos senhores condminos que, quando ocorrer qualquer problema relacionado ao recebimento de
suas correspondncias, comunique o fato ao sndico.
_______________________________________________________________________________________________
_______________________________________________________________________________________________
_______________________________________________________________________________________________
_______________________________________________________________________________________________
_______________________________________________________________________________________________
_______________________________________________________________________________________________

Um homem precisa viajar. Por sua conta, no por meio de histrias, imagens, livros ou TV. Precisa viajar por si, com
seus olhos e ps, para entender o que seu. Para um dia plantar as suas prprias rvores e dar-lhes valor.

Conhecer o frio para conhecer o calor. E o oposto. Sentir a distncia e o desabrigo para estar bem sob o prprio teto.
Um homem precisa viajar para lugares que no conhece para quebrar essa arrogncia que nos faz ver o mundo como
o imaginamos, e no simplesmente como ou pode ser; que nos faz professores e doutores do que no vimos,
quando deveramos ser alunos, e simplesmente ir ver.

Amyr Klink, Mar sem fim.
_________________________
_________________________
_________________________
_________________________
_________________________
_________________________
_________________________
_________________________
_________________________
___________________________
___________________________
___________________________
_____________________

___________________________
___________________________
___________________________
___________________________
___________________________
___________________________
___________________________
___________________________
___________________________
______________________________
______________________________
______________________________


81
19- (Fuvest SP/2006) Na frase retirada do texto acima, que nos faz professores e doutores do que no vimos, o
pronome sublinhado retoma a expresso antecedente:
a) para lugares.
b) o mundo.
c) um homem.
d) essa arrogncia.
e) como imaginamos.

20- (UFAL) Assinale como VERDADEIRAS as frases em que a CONCORDANCIA est correta e como FALSAS
aquelas em que isso no ocorre, justificando as falsas.

(___) Foi feito com arte todos os cartazes alusivo ao Congresso a ser realizado brevemente.
(___) Soavam trs horas quando, atnitos, os convidados se deram conta da ausncia do don da casa.
(___) Tornou-se necessrio providenciar urgentes no sentido de se recuperar os objetos roubados.
(___) Talvez hajam contradies, ao mesmo tempo que poder existir acordos entre os scios.
(___) Levou-se em conta as respostas referente ao questionrio elaborado pelo professor.
_______________________________________________________________________________________________
_______________________________________________________________________________________________
_______________________________________________________________________________________________
_______________________________________________________________________________________________
______________________________________________________________________________________________

- (UFG/2006) Leia os textos a seguir:
Texto 1 O cientista da cozinha

Revista Gula: [...] Qual o mtodo de suas pesquisas?

Herv This: No fundo, trata-se de verificar o que h de falso ou verdadeiro nas afirmaes categricas das receitas.
Pegue qualquer livro de receitas e encontrar algo do tipo bata as claras em neve. E l vou eu. Como isso? Por que
acontece? verdade que necessrio bater as claras? E por qu? Preciso responder a essas questes, e, se elas so
falsas, empenho-me em tentar mostrar cientificamente que so apenas procedimentos que se repetem por dcadas
sem necessidade. Essa a funo da gastronomia molecular, testar as receitas e criar outras.
GULA. N. 153. So Paulo. Jul. 2005, p. 20 [Adaptado].
Texto 2 Ingredientes de um molho maionese
3 gemas de ovos
xcara (ch) de azeite
1 colher (ch) de mostarda
1 colher (sopa) de suco de limo
1 colher (sopa) de vinagre
1 pitada de sal
WELLS, Patrcia. Cozinha de bistr. Rio de Janeiro: Ediouro, 1993. p. 276-277. [Adaptado].

Ingredientes e procedimentos adequados so fundamentais para o sucesso de uma receita culinria, universo a que
esto relacionados os textos 1 e 2.

21- Quanto ao modo de olhar de olhar as receitas culinrias, o que distingue a gastronomia molecular da gastronomia
tradicional?
_______________________________________________________________________________________________
_______________________________________________________________________________________________
_______________________________________________________________________________________________
_______________________________________________________________________________________________
______________________________________________________________________________________________

22- Elabore o Modo de fazer do molho maionese, considerando a lista de ingredientes do texto 2 e o trecho instrucional
na fala de Herv This, no texto 1.
_______________________________________________________________________________________________
_______________________________________________________________________________________________
_______________________________________________________________________________________________
_______________________________________________________________________________________________
______________________________________________________________________________________________





82
23 (ITA/2004) Leia o texto abaixo e responda s perguntas seguintes:

O sol ainda nascendo, dou a volta pela Lagoa Rodrigo de Freitas (7.450 metros e 22 centmetros) Deslumbrante.
Paro diante de uma placa da Prefeitura, feita com os maiores cuidados tcnicos, em bela tipografia, em portugus e
ingls, naturalmente escrita por altos professores e, no longo perodo com que trabalham as burocracias, vista e revista
por engenheiros, psiclogos, enfim, por toda espcie e gnero de PhDs. Certo disso, leio, cheio do desejo de
aprender, a histria da lagoa e seus dintorni, environs, neighbourhood.
L est escrito: beleza cnica integrada aos contornos dos morros que a cerca (!). Berro, no portugus mais
castio do manual do [jornal] Globo: HELP!
, como isso no tem a menor importncia, o sol continua nascendo no horizonte. Um luxo!

(FERNANDES, Millr. Caderno 2. O Estado de S. Paulo, 4/07/1999).
a) Explique por que Millr Fernandes se assusta com a placa da Prefeitura.
_______________________________________________________________________________________________
_______________________________________________________________________________________________
_______________________________________________________________________________________________
_______________________________________________________________________________________________
______________________________________________________________________________________________
b) Localize no texto um trecho que indica a ironia do autor. Explique como produzido o efeito de ironia nesse trecho.
_______________________________________________________________________________________________
_______________________________________________________________________________________________
_______________________________________________________________________________________________
_______________________________________________________________________________________________
______________________________________________________________________________________________
24 (UFG/OS/2006) Leia o texto para responder ao que se pede.
Grvida no encontra remdio caro em SP
[..] A diarista Maria do Carmo Brando, 32, no oitavo ms de uma gravidez de risco, aguarda
atendimento na Casa de Sade da Mulher Prof. Domingos Delascio, SP. Ela diz no ter problemas com a rede
de sade [..]. Maria do Carmo tem os melhores mdicos, exames, mas a ateno sua sade no integral, pena
para achar o remdio para toxoplasmose, doena que porta e pode causar deformaes no beb se no tratada.
Como a Casa no tem autorizao para distribuir o remdio de alto custo, as mulheres so obrigadas a procur-lo em
centros de distribuio especfica, onde no fcil acha-lo, relata.
FOLHA DE S. PAULO. So Paulo, 15 fev. 2005, p. C3, Cotidiano.
O titulo da reportagem permite mais de uma interpretao, ambigidade que desfeita com a leitura do texto. Elabore
duas interpretaes possveis para esse ttulo, indicando aquela que corresponde ao contedo da matria.
_______________________________________________________________________________________________
_______________________________________________________________________________________________
_______________________________________________________________________________________________
_______________________________________________________________________________________________
______________________________________________________________________________________________
25- Observe a imagem a seguir:

A imagem de uma
campanha publicitria
de uma organizao
no governamental,
Greenpeace,retrata
uma das cenas de
seca ocorrida na
Amaznia em 2005.
Explique como o efeito
de sentido pretendido
por essa campanha
atinge o leitor.

___________________________________
___________________________________
___________________________________
___________________________________
___________________________________
___________________________________
___________________________________
_____________
___________________________________
___________________________________

83





PROF.: ELIEL DE QUEIROZ






C
C
o
o
n
n
t
t
e
e

d
d
o
o
:
:

S
S
I
I
N
N
T
T
A
A
X
X
E
E
T
T
E
E
X
X
T
T
U
U
A
A
L
L
(
(
U
U
E
E
G
G
e
e

MARKENZIE
)
)
.
.






M
M
i
i
n
n
i
i
s
s
t
t
r
r
a
a
d
d
o
o
d
d
u
u
r
r
a
a
n
n
t
t
e
e
o
o
m
m

s
s
d
d
e
e
a
a
g
g
o
o
s
s
t
t
o
o














D Da at ta a d da a E En nt tr re eg ga a : : _ __ __ __ __ __ __ __ _/ /_ __ __ __ __ __ __ __ __ __ __ _/ /2 20 01 13 3
LISTA 6 GRAMTICA

84

-(UEG adaptada 2004) Leia a tira a seguir:


DICK, Browne. Hagar, o horrvel. In: AZEREDO, Jos Carlos de. (Org.). Lngua portuguesa em debate. Petrpolis: Vozes, 2000. p.
60.

01- Explique como o jogo de sentido entre linguagem metafrica e linguagem literal produz o efeito de humor na tira.

_______________________________________________________________________________________________
_______________________________________________________________________________________________
_______________________________________________________________________________________________
_______________________________________________________________________________________________
______________________________________________________________________________________________

02- Classifique as oraes do 1 balo(1 quadrinho) da tira.
_______________________________________________________________________________________________
_______________________________________________________________________________________________
_______________________________________________________________________________________________
_______________________________________________________________________________________________
______________________________________________________________________________________________

- (UEG-2004) Leia atentamente o texto abaixo.

Amarre-se na vida.
Use o cinto de segurana

DETRAN-DF. Placa de trnsito. Braslia.
- Faa o que se pede:

03- Interprete a dupla possibilidade de leitura do verbo amarrar.
_______________________________________________________________________________________________
_______________________________________________________________________________________________
_______________________________________________________________________________________________
_______________________________________________________________________________________________
______________________________________________________________________________________________

04-Aponte dois recursos usados pelo emissor da mensagem para convencer o destinatrio da importncia do uso do
cinto de segurana.
_______________________________________________________________________________________________
_______________________________________________________________________________________________
_______________________________________________________________________________________________
_______________________________________________________________________________________________
______________________________________________________________________________________________

Caderno de Atividades

Disciplina:
G Gr ra am m t ti ic ca a

Professor(a):
Eliel
Aluno:
3 ano
Ensino Mdio
Data de Recebimento:
_____/_____/_____
Lista 06
Data Entrega:
_____/_____/_____

85
-(UEG-2004) Leia o trecho a seguir:

Fernando Pessoa quem diria? foi idelogo da Coca-Cola... Quem me contou foi o professor Ademar Ferreira dos
Santos, educador portugus, da Escola da Ponte, em Portugal.
Pois ele, o professor, me contou que, nos idos dos anos 20, Fernando Pessoa, precisando ganhar dinheiro para
sobreviver, passou a trabalhar para uma empresa de propaganda. A Coca-Cola estava chegando, era desconhecida,
de gosto estranho, e precisava de uma cunha potica que lhe abrisse o caminho.
Foi ento que o escritor portugus produziu este curtssimo slogan: A princpio estranha-se. Depois, entranha-se.
Absolutamente genial!
O fragmento de texto acima de um artigo de Rubem Alves, publicado no encarte Sinapse da Folha de S. Paulo, em
24 de junho de 2003.

05- Explique a funo das aspas no slogan.

_______________________________________________________________________________________________
_______________________________________________________________________________________________
_______________________________________________________________________________________________
_______________________________________________________________________________________________
______________________________________________________________________________________________

06-Faa uma parfrase do slogan de Fernando Pessoa, mantendo, porm, o efeito de contraste produzido pelo jogo
das palavras estranha-se e entranha-se.

_______________________________________________________________________________________________
_______________________________________________________________________________________________
_______________________________________________________________________________________________
_______________________________________________________________________________________________
______________________________________________________________________________________________

07-(UEG-2004) Leia com ateno o trecho abaixo, publicado no jornal da Associao Brasileira de Engenharia
Sanitria e Ambiental de Gois, ABES-GO.
Embora a gua seja o bem mais abundante no planeta cobrindo dois teros de sua superfcie a porcentagem de gua
doce existente no mundo no suficiente para abastecer toda a populao estudos apontam que haver escassez do
produto nas prximas dcadas por outro lado o direito gua est assegurada na Declarao Universal dos Direitos do
Homem fato que tambm torna a espcie humana responsvel pela sua preservao
Reescreva o texto, pontuando-o e indicando com maisculas os incios de perodo. Ao fazer a transcrio, substitua as
palavras sublinhadas por outras equivalentes que preservem o sentido do texto.

_______________________________________________________________________________________________
_______________________________________________________________________________________________
_______________________________________________________________________________________________
_______________________________________________________________________________________________
______________________________________________________________________________________________


-(UEG-2004) Leia com ateno os textos a seguir:

TEXTO 1

A caixa-preta do pas.

CAROS AMIGOS [Anncio publicitrio]. So Paulo: Casa Amarela, ago. 2003. p. 17. [Adaptado].

Aproveite a chance de colecionar as edies que passam o Brasil e o mundo a limpo.
Agora com preos promocionais.

86
Na compra de 2 estojos completos, desconto de 10%

TEXTO 2





CONSULEX. Cidade, 31 jul. 2003.


TEXTO 3

CAIXA-PRETA: aparelhagem que grava dados sobre o funcionamento de uma aeronave, seus sistemas e as
conversas da equipagem; serve para detectar problemas e verificar suas causas em caso de acidente e para controlar
a manuteno da aeronave.
DICIONRIO HOUAISS DA LNGUA PORTUGUESA. Rio de Janeiro: Objetiva, 2001.
Observao: por SISTEMA S entende-se o conjunto de entidades de direito privado, institudas e organizadas por
legislao especfica e dirigidas pelas respectivas Confederaes Nacionais, como o caso do Sebrae, Sesc, Senac,
Sesi, Senat.

A partir da declarao do presidente Lula, h algum tempo, de que era preciso abrir a caixa-preta do judicirio, a
metfora da caixa-preta passou a ser usada com freqncia.

08- Leia atentamente os textos acima e considere as seguintes proposies:

I. Com base no significado encontrado no dicionrio, pode-se dizer que o texto da propaganda (1) usa, de forma
figurada, o termo caixa-preta, com a finalidade de, alm de aludir ao estojo em que se renem vrios nmeros da
Caros Amigos, ressaltar a qualidade da revista, que passa o Brasil e o mundo a limpo. O texto da capa da revista
Consulex (2), ao contrrio, sugere a falta de transparncia do Sistema S, que no esclarece sociedade como um
volume to grande de recursos gasto e em benefcio de quem.

II. No texto 2, a linguagem visual comunica-se com a linguagem verbal de forma direta e clara, pelo formato e cor da
caixa, em contraste com a cor das letras. O mesmo no ocorre com o texto 1, que no permite uma associao entre o
termo caixa-preta e a imagem que se apresenta na propaganda.

III. No trecho S no preo a gente deixa barato, do texto 1, a idia de excluso expressa pelo termo s sugere que,
embora de preo acessvel, a revista detecta os problemas do Brasil e do mundo, investiga suas causas e cobra caro
as decises em relao a tais problemas.

IV. Em preciso abrir esta caixa-preta, h o pressuposto de que, com a abertura da caixa-preta, ser esclarecido
para a sociedade o modo de funcionamento do Sistema S, especialmente, no que diz respeito aplicao dos recursos
financeiros.

Assinale a alternativa CORRETA:

a) As proposies I, II e III so verdadeiras.
b) As proposies I, II e IV so verdadeiras.
c) As proposies I, II, III e IV so verdadeiras.
d) As proposies I, III e IV so verdadeiras.
e) Apenas as proposies III e IV so verdadeiras.


87
- Leia o texto a seguir e responda s questes de 09 a 20.

O rapaz vinha do rio . Descalo, com as calas arregaadas acima do joelho, as pernas sujas de lama. Vestia uma
camisa vermelha, aberta no peito, onde os primeiros plos da puberdade a enegrecer. Tinha o cabelo escuro, molhado
de suor que lhe escorria pelo pescoo delgado. Dobrava-se um pouco para frente, sob o peso dos longos remos,
donde pendiam fios verdes de limos ainda gotejantes. O barco ficou balouando na gua turva, e ali perto, como se o
espreitassem, afloravam de repente os olhos globulosos de uma r. O rapaz olhou-a e ela olhou-o a ele. Depois a r
fez um movimento brusco e desapareceu. Um minuto mais e a superfcie do rio ficou lisa e calma, e brilhante como os
olhos do rapaz. A respirao do lodo desprendia lentas e moles bolhas de gs que a corrente arrastava. No calor
espesso da tarde, os choupos altos vibraram silenciosamente, e, de rajada, flor rpida que do ar nascesse, uma ave
azul passou rasando a gua. O rapaz levantou a cabea. No outro lado do rio, uma rapariga olhava-o, imvel. O rapaz
ergueu a mo livre e todo o seu corpo desenhou o gesto de uma palavra que no se ouviu. O rio flua, lento.

SARAMAGO, Jos. Desforra . In: Objeto quase. So Paulo: Cia. Das Letras, 1998.


Esse o primeiro pargrafo do conto Desforra. Trata-se de um pargrafo descritivo, com pouqussima ao: o
suficiente para fazer os personagens ingressarem na paisagem.

09- Indique as frases que exprimem aes dos dois personagens: o rapaz e a moa.

_______________________________________________________________________________________________
_______________________________________________________________________________________________
_______________________________________________________________________________________________
_______________________________________________________________________________________________
______________________________________________________________________________________________
10- Indique a nica frase descritiva com predicado nominal, que expressa o estado do rio naquele momento. Que
palavras so responsveis, nessa frase, pela descrio do rio?
_______________________________________________________________________________________________
_______________________________________________________________________________________________
_______________________________________________________________________________________________
_______________________________________________________________________________________________
______________________________________________________________________________________________

11- Indique a nica frase nominal presente nesse pargrafo. Qual sua funo?
_______________________________________________________________________________________________
_______________________________________________________________________________________________
_______________________________________________________________________________________________
_______________________________________________________________________________________________
______________________________________________________________________________________________

12- Apesar de tratar de pargrafo descritivo, os elementos que compem a paisagem no so estticos. Tudo parece
ter vida e esse efeito conseguido pela cuidadosa escolha dos verbos. Releia atentamente o pargrafo; destaque
alguns desses verbos e explique os efeitos de movimento e vida que eles produzem.
_______________________________________________________________________________________________
_______________________________________________________________________________________________
_______________________________________________________________________________________________
_______________________________________________________________________________________________
______________________________________________________________________________________________

13- H predominncia no texto de oraes coordenadas ou subordinadas? Fundamente sua resposta com trechos do
prprio texto.
_______________________________________________________________________________________________
_______________________________________________________________________________________________
_______________________________________________________________________________________________
_______________________________________________________________________________________________
______________________________________________________________________________________________

- O pronome pessoal oblquo lhe funciona na frase como complemento do verbo, especificamente como objeto
indireto: No lhe disse nada (= No disse nada ao rapaz / a ele). Mas pode assumir o significado de pronome
possessivo: No lhe ouviu os conselhos (= No lhe ouviu os conselhos.) Indique com (1) as frases em que o
lhe for objeto indireto e com (2) aquelas em que tiver valor possessivo, justificando a sua resposta.



88
14- A rapariga respondeu-lhe com um aceno.

_______________________________________________________________________________________________
_______________________________________________________________________________________________
_______________________________________________________________________________________________
_______________________________________________________________________________________________
______________________________________________________________________________________________

15- A rapariga respondeu-lhe ao aceno.
_______________________________________________________________________________________________
_______________________________________________________________________________________________
_______________________________________________________________________________________________
_______________________________________________________________________________________________
______________________________________________________________________________________________

16- Tinha o cabelo escuro, molhado de suor que lhe escorria pelo pescoo delgado.
_______________________________________________________________________________________________
_______________________________________________________________________________________________
_______________________________________________________________________________________________
_______________________________________________________________________________________________
______________________________________________________________________________________________

17- O rio obedecia-lhe.
_______________________________________________________________________________________________
_______________________________________________________________________________________________
_______________________________________________________________________________________________
_______________________________________________________________________________________________
______________________________________________________________________________________________

18- Viu-lhe as mos.

_______________________________________________________________________________________________
_______________________________________________________________________________________________
_______________________________________________________________________________________________
_______________________________________________________________________________________________
_____________________________________________________________________________________________

O rapaz olhou-a, e ela olhou-o a ele.

19- A que personagens do texto referem-se os pronomes destacados?
_______________________________________________________________________________________________
_______________________________________________________________________________________________
_______________________________________________________________________________________________
_______________________________________________________________________________________________
______________________________________________________________________________________________

20- O que voc notou? Explique a sua resposta.
_______________________________________________________________________________________________
_______________________________________________________________________________________________
_______________________________________________________________________________________________
_______________________________________________________________________________________________
______________________________________________________________________________________________

- No calor espesso da tarde, os choupos altos vibraram silenciosamente, e, de rajada, flor rpida que do ar
nascesse, uma ave azul passou rasando a gua.

21- Indique as circunstncias expressas por silenciosamente e de rajada. Esses termos referem-se a que verbos?

_______________________________________________________________________________________________
_______________________________________________________________________________________________
_______________________________________________________________________________________________
_______________________________________________________________________________________________
______________________________________________________________________________________________


89
22- D a predicao dos verbos citados na questo 21.
_______________________________________________________________________________________________
_______________________________________________________________________________________________
_______________________________________________________________________________________________
_______________________________________________________________________________________________
______________________________________________________________________________________________

23- Explique a relao existente entre a expresso flor rpida que do ar nascesse e uma ave azul.
_______________________________________________________________________________________________
_______________________________________________________________________________________________
_______________________________________________________________________________________________
_______________________________________________________________________________________________
______________________________________________________________________________________________

24- Separe e classifique as oraes do seguinte perodo: No calor espesso da tarde, os choupos altos vibraram
silenciosamente, e, de rajada, flor rpida que do ar nascesse, uma ave azul passou rasando a gua.
_______________________________________________________________________________________________
_______________________________________________________________________________________________
_______________________________________________________________________________________________
_______________________________________________________________________________________________
______________________________________________________________________________________________

25-(MARKENZIE 2007) Texto para os itens de 1 a 4.
Certa vez, chamaram minha ateno para um erro de portugus no
samba Comprimido. a crnica de um sujeito que briga com a
mulher. Ela d uma dentada nele, que resolve deixar a marca para
provar a agresso. Ganhou esse nome para enfatizar a idia de que
o indivduo estava pressionado, a ponto de tomar um comprimido
e morrer. L pelo fim do texto, h o erro: Noite de samba/ Noite
comum de novela/ Ele chegou/ Pedindo um copo dgua/ Pra tomar
um comprimido/ Depois cambaleando/ Foi pro quarto/ E se deitou/
Era tarde demais/ Quando ela percebeu que ele se envenenou. Ento
me deram um toque. A, tentei mudar. Nada encaixava. Um desespero.
A decidi deixar assim, com erro mesmo. Nunca reclamaram.
Adaptado de entrevista de Paulinho da Viola

1- O texto permite afirmar, com correo, que

a) h um contraste entre o nvel de linguagem do relato e o da cano;
nesta, o autor usa de maior de informalidade.
b) a entrevista apresenta, como marca de oralidade, o uso de a (linhas
10 e 11) para conectar partes da narrativa.
c) a letra de Comprimido apresenta diversos deslizes em relao
concordncia.
d) a inverso da ordem comum nas frases que compem os versos de
Comprimido serve para criar suspense em relao ao desfecho da
histria.
e) o entrevistado relata o que lhe aconteceu certa vez dispondo os
fatos em ordem cronolgica, sem fazer uso de interrupes,
explicaes ou comentrios.

2- O erro a que Paulinho da Viola faz aluso

a) relaciona-se com o uso de percebeu, em lugar de notou, que seria
mais apropriado ao contexto.
b) refere-se posio do pronome pessoal se, que deveria vir depois
do verbo envenenar-se (que ele envenenou-se).
c) est no emprego da forma verbal envenenou, que, considerada a
norma culta, deveria ser substituda por envenenara.

3- O texto autoriza afirmar que

a) os desvios em relao ao uso da lngua considerado correto no
incomodam o compositor, porque a popularidade de uma cano

90
tambm depende do emprego da linguagem informal.
b) o equvoco lingstico causou constrangimento e preocupao ao
compositor, que, no entanto, optou por preservar a rima entre os
versos.
c) o alegado erro nunca foi notado pelos fs de Paulinho da Viola,
que, por isso, no reclamaram.
d) o engano foi apontado para o autor em dois momentos distintos:
antes e depois de a composio ser concluda.
e) o compositor, ao que parece, no conseguiu identificar o erro que
lhe fora apontado pelos fs.

4- a crnica de um sujeito que briga com a mulher. Ela d uma dentada
nele, que resolve deixar a marca para provar a agresso.
Preserva-se o mesmo sentido do trecho acima em:

a) a histria de um indivduo que se desentende com a mulher, leva
uma mordida e decide preservar o sinal para provar que foi agredido.
b) um comentrio crtico sobre um fato do cotidiano: a discusso de
um meliante com sua esposa. Ela lhe d uma dentada e ele deixa a
marca para comprovar o insulto de que foi vtima.
c) uma narrao sobre um bate-boca entre um homem e uma mulher.
Ela o morde e resolve deixar a marca para poder dar queixa polcia.
d) um relato sinttico sobre a briga de um senhor com uma mulher.
Ela o agride com os dentes e ele decide permitir que as marcas
permaneam, confirmando, assim, os xingamentos que recebeu.
e) uma espcie de conto que fala da briga de um homem e sua
mulher, com dentadas e marcas que comprovam as mtuas
agresses.


91





PROF.:HERICK







C
C
o
o
n
n
t
t
e
e

d
d
o
o
s
s
:
:

N
N

M
M
E
E
R
R
O
O
S
S
C
C
O
O
M
M
P
P
L
L
E
E
X
X
O
O
S
S

























D Da at ta a d da a E En nt tr re eg ga a : : _ __ __ __ __ __ __ __ _/ /_ __ __ __ __ __ __ __ __ __ __ _/ /2 20 01 13 3


LISTA 1 MATEMTICA

92


Questo 01) A soma
50 49 2 1 0
50
0 j
j
i i ... i i i i S + + + + + = =
=
em que i um nmero complexo, igual a:
a)1+i
b)i
c)1i
d)i

Questo 02) Se x e y so nmeros reais tais que
(

=
(

6 - 8 -
2 0
y y
1 2
.
1 x
1 - x
, ento o mdulo do nmero complexo
yi x z + = igual a
a) 2 2
b)4
c) 5 2
d) 6 2
e)20

Questo 03)

No perodo da Revoluo Cientfica, a humanidade assiste a uma das maiores invenes da Matemtica que ir
revolucionar o conceito de nmero: o nmero complexo. Rafael Bombelli (1526 1572), matemtico italiano, foi o
primeiro a escrever as regras de adio e multiplicao para os nmeros complexos.
Dentre as alternativas a seguir, assinale aquela que indica uma afirmao incorreta.
a)o conjugado de (1 + i) (1 i)
b) 2 i 1 = +
c)(1 + i) raiz da equao 0 2 z 2 z
2
= +
d)(1 + i)
1
= (1 i)
e)(1 + i)
2
= 2i

Questo 04) Sendo 1 i = a unidade imaginria do conjunto dos nmeros complexos, o valor da expresso
6 6
i) (1 ) i 1 ( + :
a)0 b)16 c)-16 d)16i e)-16i

Questo 05) Considere a funo polinomial 1 bx ax x x ) x ( P
4 12 20
+ + + + = . Sabendo-se que a e b so nmeros reais
ambos no nulos e que i 3 4 ) i 1 ( P + = + , ento |
.
|

\
|
+i 1
2
P igual a:
a)
i 1
2
+
. b)
i 1
2

. c)
i 2
1
+
. d)4 + 3i. e)4 3i.




Caderno de Atividades

Disciplina:
M Ma at te em m t ti ic ca a

Professor(a):
Herick
Aluno:
3 ano
Ensino Mdio
Data de Recebimento:
_____/_____/_____
Lista 01
Data Entrega:
_____/_____/_____

93
Questo 06) O valor da expresso
123
i
i 1
i 8 6
) i 2 4 )( i 3 2 ( +

+
+ + igual a:
a)13 14i
b)14 + 13i
c)13 + 14i
d)14 13i
e)i
Questo 07) Se i a unidade imaginria, ento
16 15
14 13
i i
i i

+
igual a:
a)i
b) i
c)0
d)1
e) 1

Questo 08) Seja z um nmero complexo dado por
2
4
) i 3 3 (
) i 1 ( ) i 4 3 (
z

+ +
= . Considerando as aproximaes log 2 = 0,30
e log 3 = 0,48, o valor de log |z|
a)0,02
b)0,04
c)0,06
d)0,4
e)0,6

Questo 09) Os quatro vrtices de um quadrado no plano Argand-Gauss so nmeros complexos, sendo trs deles
i 2 1+ , i 2 + e i 2 1 . O quarto vrtice do quadrado o nmero complexo
a)2+i
b)2 i
c)1 2i
d)1 + 2i
e)2 i
Questo 10) O valor do nmero complexo
20
27
9
i 1
i 1
|
|
.
|

\
|
+
+
:
a)1
b)i
c)i
d)1
e)2
20


Questo 11) O nmero complexo z que verifica a equao 0 ) i 1 ( z 2 iz = + + :
a)z = 1 + i b) i
3
1
z =

c)
3
i 1
z

=

d)
3
i
1 z + =

e)z = 1 i

Questo 12) Seja o nmero complexo z , tal que i 5 10 z 2 z 3 + = + . Ento z . z (sabendo que z o conjugado de z )
igual a:
a)2 + 5i
b)29
c)5
d)2
e)24

Questo 13) Um nmero complexo z tal que i 3 a z + = , sendo a um nmero real. O valor de a para que um dos
argumentos de z seja 6 / t ser:
a) 3 3 .
b)27.
c)9.
d) 3 .
e)3.



94
Questo 14) A figura indica a representao dos nmeros Z
1
e Z
2
no plano complexo.



Se Z
1

.
Z
2
= a + bi, ento a + b igual a
a) ( ) 3 1 4
b) ( ) 1 3 2
c) ( ) 3 1 2 +
d) ( ) 1 3 8
e) ( ) 1 3 4 +

Questo 15) Sejam z
1
= i, z
2
= x + i, x > 0, nmeros complexos. Se 10 z
2
1
=
2
z teremos:
a)x = 3
b)x = 5
c)x = 2
d)x = 1
e)x = 4

Questo 16) Considere todos os nmeros complexos z = x + yi. O lugar geomtrico de todos os nmeros complexos
que possuem mdulo 1 dado pela equao:
a)x
2
+ y
2
= 1
b)x = 1
c)y = 1
d)x + y = 1
e)x
2
+ y
2
+ 1 = 0

Questo 17) Com relao aos nmeros complexos, assinale o que for correto.

01. (2 + 2 i)
6
um nmero imaginrio puro.
02.
i 1
i
z
103
+
= um nmero cujo mdulo
2
2
.
04. Se 3
1 z i
i 2 z
=
+
+
, ento
10
i 7 9
z
+
= .
08. O ponto, no plano complexo, correspondente ao nmero complexo
i 1
i
z
103
+
= est localizado no 4. quadrante.
16. |
.
|

\
| t
+
t
6
5
sen i
6
5
cos 8 a forma trigonomtrica do nmero complexo i 4 3 4 - z = .

Questo 18) As razes cbicas do nmero complexo i esto associadas aos pontos:

a) ) 0 , 1 ( ,
2
3
,
2
1
,
2
3
,
2
1

|
|
.
|

\
|

|
|
.
|

\
|

b) ) 0 , 1 ( ,
2
3
,
2
1
,
2
3
,
2
1
|
|
.
|

\
|

|
|
.
|

\
|

c) ) 1 , 0 ( ,
2
1
,
2
3
,
2
1
,
2
3

|
|
.
|

\
|

|
|
.
|

\
|

d) ) 1 , 0 ( ,
2
1
,
2
3
,
2
1
,
1
3
|
|
.
|

\
|

|
|
.
|

\
|





95
Questo 19) Dado o nmero complexo
6
sen i
6
cos z
t
+
t
= , ento, se P
1
, P
2
e P
3
so as respectivas imagens de z, z
2

e z
3
no plano complexo, a medida do maior ngulo interno do tringulo P
1
P
2
P
3

a)75
b)100
c)120
d)135
e)150

Questo 20) O nmero complexo bi z + o = vrtice de um tringulo eqiltero, como mostra a figura.

Sabendo que a rea desse tringulo igual a 3 36 , determine z
2
.

Questo 21) A respeito do nmero complexo z=1+i, assinale o que for correto.
01. z
10
= 32i
02. z z um nmero real ( z o conjugado de z)
04. z uma das razes cbicas de 4
08. A forma trigonomtrica de z |
.
|

\
| t
+
t
=
4
sen i
4
cos 2 z
Questo 22) Se u o argumento principal do nmero complexo
3
3 i
i 2
1
z

+
= , ento
a)
2
0
t
< u <
b) t < u <
t
2

c)
4
5t
< u < t
d)
2
3
4
5 t
< u <
t

e) t < u <
t
2
2
3

Questo 23) Geometricamente, a adio dos nmeros complexos ) 4 , 2 ( z
1
= e ) 1 , 1 ( z
2
=
a) b) c) d)


Questo 24) Sendo i a unidade imaginria, a soma dos termos da sequncia i
0
, i
1
, i
2
, i
3
, i
4
, i
5
,....,i
2007

a)1
b)0
c)1
d)i
e)i
Questo 25) O argumento do nmero complexo z
6
t
, e o seu mdulo 2.Ento, a forma algbrica de z
a)i
b)i
c) i 3
d) i 3
e) i 3 +

96











PROF.:HERICK





C
C
o
o
n
n
t
t
e
e

d
d
o
o
s
s
:
:

P
P
O
O
L
L
I
I
N
N

M
M
I
I
O
O
S
S


























D Da at ta a d da a E En nt tr re eg ga a : : _ __ __ __ __ __ __ __ _/ /_ __ __ __ __ __ __ __ __ __ __ _/ /2 20 01 13 3

LISTA 2 MATEMTICA

97

Questo 01) O grfico no plano cartesiano (x, y) da funo y = p(x) onde p(x) definida por d cx bx x p(x)
2 3
+ + + =
intercepta o eixo- x em x = 1 e em x = 2 , e o eixo- y em y = -2 . Nestas condies, podemos afirmar que o valor de b
- c :
a)- 3
b)- 9
c)1
d)9

Questo 02) Em relao a P(x), um polinmio de terceiro grau, sabe-se que P(1) = 2, P(0) = 1, P(1) = 2 e P(2) = 7.
a)Determine a equao reduzida da reta que passa pelo ponto em que o grfico da funo polinomial
P(x) cruza o eixo y, sabendo que essa reta tem coeficiente angular numericamente igual soma dos
coeficientes de P(x).
b)Determine P(x).

Questo 03) O nmero de razes reais distintas da equao 0 3x
3
x
3) - 3)(x (x x
3
= +
a)0
b)1
c)3
d)5
e)7

Questo 04) Considere o polinmio 24 2x 14x x x p(x)
2 3 4
+ + = . Sabendo-se que o produto de duas razes de p(x)
12 , o produto das outras duas razes :
a)2
b)2
c)4
d)4

Questo 05) Se os polinmios p, r e s so de graus 2, 3 e 4, respectivamente, pode-se afirmar que o grau de p + r
s:
a)no pode ser determinado. b) igual a 1. c) igual a 4.
d) igual a 9. e) igual a 2.

Questo 06) O quociente da diviso do polinmio
3 3 4 2
) 1 (x ) 1 x ( ) x ( P + + = por um polinmio de grau 2 um
polinmio de grau
a)5 b)10 c)13 d)15 e)18

Questo 07) Considerando que os polinmios
c bx ax 6x 3x ) x ( P
2 3 4
+ + + + = e
2 2
n) - 4)(x - m(x ) x ( Q = so idnticos, assinale o que for correto.
01. P(0) = 10
02. A equao P(x) = 0 admite 4 razes distintas.
04. Em sua forma fatorada,
2 2
2) - (x 1) 3(x ) x ( P + =
08. A soma das razes de P(x) = 0 2.
16. O produto das razes de P(x) = 0 4.

Questo 08) Se os polinmios
1 1 1
x nx 2
m n x
) x ( P
2
= e 4 x x 4 x ) x ( Q
2 3
+ + = so idnticos, ento o valor de
n
m

a)2
b)3
c)4
d)5

Caderno de Atividades

Disciplina:
M Ma at te em m t ti ic ca a

Professor(a):
Herick
Aluno:
3 ano
Ensino Mdio
Data de Recebimento:
_____/_____/_____
Lista 01
Data Entrega:
_____/_____/_____

98
Questo 09) Sejam os polinmios 5 x ) b a ( ) x ( p
4
+ = e c b x ) c a ( x 2 ) x ( q
2 4
+ + + + = , em que a, b, c so nmeros reais.
Suponha que p(x) e q(x) sejam iguais para todo r xe .
Ento, a+b+c vale:
a)7
b)
2
5

c)2
d)
2
7


Questo 10) A identidade
1 x
C
1 x
B
x
A
) 1 x ( x
1
2
+
+

+ =

vlida para todo x real exceto para x = 0, x = 1 e x = 1.


Nessas condies, os valores de A, B e C, nessa ordem so:
a)1,
2
1
,
2
1

b)0, 0, 1
c)1,
2
1
,
2
1

d)1,
2
1
,
2
1

e)0,
2
1
,
2
1


Questo 11) Seja:
4 - x
10 x ax
2 x
1 m x n
-
2 x -
n - x m
2
2
+
=
+
+ +
, onde m, n , a so nmeros reais.
Ento, correto afirmar que o produto m . n igual a:
a)-3 b)-2 c)2 d)4 e)6

Questo 12) Considere uma funo p(x), tal que 2 x 3 3x x) - p(2 - p(x) 2
2
= .
a)Calcule p(1).
b)Qual o valor da soma p(1) + p(3)?

Questo 13) Se
2 x
b
1 x
a
2 x 3 x
x
2

+

=
+
verdadeira para todo x real, 2 x , 1 x = = ento o valor de a
.
b
a) 4.
b) 3.
c) 2.
d)2.
e)6.
Questo 14) Os valores de A, B e C, de forma que
1 x
C
1 x
B Ax
1 x x x
5 x
2 2 3

+
+
+
=
+
+
, so, respectivamente,
a)2, 7 e 8.
b)5, 2 e 1.
c)2, 4 e 1.
d)3, 2 e 3.

Questo 15) Se P(x) um polinmio do segundo grau com coeficientes reais, tais que 1 ) 0 ( P = e
2 x 8 ) x ( P ) 1 x ( P + = , ento
a) 1 x 4 x 2 ) x ( P
2
+ + =
b) 1 x 2 x 4 ) x ( P
2
+ + =
c) 1 x 4 x 2 ) x ( P
2
+ + =
d) 1 x 2 x 4 ) x ( P
2
+ =

Questo 16) O polinmio 2 cx bx ax ) x ( P
2 3
+ + + = satisfaz as seguintes condies:

=
=
3
x ) x ( P ) x ( P
e
0 ) 1 ( P
, qualquer que seja x real. Ento:
a)P(1) = 1
b)P(1) = 0
c)P(2) = 0
d)P(2) = 8
e)P(2) = 12

99

Questo 17) O polinmio P(x) = ax
3
+ bx
2
+ cx + d de grau 3, tem como razes x = 1, x = 1 e x = 2, e seu grfico
est indicado na figura abaixo. Assinale a alternativa que apresenta os coeficientes desse polinmio.

a)a = 2, b = 4, c = 2, d = 4
b)a = 2, b = 4, c = 2, d = 4
c)a = 1, b = 2, c = 1, d = 2
d)a = 2, b = 4, c = 2, d = 4
e)a = 1, b = 2, c = 1, d = 2

Questo 18) O polinmio p(x) = x
3
+ ax
2
+ bx , em que a e b so nmeros reais, tem restos 2 e 4 quando dividido
por x 2 e x 1, respectivamente. Assim, o valor de a
a)6
b)7
c)8
d)9
e)10

Questo 19) Para um polinmio real, P(x), de grau 3, sabe-se que 6 ) 1 ( P = e 8 ) 2 ( P = . Ento o resto da diviso de
P(x) pelo polinmio quadrtico 2 3x - x Q(x)
2
+ = :
a)2
b)14
c)2x + 4
d)6x + 8
Questo 20) Dividindo o polinmio 4 - 2x 3x 5x ) x ( P
2 3
+ + = pelo polinmio D(x), obtm-se o quociente 18 5x ) x ( Q + = e
o resto 22 - 51x ) x ( R = . O valor de D (2) :
a)11.
b)3.
c)1.
d)3.
e)11.

Questo 21) Seja P(x) um polinmio de terceiro grau, cujo grfico est representado na Figura 5.


Ento o resto da diviso de P(x) pelo monmio x+2 :
a)0
b)16
c)-12
d)-16
e)12

Questo 22) Dados os polinmios 3 px x 2 x ) x ( P
2 3
+ = e 1 x 2 x ) x ( R
2
+ + = , assinale o que for correto.
01. Na diviso de P(x) por (x 2) e por (x + 2), os restos so iguais. Ento, p = 4.
02. Se p = 7, o quociente entre P(x) e R(x) 4 x ) x ( Q = e o resto igual a 1.
04. Se p = 3, ento P(x) divisvel por (x 1).
08. Se 2 x 5 x x ) x ( R ) x ( P
2 3
= = + , ento p um nmero mpar.
16. Se 4 x 3 X ) x ( R ) x ( P
2 3
= , ento p = 2.

100
Questo 23) O polinmio 6 x 11 x 6 x ) x ( p
2 3
+ = divisvel por:
a)(x 3)(x + 1)
b)(x 1)(x + 1)
c)(x + 1)(x 2)
d)(x 2)(x + 2)
e)(x 1)(x 2)

Questo 24) O quociente da diviso do polinmio 18 x x 3 x
2 3
+ por x2 :
a) 9 x 5 x
2
+ +
b) 5 x 9 x
2
+
c) 9 x 5 x
2
+
d) 5 x 9 x
2


Questo 25) Seja x um nmero real positivo. O volume de um paraleleppedo reto-retngulo dado, em funo de
x, pelo polinmio 8 14x 7x x
2 3
+ + + . Se uma aresta do paraleleppedo mede 1 x + , a rea da face perpendicular a essa
aresta pode ser expressa por:
a)x
2
6x + 8.
b)x
2
+ 14x + 8.
c)x
2
+ 7x + 8.
d)x
2
7x + 8.
e)x
2
+ 6x + 8.

101






PROF.:HERICK







C
C
o
o
n
n
t
t
e
e

d
d
o
o
s
s
:
:

E
E
Q
Q
U
U
A
A

E
E
S
S
A
A
L
L
G
G

B
B
R
R
I
I
C
C
A
A
S
S

























D Da at ta a d da a E En nt tr re eg ga a : : _ __ __ __ __ __ __ __ _/ /_ __ __ __ __ __ __ __ __ __ __ _/ /2 20 01 13 3

LISTA 3 MATEMTICA

102

Questo 01) Sabe-se que o produto de duas das razes do polinmio P(x) = 2x
3
x
2
+ kx + 4 igual a 1. O valor do
coeficiente k :
a)12
b)10
c)8
d)4
e)2

Questo 02) Sabendo-se que os nmeros reais no nulos, a e a, so solues da equao 0 1 px x 2 x 3
2 3
= + + ,
ento, pode-se afirmar que:
a) 1 p >
b) 1 p 0 < s
c) 0 p 1 < s
d) 1 - p <

Questo 03) Se as razes do polinmio m x
108
13
x
18
13
x ) x ( p
2 3
+ + + = esto em uma progresso geomtrica de razo
3
1
, o valor de m
a)
243
13

b)
204
1

c)
216
13

d)
243
1

e)
216
1


Questo 04) Considere a equao 0 10 mx x 6 x
2 3
= + + de incgnita x e sendo m um coeficiente real. Sabendo que
as razes da equao formam uma progresso aritmtica, o valor de m :
a)-5
b)-3
c)3
d)4
e)5

Questo 05) Em relao s razes da equao x
3
- 4x
2
+ 3x = 0, podemos afirmar, corretamente, que
a)uma delas um nmero negativo. b)uma delas um nmero irracional.
c)uma delas um nmero primo. d)o produto de todas elas igual a 3.

Questo 06) Dado um polinmio 16 x 20 x 2 2x ) x ( P
2 3
+ + = e sabendo que uma das razes x = 1, ento a mdia
aritmtica das demais razes
a)-3/2.
b)3/2.
c)-2.
d)-1.
e)1/2.

Questo 07) Uma das razes da equao polinomial 0 6 - 11x 6x - x
2 3
= + 2. O produto das outras duas razes
igual a
a) 3.
b) 2.
c)2.
d)3.
e)4.

Caderno de Atividades

Disciplina:
M Ma at te em m t ti ic ca a

Professor(a):
Herick
Aluno:
3 ano
Ensino Mdio
Data de Recebimento:
_____/_____/_____
Lista 03
Data Entrega:
_____/_____/_____

103
Questo 08) A equao 0 1 x 4 x 3
3
= apresenta 3 razes 1 - e x , x
2 1
. O valor de
2
2
2
1
x x +
a)5/3
b)8/3
c)2/5
d)1/7
e)2/3

Questo 09) A equao algbrica 0 12 x 19 x 8 x
2 3
= + tem razes a, b e c tais que c b a e c b a > > + = .
Assim, assinale o que for correto.
01. Todas as razes so nmeros naturais.
02. Uma das razes tem multiplicidade 2.
04. 26 c b a
2 2 2
= + +
08.
12
19
c
1
b
1
a
1
= +

Questo 10) Sejam p, q, r as razes distintas da equao x
3
2x
2
+ x 2 = 0. A soma dos quadrados dessas razes
igual a
a)1.
b)2.
c)4.
d)8.
e)9.

Questo 11) Uma das razes da equao 0 30 x 31 x 10 x
2 3
= + igual a 2. O produto entre as outras duas razes :
a)15
b)31
c)43
d)52
e)67

Questo 12) Sabendo que x = 1 raiz da funo 6 x 5 x 2 x ) x ( f
2 3
+ = , podemos afirmar que as outras duas razes
de f(x) tm:
a)soma igual a -1
b)soma igual a -6
c)produto igual a -6
d)produto igual a -1
e)produto igual a 1

Questo 13) As razes da equao 0 27 x 39 x 13 x
2 3
= + so reais e esto em progresso geomtrica de razo
q=3. A soma das duas razes maiores
a)18.
b)13.
c)12.
d)10.
e)9.

Questo 14) O nmero de solues da equao
3 x
x
x 5
x
2 2
+
=

,
a)0
b)1
c)2
d)3

Questo 15) Se a, b e c so as razes da equao x
3
2x
2
+ 3x 4 = 0, ento
c
1
b
1
a
1
+ + vale
a)
3
2

b)
3
4

c)
3
7

d)
4
3

e)
4
1


104
Questo 16)Os nmeros reais a, b, c e d so tais que, para todo x real, tem-se
ax
3
+ bx
2
+ cx + d = (x
2
+ x 2)(x 4) (x + 1)(x
2
5x + 3).
Desse modo, o valor de b + d :
a)2
b)0
c)4
d)6
e)10

Questo 17) As dimenses, em metros, de um paraleleppedo retngulo so dadas pelas razes do polinmio
x
3
14x
2
+56x 64. Determine, em metros cbicos, o volume desse paraleleppedo.

Questo 18) Em relao equao x
3
7x
2
+ 14x 8 = 0, assinale o que for correto.
01. A soma de suas razes 7.
02. Uma das razes nula.
04. As suas razes constituem uma progresso geomtrica.
08. O produto de suas razes um nmero mpar.
16. Uma das razes imaginria.



Questo 19) Considere o polinmio 1 bx ax ) x ( p
2
+ + = , onde a e b so nmeros reais. Calcule os valores de a e b,
sabendo que o nmero complexo 2 + i uma raiz de p(x).

Questo 20) Sabendo-se que (1 + i) raiz do polinmio 2 4x - x 3x 3x - x ) x ( P
2 3 4 5
+ + + = , pode-se afirmar que
a)1 raiz de multiplicidade 1 de P(x).
b)1 raiz de multiplicidade 2 de P(x).
c)1 raiz de multiplicidade 2 de P(x).
d)(1 + i) raiz de multiplicidade 2 de P(x).
e)(1 i) no raiz de P(x).

Questo 21) Se (5 + 3i) uma raiz complexa da equao 0 n mx 3x
2
= + + , sendo m e n nmeros reais, ento m
igual a
a)10.
b)10.
c) 30.
d)30.

Questo 22) Sabendo-se que o nmero complexo (1 i) raiz do polinmio P(x) dado por b 8x ax x ) x ( P
2 3
+ + + = ,
ento a soma do(s) valor(es) da(s) raiz(es) real(is) do referido polinmio :

Questo 23) Uma das razes da equao polinomial 0 10 13x 6x x
2 3
= + 2. As outras razes dessa equao so:
a)1 e 3.
b)(2 + i) e (2 i).
c)(2 + 2i) e (2 2i).
d)(1 + i) e (1 i).
e)(2i) e (2i).

Questo 24) A equao polinomial que possui 3 como raiz dupla e 4 e 2 como razes simples :
a)x
4
- 19x
2
- 6x + 36 = 0 .
b)x
4
- 12x
3
+ 53x
2
-102x + 72 = 0 .
c)x
4
- 6x
2
- 8x + 24 = 0 .
d)x
4
- 19x
2
- 6x + 72 = 0 .
e)x
4
+ 12x
3
- 8x - 96 = 0 .

Questo 25) O polinmio tem x 7x + 16x 12 tem
a)uma raiz real com multiplicidade 3.
b)uma raiz real com multiplicidade 2.
c)razes reais e distintas
d)uma raiz complexa
e)duas razes complexas

105






PROF.:HERICK





C
C
o
o
n
n
t
t
e
e

d
d
o
o
s
s
:
:

M
M
A
A
T
T
R
R
I
I
Z
Z
E
E
S
S
E
E
D
D
E
E
T
T
E
E
R
R
M
M
I
I
N
N
A
A
N
N
T
T
E
E
S
S



























D Da at ta a d da a E En nt tr re eg ga a : : _ __ __ __ __ __ __ __ _/ /_ __ __ __ __ __ __ __ __ __ __ _/ /2 20 01 13 3

LISTA 4 MATEMTICA

106

Questo 01) Se
|
|
.
|

\
|
=
1 0
2 1
A ,
|
|
.
|

\
|
=
1
2
B e
|
|
.
|

\
|
=
y
x
X , determine X, tal que A . X = B.

Questo 02) Dadas as matrizes
|
|
.
|

\
|


=
2 1 0
1 2 1
A e
|
|
.
|

\
|
=
0 1
1 2
B e sendo A
t
a matriz transposta de A, calcular A
t
. B.

Questo 03) Multiplicando
|
|
.
|

\
|
2
1
b
a
.
|
|
.
|

\
|
0 1
3 2
obtemos
|
|
.
|

\
|
0 2
3 4
. O produto dos elementos a e b da primeira matriz :
a)-2
b)1
c)0
d)1
e)6

Questo 04) Se A uma matriz 3 x 4 e B uma matriz n x m, ento:
a)Existe A + B se, e somente se, n = 4 e m = 3;
b)Existe AB se, e somente se, n = 4 e m = 3;
c)Existem AB e BA se, e somente se, n = 4 e m = 3;
d)Existem, iguais, A + B e B + A se, e somente se, A = B;
e)Existem, iguais, AB e BA se, e somente se, A = B.

Questo 05) Determine o valor de x para que o determinante da matriz C = A x B
t
seja igual a 602, onde:
(


=
2 1 4
3 2 1
A ,
(


=
4 7 2
5 8 1 x
B e B
t
a matriz transposta de B.

Questo 06) Seja
(

=
1 0
1 1
A .
a)Determine A
3
= A.A.A
b)Se A
n
denota o produto de A por A n vezes, determine o valor do nmero natural k tal que:
A
k
2
A
5k
+ A
6
= I, onde I a matriz identidade.

Questo 07) Uma metalrgica produz parafusos para mveis de madeira em trs tipos, denominados soft,
escareado e sextavado, que so vendidos em caixas grandes, com 2000 parafusos e pequenas, com 900, cada
caixa contendo parafusos dos trs tipos. A tabela 1, a seguir, fornece a quantidade de parafusos de cada tipo contida
em cada caixa, grande ou pequena. A tabela 2 fornece a quantidade de caixas de cada tipo produzida em cada ms
do primeiro trimestre de um ano.
Tabela 1
700 300 Sextavado
800 400 Escareado
500 200 Soft
Grande Pequena caixa Parafusos/

Tabela 2
1800 1500 1200 Grande
1300 2200 1500 Pequena
MAR FEV JAN Caixas/ms

Associando as matrizes

(
(
(

=
700 300
800 400
500 200
A e
(

=
1800 1500 1200
1300 2200 1500
B
s tabelas 1 e 2, respectivamente, o produto A x B fornece
Caderno de Atividades

Disciplina:
M Ma at te em m t ti ic ca a

Professor(a):
Herick
Aluno:
3 ano
Ensino Mdio
Data de Recebimento:
_____/_____/_____
Lista 04
Data Entrega:
_____/_____/_____

107
a)o nmero de caixas fabricadas no trimestre.
b)a produo do trimestre de um tipo de parafuso, em cada coluna.
c)a produo mensal de cada tipo de parafuso.
d)a produo total de parafusos por caixa.
e)a produo mdia de parafusos por caixa.

Questo 08) Seja dada a matriz
A =
(
(
(

x 16 6 0
6 x 2
0 2 x
,
em que x um nmero real.

a)Determine para quais valores de x o determinante de A positivo.
b)Tomando
C =
(
(
(

1
4
3
,
e supondo que, na matriz A, x = 2, calcule B = AC.

Questo 09) A matriz
(
(
(

c
b
a
a soluo da equao matricial AX = M em que:
(
(
(

=
3 0 0
4 1 0
5 2 1
A e
(
(
(

=
9
15
28
M . Ento a
2
+ b
2
+ c
2
vale:
a)67
b)68
c)69
d)70
e)71

Questo 10) Um agente secreto comunica-se com a sua base por meio de mensagens codificadas. Para isto,
associado para cada letra do alfabeto um nmero, da seguinte maneira: 1-A, 2-B, 3-C, , 26-Z. Dessa forma, para a
base enviar a mensagem HOJE para o agente secreto, ela utiliza a associao de letras e nmeros acima,
transformando a palavra HOJE na sequncia 8 15 10 5 e, em seguida, construda a matriz
(

=
5 10
15 8
A . Utilizando a
matriz codificadora
(

=
1 3
2 1
C , multiplica-se a matriz A pela C, obtendo a matriz
(

=
25 25
31 53
M . Assim, a base envia a
mensagem para o agente secreto atravs da sequncia 53 31 25 25.
O agente secreto acaba de receber a seguinte mensagem enviada pela base: 69 33 13 21. Decifre-a e responda:
qual a palavra que o agente secreto recebeu?

Questo 11) Seja A, B e C matrizes quadradas quaisquer de ordem n. Ento, correto afirmar que:
a)se AB = AC, ento B = C
b)AB = BA
c)se A
2
= 0
n
(matriz nula), ento A = 0
n

d)(AB) C = A (BC)
e)(A + B)
2
= A
2
+ 2AB + B
2


Questo 12) Considere as matrizes
(
(
(

=
p c 4
n b 4
m a 4
A e
(
(
(

=
3 c p
3 b n
3 a m
B .
Se o determinante da matriz A igual a 2, ento o determinante da matriz B igual a:
a)
2
3

b)
3
2

c) 3
d)
2
3

e)
3
2


108
Questo 13) Considere trs lojas, L
1
, L
2
e L
3
, e trs tipos de produtos, P
1
, P
2
e P
3
. A matriz a seguir descreve a
quantidade de cada produto vendido por cada loja na primeira semana de dezembro. Cada elemento aij da matriz
indica a quantidade do produto Pi vendido pela loja L
j
, i, j = 1, 2, 3.
(
(
(

11 16 12
8 10 15
20 19 30
P
P
P
L L L
3
2
1
3 2 1

Analisando a matriz, podemos afirmar que
a)a quantidade de produtos do tipo P
2
vendidos pela loja L
2
11.
b)a quantidade de produtos do tipo P1 vendidos pela loja L
3
30.
c)a soma das quantidades de produtos do tipo P
3
vendidos pelas trs lojas 40.
d)a soma das quantidades de produtos do tipo Pi vendidos pelas lojas Li, i = 1, 2, 3, 52.
e)a soma das quantidades dos produtos dos tipos P
1
e P
2
vendidos pela loja L
1
45.

Questo 14) Sabendo-se que a matriz
(
(
(
(


0 21 1
x 3 y 49
y 2 x 5

2
igual sua transposta, o valor de x + 2y :
a)-20
b)1
c)1
d)13
e)20

Questo 15) Assinale a alternativa verdadeira
a)
(
(
(

1 1 1
0 1 1
0 0 1
uma matriz diagonal.
b)
(

1 0
1 1
uma matriz triangular.
c)
(

0 1
1 0
uma matriz identidade
d)
(

1 1
0 1
uma matriz simtrica
e)
(

1 1
0 0
uma matriz transposta da matriz
(

0 1
0 1

Questo 16) O determinante da matriz P = M x M, em que
|
|
|
.
|

\
|
=
i 0 i
i i 0
0 i i
M , igual a
Observe que i o nmero complexo cujo quadrado igual a 1.
a)2i.
b)4i.
c)2.
d)4.
Questo 17) Analise as afirmaes abaixo, sabendo que: 2
i h g
f e d
c b a
=
I. 2
i h g
c b a
f e d
=

II. 6
i 3 h 3 g 3
f 3 e 3 d 3
c 3 b 3 a 3
=

III. 0
i h g
0 0 0
c b a
=
IV. 2
i h g
c 2 f b 2 e a 2 d
c b a
= + + +





Assinale a alternativa correta.
a)Apenas I, III e IV so verdadeiras.
b)Apenas a afirmao III verdadeira.

109
c)Apenas I e II so verdadeiras.
d)Todas as afirmaes so verdadeiras.


Questo 18) Sobre as matrizes abaixo, de determinantes no nulos, para quaisquer valores de a, b e c, assinale o
que for correto.
A =
(
(
(

1 4 c
5 2 b
3 1 a
B =
(
(
(

1 5 3
8 4 2
c b a

01. det A =
2
1
det B.
02. det(2A) = 4 det B.
04. det A
t
= det B.
08. det |
.
|

\
|
2
B
= det A.
16. det A = 2 det B
t
.

Questo 19) Considere as matrizes quadradas de ordem 2:
|
|
.
|

\
|
=
1 2
0 1
A e
|
|
.
|

\
|
=
2 0
1 2
B .
Seja M = AB
t
, onde B
t
a matriz transposta de B. O determinante da matriz inversa de M :
a)1/8
b)1/6
c)1/4
d)1/2

Questo 20) Sendo
|
|
.
|

\
|

=
10 2
n m
A uma matriz inversvel com inversa A
1
, suponha que
6
1
A det
1
=

, podemos
afirmar que:
a)5m + n = 3
b)5m n = 3
c)5m + n = 3
d)m + n = 1
e)n 5m = 3

Questo 21) Se A uma matriz quadrada 2 2 de determinante 10. Se B = -2 A e C = 3 B
-1
, onde B
-1
a matriz
inversa de B, ento o determinante de C
a)60
b)
20
3

c)
3
20

d)
40
9

e)
9
40


Questo 22) Dada a matriz
(

=
1 - 1
2 1
A , seja a matriz B tal que D BA A
1
=

onde
(

=
2 1 -
1 2
D , ento o determinante
de B igual a:
a)3 b)-5 c)2 d)5 e)-3

Questo 23) Seja a matriz
(
(

+
=
5 x 12 c
c b a
5 a 1
A em que a, b, c so constantes reais positivas e x uma varivel real.
Considerando que, ordenadamente, as seqncias de termos das duas primeiras linhas de A constituem
progresses aritmticas, enquanto que as seqncias de termos das duas primeiras colunas constituem progresses
geomtricas, ento, se 18 A det = , o valor de log
8
x
a)3
b)
2
3

c)1
d)
3
2

e)
3
1



110
Questo 24) Seja r a reta de equao 0
3 0 2
y 1 2
k 3 x
=

, em que k uma constante real positiva. Sabe-se que: O a


origem do sistema de eixos ortogonais; r intercepta os eixos desse sistema nos pontos P e Q; a rea do tringulo
POQ, em unidades de superfcie, igual a 16. Nessas condies, a constante k um nmero
a)maior do que 5.
b)quadrado perfeito.
c)primo.
d)divisvel por 4.
e)mltiplo de 6.

Questo 25) Seja A uma matriz. Se
(
(
(

=
34 14 0
14 6 0
0 0 1
A
3
, o determinante de A :
a)8
b) 2 2
c)2
d)
3
2
e)1

111






PROF.:HERICK






C
C
o
o
n
n
t
t
e
e

d
d
o
o
s
s
:
:

S
S
I
I
S
S
T
T
E
E
M
M
A
A
S
S
L
L
I
I
N
N
E
E
A
A
R
R
E
E
S
S
E
E
B
B
I
I
N
N

M
M
I
I
O
O
D
D
E
E

N
N
E
E
W
W
T
T
O
O
N
N

























D Da at ta a d da a E En nt tr re eg ga a : : _ __ __ __ __ __ __ __ _/ /_ __ __ __ __ __ __ __ __ __ __ _/ /2 20 01 13 3
LISTA 5 MATEMTICA

112

Questo 01) Resolva o sistema de equaes:

= +
= +
= + +
14 3z y 5x
16 z 7y x
26 2z 5y 3x


Questo 02) O par (a ; b) soluo so sistema

= +
=
22 y 2x
14 y 2 x
ento
a)a
b
< 0
b)a + b = 16
c)ab < 0
d)a + b < 0
e)a/b inteiro

Questo 03) Determine a e b de modo que sejam equivalentes os sistemas:

= +
=
2 y x
0 y x
e

=
= +
1 ay bx
1 by ax


Questo 04) So dados trs nmeros naturais a, b e c, com a < b < c. Sabese que o maior deles a soma dos
outros dois e o menor um quarto do maior. Se a b + c = 30 ento o valor de a + b + c ser:
a)45
b)60
c)900
d)120
e)150
Questo 05) Encontre o valor de a para que o sistema:

= + +
= +
= +
13 3z 4y 7x
3 z 2y x
a 3z y 2x
seja possvel. Para o valor encontrado de a
ache a soluo geral do sistema, isto , ache expresses que representem todas as solues do sistema. Explicite
duas dessas solues.

Questo 06) Dado o sistema

= +
= +
= + +
1 z y x
3 z y x
3 z y x
, ento x
2
+y
2
+z
2
vale:
a)2 b)3 c)4 d)5 e)6

Questo 07) A professora Ana Maria precisa comprar 80 unidades de material para a sua aula, entre eles, lpis,
canetas e cadernos. Espera comprar os lpis a R$ 1,00 cada, as canetas a R$ 2,00 e os cadernos a R$ 4,00.
Arrecadou dos alunos R$ 230,00 para esta compra. Se o nmero de cadernos deve ser igual ao nmero de lpis e
canetas juntos, a soluo para esta compra ser:
a)40 lpis, 35 canetas e 5 cadernos
b)5 lpis, 35 canetas e 40 cadernos
c)10 lpis, 30 canetas e 40 cadernos
d)20 lpis, 20 canetas e 40 cadernos
e)15 lpis, 25 canetas e 40 cadernos

Questo 08) Uma bolsa contm 20 moedas, distribudas entre as de 5, 10 e 25 centavos, totalizando R$ 3,25.
Sabendo que a quantidade de moedas de 5 centavos a mesma das moedas de 10 centavos, quantas moedas de
25 centavos h nessa bolsa?
a)6.
b)8.
c)9.
d)10.
e)12.
Caderno de Atividades

Disciplina:
M Ma at te em m t ti ic ca a

Professor(a):
Herick
Aluno:
3 ano
Ensino Mdio
Data de Recebimento:
_____/_____/_____
Lista 05
Data Entrega:
_____/_____/_____

113
Questo 09) Dois amigos foram fazer um passeio em um shopping na cidade de Palmas-TO. Em determinado
momento do passeio, os amigos foram praa de alimentao comprar um lanche. Um dos amigos comprou 5
bolinhos e 2 sorvetes, gastando um total de R$ 13,75. O outro amigo comprou 7 bolinhos e 1 sorvete, gastando no
total R$ 14,75. Sabendo-se que os valores unitrios dos bolinhos so os mesmos e os valores unitrios dos sorvetes
tambm so os mesmos. Ento, o preo unitrio do bolinho e do sorvete so, respectivamente:
a)R$ 1,60 e R$ 2,00
b)R$ 1,65 e R$ 2,50
c)R$ 1,75 e R$ 2,50
d)R$ 2,40 e R$ 1,65
e)R$ 2,50 e R$ 1,75

Questo 10) Um feirante vendeu todo o seu estoque de mas e peras por R$ 350,00. O preo de venda das peras
e das mas est descrito na tabela abaixo:

Se o feirante tivesse vendido somente metade das mas e
5
2
das peras, ele teria arrecadado R$ 160,00. Sendo
assim, quantas frutas o feirante vendeu?
a)200 b)300 c)400 d)500

Questo 11) Uma caixa contm 48 balas, sendo algumas de caramelo e as demais de hortel. Se a tera parte do
dobro do nmero de balas de hortel excede metade do de caramelo em 4 unidades, ento nessa caixa h:
a)igual nmero de balas de caramelo e de hortel.
b)20 balas de caramelo.
c)28 balas de hortel.
d)duas balas de caramelo a mais que de hortel.
e)duas balas de hortel a mais que de caramelo.

Questo 12) Uma pessoa necessita de 5 mg de vitamina E por semana, a serem obtidos com a ingesto de dois
complementos alimentares o e |. Cada pacote desses complementos fornece, respectivamente, 1 mg e 0,25 mg de
vitamina E. Essa pessoa dispe de exatamente R$ 47,00 semanais para gastar com os complementos, sendo que
cada pacote de o custa R$ 5,00 e de | R$ 4,00.
O nmero mnimo de pacotes do complemento alimentar o que essa pessoa deve ingerir semanalmente, para
garantir os 5 mg de vitamina E ao custo fixado para o mesmo perodo, de:
a)3 b)
16
5
3 c)5,5 d)
4
3
6 e)8

Questo 13) A figura includa nesta questo representa quatro balanas.
As duas primeiras balanas esto em equilbrio. Temos pesos de 1, 2, 5, 10 e 20 gramas. Nos pratos da esquerda,
os pesos tm a forma de cubos e cones, em que cada cubo pesa x gramas e cada cone, y gramas.
a)Qual o menor nmero de pesos que devemos colocar no prato da direita da 3 balana para que ela
fique em equilbrio?
b)Queremos colocar no prato da direita da 4 balana somente pesos de 2g e 5g. Quantos pesos
devemos colocar, de modo que ela fique em equilbrio? Descreva todos os modos possveis.

Questo 14) Os nmeros reais a e b tornam o sistema a seguir indeterminado em R, ento:

= +
= +
= +
b 2z 3y 3x
12 4z ay 2x
2 z - 2y 2x

a)a + b = 4
b)a + b = -4
c)a + b = 18
d)a + b = -18
e)a + b = 0



114
Questo 15) Procurando-se determinar a e b de modo que o sistema abaixo seja indeterminado, verifica-se que:

= +
= +
0 2 by ax
0 1 2y 5x

a)a
2
+ b
2
= 116 b) impossvel c)a e b variam com x e y
d)a
2
b
2
= 8,6 e)a
2
b
2
= 0

Questo 16) O sistema

=
= +
= +
25 7z 2y 9x
18 4z 5y 4x
13 11z 3y 5x

a)s apresenta soluo trivial
b) possvel e determinado no tendo soluo trivial
c) possvel e indeterminado
d)impossvel
e)n.d.a

Questo 17) Escolha entre as alternativas abaixo, aquela que representa o valor da constante m, de modo que o
sistema a seguir admita soluo:

= + +
= + +
= + +
m 11t 4z - 7y x
2 4t z 2y x
1 t z y - 2x

a)m = 3 b)m = 4 c)m =2/3 d)m = 5 e)m = 6

Questo 18) O sistema

= +
= +
= +
2 mz y x
0 2z 2y 3x
m z y 2x

impossvel para:
a) m = 1
b) m = 0
c) m = -3
d) 1 < m < 1
e) m > 10

Questo 19) Resolver a equao
|
|
.
|

\
| +
=
|
|
.
|

\
|
4
1 n
2
1 n


Questo 20) Calcular p, p > 3, sendo dado:
3
5
1
1 p
2
p
3
1 p
2
1 p
=
|
|
.
|

\
|

|
|
.
|

\
|
|
|
.
|

\
|
+
|
|
.
|

\
|


Questo 21) O valor de m que satisfaz a sentena 512
k
m
m
0 k
=
|
|
.
|

\
|

=
:
a)5
b)6
c)7
d)8
e)9

Questo 22) Se (n + 1)! = 10 n!, ento o valor de
|
|
.
|

\
| +
2
1 n
igual a
a)9
b)45
c)55
d)90
e)110


115
Questo 23) Para que o coeficiente do termo mdio do desenvolvimento do binmio
6
2
x
k
2
x
|
|
.
|

\
|
+ segundo as
potncias crescentes de x seja igual a 160, o valor da constante k deve ser:
a)1
b)2
c)4
d)5
e)8


Questo 24) No desenvolvimento do binmio
5
) by ax ( + , os coeficientes dos monmios
4 3 2
xy e y x so,
respectivamente, iguais a 720 e 240. A respeito do desenvolvimento desse binmio segundo potncias decrescentes
de x, sendo a e b nmeros reais, assinale o que for correto.
01. a + b = 5
02. a um nmero mpar.
04. O ltimo termo do desenvolvimento 32y
5

08. O segundo termo do desenvolvimento 810x
4
y
16. O primeiro termo do desenvolvimento 243x
5


TEXTO: 1 - Comum questo: 25

Tales, um aluno do Curso de Matemtica, depois de terminar o semestre com xito, resolveu viajar para a Europa.
Questo 25)
Em Amsterdam, uma das principais atraes tursticas a visita a museus. Tales visitou o Museu Van Gogh, o
Museu Rijks e a Casa de Anne Frank. A tabela a seguir indica o valor do ingresso para estudante, adulto e snior,
em euros () :
7,65 8,50 7,65 Frank Anne de Casa
11,25 12,50 10,00 Rijks Museu
12,60 14,00 11,20 Gogh Van Museu
Snior Adulto Estudante

Num determinado momento de um dia, com a venda de x ingressos para estudantes, y ingressos para adultos e z
ingressos para sniores, o Museu Van Gogh arrecadou 1162,00 , o Museu Rijks 1037,50 e a Casa de Anne
Frank 722,50 .
Para determinar a quantidade de ingressos vendidos, resolve-se o sistema
a)

= + +
= + +
= + +
722,50 7,65z 8,50y 7,65x
1037,50 11,25z 12,50y 10,00x
1162,00 12,60z 14,00y 11,20x

b)

= + +
= + +
= + +
2380,00 7,65z 8,50y 7,65x
3375,00 11,25z 12,50y 10,00x
3780,00 12,60z 14,00y 11,20x

c)

= + +
= + +
= + +
722,50 7,65z 11,25y 12,60x
1037,00 8,50z 12,50y 14,00x
1162,00 7,65z 10,00y 11,20x

d)

= + +
= + +
= + +
72250 7,65z 8,50y 7,65x
103750 11,25z 12,50y 10,00x
116200 12,60z 14,00y 11,20x

e)

= + +
= + +
= + +
72250 7,65z 11,25y 12,60x
103700 8,50z 12,50y 14,00x
116200 7,65z 10,00y 11,20x


116






PROF.:HERICK







C
C
o
o
n
n
t
t
e
e

d
d
o
o
s
s
:
:

A
A
N
N

L
L
I
I
S
S
E
E
C
C
O
O
M
M
B
B
I
I
N
N
A
A
T
T

R
R
I
I
A
A
E
E

P
P
R
R
O
O
B
B
A
A
B
B
I
I
L
L
I
I
D
D
A
A
D
D
E
E






















D Da at ta a d da a E En nt tr re eg ga a : : _ __ __ __ __ __ __ __ _/ /_ __ __ __ __ __ __ __ __ __ __ _/ /2 20 01 13 3

LISTA 6 MATEMTICA

117

Questo 01) Usando-se apenas as letras A, B, C e D e os algarismos do sistema decimal de numerao, o nmero
de placas de automveis usadas no Brasil (exemplo: BBA 0557) possveis de serem formadas no mximo igual a
a)120000
b)240000
c)360000
d)480000
e)640000

Questo 02) Usando as letras do conjunto {a,b,c,d,e, f , g,h,i, j}, quantas senhas de 4 letras podem ser formadas de
modo que duas letras adjacentes, isto , vizinhas, sejam necessariamente diferentes?
a)7 290 b)5 040 c)10 000 d)6 840 e)11 220

Questo 03) Para ter acesso a determinado site na Internet cada pessoa deve cadastrar uma senha numrica de
quatro dgitos e, por questo de segurana, recomenda-se que ela seja periodicamente modificada.
Para alterar sua senha atual, um indivduo decide manter os mesmos algarismos das unidades e dos milhares que
so respectivamente iguais a 5 e 9 , e continuar essa senha como um nmero divisvel por 3.
Assim, o nmero total de escolhas possveis para a nova senha igual a
01.
26
02.
27
03.
32
04.
33
05.
38

Questo 04) a)Quantos so os nmeros inteiros positivos de quatro algarismos, escolhidos sem repetio, entre
1,3,5,6,8,9?
b)Dentre os nmeros inteiros positivos de quatro algarismos citados no item a), quantos so divisveis
por 5?
c)Dentre os nmeros inteiros positivos de quatro algarismos citados no item a), quantos so divisveis
por 4?

Questo 05) A figura a seguir representa uma abelha, no ponto P, entrando nos alvolos hexagonais de sua
colmeia.


Considere que a abelha caminhe sobre as linhas tracejadas, somente no sentido indicado pelas setas, e nunca
passe mais de uma vez pelo mesmo ponto. Nestas condies, a abelha tem apenas uma maneira de ir de P a A,
uma maneira de ir de P a B, duas maneiras de ir de P a C etc. Continuando esta sequncia numrica, o nmero de
maneiras de ir de P a E, de P a F e de P a G, respectivamente, :
a)3, 4 e 5 b)4, 5 e 6 c)4, 8 e 12
d)5, 8 e 13 e)5, 10 e 13

Questo 06) De quantas maneiras podemos distribuir 10 moedas, todas idnticas, entre 7 crianas, de modo que
cada criana receba pelo menos uma moeda?
a)42.
b)60.
c)84.
d)120.
Caderno de Atividades

Disciplina:
M Ma at te em m t ti ic ca a

Professor(a):
Herick
Aluno:
3 ano
Ensino Mdio
Data de Recebimento:
_____/_____/_____
Lista 06
Data Entrega:
_____/_____/_____

118
Questo 07) Trs casais vo ao cinema e observam que existem 6 poltronas livres em uma determinada fileira. De
quantas maneiras diferentes os casais podem ocupar essas poltronas, de modo que cada casal fique sempre junto?
a)24
b)12
c)16
d)6
e)32

Questo 08) O nmero de anagramas diferentes que podem ser construdos com as letras da palavra VARGAS, e
que comecem e terminem com consoantes :
a)360
b)15
c)24
d)144
e)288

Questo 09) Todas as permutaes com as letras da palavra SORTE foram ordenadas alfabeticamente, como em
um dicionrio. A ltima letra da 86. palavra dessa lista
a)S.
b)O.
c)R.
d)T.
e)E.

Questo 10) O grmio estudantil do Colgio Alvorada composto por 6 alunos e 8 alunas. Na ltima reunio do
grmio, decidiuse formar uma comisso de 3 rapazes e 5 moas para a organizao das olimpadas do colgio. De
quantos modos diferentes pode-se formar essa comisso?
a)6720.
b)100800.
c)806400.
d)1120.

Questo 11) Um juiz dispe de 10 pessoas, das quais somente 4 so advogados, para formar um nico jri com 7
jurados. O nmero de formas de compor o jri, com pelo menos um advogado
a)70
b)7
4

c)120
d)4
7

e)140

Questo 12) Um professor apresenta 10 questes, das quais os seus alunos podero escolher 8 para serem
respondidas. De quantas maneiras diferentes um aluno pode escolher as 8 questes?
a)90
b)80
c)45
d)40
e)8

Questo 13) So dados os 8 pontos A, B, C, D, E, F, G e H sobre uma circunferncia, como na figura abaixo. De
quantas maneiras podem-se formar tringulos com vrtices nesses pontos?






119
Questo 14) Um tanque de um pesque-pague contm apenas 15 peixes, sendo 40% destes carpas. Um usurio do
pesque-pague lana uma rede no tanque e pesca 10 peixes. O nmero de formas distintas possveis para que o
usurio pesque exatamente 4 carpas :
a)151200
b)720
c)210
d)185
e)1260

Questo 15) Com base nas assertivas abaixo, assinale o que for correto.
01.
Se
)! 1 n (
) 1 n ( ! n
a
2
n
+

= ento a
2000
= 1999.
02.
Se C
n,3
= 56, ento A
n,3
= 168.
04.
Trs casais podem ocupar 6 cadeiras dispostas em fila, de tal forma que as duas extremidades sejam ocupadas
por homens, de 360 maneiras diferentes.
08.
O produto dos n primeiros nmeros pares (n e N
*
) igual a 2
n
n!
16.
A soluo da equao 7
)! 1 n (
)! 2 n (
=
+
+
um nmero par.

Questo 16) Lanam-se dois dados com faces numeradas de 1 a 6. Calcule a probabilidade de que a soma obtida
seja 10.

Questo 17) O nmero da chapa de um carro par. A probabilidade de o algarismo das unidades ser zero :
a)1/10
b)1/2
c)4/9
d)5/9
e)1/5

Questo 18) Numa caixa existem 5 balas de hortel e 3 balas de mel. Retirando-se sucessivamente e sem
reposio duas dessas balas, a probabilidade de que as duas sejam de hortel :
a)1/7
b)5/8
c)5/14
d)25/26
e)25/64

Questo 19) Considere uma urna contendo 10 bolas vermelhas e 6 bolas verdes. Retirando-se simultaneamente
duas bolas da urna, qual a probabilidade de que as duas bolas selecionadas sejam vermelhas?
a)1/4
b)3/8
c)1/2
d)2/3
e)2

Questo 20) Considere o conjunto de todos os nmeros de cinco algarismos distintos, formados com os algarismos
1, 3, 5, 8 e 9.
Escolhendo, aleatoriamente, um elemento desse conjunto, calcule a probabilidade de o nmero escolhido ser menor
que o nmero 58931.

Questo 21) Os dados a seguir referem-se aos alunos matriculados nas trs turmas de um curso de Ingls.
25 22 23 meninas de Nmero
15 18 17 meninos de Nmero
C Turma B Turma A Turma

Com base nesses dados, correto afirmar:


120
01.
Em cada turma, a razo entre o nmero de meninos e o nmero de meninas menor que
4
3
.
02.
O nmero de meninos do curso igual a 40% do total de alunos matriculados.
04.
A mdia do nmero de meninas por turma menor que 23.
08.
O nmero de duplas que podem ser formadas apenas com meninas igual a 2415.
16.
Sorteando-se um estudante do curso, a probabilidade de ser uma menina da turma A igual a
120
23
.
32.
Sorteando-se um estudante do curso, a probabilidade de ser uma menina ou ser da turma A igual a
120
87
.

Questo 22) Considere o conjunto X = {neIN / 15 s n s 64}. Escolhendo-se, ao acaso, um elemento de X, a
probabilidade de ele ser um mltiplo de 3 ou de 5 :
a)48%
b)46%
c)44%
d)42%

Questo 23) Em uma populao de aves, a probabilidade de um animal estar doente 1/25. Quando uma ave est
doente, a probabilidade de ser devorada por predadores 1/4, e, quando no est doente, a probabilidade de ser
devorada por predadores 1/40. Portanto, a probabilidade de uma ave dessa populao, escolhida aleatoriamente,
ser devorada por predadores de:
a)1,0%.
b)2,4%.
c)4,0%.
d)3,4%.
e)2,5%.

Questo 24) Um baralho comum tem 26 cartas vermelhas e 26 cartas pretas.
a)Ana Lcia retira uma carta do baralho completo, a examina e a devolve ao baralho. Depois de
embaralhar novamente as cartas, ela volta a retirar uma carta. Qual a probabilidade de que, nas duas
retiradas, a cor da carta tenha sido a mesma?
b)Ana Lcia retira, simultaneamente, duas cartas de um baralho completo. Qual a probabilidade de
que as duas cartas sejam da mesma cor?

Questo 25) Sempre que joga, um time tem probabilidade
3
2
de vencer uma partida. Em quatro jogos, a
probabilidade de esse time vencer, exatamente dois deles,
a)
27
4

b)
81
16

c)
27
8

d)
81
4

e)
27
16


121






PROF.: LEONARDO





Contedos :
Funo Polinomial do 1 grau





Ministrados durante o ms de Fevereiro












Data da Entrega : ________/___________/2013
LISTA 1 MATEMTICA

122

Questo 01) Quantos nmeros inteiros pertencem ao intervalo real
(

63 , 8 ?
a) Trs.
b) Quatro.
c) Cinco.
d) Seis.
e) Sete.

Questo 02) Dados os intervalos A = [-1, 3), B = [1, 4], C = [2, 3), D = (1, 2] e E = (0, 2], consideremos o conjunto P =
[(A B) (C D)] E. Marque a alternativa incorreta:
a) P c [-1, 4]
b) (3, 4] c P
c) 2 e P
d) O e P

Questo 03) A determinao por compreenso do conjunto A = [a; b]
a) {x e N | a s x s b}
b) {x e Z | a s x s b}
c) {x e Q | a s x s b}
d) {x e R | a s x s b}
e) {x e C | a s x s b}

Questo 04) A diferena entre
3
1
e seu valor aproximado 0,333 e igual a x% do valor exato. Ento o valor de x :
a) 0,0001
b) 0,001
c) 0,01
d) 0,1
e) 0,3

Questo 05) Dados dois nmeros reais a e b que satisfazem as desigualdades 1 s a s 2 e 3 s b s 5, pode-se afirmar
que:
a)
5
2
b
a
s
b)
3
2
b
a
>
c)
3
2
b
a
5
1
s s
d)
2
1
b
a
5
1
s s
e) 5
b
a
2
3
s s

Questo 06) A frmula
4
28 p 5
N
+
= d o valor aproximado do nmero do calado (N) em funo do comprimento
(p), em centmetros,
do p de qualquer pessoa. De acordo com a frmula, o comprimento do p de quem cala 37 , em centmetros,
aproximadamente,
a) 22,5
b) 24
c) 25,5
d) 26
e) 27,5
Caderno de Atividades

Disciplina:
M Ma at te em m t ti ic ca a

Professor(a):
Leonardo
Aluno:
3 ano
Ensino Mdio
Data de Recebimento:
_____/_____/_____
Lista 01
Data Entrega:
_____/_____/_____

123
Questo 07) Os pontos (x, y) do plano tais que

+ >
>
s
2 x y
y , 4 x
x , 10 y
definem uma regio de rea:
a) 12
b) 10
c) 8
d) 14
e) 16

Questo 08) Seja S a regio limitada pelo quadrado abaixo.

y
1
1 -1 x


Ento a regio S caracterizada pelo seguinte sistema de inequaes:
a) y s x, y > -x, y > x + 2, y > -x + 2
b) y > x, y > -x, y > x + 2, y s -x + 2
c) y > x, y > -x, y s x + 2, y s -x + 2
d) y > x, y s -x, y s x + 2, y s -x + 2

Questo 09) Se, no universo R, as inequaes 3(x 1) 2(x + 2) > 2 e ( )
1 1
1 1
3 5
x k + > tm o mesmo conjunto
soluo, ento a constante k igual a
a)
3
112

b)
3
88

c)
3
8

d)
3
11

e)
3
16

Questo 10) O maior nmero inteiro que satisfaz a inequao 3
3 x
5
>

:
a) um mltiplo de 2.
b) um mltiplo de 5.
c) um nmero primo.
d) divisvel por 3.
e) divisvel por 7.

Questo 11) Uma escola paga, pelo aluguel anual do ginsio de esportes de um clube A, uma taxa fixa de R$
1.000,00 e mais R$ 50,00 por aluno. Um clube B cobraria pelo aluguel anual de um ginsio equivalente uma taxa fixa
de R$ 1.900,00, mais R$ 45,00 por aluno. Para que o clube B seja mais vantajoso economicamente para a escola, o
menor nmero N de alunos que a escola deve ter tal que:
a) 100 s N < 150
b) 75 s N < 100
c) 190 s N < 220
d) 150 s N < 190
e) 220 s N < 250





124
Questo 12) A funo, definida para todo nmero real x, cujo grfico :



tem a seguinte lei de formao:
a)

> +
< +
=
5 x , 9 x
4
5
5 x , 4 x
2
5
) x ( f
b)

> +
< +
=
5 x , 9 x
5
4
5 x , 4 x
5
2
) x ( f
c)

> +
< +
=
5 x , 9 x
5
4
5 x , 4 x
5
2
) x ( f
d)

> +
< +
=
5 x , 9 x
5
4
5 x , 4 x
5
2
) x ( f
e)

> +
< +
=
5 x , 9 x
4
5
5 x , 4 x
2
5
) x ( f

Questo 13) Na produo de n unidades de certo artigo, o custo unitrio R$ 20,00 se n s 100. No caso de n >
100 o custo de cada artigo adicional passa a ser R$ 12,00. O grfico que MELHOR retrata o custo TOTAL na
produo de n artigos

20
12
100 200 n
Custo
(R$)
(A)


3 200
2 000
100 200 n
Custo
(R$)
(B)


125
4 000
2 000
100 200 n
Custo
(R$)
(C)


(D)
3 200
2 000
100 200 n
Custo
(R$)
(D)


Questo 14) Seja a funo f definida por

<
=
>
=
0 x se 1,
0 x se 0,
0 x se 1,
) x ( f . O melhor esboo grfico da funo g(x) = (x + 1). f(x)
:
y
x 0
a.


y
x 0
b.

y
x 0
c.


y
x 0
d.







126
Questo 15) A tabela a seguir foi gerada a partir da funo linear y = ax + b.

x
y
5,2
23,0
5,3
24,0
5,4
25,0
5,5
26,0
5,6
27,0

O valor de a b :
a) 29
b) 35
c) 39
d) 41
e) 43

Questo 16) A tabela a seguir foi gerada a partir da funo linear y = ax + b.

x
y
5,2
23,0
5,3
24,0
5,4
25,0
5,5
26,0
5,6
27,0


O grfico que melhor representa essa funo :
a.
0 x
y


b.
0 x
y

c.
0 x
y


d.
0 x
y

e.
0 x
y


Questo 17) Do nmero x, subtrai-se o inverso do nmero
5 x
3
m

= , obtendo-se (-7). O valor de x :
a) 6
b) 8
c) 11
d) 13




127
Questo 18) Um aougue est fazendo a seguinte promoo na venda de alcatra: 25% de desconto sobre o preo
total da compra de 3 quilos ou mais. O esboo de grfico que melhor representa o total pago (p) em funo da
quantidade comprada (q) :
a.
p
q 3


b.
p
q 3

c.
p
q 3


d.
p
q 3


Questo 19) Num supermercado, um cartaz anuncia a seguinte promoo:


Na compra igual a ou acima de 5 kg e abaixo de 10 kg, 10% de desconto sobre o valor total. Na compra igual a ou
acima de 10 kg, 15% de desconto sobre o valor total.
Assinale a opo cujo grfico melhor representa o valor a ser pago (em reais), expresso no eixo Oy, em funo da
quantidade comprada (em quilos), expressa no eixo Ox.
.
.
-
-
-
-
0
5 10 15
20
5
10
15
20
y
x
a.

-
-
-
-
0
5 10 15
20
5
10
15
20
y
x
b.


128

-
-
-
-
0
5 10 15
20
5
10
15
20
y
x
c.


.
-
-
-
-
0
5 10 15
20
5
10
15
20
y
x
d.
.
.


Questo 20) O grfico abaixo indica o crescimento linear de uma planta. Se a relao apresentada na figura se
mantm, ento, no 30
o
(trigsimo) dia, o comprimento da planta, em cm, :

5
10
tempo (dias)
C
o
m
p
r
i
m
e
n
t
o

(
c
m
)
1
2
6
30


a) 4
b) 5
c) 150
d) 6
e) 30
Questo 21) O grfico da funo f de [0, 4[ em R definida por

< s
< s
< s
< s
4 x 3 se 4 x
3 x 2 se 3 x
2 x 1 se 2 x
1 x 0 se 1 x

1 2 3 4
1
x
y a.
0


1
1
2
3
4
y
b.
x 0


129
1 2 3 4
1
x
y c.
0
-1


1 2 3 4
1
x
y d.
0
-1



Questo 22) O imposto de renda (I.R.) a ser pago mensalmente calculado com base na tabela da Receita Federal,
da seguinte forma: sobre o rendimento-base aplica-se a alquota correspondente; do valor obtido, subtrai-se a "parcela
a deduzir"; o resultado o valor do imposto a ser pago.

Rendimento-base
Alquota
Parcelaa
Deduzir(R$) (R$)
At900,00
De900,01a1.800,00
Acimade1.800,00
Isento
15%
27,5%
----
135,00
360,00


(Tabela da Receita Federal para agosto de 1999)
Em relao ao I.R. do ms de agosto de 99, considerando apenas as informaes da tabela, correto afirmar:
01. Sobre o rendimento-base de R$ 1.000,00, o valor do imposto R$ 15,00.
02. Para rendimentos-base maiores que R$ 900,00, ao se triplicar o rendimento-base triplica-se tambm o valor do
imposto.
03. Sendo x o rendimento-base, com x > 1800, uma frmula para o clculo do imposto y : y = 0,275x 360,
considerados x e y em reais.
04. O valor do imposto em funo do rendimento-base pode ser representado, em um sistema de coordenadas
cartesianas ortogonais, pelo grfico:

IR
400
135
0 900
1800 3000
rendimentos

Questo 23) Considere um tringulo issceles de base y, lados congruentes x e permetro igual a 30 cm.
Expressando y em funo de x, obtm-se uma funo cujo grfico :
2 4 6 8 10 12 14 16
5
10
15
20
25
30
y
x
0
a.


2 4 6 8 10 12 14 16
5
10
15
20
25
30
y
x
0
b.


130
1 2 3 4 5 6
5
10
15
20
25
30
y
0
x
c.

2 4 6 8 10 12 14 16
5
10
15
20
25
30
x
y
0
d.


Questo 24) Seja g uma funo do tipo g(x) = ax + b, com x e R. Se g(-2) = - 4 e 2g(3) = 12, os valores de a e b so,
respectivamente,
a)
2
1
e 0
b) 0 e
2
1

c) 0 e 2
d)
2
1
e 0
e) 2 e 0

Questo 25) Uma loja no centro de Belm aluga microcomputadores para usurios que desejam navegar pela
Internet. Para utilizar esse servio o usurio paga uma taxa de R$ 2,00 acrescida de R$ 3,00 por hora de utilizao
da mquina. O grfico que melhor representa o preo desse servio
R$
h
a.


R$
h
b.

R$
h
c.


R$
h
d.














131







PROF.: LEONARDO







Contedos :
Funo polinomial do 2 grau e equaes
exponenciais



Ministrados durante o ms de Maro










Data da Entrega : ________/___________/2013
LISTA 2 MATEMTICA

132

Questo 01) Em R, o conjunto soluo da inequao 0
1 x 4
4 x
2
2
>

+

Questo 02) Assinale a afirmativa correta.
O polinmio x
2
ax + 1
a) tem sempre duas razes reais.
b) tem sempre uma raiz real.
c) tem exatamente uma raiz real para a = 2
d) tem exatamente uma raiz real para infinitos valores de a.
e) tem exatamente uma raiz real para a = 0.

Questo 03) Ache os valores de k para que a equao x - kx + 1 = 0, tenha exatamente uma soluo.

Questo 04) Para que a equao kx + x + 1 = 0, com k inteiro e diferente de zero, admita uma raiz inteira,
deveremos ter k igual a:
a) 4
b) 2
c) 4
d) 2
e) 8

Questo 05) Para certos valores de k o produto das razes da equao 0 4 k 3 kx 5 x
2 2
= + + + igual a 16. Para
cada um destes valores, as razes da equao so:
a) reais e iguais
b) nmeros complexos, no reais
c) nmeros irracionais, diferentes
d) ambas positivas ou ambas negativas

Questo 06) Uma partcula desliza sobre a curva 4 3x x y
2
= , a partir de um ponto P, de ordenada 14, at chegar a
um ponto Q , de ordenada 4. A diferena, em valor absoluto, entre as abscissas de P e de Q pode ser igual a:
a) 6
b) 4
c) 5
d) 7
e) 8

Questo 07) Os pontos (0, 0) e (2, 1) esto no grfico de uma funo quadrtica f. O mnimo de f assumido no
ponto de abscissa
4
1
x = . Logo, o valor de f(1) :
a)
10
1

b)
10
2

c)
10
3

d)
10
4

e)
10
5





Caderno de Atividades

Disciplina:
M Ma at te em m t ti ic ca a

Professor(a):
Leonardo
Aluno:
3 ano
Ensino Mdio
Data de Recebimento:
_____/_____/_____
Lista 02
Data Entrega:
_____/_____/_____

133
Questo 08) O grfico de uma funo do segundo grau intercepta o eixo das ordenadas em y = 4 e o das abcissas
em x = 2 e x = 3. Essa funo definida por
a) 3 x 2 x y
2
+ =
b) 4 x x 2 y
2
+ + =
c) 4 x
3
2
x
3
2
y
2
+ + =
d) 4 x
3
2
x
3
2
y
2
=
e) 4 x
3
1
x
3
2
y
2
+ + =

Questo 09) Dos nmeros abaixo, o nico que NO pertence ao conjunto imagem da funo do segundo grau
definida por 2 x 3 x y
2
+ =
a) 1
b)
4
1

c) 0
d)
6
1

e)
3
1


Questo 10) Sejam f e g funes IR em IR definidas por f(x) = x +1 e g(x) = 1 x. Relativamente ao grfico da
funo dada por g(f(x)), correto afirmar que:
a) tangencia o eixo das abscissas.
b) no intercepta o eixo das abscissas.
c) contm o ponto (-2, 0)
d) tem concavidade voltada para cima
e) intercepta o eixo das ordenadas no ponto (0, -1)

Questo 11) Na figura temos os grficos das funes f e g. Se f(x) = 2x, ento g(3) vale:
3
-1 0 x
f
g
y

a) 6
b) 8
c) 10
d) 12
e) 14

Questo 12) Seja R um retngulo que tem 24cm de permetro. Unindo-se sucessivamente os pontos mdios dos
lados de R obtem-se um losango. Qual deve ser a medida do lado desse losango para que sua rea seja mxima?
a) 3 cm
b) cm 2 3
c) 6 cm
d) cm 2 6
e) 9 cm

Questo 13) Sendo 2 e 3 as razes de 1 bx ax ) x ( f
2
+ + = , a soma das razes de 1 ax bx ) x ( g
2
+ + = vale:
a) 2 .
b) 1 .
c) 1 .
d) 2 .
e) 3 .





134
Questo 14) O grfico representa as funes f(x) = x
2
e g(x) = x + 2. A soma das coordenadas do ponto A :
A
x 0
y

a) 2
b) 4
c) 6
d) 8

Questo 15) O grfico da funo f(x) = ax
2
+ bx + c o de uma parbola que passa pelos pontos (-2,0), (2,0) e (0,4).
Os nmeros a, b e c so tais que:
a) a < c < b
b) a < b < c
c) b < a < c
d) b < c < a

Questo 16) Se o grfico abaixo representa a parbola y = ax
2
+ bx + c, podemos afirmar que
y
x

a) a > 0, b < 0 e c < 0
b) a < 0, b > 0 e c > 0
c) a < 0, b > 0 e c < 0
d) a < 0, b < 0 e c < 0

Questo 17) Diz-se que a e IR um ponto fixo de uma funo f: IR IR quando f(a) = a. Determine, caso exista(m),
o(s) ponto(s) fixo(s) da funo f: IR IR definida por f(x) = x
2
3x + 3.

Questo 18) Sobre a funo de R em R, definida por y = - x + 2x 4, verdade que
a) admite as razes 5 1+ e 5 1
b) crescente em ]3, 10[.
c) decrescente em ]0, 2[.
d) seu conjunto imagem ]-, -3]
e) assume um valor mnimo para x = 1.

Questo 19) Chama-se Lucro (L), associado produo e venda de um certo produto, a diferena entre a Receita
referente sua venda e o Custo (C) de sua produo. Para determinado produto, uma empresa associa o Custo e a
Receita quantidade produzida q pelas equaes:
R = 8q
5
q
2
e C = 12 +
5
q 8
, onde 0 s q s 40
incorreto afirmar que:
a) o Lucro mximo ocorre quando q = 16 e igual a R$39,20;
b) para 20 < q < 30 o Lucro negativo, isto , h prejuzo;
c) o Lucro nulo para q = 2 e q = 30;
d) o Lucro crescente para 3 < q < 12;
e) o Lucro decrescente para q > 18.





135
Questo 20) Sendo o um parmetro real, tem-se que 1
1 x x
2
s
+ +
o
par a t odo x real, se, e somente se:
a) o s 1
b) o > 0,75
c) o > 1
d) o s 0,75

Questo 21) O mais amplo domnio real da funo f definida por f (x) =
243 3
1
x



Questo 22) Um conjunto de solues da inequao 0
4 x
2
> t t :
a) ] 1; 1[
b) [1; 4]
c) ] 4; 1[
d) ]2; 2[
e) [0; 1]

Questo 23) Se
50
1
5 . 3
3 . 2
x 1
2 x x
=

+
, ento x - 3 igual a:
a) 2
b) 1
c) 1
d) 2
e) 3

Questo 24) Os nmeros reais x, y so tais que y
2x+1
= 1 e
2
3
3
2
x 3
y
= |
.
|

\
|
+ . Nessas condies, o valor de x + y
a) 3/2
b) 1
c) 1/2
d) 0
e) 1/2

Questo 25) No universo U =R, a equao 3 9 0
1 x x +
=
a) no admite solues.
b) admite uma nica soluo, que um nmero natural.
c) admite uma nica soluo, que um nmero no inteiro.
d) admite duas solues distintas, que so nmeros naturais.
e)admite duas solues, sendo uma delas um nmero irracional.



136



PROF.: LEONARDO















Contedos : Funo exponencial

Ministrados durante o ms de Abril



















Data da Entrega : ________/___________/2013

LISTA 3 MATEMTICA

137

Questo 01) Seja f(x) = 2
2x + 1
. Se a e b so tais que f(a) = 4 f(b), pode-se afirmar que:
a) a + b = 2
b) a + b = 1
c) a b = 3
d) a b = 2
e) a b = 1

Questo 02) Aps beber um tanto de cachaa um motorista passa a ter 4 gramas de lcool por litro de sangue. Se
isso ocorrer na hora zero, aps t horas o motorista ter 4 . (0,5)
t
gramas de lcool por litro de sangue. Nessas
condies, a quantidade de lcool em seu sangue ser
a) inferior a 0,5 g/L se t > 3.
b) superior a 0,5 g/L se t > 5.
c) igual a 0,25 g/L se t = 8.
d) inferior a 0,25 g/L se t > 2.
e) superior a 0,25 g/L se t < 8.

Questo 03) Na figura, os grficos I, II e III referem-se, respectivamente, s funes y = a
x
, y = b
x
e y = c
x
. Ento,
est correto afirmar que:
I
y
II
III
0 x

a) 0 < a < b < c.
b) 0 < b < c < a.
c) a < 0 < b < c.
d) 0 < a < c < b.
e) a < 0 < c < b.

Questo 04) Na figura temos o esboo do grfico de y = a
x
+ 1. O valor de 2
3a- 2

;


a) 16
b) 8
c) 2
d) 32
e) 64


Questo 05) Seja f a funo de IR em IR definida por f(x) = 2
x
. O valor de
) 5 x ( f ) 4 x ( f
) 3 x ( f ) 2 x ( f ) 1 x ( f
+ + +
+ + + + +
:
a)
16
39

b)
16
21

c)
12
5

d)
24
7

e)
8
1

Caderno de Atividades

Disciplina:
M Ma at te em m t ti ic ca a

Professor(a):
Leonardo
Aluno:
3 ano
Ensino Mdio
Data de Recebimento:
_____/_____/_____
Lista 03
Data Entrega:
_____/_____/_____

138
Questo 06) O grfico representa a funo y = m.a
x
. Nessas condies, o valor de a
m
:
y
x
0
(1,6)
(2,18)

a) 6
b) 9
c) 12
d) 18
e) 27

Questo 07) Seja f a funo de R em
*
R
+
definida por f(x) = 3
-x
. verdade que:
a) f crescente em R
b) f impar
c) f(x) < 0, para todo x e R.
d) a funo inversa de f dada por
x
1
3
1
log ) x ( f =


e) f
-1
(x) > 0, para todo x e
*
R
+


Questo 08) O grfico que melhor representa a funo f: R R tal que f(x) = a
-x
, para o respectivo intervalo de a, :
y
1
0 x
0 < a < 1
a.

y
1
0 x
a < 1
b.

y
1
0 x
a < 1
c.

y
1
0 x
a >1
d.


Questo 09) Dadas as funes definidas por
x
5
4
) x ( f |
.
|

\
|
= e
x
4
5
) x ( g |
.
|

\
|
= , correto afirmar que
01. os grficos de f(x) e g(x) no se interceptam.
02. f(x) crescente e g(x) decrescente.
04. g(2).f(1) = f(1)
08. f[g(0)] = f(1)
16. f(1) + g(1) = 5/2




139
Questo 10) Sejam f e g funes de R em R dadas por f x
x
( ) . = 2 8 e g x
x
( ) . =
1
16
Os grficos de f e g
interceptam-se em um ponto pertencente ao
a) eixo das ordenadas.
b) primeiro quadrante.
c) segundo quadrante.
d) erceiro quadrante.
e) quarto quadrante.

Questo 11) Se A = log
5
5
2
2, ento o valor de A :
a) 0
b) 1
c) 5
d) 23
e) 25

Questo 12) Se
12
5
5 log 5 log
3 2
a a
= + , ento o valor de a :
a) 5
b) 5
2

c)
5
1

d) 5
e)
5
5


Questo 13) Se os inteiros x e y satisfazem a equao
x 2 y y 1 x
3 2 2 3 = +
+ +
, ento o valor de 3x :
a) 1
b)
3
1

c)
9
1

d) 3
e) 9

Questo 14) Se o par (x
1
,y
1
) soluo do sistema de equaes

=
=
19 y log . 10
x
2 . 3
0 y log . 16
x
2
, ento
1
1
y
x
igual a
a)
10
10 3

b)
3
3 10

c) 10 3
d) 3 5
e)
5
5 3


Questo 15) O valor real de x, tal que 0 ) x 5 1 log( 1 x 5 log = + , um nmero:
a) racional maior que zero.
b) irracional maior que zero.
c) inteiro.
d) racional menor que zero.
e) irracional menor que zero.

Questo 16) Se x um nmero real positivo, tal que log x > log 2 +
3
2
log x, ento,
a) o valor mximo possvel para x log 2.
b) o valor mximo possvel para x 8.
c) o valor mnimo possvel para x log 2.
d) o valor mnimo possvel para x 8.

140
Questo 17)
|
.
|

\
|
= 3
2
x
log y
2
1

Na igualdade acima, supondo x o maior valor inteiro possvel, ento, neste caso, x
y
vale:
a) 4x
b) 1
c) 8x
d) 2
e) 2x

Questo 18) Assinale a alternativa correspondente passagem em que se cometeu um erro
144
16
405
15
<
a) +
|
.
|

\
|
< |
.
|

\
|
9
1
27
1

b) +
2 3
3
1
3
1
|
.
|

\
|
< |
.
|

\
|

c) +
2
5
3
5
3
1
log
3
1
log |
.
|

\
|
< |
.
|

\
|

d) +
|
.
|

\
|
< |
.
|

\
|
3
1
log 2
3
1
log
5 5

e) +
2 3 <
Questo 19) Se
8
15
3 log
1
3 log
1
3 log
1
3 log
1
8 4
x x x
x
= + + + , ento log
3
x vale:
a)
9
1

b)
3
1

c) 3
d) 2
e) 1

Questo 20) Considere os valores inteiros de x tais que 2 ) 3 x ( log
2
1
> . A soma desses valore :
a) 9
b) 22
c) 10
d) 12
e) 15
Questo 21) Se f(x) =logx, ento ) x ( f )
x
1
( f + igual a
a) 10
b) f(x
2
)
c) f(x)
d) 1
e) 0








141
Questo 22)
I. A equao
2 x
3
2
2 x
3
x

+ =

+ no admite solues reais.


II. Se um nmero real x tal que x x > , ento 0 < x < 1.
III.. Se x > 0, ento os grficos das funes reais definidas por y = log
2
x e y = log
8
x so cincidentes.
IV. O grfico da funo real definida por
x
x x
) x ( f
+
= , x = 0, uma parbola.

Dentre as afirmaes acima, o nmero de verdadeiras :
a) 0
b) 1
c) 2
d) 3
e) 4

Questo 23) O valor do logaritmo de
32
1
na base 2 2

Questo 24) Adotando-se os valores log 2 = 0,30 e log 3 = 0,48, a raiz da equao 5
x
= 60 vale aproximadamente:
a) 2,15
b) 2,28
c) 41
d) 2,54
e) 2,67

Questo 25) O valor de um automvel (em unidades monetrias) sofre um depreciao de 4% ao ano. Sabendo-se
que o valor atual de um carro de 40.000 unidades monettiras, depois de quantos anos o valor desse carro ser de
16.000 unidades monetrias? Use o valor 0,3 para log 2 e o valor 0,48 para log 3.
a) 3
b) 6
c) 10
d) 15
e) 23




142






PROF.: LEONARDO






Contedos :
funo Modular e Geral de funes




Ministrados durante o ms de maio
















Data da Entrega : ________/___________/2013
LISTA 4 MATEMTICA

143

Questo 01) O mdulo ,x, de um nmero real x definido por ,x, = x, se x > 0, e ,x, = -x, se x < 0. Das alternativas
abaixo, a que melhor representa o grfico da funo f(x) = x ,x, - 2x + 2 :
1
1 x
y
a.

1
1 x
y b.

1
1 x
y c.

1
1 x
y
d.

1
1 x
y e.

Questo 02) Assinale a afirmativa correta.
A inequao - ,x, < x
a) nunca satisfeita
b) satisfeita em x = 0.
c) satisfeita para x negativo.
d) satisfeita para x positivo.
e) sempre satisfeita

Questo 03) Relativamente funo f(x) = 1 ,x 1,, de [0, 2] em [0, 1], considere as afirmaes:

I. A rea da figura limitada pelo seu grfico e o eixo das abscissas 1.
II. Trata-se de uma funo sobrejetora.
III. A soma das razes da equao f(x) = 0,5 2.

Ento:
a) somente I e II so verdadeiras
b) somente II e III so verdadeiras
c) somente I e III so verdadeiras
d) todas so verdadeiras
e) somente III verdadeira

Questo 04) A soma dos valores inteiros de x que satisfazem simultaneamente as desigualdades: |x 5| < 3 e |x 4| >
1 :
a) 25
b) 13
c) 16
d) 18
e) 21
Caderno de Atividades

Disciplina:
M Ma at te em m t ti ic ca a

Professor(a):
Leonardo
Aluno:
3 ano
Ensino Mdio
Data de Recebimento:
_____/_____/_____
Lista 04
Data Entrega:
_____/_____/_____

144
Questo 05) Um posto de gasolina encontra-se localizado no km 100 de uma estrada retilnea. Um automvel parte
do km 0, no sentido indicado na figura abaixo, dirigindo-se a uma cidade a 250km do ponto de partida. Num dado
instante, x denota a distncia (em quilmetros) do automvel ao km 0. Nesse instante, a distncia (em quilmetros) do
veculo ao posto de gasolina :

km 250
km 100 km 0

a) |100 + x|
b) x 100
c) 100 x
d) |x 100|

Questo 06) Com base no grfico da funo f:R R, representada ao lado, pode-se afirmar:
y
x 1 2 3 0
1

01. A imagem de f o intervalo ]0, 1].
02. A equao f(x) = 1 tem infinitas solues.
04. A equao
2
2
) x ( f = no tem soluo.
08. A funo f admite inversa.
16. O ponto (0, 2) pertence ao grfico de g(x) = 1 + f(x + 1).
32. O grfico da funo f(|x|)
Questo 07) O grfico que melhor representa a funo f : R {3} R definida por
3 x
3 x 2

:
3
-3
0
y
x
a.

3
-2
0 x
y
2
b.

3
-2
0 x
y
2
c.

3
-3
0
d. y
x

3
-2
0 x
y
2
e.

Questo 08) Sejam a b dois nmeros reais positivos tais que a < b e a + b = 4. Se o grfico da funo y = |x a| + |x
b| coincide com o da funo y = 2 no intervalo a s x s b, calcule os valores de a e b.


145
Questo 09) Considerando as afirmaes:

l.
3 x
9 x
2

= x + 3, para todo x real.


ll. |a - 3| = a - 3, para todo a real.

lll. ( ) x 1 x 1
2
= , para todo x real.

Est CORRETO afirmar que:
a) somente a ll falsa.
b) todas so verdadeiras.
c) todas so falsas.
d) somente a lll verdadeira.
e) somente a l falsa.

Questo 10) Sejam os conjuntos de nmeros inteiros, A = {x e Z / x
2
- 3x + 2 = 0} B = {x eZ / |x - 1| < 3}. O
nmero de elementos do conjunto (B - A) ser:
a) 1
b) 3
c) 2
d) 4
e) 5

Questo 11) Considere a funo dada por ) 3 x ( ) x ( f =
a) Calcule ( ) ( ) 2 3 f 2 3 f + +
b) Esboce o grfico da funo

Questo 12) Se A = {x e R: |x
2
+ x + 1| s |x
2
+ 2x 3|}, ento temos:
a) | | +
(

= 4,
2
1
2, - A
b)
(

= 4 ,
2
1
A
c) A = [-3, 1]
d) A = ]-, -3] [1, +[
e) n.d.a.

Questo 13) Julgue os itens abaixo.
00. Se |x| < 2, ento x < 2.
01. Se x < 2, ento |x| < 2.
02. Na reta real, a soluo da desigualdade 2 s |x - 7| s 4 um segmento de reta de comprimento 2.
03. a a
2
= para todo nmero real a.

Questo 14) a) Esboce, para x real, o grfico da funo 6 x | 1 x 2 | | 2 x | ) x ( f + + = . O smbolo o indica o valor
absoluto de um nmero real a e definido por |a| = a, se a > 0 e |a| = a, se a < 0.
b) Para que valores reais de x, f(x) + > 2?


146
Questo 15) O conjunto dos valores assumidos pela expresso algbrica
ab
ab
b
b
a
a
+ sendo a e b dois nmeros reais
diferentes de zero, :
a) {3, 1, 1, 3}
b) {1, 1}
c) {1, 3}
d) {3, 1}
e) {3, 3}

Questo 16) Dadas as funes reais definidas por f(x) = 4x + 1 e f(g(x)) = 3x, ento o valor de k tal que g(f(k)) = 4 :
a)
4
1

b)
5
4

c) 2
d) 3
e)
6
7


Questo 17) Se f(x) = mx + n e f(f(x)) = 4x + 9, a soma dos possveis valores de n :
a) 6
b) 6
c) 12
d) 12
e) 18

Questo 18) Se f(x) = 2x
2
3 e g(x) = x 1, o valor de g[f(2)] :
a) 3
b) 4
c) 5
d) 6
e) 7

Questo 19) Sejam f e g funes de R em R tais que f(x) = x 1 e
2
3 x
)) x ( g ( f
+
= . Nessas condies, verdade
que.
a) g(-2) = 3
b) g(-1) = 1
c)
2
5
) 0 ( g =
d) g(1) = 2
e)
2
5
) 2 ( g =

Questo 20) As funes
x
1
) x ( f = e
1 x
1
) x ( g

= (onde x e R, x = 0 e x = 1) so tais que:


a) (f o g)(x) = (g o f)(x)
b) (f o g)(x) sempre positivo
c) (f o g)(x) . (g o f)(x) = x
d) (f o g)(x) . (g o f)(x) = x . (x 1)

Questo 21) A equao que mais aproximadamente representada
pela curva abaixo :

a) xy = 1.
b) x + y 1 = 0.
c) xy = 0.
d) x
2
y = 0.
e) x y 1 = 0.



147
Questo 22) Sejam f e g funes reais de varivel real definidas por
5
4 x
) x ( g
+
= e
x
5 x
) x ( f

= , com x = 0. Assim, f
-
1
(g(f(x))) igual a
a)
x
x 5

b) 1
x 5
1
+
c) 5x
d)
x
x 5 1

e)
x 5
x 1


Questo 23) O domnio da funo inversa f
-1
(x) de
x 2
1 x 3
) x ( f

+
= :
a) {x e R / x = 2}
b)
)
`

= = e 2 x e
3
1
x / R x
c)
)
`

= e
3
1
x / R x
d) { x e R / x = -3}
e)
)
`

= = e
3
1
x e 3 x / R x

Questo 24) Dada a funo
3
) 2 x ( y + = , a funo inversa f(x)
1
dada por:
a) 2 x ) x ( f
3 1
+ =


b)
3 1
2 x ) x ( f =


c) 2 x ) x ( f
3 1
=


d)
3 1
2 x ) x ( f + =


e)
3 1
x 2 ) x ( f =



Questo 25) Em relao funo R R : f , definida por 8 x 4 ) x ( f + = e a sua inversa ) x ( f
1
, assinale o que for
correto
01. f(x) crescente e ) x ( f
1
decrescente.
02. Os grficos de f(x) e ) x ( f
1
so retas paralelas.
04. Os grficos de f(x) e ) x ( f
1
so retas perpendiculares.
08.
4
8 x
) x ( f
1

=


16. ( ) 30 f ) 2 ( f
2
1 1
=






148







PROF.: LEONARDO






Contedos :
PA e PG




Ministrados durante o ms de Setembro












Data da Entrega : ________/___________/2013
LISTA 5 MATEMTICA

149

Questo 01) Considere uma famlia de circunferncias concntricas, na qual o raio da primeira delas 1 cm, da
segunda 2 cm, da terceira 3 cm e assim por diante, cada raio com acrscimo de 1 cm em relao ao da
circunferncia anterior. A soma dos permetros das 20 primeiras dessas circunferncias , em centmetros, igual a

Questo 02) Um casal tem trs filhos cujas idades esto em progresso aritmtica. Se a soma dessas idades 36
anos e o filho mais velho tem 16 anos, quantos anos tem o filho mais novo?
a) 6
b) 8
c) 10
d) 12
e) 14

Questo 03) A seqncia de nmeros reais, com 12 termos, (89, a, b, c, ..., p, 45) uma progresso aritmrica cujo
oitavo termo vale:
a) 57
b) 59
c) 61
d) 63
e) 65

Questo 04) Os nmeros 1, 2, 3, 4, ......., 9 foram distribudos, sem repeti-los, nos quadrados da figura. Se, em cada
linha, a soma sempre S, o valor de S :

a) 16
b) 15
c) 17
d) 20
e) 18

Questo 05) Uma fita foi enrolada sobre si mesma, num total de 17 voltas, e formou um desenho parecido com a
figura.

Sabendo que a espessura da fita mede 1 mm e a primeira circunferncia formada tem raio 10 mm, o comprimento da
fita quando esticada, vale em mm, aproximadamente:
dado = 3 ~ t
a) 1 836 .
b) 1 972 .
c) 2 008 .
d) 2 144 .
e) 2 280 .


Caderno de Atividades

Disciplina:
M Ma at te em m t ti ic ca a

Professor(a):
Leonardo
Aluno:
3 ano
Ensino Mdio
Data de Recebimento:
_____/_____/_____
Lista 05
Data Entrega:
_____/_____/_____

150
Questo 06) A soma de trs nmeros naturais em progresso aritmtica trinta; a diferena entre o maior e o menor
destes nmeros doze. O menor termo dessa progresso igual a:
a) 2
b) 3
c) 4
d) 5
e) 6

Questo 07) Seja f uma funo real de varivel real tal que f(x + y) = f(x) + f(y) para todos x e y reais. Se a, b, c, d, e
formam, nessa ordem, uma PA de razo r, ento f(a), f(b), f(c), f(d), f(e) formam, nessa ordem,
a) uma PG de razo f(r).
b) uma PG de razo r.
c) uma PA de razo f(a).
d) uma PG de razo f(a).
e) uma PA de razo f(r).

Questo 08) Sejam x, y e z nmeros reais positivos. Se os nmeros log
10
x, log
10
y e log
10
z formam, nessa ordem,
uma progresso aritmtica, ento
a) 2y = xy
b) y
2
= x + z
c) 2y = x + z
d) y
2
= xz

Questo 09) Em uma progresso aritmtica sabe-se que a
3
= 5 e a
17
= 33. Calcule a
10
.

Questo 10) Numa progresso aritmtica o quinto termo excede o primeiro de 36 e o stimo termo a mdia
aritmtica dos nmeros 58, 82 e 76.
Calcule a soma dos 10 primeiros termos dessa progresso.

Questo 11) Se os lados de um tringulo retngulo esto em progresso aritmtica de razo 4, ento o cosseno do
maior ngulo agudo desse tringulo :
a) 0,6
b)
2
3

c) 0,8
d)
2
2


Questo 12) Considere o seguinte problema: As medidas, em centmetros, dos lados de um tringulo retngulo so
numericamente iguais aos termos de uma progresso aritmtica de razo 2. Determinar essas medidas. verdade
que esse problema
a) no tem soluo
b) admite infinitas solues
c) admite duas solues sendo que em uma delas o menor cateto mede 5 cm.
d) admite uma nica soluo, em que o maior cateto mede 6 cm.
e) admite uma nica soluo, em que a hipotenusa mede 10 cm.
Questo 13) Considere a seqncia ordenada de nmeros reais ... , 10 ..., , 10 , 10 , 10
12
n
12
3
12
2
12
1
. O menor nmero natural
n, tal que o produto dos primeiros n termos dessa seqncia seja maior que 100.000, igual a:
a) 11
b) 10
c) 120
d) 121
Questo 14) Em um bloco retangular (isto , paraleleppedo reto retngulo) de volume
8
27
, as medidas das arestas
concorrentes em um mesmo vrtice esto em progresso geomtrica. Se a medida da aresta maior 2, a medida da
aresta menor :
a)
8
7

b)
8
8

c)
8
9

d)
8
10

e)
8
11


151
Questo 15) A seqncia (C
n,3
, A
n,3
, 16.C
n+1,4
) uma progresso geomtrica se, e somente se, n igual a

Questo 16) O nmero de termos da progresso |
.
|

\
|
125 3 , ... ,
5
1
,
25
1
,
125
1


Questo 17) Na figura abaixo tem-se o quadrado Q
1
. Unindo-se os pontos mdios dos lados de Q
1
, construiu-se o
quadrado Q
2
. Unindo-se os pontos mdios dos lados de Q
2
, construiu-se o quadrado Q
3
.
Q
1
Q
2
Q
3

A razo entre as reas de Q
3
e Q
1
, nessa ordem,

Questo 18) Um senhor tem a anos de idade, seu filho tem f anos de idade e seu neto, n. Sobre estes valores,
podemos afirmar:
a) impossvel que a, f e n estejam em progresso aritmtica.
b) impossvel que a, f e n estejam em progresso geomtrica.
c) impossvel que a, f e n estejam simultaneamente em progresso aritmtica e geomtrica.
d) possvel que a, f e n estejam simultaneamente em progresso aritmtica e geomtrica.
e) possvel que a, f e n estejam em progresso aritmtica, mas impossvel que estejam em progresso geomtrica.

Questo 19) Supondo 9 ...
16
k
27
k
8
k
9
k
4
k
3
k
2
k
= + + + + + + + , ento:
a) sen (kt) = 1
b) cos (kt) = 1
c) 1
2
k
sen = |
.
|

\
| t

d) 1
2
k
cos = |
.
|

\
| t

e) sen (kt) > cos (kt)

Questo 20) Se numa progresso geomtrica A
1
. A
4
. A
16
= 3
12
, ento o produto A
5
. A
9
vale
OBS.: A
n
termo de ordem n, com n > 1.
a) 3
b) 3
10
c) 3
4

d) 3
8

e) 3
6


Questo 21) Se numa progresso geomtrica de termos positivos o terceiro termo igual metade da razo, o
produto dos trs primeiros termos igual a:
a)
4
1

b) 4
c)
8
1

d) 8
e)
16
1


Questo 22) Numa progresso geomtrica de 50 termos, a soma dos termos de ordem impar o triplo da soma dos
termos de ordem par. Se o primeiro termo 9, o terceiro termo :
a) 1
b) 3
c) 9
d) 18
e) 27




152
Questo 23) Numa seqncia infinita de crculos, cada crculo, a partir do segundo, tem raio igual metade do raio
do crculo anterior. Se o primeiro crculo tem raio 4, ento a soma das reas de todos os crculos :
a) 12t
b)
4
15

c)
3
64

d) 32t
e)
3
32


Questo 24) Sabe-se que a seqncia (x; y; 10) uma progresso aritmtica e a seqncia (
y
1
; 2; 3y + 4) uma
progresso geomtrica. Nessas condies, correto afirmar que:
a) a razo da progresso aritmtica 4.
b) a razo da progresso geomtrica 8.
c) x . y = 16
d) x + y = 0
e) y = 2x

Questo 25) ... 2 S
32
27
8
9
2
3
+ + + + = a soma dos infinitos termos de uma progresso geomtrica. O valor de
3
S :
a) 2
b) 4
c) 6
d) 8



153






PROF.: LEONARDO







Contedos :
Matemtica Financeira




Ministrados durante o ms de Outubro












Data da Entrega : ________/___________/2013
LISTA 6 MATEMTICA

154

Questo 01) Observe a tabela abaixo:

Considerando os dois produtos que tiveram maior crescimento nas exportaes no perodo mencionado, pode-se
afirmar que a participao desses dois produtos juntos na exportao brasileira, em porcentagem, foi de:
a) 80,8
b) 35,4
c) 27,9
d) 19,2
e) 18,4

Questo 02) Numa barraca de feira, uma pessoa comprou mas, bananas, laranjas e pras. Pelo preo normal da
barraca, o valor pago pelas mas, bananas, laranjas e pras corresponderia a 25%, 10%, 15% e 50% do preo total,
respectivamente. Em virtude de uma promoo, essa pessoa ganhou um desconto de 10% no preo das pras. O
desconto assim obtido no valor total de sua compra foi de:
a) 7,5%
b) 10%
c) 5%
d) 15%
e) 17,5%

Questo 03) O limite de consumo mensal de energia eltrica de uma residncia, sem multa, foi fixado em 320 kWh.
Pelas regras do racionamento, se este limite for ultrapassado, o consumidor dever pagar 50% sobre o excesso. Alm
disso, em agosto, a tarifa sofreu um reajuste de 16%. Suponha que o valor pago pelo consumo de energia eltrica no
ms de outubro tenha sido 20% maior do que aquele que teria sido pago sem as regras do racionamento e sem, o
aumento da tarifa em agosto. Pode-se, ento, concluir que o consumo de energia eltrica, no ms de outubro, foi de
aproximadamente:
a) 301 kWh
b) 343 kWh
c) 367 kWh
d) 385 kWh
e) 413 kWh

Questo 04) Riobaldo, ao voltar de uma viagem por Portugal, Espanha e Frana, converteu todas as suas despesas
em reais e percebeu que sua despesa em Portugal foi o dobro da despesa na Espanha, que, por sua vez, foi 20%
menor que a despesa na Frana. Se a despesa total nos trs pases foi de 4.200 reais, quanto Riobaldo gastou na
Frana a mais que na Espanha?
a) 250 reais
b) 300 reais
c) 150 reais
d) 200 reais
Caderno de Atividades

Disciplina:
M Ma at te em m t ti ic ca a

Professor(a):
Leonardo
Aluno:
3 ano
Ensino Mdio
Data de Recebimento:
_____/_____/_____
Lista 06
Data Entrega:
_____/_____/_____

155
Questo 05) Sejam os nmeros racionais x, y e z, tais que z igual a 20% de y e y igual a 40% de x. Se x + y + z =
74, ento

Questo 06) Uma inflao mensal de 2% acumula durante quatro meses uma inflao de, aproximadamente
a) 7%
b) 9%
c) 8,25%
d) 10%
e) 12%

Questo 07) Uma indstria opera com um custo fixo de produo (sem contar os impostos) de 100.000 reais por ano
e tem de pagar impostos sobre 30% de seu faturamento bruto. Quanto deve faturar para que seu lucro no ano seja de,
no mnimo, 40.000 reais?

Questo 08) Uma loja comunica a seus clientes que promover, no prximo ms, um desconto de 30 % em todos os
seus produtos. Na ocasio do desconto, para que um produto que hoje custa k mantenha este preo, ele dever ser
anunciado por:
a)
3
k 7

b)
3
k 10

c)
10
k 17

d)
3
k 17

e)
7
k 10


Questo 09) Numa loja, o preo de um produto tem um desconto de 15% se for pago vista ou um acrscimo de 5%
se for pago com carto de crdito. Tendo optado pelo carto, uma pessoa pagou R$ 80,00 de acrscimo em relao ao
que pagaria, com desconto, vista. Ento a soma dos preos do produto vista com desconto e no carto :
a) R$ 700,00
b) R$ 740,00
c) R$ 760,00
d) R$ 720,00
e) R$ 780,00
Questo 10) Numa festa, a razo entre o nmero de moas e o de rapazes
12
13
A porcentagem de rapazes na
festa :
a) 44 %
b) 45 %
c) 40 %
d) 48 %
e) 46 %

Questo 11) Um capital C, aplicado a juros compostos a uma taxa unitria i por perodo, produz, ao final de n
perodos, o montante M, dado por M = C (1 + i)
n
. Nessas condies, utilizando-se log 2 = 0,30 e log 3 = 0,48, o capital
de R$ 2000,00, aplicado a juros compostos taxa de 20% ao ano, produzir o montante de 5000,00, ao final de um
perodo.
a) 4 anos
b) 4 anos e 2 meses
c) 4 anos e 8 meses.
d) 5 anos
e) 5 anos e 6 meses

Questo 12) A tabela a seguir indica o preo de mercado de certo carro, de acordo com o ano de fabricao:

Preo(R$)1870015700132009300
Ano 2000 199919981997



156
Essa tabela mostra que o valor do veculo aumenta cerca de 19% ao ano. Com base nessas informaes, o valor
aproximado do modelo 2001 desse carro pode ser obtido multiplicando-se 18 700 por:
a) 0,81
b) 0,19
c) 1,19
d) 1,81

Questo 13) O preo vista de uma mercadoria de R$ 130,00. O comprador pode pagar 20% de entrada no ato
da compra e o restante em uma nica parcela de R$ 128,96, vencvel em 3 meses.
Admitindo-se o regime de juros simples comerciais, a taxa de juros anual cobrada na venda a prazo de:
a) 94%
b) 96%
c) 98%
d) 100%

Questo 14) Um fabricante vende determinado produto pelo preo p, para pagamento a meses aps a compra. Se o
pagamento fora feira vista, h um desconto igual a 5% de p. A taxa mensal de juros simples do financiamento :
a)
n 19
100
%
b)
n 20
100
%
c)
n 21
100
%
d)
n 22
100
%
e)
n 22
100
%

Questo 15) Uma pessoa tomou um emprstimo de R$ 6000,00 a uma taxa de juros compostos de 10% ao ano e
saldou a dvida da seguinte maneira:

- 2 anos aps ter contrado a dvida, pagou R$ 2260,00;
- 2 anos aps o primeiro pagamento, pagou mais R$ 3050,00;
- 1 ano aps o segundo pagamento, quitou a dvida.

Nessas condies, pode-se afirmar:
01. Depois do primeiro pagamento, a pessoa ficou devendo R$ 4340,00.
02. Aps o segundo pagamento, a dvida correspondia a 50% do valor do emprstimo.
04. No momento em que a pessoa quitou o emprstimo, a dvida correspondia a R$ 3300,00.
08. O montante pago pelo emprstimo foi igual a R$ 9000,00.
16. O valor pago pelos juros da dvida correspondeu a 43,5% do valor do emprstimo.

Questo 16) Andr devia, em seu carto de crdito, R$ 1.000,00. Como no conseguiu pagar, em dois meses essa
dvida aumentou para R$ 1.440,00. Nesse caso, qual foi a taxa de juros simples cobrada mensalmente pelo carto de
crdito?
a) 7,2%
b) 14,4%
c) 20%
d) 22%
e) 44%

Questo 17) Jos emprestou R$ 500,00 a Joo por 5 meses, no sistema de juros simples, a uma taxa de juros fixa e
mensal. Se no final dos 5 meses Jos recebeu um total de R$ 600,00, ento a taxa fixa mensal aplicada foi de:
a) 0,2%.
b) 0,4%.
c) 2%.
d) 4%.
e) 6%.

Questo 18) A funo P = 60 (1,04)
t
representa a estimativa do Produto Interno Bruto em bilhes de dlares (PIB)
de um pas no ano t adotando-se a seguinte conveno:

t = 0 representa o ano de 1996
t = 1 representa o ano de 1997
t = 2 representa o ano de 1998

e assim por diante.
a) Qual a estimativa do aumento percentual do PIB de 1999 em relao ao de 1998?
b) Em que ano o PIB ser aproximadamente o dobro do que era em 1996? Use aproximao por valores superiores e
adote os seguintes dados:
log 2 = 0,3010
log 13 = 1,1139

157
Questo 19) A rede Corcovado de hipermercados promove a venda de uma mquina fotogrfica digital pela seguinte
oferta: Leve agora e pague daqui a 3 meses. Caso o pagamento seja feito vista, Corcovado oferece ao consumidor
um desconto de 20%. Caso um consumidor prefira aproveitar a oferta, pagando no final do 3 ms aps a compra, a
taxa anual de juros simples que estar sendo aplicada no financiamento de:
a) 20%
b) 50%
c) 100%
d) 80%
e) 120%

Questo 20) De sua turma de 30 alunos, escolhida uma comisso de 3 representantes. Qual a probabilidade de
voc fazer parte da comisso?
a)
10
1

b)
12
1

c)
24
5

d)
3
1

e)
9
2


Questo 21) Alberto tomou um emprstimo de R$20 000,00 taxa de juro simples de 10% ao ano. Algum tempo
depois, considerando que o valor dos juros era muito alto, obteve um outro emprstimo de R$30 000,00, taxa de juro
simples de 8% ao ano. Liquidou a dvida do primeiro emprstimo, pagando tambm os juros e ainda restou algum
dinheiro. Dezoito meses depois da data do primeiro emprstimo liquidou o dbito, inclusive juros, do segundo
emprstimo.
Determine os prazos dos dois emprstimos, em meses, sabendo que Alberto pagou R$3 500,00 de juros totais nos
dois emprstimos.

Questo 22) Paulo tem R$150 000,00 aplicados num fundo de investimentos, taxa de juro composto de 20% ao
ano e quer comprar um apartamento de R$200 000,00 vista. Para adquirir o imvel, Paulo est diante de duas
possibilidades:

I. Comprar a prazo, mediante o seguinte plano de financiamento proposto pelo vendedor: R$80 000,00 de entrada,
R$84 000,00 no final de 1 ano e R$83 500,00 no final de 2 anos.
II. Comprar vista, obtendo um emprstimo de R$50 000,00 taxa de juro composto de 30% ao ano, a ser pago no
final de 2 anos.
Por qual dos dois planos Paulo deveria optar?
Justifique!

Questo 23) Carlos recebeu R$240 000,00 pela venda de um imvel. Gastou metade dessa quantia na compra de
um apartamento no litoral e investiu o dinheiro que restou em fundos de investimentos de trs instituies financeiras:
40% no Banco A, 30% no Banco B e 30% no Banco C.
Aps um ano, vendeu o apartamento do litoral por R$144 000,00 e resgatou as aplicaes, cujos rendimentos anuais
foram de +20%, 10% e +30%, respectivamente, nos Bancos A, B e C. correto afirmar que, em um ano, Carlos
aumentou o capital de R$240 000,00, recebido inicialmente, em:
a) 80%
b) 36%
c) 20%
d) 18,50%
e) 17%

Questo 24) Um investidor quer aplicar 120 mil reais. Seu corretor lhe oferece um investimento, em duas fases, com
as seguintes regras:

- Na 1 fase do investimento, ocorrer um dentre os dois eventos seguintes: com probabilidade p, o investidor ganha
metade do que investiu; com probabilidade (1 p), o investidor perde um tero do que investiu.
- Na 2 fase do investimento, a quantia final da 1 fase ser reinvestida, de forma independente da 1 fase. Neste novo
investimento, ocorrer um dentre os dois eventos seguintes: com probabilidade 1/2, o investidor ganha a quarta parte
do que foi reinvestido; com probabilidade 1/2, o investidor perde metade do que foi reinvestido.

a) Se o investidor aplicar seu dinheiro desta forma, com que valores pode ficar ao trmino do investimento? Qual a
probabilidade, em funo de p, de ficar com cada um desses valores?
b) Uma revista especializada informa que, neste investimento, a probabilidade de perder dinheiro 70%. Admitindo
como correta a informao da revista, calcule p.

Questo 25) Suponha uma inflao mensal de 4% durante um ano. De quanto ser a inflao acumulada neste ano?
(Pode deixar indicado o resultado)

158







PROF.:RINALDO JOS DE RESENDE






C
C
o
o
n
n
t
t
e
e

d
d
o
o
s
s
:
:

B
B
I
I
O
O
Q
Q
U
U

M
M
I
I
C
C
A
A
I
I
























D Da at ta a d da a E En nt tr re eg ga a : : _ __ __ __ __ __ __ __ _/ /_ __ __ __ __ __ __ __ __ __ __ _/ /2 20 01 13 3
LISTA 1 BIOLOGIA

159

01 - (UFG GO/1993) As clulas so formadas por substncias que participam da sua estrutura e de reaes
importantes que garantem a sua vida. Nas clulas existem componentes orgnicos, como os lipdios, dependendo de
inmeros fatores como idade, atividade e espcie.
a) Cite e explique 3 funes da gua no organismo pluricelular.
b) Descreva o processo de osmorregulao em peixes marinhos que, na poca da desova, retornam aos rios onde
nasceram.

02 - (UFAC/2006) Estudos experimentais realizados com membranas celulares possibilitaram a descoberta de uma
grande variedade de protenas. Algumas dessas protenas se organizam na membrana plasmtica para formarem
poros ou canais, os quais possibilitam a livre passagem de certas molculas e ons, por tal membrana. O impedimento
da formao de tais canais especficos para o trnsito de determinados ons pode se refletir como uma doena
hereditria. No caso da fibrose cstica, indique em qual das alternativas consta o on que est impedido de transitar
pela membrana plasmtica.
a) Sdio
b) Potssio
c) Cloro
d) Gs carbnico
e) Gs oxignio

03 - (UFRJ/1994) Em indivduos crnicamente subnutridos ou que ingerem dietas pobres em protenas, ocorre uma
sndrome que afeta principalmente as crianas. (Na frica, essa sndrome foi denominada de kwashiorkor,
significando, literalmente, o rejeitado, uma referncia ao desmame.)
O nvel muito baixo de protenas no sangue permite o aparecimento de edemas, principalmente na regio abdominal.
O edema resulta do extravasamento de gua dos vasos para o espao intersticial dos tecidos, onde retida. No Brasil,
esse edema conhecido como barriga dgua.
Explique a correlao entre os nveis proticos baixos e a formao da barriga dgua.

04 - (UNESP SP/1999) Os acares complexos, resultantes da unio de muitos monossacardeos, so denominados
polissacardeos.
a) Cite dois polissacardeos de reserva energtica, sendo um de origem animal e outro de origem vegetal.
b) Indique um rgo animal e um rgo vegetal, onde cada um destes acares pode ser encontrado.

05 - (UFG GO/1994) O que existe dentro do nosso crebro tem muito a ver com o que ingerimos. Sobre o papel da
alimentao no metabolismo do indivduo,
a) explique por que a ingesto de alimentos hipocalricos combate a obesidade a longo prazo;
b) explique a importncia da ingesto de alimentos vegetais e o papel da celulose no processo digestivo.

06 - (UNICAMP SP/1999) Os lipdios tm papel importante na estocagem de energia estrutura de membranas
celulares, viso, controle hormonal, entre outros. So exemplos de lipdios: fosfolipdios, esterides e carotenides.
a) Como o organismo humano obtm os carotenides? Que relao tem com a viso?
b) A quais das funes citadas no texto acima os esterides esto relacionados? Cite um esteride importante para
uma dessas funes.
c) Cite um local de estocagem de lipdios em animais e um em vegetais.

07 - (UERJ/2005)As estatinas, por seu grande xito na preveno da doena coronariana, esto entre os
medicamentos mais prescritos no mundo. Essas substncias atuam sobre a enzima que regula a sntese de colesterol
pelo fgado, denominada, simplificadamente, de HMG-CoA redutase.
Para testar a eficincia de vrios derivados de estatinas, utilizou-se uma preparao de HMG-CoA redutase isolada de
tecido heptico. A velocidade de reao dessa preparao enzimtica foi medida em funo de concentraes
crescentes de seu substrato HMG-CoA, na ausncia e na presena de uma concentrao fixa de trs derivados de
estatina. Nesses experimentos, o pH, a temperatura, a concentrao da enzima e a concentrao dos co-fatores
necessrios foram sempre mantidos constantes. O grfico abaixo representa os resultados encontrados; a curva 1 foi
obtida na ausncia de estatinas.

Caderno de Atividades

Disciplina:
B Bi io ol lo og gi ia a

Professor(a):
Rinaldo
Aluno:
3 ano
Ensino Mdio
Data de Recebimento:
_____/_____/_____
Lista 01
Data Entrega:
_____/_____/_____

160


a) Nomeie o tipo de mecanismo de ao das estatinas sobre a enzima HMG-CoA redutase heptica e justifique sua
resposta.
b) Aponte uma substncia sintetizada a partir do colesterol em nosso organismo, no caracterizada como hormnio, e
sua respectiva funo.

08 - (UFRJ/2004) Os lipdeos so os nutrientes de maior teor calrico, seguidos por carboidratos e protenas. A
elevao dos nveis sangneos de insulina um dos principais sinais responsveis pela mobilizao dos excedentes
nutricionais sob forma de lipdeos pelo tecido adiposo.
Com o intuito de evitar esse efeito da insulina, muitas pessoas recorrem a uma dieta baseada na ingesto exclusiva de
lipdeos e protenas. Apesar de seus efeitos sobre a sade serem discutveis, esse tipo de dieta pode conduzir
efetivamente a uma perda e massa corporal (peso).
Explique por que uma dieta baseada na excluso total dos carboidratos, apesar doseu alto valor calrico, no leva ao
acmulo de lipdeos no tecido adiposo.

09 - (UFTM MG/2007) Algumas bebidas base de soja e suco de frutas, vendidas em caixinhas longa-vida, estampam
com destaque em suas embalagens:
No contm lactose e colesterol.
Um dos consumidores desse tipo de bebida, preocupado com o produto que consome, procurou saber se a iseno de
lactose e colesterol na bebida era devida ao fato de os ingredientes bsicos do produto, soja e frutas, terem sido
modificados geneticamente.
a) A preocupao do consumidor justifica-se, ou seja, h a possibilidade de a soja e a fruta empregadas na composio
da bebida terem sido modificadas geneticamente para no produzirem lactose e colesterol? Justifique.
b) No organismo humano, quais as funes da lactose e do colesterol?

10 - (UFC CE/1997) Toda criana, ao nascer, deve fazer o "Teste do Pezinho", o qual capaz de detectar algumas
doenas metablicas congnitas. Uma das mais conhecidas, impede o metabolismo normal do aminocido
"fenilalanina". Pergunta-se:
a) Qual o nome dessa doena e o que ela pode causar ao indivduo se ele no for tratado precocemente?
b) Que tratamento permitir o desenvolvimento normal da criana portadora desse erro metablico?

11 - (UFRJ/1996) A fenilcetonria uma doena que resulta de um defeito na enzima fenilalanina hidroxilase, que
participa do catabolismo do aminocido fenilalanina.
A falta de hidroxilase produz o acmulo de fenilalanina que, por transaminao, forma cido fenilpirvico. Quando em
excesso, o cido fenilpirvico provoca retardamento mental severo.
Por outro lado, o portador desse defeito enzimtico pode ter uma vida normal desde que o defeito seja diagnosticado
imediatamente aps o nascimento e que sua dieta seja controlada.
A fenilcetonria to comum que mesmo nas latas de refrigerantes dietticos existe o aviso:
"Este produto contm fenilcetonricos!".
Qual o principal cuidado a tomar com a dieta alimentar de um portador desse defeito enzimtico? Por qu?

12 - (UNESP SP/1999) A figura ilustra um modelo do sistema chave-fechadura, onde observamos enzima,
substrato e produto do sistema digestivo humano.
+
enzima enzima enzima e substrato
substrato
produto

a) Se o substrato fosse uma protena que estivesse sendo degradada no estmago, qual seria a enzima especfica e
o produto obtido neste rgo?
b) Se a digesto de um determinado alimento ocorresse no intestino delgado e os produtos obtidos fossem glicerol e
cidos graxos, quais seriam, respectivamente, o substrato e a enzima?

161
13 - (UFRRJ/2006) A protena 1 foi isolada a partir do sangue de um vertebrado, e a protena 2 foi isolada a partir de
um meio de cultura de bactrias. Aps estudos de seqenciamento, foi observado que ambas as protenas
apresentavam, cada uma, 471 resduos de aminocidos em sua estrutura.
Com base nas informaes apresentadas acima, podemos afirmar que as duas protenas so iguais entre si? Explique.

14 - (UFRRJ/2006) Radicais livres um assunto da moda H muito tempo que a Medicina j reconhece os radicais
livres como verdadeiros viles do nosso organismo. Eles so tomos ou molculas livres dotados de cargas eltricas,
resultantes, muitas vezes, das prprias reaes intracelulares, ou provenientes do meio externo, que se mostram
prejudiciais sade. So conseqncias do estresse, dos desvios alimentares, do fumo, das atividades fsicas
exageradas e da poluio ambiental. Atualmente, esto sendo muito comentados, tendo em vista os avanos da
Medicina Ortomolecular.
A partir do exposto, responda:
a) De que maneira os radicais livres agem no organismo humano?
b) Cite um exemplo de local e forma de ao dos radicais livres no organismo do homem.

15 - (UNICAMP SP/1999) Cada marinheiro da esquadra de Cabral recebia mensalmente para suas refeies 15kg de
carne salgada, cebola, vinagre, azeite e 12kg de biscoito. O vinagre era usado nas refeies e para desinfetar o poro,
no qual, acreditava-se, escondia-se a mais temvel enfermidade da vida do mar. a partir do sculo XVIII essa doena
foi evitada com a introduo de frutas cidas na dieta dos marinheiros. Hoje sabe-se que essa doena era causada
pela deficincia de um nutriente essencial na dieta.
(Adaptado de: E. Bueno, A viagem do descobrimento, Rio de Janeiro, Objetiva, 1998.)
a) Que nutrientes esse?
b) Que doena causada pela falta desse nutriente?
c) Cite duas manifestaes aparentes ou sintomas dessa doena.

16 - (UFF RJ/2006) Faz tempo, dei a um beb doses de Mozart na mamadeira, Luiz Gonzaga como canes de ninar,
Vivaldi e J.S. Bach, do maternal ao ensino mdio. A boa alimentao musical deve comear na infncia, ser completa,
variada, incluindo degustao de produtos no perecveis, para evitar avitaminose da alma. Dentre os direitos da
criana deveria estar tambm a boa nutrio do esprito
(adaptado do texto A novela de Bizet escrito por Tereza Halliday e publicado no Dirio de Pernambuco, 12 de junho
de 2003).
A sade de seres humanos, principalmente crianas, sofre danos quando ficam por um longo perodo na ausncia de
luz solar, podendo ser acometidos por uma avitaminose.
a) De que vitamina esses indivduos ficariam carentes sob tais condies?
b) Qual o precursor de origem vegetal utilizado para a produo dessa vitamina, no corpo humano?
c) Que alterao ou deformidade essa avitaminose pode causar a esses indivduos? Por qu?

17 - (UFG GO/2004) A regio semi-rida brasileira caracteriza-se por baixa pluviosidade, perodos prolongados de
seca e baixa produtividade agrcola, que ocasionam carncia nutricional na populao humana. Considerando o
conjunto dos fatores mencionados, apresente e explique trs conseqncias da dieta alimentar pobre da referida
populao, associando-as com o desemprego em exerccios fsicos.

18 - (UFRN/2007) O quadro abaixo apresenta as necessidades dirias de algumas vitaminas para o ser humano
saudvel.
Os dois grficos mostram, respectivamente, a reduo da concentrao de vitaminas do complexo B e das vitaminas A
e E no organismo.

a) A partir do quadro e levando em considerao o papel das vitaminas
no organismo, explique por que o organismo humano apresenta essas
diferenas nas necessidades dirias de cido flico e de vitamina B
12

em relao s vitaminas C e E.
b) A partir dos grficos, explique a diferena existente na reduo das
concentraes das vitaminas A e E em relao ao complexo B.

19 - (UEG GO/2007) Em 1747, o mdico escocs James Lind publicou
um tratado no qual esclareceu a preveno do escorbuto. Sobre esse
tema, responda ao que se pede.
a) Que vitamina hidrossolvel encontra-se deficiente nas pessoas com
escorbuto?
b) Qual a funo das vitaminas nas reaes enzimticas?





162
QUESTES DO ENEM

01 - (ENEM Simulado/2009) O uso da gua do subsolo requer o bombeamento para um reservatrio elevado. A
capacidade de bombeamento (litros/hora) de uma bomba hidrulica depende da presso mxima de bombeio,
conhecida como altura manomtrica H (em metros), do comprimento L da tubulao que se estende da bomba at o
reservatrio (em metros), da altura de bombeio h (em metros) e do desempenho da bomba (exemplificado no grfico).
De acordo com os dados a seguir, obtidos de um fabricante de bombas, para se determinar a quantidade de litros
bombeados por hora para o reservatrio com uma determinada bomba, deve-se:

1. Escolher a linha apropriada na tabela correspondente altura (h), em metros, da entrada de gua na bomba at o
reservatrio.
2. Escolher a coluna apropriada, correspondente ao comprimento total da tubulao (L), em metros, da bomba at o
reservatrio.
3. Ler a altura manomtrica (H) correspondente ao cruzamento das respectivas linha e coluna na tabela.
4. Usar a altura manomtrica no grfico de desempenho para ler a vazo correspondente.




Disponvel em: http://www.anauger.com.br. Acesso em: 19 mai, 2009 (adaptado).

Considere que se deseja usar uma bomba, cujo desempenho descrito pelos dados acima, para encher um
reservatrio de 1.200 L que se encontra 30m acima da entrada da bomba. Para fazer a tubulao entre a bomba e o
reservatrio seriam usados 200m de cano. Nessa situao, de se esperar que a bomba consiga encher o
reservatrio
a) entre 30 e 40 minutos.
b) em menos de 30 minutos.
c) em mais de 1h e 40 minutos.
d) entre 40 minutos e 1h e 10 minutos.
e) entre 1h e 10 minutos e 1h e 40 minutos.

02 - (ENEM/2011) A produo de soro antiofdico feita por meio da extrao da peonha de serpentes que, aps
tratamento, introduzida em um cavalo. Em seguida so feitas sangrias para avaliar a concentrao de anticorpos
produzidos pelo cavalo. Quando essa concentrao atinge o valor desejado, realizada a sangria final para obteno
do soro. As hemcias so devolvidas ao animal, por meio de uma tcnica denominada plasmaferese, a fim de reduzir
os efeitos colaterais provocados pela sangria.
Disponvel em: http://www.infobibos.com. Acesso em: 28 abr. 2010 (adaptado).
A plasmaferese importante, pois, se o animal ficar com uma baixa quantidade de hemcias, poder apresentar
a) febre alta e constante.
b) reduo de imunidade.
c) aumento da presso arterial.
d) quadro de leucemia profunda.
e) problemas no transporte de oxignio.

03 - (ENEM/2011) O etanol considerado um biocombustvel promissor, pois, sob o ponto de vista do balano de
carbono, possui uma taxa de emisso praticamente igual a zero. Entretanto, esse no o nico ciclo biogeoqumico
associado produo de etanol. O plantio da cana-de-acar, matria-prima para a produo de etanol, envolve a
adio de macronutrientes como enxofre, nitrognio, fsforo e potssio, principais elementos envolvidos no
crescimento de um vegetal. Revista Qumica Nova na Escola. no 28, 2008.
O nitrognio incorporado ao solo, como consequncia da atividade descrita anteriormente, transformado em
nitrognio ativo e afetar o meio ambiente, causando

163
a) o acmulo de sais insolveis, desencadeando um processo de salinizao do solo.
b) a eliminao de microrganismos existentes no solo responsveis pelo processo de desnitrificao.
c) a contaminao de rios e lagos devido alta solubilidade de ons como NO
3

e NH
4
+
em gua.
d) a diminuio do pH do solo pela presena de NH
3
, que reage com a gua, formando o NH
4
OH (aq).
e) a diminuio da oxigenao do solo, uma vez que o nitrognio ativo forma espcies qumicas do tipo NO
2
, NO
3

,
N
2
O.

04 - (ENEM/2011) O perxido de hidrognio comumente utilizado como antissptico e alvejante. Tambm pode ser
empregado em trabalhos de restaurao de quadros enegrecidos e no clareamento de dentes. Na presena de
solues cidas de oxidantes, como o permanganato de potssio, este xido decompe-se, conforme a equao a
seguir:

5 H
2
O
2
(aq) + 2 KMnO
4
(aq) + 3 H
2
SO
4
(aq)
5 O
2
(g) + 2 MnSO
4
(aq) + K
2
SO
4
(aq) + 8 H
2
O (l)
ROCHA-FILHO, R. C. R.; SILVA, R. R. Introduo aos Clculos da Qumica.
So Paulo: McGraw-Hill, 1992.

De acordo com a estequiometria da reao descrita, a quantidade de permanganato de potssio necessria para reagir
completamente com 20,0 mL de uma soluo 0,1 mol/L de perxido de hidrognio igual a
a) 2,010
0
mol.
b) 2,010
3
mol.
c) 8,010
1
mol.
d) 8,010
4
mol.
e) 5,010
3
mol.

05 - (ENEM/2011) Em um experimento realizado para determinar a densidade da gua de um lago, foram utilizados
alguns materiais conforme ilustrado: um dinammetro D com graduao de 0 N a 50 N e um cubo macio e
homogneo de 10 cm de aresta e 3 kg de massa. Inicialmente, foi conferida a calibrao do dinammetro,
constatando-se a leitura de 30 N quando o cubo era preso ao dinammetro e suspenso no ar. Ao mergulhar o cubo na
gua do lago, at que metade do seu volume ficasse submersa, foi registrada a leitura de 24 N no dinammetro.

Considerando que a acelerao da gravidade local de 10 m/s
2
, a densidade da gua do lago,
em g/cm
3
,
a) 0,6.
b) 1,2.
c) 1,5.
d) 2,4.
e) 4,8.

06 - (ENEM/2010) O cdmio, presente nas baterias, pode chegar ao solo quando esses
materiais so descartados de maneira irregular no meio ambiente ou quando so incinerados. Diferentemente da forma
metlica, os ons Cd
2+
so extremamente perigosos para o
organismo, pois eles podem substituir ons Ca
2+
, ocasionando
uma doena degenerativa nos ossos, tornando-os muito porosos
e causando dores intensas nas articulaes. Podem ainda inibir
enzimas ativadas pelo ction Zn
2+
, que so extremamente
importantes para o funcionamento dos rins. A figura mostra a
variao do raio de alguns metais e seus respectivos ctions.

Com base no texto, a toxicidade do cdmio em sua forma inica
consequncia de esse elemento

a) apresentar baixa energia de ionizao, o que favorece a
formao do on e facilita sua ligao a outros compostos.
b) possuir tendncia de atuar em processos biolgicos mediados por ctions metlicos com cargas que variam de +1
a +3.
c) possuir raio e carga relativamente prximos aos de ons metlicos que atual nos processos biolgicos, causando
interferncia nesses processos.
d) apresentar raio inico grande, permitindo que ele cause interferncia nos processos biolgicos em que,
normalmente, ons menores participam.
e) apresentar carga +2, o que permite que ele cause interferncia nos processos biolgicos em que, normalmente,
ons com cargas menores participam.




164
07 - (ENEM/2010) O rtulo de uma garrafa de gua mineral natural contm as seguintes informaes:

As informaes qumicas presentes no rtulo de vrios produtos
permitem classificar o produto de vrias formas, de acordo com
seu gosto, seu cheiro, sua aparncia, sua funo, entre outras. As
informaes da tabela permitem concluir que essa gua

a) gasosa.
b) inspida.
c) levemente azeda.
d) um pouco alcalina.
e) radioativa na fonte.












08 - (ENEM/2010) Devido ao seu teor de sais, a gua do mar imprpria para o consumo humano e para a maioria
dos usos da gua doce. No entanto, para a indstria, a gua do mar de grande interesse, uma vez que os sais
presentes podem servir de matrias-primas importantes para diversos processos. Nesse contexto, devido a sua
simplicidade e ao seu baixo potencial de impacto ambiental, o mtodo da precipitao fracionada tem sido utilizado
para a obteno dos sais presentes na gua do mar.

Tabela 1: Solubilidade em gua de alguns compostos presentes na gua do mar a 25C



Suponha que uma indstria objetiva separar determinados sais de uma amostra de gua do mar a 25C, por meio da
precipitao fracionada. Se essa amostra contiver somente os sais destacados na tabela, a seguinte ordem de
precipitao ser verificada:

a) Carbonato de clcio, sulfato de clcio, cloreto de sdio e sulfato de magnsio, cloreto de magnsio e, por ltimo,
brometo de sdio.
b) Brometo de sdio, cloreto de magnsio, cloreto de sdio e sulfato de magnsio, sulfato de clcio e, por ltimo,
carbonato de clcio.
c) Cloreto de magnsio, sulfato de magnsio e cloreto de sdio, sulfato de clcio, carbonato de clcio e, por ltimo,
brometo de sdio.
d) Brometo de sdio, carbonato de clcio, sulfato de clcio, cloreto de sdio e sulfato de magnsio e, por ltimo,
cloreto de magnsio.
e) Cloreto de sdio, sulfato de magnsio, carbonato de clcio, sulfato de clcio, cloreto de magnsio e, por ltimo,
brometo de sdio.



165
09 - (ENEM/2010) Ao colocar um pouco de acar na gua e mexer at a obteno de uma s fase, prepara-se uma
soluo. O mesmo acontece ao se adicionar um pouquinho de sal gua e misturar bem. Uma substncia capaz de
dissolver o soluto denominada solvente; por exemplo, a gua um solvente para o acar, para o sal e para vrias
outras substncias. A figura a seguir ilustra essa citao.


Disponvel em: www.sobiologia. com.br. Acesso em: 27 abr. 2010.

Suponha que uma pessoa, para adoar seu cafezinho, tenha utilizado 3,42g de sacarose (massa molar igual a 342
g/mol) para uma xcara de 50 mL do lquido. Qual a concentrao final, em mol/L, de sacarose nesse cafezinho?

a) 0,02
b) 0,2
c) 2
d) 200
e) 2000

10 - (ENEM/2009) Sabes so sais de cidos carboxlicos de cadeia longa utilizados com a finalidade de facilitar,
durante processos de lavagem, a remoo de substncias de baixa solubilidade em gua, por exemplo, leos e
gorduras. A figura a seguir representa a estrutura de uma molcula de sabo.



Em soluo, os nions do sabo podem hidrolisar a gua e, desse modo, formar o cido carboxlico correspondente.
Por exemplo, para o estearato de sdio, estabelecido o seguinte equilbrio:

+ + OH COOH ) CH ( CH O H COO ) CH ( CH
16 2 3 2 16 2 3


Uma vez que o cido carboxlico formado pouco solvel em gua e menos eficiente na remoo de gorduras, o pH
do meio deve ser controlado de maneira a evitar que o equilbrio acima seja deslocado para a direita.

Com base nas informaes do texto, correto concluir que os sabes atuam de maneira

a) mais eficiente em pH bsico.
b) mais eficiente em pH cido.
c) mais eficiente em pH neutro.
d) eficiente em qualquer faixa de pH.
e) mais eficiente em pH cido ou neutro.


166







PROF.:RINALDO JOS DE RESENDE







C
C
o
o
n
n
t
t
e
e

d
d
o
o
s
s
:
:

B
B
I
I
O
O
Q
Q
U
U

M
M
I
I
C
C
A
A
I
I
I
I
























D Da at ta a d da a E En nt tr re eg ga a : : _ __ __ __ __ __ __ __ _/ /_ __ __ __ __ __ __ __ __ __ __ _/ /2 20 01 13 3
LISTA 2 BIOLOGIA

167
1 LETRA U C A G 3 LETRA
FEN SER TIR CIS U
U FEN SER TIR CIS C
LEU SER SS SS A
LEU SER SS TRP G
LEU PRO HIS ARG U
C LEU PRO HIS ARG C
LEU PRO GLU ARG A
LEU PRO GLU ARG G
ILE TRE ASP SER U
A ILE TRE ASP SER C
ILE TRE LIS ARG A
MET TRE LIS ARG G
VAL ALA ac.ASP GLI U
G VAL ALA ac.ASP GLI C
VAL ALA ac.GLU GLI A
VAL ALA ac.GLU GLI G
2 LETRA
CDIGO GENTICO
FEN = fenilalanina ILE = isoleucina
LEU = leucina MET = metionina
SER = serina TRE = treonina
TIR = tirosina ASP = asparagina
ClS = cistena LIS = l isina OBS: SS = sem sentido
TRP = triptofano VAL = valina
PRO = prolina ALA = alanina
HIS = histidina GLI = gl icina
GLU = glutamina ac.ASP = c. asprtico
ARG = argi nina ac.GLU = c. glutmico
SIGLAS AMINOCIDOS

01 - (FUVEST SP/1995) No DNA de um organismo, 18% das bases nitrogenadas so constitudas por citosina. Que
outras bases nitrogenadas devem existir neste DNA e em que propores? Justifique sua resposta.

02 - (EFEI MG/2000) Em Maio de 1999 , um estudo publicado na revista Nature (v.399,p.316) tenta esclarecer se a
famosa ovelha Dolly, clonada a partir de uma clula somtica (no embrionria) de uma ovelha adulta, j nasceu velha
ou no. O resultado sugere que o DNA de Dolly velho, ou seja, que o seu tamanho, que diminui a cada duplicao,
compatvel com o tamanho do DNA de ovelhas de 6 anos. Entretanto, exames clnicos no encontraram nenhum sinal
de envelhecimento precoce, ou seja, a certeza sobre se Dolly realmente nasceu velha s vir com o tempo.
Considere a seguinte seqncia de um molde de DNA.
3 A C T G G A T T G A G C C T A A G 5
a) Escreva a seqncia da fita de DNA resultante da duplicao deste molde.
b) Escreva a seqncia da fita de RNA resultante da transcrio deste molde.
c) Que tipo de RNA formado na transcrio?

03 - (UEPA/1999) Atualmente sabemos que os cidos nuclicos esto diretamente ligados a importantes funes
celulares, como a sntese protica e a hereditariedade, vistos que entram na composio de genes e,
conseqentemente, na de cromossomas. Em relao ao exposto acima, s figuras abaixo e tabela do cdigo
gentico, responda:

arroz
metionina
triptofano


feijo
leucina
fenilanina
treonina
valina
lisina
isoleucina

Clula em anfase
No-disjuno cromossmica




Caderno de Atividades

Disciplina:
B Bi io ol lo og gi ia a

Professor(a):
Rinaldo
Aluno:
3 ano
Ensino Mdio
Data de Recebimento:
_____/_____/_____
Lista 02
Data Entrega:
_____/_____/_____

168
a) Considerando esses dois exemplos de aminocidos existentes no arroz, quais os codons e os anti-codons a eles
relacionados?
b) Considerando esta seqncia de DNA: C A T A A C T G C mencione, em ordem, quais dos aminocidos
contidos no feijo esto a ela (seqncia. relacionados.
c) Quais so os chamados codons SEM SENTIDO e qual o seu significado?
d) Como est dividida a INTERFASE e em qual de seus perodos ocorre a duplicao do DNA?
e) Durante a Mitose, a no-disjuno (figura acima. propicia o aparecimento de aberraes nas clulas filhas. Diga o
nome das aberraes verificadas nas clulas Ae B.

04 - (UFRJ/1992) Observe as duas seqncias do RNA-m abaixo. Somente uma corresponde uma mensagem
capaz de codificar um peptdeo. O codon AUG no RNA mensageiro codifica o incio de uma mensagem e o codon UAA
o final. O sentido da traduo da direita para esquerda.
I. UAA UUA CAG AAG UAA CCG GAC AAU UAA AAU GUA UAA GCU UAA CCG CCG
II. GAC AUG UUU UCG AAC GAU GAC GCA CCA CCC CAU AAU GUG AAU GUG UCG
Qual das duas molculas de RNA codifica o peptdeo? Justifique sua resposta.

05 - (UFRJ/1993) Um pesquisador comparou as estruturas do duas protenas. Cada uma dessas protenas foi obtida
de uma espcie animal diferente.
A pesquisa revelou dois resultados importantes: 1) Na seqncia de aminocidos, havia 66% de homologia entre elas.
2) Na seqncia dos genes precursores das duas protenas estudadas, a homologia observada foi de apenas 45%.
Explique a discrepncia observada entre essas duas percentagens.
06 - (UFRJ/1996) O genoma da bactria E.coli tem um tamanho de 4 x 10
6
pares de nucleotdeos. J o genoma
haplide humano tem 3 x 10
9
pares de nucleotdeos. Para replicar o genoma, antes da diviso celular, existe uma
enzima, a DNA polimerase, cuja velocidade de reao equivalente a cerca de 800 nucleotdeos /s. Assim, para
replicar todo o genoma de uma bactria, a DNA polimerase consumiria cerca de 83 minutos e, para o genoma humano,
aproximadamente 43 dias!
Sabemos, no entanto, que o tempo de gerao da E.coli de cerca de 20 minutos, e que o tempo mdio de replicao
de uma clula eucariota de 12 horas.
Assumindo que a DNA polimerase apresenta uma velocidade de reao constante para todas as espcies analisadas,
explique essa aparente contradio.

07 - (UFRJ/1998) Suponha um gene de um eucarioto responsvel pela sntese de uma protena. Nesse gene existem
ntrons, ou seja, regies do ADN cujas informaes no esto presentes na protena em questo.
As regies do ARN transcrito correspondentes aos ntrons so eliminadas aps o processo de transcrio.
A figura a seguir representa o resultado de uma experincia de hibridao do ARN mensageiro com a cadeia de ADN
que lhe deu origem.

A figura mostra cinco regies, identificadas por nmeros de 1 a 5.
Quais dessas regies correspondem aos ntrons? Justifique sua resposta.

08 - (UERJ/2005) Os conhecimentos atuais de biologia celular, biologia molecular e engenharia gentica podem,
muitas vezes, estabelecer com segurana o parentesco entre pessoas, mesmo quando elas pertencem a geraes
afastadas entre si.
O heredograma abaixo mostra os descendentes do casal Joo e Maria.

Atualmente, de toda essa famlia, apenas Maria e todos os seus
bisnetos esto vivos, e se apresentaram para a identificao de
herdeiros do casal citado. Por no haver documentos legais
comprobatrios da relao de parentesco, nem ser possvel a coleta de
material gentico dos membros falecidos da famlia, foi utilizada, dentre
outras, a tcnica de identificao por meio do estudo do DNA
extranuclear.
Indique o nmero de:
a) bisnetos do sexo masculino e do sexo feminino que poderiam ser identificados com aproximadamente 100% de
certeza, por tcnicas que determinam a homologia entre amostras de DNA extranuclear, e justifique sua resposta;
b) netos e bisnetos de Joo e Maria que possuram ou possuem o cromossomo Y idntico ao de Joo e justifique sua
resposta.

169
09 - (UFRJ/1993) Duas substncias A e B ao reagirem temperatura de 25C geram um produto AB. Essa reao
muito lenta. Quando se acrescenta a substncia X que pode ser um catalisador inorgnico ou ento uma enzima, a
velocidade dessa reao aumenta acentuadamente.
Para se investigar a natureza da substncia X, realizaram-se vrios experimentos para medir a velocidade da
reao (concentrao do composto AB depois de cinco minutos de reao).
Os resultados esto no tabela a seguir:
Experimento
nmero
Temperatura
(C)
Substncia
A B X

Velocidade
da reao
I
II
III
IV
25
25
100
25 (*)
(+) (+) (-)
(+) (+) (+)
(+) (+) (+)
(+) (+) (+)
0,5
85,0
0,6
0,6

(*) no experimento IV a substncia X foi pr-aquecida 100C, depois resfriada 25C e s ento acrescentada ao
tubo contendo as substncias A e B.
(+) indica presena e (-) indica ausncia.
Com base nos resultados da tabela e sabendo-se que as substncias A e B no degradam a 100
0
C indique se a
substncia X um catalisador inorgnico ou uma enzima. Justifique sua resposta.

10 - (UFRJ/1997) A glicoquinase e a hexoquinase so duas enzimas que reagem com o mesmo substrato, a glicose.
Ambas so enzimas intracelulares que fosforilam a glicose formando glicose 6-fosfato (G6P).
Dependendo da enzima produtora, a G6P pode ou ser degradada na via da gliclise para gerar energia ou ento ser
usada para sntese de glicognio.
A gliclise ocorre nos tecidos em geral e a sntese de glicognio ocorre principalmente no fgado. A sntese do
glicognio somente acontece quando existe excesso de glicose no sangue. Essa uma forma de armazenar esse
acar.
Observe a figura a seguir, que apresenta as velocidades de reao dessas duas enzimas em funo da concentrao
da glicose. Nveis normais de glicose no sangue esto ao redor de 4 mM.
Concentrao de Glicose mM
0 2,5 5 10 15
1
3
5
7
9
V
e
l
o
c
i
d
a
d
e

d
e

R
e
a

o
(
u
n
i
d
a
d
e

a
r
b
i
t
r

r
i
a
)
Hexoquinase
Glicoquinase

Qual das duas enzimas gera G6P para sntese de glicognio heptico? Justifique sua resposta.

11 - (UFRJ/1999) O gato siams um animal de rara beleza pois a pelagem de seu corpo clara com extremidades -
orelhas, focinho, ps e cauda - pretas. A presena do pigmento que d a cor negra a essas extremidades o resultado
da atividade de uma enzima que fica inativada acima de 34C.
Explique por que esses animais tm a pelagem negra nas extremidades do corpo.

12 - (UFAL/2005) Considere uma enzima que naturalmente atue temperatura de 36C.
Explique a conseqncia do aumento de temperatura sobre a atividade dessa enzima.

13 - (UFAL/2005) Durante a reproduo sexuada a variabilidade gentica de uma populao promovida pelo
encontro ao acaso dos gametas.
Identifique, no processo de formao dos gametas (na meiose), uma das fases em que ocorre recombinao. Explique
sua resposta.

14 - (UFRJ/1994) O projeto do genoma humano pretende obter a seqncia completa do DNA dos cromossomas. As
clulas somticas humanas possuem 46 cromossomas.
Qual o nmero mnimo de cromossomas que deve ser seqenciado para que se obtenha essa informao? Justifique
sua resposta.

15 - (UFRJ/2000) No incio do projeto do genoma humano, havia duas estratgias a considerar:
I. sequenciar o ADN total dos cromossomos diretamente;
II. extrair todos os ARNs mensageiros, produzir ADN a partir desses ARNs mensageiros e sequenciar apenas esse
ADN.
Nos dois casos, a tcnica de seqenciamento era a mesma.
Por que a segunda estratgia mais rpida e, portanto, mais econmica?

170
16 - (UERJ/2002) Dois cientistas realizaram uma experincia com o objetivo de estudar a transmisso das informaes
contidas nos genes do ncleo de clulas de diferentes tecidos. Ncleos de vulos de r no-fertilizados foram
substitudos por ncleos de clulas somticas, retiradas de uma mesma r. Os cientistas observaram que a grande
maioria destas clulas, com seus novos ncleos, resultaram na formao de embries normais.
Explique por que:
a) esses ncleos transplantados de clulas somticas de diferentes tecidos deram origem a indivduos normais e
idnticos;
b) o resultado da experincia seria diferente se tivessem sido usados ncleos de clulas germinativas.

17 - (UFRJ/1996) O teste de tipagem de DNA revelou que nos seres humanos existe individualidade genmica. Isto
significa que cada indivduo possui variaes discretas e caractersticas na seqncia de seu DNA, ou seja, a
seqncia de nucleotdeos do DNA de cada pessoa nica (excetuando-se o caso de gmeos monozigticos).
Assim, a tipagem do DNA revela um padro de bandas que estvel (presente no DNA de todos os tecidos) e
transmitido aos descendentes seguindo as leis de Mendel.
Graas a essas caractersticas possvel atualmente realizar testes de paternidade que comparam os padres de
bandas de DNA das pessoas e revelam se um homem e de fato o pai biolgico de uma outra pessoa.
Suponha agora a seguinte situao: um homem acusado de ser o pai de uma criana tenta burlar o teste de tipagem
de DNA; um amigo o aconselha a receber uma transfuso de sangue 2 meses antes do teste (em geral colhe-se o
sangue como fonte de clulas nucleadas).
a) Qual a influncia da transfuso sugerida no resultado no exame?
b) Que precaues podem ser tomadas para desmascarar a tentativa de fraude?

18 - (UFG GO/2006) O exame de paternidade atravs da comparao de DNA seqenciado vem sendo utilizado para
determinar progenitores. possvel determinar o pai de um recm-nascido quando a dvida sobre a paternidade desse
recm-nascido est entre gmeos univitelinos? Justifique sua resposta.

19 - (UFRN/2005) O teste de paternidade usando o DNA tornou-se muito freqente hoje. No entanto, as pessoas
tm muitas dvidas a respeito desse tipo de exame.
As frases abaixo constam numa lista de mitos e verdades sobre o teste de DNA encontrada na internet
(http://www.gene.com.br).
I. O exame de DNA s pode ser feito com sangue.
II. Sou primo da me e estou com medo do resultado ser positivo, mesmo que eu no seja o verdadeiro pai.
III. Ele j morreu e no deixou nenhum outro parente vivo. Nunca poderei provar que ele era o pai do meu filho.

Justifique por que cada uma das frases acima constitui um mito.

20 - (UFTM MG/2007) Considere o seguinte drama familiar:
Casal 1: Marta e Joo.
Casal 2: Marina e Guilherme.
Marta e Marina so irms e deram luz no mesmo dia e na mesma maternidade. Nasceram as crianas Marcos e
Pedro. As enfermeiras entregaram Marcos ao casal 1, e entregaram Pedro ao casal 2.
Passados alguns anos, as irms Marta e Marina aventaram a hiptese de as crianas terem sido trocadas no berrio.
Suspeitavam de que, na maternidade, foi entregue a cada uma o sobrinho, e no o filho verdadeiro.
O caso foi levado justia, e o juiz solicitou que fosse feito um exame de DNA para se determinarem os verdadeiros
pais das crianas.
O teste foi feito, e o resultado est esquematizado na figura, em que cada indivduo apresenta um padro com duas
bandas (faixas) de DNA, cada uma das bandas representando um alelo do mesmo gene.


a) Se o padro de bandas de DNA de Marcos no fosse conhecido, teria sido possvel determinar de qual casal
Pedro filho? Justifique.
b) Considerando o padro de bandas de DNA de todos os envolvidos, houve troca de crianas na maternidade?
Justifique.



171
21 - (UFRJ/1994) O camundongo gigante da figura cresceu partir de um zigoto no qual foi microinjetado o gene do
hormnio do crescimento do rato. Esse camundongo chamado de transgnico.

Se ao invs de injetar o zigoto com o DNA, fosse injetado o RNA-m do gene do
hormnio do crescimento do rato, o resultado seria o mesmo? Justifique sua resposta.






22 - (UFRJ/2002) Um estudo recente realizado com espermatozides de camundongos normais descreveu o gene
que codifica uma protena (CatSper), que forma na membrana da cauda do gameta um canal de Ca2+ (esses canais
permitem a entrada rpida de clcio atravs da membrana).
Usando tcnicas de engenharia gentica, os cientistas conseguiram produzir camundongos transgnicos cujos
espermatozides no expressavam a protena CatSper. As figuras mostram os resultados de alguns experimentos
realizados com os espermatozides de camundongos normais e com os de camundongos transgnicos.
Contagemdeespermatozides
Normal Transgnico
N

m
e
r
o

(
m
i
l
h

e
s
)
2,5
2,0
1,5
1
0,5
0

Velocidadedos espermatozides
Normal Transgnico
V
e
l
o
c
i
d
a
d
e

e
m

(
m
/
s
)

100
80
60
40
20
0


Observe as figuras e responda.
De que modo um composto que inibisse especificamente a ao da protena CatSper estaria agindo como um
dispositivo anticoncepcional?

TEXTO: 1 - Comum questo: 23

Imagine-se vivendo a seguinte situao:
Voc, sua famlia e um grupo de amigos, esto reunidos numa ensolarada manh de vero, desfrutando do prazer
de sentar-se frente ao mar, sentindo nos ps a areia fina da praia do Atalaia, em Salinas, refrescando-se com a brisa
marinha e deliciando-se com gua de coco gelada (ou sua bebida favorita), acompanhada de uma boa dose de
camares fritos, ou de saborosas ostras regionais, temperadas apenas com sal e suco de limo. Todos cuidaram de
proteger a pele, adequadamente, das radiaes solares.
Nesse clima de descontrao total, sempre bem-vinda a prtica de um frescobol, ou de uma boa caminhada (quem
sabe at uma corrida!) ao longo da praia, para transpirar e exercitar um pouco a musculatura, j que o corre-corre
dirio nem sempre deixa tempo livre para uma atividade fsica regular.
Com o avanar das horas, o grupo decide desfrutar o mximo deste cenrio de muito sol, da gua esverdeada do
mar, da viso deslumbrante das verdejantes dunas de areia, e fica para o almoo. Delicia-se de um variado cardpio
composto de peixe-frito e batatas-fritas, farofa e molho, mexilho refogado, acompanhado de salada de alface e
tomates e, como no poderia deixar de ser, o indispensvel caranguejo ao toque-toque, sempre atentos com a
ingesto de lquidos, para prevenir uma indesejvel desidratao.
No final da tarde, antes de retornar, nada melhor que um bom banho no Lago da coca-cola, para retirar o sal deixado
na pele pelo mar. Finalmente em casa, todos desfrutam de um merecido descanso para a reposio de energia, pois a
noite se aproxima e ningum pode dispensar o agradvel passeio pela maravilhosa orla do Maarico e ser feliz !

23 - (UFPA/2001) Ao servir o almoo e querendo agradar o grupo, o jovem barraqueiro disps-se a elogiar a
qualidade dos alimentos servidos e, numa linguagem muito prpria, afirmou que a salada era especial, feita com folhas
que cresceram na gua e os tomates eram daqueles feitos no laboratrio, sendo mais saborosos porque demoravam
mais tempo amadurecendo no p.
Ao compreender que o rapaz falava de alface hidropnica e de tomates geneticamente modificados (ou transgnicos),
estabeleceu-se um clima de incerteza quanto ingesto ou no da salada.
Considerando que, atualmente, muitas espcies vegetais vm sendo aperfeioadas pela Engenharia Gentica atravs
da manipulao do DNA, como o caso do tomate referido pelo barraqueiro, indique as respectivas funes das
enzimas de restrio e dos plasmdeos, no processo de construo de uma molcula de DNA recombinante.




172











PROF.:RINALDO JOS DE RESENDE







C
C
o
o
n
n
t
t
e
e

d
d
o
o
s
s
:
:

E
E
V
V
O
O
L
L
U
U

O
O
I
I
























D Da at ta a d da a E En nt tr re eg ga a : : _ __ __ __ __ __ __ __ _/ /_ __ __ __ __ __ __ __ __ __ __ _/ /2 20 01 13 3
LISTA 3 BIOLOGIA

173

01 - (FUVEST SP/1995) Uma populao de bactrias foi colocada em um meio de cultura saturado de um
determinado antibitico. A maioria das bactrias morreu. No entanto, algumas sobreviveram e deram origem a
linhagens resistentes a este antibitico.
a) Explique o processo segundo a teoria lamarquista de evoluo.
b) Explique o processo segundo a teoria darvinista de evoluo.

02 - (FUVEST SP/1999) O desenvolvimento da Gentica, a partir da redescoberta das leis de Mendel, em 1900,
permitiu a reinterpretao da teoria da evoluo de Darwin. Assim, na dcada de 1940, formulou-se a teoria sinttica
da evoluo. Interprete o diagrama a seguir, de acordo com essa teoria.



a) Que fator evolutivo est representado pela letra A?
b) Que mecanismos produzem recombinao gnica?
c) Que fator evolutivo est representado pela letra B?

03 - (UFPA/1998) O uso indiscriminado de antibiticos favorece a preservao de linhagens bacterianas resistentes
a esses medicamentos. Como exemplo, temos o caso do Vibrio cholerae, agente causador da clera, do qual j so
conhecidas linhagens resistentes a, pelo menos, cinco antibiticos.
(Adaptao: Cssio Leite Vieira - Folha de So Paulo, 13/12/92)

Com base na teoria moderna da evoluo, explique a resistncia dessas bactrias aos antibiticos.

04 - (UFRJ/1992) Duas espcies de pererecas A e B, foram encontradas em trs ilhas. Na ilha I s existia a espcie
A, na ilha II s existia a espcie B e na ilha III existiam as duas espcies. Nas pererecas, o acasalamento precedido
por um canto nupcial. Os cantos nupciais das duas espcies foram gravados nas trs ilhas e esto representados
no grfico abaixo:

Ilha Espcie canto
I
II
III
A
B
A
B


Com base na teoria da evoluo. explique a mudana ocorrida no canto nupcial da espcie B na ilha II.

05 - (UFRJ/1993) Em determinada regio da Birmnia, so colocados 5 anis de metal no pescoo das meninas
quando elas atingem a idade de 5 anos. Com o passar dos anos, novos anis so a elas acrescentados. Ao chegarem
idade adulta essas mulheres apresentam um pescoo que possui o dobro do comprimento normal.
Essa tradio acabar levando ao nascimento de indivduos de pescoo mais longo nas prximos geraes? Justifique
sua resposta.

Caderno de Atividades

Disciplina:
B Bi io ol lo og gi ia a

Professor(a):
Rinaldo
Aluno:
3 ano
Ensino Mdio
Data de Recebimento:
_____/_____/_____
Lista 03
Data Entrega:
_____/_____/_____

174
06 - (UFRJ/1992) O metilmercrio um poluente que surge da organificao do mercrio que jogado nos rios, em
garimpos de ouro. O homem pode se intoxicar ao ingerir um peixe contaminado por esse poluente. Isso pode acarretar
leses no seu sistema nervoso. Caso surgisse um gene mutante na populao humana, capaz de formar uma enzima
eficiente na metabolizaco e conseqente eliminaco do metilmercrio (como ocorre em certas bactrias), os
indivduos portadores dessa mutao estariam protegidos da intoxicao por esse poluente.
Explique como a seleo natural poderia agir sobre a freqncia desse gene mutante nas seguintes situaes:
a) populao humana, portadora do mutante, vivendo em ambiente no poludo por mercrio.
b) populao humana, portadora do mutante, vivendo em ambiente poludo por mercrio.

07 - (UFRJ/1993) A mariposa Pieris brassicae em condies de laboratrio, temperatura constante de 25C produz,
geraes no perodo de um ano.
Populaes naturais dessa mariposa so encontradas em vrias localidades, como mostra o tabela a seguir:

Localidade
CEI
Polnia
Tchecoslovquia
Palestina
Latitude
60 N
53 N
48 N
32 N
Nmero de geraes
observadas em 1 ano
1
2
3
7


Com base nos dados da tabela explique a variao do nmero de geraes da mariposa nas diferentes localidades.

08 - (UFRJ/1993) Nas moscas do gnero Drosophila, a fmea identifica o macho de sua espcie mediante a
percepo do som produzido pela velocidade de batimento das asas. Cada espcie tem um som diferente e, assim,
fmeas de populaes naturais, na poca da reproduo, dificilmente se enganam na identificao dos machos de sua
espcie; contudo, quando essas moscas so criadas em laborotrio, em caixas contendo muitos indivduos, algumas
podem copular com machos de outra espcie.
Um pesquisador colocou, em uma caixa apropriada, um certo nmero de machos e fmeas da espcie A e igual
nmero de machos e fmeas da espcie B. A cada gerao, as moscas hbridas eram retiradas da caixa antes que
pudessem se reproduzir, deixando-se apenas as moscas das duas espcies. O grfico a seguir mostra a variao da
freqncia das moscas da espcie A, da espcie B e dos hbridos A/B em 32 geraes:

20
40
60
80
100
1 4 12 20 32
Espcie B
Hbridos
A/B
Espcie A
%

F
r
e
q

n
c
i
a

d
e

i
n
d
i
v

d
u
o
s
Nmero de geraes


a) Qual das espcies est melhor adaptada s condies de reproduo em laboratrio? Justifique suo resposta.
b) Qual o agente evolutivo que melhor explica a reduo do nmero de hbridos ao longo das 32 geraes? Justifique
sua resposta.

09 - (UFRJ/1995) Leia com ateno as seguintes informaes:
Informao I: O nmero de espcies de insetos que comem plantas na regio tropical , aproximadamente, trs vezes
maior que o de espcies que comem plantas na regio temperada.
Informao II: As plantas produzem substncias, como os alcalides, que so txicas para muitas espcies de insetos
que se alimentam de plantas.

Um estudo mostrou que 35% das espcies de plantas da regio tropical produzem alcalides, enquanto apenas 15%
das espcies de plantas da zona temperada produzem essas substncias.
Explique o mecanismo evolutivo que, possivelmente, gerou essa diferena percentual entre as plantas das duas
regies.

175
10 - (UFRJ/1999) Os machos de uma certa espcie de pssaros so territoriais, ou seja, so animais que delimitam e
defendem a regio em que se instalam. Os mais fortes escolhem e ocupam os melhores territrios, dos quais expulsam
qualquer outro macho que tente se aproximar. Na poca do acasalamento, as fmeas passeiam por todos os
territrios e decidem com que macho vo procriar.
O grfico a seguir mostra a ordem em que 10 machos dessa
espcie foram escolhidos.

O eixo das ordenadas indica a seqncia em que os machos foram
escolhidos e o eixo das abscissas indica a qualidade dos territrios.
a) O que determina a escolha preferencial dos machos pelas
fmeas?
b) Qual o mecanismo evolutivo que explica esse padro?





11 - (UFLA MG/2006) Em um arquiplago ocenico, todas as ilhas so habitadas por aves de um mesmo gnero.
Cada ilha possui uma nica espcie desse gnero e as diferenas morfolgicas entre elas so o tamanho e o formato
do bico.
a) Qual ter sido a primeira etapa desse processo de especiao?
b) Que presso seletiva pode ter determinado a presena de aves com bicos diferentes em diferentes ilhas?
c) Qual seria o procedimento para confirmar que as aves encontradas nas diferentes ilhas so de fato pertencentes a
espcies diferentes?

12 - (EFOA MG/2000) O processo de especiao dos seres vivos acompanhado, ao longo do tempo, por
modificaes das freqncias gnicas de suas populaes.
a) Basicamente, como a seleo natural interfere nessas modificaes?
b) Alm dos mecanismos da seleo natural, cite dois exemplos, reconhecidos evolutivamente, de fatores que
interferem nessas modificaes.

13 - (FUVEST SP/2005) Devido ao aparecimento de uma barreira geogrfica, duas populaes de uma mesma
espcie ficaram isoladas por milhares de anos, tornando-se morfologicamente distintas.
a) Explique sucintamente como as duas populaes podem ter-se tornado morfologicamente distintas no decorrer do
tempo.
b) No caso de as duas populaes voltarem a entrar em contato, pelo desaparecimento da barreira geogrfica, o que
indicaria que houve especiao?

14 - (UFBA/2006) Os peixes recifais marinhos formam um dos mais variados grupos de vertebrados existentes, e suas
comunidades tm sido estudadas em diferentes regies do mundo.
Os ambientes recifais, presentes [...] em geral em guas litorneas, so formados por estruturas slidas ou
consolidadas, que podem ter origem biognica ou no. Os de origem biognica so formados pela deposio de
material calcreo, por alguns tipos de corais e outros organismos construtores. J os de origem inorgnica podem
surgir devido cimentao natural de gros arenosos ou pela formao de costes rochosos [...]. Ambientes mais
complexos tm maior quantidade de micro-hbitats, seja pela presena de mais ramos e reentrncias de corais, ou
pela existncia de variados tamanhos de tocas formadas por pedras e/ou outros substratos consolidados. Esses
ambientes so o hbitat das mais diversas e complexas comunidades de peixes, incluindo representantes de pelo
menos 100 famlias desses animais. Somadas, as espcies dessas comunidades representam cerca de 20% de todos
os peixes atuais.
(RANGEL e outros, 2005, p. 66).
A partir da anlise das informaes do texto, apresente um argumento que possa justificar a grande diversidade de
espcies de peixes nos ambientes recifais.

15 - (UFMG/2006) A fragmentao de um ecossistema uma das principais causas da extino de espcies.
Analise esta figura:

Alm da extino de espcies, a fragmentao de
ecossistemas implica outras conseqncias igualmente
danosas natureza.






176
1. CITE duas dessas conseqncias.
Conseqncia 1:
Conseqncia 2:

2. CITE o nmero do fragmento . I, II ou III . em que a espcie remanescente apresenta maior risco de extino.
JUSTIFIQUE sua resposta.
Fragmento:
Justificativa:

3. Analise este fragmento:



a) CITE o fator evolutivo que atuou nesse fragmento.
b) EXPLIQUE como esse fator evolutivo atua.

4. Observe as espcies representadas nessas figuras:



Alm da fragmentao de ecossistemas, outras causas vm contribuindo para a extino dessas espcies.
CITE duas dessas causas.
Causa 1:
Causa 2:

16 - (UFRJ/1994) A subtilisina e a quimotripsina so enzimas que digerem protenas com estruturas primria e terciria
diferentes mas com stios ativos essencialmente idnticos.
J a lactalbumina-alfa e a lisozima so protenas que possuem estruturas primrias muito semelhantes entre si mas
que desempenham funes diversas, ou seja, possuem stios ativos diferentes.
Qual desses dois pares de protena sofreu evoluo compatvel com o modelo da convergncia evolutiva? Justifique
sua resposta.

17 - (EFOA MG/2006) A teoria moderna da evoluo considera a mutao gnica, a recombinao gnica e a seleo
natural como os principais fatores evolutivos. Sobre estes fatores, responda:
a) Que fator ocorre exclusivamente em organismos sexuados?
b) Que fator leva substituio de uma base nitrogenada por outra na molcula de DNA?
c) Qual o principal fator atuante na eliminao de indivduos menos adaptados a um ambiente particular?
d) Qual a fonte primria de variabilidade gentica?
e) O que recombinao gnica?




177

18 - (UFG GO/1994) A histria evolutiva da vida, na Terra, pode ser catalogada em diferentes perodos e eras. O
jurssico correspondente ao perodo em que predominaram os grandes rpteis, como os dinossauros, e florestas de
gimnospermas.
O filme Parque dos Dinossauros recria esses grandes rpteis e os colocam face a face com o homem.
Considerando essa biocenose e os seus descendentes, relacione:
a) dinossauros X homens;
b) irradiao adaptativa X rgos homlogos;
c) convergncia adaptativa X rgos anlogos.

19 - (UFRJ/2006) Os tigres de dentes-de-sabre so mamferos extintos. Esses animais possuam caninos superiores
muito desenvolvidos, em forma de sabre. Um fato menos conhecido que houve vrias espcies de mamferos
placentrios com dentes-de-sabre.
O diagrama a seguir mostra a filogenia provvel dos tigres de dentes-de-sabre placentrios Barbourofelis e Smilodon.


* apenas os retngulos sombreados representam tigres de dentes-de-sabre.

A presena da caracterstica dentes-de-sabre em Barbourofelis e Smilodon representa um caso de homologia ou de
analogia? Justifique sua resposta.

20 - (UFRRJ/1999) Embora sejam popularmente chamados de ursos, na realidade o urso castanho de origem
europia, Ursus arctos; o urso preto americano, Euarctos americanus; e o urso polar branco, Thalarctos maritimus, so
animais distintos.
a) Se fosse possvel o encontro do urso castanho com o urso polar, um suposto acasalamento resultaria em
reproduo? Justifique.
b) Explique por que ocorreu a diferenciao entre esses animais?

21 - (UNIRIO RJ/2005) A tabela seguinte indica a viabilidade de prole frtil resultante do cruzamento entre indivduos
de trs populaes diferentes ( I, II e III ) que vivem em regies prximas dentro de um bosque.



Quantas espcies existem no momento considerado e quantas passariam a existir, se a populao II fosse extinta?
Justifique sua resposta.

22 - (UFF RJ/2005) Diferentes espcies de peixes herbvoros marinhos do mesmo gnero so encontradas nas
regies tropicais do Oceano Atlntico, tanto na costa do Continente Americano, quanto na costa do Continente
Africano. Aps estudos sobre este grupo, foi possvel elaborar o diagrama e o quadro abaixo, onde espcies distintas
foram representadas por diferentes letras.

* Os nmeros da coluna Alimento representam:
1. algas com lminas flutuantes
2. algas incrustadas nos corais
a) Considerando os mecanismos de especiao, como poderia ser explicado o surgimento das espcies C e D a
partir de uma espcie ancestral?
b) Das espcies citadas, qual delas mais se assemelha espcie ancestral?
c) Que tipo de relao/interao ecolgica pode ocorrer entre D e E? Justifique sua resposta.


178
TEXTO: 1 - Comum questo: 23
Exrcito Cururu

Importado para exterminar besouros que atacavam canaviais, sapo brasileiro vira praga na Austrlia.
Sapos, milhes de sapos asquerosos e venenosos, em saltos pelo sol at onde a vista alcana. No se trata de uma
das famosas sete pragas do Egito. A invaso real e acontece na costa leste australiana. Esse exrcito coaxante
formado por centenas de milhes de sapos amaznicos da espcie Bufo marinus. ou, para o brasileiro leigo, o
folclrico sapo-cururu. O bicho foi introduzido na Austrlia em 1935 para o controle biolgico de um besouro que
atacava os canaviais, estratgia utilizada com sucesso nos Estados Unidos e na Amrica Central. Mas na terra dos
cangurus o cururu no funcionou. Pior: transformou-se rapidamente em praga.
Peter Moon.
(ISTO , n
o
1302, 14.09.94, p.50)
23 - (UFG GO/1997) Reconhecendo a vida como resultado de um longo processo no tempo,
a) considere os cangurus e justifique, exemplificando, a afirmativa: a ontognese recapitula a filognese;
b) enumere e explique trs caractersticas que comprovem que os besouros so mais evoludos que os sapos ou que
os sapos so mais evoludos que os besouros com relao ao esqueleto, musculatura e circulao.

Q QU UE ES ST T E ES S D DO O E EN NE EM M

01 - (ENEM Simulado/2009) A maior parte dos mamferos especialmente os grandes no pode viver sem gua
doce. Para os mamferos marinhos, gua doce ainda mais difcil de ser obtida. Focas e lees-marinhos captam gua
dos peixes que consomem e alguns comem neve para obt-la. Os peixes-boi procuram regularmente gua doce nos
rios. As baleias e outros cetceos obtm gua de seu alimento e de goladas de gua do mar. Para tanto, os cetceos
desenvolveram um sistema capaz de lidar com o excesso de sal associado ingesto de gua marinha.
WONG, K. Os mamferos que conquistaram os oceanos. In: Scientific American Brasil.
Edio Especial n5: Dinossauros e Outros Monstros. (adaptados).

A grande quantidade de sal na gua do mar

a) torna impossvel a vida de animais vertebrados nos oceanos.
b) faz com que a diversidade biolgica no ambiente marinho seja muito reduzida.
c) faz com que apenas os mamferos adaptados ingesto direta de gua salgada possam viver nos oceanos.
d) faz com que seja inapropriado seu consumo direto como fonte de gua doce por mamferos marinhos, por exemplo,
as baleias.
e) exige de mamferos que habitam o ambiente marinho adaptaes fisiolgicas, morfolgicas ou comportamentais que
lhes permitam obter gua doce.

02 - (ENEM Simulado/2009) No Perodo Permiano, cerca de 250 milhes de anos atrs (250 m.a.a.), os continentes
formavam uma nica massa de terra conhecida como Pangia. O lento e contnuo movimento das placas tectnicas
resultou na separao das placas, de maneira que j no inicio do Perodo Tercirio (cerca de 60 m.a.a.), diversos
continentes se encontravam separados uns dos outros. Uma das consequncias dessa separao foi a formao de
diferentes regies biogeogrficas, chamadas biomas. Devido ao isolamento reprodutivo, as espcies em cada bioma
se diferenciam por processos evolutivos distintos, novas espcies surgiram, outras se extinguiram, resultando na atual
diversidade biolgica do nosso planeta. A figura ilustra a deriva dos continentes e as suas posies durante um perodo
de 250 milhes de anos.

RICKLEFS, R.E. A economia da natureza. Rio de Janeiro, Guanabara Koogan, 2003. (adaptado)
De acordo com o texto, a atual diversidade biolgica do planeta resultado
a) da similaridade biolgica dos biomas de diferentes continentes.
b) do cruzamento entre espcies de continentes que foram separados.
c) do isolamento reprodutivo das espcies resultantes da separao dos continentes.
d) da interao entre indivduos de uma mesma espcie antes da separao dos continentes.
e) da taxa de extines ter sido maior que a de especiaes nos ltimos 250 milhes de anos.

179
03 - (UnB DF/2011) Evoluo no ocorre ao acaso. A aleatoriedade da evoluo no poderia proporcionar a
variabilidade da vida, ou mesmo a evoluo das espcies. Darwin explica que a seleo natural determina quem viver
o tempo suficiente para se reproduzir e perpetuar a espcie, o que proporciona a evoluo dessa espcie. Entretanto,
se h seleo, no pode haver apenas aleatoriedade. O importante ficar clara a diferena entre sorteio e seleo. No
sorteio, nenhuma caracterstica em si levada em considerao nas escolhas, tudo ao acaso, aleatrio. Em uma
seleo, pelo menos uma caracterstica utilizada para serem separados ou escolhidos alguns membros dentro de um
grupo.
O acaso desempenha um papel importante na evoluo. Entretanto, uma gama de caractersticas satisfaz s
exigncias da seleo natural, ou seja, variao gentica hereditria com fitness diferenciado. Assim, jamais se deve
interpretar a evoluo como um processo aleatrio. A seleo natural molda as populaes de modo que aquele com
maior sucesso reprodutivo passe seus genes a uma quantidade maior de descendentes. Ela opera sobre algo
preexistente e a nica explicao conhecida para as adaptaes verificadas na natureza.
Internet: <http://biociencia.org> (com adaptaes).
Com relao ao assunto abordado no texto, julgue os itens que se seguem.

( ) Infere-se do texto que o acaso foi o fator mais importante para a produo do conjunto relativamente restrito das
sequncias de aminocidos que existem nos organismos de hoje.
( ) A teoria da evoluo por seleo natural envolve o acaso e a necessidade. O acaso est presente no processo de
reproduo diferencial dos indivduos mais bem adaptados ao ambiente.
( ) A evoluo biolgica considerada processo plenamente aleatrio devido imprevisibilidade dos seus resultados
adaptativos.
( ) Entende-se como sucesso reprodutivo o quanto um indivduo contribui com prole para a gerao seguinte.
( ) Se determinado inseto adquiriu resistncia a um pesticida foi porque o pesticida causou alguma alterao no inseto
a qual o tornou resistente, o que comprova que houve evoluo.



















180






PROF.:RINALDO JOS DE RESENDE








C
C
o
o
n
n
t
t
e
e

d
d
o
o
s
s
:
:

E
E
C
C
O
O
L
L
O
O
G
G
I
I
A
A























D Da at ta a d da a E En nt tr re eg ga a : : _ __ __ __ __ __ __ __ _/ /_ __ __ __ __ __ __ __ __ __ __ _/ /2 20 01 13 3

LISTA 4 BIOLOGIA

181

01 - (UFRJ/1999) A formao de uma caracterstica fenotpica depende, em alguns casos, apenas de fatores
genticos. Em outros casos, prevalece a influncia de fatores ambientais. Na maioria das vezes h uma interao entre
fatores genticos e ambientais. Um dos mtodos utilizados para avaliar a importncia relativa dos genes e dos fatores
ambientais na formao de urna caracterstica o estudo comparativo entre irmos gmeos monozigticos criados
juntos e criados separados.
A tabela a seguir, elaborada a partir de um grande nmero de pares de gmeos, indica o grau de concordncia de
quatro caractersticas. Uma concordncia significa que quando um irmo possui a caracterstica, o outro tambm a
possui.

Indique a caracterstica que mais depende de fatores ambientais. Justifique sua resposta.

02 - (UFPR/2005) Uma floresta tropical e uma monocultura de soja so dois ecossistemas bastante distintos.
e.1) Em qual deles as redes alimentares so mais complexas? Justifique.
e.2) Qual deles mais suscetvel a pragas? Justifique.

03 - (UFC CE/1997) Compare os ecossistemas naturais com as monoculturas quanto ao nvel de biodiversidade e
susceptibilidade s pragas.

04 - (UFRJ/1993) No grfico a seguir, esto representadas as variaes de trs parmetros de um ecossistema
formado por plantas e animais que habitam por cem dias um pequeno charco. Esses parmetros so:

- Produo primria (formao de matria orgnica por fotossntese):
- taxa de respirao (energia gasta para manter vivos os animais e os vegetais):
- biomassa (quantidade de matria orgnica viva dos animais e vegetais do ecossistema).

Produo primria (fotossntese)
Taxa de respirao
Biomassa
0 20 40 60 80 100
Dias

Pela observao do grfico, verifica-se que, a partir do 70 dias, a biomassa pra de aumentar.
Explique esse fato.

05 - (UEM PR/2006) Ao estudar a vida, podem-se distinguir diversos nveis hierrquicos de organizao biolgica, que
vo do submicroscpico ao planetrio. Comeando no plano submicroscpico, observa-se que a matria viva
constituda de tomos que se renem quimicamente, formando as molculas das diversas substncias orgnicas.
Seguindo esses nveis hierrquicos, conceitue
a) tecidos;
b) rgos;
c) populao biolgica.

06 - (FUVEST SP/1995) Numa comunidade, organismos X realizam reaes que liberam nitrognio atmosfrico (N
2
);
organismos Y digerem quitina; organismos Z realizam reaes que liberam oxignio gasoso (O
2
) e os organismos W
no contm pigmentos fotoativos e produzem amilase.
a) Qual o papel desempenhado pelos organismos X, Y, Z e W nas cadeias alimentares das quais participam?
b) Considerando que outros seres vivos sejam introduzidos nessa comunidade, que alimentos (X, Y, Z ou W) lhes
fornecero maior quantidade de biomassa?
Caderno de Atividades

Disciplina:
B Bi io ol lo og gi ia a

Professor(a):
Rinaldo
Aluno:
3 ano
Ensino Mdio
Data de Recebimento:
_____/_____/_____
Lista 03
Data Entrega:
_____/_____/_____

182
07 - (UEPA/1999) Sobre a cadeia alimentar abaixo responda:

a) Qual(is) o(s) ser(es) visto(s) nesta figura realiza(m) o HERBIVORISMO, e qua(is) realiza(m) o SAPROFITISMO?
b) O que significa dizer: enquanto o produtor fotfilo o consumidor primrio fotfobo?
c) Quais dos seres focalizados so capazes de captar energia luminosa e convert-la em energia qumica? Que
organela contida em suas clulas permite realizar tal fenmeno?
d) Se compararmos esta cadeia alimentar terrestre com as cadeias aquticas, que seres contidos na figura ocupariam o
mesmo nvel trfico do fitoplancton e do zooplancton?
e) D um exemplo de uma ocorrncia ou fenmeno que poderia acontecer, caso houvesse uma caa indiscriminada s
cobras, reduzindo grandemente a populao desses rpteis.

08 - (UFRJ/1997) Em uma lagoa costeira, foram medidas as concentraes na gua de algumas substncias txicas.
Os peixes dessa lagoa tambm foram analisados para as mesmas substncias.
Os resultados dessas medies esto apresentados na tabela abaixo:
Substncia Presenada Fator de
Txica Substncia(mg/g) Bioconcentrao

gua Peixe
Lindane 0,0003 0,3 1.000
Tricloro-
benzeno
PCB-153 0,00003 240 8.000.000
Mirex 0,000008 110 13.750.000
0,0005 0,6 1.200

*Fator de Bioconcentrao = concentrao da substncia txica no peixe/ concentrao da substncia txica na gua.
Os peixes analisados so todos carnvoros.
Por que a concentrao das substncias txicas nos peixes maior do que a observada nas guas da lagoa?

09 - (UFRJ/1992) Duas ilhas tm o mesmo potencial de produo agrcola . Uma das ilhas tem uma populao
humana de hbito alimentar essencialmente vegetariano e na outra h uma populao humana de hbito alimentar
essencialmente carnvoro.
Considerando o fluxo de energia e matria num ecossistema, explique em que ilha a populao humana dever ser
maior.

10 - (UFRJ/1998) As figuras A e B representam esquematicamente a entrada de energia em dois ecossistemas.
A energia que entra igual nos dois ecossistemas e se divide de forma desigual; nas figuras,a espessura das setas
proporcional quantidade de energia.
Um dos ecossistemas um ecossistema agrcola, formado por milho e arroz, o outro uma floresta tropical primria.




Identifique o ecossistema agrcola. Justifique sua resposta.

11 - (UNICAMP SP/1999) A produtividade primria em um ecossistema pode ser avaliada de vrias formas. Nos
oceanos, um dos mtodos para medir a produtividade primria utiliza garrafas transparentes e garrafas escuras,
totalmente preenchidas com gua do mar, fechadas e mantidas em ambiente iluminado. Aps um tempo de incubao,
mede-se o volume de oxignio dissolvido na gua das garrafas. Os valores obtidos so relacionados fotossntese e
respirao.
a) Por que o volume de oxignio utilizado na avaliao da produtividade primria?
b) Explique por que necessrio realizar testes com os dois tipos de garrafas.
c) Quais so os organismos presentes na gua do mar responsveis pela produtividade primria?


183
12 - (UNIFESP SP/2005) O grfico diz respeito composio etria da populao brasileira em 1991, segundo dados
do IBGE.



Entre os vrios fatores que levaram a essa configurao, um deles a diminuio na taxa de mortalidade infantil,
devido a campanhas de vacinao em massa, que tm imunizado um nmero cada vez maior de crianas.

Responda.
a) Cite trs doenas de transmisso viral que afetam com freqncia as crianas, para as quais existem vacinas no
sistema pblico de sade brasileiro e que fazem parte do calendrio oficial de vacinas.
b) Analise a configurao do grfico e, alm da mortalidade infantil, cite uma caracterstica da pirmide que permite que
a associemos a um pas com certo grau de desenvolvimento.
Cite tambm uma caracterstica que faz com que a associemos a um pas subdesenvolvido. Justifique.

13 - (UERJ/1993) VESPA EVITA A PROLIFERAO DE PRAGAS EM PASTAGENS

Uma vespa de apenas 0,5 milmetros pode ajudar o pecuarista a combater uma praga que costuma arrasar grandes
reas de pastagens. Um estudo desenvolvido pela UFMG mostrou que cinco cepas de vespa da famlia
"Trichogrammatidae so capazes de controlar a proliferao da mariposa curuquer-dos-capinzais, cujas larvas se
alimentam de folhagens.
A vespa inibe a ploriferao da praga ao depositar os seus ovos dentro dos ovos da mariposa. Dessa forma, ao invs
de nascerem larvas que mais tarde se transformariam em mariposas, nascem vespas. Atualmente, os produtores rurais
costumam combater as larvas das mariposas com inseticidas.
(Trechos de texto de O Globo de 05/10/1992.)

O texto acima mostra como o controle biolgico pode substituir os inseticidas no combate s pragas. Vrias so as
causas biolgicas que fazem com que os inseticidas no-biodegradveis, mesmo quando lanados em pequenas
quantidades no ambiente, se acumulem gradativamente ao longo dos nveis trficos da cadeia alimentar.
Cite duas causas que concorrem para essa acumulao.

14 - (UERJ/1996)
Com certeza
ido tolo MARRECO
que descobriu o ECO.

Foi o MACACO.
que, ao passar pela caverna,
chamou o amigo:

MARRECO!
e, para surpresa geral,
a caverna respondeu:
ECO ... ECO

O MA CACO gostou
e ficava horas
conversando com a caverna:
O homem diz que ama a NATUREZA,

184
como antigamente...
MENTE... MEMTE... responde a caverna.

At que num dia de sol
(depois de uma noite sem estrelas)
apareceu um homem
de fundos olhos cor de cinza
e queimou a mata,
matou o MACACO,
comeu o MARRECO,
quebrou a caverna..
(e calou o ECO, lgico!)

A poesia de Jos de Nicola ilustra bastante bem o carter destruidor que a ao humana assume em alguns momentos
frente ao ambiente. Um exemplo tpico de desastres ecolgicos provocados pelo homem decorre da utilizao de
inseticidas clorados, como o DDT.
A partir dessas informaes, explique:
a) de que maneira a existncia das cadeias alimentares determina a presena, de poluentes, como o DDT, em animais
que no entram em contato direto com essas substncias.
b) por que existe maior concentrao de DDT, por quilo de organismo, nos indivduos de nveis trficos superiores,
como o gavio, do que nos representantes de nveis trficos inferiores, como os insetos.

15 - (UFBA/2006)


[...]Depois de sculos de crescimento mais que exponencial, a populao do [planeta] est se estabilizando. A julgar
pelas tendncias atuais, ela vai parar em cerca de 9 bilhes de pessoas em meados do sculo. Enquanto isso, a
pobreza extrema est diminuindo, tanto como percentagem da populao como em nmeros absolutos.
[...] Esse crescimento da humanidade em tamanho e riqueza, porm, pressiona os limites do planeta. Ns emitimos
dixido de carbono trs vezes mais rpido do que os oceanos e a terra conseguem absorver. No meio do sculo,
segundo estudos, o aquecimento global comear realmente a trazer grandes problemas. Na velocidade em que as
coisas vo, as florestas e os estoques de peixes vo acabar antes que isso.
(MUSSER, 2005, p. 36).

Considerando o grfico e as informaes do texto, explique a situao representada no perodo entre 1820 e 1960,
aproximadamente, com base em processos fisiolgicos associados ao ciclo biogeoqumico do carbono.

16 - (UERJ/1998) A proviso de alimentos dos animais e vegetais limitada pela disponibilidade de nitrognio fixado.
Os grficos abaixo so o resultado de uma pesquisa que analisou a relao entre o processo de fixao de nitrognio e
o de desnitrificao.
I-FIXAO
TERRESTRE
(HISTRICA)
COLHEITAS
DE LEGUMES
MARINHA
FIXAO
INDUSTRIAL
FIXAO
ATMOSFRICA
TOTAL DE
GANHOS
30
14
10
30
8
92
0 20 40 60 80 100


185
IIDESNITRIFICAO
0 20 40 60 80 100
43
40
83
TRANSFERNCIA, GANHOSEPERDASDEN(10t)
6
TERRESTRE
MARINHA
TOTALDE
PERDAS

(Adaptado de A Biosfera, textos do Scientific, American. So Paulo, Polgono, Ed. da Universidade de So Paulo, 1974, p.
87.)

A anlise dos dados mostra que o total de ganhos na fixao maior que o total de perdas, permitindo um saldo de
cerca de 9 milhes de toneladas de nitrognio fixado.
a) Explique por que a existncia deste saldo biologicamente indispensvel para contribuir com a produo de
alimentos.
b) Cite duas justificativas para que as recomendaes tcnicas relativas ao aumento da produo mundial de
alimentos enfatizem a necessidade do cultivo de leguminosas.

17 - (FUVEST SP/2005) Num campo, vivem gafanhotos que se alimentam de plantas e servem de alimento para
passarinhos. Estes so predados por gavies. Essas quatro populaes se mantiveram em nmeros estveis nas
ltimas geraes.
a) Qual o nvel trfico de cada uma dessas populaes?
b) Explique de que modo a populao de plantas poder ser afetada se muitos gavies imigrarem para esse campo.
c) Qual a trajetria dos tomos de carbono que constituem as protenas dos gavies desde sua origem inorgnica?
d) Qual o papel das bactrias na introduo do nitrognio nessa cadeia alimentar?

18 - (UNICAMP SP/2004) A cidade ideal seria aquela em que cada habitante pudesse dispor, pelo menos, de 12 m
2

de rea verde (dados da OMS). Curitiba supera essa meta com cerca de 55m
2
por habitante. A poltica ambiental da
prefeitura dessa cidade prioriza a construo de parques, bosques e praas que, alm de proporcionar reas de lazer,
desempenham funes como amenizar o clima, melhorar a qualidade do ar e equilibrar o ciclo hdrico, minimizando a
ocorrncia de enchentes.
a) Explique como as plantas das reas verdes participam do ciclo hdrico, indicando as estruturas vegetais envolvidas
nesse processo e as funes por elas exercidas.
b) Qual seria o destino da gua da chuva no utilizada pelas plantas no ciclo hdrico?

TEXTO: 1 - Comum s questes: 19, 20, 21

Exrcito Cururu

Importado para exterminar besouros que atacavam canaviais, sapo brasileiro vira praga na Austrlia.
Sapos, milhes de sapos asquerosos e venenosos, em saltos pelo sol at onde a vista alcana. No se trata de uma
das famosas sete pragas do Egito. A invaso real e acontece na costa leste australiana. Esse exrcito coaxante
formado por centenas de milhes de sapos amaznicos da espcie Bufo marinus. ou, para o brasileiro leigo, o
folclrico sapo-cururu. O bicho foi introduzido na Austrlia em 1935 para o controle biolgico de um besouro que
atacava os canaviais, estratgia utilizada com sucesso nos Estados Unidos e na Amrica Central. Mas na terra dos
cangurus o cururu no funcionou. Pior: transformou-se rapidamente em praga.
Peter Moon.
(ISTO , n
o
1302, 14.09.94, p.50)

19 - (UFG GO/1997) Considerando as relaes do homem com o meio ambiente,
a) apresente dois argumentos contra o uso excessivo de defensivos qumicos nos canaviais;
b) justifique, exemplificando, a afirmao seguinte: O relacionamento entre insetos e o homem assume mltiplos
aspectos, que vo desde a sade at a agricultura.

20 - (UFG GO/1997) Considerando as relaes do meio bitico com o meio abitico, apresente e explique trs
formas que caracterizam a participao dos sapos ou besouros ou cangurus no Ciclo do Carbono.

21 - (UFG GO/1997) Considerando o ciclo de vida,
a) relacione duas caractersticas dos canaviais que facilitam o ciclo de vida das populaes de besouros;
b) crie um grfico, relacionando o processo de crescimento dos besouros e dos sapos, em relao ao tempo.
QUESTOES DO ENEM





186
01 - (ENEM Simulado/2009) A figura a seguir ilustra as principais fontes de emisses mundiais de gs carbnico,
relacionando-as a nossas compras domsticas (familiares).

Compre verde: como nossas compras podem ajudar a salvar o
planeta. poca, n. 515, 31 maro 2008.

Com base nas informaes da figura, observado que as
emisses de gs carbnico esto diretamente ligadas s compras
domsticas. Deste modo, deduz-se das relaes de produo e
consumo apresentadas que

a) crescimento econmico e proteo ambiental so polticas
pblicas incompatveis.
b) a reduo da atividade industrial teria pouco impacto nas
emisses globais de gs carbnico.
c) os fluxos de carbono na biosfera no so afetados pela
atividade humana, pois so processos cclicos.
d) a produo de alimentos, em seu conjunto, diretamente
responsvel por 17% das emisses de gs carbnico.
e) haveria decrscimo das emisses de gs carbnico se o
consumo ocorresse em reas mais prximas da produo.



02 - (ENEM Simulado/2009) O grfico a seguir ilustra, de maneira hipottica, o nmero de casos, ao longo de 20
anos, de uma doena infecciosa e transmissvel (linha cheia), prpria de uma regio tropical especfica, transmitida por
meio da picada de inseto. A variao na densidade populacional do inseto transmissor, na regio considerada,
ilustrada (linha pontilhada). Durante o perodo apresentado no foram registrados casos dessa doena em outras
regies.



Sabendo que as informaes se referem a um caso tpico de endemia, com um surto epidmico a cada quatro anos,
percebe-se que no terceiro ciclo houve um aumento do nmero de casos registrados da doena. Aps esse surto foi
realizada uma interveno que controlou essa endemia devido

a) populao ter se tornado autoimune.
b) introduo de predadores do agente transmissor.
c) instalao de proteo mecnica nas residncias, como telas nas aberturas.
d) ao desenvolvimento de agentes qumicos para erradicao do agente transmissor.
e) ao desenvolvimento de vacina que ainda no era disponvel na poca do primeiro surto.

03 - (ENEM Simulado/2009) Ao utilizarmos um copo descartvel no nos damos conta do longo caminho pelo qual
passam os tomos ali existentes, antes e aps esse uso. O processo se inicia com a extrao do petrleo, que
levado s refinarias para separao de seus componentes. A partir da matria-prima fornecida pela indstria
petroqumica, a indstria qumica produz o polmero base de estireno, que moldado na forma de copo descartvel
ou de outros objetos, tais como utenslios domsticos. Depois de utilizados, os copos so descartados e jogados no
lixo para serem reciclados ou depositados em aterros.

Materiais descartveis, quando no reciclados, so muitas vezes rejeitados e depositados indiscriminadamente em
ambientes naturais. Em consequncia, esses materiais so mantidos na natureza por longo perodo de tempo. No caso
de copos plsticos constitudos de polmeros base de produtos petrolferos, o ciclo de existncia deste material passa
por vrios processos que envolvem


187
a) a decomposio biolgica, que ocorre em aterros sanitrios, por micro-organismos que consomem plsticos com
estas caractersticas apolares.
b) a polimerizao, que um processo artificial inventado pelo homem, com a gerao de novos compostos
resistentes e com maiores massas moleculares.
c) a decomposio qumica, devido quebra de ligaes das cadeias polimricas, o que leva gerao de
compostos txicos ocasionando problemas ambientais.
d) a polimerizao, que produz compostos de propriedades e caractersticas bem definidas, com gerao de
materiais com ampla distribuio de massa molecular.
e) a decomposio, que considerada uma reao qumica porque corresponde unio de pequenas molculas,
denominados monmeros, para a formao de oligmeros.

04 - (ENEM Simulado/2009) Nas ltimas dcadas os ecossistemas aquticos tem sido alterados de maneira
significativa em funo de atividades antrpicas, tais como minerao, construo de barragens, desvio do curso
natural de rios, lanamento de efluentes domsticos e industriais no tratados, desmatamento e uso inadequado do
solo prximo aos leitos, superexplorao dos recursos pesqueiros, introduo de espcies exticas, entre outros.
Como consequncia, tem-se observado expressiva queda da qualidade da gua e perda da biodiversidade aqutica,
em funo da desestruturao dos ambientes fsico, qumico e biolgico. A avaliao de impactos ambientais nesses
ecossistemas tem sido realizada atravs da medio de alteraes nas concentraes de variveis fsicas e qumicas
da gua. Este sistema de monitoramento, juntamente com a avaliao de variveis biolgicas, fundamental para a
classificao de rios e crregos em classes de qualidade de gua e padres de potabilidade e balneabilidade
humanas.
DAVE, M.; GOULART, C.; CALLISTO, M. Bioindicadores de qualidade de gua como ferramenta de impacto
ambiental. Disponvel em: http://www.icb.ufmg.br. Acesso em: 9 jan. 2009 (adaptado).

Se um pesquisador pretende avaliar variveis biolgicas de determinado manancial, deve escolher os testes de

a) teor de oxignio dissolvido e de temperatura e turbidez da gua.
b) teor de nitrognio amoniacal e de temperatura e turbidez da gua.
c) densidade populacional de cianobactrias e de invertebrados bentnicos.
d) densidade populacional de cianobactrias e do teor de alumnio dissolvido.
e) teor de nitrognio amoniacal e de densidade populacional de invertebrados bentnicos.

05 - (ENEM Simulado/2009) Uma colnia de formigas inicia-se com uma rainha jovem que, aps ser fecundada pelo
macho, voa e escolhe um lugar para cavar um buraco no cho. Ali dar origem a milhares de formigas, constituindo
uma nova colnia. As fmeas geradas podero ser operrias, vivendo cerca de um ano, ou novas rainhas. Os machos
provm de vulos no fertilizados e vivem aproximadamente uma semana. As operrias se dividem nos trabalhos do
formigueiro. H formigas forrageadoras que se encarregam da busca por alimentos, formigas que retiram dejetos da
colnia e so responsveis pela manuteno ou que lidam com o alimento e alimentam as larvas, e as formigas
patrulheiras. Uma colnia de formigas pode durar anos e dificilmente uma formiga social consegue sobreviver sozinha.

Melo, A. Como funciona uma sociedade de formigas? Disponvel em:
http://www.cienhoje.uol.com.br. Acesso em: 21 fev. 2009 (adaptado).

Uma caracterstica que contribui diretamente para o sucesso da organizao social dos formigueiros

a) a diviso de tarefas entre as formigas e a organizao funcional da colnia.
b) o fato de as formigas machos serem provenientes de vulos no fertilizados.
c) a alta taxa de mortalidade das formigas solitrias ou das que se afastam da colnia.
d) a existncia de patrulheiros, que protegem o formigueiro do ataque de herbvoros.
e) o fato de as rainhas serem fecundadas antes do estabelecimento de um novo formigueiro.


TEXTO: 2 - Comum s questes: 22, 23

Chuva choveu, goteira pingou...

At que fosse descoberta, pelos cientistas, a necessidade do iodo no organismo humano e que o Servio de Sade o
adicionasse ao sal de cozinha nica substncia usada pela totalidade da populao brasileira era considervel o
nmero de portadores de bcio. Alm deste, tornavam-se constantes as deficincias no desenvolvimento somtico e
mental da criana, em conseqncia do mau funcionamento da glndula tireide.
Belkiss S. C. Mendona. O popular, 15/08/97.





188
22 - (UFG GO/1998) Chuva choveu, goteira pingou...
Considerando que a chuva se constitui num fator imprescindvel para a vida,
a) apresente trs caractersticas da flora ou da fauna do Cerrado, em funo da distribuio de chuvas nesta regio;
b) esquematize o ciclo da gua.

23 - (UFG GO/1998) necessidade (...) organismo humano (...) substncia
Considerando que todos seres vivos necessitam de matria-prima e de energia para a realizao de seus processos
vitais e que essa necessidade suprida por uma grande variedade de substncias,
a) cite trs produtos obtidos a partir da atividade fngica no processo de transformao de matria-prima, utilizados
pelo homem;
b) esquematize a pirmide de energia identificando cada nvel trfico.
















































189






PROF.:RINALDO JOS DE RESENDE






C
C
o
o
n
n
t
t
e
e

d
d
o
o
s
s
:
:

B
B
O
O
T
T

N
N
I
I
C
C
A
A
I
I






















D Da at ta a d da a E En nt tr re eg ga a : : _ __ __ __ __ __ __ __ _/ /_ __ __ __ __ __ __ __ __ __ __ _/ /2 20 01 13 3

LISTA 5 BIOLOGIA

190


01 - (UFRRJ/2006) Numa comparao grosseira, as brifitas so consideradas os anfbios do mundo vegetal.
Cite duas caractersticas deste grupo vegetal que justifiquem esta comparao com os anfbios.

02 - (EFOA MG/2000) A figura abaixo ilustra trs espcies (I,
II e III) de um mesmo grupo taxonmico de plantas, conhecido
como "traquefitas", que se destaca pela sua importncia
filogentica e botnica.

a) Qual a diviso taxonmica que engloba essas trs espcies
vegetais?
b) Cite uma caracterstica bsica desse grupo de plantas.
c) Que nome recebe a estrutura indicada pela seta em I e II?





03 - (UFG GO/2005) As brifitas e as pteridfitas so vegetais caractersticos de ambientes midos.
a) Explique como ocorre o transporte da gua no interior desses organismos.
b) Apresente uma razo para o fato de as brifitas serem consideradas organismos importantes na dinmica das
comunidades.

04 - (UEPA/1999) Sobre a reproduo dos FANERGRAMOS perguntamos:
a) O que o fenmeno da PROTANDRIA?
b) Quando a polinizao dita ORNITFILA?
c) Quando a polinizao dita ENTOMFILA?
d) Nas GIMNOSPERMAS a polinizao do tipo ANEMFILA. Que tipo de vegetal este e como definir este tipo de
polinizao?
e) Qual dos integrantes de uma semente resulta de uma clula TRIPLIDE, e que estruturas se juntam para form-lo?

05 - (UFC CE/1997) Um dos smbolos do Natal a rvore, normalmente representada por um pinheiro. Sobre os
pinheiros pergunta-se:
a) A que grupo taxonmico a nvel de Diviso pertencem os pinheiros?
b) Qual o significado do nome dessa Diviso do reino Plantae?

06 - (UNESP SP/1999) Observe o ciclo reprodutivo do pinheiro.


a) Em que estgios deste ciclo ocorre reduo do nmero de cromossomos?
b) Indique as estruturas citadas no ciclo que correspondem s palavras assinaladas na seguinte estrofe popular:
Pinheiro me d uma pinha
Pinha me d um pinho
Menina me d um beijo
Que eu te dou meu corao.
Caderno de Atividades

Disciplina:
B Bi io ol lo og gi ia a

Professor(a):
Rinaldo
Aluno:
3 ano
Ensino Mdio
Data de Recebimento:
_____/_____/_____
Lista 05
Data Entrega:
_____/_____/_____

I II III

191
07 - (UFPA/1998) As angiospermas, organismos mais evoludos entre os vegetais, apresentam a semente envolvida
pelo fruto e compreendem dois grandes grupos: as dicotiledneas, que possuem dois cotildones em suas sementes, e
as monocotilidneas, que apresentam um cotildone nas sementes.
a) Em que regio da semente os cotildones so encontrados e qual a funo dos mesmos?
b) Em relao s plantas dicotiledneas e monocotiledneas, como se caracterizam as folhas e as razes?

08 - (UFRJ/1997) Na maioria dos angiospermas - plantas com flores e frutos - a reproduo depende da polinizao,
ou seja, do transporte dos gros de plen de um indivduo para outro. Em alguns casos, o transporte feito pelo vento
e, em outros, por animais polinizadores que visitam sistematicamente as flores.
Em qual dos dois casos a produo de plen deve ser maior? Justifique sua resposta.

09 - (UFRJ/1997) Nas plantas com flores e frutos (angiospermas), os esporos no so lanados no ambiente; eles
germinam no corpo do esporfito formando o gro de plen, que levado at a folha feminina. A, o gro de plen
forma um tubo polnico (gametfito masculino), no interior do qual esto os gametas masculinos. Esse tubo cresce em
direo ao vulo, onde est o saco embrionrio (gametfito feminino), com a oosfera.
Esse ciclo reprodutivo no depende tanto da gua quanto o ciclo reprodutivo de plantas pteridfitas, como as
samambaias. Por qu?

10 - (UFAL/2002) Faa um esquema de uma clula vegetal.
a) Nomeie com legendas as estruturas representadas.
b) Destaque duas estruturas que no aparecem em clulas animais e indique a funo de cada uma delas.

11 - (UFMG/2007)
1. Observe esta figura:


Na transio da gua para a terra, os seres clorofilados desenvolveram estruturas que os tornaram capazes de
conquistar o novo ambiente.
CITE a caracterstica e o modo de ao, ainda presentes nas plantas vasculares atuais, que permitiram tal conquista
em relao a
a) suporte do corpo.
Caracterstica:
Modo de ao:
b) reteno de gua.
Caracterstica:
Modo de ao:
c) respirao.
Caracterstica:
Modo de ao:
d) reproduo.
Caracterstica:
Modo de ao:

2. Os representantes do Reino Plantae apresentam peculiaridades que so percebidas desde o nvel celular.
Analise esta figura:

192
a) Considerando as informaes dessa figura e outros conhecimentos sobre o assunto,
CITE a funo desempenhada pela organela B.
b) EXPLIQUE de que modo a funo da organela A e a da organela C so realizadas nas
clulas dos seres do Reino Animalia.



12 - (UFG GO/1999) Os remdios podem conter medicamentos, produzidos a partir de
substncias vegetais, formadas nos processos fotossintticos. Considerando a morfologia
e a fisiologia vegetal,

a) Nomeie os verticilos florais de uma flor completa, com suas respectivas funes;
b) relacione sais minerais, xilema e folha

13 - (UNESP SP/2000) Recentemente, os jornais e a revista cientfica internacional Nature publicaram com destaque
um grande feito de um grupo de cientistas brasileiros, que identificou o genoma de uma bactria, Xylella fastidiosa, que
causa uma doena nas laranjeiras, conhecida como amarelinho. O xilema das plantas produtoras de laranja
parcialmente bloqueado, reduzindo a produo, pois a maioria das frutas no se desenvolve.

a) Explique o que genoma e indique uma possvel conseqncia econmica dos resultados desta pesquisa.
b) Se o xilema bloqueado, quais as principais deficincias que a planta sofre? Justifique.

14 - (FUVEST SP/1995) Realizou-se o seguinte experimento com um grupo de plantas: retirou-se um anel de casca
contendo o floema, mantendo-se folhas acima e abaixo da regio cortada. Em seguida, somente folhas abaixo do corte
foram expostas a CO
2
radioativo durante 24 horas. Em que regies da planta sero encontradas substncias com
material radioativo aps o experimento? Por que?

15 - (UNICAMP SP/1995) A remoo de um anel da casca do tronco de uma rvore provoca um espessamento na
regio situada logo acima do anel. A rvore acaba morrendo.
a) O que causa o espessamento? Por qu?
b) Por que a rvore morre?
c) Se o mesmo procedimento for feito num ramo, as folhas ou frutos desse ramo tendero a se desenvolver mais do
que os de um ramo normal. Por que isso ocorre?
d) No inverno, em regies temperadas, a remoo do anel no causa espessamento nas rovres que perdem folhas.
Por qu?

16 - (FUVEST SP/1999) Em vegetais, as taxas de fotossntese e de respirao podem ser calculadas a partir da
quantidade de gs oxignio produzido ou consumido num determinado
intervalo de tempo.

O grfico a seguir mostra as taxas de respirao e de fotossntese de
uma planta aqutica, quando se varia a intensidade luminosa.
a) Em que intensidade luminosa, o volume de gs oxignio produzido
na fotossntese igual ao volume desse gs consumido na
respirao?
b) Em que intervalo de intensidade luminosa, a planta est gastando
suas reservas?
c) Se a planta for mantida em intensidade luminosa r , ela pode
crescer? Justifique.

17 - (UFRJ/1993) Para testar as trocas gasosas das plantas com o ar, foi realizada uma experincia na qual foram
usados quatro tubos de vidro (A, B C e D)contendo soluo nutritiva, folhas vivas nos tubos A e B e razes vivas nos
tubos C e o D.
Cada tubo, depois de hermeticamente fechado, recebeu o tratamento indicado nas figuras abaixo:
Permaneceu
iluminado
Permaneceu
no escuro
A B

Permaneceu
iluminado
Permaneceu
no escuro
C D

Passado um determinado perodo de tempo, quais modificaes devemos esperar que ocorram com as concentraes
de O
2
e de CO
2
no ar do interior dos tubos? Justifique sua resposta.


193
18 - (UFRJ/1996) Tanto as folhas quanto as razes das plantas respiram. A taxa de respirao proporcional taxa
de metabolismo. Foram feitas medidas da intensidade de respirao de razes e folhas de uma rvore, localizada em
um pas da zona temperada do hemisfrio sul, onde as chuvas so constantes durante todo o ano. Os resultados esto
no grfico a seguir:
J F M A M J J A S O N D
Razes
Folhas
T
a
x
a

d
e

R
e
s
p
i
r
a

o
o
u

g
a
s
t
o

d
e

O
2
B
a
i
x
a
A
l
t
a
Meses

a) Com base no grfico, qual o parmetro climtico que explica a reduo da taxa de respirao nos meses de junho,
julho e agosto? Justifique sua resposta.
b) Por que as taxas de respirao das razes apresentam a mesma tendncia que as das folhas, mas com valores
multo menores?

19 - (UFRJ/1993) O grfico, a seguir, mostra o grau de abertura dos estmatos das folhas de uma planta ao longo do
dia.
6 12 18
Hora do dia
A
b
e
r
t
u
r
a


d
o


e
s
t
o
m
a
t
o
F
e
c
h
a
d
o
a
b
e
r
t
o

Com base no grfico, podemos dizer que a intensidade da fotossntese maior na hora mais iluminada do dia?
Justifique sua resposta

20 - (UFRJ/1996) Nos pases de clima frio, a temperatura do ar no inverno , muitas vezes, inferior a 0C. A gua do
solo congela e o ar frio e muito seco. Nesse perodo, muitas espcies vegetais perdem todas as folhas.
A perda das folhas evita um grande perigo para essas plantas.
Que problema a planta poderia sofrer caso no perdesse as folhas? Justifique sua resposta.

21 - (UFRJ/2000) As plantas chamadas de C3 perdem muito rendimento de produo de molculas orgnicas por
fotossntese quando a concentrao de CO
2
baixa. As plantas chamadas C4 possuem um metabolismo que contorna
essa situao, mantendo a velocidade da fotossntese mesmo em baixas concentraes de CO
2
. Considerando a
funo dos estmatos das folhas no controle da evapotranspirao, indique qual dos dois tipos de planta perde menos
produtividade em um clima seco e quente. Justifique sua resposta.

QUESTOES D O ENEM

Questo 01) Considere a situao em que foram realizados dois experimentos, designados de experimentos A e B,
com dois tipos celulares, denominados clulas 1 e 2. No experimento A, as clulas 1 e 2 foram colocadas em uma
soluo aquosa contendo cloreto de sdio (NaCl) e glicose (C
6
H
12
O
6
), com baixa concentrao de oxignio. No
experimento B foi fornecida s clulas 1 e 2 a mesma soluo, porm com alta concentrao de oxignio, semelhante
atmosfrica. Ao final do experimento, mediu-se a concentrao de glicose na soluo extracelular em cada uma das
quatro situaes. Este experimento est representado no quadro abaixo. Foi observado no experimento A que a
concentrao de glicose na soluo que banhava as clulas 1 era maior que a da soluo contendo as clulas 2 e esta
era menor que a concentrao inicial. No experimento B, foi observado que a concentrao de glicose na soluo das
clulas 1 era igual das clulas 2 e esta era idntica observada no experimento A, para as clulas 2, ao final do
experimento.
Pela interpretao do experimento descrito, pode-se observar
que o metabolismo das clulas estudadas est relacionado s
condies empregadas no experimento, visto que as

a) clulas 1 realizam metabolismo aerbio.
b) clulas 1 so incapazes de consumir glicose.
c) clulas 2 consomem mais oxignio que as clulas 1.
d) clulas 2 tm maior demanda de energia que as clulas 1.
e) clulas 1 e 2 obtiveram energia a partir de substratos diferentes.

194
Questo 02) Na Regio Amaznica, diversas espcies de aves se alimentam da ucuba (Virola sebifera), uma rvore
que produz frutos com polpa carnosa, vermelha e nutritiva. Em locais onde essas rvores so abundantes, as aves se
alternam no consumo dos frutos maduros, ao passo que em locais onde elas so escassas, tucanos-de-papo-branco
(Ramphastus tucanos cuvieri) permanecem forrageando nas rvores por mais tempo. Por serem de grande porte, os
tucanos-de-papo-branco no permitem a aproximao de aves menores, nem mesmo de outras espcies de tucanos.
Entretanto, um tucano de porte menor (Ramphastus vitellinus Ariel), ao longo de milhares de anos, apresentou
modificao da cor do seu papo, do amarelo para o branco, de maneira que se tornou semelhante ao seu parente
maior. Isso permite que o tucano menor compartilhe as ucubas com a espcie maior sem ser expulso por ela ou
sofrer as agresses normalmente observadas nas reas onde a espcie apresenta o papo amarelo.
PAULINO NETO, H. F. Um tucano disfarado.
Cincia Hoje, v. 252, p. 67-69, set. 2008. (com adaptaes).

O fenmeno que envolve as duas espcies de tucano constitui um caso de

a) mutualismo, pois as duas espcies compartilham os mesmos recursos.
b) parasitismo, pois a espcie menor consegue se alimentar das ucubas.
c) relao intraespecfica, pois ambas as espcies apresentam semelhanas fsicas.
d) sucesso ecolgica, pois a espcie menor est ocupando o espao da espcie maior.
e) mimetismo, pois uma espcie est fazendo uso de uma semelhana fsica em benefcio prprio.

Questo 03) Um viajante saiu de Araripe, no Cear, percorreu, inicialmente, 1000 km para o sul, depois 1000 km
para o oeste e, por fim, mais 750 km para o sul. Com base nesse trajeto e no mapa acima, pode-se afirmar que,
durante seu percurso, o viajante passou pelos estados do Cear,

a) Rio Grande do Norte, Bahia, Minas Gerais, Gois e Rio de Janeiro, tendo
visitado os ecossistemas da Caatinga, Mata Atlntica e Pantanal. Encerrou
sua viagem a cerca de 250 km da cidade de So Paulo.
b) Rio Grande do Norte, Bahia, Minas Gerais, Gois e Rio de Janeiro, tendo
visitado os ecossistemas da Caatinga, Mata Atlntica e Cerrado. Encerrou sua
viagem a cerca de 750 km da cidade de So Paulo.
c) Pernambuco, Bahia, Minas Gerais, Gois e So Paulo, tendo visitado os
ecossistemas da Caatinga, Mata Atlntica e Pantanal. Encerrou sua viagem a
cerca de 250 km da cidade de So Paulo.
d) Pernambuco, Bahia, Minas Gerais, Gois e So Paulo, tendo visitado os
ecossistemas da Caatinga, Mata Atlntica e Cerrado. Encerrou sua viagem a
cerca de 750 km da cidade de So Paulo.
e) Pernambuco, Bahia, Minas Gerais, Gois e So Paulo, tendo visitado os
ecossistemas da Caatinga, Mata Atlntica e Cerrado. Encerrou sua viagem a
cerca de 250 km da cidade de So Paulo.

Questo 04) A luz, fundamental para a vida, utilizada pelas plantas na
fotossntese, processo no qual o dixido de carbono e a gua so utilizados na formao de carboidratos. No entanto,
foram necessrios anos para que os fenmenos luminosos fossem compreendidos. (HART-DAVIS, 2010, p. 122-125).
Considerando-se os aspectos associados ao processo da fotossntese, correto afirmar:

01. A fotossntese ocorre somente na presena de luz, que fornece a entalpia-padro de formao do carboidrato.
02. O surgimento da fotossntese associada liberao de O
2
desencadeou grandes mudanas na atmosfera primitiva,
como a formao da camada de oznio, impactando na evoluo das primeiras espcies que habitaram o planeta
Terra.
04. O fenmeno da fotossntese no restrito s plantas, pois tambm ocorre em fungos clorofilados.
08. tomos da molcula de clorofila excitados por absoro de luz apresentam eltrons em nveis de energia
superiores aos de origem.
16. Uma planta, para sintetizar compostos orgnicos, absorve todo o espectro visvel da onda eletromagntica
proveniente do Sol.
32. Parte do carboidrato produzido na fotossntese utilizado pelos vegetais na respirao e na produo de
molculas orgnicas pelas clulas, o excedente armazenado em tecidos de reserva e servir, inclusive, como
fonte de energia para seres hetertrofos.





195






PROF.:RINALDO JOS DE RESENDE







C
C
o
o
n
n
t
t
e
e

d
d
o
o
s
s
:
:

B
B
O
O
T
T

N
N
I
I
C
C
A
A
I
I
I
I

























D Da at ta a d da a E En nt tr re eg ga a : : _ __ __ __ __ __ __ __ _/ /_ __ __ __ __ __ __ __ __ __ __ _/ /2 20 01 13 3
LISTA 6 BIOLOGIA

196

01 - (UFRJ/1994) A auxina um hormnio vegetal que, dependendo da concentrao, pode inibir ou estimular o
crescimento das clulas jovens do caule e da raiz. Injetando esse hormnio numa concentrao de 10
-5

molar,
lateralmente, na regio de crescimento do caule e da raiz, o rgo cresce curvando-se para o lado, como mostra a
figura abaixo:

Os efeitos da injeo de auxina no caule e na raiz esto representados no grfico a seguir.
10
-11
0
c
r
e
s
c
i
m
e
n
t
o
+
concentrao molar de auxina
A
B
10
-9
10
-7
10
-5
10
-3
10
-1

Determine e curva que corresponde ao caule e a que corresponde raiz. Justifique sua resposta.

02 - (UNICAMP SP/1999) Sabe-se que uma planta daninha de nome striga, com folhas largas e nervuras reticuladas,
invasora de culturas de milho, arroz, cana e de muitas outras espcies de gramneas na sia e na frica, a nova dor
de cabea dos tcnicos agrcola no Brasil. Sabe-se tambm que algumas auxinas sintticas so usadas como
herbicidas porque so capazes de eliminar dicotiledneas e no agem sobre monocotiledneas.
a) Qual seria o resultado da aplicao de um desses herbicidas no combate striga invasora em um carnaval? E em
uma plantao de tomates? Explique sua resposta.
b) Indique uma auxina natural e mencione uma de suas funes na planta.

03 - (UNESP SP/1999) Em ruas e avenidas arborizadas, periodicamente as companhias distribuidoras de eletricidade
realizam cortes da parte superior das rvores que esto em contato com os fios eltricos de alta tenso. As podas so
necessrias para se evitarem problemas que podem ocorrer em dias chuvosos e de fortes ventos.
a) O que dever acontecer com as rvores aps o corte da regio apical que estava atingindo os fios eltricos?
b) Que mecanismo explica o resultado obtido com o corte da regio apical?

04 - (UFAL/2005) Plantas que florescem no vero so chamadas plantas de dia longo. Para saber se a florao de
uma espcie de dia longo determinada pela durao do dia ou da noite, os pesquisadores mantiveram plantas em 3
condies. O experimento e os resultados esto esquematizados abaixo:


De acordo com os resultados, explique o que determina a florao nessa
espcie.









Caderno de Atividades

Disciplina:
B Bi io ol lo og gi ia a

Professor(a):
Rinaldo
Aluno:
3 ano
Ensino Mdio
Data de Recebimento:
_____/_____/_____
Lista 06
Data Entrega:
_____/_____/_____

197
05 - (UFPR/2005) As plantas tm um comportamento fundamentalmente diferente do comportamento animal. Os
animais podem fazer movimentos rpidos e precisos, graas a seus msculos e a sua coordenao nervosa e
sensorial. As plantas, por sua vez, realizam movimentos lentos, praticamente imperceptveis, mas que lhes permitem
um ajustamento adequado aos estmulos ambientais. Sabendo que o crescimento das plantas estimulado por fatores
externos (ambientais) e internos (hormonais), responda as seguintes perguntas:

b.1) Que fatores ambientais direcionam o crescimento dos rgos vegetais para cima ou para baixo do solo?
b.2) Qual o local (rgo vegetal) de produo das auxinas (hormnio do crescimento)?
b.3) Explique o efeito da poda para a ramificao.

06 - (UEG GO/2006) No experimento ilustrado a seguir, uma planta jovem foi exposta iluminao unilateral.



Sobre esse experimento, responda:
a) Como se denominam os fenmenos das curvaturas observadas no caule e na raiz da planta utilizada no
experimento?
b) Qual o hormnio envolvido no processo e como se d a curvatura dos rgos em questo (caule e raiz)?

07 - (UNICAMP SP/1999) Foi feito um experimento utilizando a epiderme de folha de uma e uma suspenso de
hemcias. Esses dois tipos celulares foram colocados em gua destilada e em soluo salina concentrada. Observou-
se ao microscpio que as hemcias, em presena de gua destilada, estouravam e, em presena de soluo
concentrada, murchavam. As clulas vegetais no se rompiam em gua destilada, mas em soluo salina concentrada
notou-se que o contedo citoplasmtico encolhia.
a) A que tipo de transporte celular o experimento est relacionado?
b) Em que situao ocorre esse tipo de transporte?
c) A que se deve a diferena de comportamento da clula vegetal em relao clula animal? Explique a diferena de
comportamento, considerando as clulas em gua destilada e em soluo concentrada.

08 - (FUVEST SP/2001) O esquema representa um experimento em que plantas semelhantes foram colocadas em
tubos, com igual quantidade de gua, devidamente vedados para evitar a evaporao. A planta do tubo A foi mantida
intacta; a do tubo B teve suas folhas totalmente cobertas por uma camada de vaselina. Cada tubo mostra o nvel da
gua no incio do experimento (Ni) e no final (Nf).

a) Por que os nveis da gua ficaram diferentes nos tubos A e B?
b) Que estruturas da epiderme foliar tiveram seu funcionamento afetado pela vaselina?
c) Qual o papel dessas estruturas da epiderme para que a planta realize fotossntese?



198
09 - (UFRJ/1998) No ciclo reprodutivo da maioria dos vegetais observa-se uma alternncia de geraes que
mostrada, de forma simplificada, no esquema a seguir:
Gametfito Esporfito
Meiose
Fertilizao


Em qual das fases - gametfito ou esporfito - no encontramos pares de cromossomos homlogos? Justifique sua
resposta.

10 - (UFRJ/1999) As clulas da raiz de um p de milho possuem 20 cromossomos.
Levando em conta o ciclo reprodutivo desse vegetal, quantos cromossomos voc espera encontrar nas clulas do
albume (endosperma) e do embrio de um gro de milho? Justifique sua resposta.

11 - (UFC CE/2001) A dicogamia, condio na qual os estames e carpelos de uma flor hermafrodita (monoclina)
amadurecem em perodos distintos, e a partenocarpia, formao de frutos sem sementes, so fenmenos que ocorrem
nas angiospermas.
Pergunta-se:
a) Qual a conseqncia direta da dicogamia na reproduo sexuada das plantas e por que essa caracterstica favorece
a adaptao ambiental e a sobrevivncia da populao vegetal que a apresenta?
b) Como as plantas que s formam frutos partenocrpicos se reproduzem naturalmente?
c) Como possvel a produo artificial de frutos partenocrpicos, uma prtica importante na fruticultura?

12 - (UFLA MG/2003) Considere uma rvore de 5 m de altura, que cresce 1 m por ano.
a) Se ocorrer uma leso que deixe uma marca em seu tronco, a 1,5 m do solo, a que altura ela estar aos 5 anos?
Explique.
b) Se for retirado um anel da casca do caule, logo acima do nvel do solo, provavelmente a rvore morrer. Por que isso
pode acontecer?

13 - (UFRRJ/2001) Obtm-se a cortia a partir do tecido denominado SBER, presente em plantas da regio
mediterrnea, tais como o sobreiro. O sber resulta da atividade do meristema secundrio, sendo formado por vrias
camadas de clulas mortas e ocas.
Cite duas funes do sber.

14 - (UFRJ/2000) As flores que se abrem noite, como por exemplo a dama-da-noite, em geral exalam um perfume
acentuado e no so muito coloridas. As flores diurnas, por sua vez, geralmente apresentam cores mais intensas.
Relacione essa adaptao ao processo de reproduo desses vegetais.

15 - (UFRRJ/1999) A seguir esto representados dois tipos de flor:
A. relativamente pequena, com perianto no-atrativo e grandes antenas que ficam pendentes para fora.
B. com ptalas largas, vistosas e coloridas; nectrios e glndulas odorferas.

Que planta est mais adaptada polinizao por animais? Justifique sua resposta.

16 - (FUVEST SP/1999) A conquista do meio terrestre, pelas plantas, foi possvel graas a um conjunto de
adaptaes.
a) Cite duas adaptaes dos vegetais terrestres relacionadas economia de gua.
b) Que estruturas vegetais permitem a disperso das pteridfitas e das gimnospermas, independentemente do meio
aqutico?

17 - (UFU MG/2005) A ilustrao a seguir representa, com um esquema tridimensional, a morfologia interna de uma
folha. Analise-a e responda as questes que seguem.

199

Adaptado de AMABIS, J.M. & MARTHO, G.R. "Fundamentos de Biologia Moderna". So Paulo: Moderna, 2003 e
Http://www.ualr.edu/~botany/leafstru

a) Qual o nome da estrutura apontada pelo nmero 1 e a que tecido ela pertence?
b) Qual o nome do tecido apontado pelo nmero 2 e qual a sua funo?

18 - (EFEI MG/2000) Classifique os vegetais comestveis abaixo relacionados, como sendo: raiz, caule , frutos
verdadeiros ou pseudofrutos.
Amendoim, Cebola, Cenoura, Ma, Batata, Azeitona, Tomate, Manga, Pra e Mandioca.

19 - (UNESP SP/2000) Um aluno de uma Escola de Ensino Mdio
recebeu de seu professor de Biologia uma lista de diversos vegetais
considerados comestveis. O aluno elaborou um quadro onde, com o
sinal (X), indicou o rgo da planta utilizado como principal alimento.

Aps a anlise do quadro, o professor informou ao aluno que ele
havia cometido quatro erros.
a) Indique os quatro erros cometidos pelo aluno e identifique os
verdadeiros rgos a que pertencem os vegetais assinalados
erradamente.
b) Quais so as estruturas da flor que do origem, respectivamente,
aos frutos verdadeiros e aos pseudo-frutos relacionados no quadro?

20 - (UFRJ/2000) Uma criana gravou a inicial de seu nome no tronco de uma jaqueira, a 1,5m do solo. Aps alguns
anos, ao observar a rvore percebeu que ela estava muito mais alta e que sua inicial gravada continuava mesma
altura do solo, mas com o desenho bastante alargado.
Explique por que a gravao se alargou com a passagem dos anos.

21 - (EFOA MG/2006) Os esquemas I e II abaixo correspondem a um tipo de tecido de grande importncia para as
plantas e algumas de suas especializaes celulares distribudas em dois padres distintos.




Com base nas caractersticas desse tipo de tecido, cite:
a) as duas Classes de vegetais que so diferenciadas pelos padres de distribuio I e II, respectivamente.
I: _________________________ II: __________________________
b) o nome da estrutura especializada representada nos esquemas.
c) o nome do meristema primrio que origina esse tipo de tecido.
d) o nome das nicas clulas clorofiladas apresentadas nesse tipo de tecido.
e) um exemplo de planta de interesse agronmico que apresenta o padro de distribuio do tipo de tecido observado
no esquema II.
Vegetais
comestveis
Raiz Caule Fruto Pseudo-fruto
Batata inglesa
Azeitona
Tomate
Manga
Pra
Mandioca
Ma
Cenoura
Cebola
Moranguinho
Pepino
X
X
X
X
X
X
X
X
X
X
X

200
22 - (UERJ/2005) A indstria de flores movimenta bilhes de dlares por ano e est continuamente buscando criar
novas variedades, como uma rosa azul. Pelos mtodos tradicionais, no possvel obter rosas dessa cor, pois a
enzima que sintetiza esse pigmento est ausente nas roseiras. As petnias, entretanto, possuem tal enzima. Encontra-
se em fase de teste uma tcnica que possibilita inserir em clulas de roseira o gene da petnia responsvel pela
codificao da enzima que produz o pigmento azul.

a) Cite o tipo de clula indiferenciada de roseira que, com o objetivo de produzir rosas azuis, seria o indicado para
receber esse gene e justifique sua resposta.
b) Suponha que, em vez de ser introduzido o gene da petnia que define a cor azul, fossem inseridas cpias de seu
RNA mensageiro.
Indique se h ou no possibilidade de produo de rosas azuis nesse caso e justifique sua resposta.

23 - (UFG GO/1996)
Os tecidos vegetais se renem para formar os rgos. Estes, por sua vez, formam o corpo da planta. Com relao aos
tecidos vegetais representados pelos esquemas A, B, C, D e E, pede-se o seguinte:

a) identificar os tecidos representados pelos esquemas correspondentes:
Esquema A :
Esquema B :
Esquema C :
Esquema D :
Esquema E :

b) dar a funo de cada tecido vegetal de acordo como esquema.
Esquema A:
Esquema B:
Esquema C:
Esquema D:
Esquema E:

24 - (EFOA MG/2000) Embora no sejam essenciais como nutriente, as fibras tm papel importante na dieta. Elas
estimulam os movimentos peristlticos, reduzindo o tempo de contato de substncias potencialmente prejudiciais no
intestino, evitando assim muitos distrbios como diverticulose e priso de ventre. A sua deficincia na dieta pode
causar tambm problemas mais graves como doenas coronarianas, cncer no intestino e obesidade. Em relao s
fibras, resolva os itens abaixo.

a) Qual o nome do polmero predominante nessas fibras?
b) Cite um tecido vegetal rico em fibras.
c) Alm da alimentao, cite uma importncia econmica de utilizao dessas fibras.



25 - (UFC CE/2001)
Atualmente, a cultura de clulas e tecidos est se tornando uma das tcnicas mais utilizadas no melhoramento vegetal.
a maneira mais eficiente para a obteno de plantas livres de vrus (ou bactrias) parasitas. Estes agentes
etiolgicos translocam-se atravs dos vasos condutores de seiva por toda a planta. A cultura de meristemas mostrou-
se o mtodo mais eficiente para a obteno de plantas sadias nos casos de infeco por vrus. Pergunta-se:
a) De uma maneira geral, em que consiste a tcnica da cultura de tecidos vegetais?
b) Qual a explicao para o sucesso da cultura de meristemas no caso especfico da infeco por vrus?


201










PROF.: Rmulo




C
C
o
o
n
n
t
t
e
e

d
d
o
o
s
s
:
:


























D Da at ta a d da a E En nt tr re eg ga a : : _ __ __ __ __ __ __ __ _/ /_ __ __ __ __ __ __ __ __ __ __ _/ /2 20 01 13 3
LISTA 1 BIOLOGIA

202

1. (UFRJ) A esquistossomose uma parasitose que afeta cerca de 15 milhes de brasileiros. A doena deve-se,
principalmente, s leses provocadas no fgado pelos ovos depositados pelas fmeas adultas, parte dos quais so
aprisionados pelo tecido heptico.
O ciclo de Schistosoma Mansoni, o responsvel por essa doena, est ilustrado a seguir:


Se um caramujo for invadido por um nico miracdio, as cercrias resultantes podero produzir a doena no homem?
Justifique sua resposta.

2. (UFRJ) As tnias, parasitas do intestino de vertebrados, so vermes que possuem uma pequena cabea (esclex),
de onde cresce um longo corpo formado por vrios anis (proglotes). Em cada anel, existe um sistema reprodutor
masculino e um feminino. A reproduo desses animais se faz por fecundao de gametas:
1. produzidos pelo mesmo anel;
2. produzidos por anis diferentes do mesmo animal;
3. produzidos por anis de indivduos diferentes quando ocorre mais de um indivduo por intestino.

Qual dessas formas de reproduo resulta em maior variabilidade gentica? Justifique sua resposta.

3 . (UFRN) A espcie Strongyloides stercoralis um pequeno verme que habita o intestino delgado humano.
A forma parasita desse verme a fmea partenogentica, que fica inserida na mucosa intestinal (ciclo direto). J no
solo, essa espcie pode desenvolver as formas macho e fmea, ambas de vida livre, permitindo a reproduo sexuada
(ciclo indireto).
Considerando as referidas adaptaes, em termos reprodutivos e de habitat, responda:
a) Cite e explique uma vantagem de cada um dos tipos de reproduo (sexuada e partenognese) para essa espcie.
b) Quando esse parasito submetido a uma faixa de temperatura entre 15 e 31oC, qual dos dois ciclos se desenvolve
adequadamente? Justifique sua resposta.

4. (UFBA) Podemos ter uma viso da notvel complexidade dos nematdeos examinando o pequeno Caenorhabditis
elegans, atualmente o animal que melhor se conhece.
Ele formado por exatas novecentas e cinqenta e nove clulas, cada uma localizada com grande preciso e
remontada sua origem, do vulo a uma mdia de oito a dezessete rodadas sucessivas de divises mitticas. Muitos
estudos esclareceram de que modo este desenvolvimento se desdobrou a partir do programa inscrito no genoma do
vulo.
(DE DUVE, 1997, p. 251).

Estudos realizados em C. elegans revelam a expressiva histria evolutiva dos nematdeos, que inclui a
multicelularidade. Nesse sentido, identifique dois requisitos decisivos para a evoluo da multicelularidade expressa
nesses nematdeos explicando sua importncia neste novo nvel de organizao.

Caderno de Atividades

Disciplina:
B Bi io ol lo og gi ia a

Professor(a):
Rmulo
Aluno:
3 ano
Ensino Mdio
Data de Recebimento:
_____/_____/_____
Lista 01
Data Entrega:
_____/_____/_____

203
5 . (UFRRJ) Em um estudo sobre variabilidade gentica, um pesquisador utilizou como modelo de estudo para as
suas anlises a Taenia solium e o Schistosoma mansoni.
Indique em qual dos modelos de estudo utilizados foi observada uma maior variabilidade gentica.
Justifique a razo desse resultado pelo pesquisador.

6. (UFES) A ascaridase, doena causada pelo Ascaris lumbricoides, atinge cerca de 60% da populao brasileira.
Essa doena, de endemia rural, como era entendida outrora, passa cada vez mais a ser um problema urbano.



01. Identifique o sexo dos animais da figura e d suas caractersticas morfolgicas diferenciais.
02. Uma professora relatou que um aluno, ao tossir, expeliu com a expectorao algumas larvas de lombriga.
Descreva o caminho percorrido por esses parasitas, desde a sua entrada no organismo humano at o momento em
que o fato ocorreu.
03. caracterstico dos vermes parasitas produzirem grandes quantidades de ovos. Uma fmea de Ascaris produz
cerca de 200 mil ovos por dia, que so eliminados juntamente com as fezes do hospedeiro. Descreva dois fatores que
justifiquem a necessidade de os Ascaris eliminarem esse elevado nmero de ovos no meio externo, relacionando
esses fatores ao ciclo de vida dos parasitas.

7. (UFPA) A respirao o mecanismo que permite aos seres vivos extrarem a energia qumica armazenada nos
alimentos e a sua utilizao nas diversas atividades metablicas do organismo. Uma das etapas desse processo
engloba o mecanismo de trocas gasosas entre as clulas e o meio ambiente.
Sobre esse aspecto, responda:
a) Nos organismos mais simples como as esponjas, os celenterados, os platelmintos e os asquelmintos, onde e como
ocorrem as trocas gasosas?
b) Nos mamferos, inclusive no homem, em que estrutura pulmonar ocorrem as trocas gasosas? Explique como se
processa esse fenmeno.

8. (UNESP SP) Considere os versos da cano infantil:

Minhoca, Minhoca, me d uma beijoca
No dou, no dou
Ento eu vou roubar
Minhoco, Minhoco, voc mesmo louco
Beijou o lado errado, a boca do outro lado
(www.escolapaulofreire.com.br/infantil/musica_amarelo.htm)

a) Qual a importncia das minhocas para as plantas?
b) Com relao organizao do corpo das minhocas (Annelida), justifique a frase beijou o lado errado. Com relao
reproduo das minhocas, justifique a correo ou incorreo dos termos minhoco (macho) e minhoca (fmea).

9. (FUVEST SP) O quadro abaixo mostra diferenas que ocorrem no reino animal quanto ao plano corporal e aos
sistemas digestrio, circulatrio e nervoso:

Dorsal
ventral
nervoso Cordo
Ausente nervoso Sistema - D
Fechado Aberto Ausente io circulatr Sistema - C
Completo Incompleto Ausente digestrio Sistema - B
Bilateral Radial Ausente
adulta
fase na Simetria - A
3 2 1


Os aneldeos, por exemplo, apresentam as caractersticas A3, B3, C3 e D2.
a) Que grupo animal apresenta as caractersticas A1, B1, C1 e D1?
b) Que caractersticas de A, B, C e D esto presentes em um crustceo?
c) Que caractersticas de A, B, C e D esto presentes em um anfbio?


204
10. (UFC CE) Na evoluo dos diferentes grupos animais, muitas mudanas ocorreram para dar origem a seres mais
complexos. Observe o cladograma (rvore filogentica) a seguir e responda ao que se pede:

a) Os nmeros I e II representam caractersticas que levaram ao
surgimento dos txons Cnidaria e Nematoda, respectivamente.
Indique essas caractersticas.
I.
II.
b) Cite uma caracterstica compartilhada entre protistas e animais
que culminou na teoria monofiltica, a qual sugere a origem dos
animais a partir de protistas.
c) De acordo com o cladograma, os porferos so considerados os
animais com caractersticas mais primitivas. Qual a principal
caracterstica que classifica esses organismos como primitivos?
d) Por muito tempo, os porferos no foram includos no Reino
Animal. Hoje se sabe que esses organismos apresentam
estruturas tpicas de animais. Cite uma caracterstica que inclui os porferos no Reino Animal e, ao mesmo tempo, os
exclui dos outros Reinos.

11. (UEG GO) Os celenterados so animais aquticos que apresentam formas diferentes, de acordo com o modo de
vida. Essas formas esto esquematizadas em corte longitudinal.

Sobre esse tema, responda ao que se pede.
a) Como so denominadas as formas em I e II? Cite uma caracterstica de
cada forma.
b) No corpo desses animais h a presena de clulas urticantes. Qual o
nome dessas clulas e sua funo?

12. (UFPR) O sistema circulatrio um importante sistema de integrao
entre as diferentes regies de um organismo, em funo do transporte de substncias entre essas regies. Em relao
ao sistema circulatrio, responda:

d.1) Como os cnidrios sobrevivem sem esse sistema?
d.2) Quais so as conseqncias, para os animais, da presena de um sistema circulatrio aberto? D um exemplo de
animal com esse tipo de sistema.
d.3) Quais foram as tendncias evolutivas relacionadas circulao e ao corao dos vertebrados?

13. (UERJ) BILOGOS DA UERJ DESVENDAM SEGREDOS DOS CORAIS DE BZIOS
Os moradores locais, preocupados com os danos que os barcos ancorados nas praias do balnerio, a poluio do mar
e a venda excessiva de corais no comrcio local poderiam trazer para a fauna marinha da regio, resolveram procurar
ajuda.
(Adaptado de O Globo, 24/09/2000)
Cite uma funo dos recifes de corais na preservao do ecossistema litorneo.

14. (UFRJ) O diagrama abaixo mostra o ciclo de vida do parasita Schistosoma mansoni.
Apesar de, no momento, existir tratamento quimioterpico para as pessoas infectadas e de estarem sendo
desenvolvidas vacinas protetoras, a rea endmica no Brasil est aumentando.
gua
O caramujo
lilbera cercrias
na gua doce
Caramujo hospedero
intetermedirio
Cercria
penetra na pele
do homem
O homem o
hospedeiro defi
nitivo
O parasita desenvolve-se
no homem e atinge a fase
a fmea do s. mansoni produz
ovos que migram para o intestino
Ovos na fezes atingem
a gua doce
Ovos se transformam
em miracdeos e atingem
o caramujo
gua

Examinando o diagrama, sugira um mtodo de controle da esquistossomose que no dependa da quimioterapia ou da
imunoprofilaxia.

205
15. (UMC SP) Em visita a uma pequena cidade do interior, Juvenal foi convencido pelos amigos a nadar em um
laguinho de uma fazenda. Enquanto se banhava nas guas, ele pde observar muitos exemplares de animais que
habitavam aquele lago, como peixes, caramujos e pequenos vermes aquticos. Aps o banho no lago, ele foi
acometido de uma forte coceira em diversas partes do corpo. Depois de alguns meses, Juvenal passou a apresentar
complicaes hepticas seguidas de um aumento considervel no volume de seu abdmen. Aps a realizao de uma
srie de exames mdicos, verificou-se que Juvenal se encontrava acometido de uma sria parasitose, provocada por
um platelminto, muito comum, ainda, no Brasil. Responda.
a) Qual a molstia que acomete Juvenal?
b) Ao retornar a sua casa em So Paulo, qual a possibilidade de que Juvenal transmita a molstia a seus familiares?
Justifique sua resposta.
c) Que medida deve ser tomada com relao ao lago onde Juvenal se banhou, para que outras pessoas no fiquem
doentes?

16. (UFAL) Em um jardim, Jlia via minhocas na terra. Ela ficou curiosa em saber mais sobre este animal.
a) Indique a Jlia duas caractersticas do filo ao qual as minhocas pertencem.
b) Explique a importncia desses animais do ponto de vista ecolgico.

17. (UERJ) Observe os esquemas abaixo, em que A representa a clula flama de um platelminto e B, o nefrdio de um
aneldeo.

a) Explique a funo da clula flama e descreva o papel desempenhado por seus clios.
b) Cite uma vantagem da disposio contorcida do tbulo dos nefrdios e explique a importncia da rica rede de
capilares sanguneos ao redor do tbulo.

18. (UNICAMP SP) As figuras abaixo mostram o crescimento corporal de dois grupos de invertebrados at atingirem a
fase adulta.

a) Identifique um grupo de invertebrados que pode ter o crescimento corporal como o representado na figura A e outro
como o representado na figura B. Justifique.
b) D duas caractersticas morfolgicas que permitam diferenciar entre si dois grupos de invertebrados relacionados
com o grfico A.

19. (UNESP SP) Observe a figura.

(Fernando Gonsales, Fliti. Modificado.)
Alguns inseticidas contm organofosforados e carbamatos, que inibem no organismo a ao da acetilcolinesterase,
enzima que degrada a acetilcolina. Aplicado na forma de aerossis, o produto se espalha melhor, atingindo um maior
nmero de indivduos.
Levado pelas traquias ou absorvido pela superfcie corporal dos insetos, o princpio ativo do inseticida chega aos
tecidos, onde exerce sua ao. Que tecido ou sistema fisiolgico alvo da ao do inseticida e por que esse sistema
entra em colapso, provocando a morte do inseto?


206
20. (UNESP SP) Um aracnologista ingls, Dr. W. S. Bristowe, calculou que o peso dos insetos consumidos por
aranhas anualmente, na Gr-Bretanha, excederia o peso da populao humana da ilha.
Duas caractersticas parecem contribuir para isso: produo de veneno e de seda.
(David Wise, Spiders in ecological webs, 1995)
Explique como uma dessas caractersticas pode contribuir para essa predao to eficiente. Que outro artrpode
tambm produz veneno?

21. (FUVEST SP) A figura mostra um artrpode.

a) A que grupo de artrpodes pertence esse animal? Cite uma caracterstica
observvel na figura e que permite chegar a essa concluso.
b) Em algumas espcies desse grupo, os machos cuidam dos ovos. Em
experimentos laboratoriais, quando fmeas foram colocadas em situao de
escolha de um macho para cpula, elas escolheram aqueles que estavam
cuidando de ovos. Qual seria a vantagem adaptativa desse comportamento de
escolha de machos cuidadores de ovos ?

22. (FUVEST SP) A revista PESQUISA FAPESP, de setembro de 2007, publicou
matria com o ttulo Memrias Pstumas, que destaca a importncia do
conhecimento sobre a ecologia e o desenvolvimento de diversas espcies de
insetos, para se desvendarem mortes misteriosas.
a) Como a ocorrncia de moscas e besouros, que se desenvolvem nos corpos em decomposio, pode ser til para
estimar o tempo decorrido desde a morte?
b) Os cientistas entrevistados para tal matria afirmam que os insetos podem revelar que, apesar de um corpo ter sido
encontrado no Rio de Janeiro, a morte no ocorreu nessa cidade. Como isso possvel?

23. (UFJF MG) A Entomologia Forense a cincia que estuda os insetos encontrados em corpos em decomposio
e auxilia as investigaes mdico-criminais, podendo determinar o intervalo de tempo entre a morte e a data em que o
cadver foi encontrado, a causa da morte, o local do crime, dentre outros fatores.
Insetos associados a corpos em decomposio podem ser ametbolos, hemimetbolos ou holometbolos. Considere a
tabela, a seguir, que apresenta a durao mdia (em dias) das diferentes fases de desenvolvimento dos insetos
encontrados em cadveres. Essa tabela utilizada para se determinar o tempo de decomposio de um corpo no
ambiente.
16 4 6 2 dias de Durao
ninfa pupa larva ovo mento desenvolvi de Fases

a) Imagine que voc precisa determinar o tempo de decomposio de dois corpos. No cadver X, foram encontradas
moscas-varejeiras adultas recm-produzidas e, no cadver Y, baratas adultas recm-produzidas. Considere que esses
insetos completaram seu ciclo de vida nesses cadveres. Determine o tempo mnimo (em dias) em que os cadveres X
e Y esto em decomposio. Justifique sua resposta.
b) O grfico abaixo representa duas populaes de insetos (populao 1 e populao 2) presentes em um mesmo
rgo de um corpo em decomposio, durante um determinado perodo de tempo. Que fenmeno biolgico est
representado na rea (A)? Explique sua resposta.

c) Os insetos representam a maior parte das espcies de animais que visitam corpos em decomposio. Esse fato
tambm observado quando se quantifica a fauna em um ambiente terrestre. Cite duas caractersticas
morfofisiolgicas que justificam a elevada diversidade apresentada pelos insetos.

TEXTO: 1 - Comum questo: 24

Texto 7

No ciberespao o sujeito libera-se das coeres da identidade, metamorfoseia-se, de forma provisria ou permanente,
no que ele quer, sem temer que o real o desminta. Sem rosto, no corre mais o risco sem poder ser visto, est livre de
toda responsabilidade, tendo agora apenas uma identidade voltil. No h mais o risco de ser trado ou reconhecido
por seu corpo. A rede favorece uma pluralidade de eus, o jogo libera-o de qualquer responsabilidade e favorece a
todo instante a possibilidade de desaparecer. A identidade uma sucesso de "eus" provisrios, um disco rgido que
contm uma srie de arquivos que podem ser acessados ao sabor das circunstncias. uma mscara formidvel, isto
, um estmulo ao relaxamento de toda civilidade. Toda responsabilidade desaparece. Um crime virtual no deixa
vestgios. O ciberespao instrumento da multiplicao de si, uma prtese da existncia.
NOVAES, A. O homem-mquina: a cincia manipula o corpo. So Paulo: Companhia das Letras, 2003. p. 130.

207
24. (UFG) O texto 7 da coletnea da prova de Redao menciona que No ciberespao o sujeito libera-se das
coeres da identidade, metamorfoseia-se, de forma provisria ou permanente, no que ele quer, sem temer que o real
o desminta.
A metamorfose, no contexto biolgico, um processo pelo qual algumas espcies passam durante seu ciclo de vida.
Sem temer que o real os desmintam: a borboleta vice-rei, de sabor palatvel, torna-se semelhante borboleta
monarca, que tem sabor ruim; j o camaleo modifica sua colorao de acordo com o ambiente.
Considerando os dois exemplos apresentados,
a) explique se eles correspondem ao processo de metamorfose;
b) descreva as vantagens dessas adaptaes para esses animais.

TEXTO: 2 - Comum questo: 25

A figura ilustra a diversidade em nmero de espcies de seres vivos, considerando a classificao em cinco Reinos de
Whittaker.



25. (UFBA) Justifique a maior diversidade de artrpodos entre os animais, relacionando duas aquisies que marcam
a evoluo do grupo a estratgias de sobrevivncia.

TEXTO: 3 - Comum s questes: 26, 27

A reproduo de muitas espcies de mariposas facilitada por substncias volteis lanadas no ar chamadas
feromnios. Tais substncias so produzidas por fmeas e capazes de atrair machos a milhares de metros de
distncia. Cada feromnio atrai machos da espcie da fmea que os produziu. A eficincia dos feromnios est
relacionada a grandes diferenas morfolgicas entre machos e fmeas (dimorfismo sexual), como mostram as figuras
abaixo. Armadilhas contendo feromnios so utilizadas para eliminar mariposas consideradas pragas das lavouras.



26. (UFRJ) Explique a importncia do dimorfismo sexual das antenas para a reproduo das mariposas.

27. (UFRJ) Explique por que o uso de armadilhas de feromnios considerado ambientalmente mais seguro para as
demais espcies de insetos silvestres do que o uso de inseticidas.





208








PROF.: Rmulo




C
C
o
o
n
n
t
t
e
e

d
d
o
o
s
s
:
:

Sistema Nervoso
Sistema Repiratrio









Ministrados nos meses Abril, Maio














D Da at ta a d da a E En nt tr re eg ga a : : _ __ __ __ __ __ __ __ _/ /_ __ __ __ __ __ __ __ __ __ __ _/ /2 20 01 13 3
LISTA 2 BIOLOGIA

209

01 - (UPE) Indique a alternativa que completa CORRETAMENTE as lacunas do texto abaixo.

Os atos reflexos so respostas __________ a um estmulo __________. O reflexo patelar
coordenado __________; portanto, um exemplo de reflexo __________. Mas o bulbo, a ponte e
__________ tambm so centros reflexos, que coordenam atos, como vmito, deglutio, piscar de
olhos, gritos de dor, riso e lgrimas, dentre outros.

a) involuntrias motor pelo crebro cerebral a medula espinhal
b) involuntrias sensorial pela medula medular o crebro
c) involuntrias sensorial pelo crebro cerebral a medula espinhal
d) voluntrias motor pelo crebro cerebral a medula espinhal
e) voluntrias sensorial pela medula medular o crebro
www.http://images.google.com.br
02 - (UECE) Os neurnios que conduzem o impulso do sistema nervoso central para o rgo que efetua a resposta, o
qual pode ser uma glndula, so do tipo
a) sensitivo.
b) associativo.
c) misto.
d) motor.

03 - (UEPB) O reflexo patelar um exemplo de ato reflexo, ou seja, de resposta do corpo humano a estmulos antes
que o encfalo tome conscincia destes.
Complete a descrio do reflexo patelar, de forma a torn-la coerente e, em seguida, assinale a alternativa que
apresenta a seqncia correta de preenchimento.
Um leve toque no joelho faz a perna levantar, sem que tomemos conscincia desse movimento. O toque estimula
________ de um ________, que transmite esse estmulo at a ________ da medula. Nessa regio, o estmulo
transformado em ________, que transmitida aos msculos atravs da ________ do mesmo nervo. Esse trajeto
percorrido pelo impulso nervoso forma o ________.

a) fibras sensitivas, nervo espinhal, substncia branca, ordem motora, fibra motora, ato reflexo.
b) fibras motoras, nervo misto, substncia branca, ordem motora, fibra sensitiva, arco reflexo.
c) fibras sensitivas, nervo espinhal, substncia cinzenta, ordem motora, fibra motora, arco reflexo.
d) fibras motoras, nervo motor, substncia cinzenta, ordem motora, fibra sensitiva, ato reflexo.
e) fibras motoras, nervo motor, substncia cinzenta, ordem motora, fibra sensitiva, arco reflexo.

04 - (UNIFOR CE) Quando uma pessoa encosta a mo em uma chaleira com gua fervente, ela reage imediatamente
por meio de um ato reflexo. Nesse caso, o neurnio efetuador leva o impulso nervoso para
a) o encfalo.
b) os msculos flexores do brao.
c) a medula espinhal.
d) as terminaes sensoriais de calor da mo.
e) as terminaes sensoriais de dor da mo.

05 - (UFAL) Explique o que homeostase e como ela controlada na espcie humana.

06 - (UFG) O texto que se segue foi extrado de Xadrez, truco e outras guerra, de Jos Roberto Torero. Servimo-nos
de algumas de suas estruturas, para introduzir as questes desta prova.
Os abutres, sbios animais que se alimentavam do mais farto dos pastos, j comeavam a sobrevoar a ala dos
estropiados quando o General mandou que acampassem.
Naquela tarde assaram trinta bois, quantidade nfima para abastecer os homens que ainda sobravam... O plano dos
comandantes era assaltar fazendas da regio e tomar-lhes o gado...
noite a rao foi ainda mais escassa, e, para enganar a fome, fizeram-se fogueiras para assar as ltimas batatas e
umas poucas razes colhidas pelo caminho. Como o frio tambm aumentava, surgiu um impasse: quem ficaria perto do
fogo: os colricos, que logo morreriam, ou os sos, que precisavam recuperar as foras para a luta?
noite a rao foi ainda mais escassa, e, para enganar a fome, fizeram-se fogueiras (...)
Uma rao adequada pode evitar a fome e fornecer matriaprima para o metabolismo corporal.
a) Relacione fome e sistema nervoso central.
b) Diferencie anabolismo de catabolismo.
Caderno de Atividades

Disciplina:
B Bi io ol lo og gi ia a

Professor(a):
Rmulo
Aluno:
3 ano
Ensino Mdio
Data de Recebimento:
_____/_____/_____
Lista 02
Data Entrega:
_____/_____/_____

210
07 - (UFRN) Um motorista infrator, ao dirigir, na Via Costeira, em alta velocidade, perdeu o controle do carro numa
curva, sofrendo um acidente. Ao chegar ao pronto-socorro, diagnosticou-se uma isquemia cerebral (bloqueio da
circulao nas artrias que fornecem sangue ao encfalo) no lobo frontal do crebro.
Como conseqncia, poder haver comprometimento da capacidade do motorista para
a) piscar sob o estmulo de uma luz intensa.
b) salivar ao sentir o aroma de uma comida.
c) preencher uma ficha de identificao.
d) sentir dor ao encostar num ferro quente.

08 - (UNESP SP) O dilogo entre os neurnios extrapola os limites que se supunha at hoje e se estende a neurnios
vizinhos, revelou um estudo sobre complexos processos que ocorrem no crebro dos mamferos publicado nesta
quinta-feira na revista cientfica britnica Nature.
(www.noticias.terra.com.br/ciencia. 19.12.2007. Acessado em 08.03.2009)

A que processo o termo dilogo se refere? Explique como ele ocorre.

09 - (UFU MG) Um pesquisador realizou um experimento para verificar a influncia da bainha de mielina na velocidade
de conduo do impulso nervoso nas fibras nervosas. O grfico a seguir representa os resultados obtidos nos dois
tipos diferentes de fibras nervosas analisadas,
denominadas Tipo 1 e Tipo 2.

Com base no grfico, responda as questes
propostas.
a) Quanto presena de mielina, como se
classificam
as fibras de Tipo 1 e Tipo 2?
b) A velocidade de conduo maior em que tipo de
fibra? Por qu?


10 - (UFF RJ) A figura abaixo mostra as regies de um neurnio.

a) Nomeie as estruturas indicadas.
b) Como denominada a estrutura envolvida na sinapse entre o
neurnio e uma clula do msculo estriado esqueltico? Que
neurotransmissor est geralmente envolvido nesta sinapse e
como a clula muscular responde ao seu estmulo?
c) Como esto organizadas as cargas eltricas e a concentrao de
ons nas superfcies interna e externa de uma membrana
polarizada e de uma membrana despolarizada do axnio de um
neurnio? Justifique.
d) A tabela abaixo apresenta a relao entre a velocidade do impulso
nervoso e o dimetro de dois tipos de fibras nervosas
denominadas A e B. Analise a tabela e identifique qual grupo de fibras (A ou B) apresenta bainha de mielina.
Justifique.


11 - (UNICAMP SP) Com a manchete O Vo de Maurren, O Estado de So Paulo noticiou, no dia 23 de agosto de
2008, que a saltadora Maurren Maggi ganhou a segunda medalha de ouro para o Brasil nos ltimos Jogos Olmpicos.
No salto de 7,04m de distncia, Maurren utilizou a fora originada da contrao do tecido muscular estriado
esqueltico. Para que pudesse chegar a essa marca, foi preciso contrao muscular e coordenao dos movimentos
por meio de impulsos nervosos.
a) Explique como o neurnio transmite o impulso nervoso ao msculo.
b) Para saltar, necessria a integrao das estruturas sseas (esqueleto) com os tendes e os msculos.

Explique como ocorre a integrao dessas trs estruturas para propiciar atleta a execuo do salto.

211
12 - (UFU MG) Curare o nome dado a alguns compostos orgnicos venenosos extrados de plantas. Aps entrar
em contato com um curare, um indivduo morreu por parada respiratria.
Sabendo-se que um msculo age por estmulo de um nervo motor e que tal veneno induz paralisia muscular, responda:

a) Como o curare impede a propagao do impulso nervoso?
b) Ao impedir a propagao do impulso nervoso, onde o curare age?
c) Se o nervo motor (que forma a placa motora nos msculos respiratrios) fosse estimulado com mais intensidade,
esse nervo reagiria de maneira mais intensa? Por qu?

13 - (UERJ) Todas as clulas do organismo humano possuem uma diferena de potencial eltrico entre as faces
interna e externa da membrana plasmtica. Nas clulas nervosas, essa diferena denominada potencial de repouso,
pois um estmulo capaz de desencadear uma fase de despolarizao seguida de outra de repolarizao; aps isso, a
situao de repouso se restabelece. A alterao de polaridade na membrana dessas clulas chamada de potencial
de ao que, repetindo-se ao longo dos axnios, forma o mecanismo responsvel pela propagao do impulso
nervoso.
O grfico abaixo mostra a formao do potencial de ao.


Descreva as alteraes inicas ocorridas no local do estmulo responsveis pelos processos de despolarizao e
repolarizao da membrana dos neurnios.

14 - (UERJ) Nas sinapses neuromusculares, o impulso nervoso, ao atingir a terminao do nervo motor, promove a
liberao, no espao sinptico, do mediador qumico acetilcolina. Esse mediador acopla-se, ento, a seu receptor
especfico na membrana da clula muscular, provocando despolarizao dessa membrana, o que dispara o processo
de contrao muscular.
O medicamento atropina liga-se ao receptor da acetilcolina, impedindo que esse mediador atue na despolarizao da
membrana da clula muscular.
Em um msculo, cujo nervo motor foi continuamente estimulado, mediu-se o efeito da atropina sobre a concentrao
de Ca
++
no citosol das clulas musculares.
No grfico abaixo, uma das curvas representa as variaes da concentrao de Ca
++
no citosol antes e aps a adio
de atropina.


Indique a curva que mostra o efeito da atropina e descreva o papel do Ca
++
na contrao muscular.

15 - (UNIFESP SP) A tabela mostra os efeitos da ao de dois importantes componentes do sistema nervoso
humano.

a) A que correspondem X e Y?
b) Em uma situao de emergncia, como a fuga de um
assalto, por exemplo, qual deles ser ativado de maneira
mais imediata? Fornea um outro exemplo, diferente dos
da tabela, da ao desse componente do sistema nervoso.



ejaculao da Promoo pnis do ereo Estmulo
urinria bexiga da o Relaxament urinria bexiga da Contrao
intestino e estmago do Inibio
intestinos
dos e estmago do Estmulo
salivao da Inibio salivao da Estmulo
pupila da Dilatao pupila da Contrao
Y X

212
16 - (EFOA MG) Imagine a seguinte situao: ao entrar na sala para fazer a prova de vestibular, o fiscal lhe solicita o
documento comprobatrio de inscrio e voc verifica que o esqueceu em casa. Nesse momento, seu organismo pode
passar por diversas transformaes de alerta que ilustram bem como o corpo se prepara para enfrentar as situaes
de emergncia ou de perigo. Com relao a estas transformaes, responda:
a) Qual das divises do sistema nervoso autnomo responde imediatamente nesta situao?
b) Qual o nome do principal mediador qumico liberado imediatamente nas sinapses?
c) Qual o nome do outro sistema orgnico que diretamente responsvel pela palidez facial e mos geladas,
caractersticas nesses episdios de susto?
d) Qual o nome especfico da glndula secretora hormonal que potencializa a ao desta estimulao nervosa?
e) Qual a participao fisiolgica efetiva do fgado nestes episdios de emergncia?

17 - (UNESP SP) ... Joo, com o sobrenome de Limeira, agrediu e insultou a moa, irritado naturalmente com os seus
desdns. Martinha recolheu-se casa. Nova agresso, porta. Martinha, indignada, mas ainda prudente, disse ao
importuno: No se aproxime, que eu lhe furo. Joo Limeira aproximou-se, ela deu-lhe uma punhalada, que o matou
instantaneamente.
(Machado de Assis. O punhal de Martinha, 1894.)

Perfuraes no trax, provocadas por objetos pontiagudos como facas e punhais, ainda que no atinjam qualquer
rgo vital, se permanecerem abertas podem matar o sujeito por asfixia. Explique por que isso pode ocorrer.

18 - (UNESP SP) Observe a figura.


(http://images.google.com.br/. Adaptado.)

A figura sugere que as rvores, e por implicao a floresta amaznica, representam o pulmo do mundo e seriam
responsveis pela maior parte do oxignio que respiramos. No que se refere troca de gases com a atmosfera,
podemos dizer que as rvores tm funo anloga do pulmo dos vertebrados e so produtoras da maior parte do
oxignio que respiramos? Justifique sua resposta.

19 - (UNESP SP) (...) a Fifa decidiu ratificar a proibio de jogos internacionais em estdios localizados em altitudes
acima de 2 750 metros. Para a Fifa, partidas internacionais acima desta altitude sero disputadas apenas aps um
perodo mnimo de adaptao de uma semana para os atletas. No caso de um jogo a mais de 3 mil metros do nvel do
mar, este perodo de aclimatao sobe para pelo menos duas semanas.
(www.globoesporte.globo.com. Acessado em 15.03.2008.)

Fifa suspende a proibio de partidas em altas altitudes, atendendo presso dos pases sul-americanos () O
presidente da entidade anunciou que a suspenso da proibio temporria.
(www.esporte.uol.com.br/futebol. Acessado em 25.05.2008.)

Com base nos conhecimentos sobre circulao e respirao humanas, justifique a posio anterior da Fifa que
permitiria que jogos de futebol fossem realizados em locais de elevada altitude apenas aps um perodo de adaptao
dos atletas.

20 - (UNICAMP SP) As macromolculas (polissacardeos, protenas ou lipdios) ingeridas na alimentao no podem
ser diretamente usadas na produo de energia pela clula. Essas macromolculas devem sofrer digesto (quebra),
produzindo molculas menores, para serem utilizadas no processo de respirao celular.
a) Quais so as molculas menores que se originam da digesto das macromolculas citadas no texto?
b) Como ocorre a quebra qumica das macromolculas ingeridas?
c) Respirao um termo aplicado a dois processos distintos, porm intimamente relacionados, que ocorrem no
organismo em nvel pulmonar e celular. Explique que relao existe entre os dois processos.


213
21 - (UFMG)
1. A demanda de O
2
de um animal em repouso depende de seu metabolismo basal. Em funo disso, a rea disponvel
para a realizao de trocas gasosas proporcional massa corporal de cada indivduo.
Analise, neste grfico, as curvas que representam a relao entre a massa corporal e a superfcie disponvel para
trocas gasosas de diferentes grupos de vertebrados:


Com base nessas informaes e em outros conhecimentos sobre o assunto, TRACE, no grfico acima, uma curva para
representar a mesma relao para os peixes. JUSTIFIQUE sua opo pelo traado feito.
2. No que diz respeito ao processo respiratrio dos animais, define-se ventilao como o volume de ar ou de gua que
flui, respectivamente, pelos pulmes ou pelas guelras no intervalo de 1 minuto.

Analise estes grficos:


A partir dessa anlise e considerando outros conhecimentos sobre o assunto,
a) D um ttulo nico para os dois grficos.
b) Considerando os dados representados nas curvas dos grficos I e II, ELABORE uma concluso em que se
agrupem esses dados.

22 - (UFU MG) A respirao um fenmeno complexo de vital importncia para a obteno de energia. Acerca desse
assunto, faa o que se pede.
a) em relao ao processo de respirao, recomenda-se no deixar o carro com o motor ligado, em locais fechados,
devido produo de um gs nocivo e inodoro, que forma com a hemoglobina um composto estvel. Qual esse gs?
b) quais os tipos de respirao os animais desenvolveram, de acordo com a diversidade encontrada no Reino Animal
(diferentes Filos) e com o ambiente em que os mesmos vivem?
c) cite os tipos de respirao encontrados nos anfbios, peixes e rpteis.

23 - (UERJ) O CO
2
produzido pelo metabolismo dos tecidos , em grande parte,
transportado no sangue sob a forma de bicarbonato e de cido carbnico. Em
condies normais, tais compostos encontram-se na seguinte
proporo:
1
20
CO H
HCO

3 2
-
3
=
Esse sistema de transporte, cuja parte alcalina corresponde ao

3
HCO e a
parte cida ao H
2
CO
3
, constitui o principal mecanismo de manuteno do pH
do sangue em 7,4.
Algumas situaes, como prender a respirao por tempo prolongado, podem
alterar a taxa normal desses dois compostos no plasma, conforme se observa no
grfico abaixo:
Indique a alterao observada no pH do sangue quando a respirao suspensa por tempo prolongado.
Justifique sua resposta, utilizando as informaes do grfico.

214
TEXTO: 1 - Comum questo: 24




24 - (UESC BA) A comunicao entre os personagens representados na histria em quadrinhos, embora
distintivamente humanos, se fundamenta em mecanismos bsicos que envolvem a fisiologia do sistema nervoso, como
01. a manuteno do equilbrio entre cargas eltricas positivas e negativas ao nvel das superfcies externa e interna
da membrana do neurnio, no decorrer da transmisso do impulso nervoso.
02. a suscetibilidade de clulas especializadas que convertem, nos prprios rgos sensoriais, o estmulo em
sensao consciente.
03. a dinmica das redes neuronais ser dependente da liberao de neurotransmissores pelas terminaes do axnio.
04. a possibilidade de respostas rpidas e inteligentes pelo sistema nervoso autnomo.
05. a comunicao entre neurnios adjacentes estabelecer-se por contatos de natureza mecnica.













215









PROF.: Rmulo




C
C
o
o
n
n
t
t
e
e

d
d
o
o
s
s
:
:

Sistema Digestrio
Sistema Circulatrio






















D Da at ta a d da a E En nt tr re eg ga a : : _ __ __ __ __ __ __ __ _/ /_ __ __ __ __ __ __ __ __ __ __ _/ /2 20 01 13 3
LISTA 3 BIOLOGIA

216

01 - (UERJ) A bile, secretada pelo fgado e armazenada pela vescula biliar, faz parte do processo de digesto de
alimentos em seres humanos.
Cite o tipo de alimento sobre o qual a bile age e a enzima que o digere. Em seguida, explique como a bile facilita a
ao dessa enzima.

02 - (UNESP SP) Ao comermos uma fatia de po, a ptialina (ou amilase salivar) presente na saliva inicia a digesto
do amido contido no po. Na nossa boca, o pH situa-se ao redor de 7, pH timo para ao da ptialina. Contudo, ao
chegar ao estmago, esse alimento envolvido pelo suco gstrico, de pH ao redor de 2, que inibe a ao da ptialina e
impede o prosseguimento da digesto do amido nesse local.
O que acontece com o amido a partir do estmago, at chegar ao nosso sangue?

03 - (UFRRJ) Quando voc ingere um alimento, leva algum tempo para que ele percorra o tubo digestivo e seja
digerido completamente. Durante a digesto, vrias enzimas e sucos digestivos atuam nas transformaes que sofre o
alimento. Sabe-se que nem todo amido digerido na boca, porm a ptialina no atua em outros rgos, mesmo
quando misturada no bolo alimentar; o mesmo ocorre com as enzimas do suco gstrico e do suco pancretico, que,
tambm, s atuam, respectivamente, no estmago e no intestino delgado.


Explique por que a ptialina, o suco gstrico e o suco entrico no atuam nos outros rgos.

04 - (UNICAMP SP) Ao ingerirmos alimentos, o trato digestrio secreta enzimas digestivas e outras secrees de
acordo com a caracterstica qumica desses alimentos. Foram analisadas as diferentes secrees encontradas ao
longo do trato digestrio de 3 grupos de indivduos. Cada grupo foi submetido separadamente a dietas ricas em
gorduras, ou em carboidratos, ou em protenas. Os resultados esto mostrados na tabela abaixo.



Caderno de Atividades

Disciplina:
B Bi io ol lo og gi ia a

Professor(a):
Rmulo
Aluno:
3 ano
Ensino Mdio
Data de Recebimento:
_____/_____/_____
Lista 03
Data Entrega:
_____/_____/_____

217
a) Indique o tipo de alimento ingerido pelo grupo 1 e o tipo ingerido pelo grupo 2. Explique por que na digesto do
alimento do grupo 1 no foram secretadas as mesmas enzimas secretadas pelos indivduos do grupo 2.
b) Qual a relao entre a secreo heptica e a secreo pancretica na digesto do alimento ingerido pelo grupo 3?

05 - (UFC CE) A Revista Veja (20 de dezembro de 2006) trouxe, como reportagem, o que chamou de A LTIMA
EVOLUO. Essa diz respeito a uma mutao gentica que permitiu o consumo de leite entre pessoas adultas, as
quais, normalmente, deveriam apresentar intolerncia lactose. Os itens a seguir abordam aspectos importantes
ligados a essa reportagem. Leia-os com ateno e responda as solicitaes.
a) A capacidade de digerir a lactose vai desaparecendo medida que cessa a amamentao. O que impede a
digesto da lactose entre indivduos adultos?
b) A reportagem cita que a tolerncia lactose varia entre as populaes. Entre os brasileiros, chega a 50%. J 99%
dos chineses so intolerantes, contra apenas 1% dos suecos. Considerando uma condio hipottica, onde a dieta
disponvel est baseada apenas no leite bovino, qual processamento o leite deveria sofrer para que os chineses
pudessem utilizar esse alimento?
c) Levando em conta que o leite bovino um alimento rico em muitos nutrientes, cite dois macronutrientes e dois
micronutrientes presentes no leite, essenciais para a nutrio humana. Macronutriente: 1.
Macronutriente 2.
Micronutriente: 1.
Micronutriente: 2.
d) Os cientistas que comandaram a pesquisa queriam saber se a mutao da tolerncia lactose presente nos
europeus existia tambm entre povos pastoris de outros lugares. Observaram que povos da Tanznia, do Qunia e
do Sudo possuem tambm a capacidade de digerir o leite. Entretanto, essa capacidade foi propiciada por trs
mutaes genticas distintas e independentes entre si. Qual a denominao do fenmeno que explica esse fato?

06 - (UFPA) A ressaca uma das conseqncias indesejveis do consumo excessivo de bebidas alcolicas. Uma
prtica comum para cur-la tomar tacac, alimento que contm como um dos seus ingredientes a goma (amido da
mandioca). Considerando que a hipoglicemia uma das causas do mal-estar provocado pelo excesso de lcool,
descreva o processo de digesto qumica do amido, justificando sua importncia na diminuio desse referido mal-
estar.

07 - (UEG GO) Quando os alimentos passam para o esfago, uma estrutura de cartilagem fecha a traquia. Com a
idade, a perda progressiva do tnus muscular leva a um fechamento incompleto, aumentando o risco da passagem de
alimentos para a traquia.
Qual o nome dessa estrutura e a justificativa anatmica para a sua existncia?

08 - (UFRJ) A Figura 1 a seguir mostra as vilosidades do intestino de uma serpente aps um longo perodo de jejum,
enquanto a Figura 2 mostra a mesma regio minutos aps a ingesto de alimentos.



Essa rpida alterao nas vilosidades causada por um intenso aumento da irrigao sangnea na poro interna
dessas estruturas. Tal mudana aps a alimentao importante para o aumento da eficincia do processo de nutrio
das serpentes.
Por que a alterao nas vilosidades contribui para a eficincia da nutrio das
serpentes? Justifique sua resposta.

09 - (UFG) O processo de digesto fundamental para a transformao dos
alimentos em molculas menores que podem ser absorvidas e utilizadas para a
obteno da energia necessria s funes vitais.
a) De acordo com a figura abaixo, identifique e nomeie as estruturas responsveis pela
absoro de lipdios e apresente dois fatores que o tornam possvel nessas partes do
sistema digestrio.

b) Descreva a atuao de duas enzimas sobre a digesto de protenas e de
carboidratos, respectivamente.





218
10 - (UFF RJ) Para estudar a ao de agentes estimulantes da secreo excrina do pncreas, foram introduzidos
diretamente no duodeno de uma pessoa em jejum, alguns mililitros de leo de milho. Em outra pessoa, nas mesmas
condies, o leo foi substitudo por alguns mililitros de uma soluo de HCl ajustada a pH 2,0. Em cada caso, foi
coletada uma amostra do suco pancretico produzido. Os grficos I e II abaixo apresentam os resultados das anlises
de componentes dessas amostras.




a) Identifique os grficos que correspondem, respectivamente, aos resultados obtidos aps a introduo do leo de
milho e da soluo de HCl. Descreva o mecanismo de estimulao da secreo excrina do pncreas, em cada
caso.
b) Em qual das duas situaes h, tambm, um aumento na liberao de bile no duodeno? Justifique sua resposta.

11 - (FUVEST SP) Uma enzima, extrada da secreo de um rgo abdominal de um co, foi purificada, dissolvida em
uma soluo fisiolgica com pH 8 e distribuda em seis tubos de ensaio. Nos tubos 2, 4 e 6, foi adicionado cido
clordrico (HCl), de modo a se obter um pH final em torno de 2. Nos tubos 1 e 2, foi adicionado macarro; nos tubos 3 e
4, foi adicionada carne; nos tubos 5 e 6, foi adicionada manteiga. Os tubos foram mantidos por duas horas
temperatura de 36C. Ocorreu digesto apenas no tubo 1.



a) Qual foi o rgo do animal utilizado na experincia?
b) Que alterao esperada na composio qumica da urina de um co que teve esse rgo removido
cirurgicamente? Por qu?
c) Qual foi a substncia que a enzima purificada digeriu?

12 - (UFSCar SP) Considere os seguintes componentes do sistema digestrio humano, em ordem alfabtica: nus,
boca, esfago, estmago, fgado, glndulas salivares, intestino delgado, intestino grosso e pncreas.
a) Durante seu trajeto pelo sistema digestrio, o alimento passa pelo interior de quais desses componentes e em que
seqncia?
b) De que modo o fgado participa da digesto dos alimentos?

219
13 - (UNICAMP SP) O grfico abaixo representa as atividades de duas enzimas do sistema digestrio humano,
avaliadas a 37 C (condies normais de temperatura corprea).



a) Qual o local de atuao da enzima A? Justifique.
b) Cite uma enzima digestiva que apresente o padro de atividade da enzima B e seu local de atuao.
c) Explique o que ocorreria com a atividade enzimtica se, experimentalmente, a temperatura fosse pouco a pouco
aumentada at atingir 60 C.

14 - (EFOA MG) Observe o esquema de parte do tubo digestivo e de alguns anexos, de um homem normal, indicados
(I, II, III e IV) abaixo. Considere tambm, para responder aos itens seguintes, que trs tipos de alimentos, um rico em
protenas, outro em lipdeos e outro em carboidratos, passaram pelo processo de mastigao, formaram o bolo
alimentar e esto em processo de digesto.

I
II
III
IV


a) Ordene os alimentos citados de acordo com a seqncia normal da digesto enzimtica inicial de cada um ao
longo do tubo digestivo:
b) Qual das partes indicadas secreta enzimas que participam diretamente da digesto destes trs tipos de alimentos?
c) Parte do processo da digesto de um desses alimentos citados depende de uma enzima que ativada em pH
prximo de 2. Cite o nome da enzima inativa e tambm da forma ativa:
d) Cite um exemplo especfico de funo endcrina apresentada pelo rgo indicado pelo nmero IV:

15 - (UEPB) Com o slogan Bode Rei - Ano XI - Faa chuva ou faa sol, em Cabaceiras, o bode no perde a
majestade, Cabaceiras, conhecida como a Rolide Nordestina, apresenta a XI Festa do Bode Rei - Festival de
Caprinos e Ovinos da Paraba, com o objetivo de congregar todos os agentes produtivos da cadeia da
caprinovinocultura. A festa acontece ao longo da avenida principal da cidade, onde o turista poder tornar-se um
exmio entendedor do universo bodstico. Desde a identificao da grande variedade de raas nacionais e
internacionais, at degustar pratos tpicos da culinria bodstica como buchada e picado,
pizza de bode, bodioca e linguia de bode. A buchada um dos pratos mais tradicionais do
evento feito com as vsceras do bode. Acerca dos componentes deste prato, assinale a
alternativa correta:

a) As tripas correspondem ao intestino do animal. Nele ocorre a digesto e absoro
apenas de protenas.
b) No rume ocorre a digesto dos carboidratos.
c) O bucho onde so colocados os midos corresponde ao estmago do bode, o qual
divide-se em rume, barrete, folhoso e coagulador.
d) A lngua desempenha a funo de mastigao e produo de amilase salivar.
e) O fgado uma glndula anexa ao sistema digestrio que lana sua secreo no
intestino grosso.






220
16 - (UNIFESP SP) Um estudante levantou a hiptese de que a digesto do alimento no sistema digestrio de um
aneldeo ocorre na mesma seqncia que em um ser humano. Para isso, analisou o contedo do trato digestrio do
aneldeo, segmento por segmento, medida que a digesto progredia, e encontrou o seguinte resultado:



a) Com base nos dados obtidos, a hiptese do estudante deve ser aceita ou rejeitada? Justifique.
b) Aps o final da digesto, que tipo de sistema promover o transporte dos nutrientes at as clulas do aneldeo?
Explique.

17 - (UEM PR) Sobre o sistema digestrio de anfbios, de rpteis, de aves e de mamferos, responda s questes
propostas.
a) Dos grupos nominados, cite o(s) que apresenta(m) cloaca.
b) Na espcie humana, bem como em todos os grupos nominados, o intestino delgado recebe secrees de quais
glndulas anexas?
c) Nas aves, como dividido o estmago? Qual a funo de
cada parte?

18 - (FUVEST SP) O esquema abaixo representa o corao de um
mamfero.

Indique, com os nmeros correspondentes,

a) as cmaras do corao em que o sangue apresenta maior
concentrao de gs carbnico;
b) as cmaras do corao s quais chega sangue trazido por
vasos;
c) o vaso que sai do corao com sangue venoso;
d) a cmara da qual o sangue arterial sai do corao.

19 - (UNIFESP SP) Acidentes cardiovasculares esto entre as doenas que mais causam mortes no mundo. H uma
intricada relao de fatores, incluindo os hereditrios e os ambientais, que se conjugam como fatores de riscos.
Considerando os estudos epidemiolgicos at agora desenvolvidos, altas taxas de colesterol no sangue aumentam o
risco de infarto do miocrdio.
a) Em que consiste o infarto do miocrdio e qual a relao entre altas taxas de colesterol e esse tipo de acidente
cardiovascular?
b) Considerando a relao entre os gases O
2
e CO
2
e o processo de liberao de energia em nvel celular, explique o
que ocorre nas clulas do miocrdio em uma situao de infarto.

20 - (UFF RJ) Noel Rosa, um dos maiores compositores da msica brasileira, chegou a iniciar os estudos de
Medicina, abandonando-os meses depois. Naquele perodo, escreveu os primeiros versos da msica Corao.
Depois que ele gravou a msica, os colegas da Faculdade chamaram a ateno para as descries equivocadas sobre
as funes do corao. (adaptado do site do Conselho Regional de Medicina do Estado de So Paulo, 2006)

Corao,
Grande rgo propulsor,
Transformador do sangue
Venoso em arterial;

Corao,
No s sentimental,
Mas, entretanto, dizem
Que s o cofre da paixo.

a) Identifique o erro conceitual contido na primeira estrofe dessa msica.
b) Relacione o sangue venoso e o arterial com cada um dos compartimentos do corao humano.
c) Especifique onde e como ocorre o processo de hematose no organismo humano.

221
21 - (UFRRJ) Sabemos que a regulao da temperatura corporal um processo altamente dispendioso do ponto de
vista energtico. Parte da energia originada nas mitocndrias atravs da respirao celular destinada manuteno
da temperatura corporal. Somente os mamferos e as aves tm mecanismos de regulao interna da temperatura
corporal. Coincidentemente, estes so, tambm, os nicos grupos que apresentam o corao formado por quatro
cavidades, onde nem os trios nem os ventrculos se comunicam entre si.
Relacione os fatos citados acima com a capacidade de regulao da temperatura corporal nos mamferos e nas aves.

22 - (EFOA MG) O transporte de substncias como nutrientes, gases e resduos um processo fisiolgico fundamental
para manter o equilbrio homeosttico do organismo. Considerando este processo, cite:
a) dois filos de animais metazorios que no apresentam sistema circulatrio.
b) o nome especfico da cavidade cardaca que envia o sangue venoso aos pulmes, nos tetracavitrios.
c) um exemplo de organismo que apresenta circulao dupla e incompleta.
d) um dos mecanismos que permitem ao sangue retornar ao corao, mesmo contra a gravidade.
e) o avano evolutivo anatmico no corao dos rpteis em geral, em relao ao dos anfbios.

23 - (UDESC SC) Associe a primeira coluna com a segunda quanto circulao nos vertebrados.
( 1 ) circulao completa
( 2 ) distole
( 3 ) circulao incompleta
( 4 ) sangue arterial
( 5 ) lado direito do corao
( 6 ) sstole

( ) relaxamento do corao
( ) anfbios
( ) sangue venoso
( ) contrao do corao
( ) peixe
( ) aorta

Assinale a alternativa que contm a sequncia correta, de cima para baixo.

a) 2 3 4 6 5 1
b) 6 3 5 4 2 1
c) 2 1 6 5 3 4
d) 2 3 5 6 1 4
e) 6 1 5 2 3 4

24 - (UECE) Os nicos vertebrados que, na vida adulta, apresentam somente sangue venoso no corao so:
a) Peixes.
b) Aves.
c) Anfbios.
d) Rpteis.






















222









PROF.:Rmulo




C
C
o
o
n
n
t
t
e
e

d
d
o
o
s
s
:
:

Sistema Excertor
Sistema Endcrino




Ministrados nos meses Agosto















D Da at ta a d da a E En nt tr re eg ga a : : _ __ __ __ __ __ __ __ _/ /_ __ __ __ __ __ __ __ __ __ __ _/ /2 20 01 13 3
LISTA 4 BIOLOGIA

223

01 - (UERJ) No homem, a filtrao do sangue pelos glomrulos renais produz cerca de 7,2 L de filtrado glomerular por
hora. Esse volume sofre uma significativa reduo durante a passagem do filtrado pelos tbulos contornados
proximais, que possuem um eficiente mecanismo de reabsoro ativa de sdio.
No esquema abaixo, esto representadas clulas epiteliais do tbulo contornado proximal. Observe que a bomba de
Na
+
e K
+
e os canais passivos de Na
+
esto situados em faces diferentes da membrana plasmtica.



Descreva, com base no esquema, as etapas do mecanismo pelo qual o Na
+
filtrado absorvido e retorna ao meio
interno.

02 - (UFRJ) A passagem de gua atravs da membrana plasmtica se d principalmente por canais proteicos
especficos denominados aquaporinas. A vasopressina, tambm conhecida como ADH, regula a diurese (produo de
urina) nas diversas situaes fisiolgicas, alterando a quantidade de aquaporinas na membrana das clulas do tbulo
renal responsveis pela reabsoro de gua.

A tabela a seguir mostra as concentraes normais de alguns solutos no plasma e as respectivas concentraes
apresentadas por um paciente com diarria.



Determine se a quantidade de aquaporinas na membrana plasmtica das clulas dos tbulos renais do paciente,
considerando os padres mais regulares, deve estar maior ou menor do que a de um indivduo normal. Justifique sua
resposta.

03 - (UFTM MG) Assim que chegaram lanchonete, Pedro, Flvio e Rodrigo foram ao banheiro e urinaram. mesa,
Pedro pediu uma garrafa de gua mineral; Flvio preferiu bebida alcolica e tomou duas latas de cerveja; Rodrigo
tomou duas latas de refrigerante. Antes de chegarem lanchonete, os trs amigos no haviam bebido nada e l
ingeriram o mesmo volume de lquido. Aps 1 hora de conversa, durante a qual cada um tomou apenas a sua bebida,
foram novamente ao banheiro e urinaram.
a) Sabendo-se que os trs amigos gozavam de boa sade, qual deles urinou maior volume na segunda ida ao
banheiro? Justifique.
b) Se, alm da bebida, cada um deles tivesse comido uma poro de amendoim salgado, o volume da urina seria o
mesmo? Justifique.



Caderno de Atividades

Disciplina:
B Bi io ol lo og gi ia a

Professor(a):
Rmulo
Aluno:
3 ano
Ensino Mdio
Data de Recebimento:
_____/_____/_____
Lista 04
Data Entrega:
_____/_____/_____

224
04 - (UFG) Os rins mantm o equilbrio hdrico no corpo por meio da regulao da quantidade e dos componentes do
lquido dentro e fora das clulas. Quaisquer distrbios dos canais de gua nos nfrons, ou do hormnio antidiurtico
(ADH), podem levar a doenas, como a desidratao. O grfico abaixo representa duas situaes diferentes, em que
as duas curvas se sobrepem at a metade da poro D do nfron.



Com base nas informaes acima,
a) explique qual curva poderia representar uma pessoa com desidratao;
b) quais so as partes do nfron onde o ADH atua?

05 - (UFRN) Em um ser humano, os glomrulos chegam a produzir 180 L de filtrado por dia, mas o volume de urina
excretado de apenas 1,5 L. Alm disso, no ser humano, a concentrao de substncias no filtrado pode ser bastante
diferente da concentrao na urina. A urina de um indivduo saudvel tem concentrao de glicose igual a zero,
enquanto que a urina de um indivduo diabtico pode apresentar concentraes elevadas de glicose.
a) Explique por que grande a diferena entre o volume filtrado e o volume excretado, citando as estruturas do
nfron responsveis por essa diferena.
b) Justifique as diferenas existentes entre indivduos saudveis e diabticos quanto s concentraes de glicose na
urina.

06 - (UFG) Os fluidos biolgicos sofrem mltiplas influncias que alteram seu teor e composio, constantemente, e
so corrigidos por processos homeostticos.
a) Considerando a urina como um produto da homeostase animal, explique a influncia das bebidas alcolicas sobre
a produo de urina, no homem.
b) Explique a manuteno do volume constante do sangue circulante, apesar da ingesto de grande quantidade de
lquidos.

07 - (UFRJ) Quando se faz um esforo fsico prolongado ou quando a temperatura ambiente est alta, a sudorese
(produo de suor) ajuda a baixar a temperatura do corpo. Nessas circunstncias, comum a urina se tornar mais
concentrada.
Explique a relao fisiolgica entre esses fatos.

08 - (UNIR RO) Peixes, anfbios e mamferos excretam, respectivamente,
a) amnia, uria, cido rico.
b) amnia, uria, uria.
c) cido rico, uria, uria.
d) uria, uria, cido rico.
e) cido rico, uria, amnia.

09 - (UNICAMP SP) Na tabela abaixo so apresentados os resultados das anlises realizadas para identificar as
substncias excretadas por girinos, sapos e pombos.


a) Identifique, na tabela, qual amostra corresponde s substncias excretadas por pombos. Explique a vantagem
desse tipo de excreo para as aves.
b) Identifique, na tabela, qual amostra corresponde s substncias excretadas por girinos e qual corresponde s dos
sapos. Explique a relao entre o tipo de substncia excretada por esses animais e o ambiente em que vivem.

225
10 - (UFV MG) Relacione cada um dos grupos animais com o seu principal tipo de excreta.
Grupos de animais
1. Aves
2. Peixes cartilaginosos
3. Peixes sseos
4. Aneldeos
5. Insetos

Tipo de excreta
A. Amnia
B. Uria
C. cido rico

Assinale a alternativa que contm as associaes CORRETAS:
a) 1C, 2A, 3A, 4C, 5B.
b) 1B, 2C, 3A, 4B, 5C.
c) 1B, 2A, 3C, 4B, 5B.
d) 1C, 2B, 3A, 4A, 5C.

11 - (UFRJ) Alguns anfbios passam por um processo denominado neotenia, no qual certas caractersticas das fases
juvenis so mantidas no indivduo adulto. Os axolotl (salamandras) norte-americanos so exemplos desse fenmeno.
Os animais neotnicos podem permanecer aquticos e apresentar brnquias externas como os juvenis de sua espcie,
enquanto os demais se tornam terrestres. Tais caractersticas anatmicas e ecolgicas so acompanhadas por
adaptaes fisiolgicas.
Identifique o principal composto nitrogenado excretado pelos axolotls neotnicos e pelos terrestres.
Justifique sua resposta.

12 - (UNICAMP SP) O grfico abaixo mostra a variao na concentrao de dois hormnios ovarianos, durante o ciclo
menstrual em mulheres, que ocorre aproximadamente a cada 28 dias.



a) Identifique os hormnios correspondentes s curvas A e B e explique o que acontece com os nveis desses
hormnios se ocorrer fecundao e implantao do ovo no endomtrio.
b) Qual a funo do endomtrio? E da musculatura lisa do miomtrio?

13 - (UNESP SP) VIGILNCIA SANITRIA DE SP INTERDITA LOTES DE ANTICONCEPCIONAL INJETVEL. O
Centro de Vigilncia Sanitria da Secretaria da Sade de So Paulo decidiu proibir a comercializao e o uso de trs
lotes de determinado anticoncepcional injetvel, base de medroxiprogesterona, um hormnio sinttico que, se
administrado na dose recomendada, inibe a secreo dos hormnios FSH e LH pelo organismo feminino. Anlises
feitas pelo Instituto Adolfo Lutz apontaram que ampolas do produto contm menor quantidade hormonal do que o
previsto. Na prtica, isso coloca em risco a eficcia do medicamento na preveno da gravidez.
(Folha de S.Paulo, 08.11.2007.)

Do ponto de vista fisiolgico, explique por que o medicamento com quantidades menores de medroxiprogesterona,
interditado pela Vigilncia Sanitria, coloca em risco a eficcia na preveno da gravidez.

14 - (UFRJ) O grfico a seguir mostra as fases do ciclo ovariano que ocorre ao longo do ciclo de menstruao de uma
mulher.

Sabe-se que um vulo pode viver at 48 horas e os
espermatozides podem viver at cinco dias no ambiente uterino.
Indique o dia do ciclo em que ocorre a ovulao e o perodo em
que existe chance de ocorrer uma gravidez.


226
15 - (UFF RJ) Aps a 2 guerra mundial se observou que o ndice de natalidade da populao afetada aumentou de
forma significativa como se representasse um recomeo da nossa espcie, um evento denominado babyboom. Na
populao humana, o processo de reproduo, que envolve o aparecimento de caracteres sexuais secundrios e a
formao de gametas, depende da ao seqencial de alguns hormnios.
Supondo que ocorra a fecundao aps a ovulao, como ficaro os nveis dos hormnios 4 e 5 durante a gravidez?
Justifique.
Identifique as fases, dos ciclos ovariano e uterino, respectivamente, nos quais uma mulher, que no est grvida, se
encontra no perodo entre o vigsimo e o vigsimo quinto dia do ciclo menstrual regular (28 dias).
a) Observe o esquema e preencha os espaos 1, 2, 3, 4 e 5 com os nomes dos hormnios correspondentes.
b) Supondo que ocorra a fecundao aps a ovulao, como ficaro os nveis dos hormnios 4 e 5 durante a
gravidez? Justifique.
c) Identifique as fases, dos ciclos ovariano e uterino, respectivamente, nos quais uma mulher, que no est grvida,
se encontra no perodo entre o vigsimo e o vigsimo quinto dia do ciclo menstrual regular (28 dias).

16 - (UNIMONTES MG) A plula anticoncepcional um dos mtodos contraceptivos mais eficazes e amplamente
usados por mulheres do mundo inteiro. A plula mais comum consiste numa mistura de progesterona e estrgeno
sintticos. Considerando a fisiologia do ciclo ovariano, responda s questes a seguir.
a) A plula atua inibindo a secreo de qual(is) hormnio(s)?
b) O que ocorre quando os nveis de estrgeno e progesterona caem bruscamente com a suspenso do uso da
plula?

17 - (UNESP SP) A figura mostra os nveis de diferentes hormnios ao longo do ciclo menstrual de uma mulher: em
A, os hormnios gonadotrficos e, em B, os hormnios esterides.


a) A que hormnios correspondem, respectivamente, as curvas 2 e 3? Qual a funo desses hormnios?
b) Aps a ovulao dessa mulher, teria havido fertilizao do vulo e nidao? Justifique.

18 - (UEG GO) Uma jovem comeou a submeter-se a um tratamento intensivo que consistia em exerccios e injees
intramusculares peridicas. Depois de algum tempo, ela notou que sua massa muscular, sua velocidade e sua
resistncia tinham aumentado, mas seus cabelos passaram a cair, ao mesmo tempo em que surgiram plos em seu
corpo e o seu ciclo menstrual ficou irregular.
a) Que tipo de substncia estava sendo administrada nessa jovem?
b) Do ponto de vista hormonal, por que o seu ciclo menstrual ficou irregular?

19 - (UFG) Em 2008, comemora-se o centenrio da morte de Machado de Assis, o Bruxo do Cosme Velho. O trecho
abaixo usa dilogos presentes na obra machadiana e retrata uma informao biolgica interessante.

Pena irnica molhada na tinta de melancolia lies de um bruxo

Perdo, mas o senhor no tem filhos?
verdade. No transmiti a nenhuma criatura o legado da nossa misria.
Mas evitou-se por inteno ou acaso?
O velho inclina a cabea e medita um pouco.
Creio que por acaso. Ou por fora da natureza, que tudo pode e tudo transforma.
No v pensar que Carolina e eu recorremos ao remdio que previne a concepo
para sempre, e de que ouo falar na rua do Ouvidor.
PLVORA, Hlio. Disponvel em:
<http://www.vidaslusofonas.pt/machado_de_assis.htm>.
Acesso em: 29 set. 2008.
a) Considerando que as personagens do texto no possuam nenhuma alterao cromossmica ou mutao nas
clulas germinativas, cite duas causas biolgicas da infertilidade.
b) Na poca em que estas frases foram escritas, a plula anticoncepcional feminina ainda no havia sido
desenvolvida, contudo a tcnica da vasectomia j era conhecida. Descreva como esses dois mtodos podem
prevenir a concepo.

227
20 - (UFRN) A vasectomia, um dos mecanismos de controle de natalidade disponveis atualmente, promove a
esterilizao masculina, pois interrompe o trajeto dos espermatozides entre os testculos e a prstata.
Responda aos seguintes subitens, que tratam dessa temtica:
a) O homem vasectomizado ainda ejacula? Justifique sua resposta.
b) Anos aps ter sido vasectomizado, um homem que pretendia vir a ser pai biolgico soube que seria impossvel a
reverso da cirurgia. Como ele no havia armazenado esperma antes da vasectomia, sua pretenso poderia tornar-
se realidade? Justifique sua resposta.

21 - (UFSCar SP) A plula do dia seguinte um recurso importante para mulheres que necessitam de uma
contracepo de emergncia; contudo, seu uso tem sido questionado. No incio de 2005, as Cmaras Municipais de
So Jos dos Campos e Jacare (interior de So Paulo) aprovaram leis impedindo a distribuio dessa plula s suas
muncipes. A proibio ao uso da plula foi baseada em argumentos morais e religiosos dos vereadores, que
entenderam que esse mtodo poderia ser abortivo. O Ministrio da Sade entrou com ao cvel pblica na Justia
Federal para permitir o acesso das mulheres ao medicamento.
a) A plula do dia seguinte pode ser usada como recurso para impedir a transmisso de doenas sexualmente
transmissveis entre os parceiros? Justifique.
b) Qual a ao da plula do dia seguinte no organismo que leva a uma contracepo de emergncia? Por que
considerada um mtodo abortivo no entendimento de alguns?

22 - (FUVEST SP) O grfico mostra os nveis de glicose medidos no sangue de duas pessoas, sendo uma saudvel e
outra com diabetes melito, imediatamente aps uma refeio e nas cinco horas seguintes.



a) Identifique a curva correspondente s medidas da pessoa diabtica, justificando sua resposta.
b) Como se explicam os nveis estveis de glicose na curva B, aps 3 horas da refeio?

23 - (UFU MG) Uma vez constatados elevados nveis de colesterol no sangue de um paciente, um mdico solicitou
exames laboratoriais para a avaliao dos nveis de hormnios tireoideanos.
Diante do exposto acima, responda:

a) A solicitao de dosagem de hormnios tireoidianos deve-se ao fato do mdico suspeitar da existncia de uma
patologia. Que patologia essa?
b) Cite duas caractersticas da patologia em questo.
c) Caso essa patologia se manifeste de maneira severa desde o nascimento, que problema fsico ela poder
acarretar ao referido paciente?

24 - (UFRJ) A figura mostra como feita a vasectomia, um
procedimento cirrgico simples que envolve a interrupo dos
vasos deferentes. Essa interrupo impede que os
espermatozides produzidos nos testculos atinjam a uretra,
tornando os homens infrteis. A vasectomia no inibe o ato
sexual. Para que um homem se mantenha sexualmente ativo,
preciso que haja produo e secreo do hormnio testosterona.
A testosterona, que tambm produzida nos testculos,
responsvel pela induo do desejo sexual (libido) e tambm
necessria para que ocorra a ereo do pnis.

Por que a vasectomia no bloqueia os efeitos da testosterona,
uma vez que esse hormnio tambm produzido nos testculos?




228
25 - (UFOP MG) Um rapaz diabtico foi hospitalizado em estado de coma. Aps alguns exames e coleta de
informaes de parentes, os mdicos concluram que o estado comatoso foi devido a uma dose errnea, excessiva, de
insulina. Considerando o caso descrito, responda:
a) Por que a dose excessiva de insulina provocou o coma no jovem?
b) Por que diabticos no-tratados apresentam glicosria (presena de glicose na urina) e poliria (excreo de
grandes volumes de urina)?
c) A insulina normalmente administrada a pacientes com disfuno de que rgo?

TEXTO: 1 - Comum questo: 26

Texto 3
Minha segunda vida

O trabalho de parto durou mais de uma hora. Foi o tempo que passei na burocracia do site www.secondlife.com, no
qual cadastrei o meu e-mail, alguns dados pessoais e escolhi o tipo de conta (so duas: a bsica, grtis, e a premium,
que prev mensalidade de US$ 9 e d direito a uma mesada em linden dlares, a moeda local). Escolhi um nome e
sobrenome para o meu avatar o personagem que me representa dentro da tela e gravei um software em meu
computador. A partir do prximo pargrafo, o avatar quem escreve. O estilo dele meio rebuscado, como o dos
escritores de viagem de antigamente. A seu modo, relata uma viagem a um mundo virtual, com seus cdigos prprios,
alguns to estranhos quanto os da Lilliput, do escritor irlands Jonathan Swift. No barulho difuso, no espetculo de
cores e formas estranhas, na luz que aparece de sbito tudo remete a um nascimento. No houve choro, mas alguns
segundos de silncio, como se a respirao no viesse fcil. Veio, afinal. Estou na Orientation Island, a maternidade do
Second Life. Como eu, dezenas de pessoas se materializam neste lugar a cada instante. Esto nascendo de novo.
Escolheram um sexo, um nome e um sobrenome. Aqui so todos parecidos. Como os bebs. Comeam a andar e
tropeam. Depois descobrem a fala e mexem no que est ao redor. Ento, aprendem a voar. O que torna a segunda
vida interessante no visitar lugares. Para ser feliz em Second Life, preciso ter respeito e poder. Poder e respeito.
Todos querem ser a prxima Anshe Chung, a avatar de origem chinesa que ganhou o primeiro milho de dlares reais
vendendo terrenos irreais.
POCA, So Paulo, n.461, 19 mar. 2007, p. 188-193. [Adaptado].

26 - (UFG) O avano tecnolgico aliado ao desejo de fuga da realidade a curiosidade de vivenciar novas experincias
conduzem o ser humano busca, cada vez mais intensa, de mundos virtuais. Os textos utilizados na coletnea da
prova de Redao abordam esses aspectos. Contudo, no mundo biolgico real, os processos seguem uma
organizao, os quais, muitas vezes, no esto presentes no mundo virtual. Relacionando-se com o ambiente, todos
os seres vivos nascem, crescem, desenvolvem, reproduzem, envelhecem e morrem. As questes da prova de Biologia
relacionam-se aos textos da coletnea da Redao, abordando, no entanto, as caractersticas tpicas dos seres vivos
no mundo real.

Importante: Nas questes que solicitam uma citao, um agente etc, ser considerada apenas uma resposta, neste
caso a primeira. Dessa forma, as demais sero desconsideradas, mesmo estando corretas.

O texto 3 da coletnea da prova de Redao menciona que No barulho difuso, no espetculo de cores e formas
estranhas, na luz que aparece de sbito tudo remete a um nascimento.
O ciclo da vida humana inicia-se com a fecundao do ovcito e o desenvolvimento embrionrio. Para que isso ocorra,
o sistem hipotlamo-hipfise-gnada produz e libera hormnios sexuais, como mostrado a seguir.


a) Os grficos A e B representam a ocorrncia de quais processos fisiolgicos?
b) Nomeie os hormnios X, Y, Z, indicando onde so produzidos e cite uma funo de cada um deles no ciclo
reprodutivo feminino.

229







PROF.:Rmulo




C
C
o
o
n
n
t
t
e
e

d
d
o
o
s
s
:
:

H
H
i
i
s
s
t
t
o
o
l
l
o
o
g
g
i
i
a
a
a
a
n
n
i
i
m
m
a
a
l
l












Ministrados nos ms Setembro














D Da at ta a d da a E En nt tr re eg ga a : : _ __ __ __ __ __ __ __ _/ /_ __ __ __ __ __ __ __ __ __ __ _/ /2 20 01 13 3
LISTA 5 BIOLOGIA

230


1 - (UEM PR) Quando h um corte na pele, os fibroblastos migram para a regio danificada e produzem fibras
colgenas, promovendo o fechamento do corte ou a cicatrizao.
a) A qual tecido pertencem as clulas envolvidas no processo de cicatrizao?
b) Que tipo de diviso celular ocorre no processo da cicatrizao? Geneticamente, como so as clulas-filha em relao
clula que lhes deu origem nesse tipo de diviso celular?

2 - (UNIMONTES MG) A presena de determinadas fibras, clulas e matriz extracelular faz parte dos constituintes
bsicos dos diferentes tipos de tecido conjuntivo. O esquema abaixo se refere s variedades desse tipo de tecido no
organismo humano. Observe-o.



De acordo com o esquema acima e o assunto abordado, INDIQUE a seguir um tecido representado por I, II e III.
I:
II:
III:

3 - (UNICAMP SP) Fibroblasto um tipo de clula do tecido conjuntivo que sintetiza e secreta glicoprotenas como o
colgeno. Algumas organelas citoplasmticas, como o retculo endoplasmtico rugoso, o complexo de Golgi e as
vesculas, participam de forma interativa nessas funes.
a) Qual o papel de cada uma das organelas citadas?
b) Indique duas funes do tecido conjuntivo.

4 - (UFPR) A formao de um esqueleto sseo deu-se ao longo do processo evolutivo das espcies. Nos dias de hoje,
analisando a estrutura de sustentao dos corpos dos vertebrados, deparamo-nos com organismos que apresentam o
esqueleto formado por tecidos de diferentes dureza e rigidez. Por exemplo, no organismo humano o primeiro esqueleto
de tecido cartilaginoso, o qual ser substitudo por tecido sseo. Nos elasmobrnquios (raias e tubares), o esqueleto
sempre cartilaginoso. Temos assim, em diferentes espcies, tecidos diferentes desempenhando a funo de
sustentao.
a.1) Quais as diferenas entre a matriz extracelular do tecido cartilaginoso hialino e a do tecido sseo?
a.2) Cite os tipos celulares encontrados nos referidos tecidos.

5 - (UFRJ) Certos produtos qumicos podem bloquear a diviso celular, destruindo o fuso acromtico e danificando os
cromossomos. Esses produtos so usados como quimioterpicos no tratamento de pacientes com cncer. Em geral as
clulas cancerosas esto em constante diviso.
Apesar da quimioterapia exibir uma grande eficincia teraputica, pessoas submetidas a ela podem sofrer efeitos
colaterais, devido ao efeito dos quimioterpicos sobre as clulas normais.
Que tecidos humanos so mais sensveis aos efeitos colaterais dos quimioterpicos: os tecidos epitelial e
hematopotico ou os tecidos muscular e nervoso?
Justifique sua resposta.

6 - (UERJ) Na criana, a prtica de atividades fsicas, mal orientada e com carga, pode causar alteraes no
crescimento, por provocar uma ossificao precoce.
a) D a denominao da parte do osso longo onde ocorre o processo de crescimento.
b) Identifique o tipo de tecido que serve como base no processo de ossificao dos ossos longos.


Caderno de Atividades

Disciplina:
B Bi io ol lo og gi ia a

Professor(a):
Rmulo
Aluno:
3 ano
Ensino Mdio
Data de Recebimento:
_____/_____/_____
Lista 04
Data Entrega:
_____/_____/_____

231
7 - (UFPB) A pele humana, constituda pela epiderme e pela derme, desempenha diversas funes, tais como:
proteo, sensibilidade ttil e manuteno da temperatura corporal.

Acerca dos tecidos, anexos e receptores cutneos da pele, correto afirmar:

a) As clulas de Langerhans so responsveis pelo reconhecimento e destruio de agentes estranhos que penetram
na pele.
b) A derme produz os melancitos que so responsveis pela produo e distribuio da melanina.
c) A epiderme possui quatro camadas, sendo a mais interna a espinhosa, que a responsvel pela produo de
novas clulas.
d) O corpsculo de Meissner, responsvel pela captao de estmulos de presso e trao, formado por um
conjunto de terminaes nervosas com pontas achatadas.
e) As glndulas sebceas tm por funo principal lubrificar a pele e os pelos, para manter a regulao da
temperatura corporal.

8 - (UFPEL RS) Terapia do Futuro
Os remdios podem ser administrados por via oral, venosa ou uso tpico (pele). Est sendo desenvolvida uma nova
tcnica, a iontoforese, que se baseia na aplicao de uma corrente eltrica de baixa intensidade para facilitar que os
medicamentos permeiem as membranas biolgicas e alcancem a corrente sangunea (Figura).
A iontoforese tem-se mostrado capaz de promover a entrada de molculas apolares e de grande massa molecular no
organismo, evitar a metabolizao gstrica do remdio e permitir melhor controle no transporte de frmacos.


Cincia Hoje, vol. 44, n 259, Maio de 2009 [adapt.].

Com base em seus conhecimentos e nos textos, correto afirmar que

a) os remdios, administrados por uso tpico, tm que atravessar vrias camadas de clulas da epiderme para alcanar
a corrente sangunea, presente na derme, seguindo da at o alvo.
b) os remdios, pela ingesto oral, precisam atravessar vrias camadas de clulas epiteliais do intestino para alcanar
a corrente sangunea.
c) os remdios administrados por via venosa so transportados pelas veias que dirigem o sangue ao corao sob uma
presso mais alta que as artrias.
d) o uso da iontoforese aumenta a metabolizao gstrica do remdio, dificultando a chegada do medicamento na
corrente sangunea
e) a iontoforese, assim como a administrao de medicamentos por uso tpico e via oral, alcanam apenas os rgos
alvos, evitando danos aos outros rgos.
f) I.R.

9 - (UFU MG) Em aves que voam pouco, como galinhas e perus, os msculos peitorais, que movimentam as asas,
so formados principalmente por fibras brancas. Em aves migratrias acontece o contrrio: os msculos peitorais so
formados predominantemente por fibras vermelhas.
Adaptado de LOPES, Snia. So Paulo: Saraiva, 2003. p. 393. v. 1.

De acordo com a descrio acima, faa o que se pede.

a) Estabelea diferenas fisiolgicas e morfolgicas entre fibras musculares brancas e vermelhas.
b) Determine as principais formas de obteno de energia pelas fibras musculares vermelhas e brancas durante a
atividade contrtil.




232
10 - (PAIES) Considere o trecho abaixo.
O corpo de um organismo multicelular constitudo por diferentes tipos de clulas, especializadas em realizar diversas
funes. As clulas com determinado tipo de especializao organizam-se em grupos, constituindo os tecidos.
AMABIS, JM e MARTHO, GR. Biologia das clulas.
So Paulo: Moderna, 1 ed., 1999, pp. 341-343.

Sobre os diferentes tipos de tecidos que constituem nosso organismo, faa o que se pede.
a) Classifique os tipos de tecidos representados nos desenhos esquemticos abaixo que correspondem s letras
apresentadas a seguir.

a) Quanto ao modo de secreo, estabelea trs diferenas entre os tipos de tecidos glandulares excrinos.
b) Descreva duas funes do tecido conjuntivo sangneo.

11 - (UFRN) O tecido epitelial reveste as superfcies do corpo que entram em contato com o meio externo, como
pode ser observado na pele e na mucosa que reveste o intestino. As caractersticas de cada uma dessas regies do
corpo determinam diferenas funcionais entre as clulas que compem as camadas superficiais do tecido epitelial. A
pele previne a entrada de patgenos e substncias estranhas enquanto que a mucosa intestinal tem a funo de
absorver nutrientes e proteger as paredes do tubo digestivo. A partir das informaes dadas sobre as funes da pele
e do intestino delgado, atenda s solicitaes abaixo.
a) Explique por que o tecido epitelial apresenta grande atividade mittica.
b) Cite as caractersticas que as clulas das camadas superficiais da pele e da mucosa intestinal apresentam para
poder exercer as respectivas funes. Justifique sua resposta.

12 - (UEG GO) Os tecidos conjuntivos apresentam elevada quantidade de substncia intercelular. As clulas que
constituem esse tecido apresentam acentuado polimorfismo, ou seja, tm formas e funes bastante variadas.
Sobre esses tecidos, faa o que se pede:
a) Cite quatro caractersticas tpicas (exclusivas) do tecido conjuntivo:
b) D trs exemplos de tecidos conjuntivos no corpo humano.

13 - (UNESP SP) A professora desenhou no quadro-negro duas diferentes clulas: uma clula epitelial do intestino
humano e uma clula vegetal estomtica.
a) Cite uma caracterstica da clula epitelial do intestino que, representada no desenho, permitiria seu reconhecimento.
Cite uma caracterstica da clula estomtica que, representada no desenho, permitiria seu reconhecimento.
b) Para cada uma das clulas, explique qual a relao existente entre a caracterstica citada e a principal funo da
clula no organismo.

14 - (FUVEST SP) O esquema representa dois neurnios contguos (I e II), no corpo de um animal, e sua posio
em relao a duas estruturas corporais identificadas por X e Y.
a) Tomando-se as estruturas X e Y como referncia, em que
sentido se propagam os impulsos nervosos atravs dos
neurnios I e II?
b) Considerando-se que, na sinapse mostrada, no h contato
fsico entre os dois neurnios, o que permite a transmisso do
impulso nervoso entre eles?
c) Explique o mecanismo que garante a transmisso unidirecional do impulso nervoso na sinapse.

15 - (UFRRJ) O esquema a seguir representa
alguns tecidos encontrados no corpo humano. Um
deles, no entanto, no est adequado ao local.

Identifique o tecido esquematizado de forma
inadequada ao local relacionado e justifique sua
resposta.






233
16 - (UFAM) Faa a correspondncia entre as colunas.
ovariano. to revestimen ) (
simples
o pavimentos 5.
olho do conjuntiva ) ( simples cbico 4.
urinrias. vias das e
bexiga da interno to revestimen ) (
ado estratific
cbico . 3
. pericrdio , sanguneos
vasos dos interno to revestimen ) (
ado estratific
pseudo . 2
brnquios. nasais
fossas traquia, da to revestimen ) (
transio 1.
humanos em Ocorrncia epitlio de Tipos



Identifique em uma das alternativas a seguir, qual a seqncia correta:
a) 2; 5; 1; 3; 4
b) 1; 5; 2; 3; 4
c) 1; 2; 5; 3; 4
d) 5; 2; 1; 4; 3
e) 5; 2; 4; 3; 1

17 - (UNIMONTES MG) Penfigide um tipo de doena auto-imune, caracterizada pela formao de anticorpos contra
os desmossomos do tecido epitelial. A propriedade tecidual que estar mais comprometida nessa patologia
a) transporte atravs da membrana.
b) comunicao entre as clulas.
c) adeso celular.
d) sntese de protenas.

18 - (UFV MG) Com o surgimento de invaginaes e de diferentes tubos e cavidades no corpo dos animais, os
epitlios passaram a revestir no apenas a superfcie externa, mas tambm as superfcies internas. Com relao aos
epitlios de revestimento, INCORRETO afirmar que:
a) apresentam clulas justapostas e no so vascularizados.
b) so nutridos por difuso a partir dos tecidos conjuntivos adjacentes.
c) o epitlio de revestimento pode ter funo de absoro de alimento.
d) sua classificao baseia-se no nmero de camadas e na morfologia celular.
e) o epitlio dos vasos sangneos e do peritnio originam-se do endoderma.

19 - (UEG GO) Os tecidos de revestimento ou epitlios so formados por clulas justapostas e apresentam
caractersticas peculiares nos diferentes grupos animais. Todas as funes relacionadas a seguir podem estar
associadas s diferentes funes dos epitlios, EXCETO:
a) Absoro de nutrientes
b) Trocas gasosas com o ambiente
c) Manuteno da temperatura corporal
d) Distribuio de nutrientes para todo o corpo

20 - (UPE) Analise a figura sobre as clulas do tecido nervoso.

No que se refere s caractersticas e funes desempenhadas por estas clulas, assinale a alternativa correta.
a) Figura 1 Astrcito clula cujos prolongamentos se enrolam sobre as
neurofibras presentes no sistema nervoso central, envolvendo-as com
camadas concntricas de sua membrana plasmtica e constitui a bainha de
mielina, que protege e auxilia o desempenho funcional dos neurnios.
b) Figura 2 Micrglia macrfago especializado, cuja funo fagocitar
detritos e restos celulares presentes no tecido nervoso. uma clula
grande, com muitos prolongamentos longos e pouco ramificados.
c) Figura 3 Corpo celular do neurnio consiste no centro metablico do
neurnio, contm o ncleo e a maioria das organelas da clula, tais como retculo endoplasmtico liso,
denominado corpsculo de Nissl, e se relaciona sntese de neurotransmissores.
d) Figura 4 Dendritos consistem em prolongamentos ramificados e especializados na recepo de estmulos
provenientes de outros neurnios ou de clulas sensoriais. Esses prolongamentos aumentam a superfcie dos
neurnios, o que lhes permite captar grande variedade de estmulos.
e) Figura 5 Oligodendrcito apresenta grande nmero de prolongamentos citoplasmticos, alguns destes se ligam
aos neurnios, enquanto outros se ligam a capilares sangneos por meio de expanses denominadas ps-
vasculares, estabelecendo uma ponte nutritiva entre o sangue e os neurnios.

234
21 - (UDESC SC) Relativo aos tecidos animais, CORRETO afirmar que:
a) a medula ssea vermelha responsvel pelo crescimento em espessura dos msculos.
b) os melancitos, que so clulas responsveis pela degradao da melanina, encontram-se no tecido adiposo.
c) os neutrfilos e basfilos, constituintes do tecido nervoso, so responsveis pela imunidade humoral.
d) as clulas de Schwann e os ndulos de Ranvier so responsveis pela filtrao da urina nos rins, fazendo parte do
epitlio secretor.
e) clulas plurinucleadas com miofibrilas estriadas e clulas uninucleadas sem estrias so caractersticas do msculo
estriado esqueltico e do msculo liso, respectivamente.

22 - (UEG GO) As clulas que constituem os organismos dos metazorios foram se especializando no decorrer do
processo evolutivo, agrupando-se em tecidos e rgos. Estes, por sua vez, formam os sistemas.
Considerando os nveis de organizao, analise as seguintes afirmativas:

I. Nos animais, os estmulos do ambiente so captados por rgos sensoriais, como os olhos e os ouvidos. Nesses
rgos encontram-se neurnios sensitivos encarregados de receber os estmulos e transform-los em impulsos
nervosos.
II. O revestimento do corpo dos metazorios denominado tegumento ou pele. Alm de proteger rgos, esse
revestimento tambm pode secretar substncias e atuar como superfcie respiratria.
III. Os rgos como tero e esfago so constitudos por tecido muscular liso, o que possibilita sua movimentao
involuntria.

Assinale a alternativa CORRETA:
a) Apenas a afirmativa I verdadeira.
b) Apenas a afirmativa II verdadeira.
c) Apenas as afirmativas I e III so verdadeiras.
d) Apenas as afirmativas II e III so verdadeiras.
e) Todas as afirmativas so verdadeiras.

23 - (CEFET PR) A gua a substncia mais abundante no interior da clula. Seu percentual varia de acordo com o
metabolismo celular (quanto maior atividade, maior a quantidade de gua) e com a idade do indivduo (quanto mais
velho menos gua ter). A percentagem de gua progressivamente crescente nos seguintes tecidos:

a) tecido muscular, substncia cinzenta do crebro e osso.
b) tecido nervoso do adulto, tecido nervoso do embrio e adiposo.
c) adiposo, conjuntivo fibroso e muscular.
d) nervoso, sseo e epitelial.
e) nervoso, muscular e adiposo.

24 - (UESPI) Com relao aos tendes e aos ligamentos, incorreto afirmar que:
a) eles apresentam feixes de fibras colgenas organizadas numa nica direo.
b) os tendes ligam as extremidades dos msculos aos ossos, possibilitando os movimentos esquelticos.
c) os ligamentos formam cpsulas protetoras na parede externa de rgos como rins, fgado e bao.
d) entre os feixes de fibras dos tendes e dos ligamentos, h fibroblastos.
e) o tecido que forma tendes e ligamentos denominado conjuntivo denso modelado (tendinoso).

25 - (UFSC) Os tecidos conjuntivos derivam do mesoderma do embrio e caracterizam-se por apresentar diversos
tipos de clulas imersas em material extracelular sintetizado por elas.
Com relao aos tecidos conjuntivos, assinale a(s) proposio(es) correta(s).
01. Dentre as vrias funes dos tecidos conjuntivos, podemos citar a absoro e a secreo de substncias atravs
de glndulas.
02. Entre os vrios tipos de clulas existentes nos tecidos conjuntivos encontramos os fibro-blastos, adipcitos,
melancitos e neurnios.
04. Em indivduos adultos, os elementos figurados do sangue se originam do tecido conjuntivo encontrado na medula
ssea vermelha, a qual apresenta clulas-tronco pluripotentes (ou multipotentes) que aps se multiplicarem
ativamente se diferenciam em leuccitos, hemcias e plaquetas.
08. O tecido conjuntivo denso modelado, tambm chamado de tecido tendinoso, formado por fibras grossas
orientadas paralelamente, especialmente fibras colgenas, o que confere grande resistncia a estruturas como
tendes e ligamentos.
16. Os condrcitos, aps formarem a matriz cartilaginosa, sofrem uma pequena retrao de volume e passam a ser
chamados de condroblastos.
32. Os ossos de uma criana so mais flexveis que os ossos de um adulto pois apresentam maior quantidade de
osteoblastos, os quais produzem muitas fibras colgenas.



235







PROF.:NELSON JNIOR







C
C
o
o
n
n
t
t
e
e

d
d
o
o
s
s
:
:

M
M
O
O
D
D
E
E
R
R
N
N
I
I
S
S
M
M
O
O
:
:
G
G
E
E
R
R
A
A

O
O
D
D
E
E
3
3
0
0
,
,
V
V
I
I
D
D
A
A
E
E

O
O
B
B
R
R
A
A
D
D
E
E
C
C
A
A
R
R
L
L
O
O
S
S
D
D
R
R
U
U
M
M
M
M
O
O
N
N
D
D
D
D
E
E

A
A
N
N
D
D
R
R
A
A
D
D
E
E
,
,




















D Da at ta a d da a E En nt tr re eg ga a : : _ __ __ __ __ __ __ __ _/ /_ __ __ __ __ __ __ __ __ __ __ _/ /2 20 01 13 3

LISTA 1 LITERATURA

236

Exerccios

1- (UCG- 2007) Observe com ateno a tela de Tarsila do Amaral Tendo-a como referncia, assinale os itens abaixo:




01 ( ) O ttulo da obra Abaporu tem origem na lngua Tupi-Guarani. Seus elementos mrficos aba(homem) e
poru(comer) correspondem ao grego antropos (homem ) e fagos (comer),
resultando o vocbulo antropofagia, em lngua portuguesa, significando homem que come. Os elementos da natureza
agreste, expressos disformemente, sugerem uma ruptura cultura europia que serviu como parmetro para a
representao da mentalidade brasileira. Com essa simbologia, concebe-se o conceito de o homem que come a
cultura europia para simbolizar a cultura brasileira. Prope a atitude simblica de devorao dos valores e influncias
estrangeiros num processo de assimilao crtica, a fim de imprimir- lhes carter nacional.

02 ( ) Abaporu apresenta uma figura solitria sentada numa plancie verde, com o brao dobrado repousando num
joelho. Tem ps imensos e a mo sustentando o peso da minscula cabea. Um cacto erecto, de onde parece brotar
uma flor, compe o cenrio. As formas so volumosas e exuberantes, sugerindo um qu de Brasil autntico que
parece romper com a pintura vigente.

03 ( ) O quadro Abaporu, de Tarsila do Amaral, foi o marco principal de um movimento radicalmente primitivista, a
Antropofagia, que propunha a devorao da cultura estrangeira de forma crtica e a valorizao no ufanista da cultura
brasileira. Tal movimento teve grande importncia para a afirmao do Modernismo brasileiro, que despontou com
representantes significativos como Oswald de Andrade, Mrio de Andrade, Antonio Alcntara Machado, Manuel
Bandeira.

04 ( ) Os modernistas, diferentes entre si por ideologia poltica, encontravam na experimentao criativa das revistas
encarregadas da difuso do novo credo esttico, um denominador comum. Aps a Semana de 22, surgem, em So
Paulo e no Rio, revistas como Klaxon, Esttica e Festa, abrigando manifestos, ensaios crticos, textos originais de
poetas e prosadores. Em maio de 1928, sob a gerncia de Raul Bopp, sai em So Paulo a Revista de Antropofagia. O
Manifesto Antropfago de Oswald de Andrade representava o grito de batalha pela liberdade de expresso. O nome do
movimento nascia do quadro de Tarsila do Amaral, que representava o Abaporu, o ndio antropfago, smbolo
zombeteiro do novo Brasil Modernista.

05 ( ) No Brasil, 1922 no s foi o ano da primeira revolta tenentista, da criao do PCB, como tambm de uma
importante ruptura cultural, ocorrida em So Paulo a realizao da Semana de Arte Moderna. Escritores, arquitetos,
pintores, msicos, escultores apresentaram suas obras que provocaram grande impacto ao pblico. Este reagiu com
incompreenso s novas tendncias, dentre elas, o movimento antropfago, que aceitava a cultura estrangeira desde
que fosse devorada e digerida internamente.
Caderno de Atividades

Disciplina:
Literatura

Professor(a):
Nelson
Aluno:
3 ano
Ensino Mdio
Data de Recebimento:
_____/_____/_____
Lista 01
Data Entrega:
_____/_____/_____

237

2- (UEL- 2004): Observe a figura que mostra a pintura de Tarsila do Amaral, Operrios, realizada em 1931.



Com base na anlise da imagem, assinale a alternativa correta:

a) Tarsila se refere s diferentes etnias que constituram o corpo de trabalhadores, possibilitando o desenvolvimento
industrial de So Paulo.
b) Tarsila se refere s raas e culturas que compem o pas, especialmente na cidade de Belo Horizonte.
c) Tarsila recorre ao tema tnico para dar vazo ao sentimento xenfobo que vigorava na arte
brasileira, na dcada de trinta.
d) Tarsila reflete sobre o valor da indstria nacional, pois as pessoas esto na frente da fbrica esperando para ir
trabalhar.
e) Tarsila faz uma crtica social ao mostrar as pessoas empilhadas na frente da fbrica como
se fossem produtos para serem vendidos.


3- Nas primeiras dcadas do sculo XX, as disputas polticas davam-se entre positivistas, liberais e anarquistas; no
plano artstico, os embates ocorriam entre neoclssicos, acadmicos e modernistas. No contexto desse perodo, que
acontecimento, nas artes, marcou o incio do Movimento Modernista Brasileiro?

a) A chegada de Lasar Segall ao Brasil.
b) A publicao da obra Juca Mulato, de Menotti Del Picchia.
c) O encontro de Oswald de Andrade, Guilherme de Almeida, Menotti Del Picchia e Mario de Andrade, nos Sales de
Dona Olvia Guedes Penteado.
d) A publicao da revista Klaxon.
e) A exposio de Anita Malfatti em 1917.

4- Carnaval em Madureira parte integrante da fase Pau-Brasil de Tarsila do Amaral.

Com base na obra e nos conhecimentos sobre o Manifesto Pau-
Brasil, de Oswald de Andrade, correto afirmar:

a) A obra de Tarsila do Amaral reflete profunda tristeza acerca da
dura vida na favela, sendo esta mesma tristeza professada no
Manifesto Pau-Brasil.
b) A Torre Eiffel no meio da favela refora uma das idias contidas
no Manifesto Pau-Brasil: a arte europia sempre foi superior
arte brasileira.
c) Tarsila do Amaral e Oswald de Andrade propem uma arte ligada
s razes culturais brasileiras, no perdendo de vista a expresso
artstica moderna.
d) Tarsila do Amaral e Oswald de Andrade criticam os costumes da
populao brasileira, vistos como fatores de atraso cultural.
e) A obra Carnaval em Madureira e o Manifesto Pau-Brasil fazem
uma crtica ao Modernismo europeu.


5- (UEL- 2007): 03- Quando falamos em Manifesto Pau Brasil e
Manifesto Antropofgico, estamos falando de duas ocorrncias no
campo da arte, das primeiras dcadas do sculo XX, que
provocaram transformaes no modo de fazer e entender a arte
desde aquela poca. Assinale a alternativa que corresponde a estes
manifestos e suas respectivas caractersticas:


238
a) O Manifesto Pau Brasil foi escrito por Mrio de Andrade em 1924 . O Manifesto Antropofgico foi escrito em 1928,
por Oswald de Andrade. Ambos propunham o desenvolvimento de uma esttica brasileira baseada nos valores
locais, como o artesanato e a arte plumria indgena, para que pudssemos construir nossa autonomia cultural.
b) O Manifesto Pau Brasil foi escrito por Mrio de Andrade em 1924. O Manifesto Antropofgico foi escrito em 1928,
por Oswald de Andrade. Tanto o primeiro quanto o segundo eram propostas idealizadas segundo a esttica
europia, herdada de Portugal e reforada pela Misso Artstica
Francesa, com o fim de valorizar a arte do Brasil.
c) O Manifesto Pau Brasil foi escrito por Mrio de Andrade em 1924, e conclamava os artistas
brasileiros para uma produo mais livre, criativa, inovadora e brasileira, afastando-se dos padres
europeus impostos desde a Misso Francesa. O Manifesto Antropofgico foi escrito em 1928, por
Oswald de Andrade, tomando por referncia o naufrgio do navio portugus em que viajava o bispo Sardinha,
morto e devorado pelos ndios. Este manifesto explicita a influncia da esttica
estrangeira e prope a instaurao de sua independncia.
d) O Manifesto Pau Brasil foi escrito por Mrio de Andrade em 1924. O Manifesto Antropofgico foi escrito em 1928,
por Oswald de Andrade. Estes manifestos propunham divulgar as idias destes dois autores com o fim de
promover o trabalho que realizavam na arte moderna brasileira, visando comercializao de seus livros.
e) O Manifesto Pau Brasil foi escrito por Mrio de Andrade em 1924. O Manifesto Antropofgico foi escrito em 1928,
por Oswald de Andrade. Tanto um como outro esperavam, com tais manifestos, inovar a arte brasileira, dando-lhe
visibilidade, tornando-a internacional, j que a esttica reinante no pas correspondia mesma esttica difundida
no resto do mundo, como em Paris e Nova Iorque.


5- Analise a imagem a seguir:

Com base na imagem e nos conhecimentos sobre o modernismo
brasileiro no perodo de 1920 a 1930, correto afirmar:

a) O movimento modernista prope a releitura de temas retratados por
pintores europeus do final
do sculo XIX, especialmente paisagens e alegorias histricas, como
fator de ruptura aos
princpios acadmicos.

b) Incorporando tcnicas de deformao da figura e estilizao das
linhas, o modernismo brasileiro
introduz o prosaico como pilar da nacionalidade, conceito este
discutido paralelamente ao
movimento artstico.

c) A pintura no modernismo brasileiro destaca personagens urbanos com caractersticas do
realismo, influenciando, assim, os primeiros pintores do movimento.

d) O interesse por temas populares e folclricos do Brasil, paralelamente incorporao de novas
tendncias da arte, propiciou campo frtil execuo de trabalhos plsticos pelos artistas.

e) Impulsionada pelos modernistas, a escultura congrega as principais caractersticas do
movimento, reproduzindo os modismos e integrando vrios estilos em suas peas.

5- Responda as questes propostas:
a) Tarsila do Amaral faz essa tela para comemorar uma data muito
especial para ela. Que data era essa?

b) Qual o nome da tela? O que significa esse nome?

c) A partir desta tela, Oswald de Andrade tem a idia de lanar um novo
movimento modernista. Que movimento seria esse? Quais eram suas
principais caractersticas?


239








PROF.:NELSON JNIOR





C
C
o
o
n
n
t
t
e
e

d
d
o
o
s
s
:
:

M
M
O
O
D
D
E
E
R
R
N
N
I
I
S
S
M
M
O
O
:
:
G
G
E
E
R
R
A
A

O
O
D
D
E
E
3
3
0
0
,
,
V
V
I
I
D
D
A
A
E
E

O
O
B
B
R
R
A
A
D
D
E
E
C
C
A
A
R
R
L
L
O
O
S
S
D
D
R
R
U
U
M
M
M
M
O
O
N
N
D
D
D
D
E
E

A
A
N
N
D
D
R
R
A
A
D
D
E
E
,
,























D Da at ta a d da a E En nt tr re eg ga a : : _ _0 04 4/ /_ __ __ _M MA AR R O O_ _/ /2 20 01 13 3

LISTA 2 LITERATURA

240

01 - (Unifor CE/Janeiro)
Padre Natuzzi, voz de ouro,
fala do cu, essa infinita aurora
a que seremos todos transportados
se.
Fala tambm do abismo arquimedonho
em que, a gordurosas culpas amarrado,
de ponta-cabea irei precipitar-me
se.

Est INCORRETA a seguinte afirmao sobre as estrofes acima, em que o poeta Carlos Drummond de Andrade se
lembra de um momento de sua infncia:
a) embora sintaticamente no concludas, h nas duas estrofes elementos que permitem pressupor qual seria a
complementao lgica das frases.
b) as expresses infinita aurora e abismo arquimedonho estabelecem entre si uma expressiva relao de oposio.
c) a linguagem coloquial e o abusivo aproveitamento do verso livre deixam claro que se trata de um poema tpico da
primeira fase do Modernismo de 22.
d) a sintaxe geral das frases e a disposio dos versos ajudam a estabelecer um paralelismo entre as duas estrofes.
e) a dicotomia geral ocorre entre a salvao e a danao, conforme sejam praticadas as aes abertas e sugeridas
por cada um dos se.

02 - (Udesc SC) Leia, com ateno, os trechos de dois poemas de Carlos Drummond de Andrade.

Imenso trabalho nos custa a flor.
(...)
Uma s ptala resume auroras e pontilhismos,
sugere estncias, diz que te amam, beijai a rosa,
ela sete flores, qual mais fragrante, todas exticas,
todas histricas, todas catrticas, todas patticas.
(...) J no vejo amadores de rosa.
fim do parnasiano, comeo da era difcil, a burguesia apodrece.
Aproveitem. A ltima
Rosa desfolha-se.
(In: Anncio da Rosa. A Rosa do Povo. Rio de Janeiro: Record, 2004, p.78-79.)
Vai o meu elefante
pela rua povoada,
mas no o querem ver
nem mesmo para rir
(...)
Mostra com elegncia
sua mnima vida,
e no h na cidade
alma que se disponha
a recolher em si
desse corpo sensvel
a fugitiva imagem,
o passo desastrado
mas faminto e tocante.
(In:. O Elefante. A Rosa do Povo. Rio de Janeiro: Record, 2004, p. 105-107.)
Assinale a alternativa CORRETA.

a) Nesse trecho fica evidenciado o amor do Poeta s flores, em especial s rosas.
b) No primeiro poema, Drummond se diz um poeta parnasiano, enquanto, no segundo, ele um modernista a
conclamar a figura prosaica de um elefante como fator potico.
c) A flor e o elefante, motivos dos poemas acima, no so mais que metforas dos poemas propriamente ditos; alis,
principalmente a flor assunto recorrente na obra potica de Drummond.
Caderno de Atividades

Disciplina:
Literatura

Professor(a):
Nelson
Aluno:
3 ano
Ensino Mdio
Data de Recebimento:
_____/_____/_____
Lista 02
Data Entrega:
_____/_____/_____

241
d) Quando o Poeta afirma, em Anncio da Rosa, que a burguesia apodrece, o mesmo que dizer que ningum tem
a menor sensibilidade para com a falta de elegncia de um elefante.
e) Apesar de a obra A Rosa do Povo disseminar em suas pginas a atmosfera sufocante da ditadura Vargas e os
horrores da II Guerra Mundial, os dois poemas acima fogem quele contexto por sugerir, em seus versos, a paz
das estncias e a graa dos bichos.

03 - (UFJF MG) QUADRILHA

JOO AMAVA Teresa que amava Raimundo
que amava Maria que amava Joaquim que amava Lili
que no amava ningum.
Joo foi para os Estados Unidos, Teresa para o convento
Raimundo morreu de desastre, Maria ficou para tia,
Joaquim suicidou-se e Lili casou com J. Pinto Fernandes
que no tinha entrado na histria.

Com base na leitura do poema, discuta a concepo de amor em Drummond.

04 - (IME RJ) Leia os seguintes textos, observando que eles descrevem o ambiente natural de acordo com a poca a
que correspondem, fazendo predominar os aspectos buclico, cotidiano e irnico, respectivamente:

Texto 1

Marlia de Dirceu

Enquanto pasta, alegre, o manso gado,
minha bela Marlia, nos sentemos
sombra deste cedro levantado.
Um pouco meditemos
Na regular beleza,
Que em tudo quanto vive nos descobre
A sbia Natureza.

Atende como aquela vaca preta
O novilhinho seu dos mais separa,
E o lambe, enquanto chupa a lisa teta.
Atende mais, cara,
Como a ruiva cadela,
Suporta que lhe morda o filho o corpo,
E salte em cima dela.
(GONZAGA, Toms Antnio. "Marlia de Dirceu." In: Proena Filho, Domcio. Org. "A poesia dos inconfidentes." Rio
de Janeiro, Nova Aguilar, 1996, p. 605.)

Texto 2

Buclica nostlgica

Ao entardecer no mato, a casa entre
bananeiras, ps de manjerico e cravo-santo,
aparece dourada. Dentro dela, agachados,
na porta da rua, sentados no fogo, ou a mesmo,
rpidos como se fossem ao xodo, comem
feijo com arroz, taioba, ora-pro-nobis,
muitas vezes abbora.
Depois, caf na canequinha e pito.
O que um homem precisa pra falar,
entre enxada e sono: Louvado seja Deus!
(PRADO, Adlia. "Poesia Reunida." 2 . ed, So Paulo: Siciliano, 1992, p. 42.)






242
Texto 3

Cidadezinha qualquer

Casas entre bananeiras
Mulheres entre laranjeiras
Pomar amor cantar

Um homem vai devagar.
Um cachorro vai devagar.
Um burro vai devagar.

Devagar... as janelas olham.

Eta vida besta, meu Deus.
(ANDRADE, Carlos Drummond. "Obra Completa," Rio de Janeiro: Jos Aguilar Editora, 1967, p. 67.)

Assinale a alternativa referente aos respectivos momentos literrios a que correspondem os trs textos:

a) Romntico, contemporneo, modernista.
b) Barroco, romntico, modernista.
c) Romntico, modernista, contemporneo.
d) rcade, contemporneo, modernista.
e) rcade, romntico, contemporneo.

TEXTO: 4 - Comum questo: 5

TEXTO A

Se eu fosse apenas...

Se eu fosse apenas uma rosa
com que prazer me desfolhava,
j que a vida to dolorosa
e no te sei dizer mais nada!

Se eu fosse apenas gua e vento,
com que prazer me desfaria,
como em teu prprio pensamento
vais desfazendo a minha vida!

Perdoa-me causar-te a mgoa
desta humana, amarga demora!
de ser menos breve do que a gua,
mais durvel que o vento e a rosa...
Ceclia Meireles


TEXTO B

O Bicho

Vi ontem um bicho
Na imundcie do ptio
Catando comida entre os detritos

Quando achava alguma coisa,
No examinava nem cheirava:
Engolia com voracidade.

O bicho no era um co,
No era um gato,
No era um rato.
O bicho, meu Deus, era um homem.

243
Manuel Bandeira
(PUC RS)
Todas as afirmativas que seguem podem ser associadas aos poetas dos textos em questo, EXCETO:

a) valem-se de recursos lingsticos recorrentes na msica popular atual.
b) foram contemporneos de Carlos Drummond de Andrade e Vincius de Moraes.
c) A primeira caracteriza-se pela subjetividade intimista; o segundo, pela sntese entre subjetividade e objetividade.
d) Recorrem freqentemente a imagens relacionadas natureza.
e) Dissociam-se da fase experimental do movimento modernista.

TEXTO 2

1
Minha condio humana me fascina. Conheo o limite de minha
2
existncia e ignoro por que estou nesta terra, mas s
vezes o pressinto.
3
Pela experincia cotidiana, concreta e intuitiva, eu me descubro vivo
4
para alguns homens, porque
o sorriso e a felicidade deles me
5
condicionam inteiramente, mais ainda para outros que, por acaso,
6
descobri terem
emoes semelhantes s minhas.
7
E cada dia, milhares de vezes, sinto minha vida corpo e alma
8
integralmente tributria do trabalho dos vivos e dos
mortos. Gostaria de
9
dar tanto quanto recebo e no paro de receber. Mas depois experimento
10
o sentimento satisfeito
de minha solido e quase demonstro m
11
conscincia ao exigir ainda alguma coisa de outrem. Vejo os homens se 11
12
diferenciarem pelas classes sociais e sei que nada as justifica a no ser
13
pela violncia. Sonho ser acessvel e
desejvel para todos uma vida
14
simples e natural, de corpo e de esprito.
15
(...) foram ideais que suscitaram meus esforos e me permitiram
16
viver. Chamam-se o bem, a beleza, a verdade. Se
no me identifico com
17
outras sensibilidades semelhantes minha e se no me obstino
18
incansavelmente em
perseguir esse ideal eternamente inacessvel na arte
19
e na cincia, a vida perde todo o sentido para mim. Ora, a
humanidade
20
se apaixona por finalidades irrisrias que tm por nome a riqueza, a
21
glria, o luxo. Desde moo, j as
desprezava constantemente.
Albert Einstein. Como vejo o mundo. Rio de Janeiro, Nova Fronteira, 1981
(Com adaptaes)
6 - (UFLavras MG)
O texto predominantemente:
a) enumerativo.
b) persuasivo.
c) opinativo.
d) conclusivo.
e) explicativo.

7- Assinale a alternativa incorreta sobre Ceclia Meireles.

a) Ceclia Meireles, juntamente com Murilo Mendes e Vincius de Moraes faz parte da corrente espiritualista da segunda
fase da poesia modernista.
b) Ceclia foi primeira voz potica feminina a ter repercusso significativa na nossa histria literria e expressa sua
sensibilidade em versos filiados a mais rica tradio lrica portuguesa e brasileira.
c) As imagens decorrentes da autora so a musica, a gua, o mar, o ar, o vento, o espao, tudo vago e fluido,
denotando uma influencia neo-simbolista.
d) Nada na poesia de Ceclia Meireles produto de atitude racional e seu tom fundamental o da fuga e o do sonho.
e) Ceclia aproxima-se tambm das experincias formais da poesia concreta, surgida em 1956. A obra Romanceiro da
Inconfidncia inaugura essa fase da poesia de Meireles.

Leia o texto abaixo para responder s perguntas

"Ora, daquela vez, como das outras, Fabiano ajustou o gado, arrependeu-se, enfim deixou a transao meio
apalavrada e foi consultar a mulher. Sinh Vitria mandou os meninos para o barreiro, sentou-se na cozinha,
concentrou-se, distribuiu no cho sementes de vrias espcies, realizou somas e diminuies. No dia seguinte Fabiano
voltou cidade, mas ao fechar o negcio notou que as operaes de Sinh Vitria, como de costume, diferiam das do
patro. Reclamou e obteve a explicao habitual: a diferena era proveniente de juros.
No se conformou: devia haver engano. Ele era bruto, sim senhor, via-se perfeitamente que era bruto, mas a mulher
tinha miolo. Com certeza havia um erro no papel do branco. No se descobriu o erro, e Fabiano perdeu os estribos.
Passar a vida inteira assim no toco, entregando o que era dele de mo beijada! Estava direito aquilo? trabalhar como
negro e nunca arranjar carta de alforria!
O patro zangou-se, repeliu a insolncia, achou bom que o vaqueiro fosse procurar servio noutra fazenda.
A Fabiano baixou a pancada e amunhecou. Bem, bem. No era preciso barulho no. Se havia dito palavra -toa, pedia
desculpa. Era bruto, no fora ensinado. Atrevimento no tinha, conhecia o seu lugar. Um cabra. Ia l puxar questo
com gente rica? Bruto, sim senhor, mas sabia respeitar os homens. Devia ser ignorncia da mulher, provavelmente
devia ser ignorncia da mulher. At estranhara as contas dela. Enfim, como no sabia ler (um bruto, sim senhor),
acreditara na sua velha. Mas pedia desculpa e jurava no cair noutra.
(RAMOS, Graciliano. Vidas secas.70
a
. edio. Rio, So Paulo: Record, 1995.)

244
Questo 09 Lngua Portuguesa

Sabendo que Vidas Secas foi publicado na dcada de 30, comente a contemporaneidade do romance, no que diz
respeito influncia do meio fsico na vida das personagens e do povo nordestino.

Questo 10 Lngua Portuguesa

Uma das questes que mais preocupava Graciliano Ramos era o relacionamento entre patro e empregado, vistos
como explorador e explorado, demonstrado claramente no fragmento transcrito. Estabelea o posicionamento de
Fabiano e do fazendeiro diante dessa situao.
do romance, desde que pertinentes.


245






PROF.: Fernando Viana



C
C
o
o
n
n
t
t
e
e

d
d
o
o
s
s
:
:








M
M
i
i
n
n
i
i
s
s
t
t
r
r
a
a
d
d
o
o
s
s
d
d
u
u
r
r
a
a
n
n
t
t
e
e
o
o
m
m

s
s
d
d
e
e
f
f
e
e
v
v
e
e
r
r
e
e
i
i
r
r
o
o














D Da at ta a d da a E En nt tr re eg ga a : : _ __ __ __ __ __ __ __ _/ /_ __ __ __ __ __ __ __ __ __ __ _/ /2 20 01 13 3

LISTA 1 HISTRIA

246

01 - (UFLA MG) Observe o mapa abaixo.


Assinale a alternativa que NO se relaciona com as diversas hipteses de origem do homem americano.
a) O mapa justifica a hiptese de o homem americano apresentar caractersticas mongolides ou pr-mongolides,
povos oriundos da Monglia e Sibria que penetraram no continente americano pelo Estreito de Bering.
b) Segundo o que demonstra o mapa, o homem chegou Amrica em migraes espordicas, navegando pelo
Pacfico, vindo da sia, Polinsia Oceania.
c) Segundo o mapa, o homem americano Autctone, ou seja, surgiu no prprio continente, embora no exista
nenhum fssil anterior ao Homo Sapiens Sapiens.
d) Observando o mapa, fica evidente que o Ser Humano no chegou no continente americano pelo oceano Atlntico,
apesar de esse ser o caminho mais curto .
e) No continente sul-americano, o fssil mais antigo de uma mulher conhecida por Luzia, encontrada em 1975
prximo a Lagoa Santa/MG, datada de 11500 anos.

02 - (UFPEL RS)
Texto 1
Em todo o mundo, a leste e a oeste, as populaes comearam a trocar a dependncia s hordas de grandes animais
muitas das quais em rpido declnio pela explorao de animais menores e de plantas. [...]
Onde as condies fossem particularmente adequadas [...], as peas do quebra-cabea da domesticao se
acomodaram, e os coletores transformaram-se em agricultores.
CROSBY, Alfred W. Imperialismo ecolgico. So Paulo: Companhia das Letras, 1993.

Texto 2
Os historiadores acostumaram-se a separar a coleta e a agricultura como se fossem duas etapas da evoluo humana
bastante diferentes e a supor que a passagem de uma a outra tivesse sido uma mudana repentina e revolucionria.
Hoje, contudo, admite-se que essa transio aconteceu de maneira gradual e combinada. Da etapa em que o homem
era inteiramente um caador-coletor passou-se para outra em que comeava a executar atividades de cultivo de
plantas silvestres [...] e de manipulao dos animais [...]. Mas tudo isso era feito como uma atividade complementar da
coleta e da caa.
In: VICENTINO, Cludio. Histria para o ensino mdio: histria
geral e do Brasil. So Paulo: Scipione, 2005.

Os textos analisam
a) o final do Perodo Neoltico e se posicionam de forma convergente quanto ao papel revolucionrio desempenhado
pela agricultura e pela domesticao dos animais.
b) o incio do Perodo Neoltico e divergem entre si a respeito da existncia da Revoluo Neoltica, pois enquanto um
indica uma transformao radical, o outro destaca a simultaneidade da caa, coleta e agricultura.
c) o incio do Paleoltico Inferior e so contraditrios entre si, no que se relaciona aos efeitos da agricultura, dentre
eles a sedentarizao humana.
d) o final do Paleoltico Superior, no momento em que ocorreu a Revoluo Agrcola, ambos afirmando que a caa e
a coleta foram suprimidas pela agricultura.
e) a Transio Mesoltica, e concordam que, com o cultivo das plantas e a criao de animais, ocorreu a suspenso
das atividades de caa e coleta, provocando a Revoluo Neoltica.
f) I.R.
Caderno de Atividades

Disciplina:
Histria

Professor(a):
Fernando
Aluno:
3 ano
Ensino Mdio
Data de Recebimento:
_____/_____/_____
Lista 01
Data Entrega:
_____/_____/_____

247
03 - (UFBA)
Texto I: Artigos do Cdigo de Hamurbi
Se um homem furar o olho de um homem livre, furar-se-lhe- um olho.
Se ele fura o olho de um escravo alheio ou quebra um membro ao escravo alheio, dever pagar a metade do seu
preo.
Se um arquiteto constri uma casa para algum, porm no a faz slida, resultando da que a casa venha a ruir e
matar o proprietrio, este arquiteto passvel de morte.
Se, ao desmoronar, ela mata o filho do proprietrio, matar-se- o filho deste arquiteto.
(AQUINO; FRANCO; LOPES, 1980, p. 114).

Texto II: Extratos da Lei Mosaica
Se um homem der um soco no olho do seu escravo ou da sua serva, e, em conseqncia, eles perderem esse rgo,
sero alforriados como compensao.
No se punir o homicida antes de ouvidas as testemunhas. Ningum ser condenado pelo testemunho de um s.
Aquele que ferir seu pai ou sua me ser punido de morte.
Aquele que ferir um dos seus concidados ser tratado como o tratou: receber fratura por fratura e perder olho por
olho, dente por dente.
(ARRUDA, 1981, p. 98).

Com base nesses textos e nos conhecimentos sobre o exerccio do Direito entre os antigos mesopotmios e hebreus,
indique dois fundamentos em comum, presentes no contedo dos cdigos de leis dos referidos povos.

04 - (UNIFESP SP)

(Egito: tumba de Sennedjem e de sua esposa. Sculo XIII a.C.)

A arte do Egito Antigo, alm de estar inteiramente ligada s crenas religiosas, apresenta muitas informaes sobre a
sociedade da poca.

a) Qual fator geogrfico propiciava, numa regio cercada por deserto, a atividade produtiva representada pela imagem?
b) Que significado religioso tinha para os egpcios a representao de cenas da vida cotidiana nos tmulos?

05 - (UFTM MG) Na Antigidade, a civilizao fencia particularizou-se por
a) formar um imprio teocrtico, em que se fundiram as culturas grega e asitica.
b) elaborar o primeiro cdigo de leis escritas, baseado em punies severas.
c) desenvolver o comrcio martimo, fundando colnias na bacia do Mediterrneo.
d) ter uma crena monotesta, o que modificou as sociedades do Oriente Prximo.
e) organizar-se em cidades-Estados, sob influncia da democracia ateniense.

06 - (UEG GO) O primeiro cdigo de leis de que se tem conhecimento foi estabelecido pelos babilnios por volta de
XVIII a. C.
Quais os princpios fundamentais do chamado Cdigo de Hamurbi?

07 - (UFRN) Entre as primeiras Civilizaes Orientais, a Civilizao Egpcia sobressaiu-se como uma das mais
grandiosas e a mais duradoura. As necessidades de desenvolvimento da agricultura irrigada nas margens do rio Nilo
exigiam uma direo centralizada. Nessas circunstncias, a Civilizao do Egito Antigo organizou-se em torno de uma
Monarquia.

Entre as imagens mais populares ligadas ao Egito Antigo, esto as pirmides. As mais conhecidas so as de Giz,
construdas no primeiro perodo da histria poltica do Estado Egpcio, entre 3200 e 2300 a.C., conhecido como Antigo
Imprio, retratadas na Figura abaixo.

Disponvel em:<www.suapesquisa.com/monumentos/piramides_gize.htm>.
Acesso em: 12 ago. 2010.

a) Mencione e comente duas caractersticas do Governo no Egito Antigo.
b) Explique o significado das pirmides, relacionando-as ao poder no Egito
Antigo.

248
08 - (UNESP SP) Um dos mais antigos registros escritos conhecidos surgiu no Egito. A regio foi tambm bero do
Estado e da diferenciao social. Escrever requeria anos de aprendizado e apenas alguns poucos, como os escribas,
dedicavam-se a essa tarefa.
Nos dias atuais, o conceito de analfabetismo mudou. A Unesco adota a noo de analfabeto funcional: pessoa capaz
de escrever e de ler frases simples, mas que no consegue usar informaes escritas para satisfazer suas
necessidades dirias e para desenvolver seu conhecimento. Explique para que servia a escrita no Egito antigo e
relacione o conceito contemporneo de analfabetismo com a idia de excluso social.

09 - (UFG GO) Na antiguidade Oriental, a existncia de gua e terras frteis foi fundamental para a formao de
grandes civilizaes, principalmente no chamado Crescente Frtil.
Devido presena de dois grandes rios, as terras hoje ocupadas pelo Iraque e pelo Kuwait foram alvo de disputas de
vrios povos que, sucessivamente constituram complexas sociedades, chegando a formar grandes imprios.
Aponte 2(dois) grandes imprios surgidos nesta regio, na Antiguidade, destacando 2(duas) caractersticas de cada
imprio, quer no aspecto social, poltico, econmico ou cultural.

10 - (UFAL) A busca por melhores condies de vida tanto espaciais quanto socioculturais tem sido uma
constante na histria de homens e mulheres em todos os tempos.

a) O que significou para o povo hebreu o episdio da sua histria reconhecido como o xodo?
b) Como podem ser classificadas as migraes quanto ao seu espao de deslocamento?

11 - (UFC CE) BRASLIA - Irritada com a verso de Hollywood para a guerra entre gregos e persas no filme 300 de
Esparta, a Embaixada do Ir em Braslia divulgou uma nota nesta quarta-feira na qual acusa o filme, que tem no
elenco o brasileiro Rodrigo Santoro fazendo o papel do rei persa Xerxes, de promover o conflito entre as civilizaes.
(Jornal O Globo, 04/04/2007).
Com base no texto acima e em seus conhecimentos, responda as questes que seguem.
a) Qual a ligao histrica entre os povos iraniano e persa?
b) Como ficaram conhecidas as guerras entre gregos e persas na Antigidade?
c) Qual a motivao principal das guerras mencionadas no item anterior?
d) Cite dois motivos do conflito diplomtico entre Ir e EUA nos dias de hoje.

1 12 2- - E EN NE EM MO desenho do artista uruguaio Joaqun Torres-Garca trabalha com uma representao diferente
da usual da Amrica Latina. Em artigo publicado em 1941, em que apresenta a imagem e trata do
assunto, Joaqun afirma:
Quem e com que interesse dita o que o norte e o sul?
Defendo a chamada Escola do Sul por
que na realidade, nosso norte o Sul. No deve haver
norte, seno em oposio ao nosso sul.
Por isso colocamos o mapa ao revs, desde j, e ento
teremos a justa ideia de nossa posio, e
no como querem no resto do mundo. A ponta da
Amrica assinala insistentemente o sul, nosso
norte.
TORRES-GARCA, J. Universalismo constructivo.
Buenos Aires: Poseidn, 1941. (com adaptaes).
O referido autor, no texto e imagem acima,
(A) privilegiou a viso dos colonizadores da Amrica.
(B) questionou as noes eurocntricas sobre o mundo.
(C) resgatou a imagem da Amrica como centro do
mundo.
(D) defendeu a Doutrina Monroe expressa no lema
Amrica para os americanos.
(E) props que o sul fosse chamado de norte e vice-
versa.

3 3- -A A c co on nq qu ui is st ta a d do o t te em mp po o a at tr ra av v s s d da a m me ed di id da a
c cl la ar ra am me en nt te e p pe er rc ce eb bi id da a c co om mo o u um m d do os s i im mp po or rt ta an nt te es s a as sp pe ec ct to os s d do o c co on nt tr ro ol le e d do o u un ni iv ve er rs so o p pe el lo o h ho om me em m. . D De e u um ma a m mo od do o n n o o
t t o o g ge er ra al l, , o ob bs se er rv va a- -s se e c co om mo o n nu um ma a s so oc ci ie ed da ad de e a a i in nt te er rv ve en n o o d do os s d de et te en nt to or re es s d do o p po od de er r n na a m me ed di id da a d do o t te em mp po o u um m
e el le em me en nt to o e es ss se en nt tc ci ia al l d do o s se eu u p po od de er r: : o o c ca al le en nd d r ri io o u um m d do os s g gr ra an nd de es s e em mb bl le em ma as s e e i in ns st tr ru um me en nt to os s d do o p po od de er r; ; p po or r o ou ut tr ro o l la ad do o
, , a ap pe en na as s o os s d de et te en nt to or re es s c ca ar ri is sm m t ti ic co os s d do o p po od de er r s s o o s se en nh ho or re es s d do o c ca al le en nd d r ri io o: : r re ei is s, , p pa ad dr re es s, , r re ev vo ol lu uc ci io on n r ri io os s. .
L LE E G GO OF FF F, , J Ja aq qu ue es s. . H Hi is st t r ri ia a e e m me em m r ri ia a. . C Ca am mp pi in na as s: : E Ed d. . D Da a U Un ni ic ca am mp p, , 1 19 99 90 0

C Co om m b ba as se e n no os s c co on nh he ec ci im me en nt to os s q qu ue e v vo oc c t te em m s so ob br re e a as s r re el la a e es s e en nt tr re e h hi is st t r ri ia a, , t te em mp po o e e p po od de er r, , i in nt te er rp pr re et te e o o f fr ra ag gm me en nt to o
d de e t te ex xt to o a ac ci im ma a. .

249

1 13 3- - E Em m 2 20 00 09 9, , o o g go ov ve er rn no o f fe ed de er ra al l l la an n o ou u o o p pr ro oj je et to o M Me em m r ri ia as s R Re ev ve el la ad da as s c co om m o o o ob bj je et ti iv vo o d de e a am mp pl li ia ar r n no os ss so o
c co on nh he ec ci im me en nt to o s so ob br re e o o p pe er r o od do o d da a d di it ta ad du ur ra a m mi il li it ta ar r n no o B Br ra as si il l. . U Um m d do os s m mo ot ti iv vo os s q qu ue e m mo ot ti iv vo ou u o o p pr ro oj je et to o f fo oi i a a d de es st tr ru ui i o o e e
o o d de es sa ap pa ar re ec ci im me en nt to o d de e m mu ui it to os s d do oc cu um me en nt to os s d de es ss se e p pe er r o od do o. . A A p pa ar rt ti ir r d da as s r re ef fl le ex x e es s q qu ue e f fi iz ze em mo os s s so ob br re e f fo on nt te es s h hi is st t r ri ic ca as s
e e p po od de er r, , c co om me en nt te e s so ob br re e p po os ss s v ve ei is s m mo ot ti iv vo os s p pa ar ra a o o d de es sa ap pa ar re ec ci im me en nt to o d do os s d do oc cu um me en nt to os s e e s su ua as s i im mp pl li ic ca a e es s n na a m me em m r ri ia a
c co ol le et ti iv va a. .


1 14 4- - N N o o h h c co on ns se en ns so o e en nt tr re e o os s e es st tu ud di io os so os s s so ob br re e a a d da at ta a a ap pr ro ox xi im ma ad da a d da a c ch he eg ga ad da a h hu um ma an na a a ao o c co on nt ti in ne en nt te e a am me er ri ic ca an no o. .
A As s d di iv ve er rg g n nc ci ia as s n n o o s s o o p pe eq qu ue en na as s. . P Pa ar rt ti in nd do o d de es st ta a a af fi ir rm ma a o o f fe ei it ta a p pe el lo o h hi is st to or ri ia ad do or r C Cl l u ud di io o V Vi ic ce en nt ti in no o, , a ap pr re es se en nt te e a as s
p pr ri in nc ci ip pa ai is s t te es se es s s so ob br re e a a o oc cu up pa a o o d do o n no os ss so o c co on nt ti in ne en nt te e e ex xp pl li ic ca an nd do o o os s a ar rg gu um me en nt to os s q qu ue e a as s s su us st te en nt ta am m. .


1 15 5- - A A i id de ei ia a e eu ur ro oc ce en nt tr ri ic ca a d de e q qu ue e e ex xi is st ti ir ri ia a u um ma a p pr r - -h hi is st t r ri ia a c co on nt te em mp pl la a t ta am mb b m m a a n no o o o d de e p pr ro og gr re es ss ss so o h hi is st t r ri ic co o. . A A
h hu um ma an ni id da ad de e e ev vo ol lu ui ir ri ia a d de e e es st t g gi io os s m me en no os s a ap pe er rf fe ei i o oa ad do os s p pa ar ra a s si it tu ua a e es s m me el lh ho or re es s. .. .. . c co om mo o s se e e ex xi is st ti is ss se e u um ma a r ro ot te ei ir ro o, ,
u um ma a t tr ra ag ge et t r ri ia a q qu ue e d de ev ve es ss se e s se er r o ob br ri ig ga at to or ri ia am me en nt te e c cu um mp pr ri id da a p po or r t to od do os s o os s p po ov vo os s e e s so oc ci ie ed da ad de es s, , p po or r t to od da a h hu um ma an ni id da ad de e. .
C Co om mo o b ba as se e n na a a af fi im ma a o o d de e C Cl l u ud di io o V Vi ic ce en nt ti in no o, , r re el la ac ci io on ne e a a c cr r t ti ic ca a a ao o e er ro oc ce en nt tr ri is sm mo o c co om m o os s e es st tu ud do os s d da a H Hi is st t r ri ia a d da a
o oc cu up pa a o o d do o c co on nt ti in ne en nt te e a am me er ri ic ca an no o. .



250








PROF.: Fernando Viana






C
C
o
o
n
n
t
t
e
e

d
d
o
o
s
s
:
:
G
G
r
r
e
e
c
c
i
i
a
a
e
e
R
R
o
o
m
m
a
a







M
M
i
i
n
n
i
i
s
s
t
t
r
r
a
a
d
d
o
o
s
s
d
d
u
u
r
r
a
a
n
n
t
t
e
e
o
o
m
m

s
s
d
d
e
e
M
M
a
a
r
r

o
o














D Da at ta a d da a E En nt tr re eg ga a : : _ __ __ __ __ __ __ __ _/ /_ __ __ __ __ __ __ __ __ __ __ _/ /2 20 01 13 3
LISTA 2 HISTRIA

251

01 - (UFSCAR SP) Considerando as caractersticas da democracia antiga (como a ateniense) e as da democracia
contempornea (adotada por muitos pases), indique as
a) diferenas.
b) semelhanas.

02 - (UFRN) O estudo da Histria permite a compreenso da dinmica de permanncia e mudana a que esto
submetidas as instituies, as formaes sociais, a cultura material e os modos de pensar das sociedades. Assim
sendo:

Grcia e Roma, o que que tm a ver com a gente? [] Nossa sociedade moderna ligase, de muitas maneiras, s
civilizaes clssicas e sempre h grande interesse pelos mais variados aspectos da cultura antiga que se fazem
manifestar, de forma mais ou menos explcita, aqui ou ali, gerando primeiro a curiosidade e, em seguida, o interesse
por saber mais.
FUNARI, Pedro Paulo. Grcia e Roma. So Paulo: Contexto, 2002. p. 9-11.

A partir das consideraes contidas no fragmento textual acima, mencione e explique trs elementos da cultura greco-
romana presentes no cotidiano do mundo contemporneo.

03 - (UFG GO)
Leia o texto.
A guerra no nem pode ser anomia, ausncia de regras. Ao contrrio ela se desenrola no quadro de normas aceitas
por todos os gregos, precisamente porque essas regras se originam do conjunto de prticas, de valores, de crenas
comuns. Ainda aqui, o quadro s verdadeiro at certo ponto. De incio porque a guerra jamais ficou confinada
unicamente nas fronteiras do mundo grego. Desde ento, dividida em dois campos antagnicos, a Grcia engajou-se
numa luta cujo risco, escala e forma no eram mais os mesmos. Foi todo um sistema de regras antigas que se rompeu.
VERNANT, Jean-Pierre. Mito e sociedade na Grcia Antiga.
Rio de Janeiro: Jos Olympio, 1992. p.38-39. (Adaptado).

Neste fragmento, o historiador Jean-Pierre Vernant avalia a transformao no ordenamento das cidades-Estados,
advinda com as guerras contra os persas. Diante do exposto, explique:

a) a ordem poltica das cidades-Estados, anterior guerra contra os persas;
b) a mudana ocorrida na ordenao das cidades-Estados em virtude da guerra contra os persas.

04 - (UNESP SP) Em 399 a.C., o filsofo Scrates acusado de graves crimes por alguns cidados atenienses. (...)
Em seu julgamento, segundo as prticas da poca, diante de um jri de 501 cidados, o filsofo apresenta um longo
discurso, sua apologia ou defesa, em que, no entanto, longe de se defender objetivamente das acusaes, ironiza seus
acusadores, assume as acusaes, dizendo-se coerente com o que ensinava, e recusa a declarar-se inocente ou pedir
uma pena. Com isso, ao jri, tendo que optar pela acusao ou pela defesa, s restou como alternativa a condenao
do filsofo morte.
(Danilo Marcondes. Iniciao Histria da Filosofia, 1998. Adaptado.)

Com base no texto apresentado, explique quais foram os motivos da condenao de Scrates morte.

05 - (UNIFESP SP) (...) no ltimo quartel do sculo VII [a.C.] que a economia das cidades (...) volta-se decididamente
para o exterior; o trfico por mar vai ento amplamente ultrapassar a bacia oriental do Mediterrneo, entregue a seu
papel de via de comunicao. A zona dos intercmbios estende-se a oeste at a frica e Espanha, a leste at ao Mar
Negro.
(Jean-Pierre Vernant. As origens do pensamento grego. So Paulo: Difel, 1991.)

O texto fala da expanso das cidades gregas no sculo VII a.C. Explique

a) por que o autor chama o Mar Mediterrneo de via de comunicao.
b) os principais motivos dessa expanso.

Caderno de Atividades

Disciplina:
Histria

Professor(a):
Fernando
Aluno:
3 ano
Ensino Mdio
Data de Recebimento:
_____/_____/_____
Lista 02
Data Entrega:
_____/_____/_____

252
06 - (UFJF MG) O terico franco-suo Benjamin Constant pronunciou uma conferncia em 1819 intitulada A
liberdade dos antigos comparada dos modernos. Em linhas gerais, este era o seu argumento:
A liberdade que caracterizaria as cidades antigas como Atenas na Antiguidade era a liberdade de participar
coletivamente do governo, da soberania, era a liberdade de decidir na praa pblica: era a liberdade do homem
pblico. Em contraste, a liberdade dos modernos, a que convinha aos novos tempos, era a liberdade do homem
privado, a liberdade dos direitos de ir e vir, de propriedade, de opinio, de religio. A liberdade moderna no exclui o
direito de participao poltica, mas esta se faz agora pela representao e no pelo envolvimento direto.
CARVALHO, Jos Murilo de. A Formao das Almas.
So Paulo: Companhia das Letras,1990.Adaptado

Quando tratou da liberdade dos antigos, o autor explicou, indiretamente, a diferena entre a democracia antiga e a
moderna.
a) Segundo o trecho e seus conhecimentos, qual o tipo de democracia existente em Atenas?
b) Cite e explique DUAS diferenas entre a democracia ateniense e a democracia atual.

07 - (UEG GO) Para Plato, a polis o modelo de vida em grupo. na Repblica que o autor apresenta os vrios
grupos que compem a sociedade. De acordo com suas idias, o grupo que deve governar a polis o dos:
a) comerciantes que, sabendo da importncia das riquezas para as Cidades-estado da Grcia, levariam riquezas para
a polis.
b) filsofos que, por conhecer a verdade e o bem atravs da contemplao do mundo das idias, proporcionariam o
maior bem comum a todos.
c) guerreiros, pois se caracterizavam por sua fora, integridade e seu grande amor aos sentimentos mais nobres,
como fidelidade e bravura.
d) trabalhadores que, por meio das mais diversas profisses e movidos pela ambio do lucro, garantiriam o sustento
de toda a polis.

08 - (UNICAMP SP) Nada mais presente na vida cotidiana da coletividade do que a oratria, que partilha com o
teatro a caracterstica de ser a manifestao cultural mais popular e mais praticada na Atenas clssica. A civilizao da
Atenas clssica uma civilizao do debate. As reaes dos atenienses na Assemblia eram influenciadas por sua
experincia como pblico do teatro e vice-versa. Trata-se de uma civilizao substancialmente oral. O grego era
educado para escutar. O caminho de Scrates a Aristteles ilustra perfeitamente o percurso da cultura grega da
oralidade civilizao da escrita, que corresponde, no plano poltico e social, passagem da cidade-estado ao
ecumenismo helenstico.
(Adaptado de Agostino Masaracchia, La prosa
greca del V e del IV secolo a.C.. In: Giovanni
DAnna (org.). Storia della letteratura greca. Roma:
Tascabile Economici Newton, 1995, p. 52-54.)
a) Estabelea relaes entre o modelo poltico vigente na Atenas clssica e a importncia assumida pelo teatro e pela
oratria nesse perodo.
b) Aponte caractersticas do perodo helenstico que o diferenciam da Atenas clssica.

09 - (UFC CE) A Grcia antiga era composta por vrias cidades-Estados (pleis) que tinham, cada uma, sua
autonomia poltica e econmica. No entanto, mantinham uma estreita relao entre si, o que permitia definir
uma identidade entre os gregos, e caracterizavam-se:
a) pela reunio em torno da cidade de Esparta nos perodos de guerra, pois essa era a nica polis que havia
mantido a realeza aps o fim da Idade do Bronze.
b) pela submisso religiosa cidade-Estado de Delfos onde se localizava o orculo de mesmo nome e eram
realizados os jogos olmpicos.
c) pela existncia do vnculo comum da linguagem, da religio e do padro de vida urbano, o que
configurava a existncia da Hlade.
d) pela manuteno do monoplio de rotas pelo Mediterrneo, atravs de uma liga entre as cidades com o intuito de
combater as foras persas, fencias e egpcias.
e) pela unidade militar, pois todo grego era entregue ao Estado aos sete anos de idade, para receber uma
mesma e nica formao para tornar-se guerreiro.

10 - (UEG GO) Mirem-se no exemplo daquelas mulheres de Atenas
Vivem pros seus maridos, orgulho e raa de Atenas
Quando amadas, se perfumam
Se banham com leite, se arrumam
Suas melenas
Quando fustigadas no choram
Se ajoelham, pedem, imploram
Mais duras penas
Cadenas
BUARQUE, Chico. Mulheres de Atenas. Disponvel em: <www.mundocultural.com.br.> Acesso em: 11 set. 2006.

253
O papel desempenhado pela mulher na Histria foi, por muito tempo, negligenciado pelos historiadores. Apenas a partir
da segunda metade do sculo XX, os historiadores, despertados pelas contestaes feministas, passaram a interessar
mais pela atuao da mulher. Sobre a mulher na Antiguidade, analise a validade das proposies a seguir.

I. A mulher no Egito Antigo teve um status privilegiado em comparao com outras civilizaes antigas, pois possua
direitos sociais e jurdicos que lhe garantiam uma significativa liberdade.
II. As mulheres hebraicas possuam direitos polticos e sociais equivalentes aos dos homens, derivados dos preceitos
religiosos do Pentateuco, os quais defendiam que os homens e as mulheres so iguais, pois ambos so filhos de
Deus.
III. A mulher ateniense casada vivia grande parte do seu tempo confinada ao lar, estando submissa a um regime de
quase recluso, privada de uma participao efetiva nas decises polticas.
IV. A sociedade guerreira espartana privava as mulheres de qualquer participao no exrcito e na poltica,
restringindo suas funes educao dos seus filhos e de suas filhas.

Assinale a alternativa CORRETA:
a) As proposies I e III so verdadeiras.
b) As proposies I e IV so verdadeiras.
c) As proposies II e III so verdadeiras.
d) As proposies III e IV so verdadeiras.

11 - (UFG GO) Leia o cartum a seguir.


SIMPSON, Margaret. Clepatra e sua vbora. So Paulo: Companhia das Letras, 2002, p. 139.

O cartum trata das relaes entre o Egito, na figura da rainha Clepatra, e Roma, na representao do general Marco
Antnio, durante a crise da Repblica romana. Ao elaborar uma viso contempornea dessas relaes, o cartum
remete a um contexto histrico, no qual se destacava

a) o domnio de Clepatra sobre os generais romanos, os quais lhe concediam primazia nas conquistas territoriais.
b) a postura autoritria de Clepatra, considerando a ausncia de legitimidade dos lderes do exrcito romano.
c) a atuao de Clepatra no Senado Romano, administrando suas disputas internas.
d) o conhecimento militar de Clepatra, rivalizando com a poltica expansionista romana.
e) a estratgia poltica de Clepatra, objetivando a ampliao dos seus territrios em prejuzo dos romanos.

12 - (UFG GO) Leia o fragmento da Lei das Doze Tbuas, datada de 450 a.C.

1. Se algum for chamado a Juzo, comparea.
2. Se no comparecer, aquele que o citou tome testemunhas e o prenda.
Tbua Primeira, do chamamento a Juzo.

3. Se algum cometer furto noite e for morto em flagrante, aquele que o matou no ser punido.
[]
7. Se, pela procura, a coisa furtada for encontrada na casa de algum, que esse algum seja punido como se fora um
furto manifesto.
Tbua segunda, dos julgamentos e dos furtos.
LEI DAS DOZE TBUAS. Disponvel em: <http://www.jurisciencia.com/legislacoes/
legislacao-diversa/lei-das-doze-tabuas-lei-das-12-tabuas-lei-das-xiitabuas/210/>.
Acesso em: 13 out. 2010. [Adaptado]

Esse cdigo de leis estabeleceu os princpios do Direito Romano, que forneceu as bases para o direito no Ocidente. De
acordo com o historiador Paul Veyne, os costumes romanos so traduzidos com bastante exatido pelo direito civil.
Diante do exposto e considerando a leitura do fragmento,

a) analise os conflitos sociais na Repblica Romana, que explicitam a relao entre lei e costume;
b) explique o papel da testemunha e a importncia da prova, explcitos na Lei das Doze Tbuas.




254
13 - (UFBA) Despojados de suas terras, inmeros camponeses emigravam para a cidade, onde seriam clientes das
famlias ricas ou iriam engrossar a massa de desocupados, pobres e famintos. (COTRIM, 1994, p. 99).

O texto se refere a um fenmeno social que se construiu na antiga Repblica Romana e cujo conceito clientelismo
se estendeu a vrias situaes que relacionam poder e dependncia, ao longo da histria, inclusive na histria do
Brasil.

Com base nessas consideraes e nos conhecimentos sobre o assunto, indique um fator que favoreceu o
florescimento do clientelismo na antiga Repblica Romana e outro que favoreceu sua prtica no Brasil.

Fator referente Antiga Repblica Romana:
Fator referente ao Brasil:

14 - (UFAC) Durante a dcada de 1970, Asterix, um personagem de histrias em quadrinhos, alcanou grande
popularidade. Gauls, Asterix liderava, com a ajuda de uma poo mgica e de seu companheiro Obelix, a resistncia
de sua tribo contra os invasores romanos.
ARRUDA, Jos Jobson de A.; PILETTI, Nelson.
Toda a Histria: Histria Geral e Histria do Brasil.
So Paulo: tica, s.d., p.95.


Por Tutatis! Digestivo Cultural. Disponvel em:
http://www.digestivocultural.com/colunistas/coluna.asp?codigo=1387.

O quadrinho acima apresenta a imagem de Asterix e Obelix, personagens de Uderzo e Goscinny. Ao observ-los e ler
o texto, podemos compreender que:

a) O expansionismo do Imprio Romano no significou alterao nos ordenamentos territoriais e polticos europeus.
b) Os gauleses combatiam o imperialismo grego.
c) Os gauleses foram responsveis pelo crescimento da usura, que seria uma das causas de decadncia do Imprio
Romano.
d) Os romanos tentavam dominar os povos brbaros para a erradicao do helenismo.
e) A expanso do Imprio Romano ocorreu com oposio dos povos brbaros.

15 - (UEPB) O Imprio Romano do Ocidente sofre a sua desintegrao a partir do sculo IV da era crist, ao tempo
que avanam os povos brbaros, como eram chamados aqueles que no pertenciam ao mundo romano. Quanto aos
brbaros correto afirmar:

a) os hunos, grupo tribal ordinrio da sia Central, penetraram pacificamente em territrios romanos convivendo
harmonicamente com os germnicos.
b) os brbaros no incorporaram costumes e normas romanas, ficando assim fiis s suas tradies.
c) com o aprofundamento da crise do imprio romano, integrantes de tribos brbaras adentraram pacificamente as
fronteiras romanas.
d) o renascimento carolngio condenou as obras clssicas, o teocentrismo e o trabalho dos monges copistas.
e) a adoo do direito romano, como referncia jurdica, extinguiu em todo o territrio romano a prtica do direito
consuetudinrio e dos costumes tribais brbaros.

16 - (UEG GO) Com a morte de Jlio Csar, inicia-se um perodo de conflitos na sociedade romana, o Segundo
Triunvirato. Analise os principais conflitos polticos do perodo.



255

17 - (FUVEST SP) Cesarismo/cesarista so termos utilizados para caracterizar governantes atuais que, maneira de
Jlio Csar (de onde o nome), na antiga Roma, exercem um poder
a) teocrtico.
b) democrtico.
c) aristocrtico.
d) burocrtico.
e) autocrtico.

18 - (ESPM) O mundo romano mergulhou num prolongado perodo de crises. O Baixo Imprio foi marcado pela
decadncia e pela anarquia. Finalmente as invases brbaras minaram as foras imperiais j agonizantes, tomando
pouco a pouco seus territrios e colocando fim ao imprio romano em 476.
(Cludio Vicentino. Histria Geral)

Sobre o mundo romano no Baixo Imprio correto afirmar que:

a) o perodo foi caracterizado pela continuidade da poltica de guerras de conquistas;
b) ocorreu uma expanso das reas cultivadas em consequncia da expanso territorial derivada das guerras;
c) o fim das guerras de conquistas fez escassear o nmero de prisioneiros e prejudicou a produo, acarretando a crise
do escravismo;
d) as guerras e as conquistas permitiram obter ouro e prata abundantes, ocasionando uma inflao crescente;
e) para proteger as fronteiras do imprio romano, ameaadas pelos brbaros, foi criada a guarda pretoriana.

19 - (UFCG PB) Eixo temtico: As leituras do saber histrico sobre as relaes de poder na formao do
Mundo Antigo

Durante a Roma Imperial, os cidados que moravam na cidade de Roma viviam em condies urbano-sanitrias
bastante precrias. Todavia, morar na cidade era um dos primeiros requisitos para demonstrar a sua civitas (sua
condio de cidado). Era o primeiro passo para ser considerado cidado ou encontrar uma forma de adquirir a to
sonhada liberdade.

Sobre a vida cotidiana durante o perodo imperial romano INCORRETO afirmar que:

a) Os banhos pblicos no eram uma prtica de higiene, mas um prazer recusado pelos cristos que s se banhavam
uma ou duas vezes por ms.
b) Os romanos so, de alguma forma, clientes uns dos outros e tm como obrigao saudar os seus patres todos os
dias pela manh, conferindo-lhe honorabilidade, que para o romano um smbolo de riqueza e poder.
c) O servio de abastecimento de gua em Roma surgiu no sculo 11 a.C, acompanhado por uma vasta rede de
esgotos que despejava gua servida no rio Tibre, servindo indistintamente tanto a ricos quanto a pobres.
d) A cena (ceia), principal refeio do dia, era feita a partir das 21 horas, quando os cidados ricos se recolhiam s
suas residncias, acompanhados de escravos que, carregando tochas, iluminavam as perigosas ruas romanas.
e) A habitao era uma realidade para o romano mdio. No h sem-tetos e todos aqueles que no possuiam uma
casa podem se instalar num albergue pblico, financiado pelo Estado romano.

20 - (UEG GO) O primeiro Triunvirato foi um sinal inequvoco da crise vivida pela Repblica romana. Apenas trs
homens, Pompeu, Csar e Crasso, acumularam quase todos os ttulos e cargos importantes. O fim dessa aliana,
marcado pela morte de Crasso em 53 a.C., representou imediatamente

a) o aumento da rivalidade entre os dois sobreviventes, Csar e Pompeu, que resultou em uma violenta guerra civil.
b) o enfraquecimento da influncia de Csar, em virtude do fracasso de sua campanha militar na Glia.
c) o assassinato de Csar por membros da aristocracia romana dentro do prprio senado.
d) a formao de um novo triunvirato, constitudo por Otvio, Marco Antnio e Lpido.

21 - (UFG GO) A utilizao de trabalho forado um fenmeno verificado na Roma Antiga e no Brasil atual. Ao
comparar-se esses dois contextos, observa-se como elemento comum a ocorrncia

a) do estatuto jurdico dos indivduos cativos, que sustentado pelo Estado.
b) da concepo de inferioridade racial atribuda a um grupo, que redunda na perda de sua liberdade.
c) do perodo de permanncia no cativeiro, que vitalcio e hereditrio.
d) do endividamento pessoal, que serve manuteno do trabalho compulsrio.
e) do apadrinhamento, que serve de mecanismo de atenuao das condies de trabalho.



256
22- (ENEM/2010) A poltica foi, inicialmente, a arte de impedir as pessoas de se ocuparem do que lhes diz respeito.
Posteriormente, passou a ser a arte de compelir as pessoas a decidirem sobre aquilo de que nada entendem.
VALRY, P. Cadernos. Apud BENEVIDES, M. V. M.
A cidadania ativa. So Paulo: tica, 1996.
Nessa definio, o autor entende que a histria da poltica est dividida em dois momentos principais: um primeiro,
marcado pelo autoritarismo excludente, e um segundo, caracterizado por uma democracia incompleta.

Considerando o texto, qual o elemento comum a esses dois momentos da histria poltica?

23 - (ENEM/2010) Na antiga Grcia, o teatro tratou de questes como destino, castigo e justia. Muitos gregos sabiam
de cor inmeros versos das peas dos seus grandes autores. Na Inglaterra dos sculos XVI e XVII, Shakespeare
produziu peas nas quais temas como o amor, o poder, o bem e o mal foram tratados. Nessas peas, os grandes
personagens falavam em verso e os demais em prosa. No Brasil colonial, os ndios aprenderam com os jesutas a
representar peas de carter religioso.
Esses fatos so exemplos de que, em diferentes tempos e situaes, o teatro uma forma

a) de manipulao do povo pelo poder, que controla o teatro.
b) de diverso e de expresso dos valores e problemas da sociedade.
c) de entretenimento popular, que se esgota na sua funo de distrair.
d) de manipulao do povo pelos intelectuais que compem as peas.
e) de entretenimento, que foi superada e hoje substituda pela televiso.

24-(ENEM/2010) Quem construiu a Tebas de sete portas?
Nos livros esto nomes de reis.
Arrastaram eles os blocos de pedra?
E a Babilnia vrias vezes destruda. Quem a reconstruiu tantas vezes?
Em que casas da Lima dourada moravam os construtores?
Para onde foram os pedreiros, na noite em que a Muralha da China ficou pronta?
A grande Roma est cheia de arcos do triunfo.
Quem os ergueu? Sobre quem triunfaram os csares?
BRECHT, B. Perguntas de um trabalhador que l.
Disponvel em: http://recantodasletras.uol.com.br. Acesso
em: 28 abr. 2010.

Partindo das reflexes de um trabalhador que l um livro de Histria, o autor censura a memria construda sobre
determinados monumentos e acontecimentos histricos. A crtica refere-se ao fato de que

a) os agentes histricos de uma determinada sociedade deveriam ser aqueles que realizaram feitos heroicos ou
grandiosos e, por isso, ficaram na memria.
b) a Histria deveria se preocupar em memorizar os nomes de reis ou dos governantes das civilizaes que se
desenvolveram ao longo do tempo.
c) grandes monumentos histricos foram construdos por trabalhadores, mas sua memria est vinculada aos
governantes das sociedades que os construram.
d) os trabalhadores consideram que a Histria uma cincia de difcil compreenso, pois trata de sociedades antigas
e distantes no tempo.
e) as civilizaes citadas no texto, embora muito importantes, permanecem sem terem sido alvos de pesquisas
histricas.

25-(ENEM/2010)Quando dipo nasceu, seus pais, Laio e Jocasta, os reis de Tebas, foram informados de uma profecia
na qual o filho mataria o pai e se casaria com a me. Para evit-la, ordenaram a um criado que matasse o menino.
Porm, penalizado com a sorte de dipo, ele o entregou a um casal de camponeses que morava longe de Tebas para
que o criasse. dipo soube da profecia quando se tornou adulto. Saiu ento da casa de seus pais para evitar a
tragdia. Eis que, perambulando pelos caminhos da Grcia, encontrou-se com Laio e seu squito, que, insolentemente,
ordenou que sasse da estrada. dipo reagiu e matou todos os integrantes do grupo, sem saber que entre eles estava
seu verdadeiro pai. Continuou a viagem at chegar a Tebas, dominada por uma Esfinge. Ele decifrou o enigma da
Esfinge, tornou-se rei de Tebas e casou-se com a rainha, Jocasta, a me que desconhecia.
Disponvel em: http://www.culturabrasil.org. Acesso em: 28 ago. 2010 (adaptado).
No mito dipo Rei, so dignos de destaque os temas do destino e do determinismo. Ambos so caractersticas do mito
grego e abordam a relao entre liberdade humana e providncia divina. A expresso filosfica que toma como
pressuposta a tese do determinismo :

a) Nasci para satisfazer a grande necessidade que eu tinha de mim mesmo. Jean Paul Sartre
b) Ter f assinar uma folha em branco e deixar que Deus nela escreva o que quiser. Santo Agostinho
c) Quem no tem medo da vida tambm no tem medo da morte. Arthur Shopenhauer
d) No me pergunte quem sou eu e no me diga para permanecer o mesmo. Michel Foucault
e) O homem, em seu orgulho, criou a Deus a sua imagem e semelhana. Friedrich Nietzche

257









PROF.: Fernando Viana





C
C
o
o
n
n
t
t
e
e

d
d
o
o
s
s
:
:

A
A
l
l
t
t
a
a
e
e
B
B
a
a
i
i
x
x
a
a
I
I
d
d
a
a
d
d
e
e
M
M

d
d
i
i
a
a
.
.






M
M
i
i
n
n
i
i
s
s
t
t
r
r
a
a
d
d
o
o
s
s
d
d
u
u
r
r
a
a
n
n
t
t
e
e
o
o
m
m

s
s
d
d
e
e
A
A
b
b
r
r
i
i
l
l














D Da at ta a d da a E En nt tr re eg ga a : : _ __ __ __ __ __ __ __ _/ /_ __ __ __ __ __ __ __ __ __ __ _/ /2 20 01 13 3

LISTA 3 HISTRIA

258


1. (UEPB) O Imprio Romano do Ocidente sofre a sua desintegrao a partir do sculo IV da era crist, ao tempo que
avanam os povos brbaros, como eram chamados aqueles que no pertenciam ao mundo romano. Quanto aos
brbaros correto afirmar:

a) os hunos, grupo tribal ordinrio da sia Central, penetraram pacificamente em territrios romanos convivendo
harmonicamente com os germnicos.
b) os brbaros no incorporaram costumes e normas romanas, ficando assim fiis s suas tradies.
c) com o aprofundamento da crise do imprio romano, integrantes de tribos brbaras adentraram pacificamente as
fronteiras romanas.
d) o renascimento carolngio condenou as obras clssicas, o teocentrismo e o trabalho dos monges copistas.
e) a adoo do direito romano, como referncia jurdica, extinguiu em todo o territrio romano a prtica do direito
consuetudinrio e dos costumes tribais brbaros.

02 - (UFPR) No ano 313 d.C., o imperador Constantino reconheceu o cristianismo como a religio oficial do Imprio
Romano, por meio do dito de Milo. Sobre o cristianismo na Antiguidade, INCORRETO afirmar:

a) Os primeiros cristos sofreram grandes perseguies por motivos polticos.
b) Por serem politestas, os romanos inicialmente resistiram em aceitar o monotesmo cristo.
c) Durante a Antigidade, ocorreram converses ao cristianismo de muitos povos chamados brbaros.
d) No incio de sua formao, a Igreja Crist baseou sua estrutura na organizao do Imprio Romano, reproduzindo
tambm sua diviso de poder.
e) A partir do dito de Milo, ficou estabelecido que somente autoridades religiosas poderiam determinar os rumos da
Igreja.

03 - (UNICAMP SP) A longa presena de povos rabes no norte da frica, mesmo antes de Maom, possibilitou uma
interao cultural, um conhecimento das lnguas e costumes, o que facilitou posteriormente a expanso do islamismo.
Por outro lado, deve-se considerar a superioridade blica de alguns povos africanos, como os sudaneses, que
efetivaram a converso e a conquista de vrios grupos na regio da Nbia, promovendo uma expanso do Isl que no
se apoia na presena rabe.
(Adaptado de Luiz Arnaut e Ana Mnica Lopes, Histria da frica: uma
introduo. Belo Horizonte: Crislida, 2005, p. 29-30.)

Sobre a presena islmica na frica correto afirmar que:

a) O princpio religioso do esforo de converso, a jihad, foi marcado pela violncia no norte da frica e pela
aceitao do islamismo em todo o continente africano.
b) Os processos de interao cultural entre rabes e africanos, como os propiciados pelas relaes comerciais, so
anteriores ao surgimento do islamismo.
c) A expanso do islamismo na frica ocorreu pela ao dos rabes, suprimindo as crenas religiosas tradicionais do
continente.
d) O islamismo a principal religio dos povos africanos e sua expanso ocorreu durante a corrida imperialista do
sculo XIX.

04 - (UFT TO) Os mercadores e autoridades de Meca, para quem o culto aos dolos era uma fonte de lucro, no
aceitaram o monotesmo de Maom e passaram a persegui-lo. Maom e seus seguidores fugiram para Yatreb (atual
Medina), a 400 quilmetros de Meca, onde fundou uma comunidade de fiis em 622 d.C. Esse episdio que marca as
origens do islamismo ficou conhecido como:
a) Ramad
b) Jihad
c) Muazin
d) Khaid
e) Hgira



Caderno de Atividades

Disciplina:
Histria

Professor(a):
Fernando
Aluno:
3 ano
Ensino Mdio
Data de Recebimento:
_____/_____/_____
Lista 03
Data Entrega:
_____/_____/_____

259
05 - (UPE) At o incio do sculo XX, os historiadores davam pouca importncia ao perodo final do chamado Mundo
antigo, ainda deslumbrados com a grandeza alcanada pela civilizao criada pelos gregos e romanos e entristecidos
ante trgica viso de uma Europa barbarizada. Pouco se reconhecia das contribuies trazidas pelos povos
germanos para a formao de uma nova sociedade.
(OLIVEIRA, Waldir Freitas. A Antiguidade Tardia. So Paulo: Editora tica, 1990).
Sobre as modificaes ocorridas nesse perodo de transio, analise as afirmativas a seguir:

I. No sculo IV, no Ocidente, a Igreja triunfara e se tornara, na prtica, a dirigente das mentalidades no Imprio.
II. As manifestaes artsticas da poca, a presena de cenas e de figuras ligadas ao cristianismo passam a ser
dominantes.
III. Os germanos destruram a cultura romana e adaptaram a sua nova sociedade, mudando completamente o
cenrio.
IV. O surgimento do latim vulgar foi utilizado como lngua franca, principalmente pelos proprietrios, pelos cobradores
de impostos e pelos bispos, o qual vai dar origem s lnguas neolatinas.
V. As principais estruturas germnicas que entraram na composio do feudalismo foram: colonato, vilas romanas,
direito consuetudinrio.

Esto CORRETAS

a) I, II e III.
b) I, IV e V.
c) I, II e IV.
d) II, III e IV.
e) III, IV e V.

06 - (UNESP SP) Um autor do sculo VI assim descreveu o rei tila, que, comandando os hunos, chegou s portas
de Roma:

Homem vindo ao mundo em um entrechoque de raas, terror de todos os pases, no sei como ele semeava tanto
pavor, a no ser pela ligao que se fazia de sua pessoa com um sentimento de terror. Tinha um porte altivo e um
olhar singularmente mvel, se bem que cada um de seus movimentos traduzisse o orgulho de seu poder. (...) sua
pequena-estatura, seu peito largo, sua cabea grande, seus olhos minsculos, sua barba rala, sua cabeleira eriada,
seu nariz muito curto, sua tez escura, eram sinais de suas origens.
(Jordanes. Getica XXXV (c. 551), citado por Jaime Pinsky (org.). O modo de produo feudal, 1982.)
Ao representar tila, que imagem dos brbaros o autor transmite?

07 - (UFBA) O Imprio Bizantino, que atravessou a Idade Mdia chegando at o incio da Idade Moderna,
caracterizou-se, dentre outros aspectos, pelo carter multicultural de sua sociedade.
Com base nos conhecimentos sobre essa sociedade, identifique e explique trs razes que justifiquem essa afirmao.

08 - (UFRN) Os mapas abaixo representam a Europa Ocidental em dois momentos distintos: no final da Antigidade
(Mapa 1) e no incio da Idade Mdia (Mapa 2).
Mapa 1

MOTA, Myriam Becho; BRAICK, Patrcia Ramos. Histria: das cavernas ao terceiro milnio.
So Paulo: Moderna, 2002. p. 85.

260
Mapa 2

ARRUDA, Jos Jobson de A.; PILETTI, Nelson.
Toda a histria. So Paulo: tica, 1996. p. XI.

Com base nas informaes contidas nesses dois mapas,
a) analise cada uma das organizaes polticas da Europa Ocidental representadas, destacando as diferenas
existentes entre elas;
b) analise duas causas das mudanas ocorridas na configurao espacial do territrio representado nos mapas.

09 - (UFG GO) Leia o texto abaixo, que se refere histria do significado do trabalho.

Do ponto de vista da histria, uma das revolues do cristianismo no Ocidente, reforada pela tradio monstica hostil
ao cio, ter feito do trabalho um valor.
IOGNA-PRAT, Dominique. Ordem (ns). In: Dicionrio temtico do ocidente
medieval. Bauru/SP: EDUSC: Imprensa Oficial do Estado de So Paulo.
p. 313. [Adaptado].

A respeito da histria da concepo de trabalho, pode-se afirmar que, na
a) Grcia Antiga, as atividades manuais eram consideradas socialmente superiores.
b) Roma Antiga, o estatuto da escravido limitava o trabalho do escravo s atividades no campo.
c) Roma republicana, o trabalho foi pensado como preo a ser pago pelo castigo decorrente do pecado original.
d) Idade Mdia, concebeu-se o trabalho como meio pelo qual o fiel poderia elevar-se de sua condio mundana.
e) Baixa Idade Mdia, o estatuto do trabalho nas cidades era semelhante ao da servido nos campos.

10 - (UFSCAR SP) Conforme lembrou Marc Bloch, o recurso maquinaria era apenas um meio de os monges se
conservarem disponveis para o mais importante, o essencial, quer dizer, o Opus Dei, a orao, a vida contemplativa.
Longe de ser uma instalao corrente, o moinho era uma raridade, uma curiosidade, e a sua construo por monges
passava, aos olhos contemporneos, mais como prova de saber quase sobrenatural, quase traumatrgico dos
monges, do que como exemplo de sua habilidade tcnica. (...)
Este trabalho monstico tem, sobretudo, aspecto penitencial. porque o trabalho manual se liga queda, maldio
divina e penitncia, que os monges, penitentes profissionais, penitentes de vocao, penitentes por excelncia,
devem dar esse exemplo de mortificao.
(Jacques Le Goff. Para um novo conceito de Idade Mdia, 1993.)
a) Quem exercia o trabalho manual na Europa na Idade Mdia? Quais valores predominavam em relao ao trabalho
manual?
b) Cite um exemplo de valorizao do trabalho manual na Idade Mdia europia.

11 - (UNICAMP SP) A igreja era, com freqncia, o nico edifcio de pedra em toda a redondeza; era a nica grande
construo em muitas lguas e seu campanrio era um ponto de referncia. Aos domingos e durante o culto, todos os
habitantes podiam encontrar-se ali, e o contraste entre o edifcio grandioso, com suas pinturas, talhas e esculturas, e
as casas humildes em que as pessoas viviam, era esmagador.
(Adaptado de E.H. Gombrich, Histria da Arte. Rio de Janeiro: LTC Editora, 1993, p. 126).
a) Baseado no texto, indique trs caractersticas do edifcio da igreja na cidade medieval.
b) Identifique as formas de divulgao da f catlica durante a Idade Mdia.



261
12 - (UFG GO) O Mediterrneo nem sequer um mar, antes um complexo de mares, de mares pejados de
ilhas, cortados por pennsulas, cercados por costas rendilhadas; a sua vida est ligada terra, a sua poesia
predominantemente rstica, os seus marinheiros so camponeses nas horas vagas; o mar dos olivais e das vinhas,
tanto como dos esguios barcos e remos ou dos redondos navios dos mercadores, e a sua histria no pode ser
separada do mundo terrestre que o envolve, tal como a argila o no pode ser do arteso que a modela.
(Braudel, Fernand. O Mediterrneo e o mundo mediterrnico. Martins Fontes, 1983. Vl. 1. p. 21/22)
Analise a importncia do Mediterrneo nos seguintes contextos:
a) na Antigidade clssica;
b) na alta idade mdia.

13 - (UEG GO) As relaes de suserania e vassalagem constituem as bases da estrutura social medieval. Explique o
que so essas relaes.

14 - (UNICAMP SP) At o sculo XII, a mulher era desprezada por ser considerada incapaz para o manejo de armas;
vivendo num ambiente guerreiro, no se lhe atribua outra funo alm de procriar. A sua situao no era mais
favorvel do ponto de vista espiritual; a Igreja no perdoava Eva por ter levado a humanidade perdio e continuava
a ver em suas descendentes os aclitos lbricos do demnio.
(Adaptado de Pierre Bonassie, Amor corts, em Dicionrio de Histria Medieval. Lisboa: Publicaes D. Quixote, 1985,
p. 29-30.)

a) Identifique no texto as razes para a mulher ser considerada inferior na sociedade medieval.
b) Quais caractersticas da sociedade medieval configuraram um ambiente guerreiro at o sculo XII?

15 - (UNESP SP) O sistema feudal, em ltima anlise, repousava sobre uma organizao que, em troca da proteo,
frequentemente ilusria, deixava as camadas de trabalhadores merc das camadas parasitrias, e concedia a terra
no a quem a cultivava, mas aos capazes de dela se apoderarem.
(P. Boissonade, Vida e trabalho na Europa medieval. apud Leo Huberman, Histria da Riqueza do Homem)

Explique a estrutura da sociedade feudal, destacando as relaes econmicas e as relaes de poder entre as
diferentes camadas que dela faziam parte.

16 - (UNESP SP) As horas cannicas eram anunciadas pelo toque dos sinos, que mandavam distncia o som que
funcionava como voz da eternidade, marcando o tempo de todas as pessoas. Tempo de repouso e tempo de trabalho;
tempo de orao e tempo de festa; tempo de vida e tempo de morte.
(Paulo Miceli, O feudalismo.)

O operrio transforma-se, por sua vez, num especialista em olhar o relgio, preocupado apenas em saber quando
poder escapar para gozar as suas escassas e montonas formas de lazer que a sociedade industrial lhe proporciona.
(George Woodcock, Os grandes escritos anarquistas.)

Nos dois momentos histricos descritos, considerando o cotidiano do homem, compare a percepo e o controle do
tempo.

17 - (UFG GO)
Voc me proibiu, senhora,
de que lhe dissesse qualquer coisa
sobre o quanto sofro por sua causa.
Mas ento me diga,
por Deus, senhora: a quem falarei
o quanto sofro e j sofri por voc
seno a voc mesma?
DON DINIS. Cantiga de amor. Apud CEREJA, Willian Rodrigues;
MAGALHES, Thereza Cochar. Panorama da literatura Portuguesa.
So Paulo: Atual, 1997. p. 13. [Adaptado].

Na produo potica medieval, entre outros gneros, encontram-se as cantigas de amor, que evocam o ideal de amor
corts e encenam, no jogo amoroso, as relaes entre os nobres. Com base no trecho citado,
a) identifique quem ocupa o papel de suserano, na cena do jogo amoroso;
b) caracterize o ideal de amor corts introduzido no universo da nobreza a partir do sculo XII.

18 - (FUVEST SP) Na Europa Ocidental, durante a Idade Mdia, o auge do feudalismo (sculo X ao XIII) coincide com
o auge da servido. Explique
a) no que consistia a servido.
b) por que a servido entrou em crise e deixou de ser dominante a partir do sculo XIV.

262
19 - (UFBA) Identifique e explique dois elementos marcantes da influncia da Igreja dominante na sociedade
medieval.

20 - (FUVEST SP) Na passagem da poca romana para a poca medieval, houve no s rupturas, mas tambm
continuidades.
Caracterize essas continuidades no campo da
a) religio.
b) lngua.

21 - (UFG GO) Leia o texto a seguir.
Origens do regime feudal, diz-se. Onde busc-las? Alguns responderam em Roma. Outros na Germnia. As razes
dessas miragens so evidentes []. Das duas partes, sobretudo, eram empregadas palavras tais como benefcio
(beneficium) para os latinos, feudo para os germanos das quais essas geraes persistiram em se servir, ainda que
lhes conferindo, sem se dar conta, um contedo quase inteiramente novo. Pois, para o grande desespero dos
historiadores, os homens no tm o hbito, a cada vez que mudam o costume, de mudar de vocabulrio.
BLOCH, Marc. Apologia da Histria ou o ofcio do historiador.
Rio de Janeiro: Zahar. p. 58. (Adaptado).

Neste fragmento, Marc Bloch discute de que forma os historiadores lidam com a questo das origens, indicando que a

a) origem dos fenmenos histricos deve ser buscada no encadeamento dos acontecimentos, o que confere Histria
um sentido de continuidade.
b) origem o ponto de partida da mudana que demarca a ruptura com as formas histricas precedentes.
c) ideia de origem desconsidera a cronologia, ferramenta metodolgica que concede sentido explicao histrica.
d) busca da origem dos fenmenos histricos encobre a relao entre as foras de conservao e de mudana que
compem a vida social.
e) origem dos fenmenos histricos pode ser encontrada na permanncia dos costumes e do uso do vocabulrio.

22 - (UFJF MG) Observe a ilustrao e leia a citao abaixo. Em seguida, responda ao que se pede.


Coroao de Carlos Magno como imperador do Sacro Imprio Romano-Germnico, em dezembro de 800 d.C., pelo papa Leo III.
Fonte: Disponvel em: <http://www.suapesquisa.com/historia>.Acesso em: 8 out. 2011.

Nascida nos quadros do Imprio Romano, a Igreja ia aos poucos preenchendo os vazios deixados por ele at, em fins
do sculo IV, identificar-se com o Estado, quando o cristianismo foi reconhecido como religio oficial. (...) Estreitavam-
se, portanto, as relaes Estado-Igreja. (...) No Imprio Carolngio, a aliana entre os reis e a Igreja foi fundamental
para a consolidao de ambos os poderes e, por vezes, a Igreja assumia funes que hoje consideramos ser do
Estado e este por sua vez interferia nos assuntos religiosos.
FRANCO JNIOR, Hilrio. A Idade Mdia. Nascimento do Ocidente. So Paulo: Brasiliense, 2001. p.67,71

Sobre as relaes entre Estado e Igreja, no perodo medieval, responda:
a) Qual a importncia da Igreja Catlica na administrao dos reinos e imprios?
b) De que maneira o poder rgio contribuiu para a expanso da f crist?

23 - (UEG GO) Durante seu reinado, Carlos Magno buscou reverter o quadro de estagnao cultural gerado pelas
invases brbaras, quando muito do conhecimento da Antiguidade clssica havia se perdido. Reuniu ento, com o
apoio da Igreja, grandes sbios que deveriam transmitir sua sabedoria nas escolas da poca. Esses grandes mestres
foram chamados scholasticos. As matrias ensinadas por eles nas escolas medievais eram chamadas de artes liberais
e foram divididas em
a) f e razo.
b) matemtica e gramtica.
c) trvio e quadrvio.
d) teologia e filosofia.

263

24 - (ENEM Simulado) Para uns, a Idade Mdia foi uma poca de trevas, pestes fome, guerras sanguinrias,
supersties, crueldade. Para outros, uma poca de bons cavaleiros, damas, corteses, fadas, guerras honradas,
torneios, grandes idias. Ou seja, uma Idade Mdia m e uma Idade Mdia boa.

Tal disparidade de apreciaes com relao a esse perodo da Histria se deve

a) ao Renascimento, que comeou a valorizar a comparao documental do passado, formando acervos documentais
que mostram tanto a realidade boa quanto a m.
b) tradio iluminista, que usou a Idade Mdia como contraponto a seus valores racionalistas, e ao Romantismo,
que pretendia ressaltar as boas origens das naes.
c) indstria de videojogos e cinema, que encontrou uma fonte de inspirao nessa mistura de fantasia e realidade,
construindo uma viso falseada do real.
d) ao Positivismo, que realou os aspectos positivos da Idade Mdia, e o Marxismo, que denunciou o lado negativo
do modo de produo feudal.
e) religio, que com sua viso dualista e maniquesta do mundo, alimentou tais interpretaes sobre a Idade Mdia.

25 - (UNESP SP) [Na Idade Mdia] Homens e mulheres gostavam muito de festas. Isso vinha, geralmente, tanto das
velhas tradies pags (...), quanto da liturgia crist.
(Jacques Le Goff. A Idade Mdia explicada aos meus filhos, 2007.)

Sobre essas festas medievais, podemos dizer que

a) muitos relatos do cotidiano medieval indicam que havia um confronto entre as festas de origem pag e as criadas
pelo cristianismo.
b) os torneios eram as principais festas e rompiam as distines sociais entre senhores e servos que, montados em
cavalos, se divertiam juntos.
c) a Igreja Catlica apoiava todo tipo de comemorao popular, mesmo quando se tratava do culto a alguma
divindade pag.
d) as festas rurais representavam sempre as relaes sociais presentes no campo, com a encenao do ritual de
sagrao de cavaleiros.
e) religiosos e nobres preferiam as festas privadas e pags, recusando-se a participar dos grandes eventos pblicos
cristos.

















264







PROF.: Fernando Viana






C
C
o
o
n
n
t
t
e
e

d
d
o
o
s
s
:
:
R
R
e
e
n
n
a
a
s
s
c
c
i
i
m
m
e
e
n
n
t
t
o
o
c
c
u
u
l
l
t
t
u
u
r
r
a
a
l
l
e
e
a
a
s
s

t
t
r
r
a
a
n
n
s
s
f
f
o
o
r
r
m
m
a
a

e
e
s
s
v
v
i
i
v
v
e
e
n
n
c
c
i
i
a
a
d
d
a
a
s
s
p
p
e
e
l
l
o
o
c
c
r
r
i
i
s
s
t
t
i
i
a
a
n
n
i
i
s
s
m
m
o
o

























D Da at ta a d da a E En nt tr re eg ga a : : _ __ __ __ __ __ __ __ _/ /_ __ __ __ __ __ __ __ __ __ __ _/ /2 20 01 13 3
LISTA 4 HISTRIA

265

01 - (UFF RJ) Um dos aspectos mais importantes da nova ordem decorrente do Renascimento foi a formao das
repblicas italianas. Dentre elas, se destacaram Florena e Veneza. Essas repblicas inovaram no sentido das suas
formas de governo, assim como na redefinio do lugar do homem no mundo, inspirando a partir da novas formas de
represent-lo.

a) Tomando o caso de Florena, explique como funcionavam as repblicas italianas, levando em conta a organizao
poltica e os vnculos entre os cidados e a cidade, e indique o nome do principal representante das ideias sobre a
poltica florentina no sculo XVI.
b) Analise o papel de Veneza no desenvolvimento do comrcio europeu, e suas relaes com o Oriente.

02 - (UFG GO) Analise as imagens a seguir.


Manuscrito alemo do sculo XV. Disponvel em: <http://www.
expositions.bnf.fr/ciel/grandmanall.htm>. Acesso em: 31 out. 2011.

DA VINCI, Leonardo. Homem vitruviano, 1492.
Disponvel em: <http://www.cafehistoria.ning.com/profiles/
blogs/o-homem-vitruviano-de-leonardo>. Acesso em: 10 out. 2011.

As duas imagens datam do sculo XV e integram estudos sobre o corpo humano. A primeira ilustra um manuscrito
annimo do incio do sculo, enquanto a segunda retrata o Homem vitruviano, esboado por Leonardo da Vinci em
1492. Elas expressam o convvio entre representaes distintas do corpo humano.

Tendo em vista essas informaes e comparando as duas imagens,

a) relacione a mudana na representao do corpo humano ao desenvolvimento cientfico no perodo;
b) identifique um elemento pictrico, explicando de que forma ele exemplifica essa mudana.


Caderno de Atividades

Disciplina:
Histria

Professor(a):
Fernando
Aluno:
3 ano
Ensino Mdio
Data de Recebimento:
_____/_____/_____
Lista 04
Data Entrega:
_____/_____/_____

266
03 - (UFJF MG) As imagens abaixo ilustram alguns procedimentos utilizados por um novo modo de conhecer e
explicar a realidade que se estruturou entre os sculos XVI e XVIII.


Ilustrao do Sistema solar no manuscrito de Coprnico na obra Das Revolues das esferas celestes
Fonte: Disponvel em: <http://pt.wikipedia.org>. Acesso em: 8 out. 2011.


Ilustrao de Andreas Vesalius na obra Da Organizao do Corpo Humano
Fonte: Disponvel em: <http://pt.wikipedia.org>. Acesso em: 8 out. 2011.

Com base nas informaes acima e em seus conhecimentos, responda ao que se pede:

a) Que processo histrico pode ser identificado pelas referncias acima?
b) Cite e analise uma caracterstica desse novo modo de conceber o conhecimento.
c) Explique o impacto desse novo modo de conceber o conhecimento sobre os dogmas religiosos vigentes na poca.

04 - (UFBA) O Renascimento, como expresso de concepes inovadoras de artistas, escritores e cientistas, marcou
o campo cultural e o cientfico da civilizao europeia ocidental.

Partindo dos conhecimentos sobre o movimento renascentista, indique uma concepo, relativa a cada grupo
indicado, responsvel por modificaes na mentalidade da poca.

Concepo inovadora de
artistas:
escritores:
cientistas:






267
05 - (UFG GO) Leia o texto a seguir.

Enquanto andava procura de ossos pelas estradas rurais, onde eventualmente os indivduos executados so
deixados, deparei-me com um cadver ressecado. Os ossos estavam totalmente expostos, mantendo-se unidos
apenas pelos ligamentos, e tinham sido preservadas somente a origem e a insero dos msculos. Escalei o poste e
destaquei o fmur do osso ilaco. Quando puxei a pea com fora, a omoplata, os braos e as mos tambm se
destacaram, embora faltassem os dedos de uma das mos, as duas rtulas e um dos ps. Depois de trazer
secretamente para casa as pernas e os braos e aps sucessivas idas e vindas (tinha deixado para trs a cabea e o
tronco), permaneci durante quase toda noite fora dos limites da cidade a fim de conseguir pegar o trax, que se
encontrava firmemente preso a uma corrente.
VESALIUS, Andreas. De humani corporis fabrica, 1543. Disponvel em: www.cienciahoje.uol.com.br/noticias/historia-
da-ciencia-e-epistemologia/
relancado-tratado-que-inaugurou-anatomia-moderna/.
Acesso em: 11 out.. 2010. [Adaptado]

Datada de 1543, a narrao do mdico Andreas Vesalius, considerado o precursor dos estudos de anatomia moderna,
indica a formao de um novo modelo de conhecimento, no perodo. Com base na leitura do texto e considerando o
contexto histrico,

a) explique o processo pelo qual a concepo de mundo dos homens se transformou na poca do Renascimento;
b) analise a concepo de cincia que se formava, apresentando o conflito social que essa nova concepo gerou.

06 - (UERJ) O casal Arnolfini

JAN VAN EYCK (1389-1441)
http://upload.wikimedia.org

Sempre que se evoca o tema do Renascimento, a imagem que nos vem mente a dos grandes artistas e de suas
obras mais famosas. Isso nos coloca a questo: por que razo o Renascimento implica esse destaque to grande dado
s artes visuais? De fato, as artes plsticas acabaram se convertendo num centro de convergncia de todas as
principais tendncias da cultura renascentista. E mais do que isso, acabaram espelhando os impulsos mais marcantes
do processo de evoluo das relaes sociais e mercantis.
NICOLAU SEVCENKO
Adaptado de O Renascimento. So Paulo: Atual; Campinas: Ed. Unicamp, 1984.

As diversas manifestaes da cultura renascentista na Europa ocidental, entre os sculos XIV e XVI, estiveram
relacionadas criao de novos valores e prticas sociais que se confrontaram com aqueles da sociedade medieval.
Cite dois aspectos da cultura renascentista que justifiquem a sua importncia para o incio dos Tempos Modernos.

07 - (FUVEST SP) Observe a imagem e leia o texto a seguir.


Fonte: Michelangelo, A criao de Ado, detalhe do teto da
Capela Sistina, Vaticano (c. 1511). www.rastel.com.


268
Michelangelo comeou cedo na arte de dissecar cadveres. Tinha apenas 13 anos quando participou das primeiras
sesses. A ligao do artista com a medicina foi reflexo da efervescncia cultural e cientfica do Renascimento. A
prtica da dissecao, que se encontrava dormente havia 1.400 anos, foi retomada e exerceu influncia decisiva sobre
a arte que ento se produzia.
Clayton Levy, Pesquisadores dissecam lio de anatomia de Michelangelo,
Jornal da Unicamp, n 256, junho de 2004,
http://www.unicamp.br/unicamp/unicamp_hoje/ju/junho2004/ju256pag1.html. Acessado em 11/06/2010.

a) Explique a relao, mencionada no texto, entre artes plsticas e dissecao de cadveres, no contexto do
Renascimento.
b) Identifique, na imagem acima, duas caractersticas da arte renascentista.

08 - (UNESP SP) A arte renascentista uma arte de pesquisa, de invenes, inovaes e aperfeioamentos tcnicos.
Ela acompanha paralelamente as conquistas da fsica, da matemtica, da geometria, da anatomia, da engenharia e da
filosofia.
(Nicolau Sevcenko. O renascimento. So Paulo: Atual, 1985.)


(Filippo Brunelleschi, Igreja de San Lorenzo (interior), Florena, c. 1421.)

(Michelangelo Buonarroti, Biblioteca Laurenziana (sala), Florena, construda entre 1523 e 1525.)

A partir do texto e das imagens, caracterize a concepo artstica do Renascimento e sua relao com o Humanismo
difundido no sculo XV.

09 - (UNESP SP) Em 19 de fevereiro de 1616, o Santo Ofcio passou aos seus telogos as duas proposies que
resumiam o ncleo da questo para que fossem examinadas. As duas proposies eram as seguintes: a) Que o Sol
o centro do mundo, sendo consequentemente imvel de movimento local. b) Que a Terra no est no centro do
mundo nem imvel, mas move-se por si mesma. Cinco dias depois, todos os telogos de acordo, sentenciaram que
a primeira proposio era tola e absurda em filosofia e formalmente hertica, enquanto contrastava com as sentenas
da Sagrada Escritura em seu significado literal e segundo a exposio comum dos Santos Padres e dos doutores em
teologia.
(Reale e Antiseri. Histria da Filosofia, 2000. Adaptado.)

O texto descreve os motivos que levaram condenao do filsofo Galileu Galilei por uma instituio religiosa.
Responda qual foi a instituio que o condenou e explique os motivos dessa condenao.

10 - (UNESP SP) Sobre os humanistas, afirma-se:
Eram todos cristos e apenas desejavam reinterpretar a mensagem do Evangelho luz da experincia e dos valores
da Antiguidade. Valores esses que exaltavam o indivduo, os feitos histricos, a vontade e a capacidade de ao do
homem, sua liberdade de atuao e de participao na vida das cidades.
(Nicolau Sevcenko, O Renascimento.)

A partir do texto, caracterize o contexto histrico no qual o humanismo floresceu.

269
11 - (UFJF MG) O texto e a gravura abaixo se referem ao contexto de um importante processo histrico ocorrido no
sculo XVI em vrios pases europeus.


Gravura retratando a venda de indulgncias - O Anticristo, de Lucas Cranach, 1521.
Disponvel em: <http://www.en.wikipedia.org>. Acesso em: 14 jun. 2010.

Deus chama cada um para uma vocao particular cujo objetivo a glorificao dele mesmo. O comerciante que
busca o lucro, pelas qualidades que o sucesso econmico exige: o trabalho, a sobriedade, a ordem, responde tambm
ao chamado de Deus, santificando de seu lado o mundo pelo esforo, e sua ao santa. (CALVINO, Joo apud
MOUSNIER, Roland). MOUSNIER, Roland. Os sculos XVI e XVII: os processos da civilizao europeia. In:
CROUZET, Maurice. Histria geral das civilizaes. 4. ed. So Paulo: Difel, 1973, p. 90. v. 1, tomo IV.

Com base nas referncias acima e em seus conhecimentos, responda ao que se pede.

1) Qual movimento pode ser identificado pelas referncias acima?
2) Identifique e analise dois desdobramentos desse movimento para a Europa moderna:
a) quanto aos aspectos econmicos:
b) quanto s questes religiosas:

12 - (PUC RJ) Observe a reproduo da gravura Os reformadores: Wycliffe, Huss, Lutero, Zwnglio, Calvino,
Melanchton, Bucer e Beza (1886).

http://www.virtualmuseum.ca/Exhibitions/Annodomini/THEME_13/IT/theme-it-13-1-zoom.html

a) A imagem sugere que a problemtica central desses reformadores era o retorno Bblia, s Sagradas Escrituras,
traduzidas e consideradas como o nico fundamento da f e da conduta para todos os seres humanos.
EXPLIQUE um motivo pelo qual a adoo desse princpio foi uma das causas das reformas religiosas no sculo XVI.
b) Na imagem, Calvino e Lutero esto enfileirados em primeiro plano, ressaltando a importncia de suas propostas para
a criao de novas igrejas, reformadas, na poca Moderna.
APRESENTE duas diferenas entre o luteranismo e o calvinismo.

13 - (UERJ) As relaes entre a pregao protestante e as estruturas polticas ento existentes foram muitas vezes
decisivas tanto para os destinos da pregao em si quanto para os rumos afinal tomados pela organizao das novas
Igrejas.
FRANCISCO JOS CALAZANS FALCON
In: RODRIGUES, Antonio Edmilson M. e FALCON, Francisco Jos C.
Tempos modernos: ensaios de histria cultural. Rio
de Janeiro: Civilizao Brasileira, 2000.
O texto acima se refere a processos da Reforma Religiosa ocorridos na Europa. O movimento reformista, entretanto,
conheceu diferentes reaes em distintas reas.
Indique duas causas para a Reforma Religiosa na Inglaterra e uma conseqncia econmica desse movimento.

270

14 - (UNICAMP SP) A base da teologia de Martinho Lutero reside na idia da completa indignidade do homem, cujas
vontades esto sempre escravizadas ao pecado. A vontade de Deus permanece sempre eterna e insondvel e o
homem jamais pode esperar salvar-se por seus prprios esforos. Para Lutero, alguns homens esto predestinados
salvao e outros condenao eterna. O essencial de sua doutrina que a salvao se d pela f na justia, graa e
misericrdia divinas.
(Adaptado de Quentin Skinner, As fundaes do pensamento
poltico moderno. So Paulo: Companhia
das Letras, 1996, p. 288-290.)

a) Segundo o texto, quais eram as idias de Lutero sobre a salvao?
b) Quais foram as reaes da Igreja Catlica Reforma Protestante?

15 - (UFRRJ) Votos da Companhia de Jesus, criada por Incio Loiola em 1534:
Que os membros consagraro suas vidas ao constante servio de Cristo e do Papa, lutaro sob a bandeira da Cruz e
serviro ao Senhor Pontfice romano como o vigrio de Deus na Terra, de tal forma que executaro imediatamente e
sem vacilao ou escusa tudo o que o Pontfice reinante ou seus sucessores puderem ordenar-lhes para proveito das
almas ou para propagao da f, e assim agiro em toda provncia aonde forem enviados, entre turcos ou quaisquer
outros infiis, na ndia distante, assim como em regio de hereges, cismticos ou indivduos de qualquer tipo.
LOIOLA. I. Companhia de Jesus. In: AQUINO, R. S.L & ALVARENGA,
F. J. M. & FRANCO, D.A.& LOPES, O.G.P.L.- Histria Das
Sociedades: Das Sociedades Modernas s Sociedades Atuais.
Rio de Janeiro: Ao Livro Tcnico, 1990, p.87.

O texto acima apresenta os votos da Companhia de Jesus, que foi uma reao da Igreja Catlica contra a Reforma
Protestante. Sobre a Contra-Reforma, correto afirmar que promoveu
a) o Conclio de Trento, no qual foram modificados diversos dogmas da Igreja Catlica.
b) o restabelecimento do Tribunal do Santo Ofcio, que servia para julgar aqueles que defendiam a manuteno dos
dogmas catlicos, contra a nova orientao da Igreja.
c) a reorganizao do Tribunal do Santo Ofcio, que servia para julgar os hereges, tendo uma atuao mais presente
na Pennsula Ibrica.
d) a organizao da Companhia de Jesus, que tinha como objetivo julgar os hereges que eram contra os dogmas do
catolicismo.
e) o restabelecimento do Tribunal do Santo Ofcio que determinou quem iria para as colnias da Amrica para
catequizar os ndios.

16 - (UFJF MG) Na Carta aberta aos nobres cristos, Martinho Lutero afirma que uma terra desabitada ou povoada
por pagos uma terra selvagem. Se um grupo de cristos se encontra nessas circunstncias, seu dever no consiste
em converter os pagos. Os ndios americanos eram parte da natureza e, como todas as coisas da terra, estavam
contaminados pelo pecado e pela morte. Para esses nativos, portanto, no haveria salvao possvel. J os cristos
que se estabelecessem em uma situao onde no exista um sacerdote consagrado deveriam eles mesmos eleger o
seu.
Sobre a colonizao inglesa de carter puritano na Amrica do Norte, CORRETO afirmar que:
a) a doutrina protestante foi de mais fcil aceitao entre os nativos do Novo Mundo do que a catlica, por pressupor
uma relao direta entre o homem e Deus, sem intermediao da hierarquia eclesistica.
b) as origens da sociedade norte-americana na Nova Inglaterra, no incio do sculo XVII, no tm carter religioso, ao
contrrio da catlica Amrica Latina.
c) especificamente no que se refere ao estabelecimento de seus seguidores em terras consideradas de ningum, a
viso protestante de mundo favoreceu uma concepo autoritria de sua prpria organizao social.
d) a viso de mundo que guiou os puritanos que se estabeleceram na Amrica do Norte fundava-se na crtica Igreja
papista e romana.
e) a ocupao do territrio norte-americano pelos colonos puritanos, no sculo XVII, foi mais rpida que a das
Amricas Central e do Sul pelos catlicos espanhis no sculo XVI.

17 - (UNIFESP SP) Com a Reforma e a Contra-Reforma, os dois protagonistas principais de uma e de outra foram
Calvino e Incio de Loyola. Comente o papel e a importncia de
a) Calvino para o protestantismo.
b) Incio de Loyola para o catolicismo.

18 - (UEG GO) Conhecimento a relao que se estabelece entre o sujeito cognoscente e um objeto. Na Grcia
antiga no havia fragmentao do conhecimento, e pensar sobre um assunto envolvia a totalidade dos outros. Os
filsofos gregos da antiguidade se preocupavam basicamente com os problemas do ser e do no ser, da permanncia
e do movimento, da unidade e da multiplicidade das ideias e das coisas. J para o pensador medieval, o problema
principal era a conciliao entre f e razo. No Renascimento, surgem as seguintes grandes modificaes:

271

a) a unio entre f e razo, o fidesmo e o positivismo.
b) a unio entre f e razo, o teocentrismo e o interesse pela moral.
c) a valorizao da f em detrimento da razo, o cosmocentrismo e o fidesmo.
d) a separao entre f e razo, o antropocentrismo e o interesse pelo saber ativo.

19 - (ENEM Simulado) Normalidade II Quino

Disponvel em: <http://www.filosofia.com.br/figuras/charge/49.jpg>.
Acesso em: 26 mar. 2009.
Eu no vou mais precisar de muita fora, vou usar todas as que
tenho agora ele pensou. E ele se lembrou das moscas que
rebentam suas perninhas ao tentarem escapar do mata-moscas.
KAFKA, Franz. O processo. Porto Alegre: L & PM Pocket, 2007.
p. 258. (Fragmento).

O Controle Social pode ser tomado como um conjunto de
penalidades e aprovaes, chamadas tambm de sanes. Estas
so aplicadas aos indivduos pela sociedade para assegurar a
conformidade das condutas aos modelos estabelecidos. A
imagem e o texto destacam aspectos relativos
a) garantia de liberdade coletiva pelo uso da fora.
b) aos anseios idealistas utpicos perante as convenes sociais.
c) natureza imitadora da maioria dos indivduos diante das
instituies sociais.
d) possibilidade de reao e reverso de processos
condicionantes.
e) s relaes de poder presentes nas sociedades.

20 - (ENEM Simulado) Distantes uma da outra quase 100 anos, as duas telas seguintes, que integram o patrimnio
cultural brasileiro, valorizam a cena da primeira missa no Brasil, relatada na carta de Pero Vaz de Caminha. Enquanto
a primeira retrata fielmente a carta, a segunda ao excluir a natureza e os ndios critica a narrativa do escrivo da
frota de Cabral. Alm disso, na segunda, no se v a cruz fincada no altar.


Primeira Missa no Brasil Victor Meireles (1861)
Disponvel em: http:/www.moderna.com.br Acesso em: 3 nov. 2008.

Primeira Missa no Brasil Candido Portinari (1948)
Disponvel em: http:/www.casadeportinari.com.br Acesso em: 3 nov. 2008.


272
Ao comparar os quadros e levando-se em considerao a explicao dada, observa-se que

a) a influncia da religio catlica na catequizao do povo nativo objeto das duas telas.
b) a ausncia dos ndios na segunda tela significa que Portinari quis enaltecer o feito dos portugueses.
c) ambas, apesar de diferentes, retratam um mesmo momento e apresentam uma mesma viso do fato histrico.
d) a segunda tela, ao diminuir o destaque da cruz, nega a importncia da religio no processo dos descobrimentos.
e) a tela de Victor Meireles contribuiu para uma viso romantizada dos primeiros dias dos portugueses no Brasil.

21 - (FUVEST SP) Observe as rotas no mapa e responda:



a) O que representou, para os interesses de Portugal, a rota martima Lisboa-Cabo da Boa Esperana-Calicute?
b) O que significou a expedio de Pedro lvares Cabral para o Imprio Portugus?

22 - (UEG GO) O mercantilismo foi o conjunto de prticas econmicas dos estados absolutistas europeus, durante a
idade moderna. Considerando esta afirmao, responda:

a) Como ficaram conhecidas as prticas mercantilistas na Inglaterra e na Espanha?
b) Qual era o papel das colnias dentro do Mercantilismo?

23 - (UNIFESP SP) Mercantilismo o nome normalmente dado poltica econmica de alguns Estados Modernos
europeus, desenvolvida entre os sculos XV e XVIII. Indique

a) duas caractersticas do Mercantilismo.
b) a relao entre o Mercantilismo e a colonizao da Amrica.

24 - (UFTM MG) Observe a imagem, que ilustra a capa de um manuscrito ingls do sculo XVI.


(www.bridgemanart.com)

correto relacionar a imagem

a) com prticas em vigor desde a Idade Mdia, perodo no qual se incentivou o estudo do corpo humano por meio da
dissecao.

273
b) s prticas de bruxaria ento em voga, que garantiam ser possvel determinar a localizao exata da alma
humana.
c) aos processos inquisitoriais e s torturas impostas aos acusados, que eram considerados possudos pelo demnio.
d) crena na razo e na capacidade humana de compreender os processos naturais e biolgicos, postura tpica do
Renascimento.
e) s prticas astrolgicas e mgicas, ampliadas nos sculos XV e XVI, graas ao contato com as sociedades
indgenas das Amricas.

TEXTO: 1 - Comum questo: 25

A F-1 comeou a perder as caractersticas que encantaram geraes nos anos 1990 quando o salto tecnolgico
tornou o piloto quase um coadjuvante no cockpit. Os carros de corrida so equipamentos e no mais automveis. No
volante, h mais de 100 botes. O condutor virou um operador de mquinas, reclama Bird Clemente, 72 anos,
primeiro brasileiro a guiar, profissionalmente, um carro de corrida. No passado, esse esporte dependia muito mais do
talento do piloto para regular um carro. Hoje, espremido no cockpit como mais um funcionrio de um negcio que
movimenta bilhes de dlares, o piloto cumpre religiosamente as regras do mercado.
(Adaptado de: CARDOSO, R.; LOES, J., O Esporte Perdeu. Revista Isto
, 4 ago. 2010, ano 34, n. 2125, p. 84-85.)

25 - (UEL PR) A lgica do esporte e da fruio englobada pela lgica do mercado. A importncia dada ao negcio
(negar o cio), conforme anlise de Max Weber em sua tica Protestante e o Esprito do Capitalismo, revela que

I. o trabalho atende s regras do mercado, destacando a prevalncia do negcio, em razo da necessidade de
produo capitalista.
II. a dimenso religiosa, presente nos primrdios do capitalismo, na figura do protestantismo de orientao luterana,
valoriza o carter sagrado da atividade fabril, em detrimento do trabalho braal.
III. o negcio, quando praticado de acordo com os preceitos divinos, viabiliza a distribuio igual e solidria das
riquezas produzidas.
IV. o ato de negociar, prprio do comrcio, depende da fora produtiva, conectada diviso social do trabalho no
mundo secularizado.

Assinale a alternativa correta.

a) Somente as afirmativas I e II so corretas.
b) Somente as afirmativas I e IV so corretas.
c) Somente as afirmativas III e IV so corretas.
d) Somente as afirmativas I, II e III so corretas.
e) Somente as afirmativas II, III e IV so corretas.


274









PROF.: Fernando Viana












M
M
i
i
n
n
i
i
s
s
t
t
r
r
a
a
d
d
o
o
n
n
o
o
m
m

s
s
d
d
e
e
S
S
e
e
t
t
e
e
m
m
b
b
r
r
o
o
















D Da at ta a d da a E En nt tr re eg ga a : : _ __ __ __ __ __ __ __ _/ /_ __ __ __ __ __ __ __ __ __ __ _/ /2 20 01 13 3
LISTA 5 HISTRIA

275

01 - (UFRN) A charge abaixo, publicada na Frana em 1885, refere-se a um episdio especfico de um fenmeno
histrico, cujas repercusses atingiram diversos continentes at as primeiras dcadas do sculo XX.


Disponvel em: www.chaodeareia.agcolares.org. Acesso em: 20 jun. 2011.

Analise os elementos que compem a charge e responda:

a) Qual o fenmeno histrico a que ela faz referncia? Entre o sculo XIX e as
primeiras dcadas do sculo XX, que relaes de poder existiam entre as naes?
b) Mencione dois aspectos (acontecimentos ou ideias) que se relacionam a esse
fenmeno histrico.

02 - (UEG GO)

(Folha de S. Paulo, 1/4/2003.)
LAERTE. In: CEREJA, William Roberto; MAGALHES, Thereza
Cochar. Portugus linguagens: volume 3. So Paulo: Atual, 2004. p. 221.

As tiras so um importante instrumento lingustico em que a linguagem verbal e a no verbal combinam-se na
construo de um recurso comunicativo humorstico e, s vezes, crtico da realidade. Nesse sentido, a tira citada
pertinente para se fazer uma leitura

a) adequada das lutas dos movimentos negros norte-americanos dos anos 1970, j que conseguiram significativos
avanos sociais.
b) ingnua da Abolio da escravatura brasileira, j que persistiu a desigualdade social e econmica entre negros e
brancos.
c) irnica da colonizao europia do continente africano, justificada ideologicamente pela ideia de misso
civilizadora.
d) negativa da democracia sul-africana, uma vez que o fim do Apartheid no garantiu igualdade econmica aos
negros.

03 - (UFJF MG) Observe o mapa abaixo:

Disponvel em: <www.culturabrasil.pro.br/neocolonialismo.htm>. Acesso em:
13 set. 2010.

O mapa retrata a frica partilhada por pases europeus em um processo
conhecido como imperialismo.

a) Analise as repercusses desse processo de desenvolvimento do
capitalismo desde o final do sculo XIX.
b) Relacione os impactos desse processo sobre as origens da Primeira
Grande Guerra Mundial.





Caderno de Atividades

Disciplina:
Histria

Professor(a):
Fernando
Aluno:
3 ano
Ensino Mdio
Data de Recebimento:
_____/_____/_____
Lista 05

Data Entrega:
_____/_____/_____

276
04 - (UNICAMP SP) No sculo XIX, surgiu um novo modo de explicar as diferenas entre os povos: o racismo. No
entanto, os argumentos raciais encontravam muitas dificuldades: se os arianos originaram tanto os povos da ndia
quanto os da Europa, o que poderia justificar o domnio dos ingleses sobre a ndia, ou a sua superioridade em relao
aos indianos? A nica resposta possvel parecia ser a miscigenao. Em algum momento de sua histria, os arianos
da ndia teriam se enfraquecido ao se misturarem s raas aborgenes consideradas inferiores. Mas ningum podia
explicar realmente por que essa ideia no foi aplicada nos dois sentidos, ou seja, por que os arianos da ndia no
aperfeioaram aquelas raas em vez de se enfraquecerem.
(Adaptado de Anthony Pagden, Povos e Imprios. Rio de Janeiro: Objetiva, 2002, p. 188-194.)

a) Segundo o texto, quais as incoerncias presentes no pensamento racista do sculo XIX?
b) O que foi o imperialismo?

05 - (UFRJ) A Guerra dos Beres (1899-1902), na frica do Sul, levou a Inglaterra a mobilizar aproximadamente 450
mil soldados, trazidos de todo o seu imprio. A vitria britnica fez com que fosse limitada a autonomia dos estados
beres. No entanto, o sistema eleitoral permitiu que, terminada a guerra, os africnderes (beres) dominassem o poder
poltico em diversas provncias. No mapa abaixo, pode-se observar o cenrio dessa guerra e a indicao geogrfica de
fatores a ela relacionados.


(MAPA adaptado do Atlas Historique. Paris, Hachette, 1987, p.239)

a) Apresente uma razo para o incio dessa guerra.
b) Explique o que permitiu aos beres obter o controle poltico de diversas provncias, mesmo tendo perdido a guerra
para os ingleses.

06 - (UFF RJ) Interesses econmicos, fatores demogrficos, razes humanitrias, iniciativas individuais, mas,
principalmente, motivos polticos encontram-se na origem do grande movimento de colonizao europia no mundo
durante a segunda metade do sculo XIX. Esta nova expanso levou constituio de vastos imprios coloniais que
permitiram s principais potncias europias dominar a maior parte da frica, da sia e do Pacfico. Esta colonizao,
que ps em contato a civilizao industrial do sculo XIX com as velhas sociedades tradicionais, tornou possvel a
valorizao das riquezas inexploradas, mas transformou as sociedades nativas, sujeitas ao domnio das metrpoles
europias.
(Serge Bernstein e Pierre Milza. Histria do Sculo XIX.)
A partir do texto:

a) indique e explique duas diferenas entre as expanses europias dos sculos XV e XVI e do sculo XIX;
b) explique o porqu da partilha da frica ter-se tornado um dos principais elementos deflagradores da Primeira Guerra
Mundial.

07 - (UFCG PB)
"As exposies universais queriam ser um retrato em miniatura desse mundo moderno avanado, composto de
espetculos nos campos da cincia, das artes, da arquitetura, dos costumes e da tecnologia. A idia era mostrar e
ensinar as virtudes do tempo presente e confirmar a previso de um futuro excepcional. A torre Eiffel, o palcio de
cristal e a roda gigante eram os smbolos visveis do avano tecnolgico exibido nas feiras mundiais."
(http://www.cpdoc.fgv.br/nav_historia/htm/anos20/ ev_centindep_expuniversais.ht. Acesso em 01.jun.2008)
Eixo Temtico: Alm da f, o po: permanncias, continuidades e o projeto de felicidade na modernidade

A partir do fragmento textual acima e dos seus conhecimentos sobre a temtica, INCORRETO afirmar que as
Exposies Universais no sculo XIX:

a) Ressignificaram os conceitos de espao e tempo, celebrando as virtudes do tempo presente e criando uma nova
dinmica espacial urbana.
b) Deram visibilidade ao neo-colonialismo europeu, utilizando-se da cincia e da tcnica para promoverem o
imperialismo geogrfico-cultural.
c) Respeitaram as identidades dos povos afro-asiticos, exibindo seus principais inventos nos campos da arquitetura,
da tcnica e das artes coloniais.

277
d) Representavam contedo racial, pois alguns pases levavam pequenos grupos enjaulados para mostrar ao mundo
as "raas inferiores" de suas respectivas colnias.
e) Mistificavam a modernidade e o avano tecnolgico, fazendo apologia superioridade, lucidez e eficincia do
europeu.

08 - (UNIFESP SP) A partir da Conferncia [de Berlim, em 1885],
a corrida ao continente africano foi acelerada, num gesto
inequvoco de violncia geogrfica por meio da qual quase todo o
espao recortado ganhou um mapa para ser explorado e
submetido a controle. A demarcao das fronteiras prosseguiu,
estendendo-se at depois da Primeira Grande Guerra.
O mapa e o texto tratam do expansionismo europeu na frica e da
partilha do continente. Relacione

a) o expansionismo europeu no sculo XIX e seus objetivos na
colonizao da frica.
b) a forma como se deu a diviso da frica e sua ligao com
problemas tnicos e nacionais que o continente enfrenta
atualmente.
(Diviso da frica, 1885 in H. L. Wesseling. Dividir para dominar:
a partilha da frica (1880-1914). Rio de Janeiro: UFRJ/Revan, 1998.)
(Leila Leite Hernandez. A frica na sala de aula.
So Paulo: Selo Negro Edies, 2005.)

09 - (UFSCAR SP) Leia os textos 1 e 2 e responda.

Texto 1

Difcil encontrar uma sociedade que seja monocultural. As culturas so feitas tanto de continuidade quanto de
mudanas. E as transformaes no significam o fim de sua sobrevivncia. As culturas esto o tempo todo
emprestando elementos umas das outras e isso bom. Nada parece mais monocultural do que um monastrio tibetano
e, no entanto, o budismo veio de fora, da ndia. Quem visita as aldeias ao redor da cidade em que vivi em Gana pensa
que est diante de comunidades monoculturais. Como, se quase todos os moradores so cristos? O cristianismo
chegou na regio no sculo XIX, trazido pelos europeus. , portanto, um aspecto recente da cultura local.
(Kwame Anthony Appiah. Veja, 08 de maro de 2006.)
Texto 2

Curau

Ingredientes
12 espigas de milho verde
1
2 litro de leite
2 1
2 xcaras (ch) de acar
1 pitada de sal
canela em p a gosto

1. Retirar a palha e os cabelos das espigas. Ralar o milho e raspar os sabugos com a ajuda de uma colher. Acrescentar
o leite, o sal, misturar e deixar a massa descansar por 10 minutos. Coloc-la em um pano de prato, juntar as pontas e
torcer o pano para coar o lquido.
2. Despejar o caldo em uma panela, acrescentar o acar, mexer e levar ao fogo brando.
3. Cozinhar lentamente, mexendo sempre, at que tome consistncia. Despejar em um refratrio ou em tigelinhas
molhadas.
4. Deixar esfriar e levar geladeira. Servir polvilhado com canela em p.
(Rosa Belluzzo. Fazeres e Sabores da Cozinha
Paulista. Prefeitura do Municpio de
So Paulo, 2007.)

a) Faa uma anlise comparativa dos dois textos, buscando seus aspectos semelhantes.
b) Como o argumento do texto 1 As culturas esto o tempo todo emprestando elementos umas das outras pode
ser comprovado pela receita de como fazer curau (texto 2)?




278
10 - (UNESP SP) Onde quer que tenha conquistado o Poder, a burguesia (...) afogou os fervores sagrados do xtase
religioso (...) nas guas geladas do clculo egosta. (...) Impelida pela necessidade de mercados sempre novos, a
burguesia invade todo o globo (...) Em lugar do antigo isolamento de regies e naes que se bastavam a si prprias,
desenvolvem-se um intercmbio universal, uma universal interdependncia das naes.
(Marx e Engels. Manifesto de 1848.)

Lakshmi Mittal, presidente de origem indiana da Mittal Steel, a maior siderrgica do mundo, provocou um terremoto na
Arglia. A empresa argelina (...) rompeu no incio do ms um dos tabus mais enraizados na Arglia, o chamado
popularmente fim-de-semana islmico, que inclui a quinta e a sexta-feira. (...) Para as empresas e os rgos argelinos
que mantm relaes com o estrangeiro, a defasagem entre um fim-de-semana [o islmico] e outro [o universal, no
sbado e domingo] uma tremenda complicao. Eles s tm trs dias teis por semana (segundas, teras e
quartas) para trabalhar com o resto do mundo...
(El Pas, 19.06.2007.)

Escritos em pocas distintas e tendo naturezas distintas, os textos no deixam de manifestar algumas semelhanas de
contedo.
Compare-os e indique essas semelhanas.

11 - (UERJ)


Adaptado de Jornal do Sculo, encarte do Jornal do Brasil, 12/11/2000

A Primeira Guerra Mundial provocou uma reorganizao poltico territorial da Europa, como se observa nos mapas.
Duas ideias orientaram essa reorganizao: a do Estado-nao e, no caso da fronteira russa, a do cordo sanitrio.

A partir da anlise dos mapas, identifique a mudana ocorrida na organizao poltica europeia aps a Primeira
Guerra.

Em seguida, indique o motivo que levou ao estabelecimento da poltica do cordo sanitrio naquele momento.








279
12 - (UFG GO) Observe os cartazes a seguir.

Cartaz militar ingls da I Guerra. Legenda: Mulheres da Inglaterra, digam: v! Disponvel em:
<http://historywiki.wikispaces.com/file/view/women_of_britain_say_go.jpg/
31172833/women_of_britain_say_go.jpg>. Acesso em: 11 out. 2010.

Cartaz militar estadunidense da II Guerra. Legenda: Ns podemos fazer! Disponvel em: <http://postersdeguerra.blogspot.com>.
Acesso em: 11 out. 2010.

Na primeira metade do sculo XX, durante as duas guerras mundiais, os cartazes foram importantes ferramentas para
a mobilizao de pessoas, orientando suas aes e seus sentimentos em relao aos conflitos. Com base na
comparao entre os cartazes acima,

a) descreva como a atitude da figura feminina representada em cada um deles;
b) explique a utilizao da fora de trabalho das mulheres no perodo de cada uma das guerras.

13 - (FUVEST SP) Este livro no pretende ser um libelo nem uma confisso, e menos ainda uma aventura, pois a
morte no uma aventura para aqueles que se deparam face a face com ela. Apenas procura mostrar o que foi uma
gerao de homens que, mesmo tendo escapado s granadas, foram destrudos pela guerra.
Erich Maria Remarque, Nada de novo no front. So Paulo: Abril, 1974 [1929], p.9.
Publicado originalmente em 1929, logo transformado em best seller mundial, o livro de Remarque , em boa parte,
autobiogrfico, j que seu autor foi combatente do exrcito alemo na Primeira Guerra Mundial, ocorrida entre 1914 e
1918. Discuta a ideia transmitida por uma gerao de homens que, mesmo tendo escapado s granadas, foram
destrudos pela guerra, considerando:

a) As formas tradicionais de realizao de guerras internacionais, vigentes at 1914 e, a partir da, modificadas.
b) A relao da guerra com a economia mundial, entre as ltimas dcadas do sculo XIX e as primeiras do sculo XX.

14 - (UFG GO) Leia o testemunho e analise a imagem.

A alguns passos de ns, no fundo da trincheira, jaz um corpo. de um suboficial:
est enterrado pela metade; v-se apenas a cabea, um ombro anterior, o brao se
enrijeceu e ficou estacado, e todos que andam pela vala esbarram e tropeam na
mo e no brao. Seria preciso cortar o brao ou retirar o corpo. Ningum tem
coragem.
TESTEMUNHO DE UM SOLDADO FRANCS. Apud: VINCENT, Grard.
Uma histria do segredo? In: PROST, Antoine; VINCENT, Grard.
Histria da vida privada: da Primeira Guerra aos nossos dias. So Paulo:
Companhia das Letras, 1995, v. 5. p. 206. [Adaptado].

Dizeres do monumento francs: Contra a guerra; s suas vitimas, fraternidade
dos povos.
Que o futuro console a dor. Disponvel em:
<http://guerres-mondiales.xooit.fr/t165-Dardilly.htm>. Acesso em: 26 maio 2011.

O testemunho do soldado demarca a experincia na guerra, assim como o monumento francs expe a lembrana
sobre a Primeira Guerra Mundial (1914-1918), que, para os pases europeus, ficou conhecida como a Grande Guerra.
Diante do exposto e de acordo com o texto e a imagem, explique de que forma
a) a guerra nas trincheiras levou s manifestaes pelo pacifismo, especialmente na Frana, findo o conflito, em 1918.
b) a memria sobre a Grande Guerra, expressa no monumento, se relaciona posio poltica francesa diante das
ofensivas alems durante os anos de 1930.

280
15 - (UFG GO) Leia os documentos a seguir.

Os camponeses partem para o front com incrvel entusiasmo; e as classes superiores da sociedade, quer sejam
liberais ou conservadoras, os aclamam, desejando-lhes boa sorte [] Habitualmente, os camponeses sentiam que no
tinham nada a fazer a no ser beber; mas agora no mais assim. como se a guerra lhes desse uma razo para
viver [] No ardor dos soldados russos se percebe o entusiasmo que agita o corao dos antigos mrtires se lanando
para a morte gloriosa.
LE BON, Gustave. 1916 apud JANOTTI, Maria de Lourdes. A Primeira Guerra Mundial. O confronto de imperialismos.
So Paulo: Atual, 1992. p.17.

Aps um ano de massacre, o carter imperialista da guerra cada vez mais se afirmou; essa a prova de que suas
causas encontram-se na poltica imperialista e colonial de todos os governos responsveis pelo desencadeamento
desta carnificina. [] Hoje, mais do que nunca, devemos nos opor a essas pretenses anexionistas e lutar pelo fim
desta guerra [] que provocou misrias to intensas entre os trabalhadores de todos os pases.
CONFERNCIA DE ZIMMERWALD - 5 a 8 de setembro de 1915. Apud JANOTTI, Maria de Lourdes. A Primeira
Guerra Mundial. O confronto de imperialismos. So Paulo: Atual, 1992. [Adaptado].

No incio da Primeira Guerra Mundial (1914-1918), estabeleceu-se, sobretudo na Europa, uma disputa de ideias em
torno do envolvimento nesse conflito. Com base na leitura de cada um dos documentos, explique as posies
assumidas sobre a participao na guerra.

16 - (UERJ) A Primeira Guerra Mundial no resolveu nada. As esperanas que gerou de um mundo pacfico e
democrtico de Estados-nao sob a Liga das Naes; de um retorno economia mundial de 1913; mesmo (entre os
que saudaram a Revoluo Russa) de capitalismo mundial derrubado dentro de anos ou meses por um levante dos
oprimidos logo foram frustradas. O passado estava fora de alcance, o futuro fora adiado, o presente era amargo, a
no ser por uns poucos anos passageiros em meados da dcada de 1920.
ERIC J. HOBSBAWM
A era dos extremos: o breve sculo XX (1914-1991).
So Paulo: Companhia das Letras, 1995.

O perodo entre-guerras (1919-1939) comeou com uma combinao de esperana e ressentimento.
Diversos acordos foram impostos pelos Estados vencedores aos derrotados. O mais conhecido deles o Tratado de
Versalhes de 1919. Outros tratados complementares tambm foram assinados e igualmente tiveram grande
importncia para a geopoltica mundial.
Indique duas transformaes na geopoltica mundial decorrentes desses tratados complementares.
Em seguida, cite dois pases que foram submetidos a eles.

17 - (UFTM MG) Para os que cresceram antes de 1914, o contraste foi to impressionante que muitos (...) se
recusaram a ver qualquer continuidade com o passado. Paz significava antes de 1914: depois disso veio algo que
no mais merecia esse nome. Era compreensvel.
Em 1914 no havia grande guerra fazia um sculo, quer dizer, uma guerra que envolvesse todas as grandes potncias,
ou mesmo a maioria delas (...).
(Eric Hobsbawm, Era dos Extremos: o breve sculo XX)

a) Na economia, o que caracterizou o perodo de paz, citado pelo autor?
b) Explique um motivo da guerra a que Hobsbawm se refere.

18 - (UNICAMP SP) A Primeira Guerra Mundial abalou profundamente todos os povos envolvidos, e as revolues de
1917-1918 foram, acima de tudo, revoltas contra aquele holocausto sem precedentes, principalmente nos pases do
lado que estava perdendo. Mas em certas reas da Europa, e em nenhuma outra mais que na Rssia, foram mais que
isso: foram revolues sociais, rejeies populares do Estado, das classes dominantes e do status quo.
(Adaptado de Eric Hobsbawm, Sobre Histria. So Paulo: Companhia das Letras, 1998, p. 262-263.)

a) Relacione a Primeira Guerra Mundial e a situao da Rssia na poca.
b) Cite e explique um princpio da Revoluo Russa de 1917.

19 - (UNESP SP) A Revoluo Russa o acontecimento mais importante da Guerra Mundial.
(Rosa Luxemburgo. A revoluo russa. Lisboa: Ulmeiro, 1975.)

A frase de Rosa Luxemburgo, polonesa ento radicada na Alemanha, associa diretamente a ocorrncia da Revoluo
Russa com a Primeira Guerra Mundial.

Indique e analise possveis vnculos entre os dois processos, destacando os efeitos da Guerra na vida interna da
Rssia.

281
20 - (UNESP SP) A Rssia, madura para a revoluo social, cansada de guerra e beira da derrota, foi o primeiro
dos regimes da Europa Central e Oriental a ruir sob as presses e tenses da Primeira Guerra Mundial (...) to pronta
estava a Rssia para a revoluo social que as massas de Petrogrado imediatamente trataram a queda do czarismo
como uma proclamao de liberdade, igualdade e democracia direta universais. O feito extraordinrio de Lenin foi
transformar essa incontrolvel onda anrquica popular em poder bolchevique.
(E. J. Hobsbawm, Era dos extremos.)

A partir do texto, explique as condies estruturais que permitem ao autor considerar a Rssia madura para a
revoluo social.

21 - (UNESP SP) O retorno a uma semi-economia de mercado provocou o reaparecimento da moeda e, durante o
ano de 1921, renasceu o mercado propriamente dito. A desnacionalizao de empresas comeou respectivamente
pelo pequeno e grande comrcio, atingindo, mais tarde, a indstria leve. As cooperativas foram devolvidas aos seus
antigos acionistas e, no final do ano, permaneciam nas mos do Estado apenas os setores economicamente
estratgicos, o crdito e a indstria pesada.
(Martin Malia. Entender a Revoluo Russa.)
O trecho apresentado refere-se a um momento da Revoluo Russa, no qual:
a) o Estado sovitico implementa a Nova Poltica Econmica, procurando superar as dificuldades econmicas e
sociais advindas do Comunismo de Guerra.
b) o partido bolchevista promove um processo de abertura poltica, instaurando um regime poltico democrtico e
pluripartidrio.
c) o governo leninista, enfraquecido pela guerra civil, obrigado a fazer concesses tradicional nobreza czarista.
d) o Estado sovitico aplica uma poltica de planificao econmica e de coletivizao de terras denominada de
Planos Qinqenais.
e) o conflito entre faces dentro do Estado resulta na oposio do partido bolchevista ao iderio socialista.

22 - (UFAL) A Revoluo Russa de 1917 considerada um dos episdios mais importantes da histria do sculo XX.
Tambm reconhecida como Revoluo Socialista Russa, ela exerceu considervel influncia na vida de centenas de
milhes de seres humanos. Sobre esse tema, analise as afirmaes a seguir.

1) O cenrio desolador composto na Rssia aps sua desastrosa participao na Primeira Guerra Mundial emoldurou o
quadro da Revoluo de 1917.
2) A disputa entre russos e japoneses pela posse dos territrios da Coria e da Manchria constituiu uma das
motivaes imediatas para se deflagrar a Revoluo.
3) Os partidos de esquerda que se encontravam na clandestinidade ressurgiram na conjuntura revolucionria, fazendo
eco s exigncias de derrubada da monarquia russa.
4) Os Bolcheviques liderados por Lnin preferiam adotar estratgias de negociao, mas foram vencidos pelo bloco
liderado por Trotski.
5) Os soldados russos encarregados de conter os movimentos grevistas aderiram a eles, desobedecendo s ordens
dos generais czaristas.

Esto corretas apenas:
a) 1, 2 e 3
b) 1, 3 e 4
c) 1, 4 e 5
d) 1, 3 e 5
e) 2, 4 e 5

23 - (UNESP SP) Os operrios das fbricas e das usinas, assim como as tropas rebeldes, devem escolher sem
demora seus representantes ao governo revolucionrio provisrio, que deve ser constitudo sob a guarda do povo
revolucionrio amotinado e do exrcito.
(Manifesto de 27 de fevereiro de 1917, in Marc Ferro.
A Revoluo Russa de 1917, 1974.)
O manifesto, lanado em meio s tenses de 1917 na Rssia, revela a posio dos

a) czaristas, que buscavam organizar a luta pela retomada do poder.
b) bolcheviques, que chamavam os operrios a se mobilizarem nos sovietes.
c) social-democratas, que pretendiam controlar o governo provisrio.
d) mencheviques, que defendiam o carter democrtico do novo governo.
e) militares, que tentavam controlar a revolta popular.





282
24 - (FGV) Com a NEP (Nova Poltica Econmica) o comrcio interno foi liberado, permitiu-se o funcionamento de
pequenas e mdias empresas privadas, estimularam-se os investimentos estrangeiros, instituiu-se o pagamento de
horas extras e de prmios aos trabalhadores e criou-se o imposto sobre propriedades urbanas.
(Paulo Sandroni. Dicionrio de economia, 1985.)

Durante a Revoluo Russa, a NEP foi aplicada no contexto

a) do fim da guerra civil (1918-1921), devido destruio da economia nacional e s tenses pela aplicao do
chamado comunismo de guerra.
b) da tomada do poder pelos bolcheviques, em outubro de 1917, pois a economia russa crescia em funo da
Primeira Guerra.
c) do fracasso dos planos quinquenais, que geraram a estagnao da economia sovitica a partir de 1930.
d) da revoluo de fevereiro de 1917, pois os mencheviques apostaram na reestruturao da economia russa por
meio das grandes obras de infraestrutura.
e) da morte de Lnin e da ascenso de Stalin, que estabeleceu um rgido e eficaz controle sobre as atividades
produtivas.

25 - (UFT TO) Em 1921, a Rssia conhece a mais terrvel fome da sua Histria: (...) h vrios meses, 9/10 da
populao no come mais po. Primeiro, misturaram o que restava de farinha com todo tipo de ervas, depois comeram
as ervas puras e depois, quando no restava mais nada, cozinharam e engoliram argila. (...) A cada dia, at a colheita,
aumentar o mbito das provncias e distritos onde no subsiste absolutamente mais nada que um ser humano possa
comer
(PASCAL, Pierre, Meu estado da alma. Meu dirio
da Rssia, tomo III, 1922-1926. LAge dHomme, Lausanne,
1982. apud. SALOMONI, Antonella. Lnin e a
Revoluo Russa. So Paulo: tica, 1997, p. 133).

Sobre a crise do regime sovitico INCORRETO afirmar:

a) Diante da degradao da economia agrcola, devastada pelo conflito blico, Lnin impe o abandono do comunismo
de guerra e a passagem para a fase de desenvolvimento da chamada Nova Poltica Econmica (NEP).
b) Na primavera de 1921, enquanto a degradao da produo agrcola, devastada pela guerra, e as magras colheitas
repercutiam de forma drstica na economia da Rssia, em contrapartida, assiste-se a recuperao de seu parque
industrial e sistema de transporte e prosperidade na zona rural.
c) Na cpula do partido comunista havia a conscincia de uma contra-revoluo camponesa; os dirigentes procuram
tirar lies do episdio de Kronstadt e inquietam-se com o esgotamento dos operrios. Ocorrendo um recuo
econmico, que significou, acima de tudo, abandonar o comunismo de guerra.
d) No mesmo momento em que o general Tukatchevski marcha sobre Kronstadt, abre-se em Moscou o Dcimo
Congresso do Partido Comunista. Apelando para o fim do estado de urgncia, Lnin impe a passagem para a Nova
Poltica Econmica (NEP).




283






PROF.: Fernando Viana














M
M
i
i
n
n
i
i
s
s
t
t
r
r
a
a
d
d
o
o
n
n
o
o
m
m

s
s
d
d
e
e
O
O
u
u
t
t
u
u
b
b
r
r
o
o















D Da at ta a d da a E En nt tr re eg ga a : : _ __ __ __ __ __ __ __ _/ /_ __ __ __ __ __ __ __ __ __ __ _/ /2 20 01 13 3
LISTA 6 HISTRIA

284

01 - (UERJ) Durante os ltimos trs meses, visitei uns vinte estados deste belo pas extraordinariamente rico. As
estradas do oeste e do sudoeste pululam de pessoas famintas pedindo carona. As fogueiras dos acampamentos dos
desabrigados so visveis ao longo de todas as estradas de ferro. Os fazendeiros esto sendo pauperizados pela
pobreza das populaes industriais, e as populaes industriais, pauperizadas pela pobreza dos fazendeiros. Nenhum
deles tem dinheiro para comprar o produto do outro; consequentemente h excesso de produo e carncia de
consumo, ao mesmo tempo e no mesmo pas.
Relato feito em 1932 por Oscar Ameringer Cmara dos Representantes dos Estados Unidos.
Adaptado de MARQUES, A. M. et al. Histria contempornea atravs de textos.
So Paulo: Contexto, 1990.

O depoimento acima faz referncia a efeitos da Crise de 1929 para a sociedade norte-americana.
Apresente dois fatores que contriburam para deflagrar essa crise e cite seu principal desdobramento para a economia
europeia naquele momento.

02 - (UNIFESP SP) Numa quinta-feira, 24 de outubro de 1929, 12.894.650 aes mudaram de mos, foram vendidas
na Bolsa de Nova Iorque. Na tera-feira, 29 de outubro do mesmo ano, o dia mais devastador da histria das bolsas de
valores, 16.410.030 aes foram negociadas a preos que destruam os sonhos de rpido enriquecimento de milhares
dos seus proprietrios. A crise da economia capitalista norte-americana estendeu-se no tempo e no espao. As
economias da Europa e da Amrica Latina foram duramente atingidas. Franklin Delano Roosevelt, eleito presidente
dos Estados Unidos em 1932, procurou combater a crise e os seus efeitos sociais por meio de um programa poltico
conhecido como New Deal.

a) Identifique dois motivos da rpida expanso da crise para fora da economia norte-americana.
b) Caracterize de maneira geral o New Deal e apresente uma de suas medidas de combate crise.

03 - (UFJF MG) No final de 2008, o mundo foi envolvido numa crise financeira que atingiu, e ainda atinge, a vida de
todos. Neste perodo, muitas referncias foram feitas crise econmica de 1929. A charge abaixo exemplar:


Disponvel em <http://agenciasubverta.blogspot.com> e capturado em novembro de 2009.
Com base na charge e em seus conhecimentos, responda ao que se pede.

a) Qual a principal diferena de comportamento entre os empresrios de 1929 e os atuais, expressa pela charge
acima?
b) Com base em seus conhecimentos, identifique uma semelhana e uma diferena entre as duas conjunturas
histricas.
SEMELHANA:
DIFERENA:

Caderno de Atividades

Disciplina:
Histria

Professor(a):
Fernando
Aluno:
3 ano
Ensino Mdio
Data de Recebimento:
_____/_____/_____
Lista 06

Data Entrega:
_____/_____/_____

285
04 - (UFU MG) As boas polticas do New Deal, como a construo de uma rede de segurana social bsica, fizeram
sentido por si s e teriam sido desejveis tambm durante a expanso dos anos 20. As polticas ruins deixaram as
coisas piores. Hoje, isso significa que deveramos restringir medidas extraordinrias ao setor financeiro o mximo
possvel e evitar a tentao de fazer algo para o seu prprio bem.
[...] Nossa crise atual tambm ir acabar um dia, e, assim como nos anos 30, a recuperao provavelmente vir de
razes que tm pouco a ver com a maioria das iniciativas polticas.
COWEN, Tyler. O New Deal nem sempre funcionou. The New York Times. 24/11/2008.

Sobre as realizaes do New Deal em longo prazo: as instituies que FDR [Franklin Delano Roosevelt] construiu se
mostraram to duradouras quanto essenciais. FDR pensou que estivesse sendo prudente ao conter seus planos de
gastos; na verdade, estava assumindo grandes riscos com a economia e com seu legado. Meu conselho ao pessoal de
Obama descobrir de quanta ajuda eles acham que a economia precisa e ento acrescentar 50%. Em uma economia
deprimida, muito melhor errar por excesso de estmulo do que por escassez.
KRUGMAN, Paul. Franklin Delano Obama?. The New York Times. 11/11/2008.

Compare as posies assumidas pelos articulistas citados, considerando os embates travados em ambos os contextos,
1929 e 2008, em torno do papel do Estado em um momento de crise do capitalismo.

05 - (UNICAMP SP) Aps a 1
a
Guerra Mundial, os Estados Unidos se torna o pas mais rico do mundo, consolidando
o chamado sonho americano. Essa riqueza provinha em grande parte do crescimento e avano tcnico da indstria, o
que por um lado resultava maior oferta de produtos industrializados baratos, mas por outro ocasionava um crescente
desemprego de operrios substitudos por mquinas. O desemprego e as altas aplicaes de dinheiro no mercado de
aes ajudam a desencadear uma crise de repercusso mundial.
Identifique essa crise e analise suas principais conseqncias para os Estados Unidos.

06 - (UERJ)
Exibido pela primeira vez em outubro de 1940, o filme O Grande Ditador, de Charles Chaplin, faz uma crtica a um
projeto poltico vigente na poca.

Observe abaixo o fotograma de uma cena do filme e o discurso apresentado em seu final:

http://passosdesvairados.blogspot.com
A aviao e o rdio nos aproximou. A prpria natureza dessas coisas um apelo eloquente bondade do homem, um
apelo fraternidade universal, unio de todos ns. Neste mesmo instante a minha voz chega a milhares de pessoas
pelo mundo afora. Milhes de desesperados: homens, mulheres, criancinhas, vtimas de um sistema que tortura seres
humanos e encarcera inocentes. Aos que podem me ouvir eu digo: no desespereis! A desgraa que tem cado sobre
ns no mais do que o produto da cobia em agonia, da amargura de homens que temem o avano do progresso
humano. Os homens que odeiam desaparecero, os ditadores sucumbem e o poder que do povo arrebataram h de
retornar ao povo. E assim, enquanto morrem homens, a liberdade nunca perecer.
Discurso final do filme O Grande Ditador
Adaptado de www.pedagogiaemfoco.pro.br

Identifique o projeto poltico-ideolgico criticado e apresente duas caractersticas desse projeto que se opunham aos
ideais defendidos no discurso final.

07 - (UFF RJ) Em todo caso, a interveno alem (importante tambm para a formao de oficiais e dos soldados)
foi rapidamente julgada na Europa. Nos meios populares e em certos crculos intelectuais, como prova da
agressividade nazista, anncio do perigo areo. Mas a paixo conservadora e o medo das responsabilidades, na maior
parte dos governos, determinaram uma total passividade diante daquilo que as conventrizaes [destruio por
bombardeio] e os rasantes dos Stukas [aviao de combate] anunciavam para um futuro prximo.
VILAR, Pierre. A Guerra da Espanha. 1936-1939. Rio de Janeiro, Paz e Terra, 1989, p. 101.
A partir das afirmaes do historiador espanhol Pierre Vilar:
a) destaque duas razes para a ecloso da Guerra Civil Espanhola;
b) explique em que sentido a Guerra Civil Espanhola retratada pelos historiadores como o ensaio geral da Segunda
Grande Guerra.


286
08 - (UFRN) Frequentemente o conhecimento histrico sobre a Segunda Guerra Mundial vem acompanhado de
perplexidade, quando nos deparamos com os crimes cometidos por lideranas polticas da Alemanha, inspiradas no
nazismo, amplamente divulgado entre o povo alemo. medida que os campos de concentrao foram denunciados,
todo um espetculo de barbrie veio superfcie, revelando os horrores que esses campos ocultavam.

a) Explique duas caractersticas da doutrina poltico-ideolgica nazista.
b) Mencione e comente duas influncias dessa doutrina no Brasil.

09 - (UNESP SP) Em 1922, Ele marcha sobre Roma. Ele a Itlia em movimento. A Revoluo prossegue. Depois de
meio sculo de letargia, a nao cria seu prprio regime. Surge o Estado dos italianos. Seu poder manifesta-se. Suas
virtudes vm tona. Seu imprio est em formao. Esse grande renascimento (...) ter o nome Dele. Em todo o
mundo se inaugura um sculo italiano: o sculo de Mussolini.
(Augusto Turati (1928), citado por Donald Sassoon. Mussolini e a ascenso do fascismo, 2009.)

O perfil de Benito Mussolini foi escrito em 1928 e mostra algumas caractersticas do fascismo italiano. Identifique, a
partir do documento, como esse perfil de Mussolini, traado pelo autor do texto, caracteriza a ideologia fascista e se
ope aos princpios polticos democrticos.

10 - (UNICAMP SP) Quando Colin Powell chegar s Naes Unidas hoje para defender a guerra contra Saddam
Hussein, as Naes Unidas planejam cobrir a obra-prima de Picasso, Guernica, com uma capa azul. Reprteres e
cmeras iro cercar o secretrio de Estado (cargo equivalente ao de ministro das Relaes Exteriores) na entrada do
Conselho de Segurana das Naes Unidas, onde a reproduo de Guernica est pendurada. De fato, Powell no
pode convencer o mundo sobre a necessidade de bombardear o Iraque cercado por mulheres, homens, crianas,
touros e cavalos aos gritos e mutilados. (Adaptado de Maureen Dowd, Powell without Picasso.
http://www.nytimes.com/2003/02/05/opinion/powell-without-picasso.html. Acesso em 06/12/2010.)

(Fonte: http://www.iep.uminho.pt/aac/sm/a2003/4cpintura/picasso.htm. Acesso em 06/12/2010.)
a) Quais eram as mensagens incompatveis entre a fala de Colin Powell e a obra Guernica de Picasso?
b) Identifique os acontecimentos polticos associados obra Guernica.

11 - (MACK SP) O fascismo no apenas fundador de instituies. tambm educador. Pretende reconstruir o
homem, seu carter, sua f. Para atingir esse objetivo, o fascismo conta com a autoridade e disciplina capazes de
penetrar no esprito das pessoas e a reinar completamente.
Benito Mussolini


O governo fascista italiano empenhou-se em fazer da educao pblica um instrumento capaz de impor sua doutrina
para toda a sociedade. O ideal bsico da doutrina fascista era

a) submeter o indivduo total obedincia ao Estado, comeando com a educao infantil e com a militarizao da vida
escolar.
b) promover, para os jovens, competies esportivas e desfiles paramilitares, visando exaltar a capacidade intelectual
dos indivduos.
c) a transformao das instituies educacionais, voltadas para a excelncia do conhecimento acadmico e intelectual.
d) propagar a educao fsica e a preparao militar, capazes de dotar o indivduo de uma mente analtica.
e) exaltar a inteligncia crtica e o bom desempenho acadmico dos indivduos, futuros construtores da nao.

12 - (UFRJ) O fascismo rejeita na democracia o embuste convencional da igualdade poltica, o esprito de
irresponsabilidade coletiva e o mito da felicidade e do progresso indefinido [...] No se deve exagerar a importncia do
liberalismo no sculo passado, nem convert-lo numa religio da humanidade para o presente e o futuro, quando na
realidade ele foi apenas uma das muitas doutrinas daquele sculo [...] Agora o liberalismo est prestes a fechar as
portas de seu templo deserto [...] O presente sculo o sculo da autoridade, um sculo da direita, um sculo fascista
(Benito Mussolini)
Fonte: MAZOWER, Mark. Continente sombrio: a Europa no sculo XX.
So Paulo: Companhia das Letras, 2001, p. 29.

O discurso proferido por Mussolini explicita a concepo poltica fascista nos anos 20 e 30 do sculo passado.
Cite dois aspectos do regime fascista contrrios aos princpios liberais.

287
13 - (FGV) A revoluo de Hitler uma garantia suficiente de que o Comunismo russo no ir se propagar em
direo ao Ocidente. A slida burguesia alem uma fortaleza central da Europa.
H.A.L.Fisher, History of Europe, 1936
O autor:
a) mostra iseno diante dos acontecimentos polticos que estavam ocorrendo naquele momento.
b) toma posio em favor da futura aliana entre democratas e comunistas na luta contra fascismo e nazismo.
c) manifesta uma viso pessimista sobre o futuro poltico da Europa, antevendo o perodo da guerra-fria.
d) expressa a poltica de apaziguamento praticada pelas democracias europias com relao ao regime nazista.
e) revela afinidade ideolgica com o nazismo, ao silenciar sobre o seu carter militarista e expansionista.

14 - (ENEM) A poltica foi, inicialmente, a arte de impedir as pessoas de se ocuparem do que lhes diz respeito.
Posteriormente, passou a ser a arte de compelir as pessoas a decidirem sobre aquilo de que nada entendem.
VALRY, P. Cadernos. Apud BENEVIDES, M. V. M.
A cidadania ativa. So Paulo: tica, 1996.
Nessa definio, o autor entende que a histria da poltica est dividida em dois momentos principais: um primeiro,
marcado pelo autoritarismo excludente, e um segundo, caracterizado por uma democracia incompleta.
Considerando o texto, qual o elemento comum a esses dois momentos da histria poltica?

a) A distribuio equilibrada do poder.
b) O impedimento da participao popular.
c) O controle das decises por uma minoria.
d) A valorizao das opinies mais competentes.
e) A sistematizao dos processos decisrios.

15 - (ENEM) A primeira metade do sculo XX foi marcada por conflitos e processos que a inscreveram como um dos
mais violentos perodos da histria humana.Entre os principais fatores que estiveram na origem dos conflitos ocorridos
durante a primeira metade do sculo XX esto
a) a crise do colonialismo, a ascenso do nacionalismo e do totalitarismo.
b) o enfraquecimento do imprio britnico, a Grande Depresso e a corrida nuclear.
c) o declnio britnico, o fracasso da Liga das Naes e a Revoluo Cubana.
d) a corrida armamentista, o terceiro-mundismo e o expansionismo sovitico.
e) a Revoluo Bolchevique, o imperialismo e a unificao da Alemanha.

16 - (ENEM) Do ponto de vista geopoltico, a Guerra Fria dividiu a Europa em dois blocos. Essa diviso propiciou a
formao de alianas antagnicas de carter militar, como a OTAN, que aglutinava os pases do bloco ocidental, e o
Pacto de Varsvia, que concentrava os do bloco oriental. importante destacar que, na formao da OTAN, esto
presentes, alm dos pases do oeste europeu, os EUA e o Canad. Essa diviso histrica atingiu igualmente os
mbitos poltico e econmico que se refletia pela opo entre os modelos capitalista e socialista.
Essa diviso europeia ficou conhecida como
a) Cortina de Ferro.
b) Muro de Berlim.
c) Unio Europeia.
d) Conveno de Ramsar.
e) Conferncia de Estocolmo.

17 - (ENEM) Os regimes totalitrios da primeira metade do sculo XX apoiaram-se fortemente na mobilizao da
juventude em torno da defesa de ideias grandiosas para o futuro da nao. Nesses projetos, os jovens deveriam
entender que s havia uma pessoa digna de ser amada e obedecida, que era o lder. Tais movimentos sociais juvenis
contriburam para a implantao e a sustentao do nazismo, na Alemanha, e do fascismo, na Itlia, Espanha e
Portugal.
A atuao desses movimentos juvenis caracterizava-se

a) pelo sectarismo e pela forma violenta e radical com que enfrentavam os opositores ao regime.
b) pelas propostas de conscientizao da populao acerca dos seus direitos como cidados.
c) pela promoo de um modo de vida saudvel, que mostrava os jovens como exemplos a seguir.
d) pelo dilogo, ao organizar debates que opunham jovens idealistas e velhas lideranas conservadoras.
e) pelos mtodos polticos populistas e pela organizao de comcios multitudinrios.

18 - (UFG GO) Com o fim da Segunda Guerra Mundial, os historiadores, que se voltaram para o estudo dos conflitos
armados ocorridos na primeira metade do sculo XX, denominaram a Guerra Civil Espanhola, ocorrida entre 1936 e
1939, de "Ensaio Geral".

Tendo em vista esta afirmao, explique

a) os princpios polticos defendidos pelas duas foras envolvidas na Guerra Civil Espanhola;
b) o motivo para a utilizao da denominao Ensaio Geral no contexto do ps-Segunda Guerra Mundial.


288
19 - (PUC RJ) No campo da poltica mundial, eu dedicarei esta nao poltica da boa vizinhana - uma vizinhana
que resulte do respeito mtuo e, devido a isso, respeite o direito dos outros - uma vizinhana que respeite suas
obrigaes e respeite a santidade dos seus acordos para com todos os seus vizinhos do mundo inteiro.
Discurso de posse de Franklin D. Roosevelt, em 1933.
Fonte: http://educaterra.terra.com.br/voltaire/mundo/eua_vizinhanca.htm

a) Apresente dois objetivos da poltica de Roosevelt, relacionando-os ao contexto mundial do Entre-guerras.
b) Identifique dois efeitos da poltica inaugurada por esse discurso para as relaes entre os pases americanos no
contexto da Segunda Guerra Mundial.

20 - (PUC RJ) No cartaz de propaganda norte-americano Aliados quebrando a cruz gamada (1945), os braos
simbolizam pases que se aliaram na Segunda Guerra Mundial contra as foras do Eixo.


ENDERS, A.; FERREIRA, M.; e FRANCO, R. (coords.).
Histria em curso: da Antiguidade Globalizao. SP:
Editora do Brasil; RJ: Fundao Getlio Vargas, 2008, p. 319

a) Identifique TRS pases representados no cartaz e explique UM motivo que propiciou a constituio dessa aliana.
b) Caracterize DUAS aes que permitiram s potncias aliadas assegurar a organizao da paz aps a Guerra.

21 - (UFG GO) A diviso da Alemanha entre os Aliados, no final da Segunda Guerra Mundial (1939-1945), expressa a
relao entre as disputas polticas e a definio de fronteiras territoriais. Considerando as tenses europeias nessa
circunstncia, conclui-se que a

a) zona de influncia francesa em territrio alemo pretendia pr fim ao revanchismo entre esses pases, presente
desde o Tratado de Versalhes.
b) interveno aliada desejava controlar os desejos expansionistas alemes, sustentados pela permanncia da
propaganda nazista.
c) atuao dos ingleses na partilha das zonas alems visava apoiar a presena sovitica na regio do Leste Europeu.
d) constituio das zonas de influncia na Alemanha expunha o temor europeu em relao aos interesses
nacionalistas que agitavam os Blcs.
e) criao da Repblica Democrtica Alem e da Repblica Federal Alem exps a fora poltica sovitica e norte-
americana no Continente Europeu.

22 - (UEG GO) O principal aspecto do panorama histrico do sculo XX o das duas guerras mundiais. Elas so
duas grandes cadeias de montanhas sob cujas sombras ns ainda agora vivemos. Elas mudaram o mundo mais do
que quaisquer das guerras mundiais e revolues dos sculos que as antecederam. Elas nos separaram do mundo
antes de 1914, que no somente para ns como para a gerao que se seguiu Primeira Guerra Mundial parece, e
pareceu, extremamente remoto. A revoluo bolchevista na Rssia, a ascenso dos Estados Unidos posio de
superpotncia do mundo, o fim dos imprios coloniais, a bomba atmica etc. etc., foram as consequncias dessas
guerras, no suas causas.
LUKACS, John. O duelo Churchill X Hitler, 80 dias cruciais para a Segunda
Guerra Mundial. Rio de Janeiro: Jorge Zahar Editor, 2002.

Sobre as duas guerras mundiais do sculo XX responda ao que se pede.

a) Identifique dois acontecimentos que antecederam a Primeira Guerra Mundial.
b) Discuta duas situaes histricas decorrentes da Segunda Guerra Mundial.




289
23 - (UFAC) O Dia D durante a II Guerra Mundial ocorreu quando:
a) A frente aliada desembarcou na Normandia, norte da Frana, no dia 6 de junho de 1944, anulando as foras
alems estacionadas no norte da Europa, avanando pelo continente e apertando o cerco sobre o Terceiro Reich;
b) A frente aliada desembarcou na Normandia, norte da Frana, no dia 6 de junho de 1945, anulando as foras
alems estacionadas no norte da Europa, avanando pelo continente e apertando o cerco sobre o Terceiro Reich;
c) A frente aliada desembarcou na Inglaterra no dia 06 de junho de 1945, derrotando o exrcito alemo;
d) Hitler suicidou-se;
e) Hitler entregou-se s foras aliadas no dia 06 de junho de 1945;

24 - (UFCG PB)
Pensem nas crianas
Mudas telepticas
Pensem nas meninas
Cegas inexatas
Pensem nas mulheres
Rotas alteradas
Pensem nas feridas
Como rosas clidas
Mas oh no se esqueam
Da rosa da rosa
Da rosa de Hiroshima
A rosa hereditria
A rosa radioativa
Estpida e invlida
A rosa com cirrose
A anti-rosa atmica
Sem cor sem perfume
Sem rosa sem nada.
(A Rosa de Hiroshima Vincius de Moraes)
CORRETO afirmar que a composio A Rosa de Hiroshima, de Vinicius de Moraes, faz aluso (aos)
a) campos minados da frica contempornea, que gestam corpos mutilados pelas constantes exploses.
b) bombardeios norte-americanos no Iraque a partir de 2003, destruidores das identidades locais.
c) radioatividade presente nos campos de concentrao nazistas, dizimando judeus, Testemunhas de Jeov, ciganos e
homossexuais.
d) estpida e invlida bomba de Hiroshima, fabricada pelos japoneses aps a Segunda Guerra Mundial.
e) bomba atmica lanada pelos norte-americanos em Hiroshima e Nagasaki, durante a Segunda Guerra Mundial.

25 - (PUC RJ) Nos anos de 1941 e 1942, houve mudanas na configurao das alianas polticas e militares que
ento caracterizavam a Segunda Grande Guerra (1939-1945). Frente a tais alteraes, o governo do Presidente
Getlio Vargas imprimiu novos rumos poltica externa brasileira. Sobre esses acontecimentos, podemos afirmar que:

I. o ataque japons a Pearl Harbor, em 1941, deflagrou a participao militar ostensiva dos EUA na guerra.
II. a invaso alem, na Unio Sovitica, em 1941, interferiu, entre outros aspectos, na aproximao diplomtica e
militar entre EUA, URSS e Inglaterra.
III. a crescente aproximao diplomtica com os EUA condicionou a declarao de guerra ao Eixo, por parte do
governo Vargas, em 1942.
IV. a participao militar brasileira na guerra, associada ao envio da FEB, conjugou-se ofensiva das tropas aliadas,
no front europeu, em meados de 1944.

Assinale a alternativa correta.
a) Apenas as afirmativas I e III esto corretas.
b) Apenas as afirmativas I e II esto corretas.
c) Apenas as afirmativas II e IV esto corretas.
d) Apenas as afirmativas I, III e IV esto corretas.
e) Todas as afirmativas esto corretas.



290






PROF.: JEFT RODRIGUES DE BARROS





C
C
o
o
n
n
t
t
e
e

d
d
o
o
s
s
:
:

P
P
e
e
r
r

o
o
d
d
o
o
P
P
r
r

-
-
C
C
o
o
l
l
o
o
n
n
i
i
a
a
l
l
,
,
A
A
d
d
m
m
i
i
n
n
i
i
s
s
t
t
r
r
a
a

o
o
C
C
o
o
l
l
o
o
n
n
i
i
a
a
l
l
,
,

E
E
c
c
o
o
n
n
o
o
m
m
i
i
a
a
A
A

u
u
c
c
a
a
r
r
e
e
i
i
r
r
a
a
e
e
M
M
i
i
n
n
e
e
r
r
a
a
d
d
o
o
r
r
a
a





M
M
i
i
n
n
i
i
s
s
t
t
r
r
a
a
d
d
o
o
s
s
d
d
u
u
r
r
a
a
n
n
t
t
e
e
o
o
m
m

s
s
d
d
e
e
f
f
e
e
v
v
e
e
r
r
e
e
i
i
r
r
o
o















D Da at ta a d da a E En nt tr re eg ga a : : _ __ __ __ __ __ __ __ _/ /_ __ __ __ __ __ __ __ __ __ __ _/ /2 20 01 13 3
LISTA 1 HISTRIA

291

01 - (UECE) Considere as seguintes afirmaes sobre o perodo da histria do Brasil, compreendido entre 1500 e
1530, no que concerne ao seu entendimento pela historiografia tradicional:
I. Perodo pr-colonial em virtude da ausncia de povoamento efetivo nas novas terras, em que Portugal enviava, de
vez em quando, expedies exploratrias que tambm tinham o fim de expulsar invasores.
II. Perodo de colonizao, visto que Portugal auferia lucros exorbitantes e realizava grandes investimentos nos
negcios com o corte e a venda do pau-brasil, exportando o produto para o oriente.
III. Perodo de pouco interesse de Portugal por essa possesso de terras, posto que estava envolvido com o comrcio
nas ndias e com a explorao do litoral africano.
Est correto o que se afirma em
a) II apenas.
b) II e III apenas.
c) I e III apenas.
d) I apenas.

02 - (PUC RJ) Eu, El-Rei, fao saber aos que este meu regimento virem, que sendo informado das muitas desordens
que h no serto do pau-brasil, e na conservao dele, de que se tem seguido haver hoje muita falta, cada vez ser o
dano maior se no se atalhar e der nisso a ordem conveniente (...): mando que nenhuma pessoa possa cortar, nem
mandar cortar o dito pau-brasil, por si ou seus escravos, sem expressa licena do provedor-mor da minha Fazenda
(...); e quem o contrrio fizer incorrer em pena de morte e confiscao de toda a sua fazenda.
Felipe III, Regimento do pau-brasil, 1605.
No contexto da colonizao das terras do Brasil, o regimento do rei Felipe III apresenta medidas associadas:

a) afirmao do poder da Coroa espanhola, em detrimento dos comerciantes e colonos portugueses.
b) ao carter monopolista da extrao do pau-brasil, pois era necessria autorizao expressa da Coroa para atividade
extrativista.
c) s preocupaes da Coroa na preservao da Mata Atlntica, que estava sendo devastada pelos colonos.
d) importncia do pau-brasil no comrcio colonial como principal produto de exportao da Amrica Portuguesa, em
incios do sculo XVII.
e) afirmao da poltica absolutista dos reinos europeus cerceadora de todas as iniciativas dos colonos nas Amricas.

03 - (UECE) Segundo a historiografia, o primeiro produto brasileiro a ser levado para a Europa pelos portugueses foi o
pau-brasil. Considerando a explorao desse produto, analise as seguintes afirmaes:

I. O pau-brasil era uma madeira apreciada para a tinturaria na Europa. E apesar de no ter ocupado imediatamente
as terras brasileiras, somente Portugal lucrou com o comrcio dessa madeira.
II. O pau-brasil era cortado pelos indgenas e depois por eles transportados para as feitorias, onde ficava armazenado
at a chegada de uma nau portuguesa que o levaria para a Europa.
III. Mesmo quando teve incio a produo aucareira, o corte do pau-brasil no foi interrompido e o seu transporte foi
facilitado pelo emprego de carroas puxadas por bois usadas nos engenhos.
Est correto o que se afirma em
a) III, apenas.
b) I, II e III.
c) I e II, apenas.
d) II e III, apenas.

04 - (UNICAMP SP) Em carta ao rei D. Manuel, Pero Vaz de Caminha narrou os primeiros contatos entre os indgenas
e os portugueses no Brasil: Quando eles vieram, o capito estava com um colar de ouro muito grande ao pescoo. Um
deles fitou o colar do Capito, e comeou a fazer acenos com a mo em direo terra, e depois para o colar, como se
quisesse dizer-nos que havia ouro na terra. Outro viu umas contas de rosrio, brancas, e acenava para a terra e
novamente para as contas e para o colar do Capito, como se dissesse que dariam ouro por aquilo. Isto ns
tomvamos nesse sentido, por assim o desejarmos! Mas se ele queria dizer que levaria as contas e o colar, isto ns
no queramos entender, porque no havamos de dar-lhe!
(Adaptado de Leonardo Arroyo, A carta de Pero Vaz de Caminha. So Paulo:
Melhoramentos; Rio de Janeiro: INL, 1971, p. 72-74.)
Caderno de Atividades

Disciplina:
Histria

Professor(a):
Jeft
Aluno:
3 ano
Ensino Mdio
Data de Recebimento:
_____/_____/_____
Lista 01

Data Entrega:
_____/_____/_____

292
Esse trecho da carta de Caminha nos permite concluir que o contato entre as culturas indgena e europeia foi

a) favorecido pelo interesse que ambas as partes demonstravam em realizar transaes comerciais: os indgenas se
integrariam ao sistema de colonizao, abastecendo as feitorias, voltadas ao comrcio do pau-brasil, e se
miscigenando com os colonizadores.
b) guiado pelo interesse dos descobridores em explorar a nova terra, principalmente por meio da extrao de
riquezas, interesse que se colocava acima da compreenso da cultura dos indgenas, que seria quase dizimada
junto com essa populao.
c) facilitado pela docilidade dos indgenas, que se associaram aos descobridores na explorao da nova terra,
viabilizando um sistema colonial cuja base era a escravizao dos povos nativos, o que levaria destruio da sua
cultura.
d) marcado pela necessidade dos colonizadores de obterem matria-prima para suas indstrias e ampliarem o
mercado consumidor para sua produo industrial, o que levou busca por colnias e integrao cultural das
populaes nativas.

05 - (UFTM MG) Em 1570, a Provncia de Santa Cruz contava com 60 engenhos. Destes, 41 situavam-se nas
capitanias de Pernambuco e da Bahia. Quinze anos depois, o nmero de engenhos nestas duas regies mais do que
triplicou, atingindo a marca dos 131. No final do sculo, em 1590, a colnia contava com 150 engenhos espalhados
pelas capitanias de Pernambuco, Bahia, Esprito Santo, Rio de Janeiro e So Vicente. As duas primeiras capitanias,
entretanto, continuavam a concentrar o maior nmero de unidades produtivas, que correspondia a 80% do total (...).
Em 1584, cerca de 40 navios eram utilizados para transportar o acar de Recife para Lisboa. No incio do sculo XVII,
em 1614, mais de 130 navios eram utilizados no transporte do acar de Pernambuco para a metrpole.
(Adriana Lopez e Carlos Guilherme Mota.
Histria do Brasil, uma interpretao, 2008.)
Infere-se do texto que

a) a produo aucareira distribuiu-se de forma equilibrada por toda a colnia.
b) os lucros propiciados pelo acar inibiram o desenvolvimento da pecuria em larga escala.
c) a prosperidade das regies dependia da capacidade administrativa dos donatrios.
d) a cana forneceu a base material para o estabelecimento dos portugueses nos trpicos.
e) o crescimento da produo foi lento e constante ao longo dos sculos XVI e XVII.

06 - (UFRN) O grfico abaixo mostra a destacada posio da agroindstria aucareira na economia do Brasil Colonial.


Na anlise histrica, esses dados se tornam significativos quando relacionados a outras dimenses da sociedade na
mesma poca. Neste sentido, correto afirmar que, alm de fornecer um importante produto para o comrcio mundial,
a agroindstria do acar

a) favoreceu a diversificao da economia colonial, uma vez que os lucros da produo aucareira foram investidos
na implantao de manufaturas na colnia.
b) ocasionou a ampliao do territrio da colnia portuguesa para alm dos limites estabelecidos pelo Tratado de
Tordesilhas, medida que se expandia o cultivo da cana-de-acar.
c) incorporou efetivamente ao domnio portugus as regies banhadas pelo rio So Francisco, onde se instalaram
engenhos, que utilizavam a gua do rio em obras de irrigao.
d) vinculou a explorao das terras americanas frica, fornecedora de mo de obra, e serviu de alternativa
economia portuguesa, a partir da crise no comrcio com o Oriente.

293
07 - (FATEC SP) "Os escravos so as mos e os ps do senhor de engenho, porque sem eles no Brasil no
possvel fazer, conservar e aumentar fazenda."
(ANTONIL, Andr Joo. Cultura e opulncia do Brasil. Belo Horizonte: Itatiaia, 1982, p. 89.)

No trecho citado, parte de uma obra publicada em 1711, o jesuta Antonil

a) torna evidente que o trabalho escravo constituiu a base da explorao econmica em setores essenciais da
economia colonial.
b) fornece argumentos para o combate movido pela Igreja contra a escravizao de indgenas e africanos nos
domnios coloniais portugueses.
c) explica por que a escravido foi importante no empreendimento aucareiro, mas teve papel secundrio e marginal
na explorao mineradora.
d) justifica a brandura da escravido no Brasil e sugere uma explicao para a democracia racial predominante na
sociedade colonial brasileira.
e) condena as tentativas de introduzir trabalhadores livres, trazidos da Europa, para substituir a mo-de-obra escrava
nas lavouras de caf.

08 - (FMABC SP) Nenhuma outra forma de explorao agrria no Brasil colonial resume to bem as caractersticas
bsicas da grande lavoura como o engenho de acar.
Alice Canabrava, in Srgio Buarque de Holanda (org.) Histria
geral da civilizao brasileira. Rio de Janeiro: Difel, 1963, tomo I,
vol. 2, p. 198-206.

Entre as caractersticas bsicas da grande lavoura, mencionadas no texto, podemos citar

a) pouco emprego de mo de obra e extrativismo.
b) esforo exportador e minifndio.
c) trabalho assalariado e alta tecnologia.
d) latifndio e produo voltada ao mercado interno.
e) monocultura e trabalho escravo.

09 - (UFPE) No Brasil, o acar foi a grande riqueza dos tempos coloniais, ajudando os portugueses na explorao e
no enfrentamento das suas dificuldades econmicas. Nos famosos engenhos de acar:

00. predominava o trabalho escravo, mas havia atividades desempenhadas por homens livres, embora rgidas
hierarquias estivessem presentes.
01. foram feitas muitas tentativas de romper com a centralizao administrativa, pela diviso dos latifndios e criao
de centros artesanais.
02. alimentavam-se preconceitos que dificultavam a adoo de uma vida livre e garantiam a autoridade dos senhores
de engenho.
03. aconteciam, com assiduidade, grandes revoltas de escravos, que prejudicavam os lucros e perturbavam a vida
social das elites econmicas.
04. mantinha-se um poder baseado na riqueza concentrada, no que esteve presente a atuao da Igreja Catlica e de
seus representantes.

10 - (UEG GO) O Brasil, no perodo que vai de 1500 a 1530, no foi colonizado de maneira efetiva pelos portugueses,
cujas atividades limitavam-se extrao do pau-brasil. Esse perodo denominado pr-colonial. Explique as razes do
desinteresse dos portugueses em se estabelecer no territrio descoberto.


11 - (UFC CE) Observe o mapa a seguir apresentado.


a) Exponha as razes do estabelecimento dessa linha divisria.
b) Explique os motivos que levaram o Brasil a ter uma outra
configurao, apesar das limitaes de fronteiras, ento
impostas pelo Tratado de Tordesilhas.








294
12 - (UERJ) Pelo que, comeando, digo que as riquezas do Brasil consistem em seis coisas, com as quais seus
povoadores se fazem ricos, que so estas: a primeira, a lavoura do acar; a segunda, a mercancia; a terceira, o pau a
que chamam do Brasil; a quarta, os algodes e madeiras; a quinta, a lavoura de mantimentos; a sexta e ltima, a
criao de gados. De todas estas coisas o principal nervo e substncia da riqueza da terra a lavoura dos acares.
AMBRSIO FERNANDES BRANDO, 1618
Adaptado de PRIORE, M. del; VENNCIO, R. P. O livro de ouro da histria do Brasil. Rio de Janeiro: Ediouro, 2001.

Considera-se hoje que o Brasil colonial teve um desenvolvimento bastante diferente da interpretao de Caio Prado
Jnior. que mudou a tica de observao: os historiadores passaram a analisar o funcionamento da colnia. No que
a inteno da poltica metropolitana fosse diferente do que prope o autor. Mas a realidade se revelava muito mais
complexa. No lugar da imagem de colonos engessados pela metrpole, vem tona um grande dinamismo do comrcio
colonial.
SHEILA DE CASTRO FARIA
Adaptado de www.revistadehistoria.com.br

O texto do sculo XVII enumera interesses da metrpole portuguesa em relao colonizao do Brasil; j o segundo
texto, uma anlise mais contempornea, descreve uma sociedade mais complexa que ia alm dos planos dos
exploradores europeus.
Indique dois objetivos da Coroa Portuguesa com a implantao da empresa aucareira no Brasil colonial. Em seguida,
identifique duas caractersticas da economia colonial que comprovam o seu dinamismo interno.

13 - (UNESP SP) O artista holands Albert Eckhout (c.1610-c.1666) esteve no Brasil entre 1637 e 1644, na comitiva
de Maurcio de Nassau. A tela foi pintada nesse perodo e pode ser considerada exemplar da forma como muitos
viajantes europeus representaram os ndios que aqui viviam.

(Albert Eckhout. ndia Tarairiu (tapuia), 1641.)

Identifique e analise dois elementos da imagem que expressem esse olhar europeu sobre o
Brasil.

14 - (UEG GO) Um dos principais conflitos do perodo colonial brasileiro foi a Guerra dos
Tamoios. Explique as razes que causaram esse conflito.

15 - (UFC CE) A conquista do territrio brasileiro pelos portugueses foi efetuada mediante o
confronto com as populaes indgenas que habitavam o Brasil naquele momento. Com base
nisso, responda o que se pede a seguir

a) Apresente trs caractersticas gerais das sociedades aqui encontradas pelos colonizadores
portugueses.
b) A partir dos contatos estabelecidos com os nativos, os colonizadores entenderam que
deveriam promover a salvao de suas almas. Cite a grande estratgia utilizada para esse fim pelos portugueses.
c) Vrios elementos da cultura indgena foram desvalorizados pelos portugueses no af de legitimar seu projeto
colonizador. Desse modo, indique duas prticas culturais nativas desprezadas pelos colonizadores.
d) Qual o legado cultural indgena sociedade brasileira? Enumere trs exemplos.

16 - (UFG GO) Analise os mapas a seguir.

Mapa 1 Terra Brasilis


295
Disponvel em: <http://estudoliterario.blogspot.com/2008_02_01.archive.htm>. Acesso em: 1 dez. 2009

Mapa 2 America Meridionalis

Disponvel em:<www.novomilenio.inf.br/santos/mapa68g.htm>. Acesso em: 01 dez. 2009.

Os dois mapas foram produzidos, respectivamente, em 1519 e 1638 e descrevem, de forma distinta, o continente
americano. Com base na comparao entre os mapas, explique a relao entre representao cartogrfica e o
conhecimento do territrio, em cada um deles.

17 - (UFRJ) A primeira coisa que os moradores desta costa do Brasil pretendem so ndios escravizados para
trabalharem nas suas fazendas, pois sem eles no se podem sustentar na terra.
(adaptado de GANDAVO, Pero Magalhes. Tratado descritivo da terra do Brasil.
So Paulo: Ed. Itatiaia e EDUSP, 1982, p. 42 [1576])


Nesse trecho percebe-se a adeso do cronista ao iderio dos colonos lusos no Brasil de fins do sculo XVI.

Com base no texto, e considerando que em Portugal prevalecia uma hierarquia social aristocrtica e catlica, explique
por que, ao desembarcarem na Amrica portuguesa da poca, os colonos imediatamente procuravam lanar mo do
trabalho escravo.

18 - (UFRJ) Por mais de um sculo o Brasil foi o principal exportador mundial de acar. De 1600 a 1650 o acar
respondia por 90% a 95% dos ganhos brasileiros com exportaes. Mesmo no perodo em torno de 1700, quando o
setor aucareiro declinou, ele continuava a representar 15% dos ganhos do Brasil com exportaes.
(SKIDMORE, Thomas E. Uma histria do Brasil. So Paulo: Paz e Terra, 1998, p. 36)

Explique um fator, externo Amrica portuguesa, responsvel pelo declnio relativo do setor aucareiro brasileiro na
segunda metade do sculo XVII.

19 - (UFBA) Com base nos conhecimentos sobre o Perodo Colonial, indique, para cada situao em destaque, um
fator que comprove a importncia da economia colonial brasileira para as relaes econmicas entre Portugal/Holanda
e Portugal/Inglaterra entre os sculos XVII e XVIII.
Portugal/Holanda
Portugal/Inglaterra

20- ( ENEM) Jean-Baptiste Debret. Entrudo, 1834.
Na obra Entrudo, de Jean-Baptiste Debret (1768-1848),
apresentada acima,
A- registram-se cenas da vida ntima dos senhores de engenho e
suas relaes com os escravos.
B -identifica-se a presena de traos marcantes do movimento
artstico denominado Cubismo.
C - identificam-se, nas fisionomias, sentimentos de angstia e
inquietaes que revelam as relaes conflituosas entre
senhores e escravos.
D - observa-se a composio harmoniosa e destacam-se as
imagens que representam figuras humanas.
E - constata-se que o artista utilizava a tcnica do leo sobre tela, com pinceladas breves e manchas, sem
delinear as figuras ou as fisionomias.

296
21 - (UFG GO) No sculo XVIII, um dos instrumentos utilizados para a extrao de ouro em Gois foi a bateia: um
prato na forma de cone, com o qual os mineradores executavam um movimento circular, separando o solo proveniente
do leito dos rios e o ouro. A utilizao desse instrumento na atividade mineradora
a) demonstrava o interesse pelo desenvolvimento tcnico da minerao, com insero de mecanismos de
retardamento do processo de decantao.
b) demandava mo de obra especializada, capaz de estabelecer critrios de contraste entre translucidez aurfera e
opacidade da bateia.
c) isentava a obrigatoriedade rgia da fundio do ouro,ao facilitar a extrao do minrio, quando exposto ao sol, por
meio da refrao.
d) dispensava a utilizao de outros instrumentos de trabalho, tendo em vista a eficincia do processo de decantao
aplicado ao sistema de extrao.
e) tornava o trabalho nas minas desgastante, pois havia a exigncia constante em produzir um processo de
centrifugao na bateia.

22 - (UERJ) A Estrada Real, nos dias de hoje, a reunio dos vrios caminhos
construdos no Brasil-Colnia, principalmente nos sculos XVII e XVIII, para o
transporte das riquezas do interior para o litoral do Rio de Janeiro, de onde
seguiam para a metrpole portuguesa. So 1.512 km que permitem mergulhar na
histria brasileira. A circulao de pessoas, mercadorias e riquezas era
obrigatoriamente feita por aqueles caminhos, constituindo crime de lesa-majestade
a abertura de outros no autorizados pela administrao metropolitana.
Adaptado de http://360graus.terra.com.br

A expanso da colonizao na Amrica portuguesa, nos sculos XVII e XVIII,
ocasionou o surgimento de novas atividades econmicas, de ncleos de
povoamento e de caminhos e estradas, como os que compuseram a Estrada Real.

Cite a principal atividade econmica que condicionou o surgimento dos caminhos
da Estrada Real e identifique dois interesses da Coroa portuguesa em controlar
esses caminhos, no decorrer do sculo XVIII.

23 - (UNIRG TO) Dentre as consequncias das descobertas de ouro e diamante no Brasil colnia, INCORRETO
afirmar:
a) Ocorreu o desenvolvimento de um novo eixo econmico na Colnia, deslocando as atividades principais da costa
litornea nordestina para o centro-sul, o que determinou a transferncia da capital de Salvador para o Rio de
Janeiro, em 1763.
b) Ocorreu um fortalecimento da ocupao e o povoamento da populao na rea rural, principalmente nas capitanias
de Minas Gerais, Gois e Mato Grosso.
c) Ocorreu a abertura de estradas e caminhos ligando a regio das minas ao porto do Rio de Janeiro o que propiciou
a intensificao do comrcio, visando, sobretudo, ao abastecimento da regio mineradora.
d) O mercado consumidor se ampliou e se aqueceu, em decorrncia do crescimento populacional da riqueza obtida
com o ouro.

24 - (UNICAMP SP) Durante o sculo XVIII, a capitania de So Paulo sofreu grandes transformaes territoriais e
administrativas. Em 1709, nasceu a capitania de So Paulo e das Minas do ouro, abrangendo imenso territrio
correspondente quase totalidade das atuais regies Sul, Sudeste e Centro-Oeste, exceo da ento capitania do
Rio de Janeiro e do Esprito Santo. At 1748, sucessivos desmembramentos formaram as regies de Minas, Santa
Catarina, Rio Grande de So Pedro, Gois e Mato Grosso. O novo capito-general, mais conhecido como Morgado de
Mateus, foi diretamente instrudo pelo futuro Marqus de Pombal a ocupar-se da fronteira oeste ameaada pelos
espanhis e a fomentar a produo de gneros de exportao.
(Adaptado de Ana Paula Medicci, "So
Paulo nos projetos de imprio", em Wilma Peres Costa e Ceclia
Helena de Oliveira, De um imprio a outro: formao do Brasil,
sculos XVIII e XIX. So Paulo: Hucitec/Fapesp, 2007, p. 243.)
a) Cite duas atividades econmicas que sustentavam a capitania de So Paulo no sculo XVIII.
b) Considerando a poltica territorial na Amrica Portuguesa nos sculos XVI e XVII, comente as mudanas
significativas do sculo XVIII nesse aspecto.

25 - (UNESP SP/) Entre as caractersticas da sociedade da regio das Minas Gerais no perodo da extrao de ouro,
no sculo XVIII, podemos citar:
a) maior mobilidade social que no restante da colnia.
b) pequeno desenvolvimento artstico e ausncia de estmulo produo cultural.
c) predomnio do meio rural sobre o urbano, como no restante da colnia.
d) comrcio interno restrito e ausncia de setores sociais intermedirios.
e) menor presena de irmandades religiosas que no restante da colnia.

297











PROF.: JEFT RODRIGUES DE BARROS







C
C
o
o
n
n
t
t
e
e

d
d
o
o
s
s
:
:

R
R
e
e
v
v
o
o
l
l
t
t
a
a
s
s
N
N
a
a
t
t
i
i
v
v
i
i
s
s
t
t
a
a
s
s
e
e
E
E
m
m
a
a
n
n
c
c
i
i
p
p
a
a
c
c
i
i
o
o
n
n
i
i
s
s
t
t
a
a
s
s
,
,
P
P
e
e
r
r

o
o
d
d
o
o

J
J
o
o
a
a
n
n
i
i
n
n
o
o
,
,
I
I
n
n
d
d
e
e
p
p
e
e
n
n
d
d

n
n
c
c
i
i
a
a
d
d
o
o
B
B
r
r
a
a
s
s
i
i
l
l




M
M
i
i
n
n
i
i
s
s
t
t
r
r
a
a
d
d
o
o
s
s
d
d
u
u
r
r
a
a
n
n
t
t
e
e
o
o
m
m

s
s
d
d
e
e
m
m
a
a
r
r

o
o















D Da at ta a d da a E En nt tr re eg ga a : : _ __ __ __ __ __ __ __ _/ /_ __ __ __ __ __ __ __ __ __ __ _/ /2 20 01 13 3
LISTA 2 HISTRIA

298

01 - (UECE) Considere as afirmaes a seguir em relao Guerra dos Mascates ocorrida na capitania de
Pernambuco, entre 1710 e 1711:

I. A Guerra dos Mascates foi um conflito entre os comerciantes de Recife e os proprietrios de terras de Olinda, no
contexto em que, a primeira florescia e a segunda mostrava claros sinais de decadncia.
II. A vitria dos comerciantes de Recife possibilitou a emancipao de sua vila e o fim da sujeio poltica,
administrativa e jurdica a Olinda.
III. O discurso dos olindenses derrotados era aquele que os afirmava como nobres homens da terra, destitudos de
suas prerrogativas por estrangeiros e seus descendentes aventureiros.
Est correto o que se afirma em
a) III apenas.
b) II e III apenas.
c) I apenas.
d) I, II e III.

02 - (UNICAMP SP) Emboaba: nome indgena que significa o estrangeiro, atribudo aos forasteiros pelos paulistas,
primeiros povoadores da regio das minas. Com a descoberta do ouro em fins do sculo XVII, milhares de pessoas da
colnia e da metrpole vieram para as minas, causando grandes tumultos. Formaram-se duas faces, paulistas e
emboabas, que disputavam o governo do territrio, tentando impor suas prprias leis.
(Adaptado de Maria Beatriz Nizza da Silva (coord.), Dicionrio da Histria
da Colonizao Portuguesa no Brasil. Lisboa: Verbo, 1994, p. 285.)
Sobre o perodo em questo correto afirmar que:

a) As disputas pelo territrio emboaba colocaram em confronto paulistas e mineiros, que lutaram pela posse e
explorao das minas.
b) A regio das minas foi politicamente convulsionada desde sua formao, em fins do sculo XVII, o que explica a
resistncia local aos inconfidentes mineiros.
c) A luta dos emboabas ilustra o processo de conquista de fronteiras do imprio portugus nas Amricas, enquanto
na frica os portugueses se retiravam definitivamente no sculo XVIII.
d) A monarquia portuguesa administrava territrios distintos e vrios sujeitos sociais, muitos deles em disputa entre si,
como paulistas e emboabas, ambos sditos da Coroa.

03 - (FGV) Ele virou um heri nacional, antecessor de Tiradentes e coisas do tipo. [Ele] foi enforcado realmente,
punido pela Coroa porque prendeu o governador e o mandou de volta para a metrpole. Mas quais eram suas
reivindicaes? Primeiro, reivindicava que a Coroa chamasse os jesutas de volta para Portugal, porque eles
atrapalhavam o uso dos ndios, impedindo sua escravizao. Segundo, como no se tinha ndios para trabalhar, tinha-
se que comprar escravos africanos. Mas s se podia comprar da Companhia de Comrcio, que colocava o preo nas
nuvens. Logo, as grandes reivindicaes desse heri eram o direito de escravizar ndios e de comprar africanos a
preo baixo.
(Fernando Novaes. Tendncia e debate, n.o 4, abril, maio e junho de 2000.
Apud Luiz Koshiba e Denise Manzi Frayze Pereira. Histria do Brasil
no contexto da histria ocidental, 2003.)
A rebelio analisada ocorreu no contexto

a) do reforo das restries mercantilistas, decorrente das frgeis condies do Imprio portugus aps a
Restaurao de 1640.
b) das Reformas Pombalinas, na segunda metade do sculo XVIII, que impuseram ao Brasil uma srie de
mecanismos opressores.
c) das primeiras descobertas de ouro em Minas Gerais, que geraram relaes tensas entre os paulistas e os
emboabas ou forasteiros.
d) da transio das capitanias hereditrias organizao do governo-geral, no sculo XVI, que trouxe grande prejuzo
aos senhores de terra.
e) da explorao dos bandeirantes paulistas em terras oeste de Minas Gerais, como Gois e Mato Grosso, a partir
de 1720.
Caderno de Atividades

Disciplina:
Histria

Professor(a):
Jeft
Aluno:
3 ano
Ensino Mdio
Data de Recebimento:
_____/_____/_____
Lista 01

Data Entrega:
_____/_____/_____

299
04 - (UFG GO) As regies mineratrias do Brasil colonial se constituram em espaos marcados pela experincia do
conflito e da violncia entre os diversos grupos sociais que as habitavam. A Guerra dos Emboabas (1707-1709) um
dos principais exemplos dessa experincia de conflito. Tomando como base esse acontecimento, explique:
a) o motivo para a deflagrao do conflito.
b) a ao da Coroa Portuguesa diante dos conflitos entre emboabas e paulistas.

05 - (UFMG) O sculo XVIII foi palco de uma srie de movimentos e sedies, nos quais, em diferentes graus e a
partir de diferentes estratgias, os vassalos da Amrica Portuguesa procuraram redefinir o formato de suas relaes
com a Coroa Portuguesa.
Considerando-se esse contexto, CORRETO afirmar que

a) a revolta de Filipe dos Santos, em Minas Gerais, na primeira metade desse sculo, reforou os mecanismos de
controle sobre os vassalos.
b) a revolta do Vintm e a do Quebra-quilos, na segunda metade desse sculo, ao desafiarem a Coroa, colocaram em
crise a sede do Vice-Reinado.
c) a revolta dos Tvora procurou estabelecer novos limites para a cobrana do Subsdio Literrio, destinado educao
dos vassalos.
d) os conflitos entre paulistas e emboabas, nas Minas Gerais, levaram instalao das casas de fundio nessa
Capitania.

06 - (UFTM MG) Era bvia a seduo que o enforcamento do alferes representava para o governo portugus: pouca
gente levaria a srio um movimento chefiado por um simples Tiradentes (e as autoridades lusas, depois de 1790,
invariavelmente se referiam ao alferes por seu apelido de Tiradentes). Um julgamento-exibio, seguido pela execuo
pblica de Silva Xavier, proporcionaria o impacto mximo, como advertncia, ao mesmo tempo que minimizaria e
ridicularizaria os objetivos do movimento: Tiradentes seria um perfeito exemplo para outros colonos descontentes e
tentados a pedir demais antes do tempo.
(Kenneth Maxwell. A devassa da devassa, 1978.)
O texto permite afirmar que
a) o fato de o movimento ser chefiado por um simples Tiradentes foi a razo do seu fracasso.
b) o governo tentou diminuir a relevncia da revolta e aplicou punio exemplar em Tiradentes.
c) o alferes foi enforcado por sua capacidade de liderar e seduzir os setores mais pobres do povo.
d) o despreparo de Tiradentes acabou por frustrar os planos de revolta contra os portugueses.
e) o movimento chefiado por Tiradentes no chegou a preocupar as autoridades portuguesas.

07 - (UEPB) Considerando os conflitos sociais que ocorreram na segunda metade do sculo XVIII na Amrica
portuguesa, podemos afirmar:
a) tanto na Inconfidncia Mineira, quanto na Baiana estava presente, em parcela dos integrantes, a noo de que os
governos deveriam existir para garantir direitos naturais dos homens como a liberdade e a propriedade.
b) a Inconfidncia Mineira no recebeu influncia do movimento de independncia dos EUA ocorrido em 1776.
c) as ideias liberais e iluministas no motivaram os principais proprietrios de terras e de reas extrativas de ouro.
d) tendo participao dos alfaiates como lderes da conjurao baiana, no se percebe influncia de ideais i luministas
no movimento.
e) os inconfidentes mineiros tinham como projeto a ruptura com Portugal e a criao da Repblica Federativa do Brasil,
conforme conhecemos hoje.

08 - (UFRN) Sobre a chamada Inconfidncia Mineira, a historiadora Cristina Leminski afirmou:

Sem a derrama, o movimento esvaziava-se. Para a populao em geral, se a derrama no fosse imposta, no fazia
grande diferena se Minas era ou no independente. O movimento era fundamentalmente motivado por interesses, no
por ideais [...]. A priso dos homens mais eminentes de Vila Rica provocou [...] alvoroo na cidade [...] e o Visconde de
Barbacena foi obrigado a admitir que a tentativa de manter sigilo sobre o processo seria intil.
LEMINSKI, Cristina. Tiradentes e a conspirao de Minas Gerais. So Paulo: Scipione, 1994. p. 59-64.
O movimento do sculo XVIII abordado nesse fragmento textual relaciona-se com a
a) pretenso das lideranas econmicas de Vila Rica, principais beneficiadas com a arrecadao tributria portuguesa.
b) repercusso da Revoluo Francesa no seio da elite intelectual colonial da regio aurfera nas Minas Gerais.
c) explorao tributria feita pela Metrpole sobre os colonos portugueses, no contexto da crise do antigo sistema
colonial.
d) revolta desencadeada pela deciso da Coroa de instalar as Casas de Fundio, com o propsito de cobrar o quinto.






300
09 - (UFBA) Liberdade poltica e liberdade econmica foram reivindicaes que estiveram presentes no contexto
da crise do Sistema Colonial portugus no Brasil.
Com base nessa afirmao, indique como essas reivindicaes so expressas na Revoluo Pernambucana de
1817.
Liberdade poltica
Liberdade econmica

10 - (UFPE) A leitura do processo da Revoluo dos Alfaiates nos leva a compreender que essa revoluo:
00. adotou a mesma estrutura militar da Inconfidncia Mineira.
01. defendeu idias radicais e revolucionrias para a poca.
02. no questionou a existncia do regime escravocrata na colnia.
03. contou com a participao de membros do clero baiano.
04. negou a importncia dos intelectuais para o movimento.

11- (FUVEST SP) Fui terra fazer compras com Glennie. H muitas casas inglesas, tais como celeiros e armazns
no diferentes do que chamamos na Inglaterra de armazns italianos, de secos e molhados, mas, em geral, os
ingleses aqui vendem suas mercadorias em grosso a retalhistas nativos ou franceses. (...) As ruas esto, em geral,
repletas de mercadorias inglesas. A cada porta as palavras Superfino de Londres saltam aos olhos: algodo
estampado, panos largos, loua de barro, mas, acima de tudo, ferragens de Birmingham, podem-se obter um pouco
mais caro do que em nossa terra nas lojas do Brasil.
Maria Graham. Dirio de uma viagem ao Brasil.
So Paulo, Edusp, 1990, p. 230
(publicado originalmente em 1824). Adaptado.
Esse trecho do dirio da inglesa Maria Graham refere-se sua estada no Rio de Janeiro em 1822 e foi escrito em 21
de janeiro deste mesmo ano. Essas anotaes mostram alguns efeitos

a) do Ato de Navegao, de 1651, que retirou da Inglaterra o controle militar e comercial dos mares do norte, mas
permitiu sua interferncia nas colnias ultramarinas do sul.
b) do Tratado de Methuen, de 1703, que estabeleceu a troca regular de produtos portugueses por mercadorias de
outros pases europeus, que seriam tambm distribudas nas colnias.
c) da abertura dos portos do Brasil s naes amigas, decretada por D. Joo em 1808, aps a chegada da famlia
real portuguesa Amrica.
d) do Tratado de Comrcio e Navegao, de 1810, que deu incio exportao de produtos do Brasil para a
Inglaterra e eliminou a concorrncia hispanoamericana.
e) da ao expansionista inglesa sobre a Amrica do Sul, gradualmente anexada ao Imprio Britnico, aps sua
vitria sobre as tropas napolenicas, em 1815.

12 - (UEM PR) No incio do sculo XIX, em razo das guerras napolenicas, a Corte Portuguesa transfere-se para o
Rio de Janeiro. A colnia portuguesa na Amrica torna-se a sede da Corte. A esse respeito, assinale a(s) alternativa(s)
correta(s).
01. Ao chegar ao Rio de Janeiro, o rei de Portugal realiza uma srie de reformas administrativas e culturais para
adaptar a cidade s necessidades da vida cortes.
02. Nesse perodo, o Brasil recebe uma grande influncia da cultura europeia, sobretudo, aps a chegada da misso
artstica francesa.
04. Nesse perodo, Jean Baptiste Debret documentou, em seus desenhos e aquarelas, dentre outros temas, cenas da
sociedade do Rio de Janeiro, os usos e costumes dos habitantes da colnia.
08. Na arquitetura, a misso artstica francesa desenvolveu o estilo neoclssico, abandonando os princpios barrocos.
16. Com o final do perodo joanino e o retorno da misso artstica francesa Europa, cessou, no Brasil, a influncia
cultural francesa e se iniciou o movimento modernista brasileiro.

13- (ENEM)Aps a Independncia, integramo-nos como exportadores de produtos primrios diviso internacional do
trabalho, estruturada ao redor da Gr-Bretanha. O Brasil especializou-se na produo, com brao escravo importado
da frica, de plantas tropicais para a Europa e a Amrica do Norte. Isso atrasou o desenvolvimento de nossa economia
por pelo menos uns oitenta anos. ramos um pas essencialmente agrcola e tecnicamente atrasado por depender de
produtores cativos. No se poderia confiar a trabalhadores forados outros instrumentos de produo que os mais
toscos e baratos. O atraso econmico forou o Brasil a se voltar para fora. Era do exterior que vinham os bens de
consumo que fundamentavam um padro de vida civilizado, marca que distinguia as classes cultas e naturalmente
dominantes do povaru primitivo e miservel. (...) E de fora vinham tambm os capitais que permitiam iniciar a
construo de uma infraestrutura de servios urbanos, de energia, transportes e comunicaes.
Paul Singer. Evoluo da economia e vinculao internacional.
In: I. Sachs; J. Willheim; P. S. Pinheiro (Orgs.). Brasil: um sculo
de transformaes. So Paulo: Cia. das Letras, 2001, p. 80.
Levando-se em considerao as afirmaes acima, relativas estrutura econmica do Brasil por ocasio da
independncia poltica (1822), correto afirmar que o pas


301
a) se industrializou rapidamente devido ao desenvolvimento alcanado no perodo colonial.
b) extinguiu a produo colonial baseada na escravido e fundamentou a produo no trabalho livre.
c) se tornou dependente da economia europia por realizar tardiamente sua industrializao em relao a outros
pases.
d) se tornou dependente do capital estrangeiro, que foi introduzido no pas sem trazer ganhos para a infraestrutura de
servios urbanos.
e) teve sua industrializao estimulada pela Gr-Bretanha, que investiu capitais em vrios setores produtivos.

14 - (UFG GO) As regies mineratrias do Brasil colonial se constituram em espaos marcados pela experincia do
conflito e da violncia entre os diversos grupos sociais que as habitavam. A Guerra dos Emboabas (1707-1709) um
dos principais exemplos dessa experincia de conflito. Tomando como base esse acontecimento, explique:

a) o motivo para a deflagrao do conflito.
b) a ao da Coroa Portuguesa diante dos conflitos entre emboabas e paulistas.

15 - (FUVEST SP) Indique as principais razes da insurreio pernambucana contra os holandeses, ocorrida entre
1645 e 1654.

16 - (UNICAMP SP) Comentando a Guerra dos Emboadas (1709), o historiador Antnio Srgio escreveu:

Cedo no Brasil se buscaram as minas. Para isso se organizavam expedies (bandeiras) que se internavam pelo
serto. Enfim, a descoberta fez-se e a notcia atraiu muita gente. Os habitantes de So Paulo consiideravam como
inimigos todos os que pretendiam, como eles enriquecer com o ouro.
(Adaptado de Antnio Srgio, Breve Interpretao da Histria de Portugual)

a) Quem eram os emboabas e por que os paulistas entraram na guerra contra eles?
b) Explique as transformaes econmicas que a minerao provocou no Brasil.

17 - (UFBA) Liberdade poltica e liberdade econmica foram reivindicaes que estiveram presentes no contexto
da crise do Sistema Colonial portugus no Brasil.
Com base nessa afirmao, indique como essas reivindicaes so expressas na Revoluo Pernambucana de
1817.
Liberdade poltica
Liberdade econmica

18 - (UFRN) Em 1817, com a deflagrao da Revoluo Pernambucana, a rea que atualmente compreende o
Nordeste do Brasil convulsionou-se. Nesse contexto, Pernambuco ocupava uma posio dominante em relao aos
seus vizinhos Cear, Rio Grande do Norte, Paraba, Alagoas e Sergipe. Sendo Pernambuco o principal plo
econmico, poltico e administrativo da regio, a insurreio rapidamente se espalhou pelas capitanias vizinhas,
inaugurando um ciclo revolucionrio no Nordeste.

a) Cite e explique duas caractersticas da Revoluo Pernambucana de 1817.
b) Mencione duas implicaes desse movimento revolucionrio no Rio Grande do Norte.

19 - (FGV) No texto a seguir, podemos ler, em linguagem da poca, fragmentos da instruo do processo dos
participantes de uma rebelio ocorrida na Bahia no fim do sculo XVIII; depois da leitura, responda aos subitens.

"Huma formal, inteira sublevao", devendo-se matar a todos da sua governana e "saqueando-se os cabedais das
pessoas opulentas". Para este revolucionrio o "systhema francs" era o modelo: "he melhor a regncia de muitos, do
que de um s". Propunha a "liberdade a todos os creados, estabelecendo uma Repblica de igualdade."
(Processo de Manuel de Santa Anna, "pardo", soldado
do Segundo Regimento de linha em Salvador. Autos
da Devassa do levantamento e Sedio Intentada na
Bahia em 1798, Anais do Arquivo Pblico da Bahia.
In Mota, Carlos Guilherme, "Brasil revisitado")

C.a)Como foi conhecida a rebelio mencionada, quais os seus objetivos gerais e quais os segmentos sociais e
ocupaes dos seus participantes? (7)
C.b Que movimentos de grande importncia social e poltica haviam acontecido no mundo Atlntico (Europa e
Amricas) nas ltimas dcadas do sculo XVIII e quais as idias filosficas e polticas em que haviam se baseado?
(8)
C.c)Quais as razes dos diferentes nveis de rigor na represso Inconfidncia Mineira, na qual apenas Tiradentes foi
executado, e ao movimento baiano de 1798, no qual ocorreram vrias execues e esquartejamentos? (9)


302
20 - (UNICAMP SP) No quadro das revoltas ocorridas em Minas Gerais na primeira metade do sculo XVIII entre
1707 e 1736 , verificamos, em algumas delas, elementos de marcante originalidade, por contestarem abertamente os
direitos do Rei e envolverem participao ativa de segmentos procedentes dos estratos sociais inferiores.
(Adaptado de Luciano Raposo de Almeida Figueiredo, O Imprio em apuros: notas para o estudo das relaes ultramarinas no
Imprio Portugus, sculos XVII e XVIII, em Jnia Furtado (org.). Dilogos ocenicos: Minas Gerais e as novas abordagens para
uma histria do Imprio Ultramarino Portugus. Belo Horizonte: UFMG, 2001, p. 236.)
a) Segundo o texto, quais eram as caractersticas originais apresentadas por algumas revoltas ocorridas na primeira
metade do sculo XVIII?
b) D duas caractersticas da Inconfidncia Mineira que a diferenciam das revoltas ocorridas na primeira metade do
sculo XVIII.

21 - (UFRRJ) Em 21 de abril de 1792, no Campo de So Domingos, Rio de Janeiro, a justia de Sua Majestade
Dona Maria I mandava executar o ru Joaquim Jos da Silva Xavier pelo horroroso crime de rebelio e alta traio de
que foi considerado chefe, j que, na distante capitania de Minas Gerais, atentara com a mais escandalosa temeridade
contra a Real Soberania, e Suprema autoridade da mesma senhora.
Mandado para execuo da pena imposta a Joaquim Jos da Silva
Xavier. In: Autos de devassa da Inconfidncia Mineira, 1938.

O texto se refere punio imposta a Tiradentes, preso, no Rio de Janeiro em 10 de maio de 1789, por ter participado
da Conjurao Mineira.
Aponte o movimento de idias e um evento poltico que influenciaram o movimento do qual Tiradentes participou.

22 - (UERJ) O enriquecimento da vida cultural do Rio de Janeiro, e at mesmo do pas, aps 1808, decorreu,
sobretudo, das necessidades da elite dominante. No ambiente acanhado da sociedade americana, a novidade dos
procedimentos caractersticos do crculo real exerceram extraordinrio fascnio, produzindo um poderoso efeito
civilizador em relao cidade. Em contrapartida, a Coroa no deixou de adotar tambm medidas de controle mais
eficientes. Aps a tormenta da Revoluo Francesa e ainda vivendo o turbilho do perodo napolenico, era o medo
dos princpios difundidos pelo sculo das Luzes, especialmente as perniciosas ideias francesas, que ditava essas
cautelas.
LCIA M. P. DAS NEVES E HUMBERTO F. MACHADO
Adaptado de O imprio do Brasil. Rio de Janeiro: Nova Fronteira, 1999.
O texto aborda um duplo movimento provocado pela presena da Corte portuguesa no Brasil: o estmulo s atividades
culturais na colnia e, ao mesmo tempo, o controle conservador sobre essas atividades.
Indique duas aes da Coroa que enriqueceram a vida cultural da cidade do Rio de Janeiro. Explique, ainda, como o
Estado portugus exercia controle sobre as atividades culturais.

23 - (UERJ) Possa este, para sempre memorvel dia, ser celebrado com universal jbilo por toda a Amrica
Portuguesa, por uma dilatada srie de sculos, como aquele em que comeou a raiar a aurora da felicidade,
prosperidade e grandeza, a que algum dia o Brasil se h de elevar, sendo governado de perto pelo seu soberano. Sim,
ns j comeamos a sentir os saudveis efeitos da paternal presena de to timo prncipe, que [...] nos deu as mais
evidentes provas, que muito alentam as nossas esperanas, de que viera ao Brasil a criar um grande Imprio.
Lus Gonalves dos Santos
Mamrias para servir Histria do reino do Brasil. Belo Horizonte.
Ed. Itatiaia, So Paulo: EDUSP, 1981.

O texto acima revela o entusiasmo e as esperanas daqueles que assistiram chegada da famlia real portuguesa ao
Brasil. Indique duas inovaes de carter cientfico ou cultural decorrentes da poltica de D. Joo. Indique tambm uma
mudana poltica ou econmica observada durante a permanncia da Corte e sua respectiva conseqncia para o
Brasil.

24 - (UEG GO) Uma das funes bsicas do conhecimento histrico localizar adequadamente os acontecimentos
numa linha temporal. Nesse sentido,
a) coloque os seguintes fatos histricos do Brasil e do mundo, ocorridos na primeira metade do sculo XIX, na ordem
cronolgica correta;
b) faa uma anlise da emancipao poltica do Brasil, utilizando os acontecimentos do quadro que so pertinentes.
COROAO DE PEDRO II CONGRESSO DE VIENA INDEPENDNCIA DO BRASIL PRIMAVERA DOS POVOS
VINDA DA FAMLIA REAL PORTUGUESA AO BRASIL PROIBIO DO TRFICO NEGREIRO NO BRASIL
COROAO DE NAPOLEO BONAPARTE CONJURAO BAIANA ABDICAO DE D. PEDRO I.

25 - (UNESP SP) O Visconde de Taunay, autor de A retirada da Laguna, descendia de uma famlia que viera para o
Brasil com a Misso Artstica Francesa, durante o governo de D. Joo VI. Que condies polticas europias
contriburam para a vinda da Misso Artstica para o Brasil e qual foi um dos seus resultados artstico-culturais?



303







PROF.: JEFT RODRIGUES DE BARROS





C
C
o
o
n
n
t
t
e
e

d
d
o
o
s
s
:
:

P
P
r
r
i
i
m
m
e
e
i
i
r
r
o
o
R
R
e
e
i
i
n
n
a
a
d
d
o
o
,
,
P
P
e
e
r
r

o
o
d
d
o
o
R
R
e
e
g
g
e
e
n
n
c
c
i
i
a
a
l
l
e
e
S
S
e
e
g
g
u
u
n
n
d
d
o
o

R
R
e
e
i
i
n
n
a
a
d
d
o
o
(
(
P
P
a
a
r
r
t
t
e
e
1
1
)
)





M
M
i
i
n
n
i
i
s
s
t
t
r
r
a
a
d
d
o
o
s
s
d
d
u
u
r
r
a
a
n
n
t
t
e
e
o
o
m
m

s
s
d
d
e
e
a
a
b
b
r
r
i
i
l
l













D Da at ta a d da a E En nt tr re eg ga a : : _ __ __ __ __ __ __ __ _/ /_ __ __ __ __ __ __ __ __ __ __ _/ /2 20 01 13 3
LISTA 3 HISTRIA

304


01 - (UECE) Acerca do processo de independncia no Brasil, isto , da separao poltica entre a colnia e a
metrpole portuguesas em 1822, correto afirmar-se que
a) culminou juntamente com o processo da consolidao da unidade nacional.
b) foi marcado por um movimento propriamente nacionalista e revolucionrio.
c) representou a imagem tradicional da colnia em guerra contra a metrpole.
d) resultou de uma reao conservadora provocada por interesses comuns de certos setores da elite brasileira, bem
como do Imperador.

02 - (MACK SP) No ano de sua independncia, o Brasil tinha [...] tudo para dar errado. De cada trs brasileiros, dois
eram escravos, negros forros, mulatos, ndios ou mestios. Era uma populao pobre e carente [...]. O medo de uma
rebelio dos cativos assombrava a minoria branca. O analfabetismo era geral. [...]. Os ricos eram poucos e, com raras
excees, ignorantes. O isolamento e as rivalidades entre as provncias prenunciavam uma guerra civil [...].
Laurentino Gomes, 1822
correto afirmar que a independncia do Brasil s no confirmou os temores apresentados no trecho,
a) porque ao defender a revoluo popular de inspirao camponesa, inspirou legisladores como Jos Bonifcio e
Joaquim Nabuco a defenderem a emancipao completa em relao a Portugal.
b) porque o povo conseguiu entender os anseios de D. Pedro e da elite brasileira, ao pegar em armas e defender at
a morte uma independncia que parecia condenada em sua prpria estrutura.
c) porque foi realizada revelia da populao pobre destacadamente de origem africana e indgena uma vez que
suas simpatias pela Revoluo Americana ameaavam os poderes da elite branca.
d) porque parcelas significativas da elite brasileira se aglutinaram em torno de D. Pedro, a fim de manter as antigas
bases de um Brasil colonial na estrutura do novo pas que nascia em 1822.
e) porque foi inspirada pela Revoluo Francesa e pelas ideias iluministas, no contexto da crise do Antigo Sistema
Colonial, sendo liderada pela elite burguesa contra a tirania representada por D. Pedro.

03 - (UFPB) A emancipao do Brasil de Portugal no foi um fato isolado, ocorrido s margens do Rio Ipiranga, como
ainda mostram alguns livros didticos. Na verdade, a declarao de independncia a culminncia de um rico
processo de articulao entre as classes dominantes, que assegurou reformas mas impediu uma revoluo social.
Sobre o processo de autonomia do Brasil, identifique as afirmativas corretas:
I. A ideia de separao poltica j estava presente em diversos movimentos nativistas, entre os quais podem ser
citados a Inconfidncia Mineira, a Conjurao Baiana e a Revoluo Pernambucana de 1817.
II. A abertura dos Portos, em 1808, significou o fortalecimento do Pacto Colonial, que estreitou, ainda mais, os laos
comerciais entre a Colnia e a metrpole portuguesa.
III. A Inglaterra tinha interesse direto no processo de separao do Brasil, na medida em que mantinha relaes
econmicas cada vez mais frequentes com as elites coloniais.
IV. A elite intelectual e agrria brasileira, desiludida com o sistema escravista, defendia um modelo de independncia
que mudasse a forma de trabalho e de propriedade.
V. A autonomia do Brasil se deu apenas no campo poltico, uma vez que a estrutura econmica e social, baseada no
latifndio e no trabalho escravo, foi preservada.

04 - (UFG GO) Analise a imagem a seguir.
Produzida em 1822, esta pintura constituiu uma alegoria do
Estado nacional por ocasio da Independncia. Nela se
construiu uma imagem positiva do Imprio e do papel
poltico do monarca, aclamado como Defensor Perptuo do
Brasil. Ao longo do Primeiro Reinado, entretanto, a imagem do
monarca se modificou.
DEBRET, Jean-Baptiste. Viagem pitoresca e histrica pelo Brasil.
Belo Horizonte: Itatiaia; So Paulo: Edusp, 1989. s. p. (Figura 144).
Diante do exposto e com base na anlise da pintura,
explique
a) uma caracterstica do projeto poltico monrquico do Primeiro Reinado;
b) um dos motivos que levaram mudana da imagem de D. Pedro I ao longo do Primeiro Reinado.

Caderno de Atividades

Disciplina:
Histria

Professor(a):
Jeft
Aluno:
3 ano
Ensino Mdio
Data de Recebimento:
_____/_____/_____
Lista 03

Data Entrega:
_____/_____/_____

305
05 - (UERJ) Bandeira da Confederao do Equador

www.historiabrasileira.com

O poder moderador de nova inveno maquiavlica a chave mestra da opresso da nao brasileira e o garrote mais
forte da liberdade dos povos. princpio conhecido pelas Luzes do presente sculo que a soberania reside na nao
essencialmente, logo sem questo que a mesma nao ou pessoa da comisso quem deve esboar a sua
constituio, purific-la das imperfeies e afinal estatu-la.
Frei Joaquim do Amor Divino Caneca
Crtica da constituio outorgada, 1824
Adaptado de JUNQUEIRA, Celina (org). Ensaios polticos. Rio de Janeiro: Documentrio, 1976.

A Confederao do Equador, ocorrida em 1824, apresentou propostas alternativas organizao do Imprio do Brasil,
sendo porm reprimidas pelo governo de Pedro I.
Explicite o motivo central para a ecloso da Confederao do Equador e cite duas de suas propostas para a
organizao do poder de Estado.

06 - (UFRN) Aps ser proclamada a Independncia do Brasil, o Imprio foi regido por uma Constituio outorgada por
D. Pedro I, em 1824. Nos artigos referentes s eleies, l-se:

CAPTULO VI
Das Eleies

Art. 90. As nomeaes dos deputados e senadores para a Assembleia Geral e dos membros dos conselhos gerais das
provncias sero feitas por eleies indiretas, elegendo a massa dos cidados ativos em assembleias paroquiais os
eleitores de provncias e estes os representantes da nao e provncia.

Art. 91. Tm votos nestas eleies primrias:
1) Os cidados brasileiros que esto no gozo de seus direitos polticos;
2) Os estrangeiros naturalizados.

Art. 92. So excludos de votar nas Assembleias Paroquiais: [...]
5) Os que no tiverem de renda lquida anual 100$000 [100 mil ris] por bens de raiz [imveis], indstria, comrcio ou
empregos.

Art. 93. Os que no podem votar nas Assembleias primrias de Parquias no podem ser membros, nem votar na
nomeao de alguma autoridade eletiva nacional.
MINISTRIO DA JUSTIA. Constituio poltica do imprio do Brasil,
edio fac-similar. Rio de Janeiro: Arquivo Nacional, 1972.[Adaptado]

Os artigos citados permitem inferir que

a) a doutrina de origem europeia, que inspirou a emancipao poltica e a Constituio, se opunha ideia de igualdade.
b) o voto censitrio, nos termos adotados no texto constitucional, caracterizou a chamada Constituio da mandioca.
c) o texto constitucional imposto Nao restringiu a participao poltica das camadas populares.
d) a implantao do voto censitrio tornou o direito de votar restrito s elites latifundirias.

07 - (UFPE) No Brasil, as rebelies coloniais mostraram que a dominao portuguesa encontrava dificuldades e
provocava insatisfaes. As ideias liberais contriburam para acirrar as disputas e promover discusses sobre a
existncia da opresso poltica, mesmo depois de 1822. Em Pernambuco, por exemplo, no sculo XIX, houve:

00. rebelies importantes que exigiram a ruptura em relao a princpios ditos mercantilistas e propuseram mudanas
na organizao poltica.

306
01. influncias marcantes das teorias socialistas que ajudaram na organizao de revoltas escravistas contra os
latifundirios e contra os comerciantes portugueses estabelecidos no Recife.
02. conflitos polticos entre Olinda e Recife, onde se firmou a superioridade econmica do Recife e a afirmao de
governos simpticos aos princpios antimercantilistas.
03. revoltas polticas, que contaram com a ajuda de tropas vindas do sudeste do pas, onde, no final do perodo
colonial e com a participao ativa de padres e escravos, obtiveram xitos.
04. o governo de D. Pedro I no agradava aos mais liberais e incomodava quem queria a instalao de uma
Repblica.

08 - (UECE) A renncia de D. Pedro I em 07 de abril de 1831 foi resultado de uma srie de acontecimentos que
provocaram um descontentamento geral no Imprio brasileiro. Como causas do descontentamento do povo brasileiro
com D. Pedro I, so apontadas as seguintes:

I. Dissoluo da Assemblia Constituinte por D. Pedro I bem como a nomeao de uma comisso dirigida por ele
mesmo para redigir um novo projeto constitucional.
II. Atitude de passividade e tolerncia de D. Pedro I com os grupos das provncias do Norte e Nordeste que estavam
insatisfeitos com o desempenho de suas aes polticas na conduo do Imprio.
III. Participao direta de D. Pedro I na questo da sucesso do trono aps a morte de D. Joo VI, dedicando grande
parte de seus esforos disputa poltica que ocorria em Portugal.

realmente causa do descontentamento do povo brasileiro o apontado nos itens
a) I e II, apenas.
b) I, II e III.
c) II e III, apenas.
d) I e III, apenas.

09 - (UECE) O denominado Golpe da Maioridade, processo concludo em 23 de julho de 1840, pode ser considerado
uma vitria do(s)

a) segmentos populares, que promoveram agitaes em vrias provncias em prol da entronizao do futuro Imperador,
comprometido com as demandas destes segmentos.
b) Partido Conservador, que tramava secretamente pela votao de uma emenda a favor da maioridade, visando
ampliar sua rea de influncia poltica.
c) dois partidos Liberal e Conservador pois ambos se uniram em torno da causa da Maioridade do Prncipe
Herdeiro. Afinal, no havia grandes cises ideolgicas entre os dois grupos.
d) Partido Liberal, criador do Clube da Maioridade, que dirigiu uma comisso ao Prncipe Herdeiro, com o pedido de
antecipao de sua maioridade, recebendo uma resposta positiva.

10 - (ESPM) Em Poncho Verde, regio assim denominada pelas suas verdes campinas, timas para o pastoreio de
gado, hoje sede do municpio de Dom Pedrito, foi firmado o Tratado de Poncho Verde.
(Wikipdia, Helga Piccolo: A paz dos Caramurus, caderno de Histria n 4)

O Tratado de Poncho Verde foi:
a) o acordo pelo qual Santa Catarina aderiu Guerra dos Farrapos.
b) o acordo que ps fim Revoluo Farroupilha e Repblica Riograndense.
c) o acordo de paz que encerrou, em 23 de agosto de 1895, a Revoluo Federalista.
d) o acordo que estabeleceu os limites entre o Brasil e o Uruguai, em 1909.
e) o acordo que estabeleceu a paz entre os partidrios de Borges de Medeiros (chimangos) e os partidrios de Assis
Brasil (maragatos).

11 - (UERJ) Em nome do povo do Rio Grande, depus o governador e entreguei o governo ao seu substituto legal. E
em nome do Rio Grande do Sul, digo que nesta provncia extrema, afastada da Corte, no toleramos imposies
humilhantes. O Rio Grande a sentinela do Brasil que olha vigilante o Rio da Prata. No pode e nem deve ser
oprimido pelo despotismo. Exigimos que o governo imperial nos d um governador de nossa confiana, que olhe pelos
nossos interesses, ou, com a espada na mo, saberemos morrer com honra, ou viver com liberdade.
Carta escrita em 1835 por Bento Gonalves, lder farroupilha, ao Regente Feij.
Adaptado de PESAVENTO, S. J. A Revoluo Farroupilha.
So Paulo: Brasiliense, 1990.
Rio-grandenses! Tenho o prazer de anunciar-vos que a guerra civil que por mais de nove anos devastou esta bela
provncia est terminada. Os irmos contra quem combatamos esto hoje congratulados conosco e j obedecem ao
legtimo governo do Imprio do Brasil. Unio e tranquilidade sejam de hoje em diante nossa divisa. Viva a religio, viva
o Imperador Constitucional e Defensor Perptuo do Brasil. Viva a integridade do Imprio.
Proclamao feita pelo Baro de Caxias em 1845, fim da Revoluo Farroupilha.
Adaptado de SOUZA, A. B. de. Duque de Caxias: o homem por trs do monumento.
Rio de Janeiro: Civilizao Brasileira, 2008.

307
A consolidao do Imprio do Brasil, entre as dcadas de 1830 e 1850, significou a vitria de determinado projeto
poltico e tambm o combate de propostas, como as defendidas pelos que lutaram na Revoluo Farroupilha.
Aponte uma das propostas dos lderes farroupilhas e explique por que esse movimento foi considerado ameaador
pelos dirigentes do Imprio do Brasil.

12 - (UEPB) Sobre as rebelies regenciais correto afirmar:
a) foram motivados devido existncia de apenas um partido poltico que fortalecia o poder nas mos do imperador.
b) tinham como lideranas exclusivamente integrantes da elite agrria do imprio.
c) lutavam fundamentalmente por transformaes na estrutura fundiria de base escravista.
d) seus lderes tinham como projeto a centralizao do poder e o fim da escravido.
e) o agravamento da situao econmica, as disputas entre diferentes grupos de proprietrios pelo poder e a presena
de setores populares na cena poltica foram elementos motivadores destas rebelies.

13 - (UECE) Como fatores apontados para a desarticulao do movimento conhecido como BALAIADA, foram
listados os seguintes:

I. A represso do governo que acabou esmagando a resistncia popular com a retirada do movimento de elementos da
oligarquia maranhense.
II. O papel desempenhado pelas lideranas que asseguraram a forte unidade ideolgica do movimento.
III. A ausncia de unidade ideolgica e poltica entre os envolvidos no movimento.

Dentre os fatores citados anteriormente, realmente contriburam para a desarticulao do movimento os listados

a) nos itens I, II e III.
b) nos item I e II, apenas.
c) nos itens I e III, apenas.
d) nos itens II e III, apenas.

14 - (PUC RS) A chamada Guerra dos Farrapos (1835-1845), conflito que ops o Rio Grande do Sul ao governo
central do Imprio brasileiro, pode ser entendida como

a) o resultado do descontentamento quanto tributao do charque, que criava dificuldades na concorrncia com
produtores uruguaios e argentinos.
b) uma mobilizao da elite rio-grandense na defesa da abolio da escravido, sendo o momento de pice o episdio
dos Lanceiros Negros, no Cerro dos Porongos.
c) um movimento separatista, que pretendia unir o Rio Grande do Sul Repblica Cisplatina, com base em relaes j
estabelecidas pela provncia com os pases da Bacia do Prata.
d) fruto da discordncia em relao ao fechamento da Assemblia Constituinte e ao consequente decreto da
maioridade de D. Pedro II.
e) uma luta pela expanso dos latifndios destinados pecuria, para aumentar a produo de carne conservada in
natura.

15 - (UFF RJ)

Estao central da Estrada de Ferro Central do Brasil (Marc Ferrez, c. 1870).

A inaugurao da estao da Estrada de Ferro Central do Brasil em 1870 simboliza uma aparente contradio: o Brasil
era um pas escravocrata ao mesmo tempo que buscava instalar as novas e modernas mquinas e tecnologias
oriundas da Revoluo Industrial. O trem era, assim, o smbolo da modernizao.
A partir dessa afirmao, discuta a importncia da estrada de ferro na manuteno das relaes escravistas na
economia brasileira do sculo XIX.

16 - (UFPE) A economia brasileira no se livrou da sua dependncia externa, apesar de certa modernizao em
algumas regies e do crescimento da lavoura cafeeira no sculo XIX. Havia semelhanas entre a explorao agrcola
do caf e a da cana de acar. No entanto, importante salientar que:


308
00. a produo do acar entrou numa crise imensa depois de 1850, sendo incapaz de organizar suas plantaes e
manter seus escravos.
01. ambas sofriam de limites tcnicos, dificultando a obteno de maiores lucros e maior agilidade na comercializao
de seus produtos.
02. o caf teve uma importncia decisiva, chegando a representar cerca de 61,5% das exportaes nacionais no
perodo ureo.
03. se utilizaram da mo de obra escrava, no empregando, na sua produo, trabalhadores livres, nem contando com
a presena de europeus.
04. o acar teve produo destacada em todo o sculo XIX, com a implantao dos engenhos centrais e de usinas
com tecnologia avanada.

17 - (FUVEST SP) A formao histrica do atual Estado do Rio Grande do Sul est intrinsecamente relacionada
questo fronteiria existente entre os domnios das duas coroas Ibricas na Amrica meridional. Desde o sculo XVIII,
esta regio foi cenrio de constantes disputas territoriais entre diferentes agentes sociais. Atritos que no estiveram
restritos apenas s lutas travadas entre luso-brasileiros e hispano-americanos pelo domnio do Continente do Rio
Grande.
Eduardo Santos Neumann, A fronteira tripartida, Luiz
Alberto Grij (e outros). Captulos de Histria do Rio Grande
do Sul. Porto Alegre: Editora da UFRGS, 2004, p. 25. Adaptado.

a) Caracterize a questo fronteiria, mencionada no texto acima.
b) Quais so as principais diferenas e semelhanas entre a organizao socioeconmica do Rio Grande colonial e a
de regies aucareiras, como Bahia e Pernambuco, na mesma poca?

18 - (UEMG) O cafezal a soldadesca verde
que salta morros na distncia iluminada
um dois, um dois, de batalho em batalho
na sua arremetida acelerada contra o serto.
RICARDO, Cassiano. SOLDADOS VERDES
No decorrer do sculo XIX, viu-se o crescimento da produo de caf no Brasil. O produto, que foi hegemnico na
pauta de exportao do pas por muito tempo, gerou tanta riqueza que chegou a ser conhecido como o ouro verde.

Considerando as caractersticas da produo do caf no Brasil entre os sculos XIX e XX, CORRETO afirmar que

a) a produo de caf no Vale do Paraba foi a primeira no Brasil a introduzir novidades tcnico-tecnolgicas na
agricultura do pas, mas continuava utilizando a escravido como mo de obra.
b) a expanso da produo de caf para o Oeste paulista s foi possvel com a colaborao dos indgenas, j que eles
abriam caminhos, que posteriormente seriam transformados em ferrovias.
c) a expanso da cafeicultura no Brasil foi devastadora para o meio ambiente, pois, na preparao da terra para o
plantio, diversas rvores nativas foram derrubadas e queimadas.
d) os imigrantes trazidos para trabalhar nas fazendas de caf, no final do sculo XIX, foram a soluo encontrada pelo
governo brasileiro para resolver o problema da escassez de mo de obra nas fazendas de caf, causada pela abolio
da escravido no Brasil.

19 - (UFPA) Acerca da natureza e dinmica da economia exportadora brasileira durante o Imprio (1822-1889) e a
Primeira Repblica (1889-1930), correto dizer que

a) a borracha se tornou no somente o principal produto de exportao da regio amaznica, mas o segundo produto
brasileiro da pauta de exportaes, apenas atrs do caf, sendo a exportao da borracha uma importante
atividade no cenrio econmico brasileiro.
b) houve a hegemonia da produo aucareira; o acar de cana brasileiro foi beneficiado pela expanso de
mercados consumidores europeu e norte-americano, face ao aumento do consumo de cafs, chs e chocolates
nos pases desenvolvidos. O acar de beterraba de origem russa, entretanto, ocupava uma posio cada vez
mais secundria.
c) houve o declnio da economia lastreada na cafeicultura por conta do fim da escravido, uma vez que o trabalho
escravo havia sido o suporte da produo do caf, tanto que era comum se dizer que O Brasil era o caf, o caf
era o escravo; consequentemente o Brasil passou condio de importador do caf de origem africana.
d) no houve qualquer vnculo entre o processo de industrializao brasileiro e a economia agroexportadora, uma vez
que a indstria no Brasil surgiu do trabalho e de investimentos de imigrantes europeus recm-chegados e
instalados em centros urbanos. Os brasileiros vinculados economia agroexportadora mantiveram-se, assim,
afastados.
e) a economia da borracha, apesar de sua importncia na gerao de riquezas na regio amaznica, teve tal
importncia restrita ao norte do Brasil, uma vez que entre os principais produtos brasileiros exportados a borracha
ficava entre os ltimos, por isso mesmo tornou-se incapaz de gerar recursos para a nao como um todo.

309
20 - (UESPI) O caf era um produto bem aceito, no continente europeu, desde o sculo XVIII. No Brasil, seu cultivo
ganhou espaos importantes economicamente. Sua lavoura dispunha, no sculo XIX:

a) da participao de imigrantes europeus, mas contava apenas com a escassa presena da mo de obra escrava.
b) do uso de tcnicas agrcolas avanadas, produzidas para assegurar a qualidade do solo.
c) da forma de administrar, totalmente, diferente da existente na lavoura do acar.
d) de uma maior expanso em terras do Sudeste, com destaque para a economia de So Paulo.
e) da mo de obra escrava e da participao de alguns setores ligados aos colonizadores.

21 - (UEFS BA)
I.
O que um conservador?
aquele para quem a Histria uma carroa abandonada. Ele quer conservar a sociedade do jeito que a encontrou,
evitando mudanas. Isso lgico, a sociedade o favorece:
Se est boa para mim, deve estar para todo mundo pensa o conservador. [...]
No se pense que os conservadores achavam o Brasil uma maravilha! Eles reconheciam os nossos problemas e
julgavam saber os motivos: O brasileiro preguioso e atrasado. Uma raa ruim, resultado da mistura de negros e
ndios. Um povo mestio, inferior. Ai do pas se no fssemos ns, a elite!

II.
O que um liberal?
Como o nome diz, a pessoa que defende a liberdade. Na Histria da Europa, os liberais surgiram com a burguesia,
lutando contra as imposies do tempo do feudalismo e do absolutismo.
Os liberais brasileiros eram liberais-conservadores. Donos de fazendas de gado, de acar ou de caf como os
conservadores os liberais daqui no podiam agir da mesma maneira que os liberais europeus, que surgiram na
Histria como revolucionrios. Nem sempre a nossa aristocracia conseguiu imitar a velha Europa! (ALENCAR;
RIBEIRO; CECCON, 1986, p.137).

Na aristocrtica, desigual e racista sociedade brasileira, at o incio da dcada de 70 do sculo XIX, o exerccio da
plena cidadania era reservado aos

a) donos de propriedade rural, patente militar, origem europeia.
b) possuidores de renda mnima estabelecida por lei, nascimento livre e religio catlica.
c) habitantes das capitais das provncias, senhores de engenho, ex-escravos libertos.
d) homens adultos independente de origem, alfabetizados, seguidores de qualquer religio crist.
e) homens e mulheres com direito de voto, soldados de qualquer patente, moradores das reas urbanas.

22 - (UEM PR) Ao longo do Segundo Reinado brasileiro, ocorreram importantes mudanas no mbito das atividades
produtivas. A esse respeito, assinale a(s) alternativa(s) correta(s).

01. Consolidou-se o deslocamento do eixo econmico das regies de pecuria do nordeste para o norte.
02. No mbito das atividades agrcolas, ocorreu a estagnao das lavouras tradicionais de algodo e de fumo.
04. O avano da lavoura do caf no perodo deu-se em duas direes: at aproximadamente a dcada de 1870, o vale
do Ribeira foi a grande regio produtora; nas ltimas dcadas do sculo XX, os cafezais passaram a se expandir
pelo nordeste paulista e norte do Paran.
08. No incio do Segundo Reinado, a borracha, produzida a partir do ltex da Amaznia, tornou-se o principal item da
pauta de exportaes do Brasil.
16. No decorrer do perodo, ampliou-se gradativamente a utilizao do trabalhador livre, sobretudo, imigrante, nas
lavouras, em detrimento dos escravos africanos.

23 - (UFRRJ) Leia o texto a seguir.

Mau - Um Homem a frente do seu Tempo
Irineu Evangelista de Souza, nascido no Rio Grande do Sul, em 28 de dezembro de 1813, foi um homem de
personalidade forte, controvertido e irreverente. Trabalhando como balconista no Rio, onde chegou levado por um tio
em 1822, ele comeou a observar o atraso em que o Brasil vivia. E de l saiu para ajud-lo, tornando-se poderoso e
rico. No fim faliu. Segundo o historiador Heitor Ferreira, o Baro de Mau teve muitos problemas porque era um
adversrio social do regime. Mas, como homem de negcios, ele personificou uma das aspiraes de seu tempo: a
aspirao capitalista, que era ento o ideal mais avanado.
OLIVEIRA, Ivacy F.(org.). Heris de Verdade. Tatu-SP: Casa
Publicadora Brasileira, 2. ed. 1986. Disponvel em
www.guaruja1.hpg.ig.com.br/visconde_maua.htm. Acesso em:03/09/07.


310
Alm do esforo pessoal de Mau, apresente um outro fator que tenha contribudo para o desenvolvimento industrial
do Brasil entre as dcadas de 1840 e 1850.

24 - (UNIMONTES MG) Leia o texto.
Acompanhei com vivo interesse a soluo desse grave problema. Compreendi que o contrabando no podia
reerguerse, desde que a vontade nacional estava ao lado do ministrio que decretava a supresso de to infame
comrcio. Reunir os capitais que se viam repentinamente deslocados do ilcito comrcio e fazelos convergir a um
centro onde pudessem ir alimentar as foras produtivas do pas, foi o pensamento que me surgiu na mente, ao ter
certeza de que aquele fato era irrevogvel.(...)
(Irineu Evangelista de Souza (baro de Mau) Autobiografia. In: MATTOS, I.R. O imprio da Boa Sociedade.
So Paulo: Atual, 1991)

Acerca da afirmao de Irineu Evangelista de Souza, no 2 Reinado brasileiro, esclarea:
a) a que o autor se refere ao tratar da supresso do infame comrcio?
b) o que o autor quer dizer com a frase reunir os capitais (...) e fazlos convergir a um centro onde pudessem
alimentar as foras produtivas do pas?

25 - (UFG GO) Na dcada que segue a 1850, fundaram-se 62 empresas industriais, 14 bancos, 3 caixas
econmicas, 20 companhias de navegao a vapor, 23 companhias de seguro, 4 companhias de colonizao, 8 de
minerao, 3 de transporte urbano, 2 de gs e 8 estradas de ferro.
PRADO JR., Caio. Histria econmica do Brasil. So Paulo: Brasiliense, 1973. p. 192.

Considerando as informaes no fragmento citado, identifique as origens dos recursos que financiaram o processo de
crescimento econmico responsvel pela formao de um surto industrial no Brasil, na segunda metade do sculo XIX.


































311










PROF.: JEFT RODRIGUES DE BARROS






C
C
o
o
n
n
t
t
e
e

d
d
o
o
s
s
:
:

S
S
e
e
g
g
u
u
n
n
d
d
o
o
R
R
e
e
i
i
n
n
a
a
d
d
o
o
(
(
P
P
a
a
r
r
t
t
e
e
2
2
)
)
,
,
R
R
e
e
p
p

b
b
l
l
i
i
c
c
a
a
V
V
e
e
l
l
h
h
a
a
e
e

E
E
r
r
a
a
V
V
a
a
r
r
g
g
a
a
s
s
(
(
p
p
a
a
r
r
t
t
e
e
1
1
)
)





M
M
i
i
n
n
i
i
s
s
t
t
r
r
a
a
d
d
o
o
s
s
d
d
u
u
r
r
a
a
n
n
t
t
e
e
o
o
m
m

s
s
d
d
e
e
m
m
a
a
i
i
o
o














D Da at ta a d da a E En nt tr re eg ga a : : _ __ __ __ __ __ __ __ _/ /_ __ __ __ __ __ __ __ __ __ __ _/ /2 20 01 13 3
LISTA 4 HISTRIA

312



01 - (UERJ)

www.ibge.gov.br

Diversas experincias histricas da sociedade brasileira interferiram nas variaes dos fluxos imigratrios nos sculos
XIX e XX.
Para o perodo situado entre 1880 e 1899, a variao indicada no grfico associou-se ao seguinte fator:
a) expanso cafeeira
b) crise da monarquia
c) abolio da escravido
d) modernizao industrial

02 - (UNIFOR CE) Com relao escravido e abolio ao final do sculo XIX, assinale o trecho de texto que pode
ser considerado INCORRETO:
a) O primeiro golpe na escravatura foi a abolio do trfico negreiro por meio da Lei Eusbio de Queirs, de 1850.
Para os escravocratas, isso criou um problema de reposio da mo de obra proveniente da frica. Para o imprio
britnico, a abolio do trfico fecharia um de seus principais mercados.
b) A Lei Visconde do Rio Branco, tambm conhecida como a Lei do Ventre Livre foi decretada em 1871 e estabelecia
que a partir de 1871 todos os filhos de escravos seriam considerados livres.
c) A Lei dos Sexagenrio foi promulgada em 1885, estabelecendo que depois de completar 65 anos os escravos
estariam em liberdade. Essa lei foi bastante criticada sob a argumentao de que eram poucos os escravos que
chegariam a tal idade.
d) A introduo dos imigrantes europeus funcionou como uma fonte alternativa no fornecimento de fora de trabalho e
ocupou as brechas no preenchidas pelo trabalho escravo, que se tornava escasso e caro.
e) A Lei urea, que aboliu a escravido no Brasil, foi sancionada em 13 de maio de 1888, momento em que a
Princesa Isabel exercia a Regncia no lugar do pai. A Lei teve importncia ao libertar cerca de 700 mil escravos
que ainda havia no pas.

03 - (UECE) O perodo da histria brasileira conhecido como Imprio, 1822 a 1889, tem marcos cronolgicos bem
delimitados pela historiografia.
Acerca desse momento histrico, considere as afirmaes a seguir:
I. um momento de percalos e avanos para a construo de um estado nacional, bem como da procura de
soluo para as dificuldades em concretiz-lo.
II. Este perodo, num primeiro momento, constituiu a luta contra ou a favor da centralizao monrquica e,
posteriormente, assistiu-se ao crescimento das ideias republicanas.
III. As lutas abolicionistas e a progressiva desarticulao do sistema escravista contriburam para a crise desse
sistema de governo.

Est correto o que se afirma em

a) I, II e III.
b) II e III apenas.
c) I e II apenas.
d) III apenas.

Caderno de Atividades

Disciplina:
Histria

Professor(a):
Jeft
Aluno:
3 ano
Ensino Mdio
Data de Recebimento:
_____/_____/_____
Lista 04

Data Entrega:
_____/_____/_____

313
04 - (MACK SP) Este comrcio de carne humana , pois, um cancro que corri as entranhas do Brasil (...) Torno a
dizer, porm, que eu no desejo ver abolida de repente a escravido; tal acontecimento traria consigo grandes males.
Jos Bonifcio, 1823.

Como sabido, no Brasil, a abolio tardou, s se concretizando aps longa e dolorosa agonia (...). To longo e
socialmente penoso foi o processo de abolio que, aos contemporneos (...), parecia que no viria nunca.
Maria Helena Machado.

Teremos grandes desastres, se no houver providncias enrgicas e imediatas: a rebeldia dos escravos e a abolio
da escravido.
In: Keila Grinberg e Ricardo Salles (orgs.). O Brasil Imperial, v.III (1870-1889). So Paulo: Civilizao Brasileira, 2009,
p.369

Considerando os trechos acima, conclui-se, corretamente, que uma das explicaes para a tardia abolio da
escravido, no Brasil, deveu-se

a) ao carter gradualista que ela adquiriu, satisfazendo, em grande medida, aos anseios de uma parcela da elite,
preocupada com as possibilidades sociais, econmicas e polticas dos recm-egressos da escravido.
b) s tentativas de grupos abolicionistas em erradicar a escravido desde o incio do sculo XIX como, por exemplo, o
grupo liderado pelo poeta e advogado Lus Gama, em So Paulo, denominado Caifases.
c) ao medo, por parte da elite, de que os emancipados pudessem ascender econmica e politicamente, uma vez que,
desde 1871, era assegurado o direito educao e participao poltica a esses grupos.
d) constatao de que havia o medo, em potencial, de parcelas da elite em assumir seus anseios em prol da
abolio, uma vez que, por tradio, os grandes proprietrios eram retrgrados e desfavorveis a mudanas.
e) s discusses em torno do assunto no Conselho de Estado, demoradas e no conclusivas, que s fizeram adiar as
medidas efetivas em torno da emancipao gradual, como exposto no texto de Jos Bonifcio.

05 - (UFTM MG) Observe a caricatura de Angelo Agostini, publicada
na Revista Ilustrada, em 28 de julho de 1885.

A imagem faz referncia

a) s revoltas populares, que impediam D. Pedro II e seus ministros de
sarem s ruas.
b) s dificuldades econmicas do governo, em funo da Guerra do
Paraguai, ento em curso.
c) ao enfraquecimento da monarquia frente s crises polticas e ao
crescimento do abolicionismo.
d) ao temor do terceiro reinado, que levaria ao poder o marido da
Princesa Isabel, o Conde DEu.
e) aos republicanos, que pregavam o fim da monarquia e a libertao
incondicional dos escravos.

06 - (UFT TO) O ano de 1850 significou, de fato, um marco decisivo na
histria do Segundo Reinado. No poder desde 1848, estava um
Ministrio nitidamente conservador, Arajo Lima (Marqus de Olinda),
Euzbio de Queiroz, Paulino Jos Soares de Souza e Joaquim Jos
Rodrigues Torres. Esse Ministrio legislaria sobre questes
fundamentais: o problema da estrutura agrria, a questo da escravido
e o incentivo imigrao.

So medidas vinculadas a esse Ministrio:

a) A Lei de Terras, a abolio do trfico de escravos e a reforma da Guarda Nacional.
b) A abolio do trfico de escravos, a criao de corpos de voluntrios para combater na Guerra do Paraguai e a
reforma da Guarda Nacional.
c) A Lei do Sexagenrio, a criao de corpos de voluntrios para combater na Guerra do Paraguai e a abolio do
trfico de escravos.
d) A Lei de Terras, a Lei do Ventre Livre e a reforma da Guarda Nacional.
e) A Lei do Sexagenrio, a Lei de Terras e a Lei do Ventre Livre.





314
07 - (FGV) A Lei urea, de 13 de maio de 1888, marca o fim da escravido no Imprio brasileiro. A lei assinada pela
princesa Isabel foi precedida por diversos movimentos e resistncias de escravos em diversas partes do Brasil. Com
base nessa temtica, considere as seguintes afirmaes:

I. Lderes negros, como o advogado Lus da Gama e o jornalista Jos do Patrocnio, tiveram atuao destacada na
defesa do fim da escravido no Brasil.
II. Fugas em massa foram estimuladas pelos Caifazes, que encaminhavam ex-escravos para o quilombo do Jabaquara,
em So Paulo, e at para o Cear, onde a escravido j havia sido abolida.
III. A abolio implementada pela monarquia no previa medidas que preparassem os ex- escravos para o pleno
exerccio da cidadania, o que s viria a ser realizado pelos governos republicanos a partir de 1889.

Est correto somente o que se afirma em
a) I
b) II
c) III
d) I e II.
e) I , II e III.

08 - (UFG GO) Analise a imagem e leia os artigos da Lei n. 4897 a seguir.
.
Art. 1 Joaquim Jos da Silva Xavier, o Tiradentes, declarado patrono cvico da
Nao Brasileira.
Art. 2 As Foras Armadas, os estabelecimentos de ensino, as reparties
pblicas e de economia mista homenagearo a excelsa memria desse patrono,
nela inaugurando, com festividades, no prximo dia 21 de abril, efemride
comemorativa de seu holocausto, a efgie do glorioso republicano.
LEI N 4.897, de 9 de dezembro de 1965. Declara Joaquim Jos da
Silva Xavier, o Tiradentes, Patrono da Nao Brasileira. Disponvel em:
<http://www2.camara.gov.br/legin/fed/lei/1960-1969/lei-4897-9-dezembro
-1965-368995-publicacaooriginal-1-pl.html>. Acesso em: 27 ago. 2011.

As sucessivas representaes sobre Tiradentes exemplificam o fenmeno de
apropriao do passado, tal como se observa na pintura, elaborada no incio da
Repblica, e na lei, promulgada durante o regime militar. Essas apropriaes, em
suas pocas, objetivavam
a) referendar o carter religioso da Inconfidncia.
b) unir a sociedade contra os ideais estrangeiros.
c) justificar a ao inconfidente contra o governo.
d) enfatizar o sacrifcio individual em prol da nao.
e) destacar o carter violento da histria nacional.
MELO, Francisco Aurlio Figueiredo. Martrio de Tiradentes,
1893. leo sobre tela. Museu Histrico Nacional, Rio de Janeiro.
Disponvel em: <www.clubeduquedecaxias.com.br/index.php2.
sistem=agenda&action=read&id=380>. Acesso em: 17 ago. 2011

09 - (UFJF MG) Na edio de 09 de junho de 1888, a Revista Ilustrada publicou a charge abaixo. Nela, a Repblica,
representada por uma mulher, tendo a cabea ornada pelo
barrete frgio, tenta conter fazendeiros que trazem a bandeira
Abaixo a Monarquia abolicionista. Viva a Repblica com
indenizao.

Revista Ilustrada, Rio de Janeiro, 9 jun. 1888.
Com base na ilustrao acima e em seus conhecimentos,
responda ao que se pede:

a) Aponte a principal razo que motivou muitos fazendeiros
defenderem, aps a abolio da escravido, em 13 de
maio de 1888, a substituio do regime monrquico pelo
republicano no Brasil.
b) Apesar da alterao representada pelo fim da monarquia em 1889, a implantao do regime republicano no
significou transformaes profundas para a sociedade brasileira. Aponte dois argumentos que confirmem esta
afirmativa.




315
10 - (UECE) Tomando o texto abaixo como referncia, assinale a alternativa correta:

Em 1887, a monarquia agonizava. Qualquer circunstncia contribua para enfraquec-la. At mesmo certo segmento
que sempre apoiara o Imperador passara para a oposio. Eram os republicanos de 14 de maio, expresso utilizada
por Jos do Patrocnio...
QUEIROZ, Suely Robles de. Poltica e Cultura no Imprio
Brasileiro. So Paulo: Brasiliense, 2010.

Nas palavras irnicas de Jos do Patrocnio, os Republicanos de 14 de maio seria(m)

a) os abolicionistas, que clamavam por medidas que desarticulassem o sistema escravagista.
b) a jovem oficialidade, crtica da monarquia e imbuda dos ideais positivistas e republicanos.
c) as antigas bases oligrquicas e escravocratas da monarquia que, neste momento, culpavam-na por seus prejuzos.
d) a alta oficialidade do exrcito brasileiro, que historicamente se caracterizou por se colocar como oposio
monarquia.

11 - (UEM PR) Sobre o movimento republicano no Brasil durante o Segundo Reinado, assinale a(s) alternativa(s)
correta(s).

01. Do movimento republicano participaram somente lideranas civis, como Benjamin Constant e Deodoro da
Fonseca.
02. As ideias republicanas se fortaleceram a partir de 1880 com as presses diplomticas da Argentina e dos Estados
Unidos a favor do regime republicano.
04. A chamada questo militar inciou-se como um mero problema disciplinar, porm mostrava o descontentamento
de setores do Exrcito com o governo imperial.
08. A filosofia positivista exerceu forte influncia no movimento republicano. O lema Ordem e Progresso inscrito na
bandeira brasileira expressa essa influncia.
16. O movimento republicano ganhou fora na dcada de 1850, com a fundao de partidos republicanos em todas as
provncias do Pas.

12 - (MACK SP) Rui Barbosa, quando assumiu a funo de ministro da
Fazenda durante o governo provisrio do Marechal Deodoro da
Fonseca (1889-1891), pretendeu garantir a independncia econmica
do Brasil frente ao capitalismo europeu. Para ele, a Repblica somente
se consolidaria ... sobre alicerces seguros quando suas funes se
firmarem na democracia do trabalho industrial. Sua poltica financeira,
contudo, no foi bem sucedida, como mostra a charge dada, devido

a) emisso de papel-moeda em larga escala para incentivar o crdito
para investidores do setor industrial, o que gerou uma poltica
inflacionria, visto que o aumento do meio circulante no foi
acompanhado pela elevao da produo interna.
b) restrio de crdito para financiamento de novas empresas, alm de cortes no gasto pblico e aumento dos
impostos, o que gerou diversas manifestaes, principalmente no meio do operariado nacional, prejudicado pelo
aumento no custo de vida.
c) adoo de tarifas alfandegrias protecionistas e estmulo s indstrias nacionais visando a aumentar a produo
nacional, porm congelou os salrios dos trabalhadores e aumentou os gastos na construo de obras pblicas.
d) realizao de uma poltica financeira anti-inflacionria que buscou equilibrar nossa economia frente aos prejuzos
herdados do perodo monrquico, graas aos vultosos emprstimos externos, realizados para sanar o dficit
oramentrio.
e) especulao financeira graas facilidade de crditos concedidos pelo governo, que ao invs de contribuir para a
instalao de novas indstrias no pas, foram utilizados para saldar as dvidas dos cafeicultores perante os banqueiros
estrangeiros.

13 - (UFTM MG) Sobre o processo eleitoral na 1. Repblica, correto afirmar que

a) os resultados dependiam dos interesses do partido majoritrio no Congresso Nacional.
b) as mulheres exerciam o direito de voto apenas nas cidades, graas ao das feministas.
c) polticos inescrupulosos aprovaram lei que permitiu o voto dos eleitores analfabetos.
d) vigorou um sistema fraudulento, que inclua a adulterao de atas eleitorais.
e) a adoo do voto secreto, em meados da dcada de 1920, resultou da presso popular.




316
14 - (PUC RS) O caf foi o principal produto de exportao brasileiro, desde meados de 1890 at a dcada de 1930.
Mas esta produo no esteve isenta de crises, como a ocorrida ao final do sculo XIX, devido ao excesso de
produo mundial e consequente queda nos preos. Como medida para combater a crise no perodo, destaca-se

a) o lanamento do II Plano Nacional de Desenvolvimento (PND), que iniciou o processo de abertura poltica e
garantiu, em curto prazo, o declnio do poder dos senhores de engenho, que ofereciam resistncia ao crescimento
da produo cafeeira.
b) a poltica de imigrao de mo de obra europeia, principalmente alemes, italianos e poloneses, que passam a ser
empregados em regime escravista nas fazendas de caf do interior paulista.
c) a poltica de incentivo criao de rodovias e novas fbricas, que pudessem incrementar o escoamento e
processamento da grande safra de caf brasileira, assim como a abertura de mercado para obteno de
financiamentos de investidores franceses.
d) o Convnio de Taubat, em 1906, um plano de interveno do estado, mediante a garantia de compra pelos
governos (So Paulo, Rio de Janeiro e Minas Gerais), criando estoques reguladores, promovendo a falta do
produto no mercado, com o objetivo de elevar os preos.
e) a chamada socializao das perdas, pela qual os lucros so distribudos entre a populao, e as perdas ficam
destinadas elite cafeeira e aos governos estaduais, que recorrem a emprstimos estrangeiros e queimam os
estoques excedentes do produto.

15 - (MACK SP) A linha de fora que conduziu os diversos estudos sobre a histria do So Paulo oitocentista foi o
desejo de explicar o notvel crescimento do seu ncleo urbano. Como se sabe, na segunda metade do sculo XIX, a
capital da provncia passou de 11 maior aglomerao urbana do Brasil, em 1872, para a segunda em 1920, perdendo
apenas para a capital do pas. A grande questo era entender como e por que a cidade atingiu to rapidamente tal
posio.
Maria Luiza Ferreira de Oliveira, Uma senhora na rua do Imperador:
populao e transformaes urbanas na cidade de So Paulo

Dentre as alternativas abaixo, assinale aquela que apresenta uma resposta satisfatria indagao do texto.

a) Apesar de sofrer investimentos advindos dos cafezais, So Paulo se beneficiou, principalmente, da produo
aucareira.
b) Desde sua fundao, no sculo XVI, So Paulo despontou como centro econmico do Brasil.
c) A cidade de So Paulo se beneficiou de investimentos realizados por diversos segmentos, dentre eles, o setor
cafeeiro.
d) A cidade s iria se desenvolver realmente com a industrializao, na segunda metade do sculo XX.
e) Diversos fatores explicam as transformaes vividas por So Paulo, tais como a cafeicultura, a industrializao e a
explorao das drogas do serto.

16 - (UESC BA)


A charge retrata uma realidade adequada ao processo de formao

01. das reservas indgenas, na fase de delimitao territorial dos Estados Unidos.
02. dos latifndios do cacau, no Brasil, durante a vigncia do coronelismo.
03. dos quilombos, durante a colonizao brasileira.
04. do Estado Absolutista, na Europa moderna.
05. dos feudos, no perodo medieval.






317
17 - (FATEC SP) Entre as principais caractersticas do modelo poltico adotado no Brasil durante a Repblica Velha
(1889-1930), destacaram-se

a) a poltica do Regresso Conservador, o militarismo e o voto censitrio.
b) a poltica dos governadores, o coronelismo e o voto de cabresto.
c) o parlamentarismo s avessas, o clientelismo e o voto a descoberto.
d) a poltica do caf com leite, o coronelismo e o voto secreto censitrio.
e) a poltica de valorizao do caf, o populismo e o voto universal.

18 - (UERJ) Observe a foto do grupo de Lampio e Maria Bonita e o mapa que destaca a rea do Nordeste brasileiro
onde o cangao se disseminou nas dcadas de 1920 e 1930.


http://www1.folha.uol.com.br


http://pt.wikipedia.org

O cangao representou uma manifestao popular favorecida, basicamente, pela seguinte caracterstica da conjuntura
social e poltica da poca:

a) cidadania restringida pelo voto censitrio
b) analfabetismo predominante nas reas rurais
c) criminalidade oriunda das taxas de desemprego
d) hierarquizao derivada da concentrao fundiria

19 - (Unemat MT) O movimento social surgido em 1911 [...]. nasceu reunindo seguidores de um coronel tido como
amigo dos pobres e pessoas de diversas origens, atingidas pelas mudanas que vinham ocorrendo na rea. Entre elas,
trabalhadores rurais expulsos da terra pela construo de uma ferrovia e por uma empresa madeireira e gente que
tinha sido recrutada na construo da ferrovia, ficando novamente desempregada no fim de seus contratos. Os
rebeldes se agruparam em torno de Jos Maria, uma figura que morreu nos primeiros choques com a milcia estadual e
foi santificada.
(FAUSTO, Boris. Histria do Brasil. 8. ed. So Paulo:
Editora da Universidade de So Paulo, 2000.p.296).

Sobre qual movimento social o texto faz referncia?
a) Contestado
b) Canudos
c) Revolta da Chibata
d) Juazeiro
e) Cabanagem

318
20 - (UECE) Rio de Janeiro: novembro de 1904. A divulgao do projeto de regulamentao da lei que tornara
obrigatria a vacinao antivarilica transforma a cidade em praa de guerra.
CHALHOUB, Sidney. Cidade Febril: cortios e epidemias na corte imperial.
So Paulo: Companhia das Letras, 1996.

Tomando por base o excerto acima, referente ao movimento que ficou conhecido na historiografia como Revolta da
Vacina, analise as afirmaes a seguir e assinale a opo correta:

I. Significou um raro momento em que setores sociais, com diversos interesses e insatisfaes vrias, provocaram um
protesto violento, em que centenas de pessoas saram s ruas e enfrentaram as foras da polcia, do exrcito, do
corpo de bombeiros e da marinha.
II. O saldo do confronto incluiu vrios mortos, dezenas de feridos e centenas de presos, sendo que, muitos desses
ltimos ficaram retidos na Ilha das Cobras para, em seguida, serem encaminhados a uma viagem s de ida para o
Acre.

a) I verdadeira e II falsa.
b) Ambas so falsas.
c) I falsa e II verdadeira.
d) Ambas so verdadeiras.

21 - (UEM PR) Sobre a guerra do contestado, assinale a(s) alternativa(s) correta(s).

01. A guerra do contestado inspirou Euclides da Cunha na elaborao do livro Os sertes, em que narrada a dura
vida dos sertanejos brasileiros.
02. Os conflitos do contestado relacionam-se disputa pela posse de terras na regio, entre fazendeiros e posseiros,
que tentavam manter-se nas terras devolutas, e aos interesses da Brazil Railway Company.
04. O monge Antonio Conselheiro liderou as lutas dos camponeses da regio contra as tropas do Governo Federal e
do Estado do Paran.
08. A guerra do contestado relaciona-se s disputas de limites territoriais entre o Estado do Paran e Estado de Santa
Catarina.
16. Como resultado da guerra do contestado, os territrios que formavam o Estado do Iguau foram incorporados ao
Estado de Santa Catarina.

22 - (FUVEST SP) O conceito de revoluo, aplicado ao movimento de 1930 no Brasil, alvo de polmica entre
historiadores. Independentemente da controvrsia, no h como negar que houve mudanas importantes, nessa
dcada, com relao s diretrizes da poltica econmica e questo social.
Explique as mudanas no que se refere

a) poltica econmica.
b) questo social.

23 - (UFRN) Chefe vitorioso do propalado movimento revolucionrio de 1930, Getlio Dorneles Vargas instalou-se no
Palcio do Catete, no Rio de Janeiro, onde iniciou seu governo, auxiliado por um ministrio que procurava amparar-se
em todas as foras favorveis Revoluo. As medidas iniciais visavam combater as prticas polticas do regime que
se encerrava. O contexto era conturbado. Foi assim que Juarez Tvora, delegado do movimento revolucionrio para a
regio Norte/Nordeste, veio a Natal para reunir-se com a Junta Militar.

a) Analise duas medidas tomadas por Vargas para combater as prticas polticas da Repblica Velha.
b) Cite duas repercusses, no Rio Grande do Norte, do movimento revolucionrio referido.

24 - (UFOP MG) A poltica de salvao, adotada na Repblica Velha, durante a Presidncia de Hermes da Fonseca
(1910-1914), caracterizou-se:

a) pela manuteno dos preos do caf a partir da compra do produto pelo Governo Brasileiro.
b) pelo processo de intervenes militares em vrios estados brasileiros derrubando as oligarquias locais e
estabelecendo a centralizao poltica.
c) pelo desenvolvimento da indstria de base a fim de reverter a crise do mercado interno desencadeada pelo crash da
Bolsa de Nova Iorque.
d) por sucessivas marchas, lideradas por mulheres das classes mdias, com o objetivo de deter o avano do
comunismo no Brasil.





319
25 - (UFG GO/2012)
Analise a charge.


Charge de Belmonte. Disponvel em: <www.portaldoprofessor.mec.gov/
fichaTecnicaAula.html?aula=13104>. Acesso em: 3 out. 2011. [Adaptado].

Esta charge de autoria do caricaturista brasileiro Belmonte. Com o propsito de disseminar a crtica poltica, o artista
criou a personagem Juca Pato. A composio da charge remete s contradies da vida poltica nacional durante a Era
Vargas.

Considerando o exposto, explique

a) como a abordagem do tema, na charge, ironiza a vida poltica brasileira;
b) a contradio vigente na relao entre poltica interna e externa durante o regime do Estado Novo (1937-1945).


320







PROF.: JEFT RODRIGUES DE BARROS






C
C
o
o
n
n
t
t
e
e

d
d
o
o
s
s
:
:

E
E
r
r
a
a
V
V
a
a
r
r
g
g
a
a
s
s
(
(
c
c
o
o
n
n
t
t
i
i
n
n
u
u
a
a

o
o
)
)
,
,
E
E
r
r
a
a
d
d
o
o
P
P
o
o
p
p
u
u
l
l
i
i
s
s
m
m
o
o





M
M
i
i
n
n
i
i
s
s
t
t
r
r
a
a
d
d
o
o
s
s
d
d
u
u
r
r
a
a
n
n
t
t
e
e
o
o
m
m

s
s
d
d
e
e
s
s
e
e
t
t
e
e
m
m
b
b
r
r
o
o















D Da at ta a d da a E En nt tr re eg ga a : : _ __ __ __ __ __ __ __ _/ /_ __ __ __ __ __ __ __ __ __ __ _/ /2 20 01 13 3
LISTA 5 HISTRIA

321



01 - (UFBA) Tendo como foco o carter dependente da economia brasileira, resultante do seu processo histrico,
apresente uma informao relativa a essa dependncia:
a) Na Repblica Velha (1889-1930);
b) No Perodo Desenvolvimentista (dcada de 1950).

02 - (UFTM MG) Entre os motivos alegados por Getlio Vargas para decretar o Estado Novo, em novembro de 1937,
pode-se citar
a) a iminncia do incio da 2 Guerra Mundial e a necessidade de proteger as nossas fronteiras.
b) as greves operrias, os saques e as depredaes que tomaram conta do pas no perodo.
c) a descoberta de uma suposta insurreio comunista, o chamado Plano Cohen.
d) as denncias de fraudes no processo de escolha do seu sucessor, publicadas pela imprensa.
e) a insatisfao da elite paulista com o regime, que ameaava separar-se do restante do pas.

03 - (UEM PR) Sobre o golpe de 1937 e o Estado Novo, assinale a(s) alternativa(s) correta(s).
01. O golpe de 1937, como outros que ocorreram na Europa na mesma dcada, representou a vitria de um partido
organizado e teve o apoio ativo das massas populares.
02. Um dos argumentos utilizados por Getlio Vargas para dar o golpe foi o de que pairava uma ameaa comunista
sobre a sociedade brasileira.
04. A nova Carta, outorgada em 1937, caracterizou-se pelo predomnio do poder executivo e de seu sistema de partido
nico, e representada pela base poltica da nova ordem, a Ao Integralista Brasileira.
08. As reformas administrativas que vieram aps o golpe, ao modernizarem a burocracia e introduzirem instrumentos
de controle por parte do Estado, eliminaram o federalismo da Repblica Velha.
16. Um impulso ao desenvolvimento industrial brasileiro nesse perodo foi proporcionado pela criao de indstrias de
base estatais.

04 - (UERJ) O personagem Jeca Tatu, criado por Monteiro Lobato, tornou-se mais conhecido na dcada de 1930, por
meio de anncios publicitrios, como o ilustrado abaixo:
Adaptado de www.miniweb.com.br

Esse anncio retratava aspectos da sociedade brasileira
da poca, expressando crticas principalmente s
condies de:
a) acesso escolarizao
b) assistncia mdico-hospitalar
c) salubridade nas reas rurais
d) integrao econmica regional

05 - (UERJ) In: GONALVES, Adelaide e COSTA,
Pedro E. B. (orgs.).
Mais borracha para a vitria. Braslia: Ideal Grfica,
2008.

No governo Vargas, foi criado o Servio Especial de Mobilizao de Trabalhadores
para a Amaznia - S.E.M.T.A., uma medida direcionada para a participao do
Brasil na Segunda Guerra Mundial (1939-1945).
Com base no cartaz, as aes programadas por esse servio tiveram como principal
objetivo:
a) ocupao militar relacionada redefinio das fronteiras nacionais
b) proteo dos trabalhadores rurais em resposta depresso econmica
c) estmulo migrao para explorao de recursos naturais estratgicos
d) demarcao de reservas florestais associada poltica de defesa ambiental



Caderno de Atividades

Disciplina:
Histria

Professor(a):
Jeft
Aluno:
3 ano
Ensino Mdio
Data de Recebimento:
_____/_____/_____
Lista 05

Data Entrega:
_____/_____/_____

322
06 - (UFF RJ) No perodo de 1946 a 1964, assistimos ao pleno desenvolvimento do pacto populista, que no pode ser
identificado apenas como manipulao das massas trabalhadoras. O funcionamento do regime nesse perodo
pressupe elementos de continuidade do perodo estado novista e a criao de novos mecanismos de dominao.

a) Identifique dois elementos de continuidade do perodo de 1946-1964 em relao ao perodo de Estado Novo, 1937-
1945.
b) Indique os trs maiores partidos polticos da Repblica brasileira de 1946 at o Golpe civil-militar de 1964, e analise
uma caracterstica de cada um dos trs partidos.

07 - (MACK SP) O governo de Eurico Gaspar Dutra (1946-1950) foi influenciado pelos acontecimentos internacionais
que marcaram o ps-guerra. A poltica econmica adotada em seu governo tinha como principal objetivo

a) o aumento da interveno do Estado, que passou a controlar as importaes, diminuindo as tarifas alfandegrias.
b) a manuteno de uma poltica de confisco para combater a inflao que, entretanto, no prejudicou os ajustes
salariais dos trabalhadores.
c) a liberalizao do cmbio, aumentando as importaes de produtos suprfluos, sem adotar uma poltica de seleo
nas importaes.
d) a adoo de uma poltica liberal e nacionalista, favorvel aos negcios das empresas nacionais.
e) a manuteno das condies favorveis acumulao de capital, por meio de uma poltica social democrtica e
nacionalista.

08 - (FGV) Do ponto de vista da poltica econmica, o governo Dutra se iniciou seguindo um modelo liberal. A
interveno estatal foi condenada [e] passou-se a acreditar que o desenvolvimento econmico do pas e o fim da
inflao gerada nos ltimos anos da guerra dependiam da liberdade dos mercados em geral e principalmente da livre
importao de bens. [...] em junho de 1947, o governo mudou de orientao, estabelecendo um sistema de licenas
para importar. Na prtica, o critrio de licenas favorecia a importao de itens essenciais, como equipamentos,
maquinaria e combustveis, e restringiu a importao de bens de consumo.
Boris, Fausto. Histria do Brasil. So Paulo: Edusp, 1995, p.403.
De acordo com a leitura do texto, podemos concluir que:

a) Dutra iniciou seu governo com uma orientao econmica intervencionista, mas, a partir de 1947, adotou uma
poltica liberal, que proibia as importaes de bens de capital e de consumo.
b) houve uma mudana na orientao econmica do governo Dutra a partir de 1947, no sentido de permitir a
importao de bens de capital e restringir a entrada de bens de consumo.
c) o governo Dutra, ainda sob o impacto do Estado Novo getulista, adotou uma poltica econmica intervencionista que
se acentuou, a partir de 1947, com medidas restritivas importao de bens de capital no Brasil.
d) a orientao liberal da poltica econmica do governo Dutra acarretou a desindustrializao do pas, processo
intensificado com a poltica intervencionista adotada a partir de 1947.
e) a adoo de uma poltica econmica liberal foi a plataforma pela qual a UDN conseguiu chegar ao poder, com Dutra.
A mudana para uma linha intervencionista marcou a reaproximao de Dutra com Getlio e o rompimento com a
UDN.

09 - (UFPE) Modernizar fazia parte dos projetos do governo Juscelino Kubitschek. Braslia tornou-se um smbolo da
sua gesto pelas renovaes urbanas que consolidou. Alm disso, no governo de JK houve:

00. uma preocupao com o controle dos ndices inflacionrios que devastavam o equilbrio da economia brasileira
desde o governo de Caf Filho.
01. uma grande exaltao ao desenvolvimentismo, com ambies de firmar os caminhos de uma industrializao mais
ampla.
02. um forte esquema de controle da impressa, devido s crticas constantes feitas pelos seus adversrios aos gastos
administrativos.
03. uma preocupao com questes relacionadas com a situao do Nordeste, procurando diminuir seus problemas
sociais.
04. um desprezo crescente pelas prticas polticas do populismo, enfatizando os xitos da burocracia e dos
planejamentos tcnicos.










323
10 - (PUC RJ)

Posio de Braslia no Planalto Central brasileiro e as distncias a que se
acha das capitais e territrios. Revista Braslia, jan. 1957.

Desenvolvimento e integrao nacional foram objetivos de vrios governos republicanos brasileiros. O presidente
Juscelino Kubitschek (1956-1960) considerava a construo de Braslia um dos objetivos do seu projeto de governo.

a) Qual a importncia da construo de Braslia para as aes desenvolvimentistas propostas no Plano de Metas.
b) Assim como JK, os governos militares (1964-1985) tambm buscaram implementar aes para a integrao
nacional. Identifique duas dessas aes utilizando a concepo de integrao presente na imagem apresentada.

11 - (UERJ) Deste Planalto Central, desta solido que em breve se transformar em crebro das altas decises
nacionais, lano os olhos mais uma vez sobre o amanh do meu pas e antevejo esta alvorada com f inquebrantvel e
uma confiana sem limites no seu grande destino.
Juscelino Kubitschek, 02/10/1956 O Globo, 21/04/2010

A realizao mais conhecida do governo de Juscelino Kubitschek foi a construo de Braslia. No entanto, essa obra
contemplava objetivos mais abrangentes desse governante.
Dentre esses objetivos, destaca-se o de promover a integrao nacional por meio da seguinte ao:

a) modernizao do setor tercirio
b) ampliao da infraestrutura de transportes
c) interligao das redes de telecomunicaes
d) explorao das regies Nordeste e Centro-Oeste

12 - (UEG GO) O governo de Juscelino Kubitschek foi marcado pelas ideias de desenvolvimento e planejamento. O
chamado Plano de Metas tinha como slogan 50 anos em 5. Destaque os trs principais pontos desse plano e suas
consequncias para o desenvolvimento socioeconmico brasileiro.

13 - (UNESP SP) A construo de Braslia durante o governo Juscelino Kubitschek (1956-1961) teve, entre suas
motivaes oficiais,

a) afastar de So Paulo a sede do governo federal, impedindo que a elite cafeicultora continuasse a control-lo.
b) estimular a ocupao do interior do pas, evitando a concentrao das atividades econmicas em reas litorneas.
c) deslocar o funcionalismo pblico do Rio de Janeiro, permitindo que a cidade tivesse mais espaos para acolher os
turistas.
d) tornar a nova capital um importante centro fabril, reunindo a futura indstria de base do Brasil.
e) reordenar o aparato militar brasileiro, expandindo suas reas de atuao at as fronteiras dos pases vizinhos.

14 - (FEPECS DF) Bossa Nova mesmo ser presidente
Desta terra descoberta por Cabral
Para tanto basta ser, to simplesmente,
Simptico, risonho, original.
(Juca Chaves. Presidente Bossa Nova.)
O governo JK (1956/1960) ficou marcado pelo forte desenvolvimento econmico do pas atravs do Plano de Metas
que, entre outras realizaes, permitiu a construo da nova capital, Braslia, inaugurada no dia 21 de abril de 1960. O
presidente JK, conhecido como presidente Bossa Nova, gerou uma euforia interna durante seu governo, permitindo
que esse perodo fosse identificado como os Anos Dourados do Brasil. Uma caracterstica de destaque dos Anos
Dourados de JK foi:


324
a) o avano somente na rea econmica, deixando o pas ainda preso a prticas polticas autoritrias;
b) o desenvolvimento da poltica nacional com o livre exerccio democrtico, porm, sem a consolidao econmica
do pas;
c) o pleno desenvolvimento da economia, triunfos esportivos, reconhecimento cultural e a estabilidade poltica do
pas;
d) a acelerao da economia, mas sem o avano das reas de cultura e esportes em funo de baixos investimentos;
e) o pleno desenvolvimento da poltica, devido liberdade da imprensa, mas sem a consolidao cultural do pas no
plano externo.

15 - (FGV) A eleio de Jnio Quadros, em 1960, significou certa alterao de rumos da poltica brasileira com
relao ao perodo iniciado em 1945. Tal alterao baseou-se:

a) No apoio que os comunistas emprestaram candidatura de Jnio em troca da legalizao do PCB, que ocorreria
em 1961.
b) Na primeira vitria das foras trabalhistas em pleitos nacionais e no fortalecimento de novas lideranas sindicais.
c) No rompimento da hegemonia paulista e no descontentamento militar provocado pelas propostas eleitorais
janistas.
d) Na vitria de uma candidatura da UDN, que interrompeu a srie de vitrias do PSD e do PTB, em arranjo poltico
orquestrado por Getlio Vargas.
e) Na inaugurao de um novo estilo poltico baseado na valorizao das estruturas partidrias e na definio clara
de propostas polticas programticas.

16 - (UFPE) No Brasil, as campanhas eleitorais agitam a populao e, muitas vezes, surgem candidatos que recebem
votaes consideradas espetaculares. Foi o caso do sucessor de Juscelino, Jnio Quadros, vencedor com uma
vantagem bastante significativa. No seu governo, Jnio Quadros:

00. fez grandes reformas na agricultura, mantendo o poder de exportao de sua bem organizada produo de caf.
01. renunciou o mandato, com pouco tempo de exerccio, pressionado pela ao da UDN e das foras militares.
02. fechou o Congresso Nacional e abriu espao para a fundao da SUDENE, responsvel pelo crescimento do
Nordeste.
03. tomou medidas coerentes com seu liberalismo conservador, mas se manteve no poder com popularidade sempre
crescente.
04. seguiu comportamentos populistas, porm renunciou ao poder sem deixar claro quais os motivos do seu ato.

17 - (IBMEC RJ) Varre, varre, varre, vassourinha era a introduo do jingle da campanha presidencial de Jnio
Quadros, um compromisso pblico no sentido de varrer do pas:

a) as corridas noturnas de cavalo;
b) a corrupo;
c) todas as formas de jogo;
d) os biqunis, por sua imoralidade;
e) todos os que foram favorveis criao da Petrobras.

18 - (MACK SP) Um dos episdios mais controvertidos e menos elucidados da Histria do Brasil foi a renncia do
presidente Jnio Quadros, em 1961, que completa cinquenta anos. A esse respeito, considere I, II, III e IV abaixo.

I. O vice- presidente Joo Goulart, considerado, por setores da elite, propenso ao comunismo, foi impedido de
assumir o governo logo aps a renncia de Jnio.
II. A Campanha da Legalidade, liderada por Leonel Brizola, defendeu o cumprimento da Constituio federal, exigindo
a posse de Jango na presidncia.
III. Para diminuir os conflitos resultantes da renncia de Jnio, o Congresso instituiu o parlamentarismo, sendo, assim,
garantida a posse de Jango, mas com poderes sensivelmente diminudos.
IV. Resultou em uma crise poltica, uma vez que setores poderosos faziam oposio a Jango, acusado de comunismo,
sendo esse fato uma das justificativas para o Golpe militar em 1964.

Ento,

a) somente I, III e IV esto corretas.
b) somente I, II e III esto corretas.
c) somente II e III esto corretas.
d) somente II e IV esto corretas.
e) I, II, III e IV esto corretas.



325
19 - (ESPM) A questo dominou completamente o cenrio poltico nacional, numa gravssima conjuntura conspiratria
e golpista. No Rio Grande do Sul, sob o comando de Leonel Brizola, formou-se a Rede da Legalidade, que conseguiu
polarizar todos os segmentos tradicionais da poltica gacha em defesa de Joo Goulart.
(Edgard Luiz de Barros. O Brasil de 1945 a 1964)

O texto trata da Rede da Legalidade, que:
a) ofereceu apoio ao presidente Joo Goulart para enfrentar o golpe de 1964.
b) foi formada para apoiar as reformas de base propostas pelo presidente Goulart.
c) foi formada para garantir a existncia da Frente Ampla articulada por Leonel Brizola.
d) foi formada por Leonel Brizola para garantir a posse do vice-presidente Joo Goulart aps a renncia de Jnio
Quadros.
e) foi formada para apoiar a execuo do plano trienal dos economistas Celso Furtado e San Thiago Dantas.

20 - (UFU MG) Sobre o governo de Joo Goulart (1963-1964), correto afirmar:

a) Goulart procurou implementar todas as reformas de base, como a reforma agrria, a reforma urbana e a maior
interveno do Estado na economia, sendo impedido pelo golpe militar de 1964.
b) Goulart realizou acordos multilaterais com pases europeus e os Estados Unidos para a criao de filiais das
principais empresas automobilsticas do mundo.
c) Goulart tinha amplo apoio do empresariado nacional, pois possua ideias arrojadas para a poca, como fazer as
reformas de base, que aumentariam os lucros das empresas sediadas no Brasil.
d) A Marcha da Famlia com Deus pela Liberdade, realizada em 1964, foi uma manifestao de homenagem a Joo
Goulart em defesa de seu governo e contra as ameaas dos militares.

21 - (PUC RS) Em 25 de agosto de 1961, Jnio Quadros renunciou ao seu cargo de Presidente da Repblica,
alegando presso de foras terrveis, sem revelar seus verdadeiros motivos. Um movimento popular foi
desencadeado em seguida, comandado por Leonel Brizola, governador do Rio Grande do Sul, para garantir a
obedincia Constituio e a nomeao do vice-presidente da Repblica, Joo Goulart, que estava em viagem
diplomtica Repblica Popular da China.

Quem assumiu o poder, aps a Campanha da Legalidade?

a) Jnio Quadros, que retoma seu cargo de presidente, aclamado pelo povo.
b) Leonel Brizola, que ganha amplo poder popular, aps a Campanha.
c) Jango, que passa condio de chefe-de-estado em um regime parlamentarista.
d) Humberto Castelo Branco, que foi chamado a assumir, levando os militares ao poder.
e) Ulisses Guimares, que assume de forma interina, at a eleio seguinte.

22 - (FGV) A charge mostra o presidente Jango

(Augusto Bandeira, O Correio da Manh, 10.07.1962, apud Jayme Brener,
Jornal do Sculo XX, p. 226).

a) interessado no restabelecimento do presidencialismo, pois a sua posse, em 1961, s foi garantida com a instituio
do parlamentarismo.
b) defendendo a manuteno do sistema parlamentarista, posio contrria de Carlos Lacerda (UDN) e de Adhemar de
Barros (PSP).
c) responsabilizado pela radicalizao poltica do governo em funo da opo pelo sistema parlamentarista e pela
reforma agrria.
d) recebendo apoio das Ligas Camponesas para defender a aprovao de uma emenda constitucional que
estabeleceria a reforma agrria.
e) alheio grave crise poltica gerada pela institucionalizao do parlamentarismo e preocupado com as eleies
presidenciais de 1965.



326
23 - (UFSCAR SP) A partir de 1964, com o golpe militar que deps Joo Goulart, o novo governo do Brasil fez
interveno nos sindicatos, destituindo lderes, prendendo militantes e dando incio a um processo de controle rigoroso
no movimento sindical.
Contudo, o movimento operrio foi importante no processo de redemocratizao do pas.

a) Explique como os operrios participaram do processo de luta contra o regime militar.
b) Diante da poltica neoliberal da dcada de 1990, quais os novos desafios que os operrios, junto com os demais
trabalhadores brasileiros, passaram a enfrentar?

24 (MACK SP)
J vou embora/ Mas sei que vou voltar
Amor no chora/ se eu volto pra ficar
Amor no chora/ Que a hora de deixar
O amor de agora/ Pra sempre ele ficar
Eu quis ficar aqui/ Mas no podia
O meu caminho a ti/ No conduzia
Um rei mal coroado/ No queria
O amor em seu reinado/ Pois sabia/ No ia
[ser amado/
Amor no chora/ Eu volto um dia/
O rei velho e cansado/ J morria
Perdido em seu reinado/ Sem Maria
Quando me despedia/ No meu canto lhe dizia.
Geraldo Vandr, Cano da despedida

Os versos acima so de uma cano que, assim como outras do compositor, foram alvo da censura oficial da poca,
por sua crtica mais ou menos velada ao regime poltico ento vigente.
Considere as trs proposies, a seguir, a respeito desse perodo da histria brasileira.
I. A represso poltica exercida pelo governo voltou-se violentamente para o meio artstico- cultural do pas, como o
teatro e a msica popular, submetendo-o censura e, alm disso, obrigando muitos intelectuais e artistas a optar
pelo exlio.
II. O regime ditatorial revelava uma grande contradio interna, na medida em que combinava a mais severa
perseguio poltica aos opositores com uma clara normalidade democrtica, visvel na existncia livre dos vrios
partidos polticos, na regularidade de eleies diretas para presidente, e na liberdade de ao sindical.
III. A decretao de atos institucionais foi uma prtica recorrente do Executivo com o intuito de manter o controle
sobre a situao poltica, pois permitiu ao presidente, entre outras coisas, fechar o Congresso Nacional e cassar
direitos polticos.

Assinale
a) se apenas I correta.
b) se apenas I e II so corretas.
c) se apenas I e III so corretas.
d) se apenas II e III so corretas.
e) se I, II e III so corretas.

25 (UEPG PR) Sobre a msica popular brasileira, assinale o que for correto.
01. O Ato Institucional no 5 atingiu apenas alguns setores da sociedade. A msica e o cinema tiveram oportunidade de
expresso e divulgao.
02. A ditadura militar encontrou a cultura brasileira numa fase de grande agitao. Movimentos estticos e polticos,
como o Tropicalismo, liderado, entre outros, pelos compositores Gilberto Gil e Caetano Veloso, propunham uma
nova leitura e uma interpretao crtica do pas.
04. Os anos do governo Mdici corresponderam, no campo da criao artstica, a uma fase de pouca renovao: as
restries s manifestaes artsticas dificultaram enormemente a expresso. Por outro lado, predominaram
msicas ufansticas, que procuravam divulgar uma imagem nacional de felicidade e progresso.
08. A dcada de 1950 foi marcada por uma srie de movimentos que, impulsionados pelo otimismo advindo das
transformaes sociais e industriais ou pela vontade de conhecer, criticar e interpretar a nova realidade urbana e
rural do pas, deram novos ares cultura nacional. Nesse contexto surgiram manifestaes como a bossanova, um
movimento de modernizao e internacionalizao da msica popular brasileira.
16. Enquanto o cinema, esgotada a fase da pornochanchada, produz filmes ligados nossa realidade, a msica
popular brasileira se renova, rejeitando compositores tradicionais como Nelson do Cavaquinho, Adoniram Barbosa
e Cartola.


327











PROF.: JEFT RODRIGUES DE BARROS








C
C
o
o
n
n
t
t
e
e

d
d
o
o
s
s
:
:

D
D
i
i
t
t
a
a
d
d
u
u
r
r
a
a
M
M
i
i
l
l
i
i
t
t
a
a
r
r
e
e
B
B
r
r
a
a
s
s
i
i
l
l
A
A
t
t
u
u
a
a
l
l




M
M
i
i
n
n
i
i
s
s
t
t
r
r
a
a
d
d
o
o
s
s
d
d
u
u
r
r
a
a
n
n
t
t
e
e
o
o
m
m

s
s
d
d
e
e
o
o
u
u
t
t
u
u
b
b
r
r
o
o














D Da at ta a d da a E En nt tr re eg ga a : : _ __ __ __ __ __ __ __ _/ /_ __ __ __ __ __ __ __ __ __ __ _/ /2 20 01 13 3
LISTA 6 HISTRIA

328


01 - (PUC SP) "Os anos 70, que se iniciaram em 1969, foram terrveis. Todo mundo parecia apoiar a ditadura. Os
brasileiros comearam a dcada torcendo pelo Brasil na Copa, '90 milhes em ao', unidos em torno da excelente
seleo, que levou o tricampeonato. A vitria deu grande prestgio a Emlio Garrastazu Mdici, o militar de planto no
governo. O plano econmico, apelidado de 'milagre brasileiro', alm de enriquecer ainda mais a burguesia, propiciou a
expanso da classe mdia e elevou os padres de consumo de muitas famlias: eletrodomsticos, um carro, o segundo
carro, financiamentos da casa prpria pelo Banco Nacional da Habitao, o BNH. Mas, principalmente, o comeo dos
anos 70 marca o incio da era da televiso no Brasil."
Maria Rita Kehl. As duas dcadas dos anos 70, in Anos 70:
trajetrias. So Paulo: Iluminuras, 2006, p. 32. Adaptado.
O texto faz um balano da dcada de 1970 no Brasil e destaca, entre outros aspectos, o

a) aumento da carestia, o avano do populismo e a exploso no consumo de bens de primeira necessidade.
b) surgimento da indstria automobilstica, a vitria eleitoral dos militares e a forte represso oposio institucional.
c) aumento da prtica de esportes, a militarizao do cotidiano e o declnio do regime militar.
d) surgimento do sonho da casa prpria, a superao da hiperinflao e a plena democratizao do pas.
e) aumento das desigualdades sociais, o avano da cultura de massa e o autoritarismo poltico.

02 - (UFRN) Durante o governo Mdici, a propaganda foi amplamente utilizada para divulgar o projeto poltico-
ideolgico dos Governos Militares. O slogan abaixo representativo dessa propaganda.


Disponvel em: www.mundovestibular.com.br. Acesso em: 20 jun. 2011.

a) Justifique por que esse slogan representativo do projeto poltico-ideolgico dos Governos Militares e cite duas
medidas adotadas por esses governos para concretiz-lo.
b) Mencione e explique duas reaes contrrias da sociedade brasileira a esse modelo poltico.

03 - (UNESP SP) A situao de harmonia no Congresso entraria em crise nas eleies de 1974, marco importante do
avano pela retomada do Estado de Direito.
(Edgard Leite Ferreira Neto. Os partidos polticos no Brasil, 1988.)
O texto menciona as eleies parlamentares de 1974, ocorridas durante o regime militar. Pode-se dizer que essas
eleies

a) representaram uma vitria significativa do partido da situao e eliminaram os esforos reformistas de deputados e
senadores.
b) revelaram a ampla hegemonia de que o governo desfrutava nos estados economicamente mais fortes do Sudeste
e sua fragilidade no Centro-Norte do pas.
c) reforaram a convico de que o bipartidarismo era o modelo poltico-partidrio adequado para a consolidao da
Repblica brasileira.
d) demonstraram insatisfao de parte expressiva da sociedade brasileira e provocaram forte reao do governo, que
alterou as leis eleitorais para assegurar a manuteno do controle sobre o Congresso Nacional.
e) expressaram a popularidade dos candidatos do partido de oposio e o desejo dos oposicionistas de manterem a
ordem poltica ento predominante.

04 - (Unemat MT) O Regime Militar no Brasil foi instaurado em 1964 e estendeu-se por 21 anos. Nesse perodo, a
presidncia da Repblica foi ocupada sucessivamente por generais do exrcito.

Assinale a alternativa cujo fato no corresponde ao perodo do Regime Militar.
a) Plano Cohen
b) Cinema Novo
c) Guerrilha do Araguaia
d) Operao Bandeirantes
e) Movimento Artstico Tropiclia
Caderno de Atividades

Disciplina:
Histria

Professor(a):
Jeft
Aluno:
3 ano
Ensino Mdio
Data de Recebimento:
_____/_____/_____
Lista 06

Data Entrega:
_____/_____/_____

329
05 - (UNIFOR CE) Em 13 de dezembro de 1968, o Presidente Marechal Arthur da Costa e Silva decretou o Ato
Institucional n 5 (AI-5). Chamado de golpe dentro do golpe, o AI-5 concretizou o golpe de 1964 e deixou claro que os
militares estavam dispostos a abandonar sua posio de poder moderador. A propsito desse ato, assinale a
alternativa INCORRETA.

a) Decretou o fechamento do Congresso Nacional, autorizando o Executivo a legislar em todas as matrias previstas
nas Constituies.
b) Suspendeu a garantia de habeas corpus nos casos de crimes polticos contra a segurana nacional, a ordem
econmica e social e a economia popular.
c) Restabeleceu, no cenrio nacional, as demisses sumrias, cassaes de mandatos e suspenso de direitos
polticos.
d) Preservou, como exceo, as garantias constitucionais ou legais de vitalicidade, inamovibilidade e estabilidade dos
militares e juzes.
e) Resultou nas prises do ex-governador do Rio de Janeiro Carlos Lacerda e do ex-presidente da Repblica
Juscelino Kubitschek.

06 - (UECE) No perodo compreendido entre 1950 e 1980, o Brasil passou por um intenso processo de modernizao
que alterou em profundidade sua fisionomia social, econmica e poltica.

No que tange a essas transformaes, considere as afirmaes a seguir:

I. A maior alterao foi a inverso da relao campo/cidade. Em 1950, a populao rural representava mais da
metade da populao do pas; em 1980, a populao urbana havia superado a rural.
II. Nesse perodo o eixo econmico deslocou-se da cidade para o campo, cenrio tradicionalmente produtor de
riquezas no pas desde os tempos coloniais.
III. O desenvolvimento da industrializao e a consequente integrao do Brasil no conjunto econmico capitalista
mundial foram caractersticas importantes deste momento histrico.

Est correto o que se afirma em

a) I e II apenas.
b) I, II e III.
c) II e III apenas.
d) I e III apenas.

07 - (UFPE) O sucesso do cinema atingiu a sociedade brasileira, que divertiu muito com as chanchadas e os
desempenhos de Oscarito e Grande Otelo. Mas, com a chegada do chamado Cinema Novo e suas influncias, a
produo cinematogrfica do Brasil:

00. dedicou-se a defender o nacionalismo, com filmes histricos sobre a poca da colonizao portuguesa.
01. mudou de rumos, com reflexes mais intelectualizadas e renovadoras sobre a identidade nacional.
02. foi monopolizada por produes pouco crticas e originais, ligadas ao governo federal.
03. com temas diferentes, fez sucesso em festivais internacionais e teve filmes premiados.
04. passou a ser financiado pelo capital estrangeiro, com destaques para os filmes de Gluber Rocha e Anselmo
Duarte.

08 - (FGV) Na mesma poca da Bossa Nova na msica, surgia o Cinema Novo. Entre 1960 e 1962, um grupo de
jovens cineastas, entre eles Glauber Rocha, Arnaldo Jabor, Ruy Guerra, alm do veterano Nelson Pereira dos Santos,
preconizava a necessidade de um cinema ousado, em forma e contedo (...)
(Marcos Napolitano. Cultura brasileira: utopia
e massificao (1950-1980), 2001.)

Sobre essa ousadia em forma e contedo, correto afirmar que o Cinema Novo

a) trabalhava com pardias de superprodues do cinema europeu e usava de referncias carnavalescas para
representar o gosto popular pelos melodramas.
b) defendia um cinema de autor, com a utilizao de um cenrio natural, mostrando a realidade brasileira marcada
por relaes sociais conflituosas.
c) negava destaque aos problemas contemporneos e tinha como temtica a recuperao de um passado mtico
brasileiro, sob uma esttica futurista.
d) seguia os padres hollywoodianos quanto temtica do progresso e recebia decisivo apoio financeiro da Ancine
Agncia Nacional do Cinema.
e) reconhecia, na harmonia social e racial brasileira, o elemento bsico para a compreenso da realidade econmica
do pas.

330
09 - (UFTM MG) Nos festivais da cano, msicas de jovens compositores expressavam os sentimentos de protesto
de toda uma gerao. Nos Teatros Opinio, de Arena e Oficina, montavam-se shows e peas revolucionrias sobre a
realidade do pas. A rebeldia ganhava as artes plsticas, enquanto o Cinema Novo buscava uma nova linguagem para
exprimir a identidade nacional. Seria, alis, a criao de uma verdadeira cultura brasileira o objetivo principal do
movimento que foi chamado de tropicalismo.
(Amrico Freire, Marly Motta e Dora Rocha, Histria em curso: o Brasil e suas relaes com o mundo ocidental)

Essa efervescncia cultural relaciona-se

a) crise do Estado oligrquico.
b) s crticas contra o Estado Novo.
c) aos desmandos do governo JK.
d) a formas de resistncia ditadura militar.
e) redemocratizao concretizada em 1988.

10 - (FUVEST SP) No incio de 1969, a situao poltica se modifica. A represso endurece e leva retrao do
movimento de massas. As primeiras greves, de Osasco e Contagem, tm seus dirigentes perseguidos e so
suspensas. O movimento estudantil reflui. A oposio liberal est amordaada pela censura imprensa e pela
cassao de mandatos.
Apolnio de Carvalho. Vale a pena sonhar. Rio de Janeiro:
Rocco, 1997, p. 202.

O testemunho, dado por um participante da resistncia ditadura militar brasileira, sintetiza o panorama poltico dos
ltimos anos da dcada de 1960, marcados

a) pela adeso total dos grupos oposicionistas luta armada e pela subordinao dos sindicatos e centrais operrias
aos partidos de extrema esquerda.
b) pelo bipartidarismo implantado por meio do Ato Institucional n 2, que eliminou toda forma de oposio institucional
ao regime militar.
c) pela desmobilizao do movimento estudantil, que foi bastante combativo nos anos imediatamente posteriores ao
golpe de 64, mas depois passou a defender o regime.
d) pelo apoio da maioria das organizaes da sociedade civil ao governo militar, empenhadas em combater a
subverso e afastar, do Brasil, o perigo comunista.
e) pela decretao do Ato Institucional n 5, que limitou drasticamente a liberdade de expresso e instituiu medidas
que ampliaram a represso aos opositores do regime.

11 - (UFTM MG) O refro Um, dois, trs, quatro, cinco, mil, queremos eleger o presidente do Brasil! foi entoado nos
vrios comcios do movimento Diretas J, iniciado em fins de 1983 e que tomou conta das ruas do pas em 1984.

Sobre esse movimento, correto afirmar que

a) resultou na eleio do Presidente Fernando Collor de Mello, que no chegou a terminar o seu mandato.
b) preocupou os militares, que tentaram acalmar os nimos por meio da lei que anistiou os presos polticos.
c) renovou o cenrio poltico nacional, pois foi a causa do surgimento de novos partidos e lideranas polticas.
d) contou com o apoio do Presidente Figueiredo, que autorizou a realizao dos comcios e retirou o exrcito das ruas.
e) terminou por no atingir seus objetivos, pois no se obtiveram os votos necessrios para alterar a Constituio ento
em vigor.

12 - (UNICAMP SP) O movimento pelas Diretas J provocou uma das maiores mobilizaes populares na histria
recente do Brasil, tendo contado com a cobertura nos principais jornais do pas.

Assinale a alternativa correta.

a) O movimento pelas Diretas J, baseado na emenda constitucional proposta pelo deputado Dante de Oliveira,
exigia a antecipao das eleies gerais para deputados, senadores, governadores e prefeitos.
b) O fato de que os protestos populares pelas Diretas J pudessem ser veiculados nas pginas dos jornais indica que
o governo vigente, ao evitar censurar a imprensa, mostrava-se favorvel s eleies diretas para presidente.
c) O movimento pelas Diretas J exigia que as eleies presidenciais de 1985 ocorressem no de forma indireta, via
Colgio Eleitoral, mas de forma direta por meio do voto popular.
d) As manifestaes populares pelas Diretas J consistiram nas primeiras marchas e protestos civis no espao
pblico desde a instituio do AI-5, em dezembro de 1968.




331
13 - (FGV) Em 15 de janeiro de 1985, Tancredo Neves e Jos Sarney foram eleitos, respectivamente, presidente e
vice-presidente pelo Colgio Eleitoral. A respeito do funcionamento das eleies indiretas no Brasil, no tempo da
ditadura militar, correto afirmar:

a) As eleies diretas para presidente foram mantidas entre 1964 e 1982 e o Colgio Eleitoral institudo em 1983,
diante do avano das foras oposicionistas.
b) Entre 1964 e 1973, os presidentes da repblica foram eleitos pelos governadores estaduais, prefeitos das capitais
e pelos comandantes das Foras Armadas.
c) Senadores, deputados federais e deputados escolhidos nas Assembleias Legislativas Estaduais tinham direito a
voto no Colgio Eleitoral de 1985.
d) At 1985, os cinco candidatos mais votados nas Assembleias Legislativas Estaduais eram submetidos escolha
dos integrantes do Colgio Eleitoral.
e) As duas chapas mais votadas pelos deputados federais e senadores eram submetidas ao Colgio Eleitoral
composto pelos comandantes das Foras Armadas.

14 - (UNESP SP) A campanha pelo restabelecimento das eleies diretas para presidente da Repblica do Brasil, em
1984, intitulada Diretas J!,

a) tentava garantir que o primeiro presidente ps-regime militar fosse escolhido, em 1985, pelo Colgio Eleitoral.
b) defendia a continuidade dos militares no poder, desde que fossem escolhidos pelo voto direto dos brasileiros.
c) foi a primeira mobilizao pblica de membros da sociedade civil brasileira desde o golpe militar de 1964.
d) reuniu diferentes partidos polticos em torno da aprovao de emenda constitucional que reintroduzia o voto direto
para presidente.
e) teve sucesso, pois contou com apoio oficial da Igreja Catlica, dos sindicatos, das foras armadas e do partido
situacionista.

15 - (UNIRG TO) Maro de 1985, Jos Sarney (PMDB) assumiu de forma inesperada a Presidncia da Repblica e
encontrou um Brasil com graves problemas estruturais, que entravavam o desenvolvimento e prejudicavam a
competitividade do pas no mercado mundial.

Todas as assertivas indicam caractersticas desse perodo, EXCETO:

a) As qualidades dos servios pblicos, como nas reas de sade e de educao, encontravam-se em queda.
b) As rodovias careciam de manuteno.
c) A capacidade da rede telefnica estava muito aqum da populao, mas as leis brasileiras estimulavam as empresas
estrangeiras de informtica a atuarem no Brasil.
d) Os ndices inflacionrios subiam em ritmo alarmante.

16 - (UECE) Em 28 de fevereiro de 1986, o Presidente Jos Sarney anunciou ao pas, em cadeia de rdio e televiso,
a mais radical mudana na economia nos ltimos anos: o Plano Cruzado.

Analise as seguintes afirmaes acerca dessa medida econmica:

I. Os preos e o cmbio foram congelados, o mesmo ocorrendo com os aluguis, as prestaes e os salrios.
II. O Plano foi lanado na forma de um decreto-lei, imposto sociedade de cima pra baixo, mas, ainda assim,
agradou a populao.
III. O Plano, logo comeou a dar mostras de sua fragilidade, fruto de sua desvinculao com a realidade brasileira,
embora tenha propiciado algumas manifestaes coletivas pouco vistas na sociedade brasileira.
Est correto o que se afirma em
a) I e II, apenas.
b) III, apenas.
c) I, II e III.
d) II, apenas.

17 - (UNIFOR CE) O Brasil visto hoje como um pas preparado para receber investimentos externos. No entanto, a
violncia, especialmente urbana, vista com muita preocupao pelos investidores estrangeiros. Recentemente, o
Governo do Estado do Rio de Janeiro, com apoio do Governo Federal, realizou um conjunto de operaes para
pacificar os pontos mais crticos da Cidade Maravilhosa. Sobre este assunto, correto afirmar que:

a) A violncia no Rio de Janeiro est associada ao trfico de drogas.
b) A principal operao foi realizada no morro do Borel, Complexo do Alemo e Cidade de Deus e Paraispolis.
c) A favela da Rocinha hoje um lugar pacificado pela operao.
d) A violncia concentra-se nos bairros de classe mdia e no nas favelas.
e) A relao entre traficantes, polcia e populao de cordialidade.

332
18 - (FUVEST SP) O presidente do Senado, Jos Sarney (PMDB-AP), disse nesta segunda-feira [30/5] que o
impeachment do ex-presidente Fernando Collor de Mello foi apenas um acidente na histria do Brasil. Sarney
minimizou o episdio em que Collor, que atualmente senador, teve seus direitos polticos cassados pelo Congresso
Nacional. Eu no posso censurar os historiadores que foram encarregados de fazer a histria. Mas acho que talvez
esse episdio seja apenas um acidente que no devia ter acontecido na histria do Brasil, disse o presidente do
Senado.Correio Braziliense, 30/05/2011.

Sobre o episdio mencionado na notcia acima, pode-se dizer acertadamente que foi um acontecimento

a) de grande impacto na histria recente do Brasil e teve efeitos negativos na trajetria poltica de Fernando Collor, o
que faz com que seus atuais aliados se empenhem em desmerecer este episdio, tentando diminuir a importncia
que realmente teve.
b) nebuloso e pouco estudado pelos historiadores, que, em sua maioria, trataram de censur-lo, impedindo uma justa
e equilibrada compreenso dos fatos que o envolvem.
c) acidental, na medida em que o impeachment de Fernando Collor foi considerado ilegal pelo Supremo Tribunal
Federal, o que, alis, possibilitou seu posterior retorno cena poltica nacional, agora como senador.
d) menor na histria poltica recente do Brasil, o que permite tomar a censura em torno dele, promovida oficialmente
pelo Senado Federal, como um episdio ainda menos significativo.
e) indesejado pela imensa maioria dos brasileiros, o que provocou uma onda de comoo popular e permitiu o retorno
triunfal de Fernando Collor cena poltica, sendo candidato conduzido por mais duas vezes ao segundo turno das
eleies presidenciais.

19 - (UNIFOR CE) O governo de Lus Incio Lula da Silva foi importante para a consolidao da democracia no Brasil,
dado que nenhum elemento apontou para a interrupo do processo democrtico. Do ponto de vista econmico e
social, pode-se destacar como realizaes do governo:
a) A preocupao primordial com o controle da inflao e a manuteno de um supervit primrio.
b) O aprofundamento do endividamento junto ao FMI.
c) A diminuio da carga tributria, particularmente nos aspectos do imposto sobre a renda.
d) A desvalorizao sistemtica da taxa de cmbio com o intuito de incentivar as exportaes.
e) A criao do programa Fome Zero para o atendimento social s famlias de classe mdia.

20 - (FGV) Recentemente, em julho de 2011, faleceu o ex-presidente Itamar Franco. A respeito da sua chegada ao
poder e do seu governo, correto afirmar:
a) Venceu Luiz Incio Lula da Silva no primeiro turno das eleies disputadas em 1994, graas ao sucesso do Plano
Real, implementado no governo de Fernando Henrique Cardoso.
b) Venceu Luiz Incio Lula da Silva nas eleies de 1989 e organizou um governo de coalizo nacional, do qual
participaram todos os demais partidos polticos brasileiros, inclusive o PT.
c) Assumiu a presidncia aps o processo de impeachment do presidente Fernando Collor de Mello e, com seu
ministro Fernando Henrique Cardoso, implementou o Plano Real.
d) Foi eleito em janeiro de 1985, em eleio direta pelo colgio eleitoral, e organizou um governo de reformas
polticas e econmicas que permitiram sua reeleio em 1994.
e) Foi eleito em 1994 devido ao sucesso do Plano Real implementado no governo do presidente Fernando Henrique
Cardoso, do qual participou como ministro da Fazenda.

21 - (UCS RS) Nos ltimos 30 anos, o Brasil deu passos importantes para a consolidao do regime democrtico,
com a conquista de vrios direitos, como a liberdade de expresso e o direito de escolher, por meio de eleies diretas,
todos os governantes do pas.

Sobre esse perodo, considere as proposies abaixo.
I. A Nova Repblica, que ps fim a 21 anos de ditadura militar, comeou de forma trgica: Tancredo Neves, o
presidente eleito pelo Colgio Eleitoral, no pde assumir por motivo de doena. Em 15 de maro de 1985, Jos
Sarney, eleito vice-presidente, assumiu o cargo de presidente da Repblica. Tancredo Neves veio a falecer em 21
de abril do mesmo ano.
II. A Constituio de 1988, conhecida como Constituio Cidad, assegurou amplamente os direitos civis, sendo as
prticas do racismo e da tortura classificadas como crimes inafianveis.
III. Em 1989 o Brasil realizou a sua primeira eleio presidencial, desde 1960, polarizada entre Fernando Collor e
Ulysses Guimares. O primeiro era identificado com os polticos apoiadores do regime militar; o segundo
simbolizava a oposio ditadura.
Das afirmativas acima, pode-se dizer que
a) apenas I est correta.
b) apenas II est correta.
c) apenas I e II esto corretas.
d) apenas II e III esto corretas.
e) I, II e III esto corretas.

333
22 - (UFU MG) Caras pintadas foi o nome dado aos jovens e estudantes que, em agosto e setembro de 1992,
pintaram o rosto de verde e amarelo e organizaram passeatas pelo impeachment do ento presidente Fernando Collor
de Mello.

Sobre os fatos e o contexto histrico relacionados ao movimento dos caras pintadas, assinale a alternativa
INCORRETA.

a) Os estudantes, menores de idade, estavam excludos do cenrio poltico eleitoral naquele momento, o que
estimulou a reivindicao pela ampliao do direito de voto levada a cabo pela liderana da UNE e pelo Partido
dos Trabalhadores.
b) Os meios de comunicao de massa tiveram papel relevante no processo de denncia e investigao contra o
esquema de corrupo do governo Collor, bem como na divulgao das manifestaes populares pelo
impeachment.
c) O movimento nas ruas das cidades surgiu como resposta ao apelo de Fernando Collor populao brasileira para
que esta vestisse verde e amarelo em sinal de apoio ao presidente; a populao saiu s ruas vestida de preto.
d) Collor construiu uma imagem de homem empreendedor e moderno explorando a visibilidade na mdia, pretendeu
imprimir uma imagem de probidade administrativa e sofreu o impeachment por crime de responsabilidade.


23 - (Unimontes MG) Observe a charge de Nani


A expresso Baixo Clero vem da Frana, sculo XVIII. No Brasil, ganhou popularidade e outro significado aps a
eleio de Severino Cavalcanti para a Presidncia da Cmara.

Explique o significado da expresso Baixo Clero:
a) no mbito da sociedade do sculo XVIII.
b) no mbito das relaes polticas do Legislativo brasileiro com o Executivo, no governo de Lus Incio Lula da Silva.

24 - (FGV) Nesse momento alto da histria orgulhamo-nos de pertencer a um povo que no se abate, que sabe
afastar o medo e no aceita colher o dio. A Nao inteira comunga desse ato de esperana. Reencontramos, depois
de iluses perdidas e pesados sacrifcios, o bom e velho caminho democrtico. (...) A primeira tarefa de meu governo
promover a organizao institucional do Estado. (...) Faz algumas semanas eu anunciava a construo de uma Nova
Repblica. Vejo nessa fase da vida nacional a grande oportunidade histrica de nosso povo.
(Discurso de Tancredo Neves em 15 de janeiro de
1985, aps ser eleito pelo Colgio Eleitoral)

(Comcio das Diretas em frente igreja da Candelria,
no Rio de Janeiro, em 10 de abril de 1984)

No alvorecer dos anos oitenta, na sociedade brasileira, uma srie de
manifestaes polticas e sociais criticaram o governo vigente sob a
bandeira do retorno dos direitos e prerrogativas democrticas. A partir
dessas informaes e dos documentos acima:
a) explique o movimento Diretas J;
b) identifique uma medida implementada pelo governo da Nova Repblica
favorvel ao restabelecimento dos direitos democrticos.


334







PROF.: J. CNDIDO






C
C
o
o
n
n
t
t
e
e

d
d
o
o
s
s
:
:

E
E
l
l
e
e
t
t
r
r
i
i
z
z
a
a

o
o
e
e
f
f
o
o
r
r

a
a
e
e
l
l

t
t
r
r
i
i
c
c
a
a





M
M
i
i
n
n
i
i
s
s
t
t
r
r
a
a
d
d
o
o
s
s
d
d
u
u
r
r
a
a
n
n
t
t
e
e
o
o
m
m

s
s
d
d
e
e
f
f
e
e
v
v
e
e
r
r
e
e
i
i
r
r
o
o















D Da at ta a d da a E En nt tr re eg ga a : : _ __ __ __ __ __ __ __ _/ /_ __ __ __ __ __ __ __ __ __ __ _/ /2 20 01 13 3
LISTA 1 FSICA

335

01 - (UFTM) Em uma festa infantil, o mgico resolve fazer uma demonstrao que desperta a curiosidade das
crianas ali presentes. Enche uma bexiga com ar, fecha-a, e, a seguir, aps esfreg-la vigorosamente nos cabelos de
uma das crianas, encosta o balo em uma parede lisa e perfeitamente vertical. Ao retirar a mo, a bexiga permanece
fixada parede. Qual foi a mgica?

a) O ar da bexiga interage com a parede, permitindo o repouso da bexiga.
b) Ao ser atritada, a bexiga fica eletrizada e induz a distribuio das cargas da parede, o que permite a atrao.
c) O atrito esttico existente entre a bexiga e a parede suficiente para segur-la, em repouso, na parede.
d) A bexiga fica eletrizada, gerando uma corrente eltrica que a segura parede.
e) Por ser bom condutor de eletricidade, o ar no interior da bexiga absorve energia eltrica da parede, permitindo a
atrao.

02 - (UNESP) Induo eletrosttica o fenmeno no qual pode-se provocar a separao de cargas em um corpo
neutro pela aproximao de um outro j eletrizado. O condutor que est eletrizado chamado indutor e o condutor no
qual a separao de cargas ocorreu chamado induzido. A figura mostra uma esfera condutora indutora positivamente
eletrizada induzindo a separao de cargas em um condutor inicialmente neutro.


(http://efisica.if.usp.br. Adaptado.)

Analisando a figura e sobre o processo de eletrizao por induo, so feitas as seguintes afirmaes:

I. Para eletrizar o corpo neutro por induo, deve-se aproximar o indutor, conectar o induzido terra, afastar o indutor
e, finalmente, cortar o fio terra.
II. Para eletrizar o corpo neutro por induo, deve-se aproximar o indutor, conectar o induzido terra, cortar o fio terra
e, finalmente, afastar o indutor.
III. Na situao da figura, a conexo do induzido terra, com o indutor nas suas proximidades, faz com que prtons do
induzido escoem para a terra, por repulso.
IV. No final do processo de eletrizao por induo, o corpo inicialmente neutro e que sofreu induo, adquire carga de
sinal negativo.

Est correto, apenas, o contido em
a) II.
b) I e III.
c) I e IV.
d) II e IV.
e) II, III e IV.

03 - (FUVEST SP) A lei de conservao da carga eltrica pode ser enunciada como segue:

a) A soma algbrica dos valores das cargas positivas e negativas em um sistema isolado constante.
b) Um objeto eletrizado positivamente ganha eltrons ao ser aterrado.
c) A carga eltrica de um corpo eletrizado igual a um nmero inteiro multiplicado pela carga do eltron.
d) O nmero de tomos existentes no universo constante.
e) As cargas eltricas do prton e do eltron so, em mdulo, iguais.

Caderno de Atividades

Disciplina:
Fsica

Professor(a):
J. Cndido
Aluno:
3 ano
Ensino Mdio
Data de Recebimento:
_____/_____/_____
Lista 01

Data Entrega:
_____/_____/_____

336
04 - (UFTM) A induo eletrosttica consiste no fenmeno da separao de cargas em um corpo condutor (induzido),
devido proximidade de outro corpo eletrizado (indutor).
Preparando-se para uma prova de fsica, um estudante anota em seu resumo os passos a serem seguidos para
eletrizar um corpo neutro por induo, e a concluso a respeito da carga adquirida por ele.

PASSOS A SEREM SEGUIDOS:
I. Aproximar o indutor do induzido, sem toc-lo.
II. Conectar o induzido Terra.
III. Afastar o indutor.
IV. Desconectar o induzido da Terra.

CONCLUSO:
No final do processo, o induzido ter adquirido cargas de sinais iguais s do indutor.

Ao mostrar o resumo para seu professor, ouviu dele que, para ficar correto, ele dever

a) inverter o passo III com IV, e que sua concluso est correta.
b) inverter o passo III com IV, e que sua concluso est errada.
c) inverter o passo I com II, e que sua concluso est errada.
d) inverter o passo I com II, e que sua concluso est correta.
e) inverter o passo II com III, e que sua concluso est errada.

05 - (UNESP) Um dispositivo simples capaz de detectar se um corpo est ou no eletrizado, o pndulo eletrosttico,
que pode ser feito com uma pequena esfera condutora suspensa por um fio fino e isolante. Um aluno, ao aproximar um
basto eletrizado do pndulo, observou que ele foi repelido (etapa I). O aluno segurou a esfera do pndulo com suas
mos, descarregando-a e, ento, ao aproximar novamente o basto, eletrizado com a mesma carga inicial, percebeu
que o pndulo foi atrado (etapa II). Aps tocar o basto, o pndulo voltou a sofrer repulso (etapa III). A partir dessas
informaes, considere as seguintes possibilidades para a carga eltrica presente na esfera do pndulo:

Negativa Neutra Negativa 5
Negativa Negativa Positiva 4
Negativa Positiva Negativa 3
Positiva Neutra Positiva 2
Neutra Negativa Neutra 1
III E II E I E P TAPA TAPA TAPA DE OSSIBILIDA


Somente pode ser considerado verdadeiro o descrito nas possibilidades

a) 1 e 3.
b) 1 e 2.
c) 2 e 4.
d) 4 e 5.
e) 2 e 5.

06 - (UECE) Quatro cargas eltricas fixas, com valores +q, +2q, +3q e +4q, so dispostas nos vrtices de um
quadrado de lado d. As cargas so posicionadas na ordem crescente de valor, percorrendo-se o permetro do
quadrado no sentido horrio. Considere que este sistema esteja no vcuo e que e
0
a permissividade eltrica nesse
meio. Assim, o mdulo do campo eltrico resultante no centro do quadrado

a)
2
0
d
q
e 4
1
t
.
b)
2
0
d
q
e
2
t
.
c)
2
0
d
q
e
1
t
.
d)
2
0
d
q
e
4
t
.





337
07 - (UDESC) Duas cargas puntiformes +4q e +q esto dispostas ao longo de uma linha reta horizontal e separadas
por uma distncia d. Em que posio x, ao longo da linha horizontal, e em relao carga +4q, deve-se localizar uma
terceira carga +q a fim de que esta adquira uma acelerao nula?

a) 2d / 3
b) 3d / 2
c) 5d / 4
d) d / 3
e) 3d / 4

08 - (UEFS BA) Duas esferas condutoras idnticas foram suspensas utilizando-se dois fios isolantes de peso
desprezvel. Separando-as, eletrizou-se uma com carga Q, positiva, mantendo a outra neutra. Aps o contato entre
elas, foram abandonadas e se equilibraram na posio mostrada na figura.




Considerando-se a constante eletrosttica do meio e o mdulo da acelerao da gravidade iguais a 9,010
9
Nm
2
/C
2
e
10,0m/s
2
, respectivamente, e sabendo-se que a massa de cada esfera de 0,60g, o valor da carga Q, em C, era de,
aproximadamente,

a) 1,2
b) 1,7
c) 2,4
d) 3,0
e) 3,6

09 - (UEG GO) Duas partculas de massas m
1
e m
2
esto presas a uma haste retilnea que, por sua vez, est presa, a
partir de seu ponto mdio, a um fio inextensvel, formando uma balana em equilbrio. As partculas esto
positivamente carregadas com carga Q
1
= 3,0 C e Q
2
= 0,3 C. Diretamente acima das partculas, a uma distncia d,
esto duas distribuies de carga Q
3
= -1,0 C e Q
4
= -6,0 C, conforme descreve a figura.
Dado: k
0
= 9,0 x 10
9
N.m
2
/C
2



Sabendo que o valor de m
1
de 30 g e que a acelerao da gravidade local de 10 m/s
2
, determine a massa m
2
.

10 - (UDESC) Duas partculas eletrizadas idnticas so mantidas fixas, isoladas e distanciadas 1,0 cm entre si. Suas
massas e cargas eltricas so, respectivamente, 3,0 g e C 10 67 , 0
6
. A razo entre os mdulos das foras
eletrosttica e gravitacional existentes entre ambas as partculas :

a) 1,0 x 10
13

b) 1,0 x 10
7

c) 2,0 x 10
6
C/N
d) 1,0 x 10
14
C
2
/m
2

e) 1,0 x 10
12
N

11 - (UFTM) Uma haste isolante, homognea e apoiada em seu centro geomtrico equilibra quatro pequenas esferas
idnticas e de massas desprezveis, carregadas com cargas eltricas Q
A
, Q
B
, Q
C
e Q
D
, posicionadas como mostra a
figura.


338


Se as intensidades das cargas eltricas Q
B
, Q
C
e Q
D
so iguais a Q, a carga eltrica Q
A
, para que seja mantido o
equilbrio horizontal da haste, igual a

a) Q/4.
b) Q/2.
c) Q.
d) 2Q.
e) 4Q.

12 - (UFU MG) Nos vrtices A, B e C de um quadrado cuja diagonal mede m 2 3 , esto trs cargas eltricas positivas
puntiformes de intensidades Q
a
= 1 10
5
C; Q
b
= 3 10
4
C e Q
C
= 9 10
4
C. O valor de K 9 10
9
Nm
2
/C
2
.



O valor da fora eltrica resultante na carga A de

a) 12N
b) N 10 3
c) N 10 9
d) 0N

13 - (ASCES PE) Na figura a seguir, as cargas pontuais positivas Q
1
, Q
2
e Q
3
encontram-se fixas no vcuo. A carga
pontual negativa Q
4
est em equilbrio apenas sob a ao das foras gravitacional e eletrosttica. A fora eletrosttica
que a carga Q
1
exerce na carga Q
4
tem mdulo 20 N. Se L = 2D e Q3 = 3Q
1
, o peso da carga Q
4
igual a:



a) 15 N
b) 20 N
c) 120 N
d) 240 N
e) 360 N

14 - (FAVIP PE) Trs cargas pontuais positivas encontram-se arranjadas no vcuo de acordo com a figura a seguir,
onde D
2
/D
1
= 1/4. As cargas Q
1
e Q
2
esto fixas e a carga Q
3
encontra-se em equilbrio. Considerando apenas a ao
de foras eletrostticas, quanto vale a razo Q
2
/Q
1
?

a) 1/16
b) 1/4
c) 1
d) 4
e) 16

339
15 - (UEL PR) Devido ao balanceamento entre cargas eltricas positivas e negativas nos objetos e seres vivos, no
se observam foras eltricas atrativas ou repulsivas entre eles, em distncias macroscpicas. Para se ter, entretanto,
uma ideia da intensidade da fora gerada pelo desbalanceamento de cargas, considere duas pessoas com mesma
altura e peso separadas pela distncia de 0,8 m. Supondo que cada uma possui um excesso de prtons
correspondente a 1% de sua massa, a estimativa da intensidade da fora eltrica resultante desse desbalanceamento
de cargas e da massa que resultar numa fora-peso de igual intensidade so respectivamente:

Dado: Massa de uma pessoa: m = 70 kg
a) 910
17
N e 610
3
kg
b) 6010
24
N e 610
24
kg
c) 910
23
N e 610
23
kg
d) 410
17
N e 410
16
kg
e) 6010
20
N e 410
19
kg

16 - (UEFS BA) O objetivo primordial da Fsica entender a natureza de forma unificada. Tem-se algumas ideias
sobre como unificar as interaes fortes com as fracas e eletromagnticas a chamada Grande Unificao , mas
isso s pode ocorrer se a gravidade for includa na equao, o que traz grandes dificuldades.
Sabendo-se que a ordem de grandeza da constante de gravitao universal 10
10
Nm
2
/kg
2
, da constante eletrosttica
10
10
Nm
2
/C
2
, da massa do eltron 10
30
kg, da massa do prton 10
27
kg, da carga eltrica elementar 10
19
C, do
raio do tomo de hidrognio 10
10
m, a intensidade da atrao gravitacional entre um eltron e um prton, no tomo
de hidrognio, menor que a fora de atrao eltrica um nmero de vezes da ordem de
a) 10
19

b) 10
23

c) 10
39

d) 10
41

e) 10
44


17 - (PUC RJ) Uma carga Q
1
= 1,0 10
6
C est fixa no ponto x = 0. No instante t = 0 s, em x = 1,0 m se encontra
uma carga Q
2
= 2 Q
1
, em repouso, porm livre para se mover. Considere que o eixo x a linha que une as duas
cargas.
Dado que a constante k
e
= 9,0 10
9
N m
2
/C
2
, indique a fora em Newtons na direo x que a carga Q
2
faz sobre a
carga Q
1
.

a) 18,0 10
3
.
b) 4,5 10
3
.
c) 9,0 10
3
.
d) 18,0 10
3
.
e) 9,0 10
3
.

18 - (FEPECS DF) Trs cargas de valores Q, q e Q esto alinhadas como mostra a figura abaixo.



A distncia entre as extremidades, onde se encontram cargas Q, D, e a carga q se encontra no ponto mdio desse
segmento. Para que as resultantes das foras eletrostticas em cada carga seja nula, o valor da carga q deve ser:
a) Q/4;
b) 4Q;
c) Q/4;
d) 4Q;
e) Q/2.

19 - (UFU MG) Duas cargas +q esto fixas sobre uma barra isolante e distam entre si uma distncia 2d. Uma outra
barra isolante fixada perpendicularmente primeira no ponto mdio entre essas duas
cargas. O sistema colocado de modo que esta ltima haste fica apontada para cima.
Uma terceira pequena esfera de massa m e carga +3q furada atravessada pela haste
vertical de maneira a poder deslizar sem atrito ao longo desta, como mostra a figura
abaixo. A distncia de equilbrio da massa m ao longo do eixo vertical z.
Com base nessas informaes, o valor da massa m em questo pode ser escrito em
funo de d, z, g e k, onde g a acelerao gravitacional e k a constante eletrosttica.
A expresso para a massa m ser dada por:


340

a)
( )
2 3
2 2
2
z d
z kq
m
+
=
b)
( )
2 3
2 2
2
z d g
z kq 6
m
+
=
c)
( )
2
2 2
2
z d g
z kq 6
m
+
=
d)
( )
3
2 2
2
z d g
z kq 6
m
+
=

TEXTO: 1 - Comum questo: 20

Se necessrio considerar os dados abaixo:

Acelerao da gravidade: 10 m/s
2

Densidade da gua: 1 g/cm
3
= 10
3
kg/m
3

Calor especfico da gua: 1 cal/g.C
Carga do eltron = 1,6 x 10
19
C
Massa do eltron = 9 x 10
31
kg
Velocidade da luz no vcuo = 3 x 10
8
m/s
Constante de Planck = 6,6 x 10
34
J.s
sen 37 = 0,6
cos 37 = 0,8

20 - (UFPE) Considerando que as trs cargas da figura esto em equilbrio, determine qual o valor da carga Q
1
em
unidades de 10
9
C. Considere Q
3
= 3 10
9
C.



21 - (ENEM) O tempo que um nibus gasta para ir do ponto inicial ao ponto final de uma linha varia, durante o dia,
conforme as condies do trnsito, demorando mais nos horrios de maior movimento. A empresa que opera essa
linha forneceu, no grfico abaixo, o tempo mdio de durao da viagem conforme o horrio de sada do ponto inicial,
no perodo da manh.


De acordo com as informaes do grfico, um passageiro que necessita chegar at as 10h30min ao ponto final dessa
linha, deve tomar o nibus no ponto inicial, no mximo, at as:

a) 9h20min
b) 9h30min
c) 9h00min
d) 8h30min
e) 8h50min

22 - (ENEM) Joo e Antnio utilizam os nibus da linha mencionada na questo anterior para ir trabalhar, no perodo
considerado no grfico, nas seguintes condies:
trabalham vinte dias por ms;
Joo viaja sempre no horrio em que o nibus faz o trajeto no menor tempo;
Antnio viaja sempre no horrio em que o nibus faz o trajeto no maior tempo;
na volta do trabalho, ambos fazem o trajeto no mesmo tempo de percurso.
Considerando-se a diferena de tempo de percurso, Antnio gasta, por ms, em mdia,

a) 05 horas a mais que Joo.
b) 10 horas a mais que Joo.
c) 20 horas a mais que Joo.
d) 40 horas a mais que Joo.
e) 60 horas a mais que Joo.

341
23 - (ENEM) O consumo total de energia nas residncias brasileiras envolve diversas fontes, como eletricidade, gs
de cozinha, lenha, etc. O grfico mostra a evoluo do consumo de energia eltrica residencial, comparada com o
consumo total de energia residencial, de 1970 a 1995.


Fonte: valores calculados atravs dos dados
obtidos de: http://infoener.iee.usp.br/1999.

Verifica-se que a participao percentual da energia eltrica no total de energia gasto nas residncias brasileiras
cresceu entre 1970 e 1995, passando, aproximadamente, de
a) 10% para 40%.
b) 10% para 60%.
c) 20% para 60%.
d) 25% para 35%.
e) 40% para 80%.

24 - (ENEM) As sociedades modernas necessitam cada vez mais de energia. Para entender melhor a relao entre
desenvolvimento e consumo de energia, procurou-se relacionar o ndice de Desenvolvimento Humano (IDH) de vrios
pases com o consumo de energia nesses pases.
O IDH um indicador social que considera a longevidade, o grau de escolaridade, o PIB (Produto Interno Bruto) per
capita e o poder de compra da populao. Sua variao de 0 a 1. Valores do IDH prximos de 1 indicam melhores
condies de vida.
Tentando-se estabelecer uma relao entre o IDH e o consumo de energia per capita nos diversos pases, no binio
1991-1992, obteve-se o grfico abaixo, onde cada ponto isolado representa um pas, e a linha cheia, uma curva de
aproximao.


Fonte: GOLDEMBERG, J. Energia, meio ambiente e
desenvolvimento. So Paulo: Edusp, 1998.

Com base no grfico, correto afirmar que:

a) quanto maior o consumo de energia per capita, menor o IDH.
b) os pases onde o consumo de energia per capita menor que 1 TEP no apresentam bons ndices de
desenvolvimento humano.
c) existem pases com IDH entre 0,1 e 0,3 com consumo de energia per capita superior a 8 TEP.
d) existem pases com consumo de energia per capita de 1 TEP e de 5 TEP que apresentam aproximadamente o
mesmo IDH, cerca de 0,7.
e) os pases com altos valores de IDH apresentam um grande consumo de energia per capita (acima de 7 TEP).



342
TEXTO: 1 - Comum questo: 25

O grfico a seguir ilustra a evoluo do consumo de eletricidade no Brasil, em GWh, em quatro setores de consumo,
no perodo de 1975 a 2005.


Balano Energtico Nacional. Braslia:
MME, 2003 (com adaptaes).

25 - (ENEM) A racionalizao do uso da eletricidade faz parte dos programas oficiais do governo brasileiro desde
1980. No entanto, houve um perodo crtico, conhecido como apago, que exigiu mudanas de hbitos da popul ao
brasileira e resultou na maior, mais rpida e significativa economia de energia. De acordo com o grfico, conclui-se que
o apago ocorreu no binio

a) 1998-1999.
b) 1999-2000.
c) 2000-2001.
d) 2001-2002.
e) 2002-2003.


343






PROF.: J. CNDIDO








C
C
o
o
n
n
t
t
e
e

d
d
o
o
s
s
:
:

C
C
a
a
m
m
p
p
o
o
e
e
l
l

t
t
r
r
i
i
c
c
o
o
e
e
p
p
o
o
t
t
e
e
n
n
c
c
i
i
a
a
l
l
e
e
l
l

t
t
r
r
i
i
c
c
o
o





M
M
i
i
n
n
i
i
s
s
t
t
r
r
a
a
d
d
o
o
s
s
d
d
u
u
r
r
a
a
n
n
t
t
e
e
o
o
m
m

s
s
d
d
e
e
m
m
a
a
r
r

o
o















D Da at ta a d da a E En nt tr re eg ga a : : _ __ __ __ __ __ __ __ _/ /_ __ __ __ __ __ __ __ __ __ __ _/ /2 20 01 13 3
LISTA 2 FSICA

344


01 - (UFG GO) A figura a seguir mostra o potencial de repouso de uma clula, o qual constante dentro e fora da
clula e varivel no interior da membrana. Esta variao do potencial V, representado no grfico, apenas hipottica.

Forada
clula
V(mV)
V=-70
0
Regiodamembrana
Dentrodaclula
Distncia(A)


a) Represente, graficamente, o campo eltrico dentro e fora da clula e no interior da membrana, supondo que a
espessura da membrana seja de 80 (1 = 10
10
m).
b) Em uma clula nervosa excitada, penetram, atravessando sua membrana, cerca de 2.10
6
ons Na
+
, em um intervalo
de tempo de 1,6 ms (1 ms =10
3
s). A rea da membrana celular de aproximadamente 5.10
10
m
2
. Calcule a
densidade mdia de corrente eltrica (corrente por unidade de rea) atravs dessa membrana.

02 - (UFOP MG) Duas cargas eltricas, Q
1
= +1x10
-6
C e Q
2
= +2,0x10
-6
C, esto fixas e separadas por uma distncia
d = 0,30m, conforme a figura abaixo.
a) Determine o ponto em que o campo eltrico resultante nulo. Este ponto chamado ponto de equilbrio.
b) Diga se este ponto de equilbrio estvel ou instvel. Justifique a resposta.

Q
1
Q
2
d


03 - (UFOP MG) Duas cargas, uma positiva, Q = 1,0x10
-6
C, e uma negativa, Q = -1,0x10
-6
C, esto fixas e
separadas pela distncia d = 0,8m.
Dado: K
0
= 9x10
9
Nm
2
/C
2
.

P
0,3m
0,4m 0,4m

a) Calcule e represente o campo eltrico no ponto P.
b) Desenhe as linhas de fora do campo resultante criado pelas duas cargas, inclusive aquela que passa pelo ponto P.

04 - (UFOP MG) A figura desta questo mostra uma esfera condutora de raio R = 2m, carregada com uma carga
positiva Q = 5x10
-6
C. Considere que a carga de prova q, nos itens (b) e (c) no muda a distribuio de carga na esfera
condutora.
Dado: 1/(4tc
0
) = 9x10
9
N.m
2
/C
2
.


Diga se as afirmativas abaixo esto certas ou erradas, justificando a sua resposta.
a) No interior da esfera, o campo eltrico constante, tem a direo radial e est dirigido para o seu centro.
b) A carga de prova positiva q = 1x10
-9
, colocada no ponto B, fica submetida a uma fora de mdulo 1,8x10
-6
N, na
direo da reta definida pelos pontos O e C e dirigida para o centro da esfera.
c) O trabalho para deslocar a carga de prova de O at A igual ao trabalho par desloc-la de B para C.

Caderno de Atividades

Disciplina:
Fsica

Professor(a):
J. Cndido
Aluno:
3 ano
Ensino Mdio
Data de Recebimento:
_____/_____/_____
Lista 02

Data Entrega:
_____/_____/_____

345
05 - (UFOP MG) A figura mostra duas cargas puntiformes, +Q
1
e Q
2
, distantes entre si de um valor d. Admita
que o mdulo de +Q
1
seja quatro vezes o de Q
2
, ou seja, Q
1
= 4Q
2
. Considere Q
1
na origem O do eixo Ox, conforme a
figura abaixo.
a) Argumente, qualitativamente, e mostre que o campo eltrico resultante em qualquer ponto do eixo Ox, esquerda
de +Q
1
, no ser nulo.
b) Faa o mesmo para qualquer ponto do eixo Ox situado entre +Q
1
e Q
2
.
c) Use a mesma argumentao para mostrar que o campo resultante s ser nulo em um ponto do eixo Ox direita de
Q
2
.


06 - (UFPE) Duas cargas puntiformes no vcuo, de mesmo valor Q = 125C e de sinais opostos, geram campos
eltricos no ponto P (vide figura). Qual o mdulo do campo eltrico resultante, em P, em unidades de 10
7
N/C?
+ Q
- Q
4cm
3
c
m
3
c
m
P


07 - (UFRJ) Em um condutor em equilbrio eletrosttico, o campo eltrico prximo s pontas bem mais intenso do
que em outros pontos de sua superfcie (veja a figura 1). Este fenmeno conhecido como o "poder das pontas".


Podemos explic-lo com um modelo simples: considere uma esfera condutora A, de raio igual a 10R, submetida a um
potencial eltrico constante igual a V
O
e uma pequena esfera B, tambm condutora, de raio igual a R, ligada primeira
por um fio condutor muito fino (veja figura 2).
.
.
R
B
fio
A
10R
Vo

Suponha as esferas suficientemente afastadas, de modo que possamos desfazer os efeitos de induo de carga entre
elas e considerar a distribuio de cargas nas esferas uniforme. Nestas condies, calcule:
a) A razo Q
A
/Q
B
entre as cargas da esfera A (Q
A
) e da esfera B (Q
B
).
b) A razo E
A
/ E
B
entre o mdulo do campo eltrico na superfcie da esfera (E
A
) e o mdulo do campo eltrico na
superfcie da esfera B (E
B
).

08 - (UFU MG) Duas esferas metlicas de raios r e R, com cargas respectivas q e Q, tm seus centros separados
pela distncia 0,8m.
a) calcule o campo eltrico (mdulo, direo e sentido) resultante num ponto P, situado na reta que contm os centros
das duas esferas e distante 0,2 m do centro da esfera maior, conforme a figura abaixo.
0,8m 0,2m
q
Q
P

Dados:
q = 20 C
Q = 4 C
r = 4 x 10
4
m
R = 20 x 10
4
m



346
A seguir as esferas so postas em contato e levadas novamente posio original.
Aps este processo, determine:
a) as novas cargas (q e Q) em cada esfera.
b) o nmero de eltrons transferidos de uma esfera para outra.
c) se os eltrons (cargas negativas) se transferiram da esfera maior para a menor ou vice-versa, justifique.
d) o potencial eltrico (em relao ao infinito) no ponto P.
Dados:
Carga do eltron = 1,6 x 10
19
C.
K = 9,0 x 10
9
N m
2
/C
2


09 - (FMTM MG) Duas cargas eltricas puntiformes, q
1
= 1,0 10
8
C e q
2
= 2,0 10
8
C, encontram-se fixas no
vcuo, respectivamente, no ponto E e no ponto A. O ponto E o centro de uma circunferncia, de raio 10 cm, e os
pontos A, B, C e D so pertencentes circunferncia. Considere desprezveis as aes gravitacionais.
Dado: k = 9 10
9
Nm
2
/C
2




a) Determine o mdulo do vetor campo eltrico resultante, criado pelas cargas q
1
e q
2
, no ponto C.
b) Uma terceira carga eltrica, q
3
= 3,0 10
12
C, pontual, descreve o arco . Qual o trabalho realizado, nesse
deslocamento, pela fora eltrica que atua na carga q
3
devido ao das cargas eltricas q
1
e q
2
? Justifique sua
resposta.

10 - (UFPA) Quase todo mundo tem um Tubo de Raios Catdicos (TRC) em casa. Uma das aplicaes mais comuns
de um TRC a gerao de imagens em televiso e em monitores de computador. Como mostrado na figura (I) abaixo,
um TRC basicamente um tubo de vidro (em cujo interior a presso muito baixa), no qual produzido um feixe de
eltrons no catodo (da o nome raios catdicos), que so acelerados pelo anodo, at atingirem a tela fluorescente
localizada direita, produzindo nessa uma rea brilhante ou imagem. Os dois conjuntos de placas de deflexo, por
meio da aplicao de campos eltricos (ou de campos magnticos nos televisores e monitores), servem para deslocar
o feixe de eltrons, respectivamente, nas direes horizontal e vertical. Na figura (II) est destacada a trajetria
seguida pelo feixe de eltrons que ser objeto de nossa anlise. Este feixe abandona o catodo (A), passa pelo anodo
(B) e atravessa a regio das placas de deflexo vertical, onde est aplicado o campo eltrico E, at atingir a tela
fluorescente.



Analisando a figura (II),
a) responda: Aps penetrar na regio entre as placas de deflexo vertical, onde est aplicado o campo eltrico E, o
eltron seguir qual trajetria: a, b ou g ? Justifique.
b) considerando o movimento do feixe de eltrons, calcule, aplicando o Princpio da Conservao da Energia nos
pontos A e B, a velocidade de um eltron ao sair pelo orifcio do anodo acelerador. Use os dados abaixo:


347


11 - (UFRRJ ) Usar g = 10 m/s
2
sempre que necessrio.
Em cada um dos vrtices de um cubo de aresta a h uma carga pontual +q.



Determine
a) o campo eltrico no centro C do cubo;
b) o potencial eltrico no centro C do cubo, tomando 0 ) ( V = .

12 - (UnB DF) Considere quatro cargas puntuais q
1
= 3C, q
2
= 3C, q
3
= 3C e q
4
= -3C, dispostas nos vrtices de um
retngulo imaginrio de lados L
1
= 3m e L
2
= 4m, como representado na figura. Calcule o trabalho, em Joules,
necessrio para afastar as cargas, at que a distncia entre elas seja duplicada. D sua resposta dividindo o valor
encontrado por 9 x 10
9
.

q
1
q
2
q
3
q
4
L
1
L
2


A energia potencial armazenada num sistema de duas cargas eltricas q e q dada por:
d
' kqq
E = , onde k a
constante eletrosttica e d a distncia entre elas.

13 - (UFOP MG) Um prton penetra com energia cintica K = 2,4x10
-16
J numa regio extensa de um campo eltrico
uniforme, cujo intensidade E = 3,0x10
4
N/C. A trajetria descrita retilnea, com a partcula invertendo o sentido do
movimento aps percorrer uma distncia d. Sabendo-se que a massa do prton e m = 1,67x10
-27
kg e que sua carga
q = 1,6x10
-19
C, determine:
a) o valor de d;
b) o tempo gasto para percorrer a distncia d.

14 - (UFOP MG) O campo eltrico em uma dada regio uniforme e tem mdulo E = 100N/C, como mostra a figura
abaixo.
C
A B
0,10m
0,05m
E


a) Determine a diferena de potencial entre os pontos A e B, B e C e A e C.
b) Determine a fora eltrica que age sobre uma carga puntual q = 1x10
-6
C, colocada no ponto A deste campo.

15 - (UFOP MG) Considere o capacitor de placas paralelas abaixo, sendo o campo eltrico no espao
entre as placas E = 6,0 x 10
4
V/m e a distncia entre as placas, 10mm.
a) a diferena de potencial entre os pontos M e N;
b) o trabalho realizado por uma fora externa para transportar, em equilbrio, uma carga Q = 5x10
-6
C do
ponto M ao ponto N segundo a trajetria MLN da figura.
M
L
N
6mm
6mm
6
m
m
6
m
m

348
16 - (UFOP MG) Nas figuras abaixo, esto representados um capacitor de placas paralelas e o grfico da diferena
de potencial entre dois pontos A e B do seu interior em funo da distncia entre eles.
a) Desenhe, no interior do capacitor, uma seta que represente a direo e o sentido do vetor campo eltrico.
b) Calcule o mdulo do vetor campos eltrico.
c) Calcule o trabalho para transportar, em equilbrio, uma carga de prova positiva q = 2,0x10
-9
C, do ponto C at o ponto
D, segundo a trajetria indicada na figura.



17 - (UFPE) Um eltron com energia cintica de 2,4 x 10
16
J entra em uma regio de campo eltrico uniforme, cuja
intensidade 3,0 x 10
4
N/C. O eltron descreve uma trajetria retilnea, invertendo o sentido do seu movimento aps
percorrer uma certa distncia. Calcule o valor desta distncia, em cm.

18 - (UFPE) Uma nuvem eletrizada est situada a 1000m de altura, paralelamente superfcie da Terra, formando
com esta um capacitor plano de 15nF. Quando o campo eltrico no ar (entre a nuvem e a Terra) atinge o valor de 3,0 x
10
6
N/C, ocorre um relmpago. Calcule a carga eltrica, em C, que se encontrava armazenada na nuvem, no instante
da descarga eltrica.

19 - (FUVEST SP) Duas pequenas esferas metlicas, A e B, so mantidas em potenciais eletrostticos constantes,
respectivamente, positivo e negativo. As linhas cheias do grfico a seguir representam as interseces, com o plano do
papel, das superfcies equipotenciais esfricas geradas por A, quando no h outros objetos nas proximidades. De
forma anloga, as linhas tracejadas representam as interseces com o plano do papel, das superfcies equipotenciais
geradas por B. Os valores dos potenciais eltricos dessas superfcies esto indicados no grfico. As questes se
referem situao em que A e B esto na presena uma da outra, nas posies indicadas no grfico, com seus
centros no plano do papel.
NOTE/ADOTE Uma esfera com carga Q gera, fora dela, a uma distncia r do seu centro, um potencial V e um campo
eltrico de mdulo E, dados pelas expresses:
V = K (Q/r)
E = K (Q/r
2
) = V/r
K = constante;
1 volt / metro = 1 newton / coulomb
a) Trace, com caneta, em toda a extenso do grfico a seguir, a linha de potencial V = 0, quando as duas esferas
esto nas posies indicadas. Identifique claramente essa linha por V = 0.
b) Determine, em volt / metro, utilizando dados do grfico, os mdulos dos campos eltricos E
PA
e E
PB
criados, no
ponto P, respectivamente, pelas esferas A e B.
c) Represente, em uma escala conveniente, no grfico, com origem no ponto P, os vetores E
PA
, E
PB
e o vetor campo
eltrico E
P
resultante em P. Determine, a partir desta construo grfica, o mdulo de E
P
, em volt / metro.
d) Estime o mdulo do valor do trabalho t, em joules, realizado quando uma pequena carga q = 2,0nC levada do
ponto P ao ponto S, indicados no grfico.
(2,0nC = 2,0 nanocoulombs = 2,0 x 10
9
C).



349
20 - (FUVEST SP) Duas pequenas esferas, com cargas positivas e iguais a Q, encontram-se fixas sobre um plano,
separadas por uma distncia 2a. Sobre esse mesmo plano, no ponto P, a uma distncia 2a de cada uma das esferas,
abandonada uma partcula com massa m e carga q negativa. Desconsidere o campo gravitacional e efeitos no
eletrostticos.
Determine, em funo de Q, K, q, m e a,

2a
a
2a
a
q P
Q Q
O


a) A diferena de potencial eletrosttico V = V
O
V
P
, entre os pontos O e P.
b) A velocidade v com que a partcula passa por O.
c) A distncia mxima D
max
, que a partcula consegue afastar-se de P. Se essa distncia for muito grande, escreva D
max

= infinito.
Dados:
A fora F entre duas cargas Q
1
e Q
2
dada por F=K Q
1
.Q
2
/r
2
onde r a distncia entre as cargas.
O potencial V criado por uma carga Q, em um ponto P, a uma distncia r da carga , dado por: V=K Q/r.

21 - (UFOP MG) A velocidade da luz no vcuo uma constante universal com valor, aproximadamente, c = 3x10
8
m/s.
Considere uma carga eltrica pontual positiva Q = 10C.
Dada a constante de Coulomb no vcuo k
0
= 9x10
9
Nm
2
/C
2
:
a) Calcule o valor do potencial eltrico medido por um fsico em um ponto a 106m da carga eltrica Q.
b) Se a carga eltrica Q for colocada repentinamente em movimento uniforme, segundo a reta definida pela carga e por
esse fsico, com velocidade v
0
= 1000m/s, afastando-se do fsico, calcule depois de quanto tempo o aparelho de
medida indicar que a carga Q entrou em movimento.
c) Deduza a expresso do potencial eltrico medido por esse fsico, aps a carga eltrica entrar em movimento.

22 - (UNIMONTES MG) Uma casca esfrica de raio R
1
possui uma densidade D
0
(carga por rea), sendo D
0
> 0.
Considere E
0
e V
0
os valores do potencial e do campo eltrico, respectivamente, na superfcie da esfera.



a) Qual o valor de V
0
, em termos de R
1
e D
0
? Faa um grfico para o potencial eltrico, em funo da distncia r at o
centro da esfera, considerando nele 0 s r s R
1
e r > R
1
.
b) Qual o valor de E
0
, em termos de R
1
e D
0
? Faa um grfico para o mdulo do campo eltrico, em funo da distncia
r at o centro da esfera, considerando nele os casos 0 r R
1
e r > R
1
.

23 - (ENEM) Observe o fenmeno indicado na tirinha.

(Adaptado. Luisa Daou & Francisco Caruso, Tirinhas
de Fsica, vol. 2, CBPF, Rio de Janeiro, 2000.)

A fora que atua sobre o peso e produz o deslocamento vertical da
garrafa a fora

a) de inrcia.
b) gravitacional.
c) de empuxo.
d) centrpeta.
e) elstica.




350
24 - (ENEM) A figura mostra o tubo de imagens dos aparelhos de televiso usado para produzir as imagens sobre a
tela. Os eltrons do feixe emitido pelo canho eletrnico so acelerados por uma tenso de milhares de volts e passam
por um espao entre bobinas onde so defletidos por campos magnticos variveis, de forma a fazerem a varredura da
tela.



Nos manuais que acompanham os televisores comum encontrar, entre outras, as seguintes recomendaes:

I. Nunca abra o gabinete ou toque as peas no interior do televisor.
II. No coloque seu televisor prximo de aparelhos domsticos com motores eltricos ou ms.

Estas recomendaes esto associadas, respectivamente, aos aspectos de

a) riscos pessoais por alta tenso / perturbao ou deformao de imagem por campos externos.
b) proteo dos circuitos contra manipulao indevida / perturbao ou deformao de imagem por campos externos.
c) riscos pessoais por alta tenso / sobrecarga dos circuitos internos por aes externas.
d) proteo dos circuitos contra a manipulao indevida / sobrecarga da rede por fuga de corrente.
e) proteo dos circuitos contra manipulao indevida / sobrecarga dos circuitos internos por ao externa.

25 - (ENEM) Segundo um especialista em petrleo (Estado de S. Paulo, 5 de maro de 2000), o consumo total de
energia mundial foi estimado em 8,3 bilhes de toneladas equivalentes de petrleo (tep) para 2001. A porcentagem das
diversas fontes da energia consumida no globo representada no grfico.



Segundo as informaes apresentadas, para substituir a energia nuclear utilizada necessrio, por exemplo, aumentar
a energia proveniente do gs natural em cerca de

a) 10%.
b) 18%.
c) 25%.
d) 33%.
e) 50%.


351







PROF.: J. CNDIDO






C
C
o
o
n
n
t
t
e
e

d
d
o
o
s
s
:
:

C
C
a
a
m
m
p
p
o
o
e
e
l
l

t
t
r
r
i
i
c
c
o
o
u
u
n
n
i
i
f
f
o
o
r
r
m
m
e
e
e
e
c
c
o
o
n
n
d
d
u
u
t
t
o
o
r
r
e
e
s
s
e
e
m
m

e
e
q
q
u
u
i
i
l
l

b
b
r
r
i
i
o
o




M
M
i
i
n
n
i
i
s
s
t
t
r
r
a
a
d
d
o
o
s
s
d
d
u
u
r
r
a
a
n
n
t
t
e
e
o
o
m
m

s
s
d
d
e
e
a
a
b
b
r
r
i
i
l
l















D Da at ta a d da a E En nt tr re eg ga a : : _ __ __ __ __ __ __ __ _/ /_ __ __ __ __ __ __ __ __ __ __ _/ /2 20 01 13 3
LISTA 3 FSICA

352

01 - (UFG GO) Uma mutao no DNA pode alterar as cargas de algumas protenas sem alterar suas massas. Para
separar as protenas normais das mutantes emprega-se a tcnica chamada eletroforese, que consiste, basicamente,
em se aplicar uma diferena de potencial entre duas placas paralelas separadas por uma distncia conhecida. Para
permitir a visualizao da separao das protenas, elas so colocadas em um gel que produz sobre elas uma fora
resistiva proporcional velocidade,


= v b Fres ,

sendo b a constante de proporcionalidade. As protenas atingem suas velocidades terminais, em um intervalo de tempo
desprezvel, e ento percorrem todo o gel com velocidade terminal constante. Aps um tempo de espera, as protenas
encontram-se em diferentes posies no gel, dando a ele uma tonalidade escura.

Um experimento foi realizado aplicando-se uma diferena de potencial de 120 V. Com um tempo de espera igual a 40
minutos obteve-se a separao apresentada na figura abaixo.


No caso em que as protenas mutantes perdem eltrons, determine:
a) a intensidade do campo eltrico aplicado e sua direo;
b) as velocidades terminais das protenas normais e mutantes;
c) a razo entre as cargas eltricas dos dois tipos de protenas.

02 - (MACK SP) Uma pequena esfera de isopor, de massa 0,512 g, est em equilbrio entre as armaduras de um
capacitor de placas paralelas, sujeito s aes exclusivas do campo eltrico e do campo gravitacional local.
Considerando
2
s / m 10 | g | =

, pode-se dizer que essa pequena esfera possui




a) um excesso de 1,010
12
eltrons, em relao ao nmero de prtons.
b) um excesso de 6,410
12
prtons, em relao ao nmero de eltrons.
c) um excesso de 1,010
12
prtons, em relao ao nmero de eltrons.
d) um excesso de 6,410
12
eltrons, em relao ao nmero de prtons.
e) um excesso de carga eltrica, porm, impossvel de ser determinado.
Caderno de Atividades

Disciplina:
Fsica

Professor(a):
J. Cndido
Aluno:
3 ano
Ensino Mdio
Data de Recebimento:
_____/_____/_____
Lista 03

Data Entrega:
_____/_____/_____

353
03 - (FGV) A figura seguinte representa algumas linhas de fora de um campo eltrico uniforme e trs pontos internos
A, B e C desse campo. A reta que passa pelos pontos A e C perpendicular s linhas de fora.



correto afirmar que

a) A e B tm o mesmo potencial eltrico, sendo este maior que o de C.
b) A e B tm o mesmo potencial eltrico, sendo este menor que o de C.
c) A e C tm o mesmo potencial eltrico, sendo este maior que o de B.
d) os potenciais eltricos dos pontos A, B e C guardam a relao V
A
<V
B
<V
C
.
e) os potenciais eltricos dos pontos A, B e C guardam a relao V
A
>V
B
>V
C
.

04 - (UECE) Uma partcula carregada negativamente posta na presena de um campo eltrico de direo vertical,
com sentido de cima para baixo e mdulo constante E, nas proximidades da superfcie da Terra. Denotando-se por g o
mdulo da acelerao da gravidade, a razo entre a carga e a massa da partcula para que haja equilbrio esttico
deve ser

a)
g
E

b)
E
g

c)
g 8 , 9
E

d)
g
E 8 , 9


05 - (UFAC) A figura abaixo, mostra um eltron (e) entrando com velocidade horizontal (v) em uma regio limitada por
duas placas paralelas condutoras com cargas opostas.



Considerando que o peso do eltron desprezvel, e que o campo eltrico entre as placas essencialmente uniforme
e perpendicular s mesmas, correto afirmar que:

a) Quanto menor a velocidade v, mais rapidamente o eltron se aproximar da placa positiva.
b) A velocidade de aproximao do eltron placa positiva independe do valor da velocidade horizontal v.
c) A direo da acelerao do eltron, na regio limitada pelas placas, est mudando ao longo da sua trajetria.
d) O eltron no est acelerado.
e) Quanto maior a velocidade v, mais rapidamente o eltron se aproximar da placa positiva.

06 - (UFG GO) Conecta-se placa positiva de um capacitor de placas paralelas um fio isolante inextensvel, de
comprimento L e massa desprezvel, que tem preso sua extremidade uma bolinha de massa m e carga +q, conforme
ilustra a figura.




354
Sendo E o mdulo do campo eltrico entre as placas e desprezando a resistncia do ar, o perodo de pequenas
oscilaes desse pndulo :

a)
g
L
2 T t =
b)
qE
mL
2 T t =
c)
qE
mL
2
1
T
t
=
d)
mg qE
mL
2
1
T
+ t
=
e)
mg qE
mL
2 T
+
t =

07 - (UEFS BA) A figura representa uma linha de fora, LF, de um campo eletrosttico gerado por cargas eltricas em
repouso.


Durante o deslocamento de uma partcula eletrizada com carga positiva q do ponto 1 para o ponto 2, correto afirmar:

a) O potencial eltrico decrescente no sentido da linha de fora.
b) A fora eltrica resultante F, em cada ponto da trajetria, que age sobre q, tem direo perpendicular linha.
c) A fora resultante sobre a carga nula.
d) As linhas de fora de um campo eltrico, gerado por cargas eltricas em repouso, podem ser linhas fechadas.
e) O trabalho da fora eltrica durante o deslocamento da carga q sobre uma superfcie equipotencial sempre
positivo.

08 - (UESPI) Uma partcula de massa 0,1 kg e carga 10
6
C cai verticalmente numa regio de campo eltrico uniforme
e vertical, de mdulo 10
5
N/C. Desprezando a resistncia do ar e considerando a acelerao da gravidade igual a 10
m/s
2
, os valores mnimo e mximo da acelerao dessa partcula valem:
a) 8 m/s
2
e 10 m/s
2

b) 9 m/s
2
e 10 m/s
2

c) 8 m/s
2
e 12 m/s
2

d) 9 m/s
2
e 11 m/s
2

e) 8 m/s
2
e 9 m/s
2


09 - (UFAC) As clulas so as unidades bsicas da vida. O entendimento do funcionamento delas muito importante
dos pontos de vista fsico e qumico, a fim de saber como funcionam os seres vivos e como eles reagem frente a
diversos estmulos externos. Um dos avanos do ponto de vista fsico foi descoberta da existncia de excesso de
ons positivos, na parede externa, e excesso de ons negativos na parede interna da membrana celular. Essa
descoberta indica que a membrana celular, se comporta, efetivamente, como um capacitor eltrico, que podemos
chamar capacitor celular. Sabe-se, tambm, que a diferena de potencial eltrico entre as paredes da membrana de
uma clula nervosa varia entre 55 mV e 100 mV, para animais de sangue quente. Suponha que o capacitor celular
pode ser aproximado por um capacitor de placas paralelas e que a espessura da membrana celular de 7 nm (1 nm =
10
9
m).
Escolha o item correto:

a) O sentido do campo eltrico no interior da membrana de dentro para fora.
b) Os valores do campo eltrico no interior da membrana encontram-se entre 7,8610
6
V/m e 1,4310
7
V/m.
c) O campo eltrico no interior da membrana celular nulo.
d) O potencial eltrico na parede externa da membrana menor do que o potencial eltrico na parede interna.
e) O potencial eltrico constante no interior da membrana celular, ou seja, na regio limitada entre a parede interna e
a parede externa.

10 - (ITA SP) Considere as cargas eltricas q
1
= 1C, situada em x = 2m, e q
2
= 2C, situada em x = 8m.

Ento, o lugar geomtrico dos pontos de potencial nulo

a) uma esfera que corta o eixo x nos pontos x = 4m e x = 4m.
b) uma esfera que corta o eixo x nos pontos x = 16m e x = 16m.
c) um elipsoide que corta o eixo x nos pontos x = 4m e x = 16m.
d) um hiperboloide que corta o eixo x no ponto x = 4m.
e) um plano perpendicular ao eixo x que o corta no ponto x = 4m.

355
11 - (UECE) Quatro esferas metlicas, com raios R
1
> R
2
> R
3
> R
4
, esto interligadas eletricamente. No conjunto est
distribuda uma carga eltrica Q. A esfera com maior densidade de carga
a) R
1
.
b) R
2
.
c) R
3
.
d) R
4
.

12 - (UFTM) Considere uma esfera oca metlica eletrizada. Na condio de equilbrio eletrosttico,
a) o vetor campo eltrico no interior da esfera no nulo.
b) o potencial eltrico em um ponto interior da esfera depende da distncia desse ponto superfcie.
c) o vetor campo eltrico na superfcie externa da esfera perpendicular superfcie.
d) a distribuio de cargas eltricas na superfcie externa da esfera depende do sinal da carga com que ela est
eletrizada.
e) o mdulo do vetor campo eltrico em um ponto da regio externa da esfera no depende da distncia desse ponto
superfcie.

13 - (UEPG PR) Considere duas esferas condutoras A e B, de raios R e 3R, respectivamente, separadas por uma
distncia d. Inicialmente a esfera A tem carga eltrica lquida nula e a esfera B tem uma carga eltrica lquida 3Q. As
duas esferas so conectadas entre si por meio de um fio condutor que logo aps desconectado das esferas. Com
relao ao estado final das esferas, assinale o que for correto.

01. Todos os excessos de carga nas esferas A e B esto localizados na superfcie das esferas.
02. A esfera A tem carga
4
3
Q e a esfera B tem carga
4
9
Q .
04. O potencial eltrico da esfera A menor do que o potencial eltrico da esfera B.
08. O potencial eltrico no interior das esferas A e B so constantes e iguais entre si.
16. A fora eletrosttica entre as duas esferas k
2
2
d 16
Q 27
.

14 - (UFG GO) Os foges residenciais a gs com acendimento automtico so alimentados por uma ddp de 220 V.
Neles, um circuito eltrico produz uma tenso de 17 kV no faiscador. Com o gs aberto, a chama acende porque a ddp
de
a) 17 kV gera uma corrente eltrica na superfcie do material, que aquece a boca do fogo e a mistura gs-nitrognio.
b) 220 V supera o campo de ruptura do ar e gera uma centelha que fornece calor para a mistura gs-oxignio.
c) 17 kV gera uma corrente eltrica na superfcie do material; essa aquece a boca do fogo que, por sua vez, aquece a
mistura gs-oxignio.
d) 17 kV supera o campo de ruptura do ar e gera uma centelha que fornece calor para a mistura gs-oxignio.
e) 220 V gera uma corrente eltrica no ar condutor, que aquece a mistura gs-nitrognio.

15 - (UEG GO) Um eletrodomstico bastante popular nos dias de hoje o aparelho de micro-ondas que tem como
princpio de funcionamento a produo de micro-ondas para aquecer alimentos. Porm, a radiao produzida dentro do
aparelho no consegue escapar. Com base nesta explicao,
a) explique o que a gaiola de Faraday;
b) d trs exemplos de gaiola de Faraday.

16 - (UPE) Imagine uma pequena gota esfrica de um fluido incompressvel, com uma certa carga que tem um
potencial eletrosttico V em sua superfcie. Se n gotas idnticas e de mesma carga desse fluido se unem para formar
uma gota esfrica maior, qual o potencial eltrico na superfcie da nova gota?

a) V
b) V n
-1

c) V
3
n
d) V
3 2
n
e) V
3 1
n



17 - (UFG GO) A umidade relativa do ar no inverno de 2010 em Goinia atingiu nveis muito baixos. Essa baixa
umidade pode provocar descargas eltricas nas pessoas quando elas aproximam seus dedos de superfcies
condutoras de eletricidade. Considere que a descarga ocorre quando uma pessoa aproxima seu dedo a uma distncia
de 3 mm da superfcie metlica e a carga eltrica na ponta do dedo corresponda metade daquela que deve estar
uniformemente distribuda em uma pequena esfera de raio 6 mm. Nessas condies, a carga acumulada na ponta do
dedo, em Coulomb, ser de
Dados
Campo de ruptura do ar: 3 x 10
6
V/m
k = 9 x 10
9
Nm
2
/C
2

a) 1,50 x 10
9

b) 6,00 x 10
9

c) 1,20 x 10
8

d) 1,35 x 10
8

e) 2,70 x 10
6

356
18 - (UNESP) Uma esfera condutora descarregada (potencial eltrico nulo), de raio R
1
= 5,0 cm, isolada, encontra-se
distante de outra esfera condutora, de raio R
2
= 10,0 cm, carregada com carga eltrica Q = 3,0 C (potencial eltrico
no nulo), tambm isolada.


Em seguida, liga-se uma esfera outra, por meio de um fio condutor longo, at que se estabelea o equilbrio
eletrosttico entre elas. Nesse processo, a carga eltrica total conservada e o potencial eltrico em cada condutor
esfrico isolado descrito pela equao
r
q
k V = , onde k a constante de Coulomb, q a sua carga eltrica e r o seu
raio.


Supondo que nenhuma carga eltrica se acumule no fio condutor, determine a carga eltrica final em cada uma das
esferas.

19 - (UNEB) Alm de dissolver cimento e calcrio e reduzir o pH de lagos e riachos, a chuva cida leva importantes
nutrientes do solo, prejudicando plantas e liberando minerais txicos que podem alcanar hbitats aquticos. Para
combater esse problema quando surgiu pela primeira vez, a Agncia de Proteo Ambiental dos Estados Unidos
conseguiu aprovar, em 1990, alteraes na Lei do Ar Limpo, que cortou em 59% as emisses de compostos sufurados
das fbricas de 1990 a 2008. As emisses de compostos de nitrognio, entretanto, no caram to abruptamente.
De maneira geral, usinas termeltricas a carvo mineral e veculos motorizados expelem a maior parte dos xidos de
nitrognio do pas, a matria-prima para a chuva de cido ntrico, HNO
3
. Mas uma grande porcentagem deles tambm
vem do setor agrcola na forma de amnia, NH
3
, que bactrias convertem a cido ntrico no solo. Os maiores
responsveis so os fabricantes de fertilizantes, que transformam o gs nitrognio no reativo da atmosfera em amnia
por meio do chamado processo de Haber-Bosch. (TENNESEN, 2010).

Sobre a descarga eltrica que desencadeia reaes qumicas entre poluentes lanados por diversas fontes na
atmosfera terrestre, formando compostos cidos que caem em forma de chuva cida, correto afirmar:
01. Os raios caem preferencialmente sobre objetos pontiagudos porque, nas vizinhanas desses objetos, existem
campos eltricos intensos que ionizam o ar dessas regies.
02. A descarga eltrica que ocorre entre partes de uma mesma nuvem resulta da movimentao das cargas eltricas
dos pontos de maior potencial para os de menor potencial.
03. Os raios, formados por cargas eltricas em movimento ordenado, produzem ondas mecnicas que podem ser
ouvidas por observadores que se encontram em repouso no solo.
04. A descarga eltrica entre as nuvens ocorre quando se estabelece, nessa regio, um campo eltrico uniforme de
intensidade menor do que a rigidez dieltrica do ar.
05. O trovo, resultante do efeito trmico das correntes, uma onda transversal que apresenta o fenmeno de
polarizao.

20 - (ENEM) Um sistema de radar programado para
registrar automaticamente a velocidade de todos os veculos
trafegando por uma avenida, onde passam em mdia 300
veculos por hora, sendo 55 km/h a mxima velocidade
permitida. Um levantamento estatstico dos registros do radar
permitiu a elaborao da distribuio percentual de veculos de
acordo com sua velocidade aproximada.

A velocidade mdia dos veculos que trafegam nessa avenida de:
a) 35 km/h
b) 44 km/h
c) 55 km/h
d) 76 km/h
e) 85 km/h

357
21 - (ENEM) Em uma fbrica de equipamentos eletrnicos, cada componente, ao final da linha de montagem,
submetido a um rigoroso controle de qualidade, que mede o desvio percentual d) de seu desempenho em relao a um
padro ideal. O fluxograma a seguir descreve, passo a passo, os procedimentos executados por um computador para
imprimir um selo em cada componente testado, classificando-o de acordo com o resultado do teste:



Os smbolos usados no fluxograma tm os seguintes significados:
Entrada e sada de dados
Deciso (testa uma condio, executando operaes diferentes caso essa condio seja verdadeira ou falsa)
Operao

Segundo essa rotina, se D =1,2%, o componente receber um selo com a classificao

a) Rejeitado, impresso na cor vermelha.
b) 3 Classe, impresso na cor amarela.
c) 3 Classe, impresso na cor azul.
d) 2 Classe, impresso na cor azul.
e) 1 Classe, impresso na cor azul.

22 - (ENEM) Uma fonte de energia que no agride o ambiente, totalmente segura e usa um tipo de matria-prima
infinita a energia elica, que gera eletricidade a partir da fora dos ventos. O Brasil um pas privilegiado por ter o
tipo de ventilao necessria para produzi-la. Todavia, ela a menos usada na matriz energtica brasileira. O
Ministrio de Minas e Energia estima que as turbinas elicas produzam apenas 0,25% da energia consumida no pas.
Isso ocorre porque ela compete com uma usina mais barata e eficiente: a hidreltrica, que responde por 80% da
energia do Brasil. O investimento para se construir uma hidreltrica de aproximadamente US$ 100 por quilowatt. Os
parques elicos exigem investimento de cerca de US$ 2 mil por quilowatt e a construo de uma usina nuclear, de
aproximadamente US$ 6 mil por quilowatt. Instalados os parques, a energia dos ventos bastante competitiva,
custando R$ 200,00 por megawatt-hora frente a R$ 150,00 por megawatt-hora das hidreltricas e a R$ 600,00 por
megawatt-hora das termeltricas.
poca. 21/4/2008 (com adaptaes).

De acordo com o texto, entre as razes que contribuem para a menor participao da energia elica na matriz
energtica brasileira, inclui-se o fato de

a) haver, no pas, baixa disponibilidade de ventos que podem gerar energia eltrica.
b) o investimento por quilowatt exigido para a construo de parques elicos ser de aproximadamente 20 vezes o
necessrio para a construo de hidreltricas.
c) o investimento por quilowatt exigido para a construo de parques elicos ser igual a 1/3 do necessrio para a
construo de usinas nucleares.
d) o custo mdio por megawatt-hora de energia obtida aps instalao de parques elicos ser igual a 1,2 multiplicado
pelo custo mdio do megawatt-hora obtido das hidreltricas.
e) o custo mdio por megawatt-hora de energia obtida aps instalao de parques elicos ser igual a 1/3 do custo
mdio do megawatt-hora obtido das termeltricas.

23 - (ENEM) Observe a situao descrita na tirinha abaixo.


(Francisco Caruso & Luisa Daou, Tirinhas de Fsica, vol. 2, CBPF, Rio de Janeiro, 2000.)


358
Assim que o menino lana a flecha, h transformao de um tipo de energia em outra. A transformao, nesse caso,
de energia
a) potencial elstica em energia gravitacional.
b) gravitacional em energia potencial.
c) potencial elstica em energia cintica.
d) cintica em energia potencial elstica.
e) gravitacional em energia cintica.

TEXTO: 1 - Comum questo: 24
Os Dez Mais Belos Experimentos da Fsica

A edio de setembro de 2002 da revista Physics World apresentou o resultado de uma enquete realizada entre seus
leitores sobre o mais belo experimento da Fsica. Na tabela abaixo so listados os dez experimentos mais votados.

Foucault. de pndulo
do a Experinci ) 10
Young. por realizada
luz, da cia interfern
da o Experiment ) 5
. Rutherford
de o Experiment 9)
Newton. por realizada
prisma, um com solar
luz da o Decomposi 4)
Galileu. por
realizado inclinado, plano
num corpos de movimento
o sobre o Experiment ) 8
Millikan.
por realizada leo, de
gota da o Experiment 3)
es. Eraststen
por realizada Terra, da
ncia circunfer da Medida 7)
Galileu. por realizada corpos, dos
queda da o Experiment 2)
Cavendish. por
realizada
torso, de balana
a com o Experiment ) 6
eltrons. com realizado
Young, de fenda
dupla da o Experiment 1)

24 - (UEG GO) Embora as experincias realizadas por Millikan tenham sido muito trabalhosas, as ideias bsicas nas
quais elas se apoiam so relativamente simples. Simplificadamente, em suas experincias, R. Millikan conseguiu
determinar o valor da carga do eltron equilibrando o peso de gotculas de leo eletrizadas, colocadas em um campo
eltrico vertical e uniforme, produzido por duas placas planas ligadas a uma fonte de voltagem, conforme ilustrado na
figura abaixo.

Carga do eltron (em mdulo) e = 1,6 10
19
C
g = 10 m/s
2

Supondo que cada gotcula contenha cinco eltrons em excesso, ficando em
equilbrio entre as placas separadas por d = 1,50 cm e submetendo-se a uma
diferena de potencial V
AB
= 600 V, a massa de cada gota vale, em kg:
a) 1,6x10
15

b) 3,2x10
15

c) 6,4x10
15

d) 9,6x10
15


TEXTO: 2 - Comum questo: 25

25 - (Unifacs BA)
Com base na figura, que representa as trajetrias dos feixes
descritos pelas partculas o e | e pelos raios , ao serem lanados
na regio de um campo eltrico uniforme gerado pelas placas
paralelas eletrizadas, correto afirmar:

01. As partculas | so eletrizadas positivamente.
02. As partculas o so dotadas de cargas eltricas negativas.
03. As trajetrias descritas pelas partculas o e | so linhas
equipotenciais do campo eletrosttico.
04. Os raios so ondas eletromagnticas que possuem caractersticas semelhantes das ondas de rdio.
05. As partculas beta, cerca de sete mil vezes mais leves do que as partculas alfa, atingem a chapa fotogrfica com a
mesma velocidade que as partculas alfa.

359







PROF.: J. CNDIDO



C
C
o
o
n
n
t
t
e
e

d
d
o
o
s
s
:
:

C
C
a
a
p
p
a
a
c
c
i
i
t
t
o
o
r
r
e
e
s
s







M
M
i
i
n
n
i
i
s
s
t
t
r
r
a
a
d
d
o
o
s
s
d
d
u
u
r
r
a
a
n
n
t
t
e
e
o
o
m
m

s
s
d
d
e
e
m
m
a
a
i
i
o
o














D Da at ta a d da a E En nt tr re eg ga a : : _ __ __ __ __ __ __ __ _/ /_ __ __ __ __ __ __ __ __ __ __ _/ /2 20 01 13 3
LISTA 4 FSICA

360

01 - (FEI SP) Dispem-se de cinco condensadores de capacidade C = 1F cada um. Eletriza-se um deles com uma
diferena de potencial de 1000 volts. Coloca-se o segundo em paralelo com o primeiro, separando-os. Depois o 3
o
em
paralelo com o 2
o
, separando-os. Assim sucessivamente, at colocar o 5
o
em paralelo com o 4
o
. Calcular as cargas Q
1
,
Q
2
, Q
3
, Q
4
e Q
5
dos condensadores.

02 - (UFC CE) Dois capacitores desconhecidos so ligados em srie a uma bateria de fora eletromotriz c , de modo
que a carga final de cada capacitor q. Quando os mesmos capacitores so ligados em paralelo mesma bateria, a
carga total final da associao 4q. Determine as capacitncias dos capacitores desconhecidos.

03 - (UFES) No interior de um veculo espacial, encontramos dois capacitores isolados de placas finas planas
paralelas, com capacitncia C
1
= 10 F, C
2
= 30 F e cargas Q
1
= 1 C, Q
2
= 3 C, respectivamente. A distncia entre as
placas para cada um dos capacitores d = 1 mm. Aps o lanamento, esse veculo apresenta um vetor velocidade
constante de mdulo 36.000 km/h e de direo paralela ao vetor distncia d entre as placas. Sabendo que as placas
planas paralelas dos capacitores so perpendiculares ao vetor velocidade, determine

a) a capacitncia total do sistema antes do lanamento, quando se associam os capacitores em paralelo;
b) a tenso entre as placas do capacitor com carga Q
1
antes do lanamento;
c) a capacitncia C
2
, aps o lanamento, para um observador fixo na terra;
d) a velocidade do foguete para que a capacitncia de C
1
aumente em 2%.
Se necessrio, use
2
2 2
1
1
1
1
x
x
+ ~

, para x
2
<< 1.

04 - (ITA SP) Um capacitor de capacitncia igual a 0,25 10
6
F carregado at um potencial de 1,00 . 10
5
V, sendo
ento descarregado at 0,40 . 10
5
V num intervalo de tempo de 0,10s, enquanto transfere energia para um
equipamento de raios-X. A carga total, Q, e a energia, e, fornecidas ao tubo de raios-X, so melhor representadas
respectivamente por:
a) Q = 0,005C e e = 1250J
b) Q = 0,025C e e = 1250J
c) Q = 0,025 C e e = 1050J
d) Q = 0,015C e e = 1250J
e) Q = 0,015C e e = 1050J

05 - (UFMT) Em equipamentos eletrnicos, como a televiso, encontram-se diversos elementos de circuito eltrico.
Assinale a alternativa que apresenta as funes dos resistores, capacitores e indutores, respectivamente.
a) Queda de potencial, produo de campo magntico e produo de campo eltrico.
b) Variao do potencial, dissipao do campo eltrico e amplificao da corrente eltrica.
c) Diviso de corrente, conservao da carga e conservao da energia.
d) Dissipao de energia, armazenamento de carga e produo de campo magntico.
e) Distribuio da carga, diviso da corrente e produo de energia eltrica.

06 - (IME RJ) A figura acima ilustra um cubo de madeira parcialmente submerso em um lquido
de densidade . Sua face superior est coberta por uma placa metlica quadrada P
1
. Uma placa
idntica P
2
, fixada em um supor-te, forma com a primeira um capacitor de placas paralelas. As
placas esto carregadas com uma carga Q, havendo entre elas uma capacitncia C e uma
tenso eltrica V, armazenando o capacitor uma energia E. Se o lquido for substitudo por igual
quantidade de outro com densidade maior, a capacitncia (I) , a tenso entre as placas (II)___ e
a energia armazenada (III) .
A opo que corresponde ao preenchimento correto das lacunas (I), (II) e (III) :
(I) (II) (III)
a) aumenta aumenta aumenta
b) aumenta diminui aumenta
c) aumenta diminui diminui
d) diminui aumenta aumenta
e) diminui diminui diminui

Caderno de Atividades

Disciplina:
Fsica

Professor(a):
J. Cndido
Aluno:
3 ano
Ensino Mdio
Data de Recebimento:
_____/_____/_____
Lista 04

Data Entrega:
_____/_____/_____

361
07 - (IME RJ) Um raio luminoso incide ortogonalmente no ponto central de um espelho plano quadrado MNPQ,
conforme a figura abaixo. Girando-se o espelho de um certo ngulo em torno da aresta PQ, consegue-se que o raio
refletido atinja a superfcie horizontal S paralela ao raio incidente. Com a seqncia do giro, o ponto de chegada em S
aproxima-se da aresta PQ.
No ponto de chegada em S que fica mais prximo de PQ est um sensor que, ao ser atingido pelo raio refletido, gera
uma tenso eltrica U proporcional distncia d entre o referido ponto e aquela aresta: U = kd.
Fixando o espelho na posio em que a distncia d mnima, aplica-se a tenso U aos terminais A e B do circuito.
Dado que todos os capacitores esto inicialmente descarregados, determine a energia que ficar armazenada no
capacitor C
3
se a chave Y for fechada e assim permanecer por um tempo muito longo.

Dados: comprimento PQ = 6m ;
constante k = 12 V/m.

08 - (UFLA MG) A energia armazenada num capacitor de 10.000 F, submetido a uma diferena de potencial de 16V, se
descarrega num motor sem atrito, arranjado para erguer um bloco de 0,10 kg de massa. Qual a altura mxima atingida
pelo bloco, em metros? (Adote g = 10 m/s
2
)
a) 0,24 m
b) 1,56 m
c) 1,28 m
d) 2,15 m
e) 3,21 m

09 - (PUC RJ) Dois condutores A e B so esfricos e concntricos. O condutor A macio e tem raio 2cm e carga 5
C; o condutor B, ligado Terra, tem raio interno 4 cm e raio externo 5 cm. Um condutor C, inicialmente neutro,
aproximado do condutor B, sem toc-lo. Nestas condies, podemos afirmar que, aps a aproximao do condutor C:

A
B
C


a) a carga do condutor A passa a ser nula.
b) a carga total do condutor B nula.
c) a carga induzida no condutor C de +5 C.
d) a carga induzida no condutor C nula.
e) a carga induzida no condutor C de 5 C.

10 - (UNIFOR CE) Os capacitores C
1
e C
2
, representados no esquema, tm placas de mesma rea, mas em C
1
a
distncia entre as placas d, e em C
2
a distncia entre as placas d/2

Nessas condies, sendo Q a carga do capacitor C
1
, a carga do capacitor
C
2

a) Q/4
b) Q/2
c) Q
d) 2Q
e) 4Q



362
11 - (UFLA MG) A energia armazenada num capacitor de 10.000 F, submetido a uma diferena de potencial de 16V, se
descarrega num motor sem atrito, arranjado para erguer um bloco de 0,10 kg de massa. Qual a altura mxima atingida
pelo bloco, em metros? (Adote g = 10 m/s
2
)
a) 0,24 m
b) 1,56 m
c) 1,28 m
d) 2,15 m
e) 3,21 m

12 - (UFJF MG) Um capacitor de placas planas e paralelas, isolado a ar, carregado por uma bateria. Em seguida o
capacitar desligado da bateria e a regio entre as placas preenchida com leo isolante. Sabendo-se que a
constante dieltrica do leo maior do que a do ar, pode-se afirmar que:
a) a carga do capacitar aumenta e a ddp entre as placas diminui;
b) a capacitncia do capacitor aumenta e a ddp entre as placas diminui;
c) a capacitncia do capacitor diminui e a ddp entre as placas aumenta;
d) a carga do capacitor diminui e a ddp entre as placas aumenta.

13 - (UEM PR) Considere um capacitor plano de placas paralelas carregado, contendo ar como dieltrico, e cuja
capacitncia vale 8F. Desprezando os efeitos de borda, correto afirmar que :
01. a capacitncia no se altera, quando se introduz um dieltrico de constante dieltrica 7,5 entre as placas do
capacitor.
02. o capacitor ter uma carga de 160C, quando estiver submetido a uma d.d.p. de 20 volts.
04. a unidade de capacidade do capacitor pode ser dada por (coulomb)
2
/joule .
08. a intensidade do campo eltrico entre as placas do capacitor carregado variar, em mdulo, direo e sentido.
16. a capacitncia aumentar, quando for introduzido um dieltrico de constante dieltrica 12 que preencha todo o
espao entre as placas, e o sistema fique submetido mesma d.d.p. inicial.

14 - (UFJF MG) A cpula de um gerador Van de Graaff constituda de uma casca esfrica de raio 10 cm. Deixa-se o
gerador ligado at que sua cpula adquira carga de 6 x 10
-8
C e fique em equilbrio eletrosttico. Uma carga de prova
de 10
-9
C colocada no centro da cpula do gerador. A respeito da fora eletrosttica e do potencial a que a carga de
prova fica submetida, podemos afirmar que seus mdulos so, respectivamente:
a) 5,4 x 10
-5
N ; 5,4 x 10
3
V;
b) zero; 5,4 x 10
3
V;
c) 5,4 x 10
-5
N; depende da localizao do ponto;
d) zero; zero.

15) Qual deveria ser o raio de uma esfera condutora, imersa no vcuo, para que sua capacitncia fosse a 1 farad?
Nota: K
o
= 9 . 10
9
unidades do S.I.

16) A capacitncia eletrosttica de um condutor esfrico imerso no vcuo C = 100nF. O seu potencial eletrosttico
V = 1,00 . 10
4
volts.
Estando ele isolado, determinar:
Dado: K
0
= 9 . 10
9
unidades S.I.
a) seu raio
b) sua carga eltrica
c) a energia potencial eletrosttica armazenada
d) o grfico da sua carga em funo de seu potencial, quando ambos variam.

17 - (PUCCAMP SP) Trs esferas E1, E2, E3, bem afastadas entre si, tm seus raios na razo 1:3:5 e esto
eletrizadas com cargas respectivamente iguais a 3C, 6C e 18C.
Supondo que as esferas sejam ligadas entre si por fios de capacidade desprezvel, as cargas de E1, E2 e E3, aps a
ligao, sero respectivamente iguais a:
a) 1C, 3C, 5C
b) 5C, 3C, 1C
c) 10C, 6C, 2C
d) 2C, 6C, 10C
e) 5C, 3C, 1C

18 - (FEI SP) Duas esferas condutoras de raio R
1
= 10cm e R
2
= 15cm esto eletrizadas, no vcuo, e os seus
potenciais so respectivamente V
1
= 1000V e v
2
= 2000V. As esferas so colocadas em contato e depois afastadas
uma da outra. Qual o novo potencial de cada esfera?




363
19 - (ITA SP) Uma diferena de potencial eletrosttico V estabelecida entre os pontos M e Q da rede cbica de
capacitores idnticos mostrada na figura. A diferena de potencial entre os pontos N e P



a) V/2.
b) V/3.
c) V/4.
d) V/5.
e) V/6.

20 - (UFRR) Considere a associao de capacitadores conforme mostrado na figura abaixo.



Os capacitores tm capacitncia de C
1
= C
2
= C
3
= 6 F alimentados por uma bateria de 12 V, a carga nos capacitores
:

a) Q
1
= 36 C, Q
2
= 72 C e Q
3
= 36 C
b) Q
1
= 36 C, Q
2
= 36 C e Q
3
= 36 C
c) Q
1
= 72 C, Q
2
= 36 C e Q
3
= 36 C
d) Q
1
= 36 C, Q
2
= 36 C e Q
3
= 72 C
e) Q
1
= 72 C, Q
2
= 36 C e Q
3
= 18 C

21 - (UECE) Trs capacitores, de placas paralelas, esto ligados em paralelo. Cada um deles tem armaduras de rea
A, com espaamento d entre elas. Assinale a alternativa que contm o valor da distncia entre as armaduras, tambm
de rea A, de um nico capacitor, de placas paralelas, equivalente associao dos trs.
a)
3
d

b) 3d
c)
2
d 3

d)
3
d 2


22 - (UFU MG) Os circuitos abaixo so formados por capacitores idnticos associados de diferentes formas, conforme
figura. Esses circuitos, designados por A, B e C, so todos submetidos mesma diferena de potencial V.


Considerando que U
A
,U
B
e U
C
so respectivamente as energias totais dos circuitos A, B e C, pode-se afirmar que:
a) U
C
> U
A
> U
B
.
b) U
A
> U
C
> U
B
.
c) U
A
> U
C
< U
B
.
d) U
C
< U
B
> U
A
.


364
23 - (ENEM) Para medir o tempo de reao de uma pessoa, pode-se realizar a seguinte experincia:

I. Mantenha uma rgua (com cerca de 30 cm) suspensa verticalmente, segurando-a pela extremidade superior, de
modo que o zero da rgua esteja situado na extremidade inferior.
II. A pessoa deve colocar os dedos de sua mo, em forma de pina, prximos do zero da rgua, sem toc-la.
III. Sem aviso prvio, a pessoa que estiver segurando a rgua deve solt-la. A outra pessoa deve procurar segur-la o
mais rapidamente possvel e observar a posio onde conseguiu segurar a rgua, isto , a distncia que ela percorre
durante a queda.

O quadro seguinte mostra a posio em que trs pessoas conseguiram segurar a rgua e os respectivos tempos de
reao.

2009. fev. 1 : em Acesso ties.com. //br.geoci : http
: em Disponvel
0,14 0,10
0,17 0,15
0,24 0,30
(segundo)
reao de Tempo
(metro) queda a durante
rgua pela percorrida Distncia


A distncia percorrida pela rgua aumenta mais rapidamente que o tempo de reao porque a

a) energia mecnica da rgua aumenta, o que a faz cair mais rpido.
b) resistncia do ar aumenta, o que faz a rgua cair com menor velocidade.
c) acelerao de queda da rgua varia, o que provoca um movimento acelerado.
d) fora peso da rgua tem valor constante, o que gera um movimento acelerado.
e) velocidade da rgua constante, o que provoca uma passagem linear de tempo.

24 - (ENEM) Uma das modalidades presentes nas olimpadas o salto com vara. As etapas de um dos saltos de um
atleta esto representadas na figura:



Desprezando-se as foras dissipativas (resistncia do ar e atrito), para que o salto atinja a maior altura possvel, ou
seja, o mximo de energia seja conservada, necessrio que

a) a energia cintica, representada na etapa I, seja totalmente convertida em energia potencial elstica representada
na etapa IV.
b) a energia cintica, representada na etapa II, seja totalmente convertida em energia potencial gravitacional,
representada na etapa IV.
c) a energia cintica, representada na etapa I, seja totalmente convertida em energia potencial gravitacional,
representada na etapa III.
d) a energia potencial gravitacional, representada na etapa II, seja totalmente convertida em energia potencial
elstica, representada na etapa IV.
e) a energia potencial gravitacional, representada na etapa I, seja totalmente convertida em energia potencial elstica,
representada na etapa III.


365
25 - (ENEM) Considere os seguintes acontecimentos ocorridos no Brasil:

- Gois, 1987 - Um equipamento contendo csio radioativo, utilizado em medicina nuclear, foi encontrado em um
depsito de sucatas e aberto por pessoa que desconhecia o seu contedo. Resultado: mortes e conseqncias
ambientais sentidas at hoje.
- Distrito Federal, 1999 - Cilindros contendo cloro, gs bactericida utilizado em tratamento de gua, encontrados em
um depsito de sucatas, foram abertos por pessoa que desconhecia o seu contedo. Resultado: mortes, intoxicaes e
conseqncias ambientais sentidas por vrias horas.

Para evitar que novos acontecimentos dessa natureza venham a ocorrer, foram feitas as seguintes propostas para a
atuao do Estado:

I. Proibir o uso de materiais radioativos e gases txicos.
II. Controlar rigorosamente a compra, uso e destino de materiais radioativos e de recipientes contendo gases txicos.
III. Instruir usurios sobre a utilizao e descarte destes materiais.
IV. Realizar campanhas de esclarecimentos populao sobre os riscos da radiao e da toxicidade de determinadas
substncias.

Dessas propostas, so adequadas apenas

a) I e II.
b) I e III.
c) II e III.
d) I, III e IV.
e) II, III e IV.

366






PROF.: J. CNDIDO







C
C
o
o
n
n
t
t
e
e

d
d
o
o
s
s
:
:

E
E
l
l
e
e
t
t
r
r
o
o
d
d
i
i
n
n

m
m
i
i
c
c
a
a





M
M
i
i
n
n
i
i
s
s
t
t
r
r
a
a
d
d
o
o
s
s
d
d
u
u
r
r
a
a
n
n
t
t
e
e
o
o
m
m

s
s
d
d
e
e
j
j
u
u
n
n
h
h
o
o














D Da at ta a d da a E En nt tr re eg ga a : : _ __ __ __ __ __ __ __ _/ /_ __ __ __ __ __ __ __ __ __ __ _/ /2 20 01 13 3

LISTA 5 FSICA

367


01 - (UFPR) Considere que uma pessoa tome diariamente um banho de t minutos de durao e que utilize para isso
um chuveiro eltrico de P W de potncia. Determine, desprezando na resposta os centavos que excederem ao nmero
inteiro em reais, o quanto essa pessoa ir pagar pela energia eltrica consumida em 30 dias, caso 1 kWh custe R$ R.

02 - (UFF RJ) Um motorista acende os dois faris (F
1
e F
2
) e as quatro lanternas (L
1
, L
2
, L
3
e L
4
) de um automvel com
o motor desligado. Todos os elementos esto ligados bateria de 12 V, conforme ilustra o esquema a seguir.

. .
. . .
. . . . .
.
F
1
F
2
L
1
L
2
L
3
L
4

Os valores nominais de potncia e ddp das lmpadas so, para os faris, respectivamente, 40 W e 12 V e para as
lanternas, 6,0 W e 12 V.
Nesta situao, determine:
a) as intensidades das correntes que passam no farol F
1
e na lanterna L
1
;
b) a resistncia do farol F
1
;
c) a intensidade da corrente total que atravessa a bateria.

03 - (UFF RJ) O circuito da figura mostra uma forte varivel de tenso E alimentando um circuito com cinco resistores,
sendo um deles de resistncia R, varivel. Um ampermetro A e um voltmetro V, ideais, so colocados como indicado.
A fonte de tenso varia at, no mximo, 40 volts. Sabe-se que a tenso E e a resistncia R variam de modo a manter a
leitura no voltmetro sempre igual a 10 volts.

a) Para qualquer valor de E e R, qual a leitura no ampermetro ?
b) Expresse E em funo de R.
c) Trace o grfico de E x R at o valor mximo de E.
d) Para quais valores de E e R ser mxima a potncia dissipada por R?

04 - (UFF RJ) Determine o valor, em Ohms, da resistncia R no circuito
da figura abaixo, para que a potncia total dissipada seja igual a 20 W.

05 - (UNESP) Trs resistores idnticos, cada um com resistncia R, e uma pilha de 1,5V e resistncia interna
desprezvel so ligados como mostra a figura.

a) Determine a diferena de potencial entre A e B.
b) Supondo R = 100O, determine a capacidade da corrente eltrica que passa pela pilha.







Caderno de Atividades

Disciplina:
Fsica

Professor(a):
J. Cndido
Aluno:
3 ano
Ensino Mdio
Data de Recebimento:
_____/_____/_____
Lista 05

Data Entrega:
_____/_____/_____
1,5V
R
R
A
R
B

368
06 - (UNESP) Os grficos na figura mostram o comportamento da corrente em dois resistores, R
1
e R
2
, em funo da
tenso aplicada.

4 8 12
v(v)
0,20
0,40
i(A)
R
R
1
2
0
0

a) Considere uma associao em srie desses dois resistores, ligada a uma bateria. Se a tenso no resistor R
1
for
igual a 4 V, qual ser o valor da tenso em R
2
?
b) Considere, agora, uma associao em paralelo desses dois resistores, ligada a uma bateria. Se a corrente que
passa pelo resistor R
1
for igual a 0,30 A, qual ser o valor da corrente por R
2
?

07 - (ITA SP) Sabe-se que a mxima transferncia de energia de uma bateria ocorre quando a resistncia do circuito
se iguala resistncia interna da bateria, isto , quando h o casamento de resistncias. No circuito da figura, a
resistncia de carga R
c
varia na faixa O s s O 400 R 100
c
. O circuito possui um resistor varivel, R
x
, que usado para o
ajuste da mxima transferncia de energia. Determine a faixa de valores de R
x
para que seja atingido o casamento de
resistncias do circuito.


08 - (UnB DF) Uma pessoa utiliza aproximadamente 0,7 kWh de energia eltrica para tomar um banho de 10 minutos
com um chuveiro eltrico. Se a energia eltrica utilizada for proveniente de uma usina hidreltrica, pode-se dizer que a
quantidade de gua mobilizada em virtude de um banho bem maior que aquela que flui pelo chuveiro. No caso da
usina de Itaipu, cujo lago comporta um volume de 2,9 x 10
10
m
3
, cada 10.800 m
3
de gua que flui pelas turbinas
fornece 10,6GJ (12,6 x 10
9
J) de energia. Suponha que essa energia seja utilizada unicamente para alimentar o
funcionamento dos chuveiros eltricos existentes e que cada habitante, de uma populao de 160 milhes, utilize o
chuveiro para tomar um banho dirio de 10 minutos. Nessas condies, considerando que a capacidade geradora
dessa usina seja independente do volume de gua no reservatrio e que nenhuma gua seja adicionada a ele, calcule
o nmero de dias que seriam necessrios para esvaziar completamente o lago de Itaipu, supondo-o inicialmente cheio.
Despreze, caso exista, a parte fracionria do valor calculado.

09 - (UnB DF) Tipicamente, uma pessoa utiliza 100L de gua para tomar um banho dirio em um chuveiro eltrico que
aquece a gua em 10C. Admita que a energia eltrica utilizada para o aquecimento seja gerada na usina de Itaipu,
onde cada 10,8 x 10
6
L de gua que verte, pelas turbinas geram 1,26 x 10
6
kJ de energia eltrica. Considerando o calor
especfico da gua igual a 4,2 kJ x kg
-1
x (C)
-1
e a densidade da gua igual a 1kg/L, calcule uma das seguintes
quantidades, desprezando, para a marcao na Folha de Respostas, a parte fracionria do resultado final obtido aps
efetuar todos os clculos solicitados.
a) a energia, em kJ, consumida no banho, dividindo a quantidade calculada por 10.
b) O volume de gua utilizada em Itaipu, em m
3
, para aquecer a gua do banho mencionado.

10 - (UFG GO) O PRA-RAIO PRA O RAIO
Uma nuvem eletrizada se descarrega atravs de um pra-raio de cobre. O fenmeno dura 10
4
segundos e funde cerca
de 500g de cobre, inicialmente a 30C.
a) Considerando a temperatura de fuso do cobre igual a 1100C, o calor especfico mdio do cobre igual a 0,080
cal/gC
4
o calor latente de fuso a 43 cal/g e que 1 cal = 4,2 J, qual a energia em joules desprendida para aquecer
e fundir esta massa de cobre?
b) Qual a potncia mdia da descarga?
c) Quantas lmpadas de 100W poderiam ser acendidas, com luminosidade total, com esta energia desprendida?




369
11 - (UFOP MG) As figuras abaixo mostram os diagramas tenso versus corrente para dois condutores I e II.

1,5
1,0
0,5
0
3,5 7,5 10,5
V(V)
i(A)
(I)

1,0
0
V(V)
i(A)
(II)
0,5
1,0 2,0 3,0 4,0 5,0

a) Qual dos dois condutores obedece a lei de Ohm? Determine a resistncia eltrica deste condutor.
b) Os dois condutores so ligados em srie a uma bateria de fora eletromotriz c. Se a diferena de potencial no
condutor II de 5,0V, determine a fora eletromotriz c da bateria.

12 - (UFOP MG) Um laboratrio de homologao recebe, para anlise, uma bateria B. O tcnico do laboratrio faz a
montagem mostrada na figura, composta, alm da bateria B, por um reostato R, um argumento A e um voltmetro V.


Para um determinado valor da resistncia do reostato, o voltmetro mede a diferena de potencial eltrico V
ab
entre os
pontos a e b, e o ampermetro mede o valor da corrente eltrica fornecida pela bateria. Diminuindo o valor da
resistncia, o tcnico verificou que a corrente lida no ampermetro aumentava, enquanto a diferena de potencial
eltrico diminua. Ento, construiu a tabela abaixo com os resultados de suas observaes.

I (ampres) 0,0 2,0 4,0 6,0 8,0 10,0
V
ab
(Volts) 10,0 12,0 8,0 6,0 4,0 2,0

a) Desenhe, na figura quadriculada abaixo, o grfico da diferena de potencial eltrico V
ab
em funo da corrente
eltrica i.
b) Determine o valor da fora eletromotriz da bateria.
c) Calcule o valor da resistncia interna da bateria.

13 - (UFSC) Qual o valor, em ohms, da resistncia equivalente R
AB
, da associao de resistores abaixo
representada:
10O
A
B
2O 6O
6O 2O 3O
20O
6O


14 - (UFU MG) Liga-se os terminais de uma fonte E
V
varivel de resistncia interna r = 10O uma folha de grama.
Mantendo-se os terminais nas extremidades da folha, procura-se medir a resistncia R que ela oferece passagem de
corrente eltrica. Ao se variar E
V
, a corrente que passa pela folha de grama medida por um ampermetro, obtendo-se
os seguintes resultados:

Situao E
V
(volts) i(A)
1 2,70 3 x 10
3

2 4,50 5 x 10
3


Determine:
a) A diferena de potencial (d.d.p.) fornecida pela fonte folha em cada situao (1 e 2).
b) A resistncia apresentada pela folha em cada situao (1 e 2).
c) se essa folha um resistor hmico.



370
15 - (UFRRJ) Um chuveiro quando ligado a uma diferena de potencial constante de 110V, desenvolve uma potncia
de 2200W. Este mesmo chuveiro fica ligado nesta ddp todos os dias durante, apenas, 10 minutos. Neste caso:
a) qual a energia, em joule, consumida por este chuveiro durante 10 minutos?
b) se 1KWh custa R$ 0,20 (vinte centavos), qual a despesa em reais com este chuveiro durante em ms (30 dias)?

16 - (UFRRJ) Se um eletricista tem 100 lmpadas, de 100W e 100V, para iluminar uma festa junina, pergunta-se:
a) qual a resistncia eltrica de cada lmpada, quando acesa?
b) qual a resistncia equivalente das lmpadas, caso estas sejam associadas em paralelo?

17 - (UFRJ) O fabricante de cerveja e fsico amador James Joule estimou, em meados do sc. XIX, a diferena entre
a temperatura da gua no sop e no topo das Cataratas de Nigara.
A fim de fazer uma estimativa similar para uma das quedas de Iguau, com altura de 84 m, considere que o modulo da
velocidade com que a gua corre no sop, aps a queda, igual ao modulo da velocidade com que a gua corre no
topo, antes de iniciar a queda. Considere tambm que toda energia mecnica perdida pela gua reabsorvida na
forma de calor, e que provoca seu aquecimento.

Calcule a diferena ente a temperatura da gua no sop e no topo desta queda (o calor especfico da gua 1,0
cal/gC e 1,0 cal = 4,2 J)

18 - (UFSC) Um fio de ferro com 10,0 metros de comprimento e rea da seo transversal igual a 0,01 m, dissipa
uma potncia de 0,81 watts. Sabendo que a resistividade do ferro 10,0 10
-8
O.m, determine, em ampre, a corrente
que circula no resistor

19 - (ITA SP) A figura representa o esquema simplificado de um circuito eltrico em uma instalao residencial. Um
gerador bifsico produz uma diferena de potencial (d.d.p) de 220 V entre as fases ( 110 V e 110 V) + e uma ddp de
110 V entre o neutro e cada uma das fases. No circuito esto
ligados dois fusveis e trs aparelhos eltricos, com as respectivas
potncias nominais indicadas na figura.

Admitindo que os aparelhos funcionam simultaneamente durante
duas horas, calcule a quantidade de energia eltrica consumida em
quilowatt-hora (kWh) e, tambm, a capacidade mnima dos fusveis,
em ampre.

20 - (UNICAMP SP) Uma lmpada incandescente (100W, 120V) tem um filamento de tugstnio de comprimento igual a
31,4 cm e dimetro 4,0x10
-2
mm. A resistividade do tungstnio temperatura ambiente de 5,6x10
-8
O.m.
a) Qual a resistncia do filamento quando ele est temperatura ambiente?
b) Qual a resistncia do filamento com a lmpada acesa?

21 - (ENEM)Os nmeros de identificao utilizados no cotidiano (de contas bancrias, de CPF, de Carteira de
Identidade etc) usualmente possuem um dgito de verificao, normalmente representado aps o hfen, como em
17326-9. Esse dgito adicional tem a finalidade de evitar erros no preenchimento ou digitao de documentos.
Um dos mtodos usados para gerar esse dgito utiliza os seguintes passos:
- multiplica-se o ltimo algarismo do nmero por 1, o penltimo por 2, o antepenltimo por 1, e assim por diante,
sempre alternando multiplicaes por 1 e por 2.
- soma-se 1 a cada um dos resultados dessas multiplicaes que for maior do que ou igual a 10.
- somam-se os resultados obtidos .
- calcula-se o resto da diviso dessa soma por 10, obtendo-se assim o dgito verificador.

O dgito de verificao fornecido pelo processo acima para o nmero 24685

a) 1.
b) 2.
c) 4.
d) 6.
e) 8.

371

22 - (ENEM) O uso mais popular de energia solar est associado ao fornecimento de gua quente para fins
domsticos.
Na figura ao lado, ilustrado um aquecedor de gua constitudo de dois tanques pretos dentro de uma caixa
termicamente isolada e com cobertura de vidro, os quais absorvem energia solar.

A. Hinrichs e M. Kleinbach. Energia e meio ambiente. So Paulo:
Thompson, 3. ed., 2004, p. 529 (com adaptaes).

Nesse sistema de aquecimento,
a) os tanques, por serem de cor preta, so maus absorvedores de calor e reduzem as perdas de energia.
b) a cobertura de vidro deixa passar a energia luminosa e reduz a perda de energia trmica utilizada para o
aquecimento.
c) a gua circula devido variao de energia luminosa existente entre os pontos X e Y.
d) a camada refletiva tem como funo armazenar energia luminosa.
e) o vidro, por ser bom condutor de calor, permite que se mantenha constante a temperatura no interior da caixa.

23 - (ENEM) Podemos estimar o consumo de energia eltrica de uma casa considerando as principais fontes desse
consumo.
Pense na situao em que apenas os aparelhos que constam da tabela abaixo fossem utilizados diariamente da
mesma forma.

Tabela: A tabela fornece a potncia e o tempo efetivo
de uso dirio de cada aparelho domstico.
6 0,10 Lmpadas
10 0,35 Geladeira
10 0,2 Freezer
1/3 3,3 eltrico Chuveiro
8 1,5 do condiciona Ar
(horas) dirio
uso de Tempo
(KW) Potncia Aparelho


Supondo que o ms tenha 30 dias e que o custo de 1 KWh de R$ 0,40, o consumo de energia eltrica mensal dessa
casa, de aproximadamente
a) R$ 135.
b) R$ 165.
c) R$ 190.
d) R$ 210.
e) R$ 230.


372
24 - (ENEM)
H estudos que apontam razes econmicas e ambientais para que o gs natural possa vir a tornar-se, ao longo deste
sculo, a principal fonte de energia em lugar do petrleo. Justifica-se essa previso, entre outros motivos, porque o gs
natural

a) alm de muito abundante na natureza um combustvel renovvel.
b) tem novas jazidas sendo exploradas e menos poluente que o petrleo.
c) vem sendo produzido com sucesso a partir do carvo mineral.
d) pode ser renovado em escala de tempo muito inferior do petrleo.
e) no produz CO
2
em sua queima, impedindo o efeito estufa.

TEXTO: 1 - Comum questo: 25


O grfico a seguir ilustra a evoluo do consumo de eletricidade no Brasil, em GWh, em quatro setores de consumo,
no perodo de 1975 a 2005.


Balano Energtico Nacional. Braslia:
MME, 2003 (com adaptaes).

25 - (ENEM)
Observa-se que, de 1975 a 2005, houve aumento quase linear do consumo de energia eltrica. Se essa mesma
tendncia se mantiver at 2035, o setor energtico brasileiro dever preparar-se para suprir uma demanda total
aproximada de

a) 405 GWh.
b) 445 GWh.
c) 680 GWh.
d) 750 GWh.
e) 775 GWh.

373







PROF.: J. CNDIDO





C
C
o
o
n
n
t
t
e
e

d
d
o
o
s
s
:
:

G
G
e
e
r
r
a
a
d
d
o
o
r
r
e
e
s
s
e
e
r
r
e
e
c
c
e
e
p
p
t
t
o
o
r
r
e
e
s
s






M
M
i
i
n
n
i
i
s
s
t
t
r
r
a
a
d
d
o
o
s
s
d
d
u
u
r
r
a
a
n
n
t
t
e
e
o
o
m
m

s
s
d
d
e
e
s
s
e
e
t
t
e
e
m
m
b
b
r
r
o
o














D Da at ta a d da a E En nt tr re eg ga a : : _ __ __ __ __ __ __ __ _/ /_ __ __ __ __ __ __ __ __ __ __ _/ /2 20 01 13 3

LISTA 6 FSICA

374


01 - (UnB DF) Calcule a corrente, em ampres, que passa pela resistncia R
3
, sabendo-se que V = 30V, R
1
= 1O, R
2

= 4O e R
3
= 6O.
R
3
R
1
R
2


02 - (UnB DF) Duas pilhas com foras eletromotriz (f.e.m.) e resistncia interna c
1
= 2V, r
1
= 1O e c
2
= 1O e r
2
= 1O
respectivamente, esto ligadas conforme o circuito abaixo. Para R
2
= 5O, calcule a intensidade da corrente que passa
atravs da resist~encia R (em ampres). Multiplique o resultado por 11.

r
1
r
2
c
1
c
2
R


03 - (UnB DF) Um perigo para os mergulhadores em rios e oceanos o contato com peixes eltricos. Sabe-se que
essa espcie produz eletricidade a partir de clulas biolgicas (eletroplacas) que funcionam como baterias eltricas.
Certos peixes eltricos encontrados na Amrica do Sul contm um conjunto de eletroplacas organizadas de forma
anloga ao circuito eltrico representado na figura acima. Existem, ao longo do corpo deles, 150 linhas horizontais,
com 5.000 eletroplacas por linha. Cada eletroplaca tem uma fora eletromotriz - c - de 0,15V e uma resistncia eltrica
R interna de 0,30O. A resistncia da gua R
gua
em torno do peixe deve ser considerada igual a 740O. Com
base nessas informaes, calcule uma das seguintes quantidades, desprezando, para a marcao na Folha de
Respostas, a parte fracionria do resultado final obtido aps efetuar todos os clculos solicitados.


a) O nmero total de eletroplacas do peixe eltrico, expressando a quantidade calculada em milhares de eletroplacas.
b) A resistncia equivalente em cad linha de eletroplacas, em ohms, dividindo a quantidade calculada por 10.
c) A resistncia equivalente do peixe eltrico, observada entre os pontos A e B, em ohms.
d) A potncia dissipada no peixe eltrico, em watts, quando esta est submerso na gua. Multiplique a quantidade
calculada por 10.
Caderno de Atividades

Disciplina:
Fsica

Professor(a):
J. Cndido
Aluno:
3 ano
Ensino Mdio
Data de Recebimento:
_____/_____/_____
Lista 06

Data Entrega:
_____/_____/_____

375
04 - (FUVEST SP) Considere o circuito representado esquematicamente na figura. O ampermetro ideal A indica a
passagem de uma corrente de 0,50 A. Os valores das resistncias dos resistores R
1
e R
3
e das foras eletromotrizes
E
1
e E
2
dos geradores ideais esto indicados na figura. O valor do resistor R
2
no conhecido. Determine:

+
+
-
-
R =0,50
1
O
E =6,0V
1
E =4,0
2
O
R
2 R =10
3
O
C
A
D

a) O valor da diferena de potencial entre os pontos C e D.
b) A potncia fornecida pelo gerador E
1
.

05 - (UFG GO) Considerando o circuito abaixo:

_
__
.
.
.
.
R
2R
2R
2R
fio retilneo
e longo
P
A B
d
D C
c
.

Dados: R = 3O, c = 12V, d = 1,0 cm e
0
= 4t . 10
7
T.m/A
a) Calcule a corrente total no circuito.
b) Admita que o comprimento do fio no trecho BC seja muito maior que a distncia d entre o fio e o ponto P, ou seja,
em relao ao ponto P, o fio pode ser considerado como retilneo e longo. Calcule o campo magntico nesse ponto,
devido somente ao trecho BC.

06 - (UENF RJ) Uma bateria de 12V e resistncia interna desprezvel o elemento ativo do circuito abaixo.

A
B
4,0O 4,0O
A
10,0O
12,0V V

Determine a corrente e a tenso mediadas pelos ampermetros A e pelo voltmetro V nas seguintes condies.
a) no circuito configurado;
b) quando estabelecido um curto-circuito entre os pontos A e B.

07 - (UFOP MG) A figura abaixo mostra duas lmpadas L
1
e L
2
e as suas respectivas potncias. Estas lmpadas esto
ligadas a uma fonte de tenso contnua de 100V.

100V
L - 25W
1
L - 50W
2

a) Determine a resistncia eltrica de cada lmpada.
b) Determine a corrente eltrica em cada lmpada.
c) Se a lmpada L
2
se queimar, a corrente eltrica na lmpada L
1
ser diferente do valor determinado no item b?
Justifique sua resposta.




376
08 - (UFOP MG) Uma resistncia ligada rede eltrica que fornece uma tenso de 110V. C uma chave que
desarma quando a corrente maior do que 30 A. Os aparelhos e lmpadas, com suas potncias especificadas, esto
como no circuito esquemtico abaixo.

8lmpadas
60Wcada
Chuveiro
2400W
C
110V
Som
120W
TV
120W
Geladeira
160W
Microondas
240W


a) Quantas lmpadas podem ser ligadas quando todos os aparelhos esto em funcionamento?
b) Se a fiao suporta correntes de at 5 A, aponte uma soluo para que todas as lmpadas e aparelhos possam
funcionar simultaneamente.

09 - (UFF RJ) O funcionamento de uma bateria eltrica pode ser descrito pelo grfico U x i a seguir, onde U a
diferena de potencial entre os terminais da bateria quando a mesma atravessada pela corrente eltrica i.


a) Determine os valores da f. e. m. da bateira e de sua resistncia interna, bem como o valor da corrente para a qual
a bateria estar em curto-circuito. Justifique suas respostas.
b) Esboce o grfico da potncia P fornecida pela bateria a um circuito externo em funo da corrente eltrica i que a
atravessa, determinando o valor mximo dessa potncia.

10 - (ITA SP) Quando se acendem os faris de um carro cuja bateria possui resistncia interna O = 050 , 0 r
i
, um
ampermetro indica uma corrente de 10 A e um voltmetro uma voltagem de 12 V. Considere desprezvel a resistncia
interna do ampermetro. Ao ligar o motor de arranque, observa-se que a leitura do ampermetro de 8,0 A e que as
luzes diminuem um pouco de intensidade. Calcular a corrente que passa pelo motor de arranque quando os faris
esto acesos.


11 - (UFF RJ) Um certo dispositivo, quando submetido a uma
diferena de potencial varivel, apresenta corrente eltrica I em
ampres, como funo da diferena de potencial V
D
em volts
aplicada aos seus terminais, conforme mostra o grfico abaixo.








Esse dispositivo utilizado, com uma lmpada de resistncia R = 50O e uma fonte de
d.d.p. varivel c, no circuito esquematizado na figura a seguir.

O dispositivo simbolizado por uma caixa preta e designado pela letra D.

a) Desenhe o grfico da diferena de potencial da fonte em funo da corrente eltrica
no circuito.
b) Determine a diferena de potencial da fonte para que a potncia dissipada na
lmpada seja de 4,5 W.

377
12 - (MACK SP) A tenso nos terminais de um receptor varia com a corrente, conforme o grfico abaixo:

25
22
0 2,0 5,0
I(A)
U(V)

A f.c.a.m. e a resistncia interna deste receptor so, respectivamente:
a) 11V e 1,0O
b) 12,5V e 2,5O
c) 20V e 1,0O
d) 22V e 2,0O
e) 25V e 5,0O

13) O esquema abaixo representa um circuito contendo duas pilhas e dois resistores:

10O 20O
9,0V
3,0V

a) Qual a intensidade da corrente eltrica?
b) Qual a ddp nos terminais do receptor?

14 - (FACMAUA SP) Um gerador de fora eletromotriz E
1
=12V e resistncia interna r
1
= 0,48O ligado em oposio
a outro, de fora eletromotriz E
2
= 6V e resistncia interna r
2
= 0,20O. Calcule a corrente que circula pelos geradores,
indicando qual deles estar fornecendo e qual estar absorvendo energia.

15 - (PUC SP) No circuito da figura, a diferena de potencial V
A
V
B
, com a chave K aberta, tem valor:

20V
15V
B
k
A
2O
3O

a) 35V
b) 20V
c) 15V
d) 5V
e) zero

16 - (PUC SP) Fechando a chave K da figura, a diferena de potencial V
A
V
B
passa a ter valor:

20V
15V
B
k
A
2O
3O

a) 35V
b) 23V
c) 20V
d) 17V
e) 15V


378
17 - (ITA SP) As duas baterias da figura esto ligadas em oposio. Suas f.e.m e resistncia so, respectivamente,
18,0V e 2,00O, 6,00V e 1,00O. Sendo i a corrente no circuito, V
ab
a tenso V
a
V
b
, podemos afirmar que:


a) i = 9,00 A; V
ab
= 10,0V
b) i = 6,00 A; V
ab
= 10,0V
c) i = 4,00 A; V
ab
= 10,0V
d) i = 4,00 A; V
ab
= 10,0V
e) i = 4,00 A; V
ab
= 24,0V

18) Considere a montagem abaixo esquematizada, onde a chave CH pode ocupar as posies (1), (2), (3) e (4).

E = 10V
r = 0,50O
(4)
CH
(1)
(2)
(3)
R = 2,0O
r = 0,50O
E = 8,0V

So feitas quatro proposies; assinale de acordo com o cdigo abaixo:
a) Se todas forem corretas.
b) Se apenas I e II forem corretas.
c) Se apenas III e IV forem corretas.
d) Se apenas II e IV forem corretas.
e) Se todas forem falsas.
I. com a chave CH na posio (1), a ddp no gerador vale 10V.
II. com a chave CH na posio (2), a ddp no gerador vale 8,0V.
III. com a chave CH na posio (3), a ddp no receptor vale 9,0 V.
IV. com a chave CH na posio (4), a ddp no gerador nula.

19 - (FUVEST SP) Uma lmpada L est ligada a uma bateria B por 2 fios, F
1
e F
2
, de mesmo
material, de comprimentos iguais e de dimetros d e 3d, respectivamente. Ligado aos
terminais da bateria, h um voltmetro ideal M (com resistncia interna muito grande), como
mostra a figura. Nestas condies a lmpada est acesa, tem resistncia R
L
= 2,0 O e dissipa
uma potncia igual a 8,0W. A fora eletromotriz da bateria c = 9,0V e a resistncia do fio F
1

R
1
= 1,8 O.
Determine o valor da:
a) corrente I, em ampres, que percorre o fio F
1
.
b) potncia P
2
, em watts, dissipada no fio F
2
.
c) diferena de potencial V
M
, em volts, indicada pelo voltmetro M.

20 - (UFRJ) Um estudante dispunha de duas baterias comerciais de mesma resistncia interna de 0,10 O, mas
verificou, por meio de um voltmetro ideal, que uma delas tinha fora eletromotriz de 12 Volts e a outra, de 11Volts. A
fim de avaliar se deveria conectar em paralelo as baterias para montar uma fonte de tenso, ele desenhou o circuito
indicado na figura a seguir e calculou a corrente i que passaria pelas baterias desse circuito.

a) Calcule o valor encontrado pelo estudante para a corrente i.
b) Calcule a diferena de potencial V
A
V
B
entre os pontos A e B
indicados no circuito.









18,0V
6,00V
b a
2,00O
1,00O

379
21 - (ENEM) A energia eltrica consumida nas residncias medida, em quilowatt-hora, por meio de um relgio
medidor de consumo. Nesse relgio, da direita para esquerda, tem-se o ponteiro da unidade, da dezena, da centena e
do milhar, Se um ponteiro estiver entre dois nmeros, considera-se o ltimo nmero ultrapassado pelo ponteiro.
Suponha que as medidas indicadas nos esquemas seguintes tenham sido feitas em uma cidade em que o preo do
quilowatt-hora fosse de R$ 0,20.


FILHO, A.G.; BAROLLI, E. Instalao Eltrica.
So Paulo: Scipione, 1997.

O valor a ser pago pelo consumo de energia eltrica registrado seria de

a) R$ 41,80.
b) R$ 42.00.
c) R$ 43.00.
d) R$ 43,80.
e) R$ 44,00.

22 - (ENEM) Usando presses extremamente altas, equivalentes s encontradas nas profundezas da Terra ou em um
planeta gigante, cientistas criaram um novo cristal capaz de armazenar quantidades enormes de energia. Utilizando-se
um aparato chamado bigoma de diamante, um cristal de difluoreto de xennio (XeF
2
) foi pressionado, gerando um
novo cristal com estrutura supercompacta e enorme quantidade de energia acumulada.
Inovao Tecnolgica. Disponvel em: http://www.inovacaotecnologica.com.br.
Acesso em: 07 jul. 2010 (adaptado).

Embora as condies citadas sejam diferentes do cotidiano, o processo de acumulao de energia descrito anlogo
ao da energia

a) armazenada em um carrinho de montanha russa durante o trajeto.
b) armazenada na gua do reservatrio de uma usina hidreltrica.
c) liberada na queima de um palito de fsforo.
d) gerada nos reatores das usinas nucleares.
e) acumulada em uma mola comprimida.

TEXTO: 1 - Comum questo: 23

Esta prova aborda fenmenos fsicos em situaes do cotidiano, em experimentos cientficos e em avanos
tecnolgicos da humanidade. Em algumas questes, como as que tratam de Fsica Moderna, as frmulas necessrias
para a resoluo da questo foram fornecidas no enunciado. Quando necessrio use g = 10 m/s
2
para a acelerao da
gravidade na superfcie da Terra e t = 3.

23 - (UNICAMP SP) Telas de visualizao sensveis ao toque so muito
prticas e cada vez mais utilizadas em aparelhos celulares, computadores e
caixas eletrnicos. Uma tecnologia frequentemente usada a das telas
resistivas, em que duas camadas condutoras transparentes so separadas
por pontos isolantes que impedem o contato eltrico.

a) O contato eltrico entre as camadas estabelecido quando o dedo exerce
uma fora F sobre a tela, conforme mostra a figura ao lado. A rea de
contato da ponta de um dedo igual a A = 0,25 cm
2
. Baseado na sua
experincia cotidiana, estime o mdulo da fora exercida por um dedo em
uma tela ou teclado convencional, e em seguida calcule a presso exercida
pelo dedo. Caso julgue necessrio, use o peso de objetos conhecidos como
guia para a sua estimativa.

380
b) O circuito simplificado da figura no espao de resposta ilustra como feita a deteco da posio do toque em telas
resistivas. Uma bateria fornece uma diferena de potencial U = 6 V ao circuito de resistores idnticos de R = 2 kO. Se o
contato eltrico for estabelecido apenas na posio representada pela chave A, calcule a diferena de potencial entre
C e D do circuito.

TEXTO: 2 - Comum s questes: 24, 25

A Revoluo Industrial consistiu em um conjunto de mudanas tecnolgicas com profundo impacto no processo
produtivo em nvel econmico e social. Iniciada na Inglaterra em meados do sculo XVIII, expandiu-se pelo mundo a
partir do sculo XIX. James Hargreaves, 1764, na Gr-Bretanha, inventa a fiadora spinning Jenny, uma mquina de
fiar rotativa que permitia a um nico arteso fiar oito fios de uma s vez.; James Watt, 1768, inventa a mquina a
vapor; Gottlieb Daimler, 1885, inventou um motor a exploso etc.

TEXTO: 3 - Comum s questes: 24, 25

Em 1820, o cientista dinamarqus Hans Christian Oersted (1777-1851) no imaginava que, com uma singela
experincia, descobriria um princpio fsico fundamental para o funcionamento do motor eltrico, que possibilitou o
surgimento e o desenvolvimento de um grande nmero de aparelhos eltricos, tais como: bateria, ventilador, furadeira,
liquidificador, aspirador de p, enceradeira, espremedor de frutas, lixadeira, alm de inmeros brinquedos movidos a
pilha e/ou tomada, como robs, carrinhos, etc, utilizados em todo o mundo.

24 - (UEPB) Acerca do assunto tratado no texto I, em relao ao motor eltrico, analise as proposies a seguir,
escrevendo V ou F conforme sejam verdadeiras ou falsas, respectivamente:

( ) O motor eltrico um elemento de trabalho que converte energia eltrica em energia mecnica de rotao.
( ) O motor eltrico uma mquina que converte energia mecnica de rotao em energia eltrica.
( ) Um motor eltrico uma aplicao do princpio fundamental do eletromagnetismo que afirma que uma fora
magntica vai atuar sobre um condutor eltrico se esse condutor estiver convenientemente colocado num campo
magntico e for percorrido por uma corrente eltrica.
Aps a anlise feita, assinale a alternativa que corresponde sequncia correta:
a) VVV
b) FVF
c) VVF
d) FVV
e) VFV

TEXTO: 4 - Comum s questes: 24, 25

A Revoluo Industrial consistiu em um conjunto de mudanas tecnolgicas com profundo impacto no processo
produtivo em nvel econmico e social. Iniciada na Inglaterra em meados do sculo XVIII, expandiu-se pelo mundo a
partir do sculo XIX. James Hargreaves, 1764, na Gr-Bretanha, inventa a fiadora spinning Jenny, uma mquina de
fiar rotativa que permitia a um nico arteso fiar oito fios de uma s vez.; James Watt, 1768, inventa a mquina a
vapor; Gottlieb Daimler, 1885, inventou um motor a exploso etc.

TEXTO: 5 - Comum s questes: 24, 25

Em 1820, o cientista dinamarqus Hans Christian Oersted (1777-1851) no imaginava que, com uma singela
experincia, descobriria um princpio fsico fundamental para o funcionamento do motor eltrico, que possibilitou o
surgimento e o desenvolvimento de um grande nmero de aparelhos eltricos, tais como: bateria, ventilador, furadeira,
liquidificador, aspirador de p, enceradeira, espremedor de frutas, lixadeira, alm de inmeros brinquedos movidos a
pilha e/ou tomada, como robs, carrinhos, etc, utilizados em todo o mundo.

25 - (UEPB) Ainda acerca do assunto tratado no texto I, resolva a seguinte situao-problema:
Um motor eltrico tem resistncia interna de 2,0O e est ligado a uma ddp de 100 V. Verifica-se que ele percorrido
por uma corrente eltrica de intensidade igual a 5,0 A. A fora contra-eletromotriz do motor e a potncia total recebida
pelo motor, respectivamente, so
a) 80 V; 350 W
b) 90 V; 450 W
c) 90 V; 500 W
d) 70 V; 300 W
e) 100 V; 400 W

381








PROF.: J. CNDIDO





C
C
o
o
n
n
t
t
e
e

d
d
o
o
s
s
:
:

M
M
a
a
g
g
n
n
e
e
t
t
i
i
s
s
m
m
o
o






M
M
i
i
n
n
i
i
s
s
t
t
r
r
a
a
d
d
o
o
s
s
d
d
u
u
r
r
a
a
n
n
t
t
e
e
o
o
m
m

s
s
d
d
e
e
o
o
u
u
t
t
u
u
b
b
r
r
o
o














D Da at ta a d da a E En nt tr re eg ga a : : _ __ __ __ __ __ __ __ _/ /_ __ __ __ __ __ __ __ __ __ __ _/ /2 20 01 13 3

LISTA 7 FSICA

382


01 - (IME RJ) Considere duas barras condutoras percorridas pelas correntes eltricas i
1
e i
2
, conforme a figura abaixo.
A primeira est rigidamente fixada por presilhas e a segunda, que possui liberdade de movimento na direo vertical,
est presa por duas molas idnticas, que sofreram uma variao de 1,0 m em relao ao comprimento nominal.
Sabendo-se que i
1
= i
2


a) o valor das correntes para que o sistema permanea esttico;
b) a nova variao de comprimento das molas em relao ao comprimento nominal, mantendo o valor das correntes
calculadas no pedido anterior, mas invertendo o sentido de uma delas.

Dados: comprimento das barras = 1,0 m;
massa de cada barra = 0,4 kg;
distncia entre as barras = 3,0 m;
constante elstica das molas = 0,5 N/m;
acelerao da gravidade (g) = 10 m/s
2
;
permeabilidade do vcuo (
o
) = 4t10
7
Tm/A.

02 - (UFOP MG) A figura abaixo mostra uma barra metlica horizontal de comprimento L = 50cm e peso P = 3,0N
suspensa por molas tambm metlicas de constante elstica k = 5,0N/m cada. A barra est imersa em uma regio
onde atua um campo de induo magntica uniforme B, cuja direo, na figura, perpendicular ao plano do papel e
apontando para o leitor.



Sabendo-se que a barra conduz uma corrente i = 6,0 A e que o campo no exerce influncia sobre as molas:
a) determine o sentido da corrente, par que as molas, no equilbrio, no exeram foras sobre a barra. Justifique sua
resposta;
b) calcule o valor da induo magntica, B, para que as molas, no equilbrio, fiquem alongadas de 15,0cm.

03 - (UFPE) Um fio MN, de 40cm de comprimento e massa igual a 30g, est suspenso horizontalmente por uma mola
ideal de constante elstica k = 10N/m. O conjunto encontra-se em uma regio de campo magntico uniforme B =
0,1Wb/m
2
, como indicado na figura. Quando a corrente no fio for 10A, dirigida de N para M, atuar sobre o fio uma
fora magntica dirigida verticalmente para baixo. Determine a elongao total, devido fora magntica e fora
gravitacional, sofrida pela mola, em cm.

M N
L
i
n
h
a
s

d
o

c
a
m
p
o
m
a
g
n

t
i
c
o
i = 10A

Caderno de Atividades

Disciplina:
Fsica

Professor(a):
J. Cndido
Aluno:
3 ano
Ensino Mdio
Data de Recebimento:
_____/_____/_____
Lista 07

Data Entrega:
_____/_____/_____
BATERIA
B

383
04 - (UNICAMP SP) Um fio condutor rgido de 200 g e 20 cm de comprimento ligado ao restante do circuito
atravs de contatos deslizantes sem atrito, como mostra a figura ao lado.




O plano da figura vertical. Inicialmente a chave est aberta. O fio condutor preso a um dinammetro e se encontra
em uma regio com campo magntico de 1,0 T, entrando perpendicularmente no plano da figura.
a) Calcule a fora medida pelo dinammetro com a chave aberta, estando o fio em equilbrio.
b) Determine a direo e a intensidade da corrente eltrica no circuito aps o fechamento da chave, sabendo-se que o
dinammetro passa a indicar leitura zero.
c) Calcule a tenso da bateria sabendo-se que a resistncia total do circuito de 6,0.

05 - (FEI SP) Uma barra condutora AB de peso P = 10,0 N e comprimento = 2,0m, apoia-se em dois trilhos
condutores e paralelos que formam com a horizontal um ngulo de 60. No h atrito entre a barra e esses condutores
e o conjunto est imerso em um campo de induo magntica uniforme e vertical, de intensidade B = 0,5 T. Qual a
corrente eltrica que deve passar pela barra para que permanea em repouso na posio indicada? Qual o sentido da
corrente na barra?

x
z
y
A
B

B
60
60


06 - (FACMAUA SP) Um condutor retilneo, de massa m = 10g por metro linear, mantido horizontal na direo Leste-
Oeste. Neste local pode-se considerar a induo magntica terrestre horizontal e de valor B = 5x10
5
T.
a) que corrente deveria circular pelo condutor, se isso fosse exeqvel, para que, retirando-se o seu apoio, ele
permanecesse em equilbrio?
b) qual o sentido desta corrente?
Admitir coincidentes as direes Norte-Sul geogrfica e magntica e adotar o valor g = 10m/s
2
.

07 - (UFV MG) Uma espira quadrada, de lado L = 1,0x10
1
m e massa m = 4,0x10
2
kg, percorrer por uma corrente i =
2,0 A, est suspensa por uma mola de constante elstica k = 10N/m. A parte
inferior da espira est imersa num campo magntico uniforme B

, com sentido
indicado na figura e mdulo T 0 , 1 | B | =

.

Considerando o mdulo da acelerao da gravidade | g

| = 10m/s
2
, determine:
a) o peso da espira;
b) o mdulo da fora magntica;
c) a deformao da mola devida s foras na espira.






dinammetro
contato-B
contato-A
B
x x
x x x x
x x x x x x
condutor rgido
bateria
chave
x x x x x x
x x x x x x
x x x x x x
i i
B


384
tela






B
b
x
y
eltron
P
L
anodo
catodo
U
08 - (UNIFESP SP) A figura mostra uma espira retangular imersa em um campo magntico uniforme, elemento
bsico de um motor eltrico de corrente contnua.
O plano da espira paralelo ao vetor campo magntico, B . A extremidade da espira junto ao ponto D est ligada ao
plo positivo da bateria e a extremidade B ao plo negativo; a corrente percorre o circuito no sentido de D para B.
So dados:
intensidade da corrente que percorre a espira: i = 0,80 A;
resistncia do fio no trecho DCAB: O = 5 , 2 R
mdulo do vetor campo magntico: B = 0,50 T;
comprimento dos lados da espira: AB = CD = 0,050 m.
Determine:
a) a diferena de potencial entre os pontos D e B.
b) o mdulo da fora magntica que atua em um dos lados, AB ou CD.

09 - (UnB DF) A figura abaixo mostra o esquema de um espectrmetro de massa idealizado por Dempster.
.
. . . . . . .
. . . . . . .
. . . . . . .
. . . . . . .
. . . . . . .
. . . . . . .
| | | | | | |
U
E
|
r
janela de inspeo
regio de campo
magntico uniforme
trjetria
regio de campo
eltrico uniforme

Esse aparelho foi usado para medir a massa de ons. Na fonte F, so produzidos ons de massa M e carga +q, que so
inseridos em uma regio onde existe um campo eltrico uniforme E

, sendo, ento, acelerados devido a uma diferena


de potencial U, adquirindo uma velocidade dada pela expresso
2
1
M
qU 2
v |
.
|

\
|
= . Em seguida, os ons penetram em uma
regio onde existe um campo magntico uniforme B

, de direo perpendicular ao plano desta folha de papel e sentido


para fora desta pgina, descrevendo uma trajetria semi-circular de raio r, cujo plano perpendicular ao campo B

,
conforme ilustra a figura. Sabendo que o mdulo da fora magntica que atua sobre os ons dado pela expresso F =
qvB e considerando U = 5,0 x 10
3
V, B = 0,5 T, r = 0,1 m e q = 1,6 x 10
19
C, calcule, em unidades de massa atmica
(uma), a massa M de um desses ons. Para isso, considere 1 uma = 1,6 x 10
27
kg e despreze a parte fracionria de
seu resultado, caso exista.

10 - (UFRJ) Um dos aparelhos de medida mais utilizados na fsica de partculas a cmara de bolhas. Ela foi
concebida em 1952 por D. A. Glaser quando observava as bolhas de um copo de cerveja. A cmara consiste de um
tanque contendo um lquido muito prximo da ebulio, mas que ainda no ferveu. Quando uma partcula carregada e
veloz passa pela cmara, produz-se um rastro de ons ao longo de sua trajetria e o lquido ferve em volta destes ons,
formando bolhas; fotografando-se estas bolhas, obtm-se a trajetria da partcula. A cmara ainda colocada em um
forte campo magntico uniforme B.
A figura mostra a trajetria de uma partcula carregada obtida a partir de uma de tais fotografias.

Suponha que o movimento ocorra no plano do papel e que o campo B aponte na direo
perpendicular a este plano e com sentido para fora. A partcula entra na cmara pelo ponto A da
figura.
a) Represente, por meio de segmentos de reta orientados, a fora magntica que atua nesta partcula
e sua velocidade quando esta se encontra no ponto P da figura.
b) Determine o sinal da carga dessa partcula. Justifique sua resposta.

11 - (ITA SP) Tubos de imagem de televiso possuem bobinas magnticas
defletoras que desviam eltrons para obter pontos luminosos na tela e, assim,
produzir imagens. Nesses dispositivos, eltrons so inicialmente acelerados por uma
diferena de potencial U entre o catodo e o anodo. Suponha que os eltrons so
gerados em repouso sobre o catodo. Depois de acelerados, so direcionados,
ao longo do eixo x, por meio de uma fenda sobre o anodo, para uma regio de
comprimento L onde atua um campo de induo magntica uniforme B , que
penetra perpendicularmente o plano do papel, conforme mostra o esquema.
Suponha, ainda, que a tela delimita a regio do campo de induo magntica.
A
P
.

385
Se um ponto luminoso detectado a uma distncia b sobre a tela, determine a expresso da intensidade de
B necessria para que os eltrons atinjam o ponto luminoso P, em funo dos parmetros e constantes fundamentais
intervenientes. (Considere b << L).

12 - (UFU MG) Numa regio onde existe um campo magntico vertical B

, uma calha muito fina, de comprimento 4m,


gira livremente na horizontal em torno de um eixo vertical, passando pelo seu centro, com uma velocidade angular e,
tendo duas bolinhas iguais, de massas m e carregadas com cargas +q, uma de cada lado (da calha), conforme a
figura.
Desprezando-se todos os atritos e sabendo-se que as bolinhas carregadas esto em equilbrio (paradas) a meia
distncia entre o eixo de rotao e as extremidades da calha (posies da figura), determine:
a) a fora centrpeta (fora resultante necessria) para manter essas bolinhas em equilbrio nessas posies;
b) o mdulo, a direo e o sentido da fora eltrica (Coulombiana) sobre cada bolinha;
c) a direo e o sentido do campo magntico nessa regio;
d) a intensidade desse campo magntico.

W
EIXO DE
ROTAO
1
m
1
m
1
m
1
m

Dados:
K = 9 x 10
9
Nm
2
/C
2

e = 2 rad/s
m = 11 x 10
11
kg
q = 4 x 10
10
C

13 - (FUVEST SP) Uma espira condutora ideal, com 1,5 m por 5,0 m, deslocada com velocidade constante, de tal
forma que um de seus lados atravessa uma regio onde existe um campo magntico B, uniforme, criado por um
grande eletrom. Esse lado da espira leva 0,5 s para atravessar a regio do campo. Na espira est inserida uma
resistncia R com as caractersticas descritas.
Em conseqncia do movimento da espira, durante esse intervalo de tempo, observa- se uma variao de
temperatura, em R, de 40C. Essa medida de temperatura pode, ento, ser utilizada como uma forma indireta para
estimar o valor do campo magntico B.
Assim determine

CARACTERSTICAS DO RESISTOR R:
Massa = 1,5g
Resistncia = 0,40 O
Calor especfico = 0,33 cal/g
a) a energia E, em joules, dissipada no resistor sob a forma de calor.
b) a corrente I, em ampres, que percorre o resistor durante o aquecimento.
c) o valor do campo magntico B, em teslas.
NOTE E ADOTE:
1 cal ~ 4 J
F = I B L a fora F que age sobre um fio de comprimento L, percorrido por uma corrente I, em um campo magntico
B.
|fem| = A|/At, ou seja, o mdulo da fora eletromotriz induzida igual variao de fluxo magntico | por unidade de
tempo.
| = BS, onde B a intensidade do campo atravs de uma superfcie de rea S, perpendicular ao campo.

386

14 - (UnB DF)
O grfico abaixo mostra o mdulo do campo magntico que atravessa uma bobina com 100 espiras em funo do
tempo. Sabendo-se que as espiras so quadradas e que tm 20cm de lado, calcule a fora eletromotriz induzida na
bobina, se o campo magntico dado em Tesla e o tempo em segundos. D a resposta em volt.

4
2
0 1 2
t(s)
B(T)


15 - (UnB DF)
Temos 1440 cm de fio de cobre especial para a fabricao de transformadores. Queremos construir um transformador
de espiras quadradas, cujos lados tm 3 cm, com entrada de 110 V e sada de 220 V, usando todo o fio,q eu,
obviamente, deve ser cortado, apenas uma vez. Calcule o produto do nmero de espiras do primrio pelo nmero de
espiras do secundrio. Divida este resultado por 100.

16 - (UFOP MG)
I. A figura abaixo mostra uma espira circular condutora prxima a uma barra imantada. A barra imantada se acha
disposta perpendicularmente ao plano da espira.

A B
N S


Ao aproximarmos a espira da barra afirmamos que:

- Aparecer uma corrente eltrica induzida na espira.
- O fluxo magntico atravs da espira aumenta.
- A corrente eltrica induzida na espira cria u campo magntico cujo sentido, no centro da espira, o de A para B.

Se V representa VERDADEIRO e F representa FALSO, a combinao correta para as afirmativas acima :
a) V, V, F
b) F, F, V
c) F, V, V
d) V, F, V
e) F, V, F

II. Com base nas leis de Faraday e Lenz, justifique a opo dada no item anterior.

17 - (UFOP MG)
Michel Faraday realizou um grande nmero de experincias e verificou que sempre que uma fora eletromotriz
induzida aparecia em um circuito, estava ocorrendo uma variao do fluxo magntico atravs desse circuito.
a) Descreva uma dessas experincias.
b) Descreva como Heinrich Lenz explicou o sentido da corrente induzida na espira.

18 - (UFF RJ)
Um prton, movendo-se com uma velocidade inicial v
0
= 4,0 x 10
5
m/s, atravessa um campo eltrico uniforme
estabelecido entre duas placas metlicas, planas e paralelas, distantes 4,5 mm uma da outra.


O movimento do prton se d na mesma direo do campo eltrico entre as placas e o
prton emerge deste campo com uma velocidade v = 5,0 x 10
5
m/s.
Sabendo que a razo massa/carga eltrica para o prton vale m/q = 1,04 x 10
-8
kg/C,
determine:
a) a acelerao do prton no campo eltrico;
b) o tempo de trnsito do prton entre as duas placas;
c) a diferena de potencial entre as placas;
d) o valor da intensidade do campo eltrico entre as placas.


387
19 - (ITA SP) A figura mostra uma regio de superfcie quadrada de lado L na qual atuam campos magnticos B
1
e
B
2
orientados em sentidos opostos e de mesma magnitude B. Uma partcula de massa m e carga 0 q > lanada do
ponto R com velocidade perpendicular s linhas dos campos magnticos. Aps um certo tempo de lanamento, a
partcula atinge o ponto S e a ela acrescentada uma outra partcula em repouso, de massa m e carga q (choque
perfeitamente inelstico). Determine o tempo total em que a partcula de carga 0 q > abandona a superfcie quadrada.


20 - (UnB DF) Dois eltrons chegam Terra e suas trajetrias so desviadas pelo campo magntico terrestre
tornando-se circulares. Se a velocidade do primeiro eltron tem mdulo igual a V
1
= 300km/s e a do segundo vale V
2
=
350km/s, calcule a razo entre as velocidades angulares e
1
/e
2
dos dois eltrons na sua trajetria perpendicular ao
campo magntico, supondo que as rbitas dos mesmos ocorrem em uma regio onde o campo magntico terrestre
homogneo.

21 - (UFG GO) Uma partcula de massa igual a 20 mg (miligramas) com carga de 100 C (1 = 10
6
), deslocando-se
com velocidade de 1,0 cm/s ao longo da direo x, entra em uma regio com campo magntico uniforme, de
intensidade igual a 10T, apontando na direo perpendicular ao plano do papel e sentido indicado na figura:

B
V
0
g
x
y
Regiocomcampo
magntico
Regiosemcampo
magntico
x x x x
x x x x
x x x x
x x x x
x x x x


a) Qual seria a trajetria descrita pela partcula, se, na regio de campo magntico, os efeitos da acelerao
gravitacional fossem desprezados?
b) Determine o mdulo, a direo e o sentido da fora magntica exercida sobre a partcula, no instante em que ela
penetra na regio de campo magntico.
c) Considerando, agora, que a partcula esteja tambm sujeita a uma acelerao gravitacional de intensidade g = 10
m/s
2
, no sentido indicado na figura, calcule a acelerao resultante sobre a partcula, no exato instante em que ela
penetra na regio com campo magntico.

22 - (UNESP) Um feixe constitudo de dois tipos de partculas com cargas eltricas iguais, mas massas m
1
e m
2

) m m (
2 1
= . Ao adentrarem, com velocidades iguais, uma regio onde existe um campo magntico uniforme, as
partculas de massa m
1
e m
2
descrevem, num mesmo plano, trajetrias semi-circulares diferentes, com raios R
1
e R
2
,
respectivamente, como ilustradas na figura.


Expresse a razo entre as massas m
1
e m
2
, em termos de R
1
e R
2
.

388
23 - (ENEM) O manual de funcionamento de um captador de guitarra eltrica apresenta o seguinte texto:
Esse captador comum consiste de uma bobina, fios condutores enrolados em torno de um m permanente. O campo
magntico do m induz o ordenamento dos polos magnticos na corda da guitarra, que est prxima a ele. Assim,
quando a corda tocada, as oscilaes produzem variaes, com o mesmo padro, no fluxo magntico que atravessa
a bobina. Isso induz uma corrente eltrica na bobina, que transmitida at o amplificador e, da, para o alto-falante.

Um guitarrista trocou as cordas originais de sua guitarra, que eram feitas de ao, por outras feitas de nilon. Com o uso
dessas cordas, o amplificador ligado ao instrumento no emitia mais som, porque a corda de nilon

a) isola a passagem de corrente eltrica da bobina para o alto-falante.
b) varia seu comprimento mais intensamente do que ocorre com o ao.
c) apresenta uma magnetizao desprezvel sob a ao do m permanente.
d) induz correntes eltricas na bobina mais intensas que a capacidade do captador.
e) oscila com uma frequncia menor do que a que pode ser percebida pelo captador.

24 - (ENEM) O crescimento da produo de energia eltrica ao longo do tempo tem influenciado decisivamente o
progresso da humanidade, mas tambm tem criado uma sria preocupao: o prejuzo ao meio ambiente. Nos
prximos anos, uma nova tecnologia de gerao de energia eltrica dever ganhar espao: as clulas a combustvel
hidrognio/oxignio.

Com base no texto e na figura, a produo de energia eltrica por meio
da clula a combustvel hidrognio/oxignio diferencia-se dos
processos convencionais porque

a) transforma energia qumica em energia eltrica, sem causar danos
ao meio ambiente, porque o principal subproduto formado a
gua.
b) converte a energia qumica contida nas molculas dos
componentes em energia trmica, sem que ocorra a produo de
gases poluentes nocivos ao meio ambiente.
c) transforma energia qumica em energia eltrica, porm emite
gases poluentes da mesma forma que a produo de energia a
partir dos combustveis fsseis.
d) converte energia eltrica proveniente dos combustveis fsseis em energia qumica, retendo os gases poluentes
produzidos no processo sem alterar a qualidade do meio ambiente.
e) converte a energia potencial acumulada nas molculas de gua contidas no sistema em energia qumica, sem que
ocorra a produo de gases poluentes nocivos ao meio ambiente.
VILLULLAS, H.M; TICIANELLI, E. A; GONZLEZ, E.R.
Qumica Nova na Escola. N. 15, maio 2002.


25 - (ENEM) Observe a tabela seguinte. Ela traz especificaes tcnicas constantes no manual de instrues
fornecido pelo fabricante de uma torneira eltrica.

Disponvel em:http://www.cardeal.com.br.manualprod/Manuais/Torneira%20
Suprema/ManualTorneiraSupremaroo.pdf
Considerando que o modelo de maior potncia da verso 220 V da torneira suprema foi inadvertidamente conectada a
uma rede com tenso nominal de 127 V, e que o aparelho est configurado para trabalhar em sua mxima potncia.
Qual o valor aproximado da potncia ao ligar a torneira?
a) 1.830 W
b) 2.800 W
c) 3.200 W
d) 4.030 W
e) 5.500 W

389







PROF.: MAXMILIANO N.C.






C
C
o
o
n
n
t
t
e
e

d
d
o
o
s
s
:
:

D
D
i
i
n
n

m
m
i
i
c
c
a
a
.
.






M
M
i
i
n
n
i
i
s
s
t
t
r
r
a
a
d
d
o
o
s
s
d
d
u
u
r
r
a
a
n
n
t
t
e
e
o
o
m
m

s
s
d
d
e
e
f
f
e
e
v
v
e
e
r
r
e
e
i
i
r
r
o
o














D Da at ta a d da a E En nt tr re eg ga a : : _ __ __ __ __ __ __ __ _/ /_ __ __ __ __ __ __ __ __ __ __ _/ /2 20 01 13 3
LISTA 1 FSICA

390

Vetores
Denomina-se por vetor os segmentos de reta orientados, que possuem todos mesma intensidade (denominada norma
ou mdulo), mesma direo e mesmo sentido.Por meio de vetores podemos dar a grandezas como fora, velocidade,
acelerao dentre outras, um mdulo, direo e sentido. poderemos utilizar dois processos como indicamos a seguir:









Determinao da Resultante
O mdulo da resultante pode ser calculado pela expresso matemtica abaixo.



1. (UFAL) Uma partcula est sob ao das foras coplanares conforme o esquema abaixo. A resultante delas uma
fora, de intensidade, em N, igual a:

a) 110
b) 70
c) 60
d) 50
e) 30


2. (ACAFE) Os mdulos das foras representadas na figura so F1 = 30N, F2 = 20 N e F3 = 10N. Determi ne o mdulo
da fora resultante:

Caderno de Atividades

Disciplina:
Fsica

Professor(a):
Maximiliano
Aluno:
3 ano
Ensino Mdio
Data de Recebimento:
_____/_____/_____
Lista 01

Data Entrega:
_____/_____/_____

391
3. (PUCC ) A soma de dois vetores ortogonais, isto , perpendiculares entre si, um de mdulo 12 e outro de mdulo
16, ter mdulo igual a?


Composio de movimentos




4.(INATEL) Dois corpos A e B se deslocam segundo trajetria perpendiculares, com velocidades constantes, conforme
est ilustrado na figura adiante.

As velocidades dos corpos medidas por um observador fixo tm intensidades iguais a: VA = 5,0 (m/s) e VB = 12 (m/s).
Quanto mede a velocidade do corpo A em relao ao corpo B?

5.(UfU) A trajetria descrita por um ponto material P e a equao horria da projeo horizontal de P, num sistema de
coordenadas cartesiano ortogonal Oxy, expressas em unidades do sistema internacional, so respectivamente: y =
0,125x2 e x = 6,0t, onde x e y so coordenadas de P e t tempo. A velocidade de P segundo Ox e a acelerao de P
segundo Oy, em unidades do sistema internacional, tm densidades iguais a?

6.Um homem rema um barco com velocidade de 5,00 km/h na ausncia de correnteza. Quanto tempo ele gasta para
remar 3,00 km rio abaixo e voltar ao ponto de partida num dia em que a velocidade da correnteza de 1,0 km/h?

Leis de Newton

I Lei.: Inrcia a propriedade comum a todos os corpos materiais, mediante a qual eles tendem a manter o seu
estado de movimento ou de repouso.
II.: A Segunda lei de Newton trata dos casos em que a resultante das foras que actuam num corpo no nula. Neste
caso, nota-se o aparecimento de uma outra grandeza conhecida: a acelerao.
III.: Sabemos que fora fruto da interao, ou seja, uma fora atuante em um corpo representa a ao que este
corpo recebe de um outro corpo.

07. (UFG GO) Em uma torneira gotejante, as gotas caem quando o dimetro atinge o valor limiar D. Nessa situao,
considerando que as gotas possuem forma esfrica, o valor mximo da fora devido tenso superficial, em N, que
mantm a gota presa torneira, ?
Dados:
O H
2
d
= 1,0 g/cm3
D = 5,0 mm
t = 3
g = 10 m/s2

08.(UERJ) Um corpo de massa igual a 6,0 kg move-se com velocidade constante de
0,4 m/s, no intervalo de 0 s a 0,5 s. Considere que, a partir de 0,5 s, esse corpo
impulsionado por uma fora de mdulo constante e de mesmo sentido que a
velocidade, durante 1,0 s. O grfico abaixo ilustra o comportamento da fora em
funo do tempo.


Calcule a velocidade do corpo no instante t = 1,5s


392
09.(UERJ) Um patinador cujo peso total 800 N, incluindo os patins, est parado em uma pista de patinao em gelo.
Ao receber um empurro, ele comea a se deslocar. A fora de atrito entre as lminas dos patins e a pista, durante o
deslocamento, constante e tem mdulo igual a 40 N. Estime a acelerao do patinador imediatamente aps o incio
do deslocamento.

10.(UNEMAT) Um nibus de peso igual a 10.000 N est em movimento com velocidade de 15 m/s. O motorista que
dirige o nibus avista na pista de rolamento um animal e aciona o freio. O nibus percorre 9 metros durante a frenagem
at parar completamente. O mdulo da fora de frenagem igual a: (Dado: g=10m/s2)


11.A respeito do conceito da inrcia, assinale a frase correta:
a) Um ponto material tende a manter sua acelerao por inrcia.
b) Uma partcula pode ter movimento circular e uniforme, por inrcia.
c) O nico estado cinemtico que pode ser mantido por inrcia o repouso.
d) No pode existir movimento perptuo, sem a presena de uma fora.
e)A velocidade vetorial de uma partcula tende a se manter por inrcia; a fora usada para alterar a velocidade e no
para mant-la.

12.(UNESP) As estatsticas indicam que o uso do cinto de segurana deve ser obrigatrio para prevenir leses mais
graves em motoristas e passageiros no caso de acidentes. Fisicamente, a funo do cinto est relacionada com a:
a) Primeira Lei de Newton;
b) Lei de Snell;
c) Lei de Ampre;
d) Lei de Ohm;
e) Primeira Lei de Kepler.

13.(Direito-C.L) Um corpo de massa 5 Kg, inicialmente em repouso, sofre a ao de uma fora constante de 30N. Qual
a velocidade do corpo (em m/s) depois de 5 s? A resultante das foras que agem sobre um ponto material igual ao
produto de sua massa pela acelerao adquirida.

14.(PUC) Uma fora constante atuando sobre um certo corpo de massa m produziu uma acelerao de 4,0 m/s 2. Se a
mesma fora atuar sobre outro corpo de massa igual a m/2 , a nova acelerao ser, em m/s2

15(UESPI) O coeficiente de atrito esttico entre o bloco e a parede vertical, mostrados na figura abaixo, 0,25. O bloco
pesa 100N. O menor valor da fora F para que o bloco permanea em repouso :


16.(Direito-C.L.-98)Uma fora de12 N aplicada em um corpo de massa 50 kg. A) Qual a acelerao produzida por
essa fora? B) Se a velocidade do corpo era 3 m/s quando se iniciou a ao da fora, qual ser o seu valor 5 s depois?

17. (MACK SP) O bloco A est na iminncia de movimento de descida, quando equilibrado pelo bloco B, como mostra
a figura. Os fios e as polias so ideais e o coeficiente de atrito esttico entre o bloco A e a superfcie de apoio 0,2. A
massa do bloco B aproximadamente de? Dados:
0,8 53 sen e 0,6 53 cos = =
e g=10m/s2




18. (UPE)Um bloco de ao colocado sobre uma tbua de apoio que vai se inclinando aos poucos. Quando o bloco
fica na iminncia de escorregar, a tbua forma com a horizontal o ngulo |, de acordo com a figura a seguir:


393


Sabendo-se que o coeficiente de atrito esttico entre o bloco e a tbua vale e = 0,40, CORRETO afirmar que a
distncia x indicada na figura, em centmetros, vale?



19. (UFG GO) A fora muscular origina-se nas fibras musculares, conforme figura (a), como resultado das interaes
entre certas protenas que experimentam mudanas de configurao e proporcionam a contrao rpida e voluntria
do msculo. A fora mxima que um msculo pode exercer depende da sua rea da seo reta e vale cerca de 30
N/cm2. Considere um operrio que movimenta com uma velocidade constante uma caixa de 120 kg sobre uma
superfcie rugosa, de coeficiente de atrito 0,8, usando os dois braos, conforme ilustrado na figura (b).



Dessa forma, a menor seo reta dos msculos de um dos braos do operrio, em cm2, e uma das protenas
responsveis pela contrao das miofibrilas so:
Dados: g = 10,0 m/s2
a) 16 e actina.
b) 16 e mielina.
c) 20 e miosina.
d) 32 e actina.
e) 32 e miosina.

20. (MACK SP) Um balde de 400 g suspenso por um fio ideal que tem uma extremidade presa a um bloco de massa
12 kg. O conjunto est em repouso, quando se abre a torneira, que proporciona uma vazo de gua ( = 1 kg/L),
constante e igual a 0,2 L/s. Sabendo-se que o coeficiente de atrito esttico entre o bloco e a superfcie horizontal que o
suporta E = 0,4 e que a polia ideal, esse bloco iniciar seu deslocamento no instante imediatamente aps quanto
tempo?
Dado: g =10 m/s2




394

21.(UPE) No dispositivo representado na figura abaixo, um bloco de granito de massa 1500 kg puxado para cima em
um plano inclinado, com uma velocidade constante de 2,0 m/s por uma fora F aplicada ao cabo. As distncias
indicadas so d1 = 40 m e d2 = 30 m, e o coeficiente de atrito cintico entre o bloco e o plano inclinado 0,50.
Considere g = 10 m/s2. O atrito na roldana e as massas da corda e da roldana so desprezveis. Nessas condies, a
potncia desenvolvida pela fora F aplicada ao bloco pelo cabo vale em kW:




22. (UNESP) Um professor de fsica pendurou uma pequena esfera, pelo seu centro de gravidade, ao teto da sala de
aula, conforme a figura:



Em um dos fios que sustentava a esfera ele acoplou um dinammetro e verificou que, com o sistema em equilbrio, ele
marcava 10N. O peso, em newtons, da esfera pendurada de?

23. (PUC RS) Dois operrios suspendem um balde por meio de cordas, conforme mostra o esquema a seguir.



So dados: sen 30 = cos 60 =
2
1
e
sen 60 = cos 30 =
2
3


Sabe-se que o balde, com seu contedo, tem peso 50N, e que o ngulo formado entre as partes da corda no ponto de
suspenso 60. A corda pode ser considerada como ideal (inextensvel e de massa desprezvel). Quando o balde
est suspenso no ar, em equilbrio, a fora exercida por um operrio, medida em newtons, vale?


395
24.(UFTM) As dependncias da escola no possuam tomadas no local em que estava montada a barraca do
churrasco e, por isso, uma extenso foi esticada, passando por uma janela do segundo andar do prdio das salas de
aula.



Para posicionar a lmpada logo frente da barraca, uma corda presa lona foi amarrada ao fio da extenso, obtendo-
se a configurao indicada na figura. Considere
2
1
30 sen =
,
2
3
30 cos =
e g = 10 m/s2.



O conjunto formado pela cpula, lmpada e soquete, de massa total 0,5 kg, sustentado pela corda e pelo fio
condutor. Desprezando-se os pesos do fio e da corda, possvel afirmar que o fio condutor esticado atravs da janela
sofre ao de uma fora de intensidade, em newtons, de?


25.(UFTM)A figura 1 mostra um carrinho transportando um corpo de massa m por um plano sem atrito, inclinado em
30 com a horizontal. Ele empurrado para cima, em linha reta e com velocidade constante, por uma fora constante
de intensidade F1 = 80 N. A figura 2 mostra o mesmo carrinho, j sem o corpo de massa m, descendo em linha reta, e
mantido com velocidade constante por uma fora tambm constante de intensidade F2 = 60 N.



Adotando g = 10 m/s2, quanto vale a massa m em kg?

26.(UPE) Prximo a um abismo, solto do repouso um bloco de massa M = 5,0kg, de uma altura de h = 5,0m acima do
nvel do incio da parede do referido abismo, do alto de uma rampa com ngulo de inclinao u = 30, sem atrito,
adjacente parede do abismo de altura H = 10,0m, como observado na figura a seguir:




396
Dados: considere a acelerao da gravidade g = 10 m/s2; sen30 = 0,5 e cos30 = 0,87.

Analise as proposies a seguir e conclua.

00. A acelerao do bloco, enquanto ele desce escorregando pela rampa, de 5,0m/s2.
01. A velocidade escalar do bloco, quando ele deixa a rampa, de 10,0m/s.
02. A distncia A da parede do abismo at o bloco atingir o solo de 8,7m.
03. O tempo que o bloco leva desde o momento em que solto at o instante em que atinge o solo de 1,0s.
04. A acelerao do bloco depende da sua massa M.


27. (UFPE) Considere dois blocos empilhados, A e B, de massas mA = 1,0 kg e mB = 2,0 kg. Com a aplicao de uma
fora horizontal
F
sobre o bloco A, o conjunto move-se sem ocorrer deslizamento entre os blocos. O coeficiente de
atrito esttico entre as superfcies dos blocos A e B igual a 0,60, e no h atrito entre o bloco B e a superfcie
horizontal. Determine o valor mximo do mdulo da fora
F
, em newtons, para que no ocorra deslizamento entre os
blocos, ser?




































397













PROF.:MAXMILIANO N.C.





C
C
o
o
n
n
t
t
e
e

d
d
o
o
s
s
:
:

E
E
s
s
t
t

t
t
i
i
c
c
a
a
e
e
E
E
q
q
u
u
i
i
l
l
i
i
b
b
r
r
i
i
o
o
.
.






M
M
i
i
n
n
i
i
s
s
t
t
r
r
a
a
d
d
o
o
s
s
d
d
u
u
r
r
a
a
n
n
t
t
e
e
o
o
m
m

s
s
d
d
e
e
m
m
a
a
r
r

o
o















D Da at ta a d da a E En nt tr re eg ga a : : _ __ __ __ __ __ __ __ _/ /_ __ __ __ __ __ __ __ __ __ __ _/ /2 20 01 13 3
LISTA 2 FSICA

398


1. (ITA SP) Considere uma balana de braos desiguais, de comprimentos 1 e 2, conforme mostra a figura. No lado
esquerdo encontra-se pendurada uma carga de magnitude Q e massa desprezvel, situada a uma certa distncia de
outra carga, q. No lado direito encontra-se uma massa m sobre um prato de massa desprezvel. Considere as cargas
como pontuais e desprezvel a massa do prato da direita, o valor de q para equilibrar a massa m dado por

a) mg2d2/(k0Q1)
b) 8mg2d2/(k0Q1)
c) 4mg2d2/(3k0Q1)
d) 2mg2d2/(
3
k0Q1)
e) 8mg2d2/(
3 3
k0Q1)

2.(UERJ) Uma prancha homognea de comprimento igual a 5,0 m e massa igual a 10,0 kg encontra-se apoiada nos
pontos A e B, distantes 2,0 m entre si e equidistantes do ponto mdio da prancha.
Sobre a prancha esto duas pessoas, cada uma delas com massa igual a 50 kg.
Observe a ilustrao:



Admita que uma dessas pessoas permanea sobre o ponto mdio da prancha.
Nessas condies, calcule a distncia mxima, em metros, que pode separar as duas pessoas sobre a prancha,
mantendo o equilbrio.

03.(MACK SP) Em uma experincia, a barra homognea, de seco reta constante e peso 100 N, suspensa pelo seu
ponto C, por um fio ideal, e mantida em equilbrio como mostra a figura. Nas extremidades da barra, so colocados os
corpos A e B. Sabe-se que o peso do corpo B 80 N. A trao no fio que sustenta essa barra tem intensidade?



04. (IME RJ) A figura apresenta um perfil metlico AB, com dimenses AC = 0,20 m e CB = 0,18 m, apoiado em C por
meio de um pino sem atrito. Admitindo-se desprezvel o peso do perfil AB, o valor da fora vertical F, em newtons, para
que o sistema fique em equilbrio na situao da figura :
Dados:
- sen(15) = 0,26
A
B
175,0 N
216,0 N
15
75
F
C

- cos(15) = 0,97
Caderno de Atividades

Disciplina:
Fsica

Professor(a):
Maximiliano
Aluno:
3 ano
Ensino Mdio
Data de Recebimento:
_____/_____/_____
Lista 02

Data Entrega:
_____/_____/_____

399
05.(FGV) Em um poste, uma trave horizontal feita de madeira serve de suporte para os trs isoladores de alta tenso,
responsveis, tambm, por manter os fios sobrelevados.



Os pesos da trave e dos isoladores podem ser considerados desprezveis. Cada fio exerce sobre seu isolador uma
fora vertical de intensidade 400 N e, por essa razo, alm da trave ser presa diretamente ao poste, uma haste
inclinada exerce um esforo adicional para cima, em newtons, de intensidade?

06. (UPE) A figura abaixo mostra uma barra homognea de peso 10 N e de comprimento 10 m que est apoiada sobre
um suporte distante de 3,0 m da sua extremidade esquerda.



Pendura-se um bloco de massa m = 2,0 kg na extremidade esquerda da barra e coloca-se um bloco de massa M = 4,0
kg sobre a barra do lado direito ao suporte. O valor de D, para que a barra esteja em equilbrio, em metros, vale?

Dado: considere a acelerao da gravidade g = 10 m/s2.


07.(UPE) Uma barra de peso desprezvel est sobre um apoio situado no meio dela. Aplicam-se 3 foras sobre a barra
como indicado na figura.



Dados: considere cos 30 = 0,86 e sen 30 = 0,5
Para que a barra esteja em equilbrio, o valor de F, em newtons, vale
a) 17,2
b) 12,7
c) 10,0
d) 20,0
e) 18,0

08. (UNICAMP SP) O homem tem criado diversas ferramentas especializadas,
sendo que para a execuo de quase todas as suas tarefas h uma ferramenta
prpria.
a) Uma das tarefas enfrentadas usualmente a de levantar massas cujo peso
excede as nossas foras. Uma ferramenta usada em alguns desses casos o
guincho girafa, representado na figura abaixo.
Um brao mvel movido por um pisto e gira em torno do ponto O para
levantar uma massa M. Na situao da figura, o brao encontra-se na posio

400
horizontal, sendo D = 2,4m e d = 0,6m. Calcule o mdulo da fora
F
exercida pelo pisto para equilibrar uma massa M
= 430kg.
Despreze o peso do brao. Dados: cos30 = 0,86 e sen30 = 0,50.

b) Ferramentas de corte so largamente usadas nas mais diferentes situaes como, por exemplo, no preparo dos
alimentos, em intervenes cirrgicas, em trabalhos com metais e em madeira. Uma dessas ferramentas o formo,
ilustrado na figura ao lado, que usado para entalhar madeira. A rea da extremidade cortante do formo que tem
contato com a madeira detalhada com linhas diagonais na figura, sobre uma escala graduada. Sabendo que o
mdulo da fora exercida por um martelo ao golpear a base do cabo do formo F = 4,5N, calcule a presso exercida
na madeira.



9. (IME RJ)A figura ilustra uma mola feita de material isolante eltrico, no deformada, toda contida no interior de um
tubo plstico no condutor eltrico, de altura h = 50 cm. Colocando-se sobre a mola um pequeno corpo (raio
desprezvel) de massa 0,2 kg e carga positiva de 9 10-6 C, a mola passa a ocupar metade da altura do tubo. O valor
da carga, em coulombs, que dever ser fixada na extremidade superior do tubo, de modo que o corpo possa ser
posicionado em equilbrio esttico a 5 cm do fundo,



Dados:
Acelerao da Gravidade: g = 10 m/s2
Constante Eletrosttica: K = 9 109 N.m2/C2

a) 2 106
b) 4 104
c) 4 106
d) 8 104
e) 8 106

10. (UFMG) Para pintar uma parede, Miguel est sobre um andaime suspenso por duas cordas. Em certo instante, ele
est mais prximo da extremidade direita do andaime, como mostrado nesta figura:

Sejam TE e TD os mdulos das tenses nas cordas, respectivamente, da esquerda e da direita e P o mdulo da soma
do peso do andaime com o peso de Miguel.
Analisando-se essas informaes, CORRETO afirmar que
a) TE = TD e TE + TD = P.
b) TE = TD e TE + TD > P.
c) TE < TD e TE + TD = P.
d) TE < TD e TE + TD > P.



401
11. (UEM PR) Com uma balana de braos desiguais, de tamanhos L1 > L2, foram realizadas as seguintes medidas:
primeiro, uma massa M0 colocada no prato esquerda e equilibrada por uma massa M1 colocada no prato direita
(figura a); depois, a massa M0 colocada no prato direita e equilibrada por uma massa M2 (figura b). Dessas
medidas, pode-se concluir que


01.
0
1
2
0
M
M
M
M
=
.
02. M2 > M1
04. M1L1 < M2L2
08.
2
M M
M
2 1
0
+
=

16. M0L1 + M0L2 = M1L1 + M2L2

12.Unicap-PE A figura abaixo representa uma barra homognea de peso igual a 200 N,
articulada em A e mantida em equilbrio por meio de um fio ideal BC, de comprimento igual
a 1 m., sob ngulo de 45.
Calcule a trao no cabo e a fora F sobre o piv.







13. (UEPG PR) A figura abaixo ilustra um esquema de uma balana de pratos. Sobre o equilbrio dessa balana
assinale o que for correto.



01. A condio de equilbrio exige que a resultante das foras que atuam sobre a parte mvel da balana seja zero.
02. Para que a balana esteja em equilbrio o momento resultante das foras deve ser nulo.
04. A balana em questo no est em equilbrio, pois, a resultante diferente de zero.
08. Se o brao A for diferente do brao B a balana poder estar em equilbrio, porm, a massa medida no ser
confivel.

14. (FGV) Todo carrinho de churros possui um acessrio peculiar que serve para injetar doce de leite nos churros.
Nele, a fora sobre um mbolo, transmitida por alavancas, empurra o recheio para
dentro do churro.
Em cada lado do recheador, h duas alavancas unidas por um piv, uma delas, reta e
horizontal, e a outra, parte vertical e parte transversal. A alavanca maior encontra na
base do aparelho outro piv e, na outra extremidade, um manete, onde aplicada a
fora. A alavanca menor se conecta extremidade do mbolo que est em contato
com o doce de leite, pronta para aplicar, no incio do processo, uma fora horizontal.
No momento em que vai rechear um churro, o vendedor posiciona sua mo sobre o
manete e aplica sobre ele uma fora de 2 N, constante, de direo e sentido indicados
no esquema, desenhado sobre uma malha quadriculada, cujas unidades tm
dimenses 1 cm x 1 cm.



402


Se, devido a uma obstruo do canal de sada do recheio, o mecanismo no se move, desconsiderando-se as massas
das alavancas e do manete, a intensidade da fora que, nessa condio, o mecanismo aplica sobre o mbolo, tem
valor, em N, de ?

15. (UFPB) Uma balana constituda por um contrapeso de 4 kg que pode ser movimentado sobre uma barra de 1 m
de comprimento e massa desprezvel. A extremidade esquerda da barra pode girar livremente em torno de um piv
fixo. Uma corda de massa desprezvel amarrada a outra extremidade da barra, passando por uma polia que pode girar
sem atrito, sustenta um bloco cuja massa se deseja medir, conforme figura abaixo.


Sabendo que o sistema encontra-se em equilbrio com a barra na horizontal e que o contrapeso est a 0,6 m da
extremidade direita da barra, correto afirmar que a massa do bloco de?

16. (FUVEST SP) Em uma academia de musculao, uma barra B, com 2,0 m de comprimento e massa de 10 kg, est
apoiada de forma simtrica em dois suportes, S1 e S2, separados por uma distncia de 1,0 m, como indicado na figura.
Para a realizao de exerccios, vrios discos, de diferentes massas M, podem ser colocados em encaixes, E, com
seus centros a 0,10 m de cada extremidade da barra. O primeiro disco deve ser escolhido com cuidado, para no
desequilibrar a barra. Dentre os discos disponveis, cujas massas esto indicadas abaixo, aquele de maior massa e
que pode ser colocado em um dos encaixes, sem desequilibrar a barra, o disco de ?

17. (ITA SP) Na experincia idealizada na figura, um halterofilista sustenta, pelo ponto M, um conjunto em equilbrio
esttico composto de uma barra rgida e uniforme, de um peso
N 100 P
1
=
na extremidade a 50 cm de M, e de um peso
60N P
2
=
, na posio x2 indicada. A seguir, o mesmo equilbrio esttico verificado dispondo-se, agora, o peso P2 na
posio original de P1, passando este posio de distncia
2 1
1,6x x =
da extremidade N. Sendo de 200 cm o
comprimento da barra e
2
10m/s g =
a acelerao da gravidade, a massa da barra de ?



403
18. (UFPE) A figura mostra uma corda que passa por uma polia ideal, tendo uma de suas extremidades presa ao
bloco de massa M, e a outra presa na extremidade B de uma viga uniforme. Considerando que a viga, de comprimento
L e massa igual a 50 kg, mantida em equilbrio na horizontal com o auxlio do apoio em A, determine a massa do
bloco, em kg.


19. (UNIMONTES MG) Observe o sistema, em equilbrio, ilustrado na figura abaixo. Nele, um bloco de massa M e
volume V est suspenso pelas cordas A e B e imerso num lquido de densidade dL. O valor do mdulo da acelerao
da gravidade, no local, g. Sobre as traes TA e TB em cada uma das cordas, CORRETO afirmar que


a)
u

=
sen
) Vd M ( g
T
L
B

b)
u

=
sen 2
) Vd M ( g
T
L
A

c)
u

=
sen 2
) M 2 Vd ( g
T
L
B

d)
u

=
sen
) Vd M 2 ( g
T
L
A


20. (UNIFESP SP) De posse de uma balana e de um dinammetro (instrumento para medir foras), um estudante
decide investigar a ao da fora magntica de um m em forma de U sobre uma pequena barra de ferro.
Inicialmente, distantes um do outro, o estudante coloca o m sobre uma balana e anota a indicao de sua massa.
Em seguida, ainda distante do m, prende a barra ao dinammetro e anota a indicao da fora medida por ele.
Finalmente, monta o sistema de tal forma que a barra de ferro, presa ao dinammetro, interaja magneticamente com o
m, ainda sobre a balana, como mostra a figura.


A balana registra, agora, uma massa menor do que a registrada na situao
anterior, e o dinammetro registra uma fora equivalente

a) fora peso da barra.
b) fora magntica entre o m e a barra.
c) soma da fora peso da barra com metade do valor da fora magntica
entre o m e a barra.
d) soma da fora peso da barra com a fora magntica entre o m e a barra.
e) soma das foras peso da barra e magntica entre o m e a barra, menos a
fora elstica da mola do dinammetro.







404
21. (UECE) Uma escada est apoiada entre uma parede vertical sem atrito e o cho (horizontal), conforme mostra a
figura a seguir.



Considerando que a escada se comporta como uma barra homognea de 5 m e peso 100 N, e sabendo que o
coeficiente de atrito esttico entre a escada e o cho 0,5, a distncia mxima x que a base da escada pode estar da
parede, sem deslizar, , aproximadamente?


22. (UFU MG) Um recipiente cilndrico vazio foi pendurado em uma mola de massa desprezvel. Diferentes
quantidades de gua foram sendo colocadas nesse cilindro para a determinao da constante elstica da mola.
O grfico abaixo mostra a fora F aplicada mola pelo peso do cilindro com gua como funo da elongao (x) da
mola. Quando havia 2,1 kg de gua no cilindro, a mola apresentava 10 cm de elongao.



Considerando g = 10 m/s2, a alternativa que fornece a massa do cilindro (vazio) e a constante elstica da mola,
respectivamente, ?

23. (UNIFOR CE) Uma escada AB, uniforme, homognea e de peso P, tem sua extremidade A apoiada numa parede
perfeitamente lisa e a outra extremidade B no cho spero. Na posio representada, a escada est prestes a
escorregar.


Nessas condies, a fora de atrito entre o cho e a escada vale?


24. (FUVEST SP) Para vencer o atrito e deslocar um grande continer C, na direo indicada, necessria uma fora
F = 500 N. Na tentativa de mov-lo, blocos de massa m = 15 kg so pendurados em um fio, que esticado entre o
continer e o ponto P na parede, como na figura. Para movimentar o continer, preciso pendurar no fio, no mnimo,

a) 1 bloco
b) 2 blocos
c) 3 blocos
d) 4 blocos
e) 5 blocos



405









PROF.:MAXMILIANO N.C.






C
C
o
o
n
n
t
t
e
e

d
d
o
o
s
s
:
:

C
C
i
i
n
n
e
e
m
m

t
t
i
i
c
c
a
a
E
E
s
s
c
c
a
a
l
l
a
a
r
r
.
.






M
M
i
i
n
n
i
i
s
s
t
t
r
r
a
a
d
d
o
o
s
s
d
d
u
u
r
r
a
a
n
n
t
t
e
e
o
o
m
m

s
s
d
d
e
e
a
a
b
b
r
r
i
i
l
l














D Da at ta a d da a E En nt tr re eg ga a : : _ __ __ __ __ __ __ __ _/ /_ __ __ __ __ __ __ __ __ __ __ _/ /2 20 01 13 3
LISTA 3 FSICA

406


Referencial e Deslocamento

Referencial.: Um corpo est em repouso quando a distncia entre este corpo e o referencial no muda com o tempo.
Um corpo est em movimento quando a distncia entre este corpo e o referencial muda com o tempo.
Deslocamento.: O deslocamento de um mvel determinado pela diferena entre sua posio final e sua posio
inicial.
S = Sf - Si
S = deslocamento (m, km)

ALGUMAS MEDIDAS DE COMPRIMENTO

a) Uma polegada equivale a 2,54 cm (origem anglo-saxnica)
b) Um p equivale a 12 polegadas (30,48 cm)
c) Uma jarda equivale a 3 ps (91,4 cm)
d) Uma milha equivale a 1760 jardas (1609 m)
e) um palmo equivale a 22 cm
f) Um cbito equivale a 3 palmos (66 cm)
g) Uma vara equivale a 5 palmos (1,10 m)
h) Uma braa equivale a 10 palmos ( 2,20 m)
i) Uma lgua equivale a 3 mil braas ( 6600 m)
j) Um quilmetro equivale a 1000 m
l) Um are equivale a 100 m
m) Um alqueire mineiro equivale a 48.400 m
n) Um alqueire paulista equivale a 24.200m

Questes:

1.Partindo do ponto A, uma pessoa caminha, passando na ordem,
pelos pontos B, C, D, B e E, onde pra. Calcule o espao
percorrido e o deslocamento da pessoa nesse trajeto,
respectivamente.Novamente ser preciso usar o teorema de
Pitgoras para encontrar a medida BE.






2.Uma pessoa, em um carro, observa uma placa na calada de uma rua, ao passar por ela. A placa est em repouso
ou em movimento? Explique.

3. Um cachorro, partindo do ponto A, d voltas em torno do terreno ABCD, cuja forma de trapzio indicada na figura.
a)Qual o espao percorrido e o deslocamento do cachorro em uma volta completa?
b)Qual o espao percorrido e o deslocamento no percurso ABC?
Caderno de Atividades

Disciplina:
Fsica

Professor(a):
Maximiliano
Aluno:
3 ano
Ensino Mdio
Data de Recebimento:
_____/_____/_____
Lista 03

Data Entrega:
_____/_____/_____


407

4.Um carro parte do km 12 de uma rodovia e desloca-se sempre no mesmo sentido at o km 90. Determine o
deslocamento do carro.

5. Sobre o cho de um elevador coloca-se um trenzinho de brinquedo, em movimento circular. O elevador sobe com
velocidade constante. Que tipo de trajetria descreve o trenzinho, em relao:
Ao elevador?
Ao solo?

6.Um caminho fez uma viagem a partir do km 30 de uma rodovia at o km 120 da mesma. Depois retorna ao km 10.
Qual foi o deslocamento do caminho?

7. Um produtor rural resolve verificar a cerca de sua propriedade, acompanhado de seu co, parte do ponto A, d uma
volta em torno do terreno ABCDEA, enquanto seu co ao chegar no ponto C atalha pela propriedade at o ponto E e a
partir da chega junto de seu dono na origem do movimento.
a) Quanto mede o lado CD?
b)Qual o espao percorrido e o deslocamento em uma volta completa para o homem e para o co?


8. (Enem 2008) A figura abaixo mostra um reservatrio de gua na forma de um cilindro circular reto, com 6 m de
altura. Quando est completamente cheio, o reservatrio suficiente para abastecer, por um dia, 900 casas cujo
consumo mdio dirio de 500 litros de gua. Suponha que, um certo dia, aps uma campanha de conscientizao do
uso da gua, os moradores das 900 casas abastecidas por esse reservatrio tenham feito economia de 10% no
consumo de gua. Nessa situao,

a) a quantidade de gua economizada foi de 4,5 m3.
b) a altura do nvel da gua que sobrou no reservatrio, no final do dia, foi igual a 60 cm.
c) a quantidade de gua economizada seria suficiente para abastecer, no mximo, 90 casas cujo consumo dirio fosse
de 450 litros.
d) os moradores dessas casas economizariam mais de R$ 200,00, se o custo de 1 m3 de gua para o consumidor
fosse igual a R$ 2,50.
e) um reservatrio de mesma forma e altura, mas com raio da base 10% menor que o representado, teria gua
suficiente para abastecer todas as casas.

Movimento uniforme
Velocidade mdia e funo horria
No movimento uniforme a velocidade constante em qualquer instante.
s = so + v.t
s = posio em um instante qualquer (m, km)
so = posio inicial (m, km)
v = velocidade (m/s, km/h)
t = tempo (s, h)


408
9. (UFRN) Um mvel desloca-se em MRU, cujo grfico ( v X t ) est representado na figura ao lado. Determine o valor
do deslocamento do mvel entre os instantes t=0s e t=4,0s.












10.Uma bicicleta movimenta-se sobre uma trajetria retilnea segundo a funo horria s=10+2t (no SI). Pede-se: A)
sua posio inicial; B) sua velocidade.

11. (UERJ)Um foguete persegue um avio, ambos com velocidades constantes e mesma direo. Enquanto o foguete
percorre 4,0 km, o avio percorre apenas 1,0 km. Admita que, em um instante t1, a distncia entre eles de 4,0 km e
que, no instante t2, o foguete alcana o avio.No intervalo de tempo t2 t1, a distncia percorrida pelo foguete, em
quilmetros, corresponde aproximadamente a:

a) 4,7
b) 5,3
c) 6,2
d) 8,6

12.Uma partcula move-se em linha reta, obedecendo funo horria s = -5 + 20t, no S.I. Determine: A) a posio
inicial da partcula; B) a velocidade da partcula; C) a posio da partcula no instante t = 5 s.

13. (UFG GO)O tempo de reao o tempo entre a percepo de um evento e o incio efetivo da reao. As pessoas
com condies fisiolgicas normais apresentam tempo de reao da ordem de 0,75 segundos. Uma pessoa com
alguma alterao fisiolgica tem este tempo aumentado para 2,0 segundos. Admitindo-se que, no trnsito, a distncia
de segurana entre dois veculos a 72 km/h seja de 15 m no primeiro caso, qual deve ser esta distncia para o
segundo caso, ou seja, com tempo de reao de 2,0 segundos?
a) 20 m
b) 28 m
c) 33 m
d) 36 m
e) 40 m

14. (UNESP) O atleta jamaicano Usain Bolt foi um dos grandes protagonistas dos Jogos Olmpicos de Pequim. Ele
bateu seu prprio recorde mundial dos 100 metros com o tempo de 9,69 segundos e, na prova dos 200 metros rasos,
ele registrou o tempo de 19,3 segundos. Se Bolt corresse a prova de 200 metros rasos com a mesma velocidade
mdia com que correu a prova dos 100 metros, ele teria completado a prova em
a) 15,4 segundos.
b) 19,4 segundos.
c) 25,5 segundos.
d) 29,3 segundos.
e) 30,4 segundos.

15.(UERJ) Uma partcula se afasta de um ponto de referncia O, a partir de uma posio inicial A, no instante t = 0 s,
deslocando-se em movimento retilneo e uniforme, sempre no mesmo sentido.A distncia da partcula em relao ao
ponto O, no instante t = 3,0 s, igual a 28,0 m e, no instante t = 8,0 s, igual a 58,0m. Determine a distncia, em
metros, da posio inicial A em relao ao ponto de referncia O.

16.Um mvel passa pela posio 10 m no instante zero (t0 = 0) com a velocidade de +5 m/s. Escreva a funo horria
desse movimento.

17.Numa noite de neblina, um carro, sem nenhuma sinalizao, percorre um trecho retilneo de uma estrada com
velocidade constante de 6 m/s. Em um certo instante, uma moto com velocidade constante de 8 m/s est 12 m atrs do
carro. Quanto tempo aps esse instante a moto poder chocar-se com o carro?


409
18. Determine o instante e a posio de encontro dos mveis. Dois mveis percorrem a mesma trajetria e suas
posies em funo do tempo so dadas pelas equaes: sA = 30 - 80t e sB = 10 + 20t (no SI). Determine o instante e
a posio de encontro dos mveis

Acelerao
a = v/t
v = v - v0
t = t - t0
Velocidade em funo do tempo
v = v0 + a.t
Posio em funo do tempo
s = s0 + v0.t + 1/2 at2
Equao de Torricelli
v2 = v02 + 2.a. s

19. (Enem 2009)-O Brasil pode se transformar no primeiro pas das Amricas a entrar no seleto grupo das naes que
dispem de trens-bala. O Ministrio dos Transportes prev o lanamento do edital de licitao internacional para a
construo da ferrovia de alta velocidade Rio-So Paulo. A viagem ligar os 403 quilmetros entre a Central do Brasil,
no Rio, e a Estao da Luz, no centro da capital paulista, em uma hora e 25 minutos.
Disponvel em: http://oglobo.globo.com.
Acesso em: 14 jul. 2009.
Devido alta velocidade, um dos problemas a ser enfrentado na escolha do trajeto que ser percorrido pelo trem o
dimensionamento das curvas. Considerando-se que uma acelerao lateral confortvel para os passageiros e segura
para o trem seja de 0,1 g, em que g a acelerao da gravidade (considerada igual a 10 m/s2), e que a velocidade do
trem se mantenha constante em todo o percurso, seria correto prever que as curvas existentes no trajeto deveriam ter
raio de curvatura mnimo de, aproximadamente.
A 80 m.
B 430 m.
C 800 m.
D 1.600 m.
E 6.400 m.

20.(UNESP) O atleta jamaicano Usain Bolt foi um dos grandes protagonistas dos Jogos Olmpicos de Pequim. Ele
bateu seu prprio recorde mundial dos 100 metros com o tempo de 9,69 segundos e, na prova dos 200 metros rasos,
ele registrou o tempo de 19,3 segundos. Se Bolt corresse a prova de 200 metros rasos com a mesma velocidade
mdia com que correu a prova dos 100 metros, ele teria completado a prova em?

21.(UFG GO) O tempo de reao o tempo entre a percepo de um evento e o incio efetivo da reao. As pessoas
com condies fisiolgicas normais apresentam tempo de reao da ordem de 0,75 segundos. Uma pessoa com
alguma alterao fisiolgica tem este tempo aumentado para 2,0 segundos. Admitindo-se que, no trnsito, a distncia
de segurana entre dois veculos a 72 km/h seja de 15 m no primeiro caso, qual deve ser esta distncia para o
segundo caso, ou seja, com tempo de reao de 2,0 segundos?

22.(UFRJ) Um avio vai decolar em uma pista retilnea. Ele inicia seu movimento na cabeceira da pista com velocidade
nula e corre por ela com acelerao mdia de 2,0 m/s2 at o instante em que levanta voo, com uma velocidade de 80
m/s, antes de terminar a pista.
a) Calcule quanto tempo o avio permanece na pista desde o incio do movimento at o instante em que levanta voo.
b) Determine o menor comprimento possvel dessa pista.

23.(FMABC) O movimento de um corpo descrito pela funo: S = 5t2 30t + 50 (unidades do Sistema Internacional).
Esse corpo inverte o sentido de seu movimento no instante de ?

24.(UERJ) Um trem de brinquedo, com velocidade inicial de 2 cm/s,
acelerado durante 16 s.
O comportamento da acelerao nesse intervalo de tempo mostrado
no grfico a seguir.


Calcule, em cm/s, a velocidade do corpo imediatamente aps esses
16 s.



410
25.(UFCG PB) dever de todo/a cidado/ respeitar as regras de trnsito, a vida prpria e a dos outros, o que no faz
um motorista alcoolizado direo. Como exemplo, considere um motorista viajando a 72km/h que observando o sinal
vermelho, aplica instantaneamente os freios, e para em 10 segundos, justamente na borda da faixa de pedestres.
Suponha que, num outro dia, cometendo a imprudncia de consumir bebida alcolica e dirigir e viajando mesma
velocidade e exatamente na mesma estrada e no mesmo ponto, ele observa a mudana de cor do sinal para o
vermelho. Acontece que agora ele demora 0,20 segundo at aplicar os freios. Considerando que o carro freie com a
mesma acelerao anterior, pode-se afirmar que avana sobre a faixa de pedestre quantos metros?

26.(UFPE) Um motorista dirige um carro com velocidade constante de 80 km/h, em linha reta, quando percebe uma
lombada eletrnica indicando a velocidade mxima permitida de 40 km/h. O motorista aciona os freios, imprimindo
uma desacelerao constante, para obedecer sinalizao e passar pela lombada com a velocidade mxima
permitida. Observando-se a velocidade do carro em funo do tempo, desde o instante em que os freios foram
acionados at o instante de passagem pela lombada, podemos traar o grfico abaixo. Determine a distncia
percorrida entre o instante t = 0, em que os freios foram acionados, e o instante t = 3,0 s, em que o carro ultrapassa a
lombada. D sua resposta em metros.


Movimento Circular Uniforme

27-(Enem-2006) Na preparao da madeira em uma industria de moveis, utiliza-se uma
lixadeira constituda de quatro grupos de polias, como ilustra o esquema ao lado. Em cada
grupo, duas polias de tamanhos diferentes so interligadas por uma correia provida de lixa.
Uma prancha de madeira e empurrada pelas polias, no sentido A B (como indicado no
esquema), ao mesmo tempo em que um sistema e acionado para frear seu movimento, de
modo que a velocidade da prancha seja inferior a da lixa. O equipamento acima descrito
funciona com os grupos de polias girando da seguinte forma:
a) 1 e 2 no sentido horario; 3 e 4 no sentido anti-horario.
b) 1 e 3 no sentido horario; 2 e 4 no sentido anti-horario.
c)1 e 2 no sentido anti-horario; 3 e 4 no sentido horario.
d) 1 e 4 no sentido horario; 2 e 3 no sentido anti-horario.
e)1, 2, 3 e 4 no sentido anti-horario.


28. (ENEM) Observe o fenmeno indicado na tirinha.

Adaptado. Luisa Daou & Francisco Caruso, Tirinhas de Fsica, vol. 2, CBPF, Rio de Janeiro, 2000.)
A fora que atua sobre o peso e produz o deslocamento vertical da garrafa a fora
a) de inrcia. b) gravitacional. c) de empuxo. d) centrpeta. e) elstica.






411














PROF.:MAXMILIANO N.C.




C
C
o
o
n
n
t
t
e
e

d
d
o
o
s
s
:
:

L
L
a
a
n
n

a
a
m
m
e
e
n
n
t
t
o
o
s
s
.
.






M
M
i
i
n
n
i
i
s
s
t
t
r
r
a
a
d
d
o
o
s
s
d
d
u
u
r
r
a
a
n
n
t
t
e
e
o
o
m
m

s
s
d
d
e
e
m
m
a
a
i
i
o
o















D Da at ta a d da a E En nt tr re eg ga a : : _ __ __ __ __ __ __ __ _/ /_ __ __ __ __ __ __ __ __ __ __ _/ /2 20 01 13 3
LISTA 4 FSICA

412

01.(UFF RJ) Aps um ataque frustrado do time adversrio, o goleiro se prepara para lanar a bola e armar um
contraataque. Para dificultar a recuperao da defesa adversria, a bola deve chegar aos ps de um atacante no
menor tempo possvel. O goleiro vai chutar a bola, imprimindo sempre a mesma velocidade, e deve controlar apenas o
ngulo de lanamento. A figura mostra as duas trajetrias possveis da bola num certo momento da partida.


Assinale a alternativa que expressa se possvel ou no determinar qual destes dois jogadores receberia a bola no
menor tempo. Despreze o efeito da resistncia do ar.
a) Sim, possvel, e o jogador mais prximo receberia a bola no menor tempo.
b) Sim, possvel, e o jogador mais distante receberia a bola no menor tempo.
c) Os dois jogadores receberiam a bola em tempos iguais.
d) No, pois necessrio conhecer os valores da velocidade inicial e dos ngulos de lanamento.
e) No, pois necessrio conhecer o valor da velocidade inicial.

02.(FUVEST SP)Uma menina, segurando uma bola de tnis, corre com velocidade constante, de mdulo igual a 10,8
km/h, em trajetria retilnea, numa quadra plana e horizontal. Num certo instante, a menina, com o brao esticado
horizontalmente ao lado do corpo, sem alterar o seu estado de movimento, solta a bola, que leva 0,5 s para atingir o
solo. As distncias sm e sb percorridas, respectivamente, pela menina e pela bola, na direo horizontal, entre o
instante em que a menina soltou a bola (t = 0 s) e o instante t = 0,5 s, valem:

a) sm = 1,25 m e sb = 0 m. b) sm = 1,25 m e sb = 1,50 m. c) sm = 1,50 m e sb = 0 m.
d) sm = 1,50 m e sb = 1,25 m. e) sm = 1,50 m e sb = 1,50 m.

03 - (UFTM)Num jogo de vlei, uma atacante acerta uma cortada na bola no instante em que a bola est parada numa
altura h acima do solo. Devido ao da atacante, a bola parte com velocidade inicial V0, com componentes horizontal
e vertical, respectivamente em mdulo, Vx = 8 m/s e Vy = 3 m/s, como mostram as figuras 1 e 2.


Aps a cortada, a bola percorre uma distncia horizontal de 4 m, tocando o cho no ponto P.

Considerando que durante seu movimento a bola ficou sujeita apenas fora gravitacional e adotando g = 10 m/s2, a
altura h, em m, onde ela foi atingida de ?
Caderno de Atividades

Disciplina:
Fsica

Professor(a):
Maximiliano
Aluno:
3 ano
Ensino Mdio
Data de Recebimento:
_____/_____/_____
Lista 04

Data Entrega:
_____/_____/_____

413
04 - (PUC RJ)Um objeto lanado horizontalmente de um penhasco vertical, com uma velocidade inicial vhorizontal =
10 m/s.
Ao atingir o solo, o objeto toca um ponto situado a 20 m da base do penhasco.
Indique a altura H (em metros) do penhasco considerando que a acelerao da gravidade g = 10 m/s2 e
desprezando a resistncia do ar ser?.

05.(UEG GO) Um objeto de massa M lanado obliquamente com um ngulo de 60 com a horizontal, conforme
mostra a figura abaixo. Desconsidere a resistncia do ar.



a) O trabalho realizado pela fora gravitacional no objeto nulo, negativo ou positivo na subida? E na descida?
Justifique sua resposta.
b) Durante o processo de descida, qual a relao do trabalho da fora gravitacional com as mudanas dos valores
das energias potencial gravitacional e cintica? Justifique sua resposta.
c) No ponto B, qual o mdulo da velocidade do objeto?


06.(UEPG PR)Um projtil quando lanado obliquamente, no vcuo, ele descreve uma trajetria parablica. Essa
trajetria resultante de uma composio de dois movimentos independentes. Analisando a figura abaixo, que
representa o movimento de um projtil lanado obliquamente, assinale o que for correto.



01. As componentes da velocidade do projtil, em qualquer instante nas direes x e y, so respectivamente dadas
por, Vx = V0 cosu e Vy = V0 senu gt
02. As componentes do vetor posio do projtil, em qualquer instante, so dadas por, x = V0 cosu t e y = V0 senu

2
1
gt2
04. O alcance do projtil na direo horizontal depende da velocidade e do ngulo de lanamento.
08. O tempo que o projtil permanece no ar t = 2
g
sen . V
o
u

16. O projtil executa simultaneamente um movimento variado na direo vertical e um movimento uniforme na direo
horizontal.

07. (UFOP MG)Uma pessoa lana uma pedra do alto de um edifcio com
velocidade inicial de 60m/s e formando um ngulo de 30 com a horizontal,
como mostrado na figura abaixo. Se a altura do edifcio 80m, qual ser o
alcance mximo (xf) da pedra, isto , em que posio horizontal ela atingir
o solo? (dados: sen 30 = 0,5, cos 30 = 0,8 e g = 10 m/s2).




414
08.(UFMS) Uma bola de bilhar de massa m lanada horizontalmente com velocidade Vo da borda de uma mesa que
est a uma altura H do solo tambm horizontal. A acelerao da gravidade no local g e uniforme, veja a figura.
Considerando que o ar exerce uma fora Fa de arrasto na bola dada pelo formalismo vetorial Fa = bV, onde b uma
constante de proporcionalidade, e V o vetor velocidade da bola vista de um referencial inercial, assinale a(s)
proposio(es) correta(s).


( ) A trajetria da bola no ser uma parbola.
( ) A componente da velocidade da bola na direo horizontal permanece constante durante a queda.
( ) A fora de arrasto sempre vertical para cima.
( ) O alcance A na horizontal igual a Vo(2H/g)1/2.
( ) A intensidade do vetor acelerao da bola vai diminuindo durante a queda.

09.(UNIR RO) O lanamento do dardo uma modalidade olmpica do atletismo praticada desde os jogos olmpicos da
antiga Grcia. Nas olimpadas de 2016, no Rio de Janeiro, atletas do sexo masculino devero arremessar uma lana
de 800 g de massa; j os do sexo feminino arremessaro uma lana de 600 g de massa. Considere que o dardo saia
das mos dos atletas com velocidades de 90 km/h (mulheres) e 108 km/h (homens) e que as posies verticais inicial e
final da lana so as mesmas. Sem considerar a resistncia do ar, qual o alcance mximo esperado para homens e
mulheres, respectivamente?Considere: g = 10 m/s2

10.(UFU MG) Em um dado instante t0, um mssil lanado do solo, com velocidade inicial de 120 m/s formando um
ngulo de 30 em relao ao plano horizontal. Um lanador de antimsseis est posicionado a certa distncia d,
conforme a figura.



O valor de d igual posio horizontal em que o mssil atinge seu ponto mais alto na trajetria. Alguns instantes aps
o lanamento do mssil, um antimssil lanado verticalmente com velocidade v0A.
Considere g = 10 m/s2, despreze velocidade a resistncia do ar e considere tanto o mssil quanto o antimssil como
pontos materiais.

Dado:
1,7 3 =


Com base nessas informaes, faa o que se pede.

a) Determine o valor da posio horizontal d.
b) Calcule em que instante aps o lanamento do mssil, o antimssil deve ser lanado para atingir o mssil com uma
velocidade de 80 m/s.

11. Duas bolinhas idnticas A e B partem ao mesmo tempo de uma certa
altura h acima do solo, sendo que A cai em queda
livre e B tem uma velocidade v0 horizontal. Assinalar a alternativa correta.
a) As duas chegam juntas ao solo.
b) A chega primeiro ao solo.
c) A chega logo depois de B.
d) A ou B chega primeiro, dependendo da velocidade inicial
v0 de B.
e) A ou B chega primeiro, dependendo da altura do lanamento.





415

12. (FUVEST) Em decorrncia de fortes chuvas, uma cidade do interior paulista ficou isolada. Um avio sobrevoou a
cidade,
com velocidade horizontal constante, largando 4 pacotes de alimentos, em intervalos de tempos iguais. No caso ideal,
em
que a resistncia do ar pode ser desprezada, a figura que melhor poderia representar as posies aproximadas do
avio e dos pacotes em um mesmo instante :




13. (FUVEST-adaptada) Durante um jogo de futebol, um chute forte, a partir do cho, lana a bola contra uma parede
prxima. Com auxlio de uma cmara digital, foi possvel reconstruir a trajetria da bola desde o instante em que ela
atingiu a altura mxima (ponto A) at o ponto que bateu na parede (ponto B). Os pontos A e B esto representados na
figura sem escala. Desprezar a resistncia do ar e considerar g = 10m/s2. Determinar:
a) O tempo gasto pela bola para se deslocar do ponto A ao ponto B.
b) A velocidade da bola no instante em que passa por A.
c) A intensidade da velocidade da bola no instante em que passa por B.



14. Um corpo de massa m lanado obliquamente no vcuo com velocidade inicial 100m/s, que forma um ngulo de
60 com a horizontal. Com relao ao movimento desse corpo, so feitas 3 afirmaes. Indicar as que esto corretas,
desprezando- se a resistncia do ar.
I No ponto mais alto do lanamento, a velocidade mnima e vale 50 m/s.
II As velocidades do corpo ao passar pelos pontos A e B de mesma altura apresentam a mesma intensidade.
III Se o corpo lanado de uma superfcie horizontal, o tempo de subida igual ao de descida.



416
15. A figura desta questo mostra uma esfera lanada com velocidade horizontal de 5m/s de uma plataforma de altura
1,8m. Ela deve cair dentro do pequeno frasco colocado a uma distancia X do p da plataforma A distancia X deve ser
de,
aproximadamente?


16. Um homem sobre uma plataforma aponta sua arma na direo de um objeto parado no ar e situado na mesma
horizontal a 200m de distancia, como mostra o esquema. No instante em que a arma disparada, o objeto, que
inicialmente se encontrava a 80m do solo, inicia seu movimento de queda. Desprezando a resistncia do ar e adotando
g = 10m/s2, determine a velocidade mnima que deve ter a bala para atingir o objeto.





17. Uma bola est parada sobre o gramado de um campo horizontal, na posio A. Um jogador chuta a bola para cima,
imprimindo-lhe velocidade v0 de modulo 8m/s, fazendo com a horizontal um ngulo de 60, como mostra a figura. A
bola sobe e desce, atingindo o solo novamente, na posio B. Desprezando-se a resistncia do ar, q


ual ser a distancia entre as posies A e B? (Considere g = 10m/s2).

18. Em um jogo de basquete, uma bola arremessada por um atleta em direo a cesta, distante 10m, a uma
velocidade de 20m/s e fazendo ngulo de 45 com a horizontal. Com que velocidade a bola chega no alvo,
considerando que, na hora do arremesso, a mo do atleta est no mesmo nvel horizontal que a cesta? Adote g =
10m/s2 e despreze os efeitos do ar. (Dado: sen 45 = cos 45 = 2/2).



417

19.(FUC-MT) Um corpo lanado verticalmente para cima com uma velocidade inicial de vo= 30 m/s. Sendo g=10
m/s2 e desprezando a resistncia do ar qual ser a velocidade do corpo 2,0 s aps o lanamento?


20.(UECE) De um corpo que cai livremente desde o repouso, em um planeta X, foram tomadas
fotografias de mltipla exposio razo de 1 200 fotos por minuto. Assim, entre duas posies
vizinhas, decorre um intervalo de tempo de 1/20 de segundo. A partir das informaes constantes
da figura, podemos concluir que a acelerao da gravidade no planeta X, expressa em metros por
segundo ao quadrado, de?






21.(MACK-SP) Uma equipe de resgate se encontra num helicptero, parado em relao ao solo a
305 m de altura. Um pra-quedista abandona o helicptero e cai livremente durante 1,0 s, quando
abre-se o pra-quedas. A partir desse instante, mantendo constante seu vetor velocidade, o pra-
quedista atingir o solo em: (Dado: g=10 m/s2)
a) 7,8 s b) 15,6 s c) 28 s d) 30 s e) 60 s


22.(Fameca-SP) De um avio descrevendo uma trajetria paralela ao solo, com velocidade v, abandonada uma
bomba de uma altura de 2 000 m do solo, exatamente na vertical que passa por um observador colocado no solo. O
observador ouve o estouro da bomba no solo depois de 23 segundos do lanamento da mesma. So dados:
acelerao da gravidade g=10 m/s2; velocidade do som no ar: 340 m/s. A velocidade do avio no instante do
lanamento da bomba era, em quilmetros por hora, um valor mais prximo de?



23.Um menino lana uma bola verticalmente para cima do nvel da rua. Uma
pessoa que est numa sacada a 10 m acima do solo apanha essa bola
quando est a caminho do cho. Sabendo-se que a velocidade inicial da bola
de 15 m/s, pode-se dizer que a velocidade da bola, ao ser apanhada pela
pessoa, era de







24. (ENEM) O nibus espacial Atlantis foi lanado ao espao com cinco astronautas a bordo e uma cmera nova, que
iria substituir uma outra danificada por um curto-circuito no telescpio Hubble. Depois de entrarem em rbita a 560 km
de altura, os astronautas se aproximaram do Hubble. Dois astronautas saram da Atlantis e se dirigiram ao
telescpio.Ao abrir a porta de acesso, um deles exclamou: Esse telescpio tem a massa grande, mas o peso
pequeno.

Considerando o texto e as leis de Kepler, pode-se afirmar que a frase dita pelo
astronauta
a) se justifica porque o tamanho do telescpio determina a sua massa,
enquanto seu pequeno peso decorre da falta de ao da acelerao da
gravidade.
b) se justifica ao verificar que a inrcia do telescpio grande comparada
dele prprio, e que o peso do telescpio pequeno porque a atrao
gravitacional criada por sua massa era pequena.
c) no se justifica, porque a avaliao da massa e do peso de objetos em
rbita tem por base as leis de Kepler, que no se aplicam a satlites
artificiais.
d) no se justifica, porque a fora-peso a fora exercida pela gravidade terrestre, neste caso, sobre o telescpio e
a responsvel por manter o prprio telescpio em rbita.
e) no se justifica, pois a ao da fora-peso implica a ao de uma fora de reao contrria, que no existe naquele
ambiente. A massa do telescpio poderia ser avaliada simplesmente pelo seu volume.



418
25.(UEPB) Um marceneiro est trabalhando na cobertura de um edifcio. Por descuido, o martelo de massa 300 g
escapa de sua mo e cai verticalmente. Sabendo-se que a velocidade do martelo imediatamente antes de tocar o solo
de 25 m/s num tempo de queda igual a 2 s e considerando a acelerao da gravidade 10m/s2, a altura do edifcio,
em metros, ?

26. (PUC RJ) Um objeto lanado verticalmente para cima a partir do solo.
Sua velocidade descrita no grfico abaixo.



A altura mxima atingida pelo objeto em metros de ?

27.(UEFS BA) Um objeto foi abandonado do sexto andar de um prdio, a vinte metros do solo, causando um acidente.
A percia determinou a velocidade com que o objeto chegou ao solo. Considerando-se o mdulo da acelerao da
gravidade local, 10,0m/s2, e desprezando-se a resistncia do ar, o corpo atingiu o solo com velocidade, em km/h, igual
a?





















419










PROF.:MAXMILIANO N. C.




C
C
o
o
n
n
t
t
e
e

d
d
o
o
s
s
:
:

E
E
n
n
e
e
r
r
g
g
i
i
a
a
e
e
C
C
o
o
n
n
s
s
e
e
r
r
v
v
a
a

o
o
.
.







M
M
i
i
n
n
i
i
s
s
t
t
r
r
a
a
d
d
o
o
s
s
d
d
u
u
r
r
a
a
n
n
t
t
e
e
o
o
m
m

s
s
d
d
e
e
s
s
e
e
t
t
e
e
m
m
b
b
r
r
o
o














D Da at ta a d da a E En nt tr re eg ga a : : _ __ __ __ __ __ __ __ _/ /_ __ __ __ __ __ __ __ __ __ __ _/ /2 20 01 13 3

LISTA 5 FSICA

420

Trabalho; Energia ; Impulso e Quantidade de movimento

Trabalho.: Quando aplicamos uma fora sobre um corpo, provocando um deslocamento, estamos gastando energia,
estamos realizando um trabalho.

Energia Mecnica.: A energia mecnica (Em) a soma da energia cintica e potencial em um ponto. A energia
mecnica permanece constante (EmA = EmB) enquanto o corpo sobe ou desce.
Em = Ec + Ep
Ep = m.g.h
Ec = 1/2.m.v2
EmA = EmB
EmA = EcA + EpA
EmB = EcB + EpB


Trabalho




1. Um corpo de massa 5kg e retirado de um ponto A e levado para um ponto B, distante 40m na horizontal e 30m na
vertical traadas a partir do ponto A. Qual e o modulo do trabalho realizado pela forca peso?

2.(UFRS) Num sistema de referncia inercial, exercida uma fora resultante sobre um corpo de massa igual a 0,2 kg,
que se encontra inicialmente em repouso. Essa fora resultante realiza sobre o corpo um trabalho de 1 J, produzindo
nele apenas movimento de translao. No mesmo sistema de referncia, qual o mdulo da velocidade adquirida pelo
corpo em conseqncia do trabalho realizado sobre ele?

3. Calcular o trabalho realizado por uma fora de 28 N que desloca um objeto numa distncia de 2 m na mesma
direo e sentido da fora.

4.(Enem-2009) Os motores eltricos so dispositivos com diversas aplicaes, dentre elas, destacam-se aquelas que
proporcionam conforto e praticidade para as pessoas. E inegvel a preferncia pelo uso de elevadores quando o
objetivo o transporte de pessoas pelos andares de prdios elevados. Nesse caso, um dimensionamento preciso da
potncia dos motores utilizados nos elevadores muito importante e deve levar em considerao fatores como
economia de energia e segurana.
Considere que um elevador de 800 kg, quando lotado com oito pessoas ou 600 kg, precisa ser projetado. Para tanto,
alguns parmetros devero ser dimensionados. O motor ser ligado rede eltrica que fornece 220 volts de tenso. O
elevador deve subir 10 andares, em torno de 30 metros, a uma velocidade constante de 4 metros por segundo. Para
fazer uma estimativa simples da potncia necessria e da corrente que deve ser fornecida ao motor do elevador para
ele operar com lotao mxima, considere que a tenso seja contnua, que a acelerao da gravidade vale 10 m/s2 e
que o atrito pode ser desprezado. Nesse caso, para um elevador lotado, a potncia mdia de sada do motor do
elevador e a corrente eltrica mxima que passa no motor sero respectivamente de

a) 24kW e 109A. d) 180 kW e 818A.
b) 32kW e 145A. e) 240 kW e 1090 A.
c) 56kW e 255A.


Caderno de Atividades

Disciplina:
Fsica

Professor(a):
Maximiliano
Aluno:
3 ano
Ensino Mdio
Data de Recebimento:
_____/_____/_____
Lista 05

Data Entrega:
_____/_____/_____

421
5.Uma fora constante de 20 N produz, em um corpo, um deslocamento de 0,5 m no mesmo sentido da fora. Calcule
o trabalho realizado por essa fora.

6. Aplica-se uma fora horizontal de 10 N sobre um corpo que desloca-se numa trajetria retilnea de acordo com a
equao s = 10 + 3t + t2, no SI. Calcule o trabalho realizado pela fora em 5 s.

7. (Vunesp) Deslocando-se por uma rodovia a 108 km/h (30 m/s), um motorista chega praa de pedgio e passa a
frear o carro a uma taxa constante, percorrendo 150 m, numa trajetria retilnea, at a parada do veculo.
Considerando a massa total do veculo como sendo 1000 kg, o mdulo do trabalho realizado pelas foras de atrito que
agem sobre o carro, em joules, igual a?

8.(UFR-RJ) Um corpo de massa 2,0 kg sofre a ao de um conjunto de foras e sua velocidade varia em mdulo de
2,0 m/s para 12 m/s. Determine o trabalho realizado pela resultante desse conjunto de foras.

9.(ENEM) O Sol representa uma fonte limpa e inesgotvel de energia para o nosso planeta. Essa energia pode ser
captada por aquecedores solares, armazenada e convertida posteriormente em trabalho til. Considere determinada
regio cuja insolao potncia solar incidente na superfcie da Terra seja de 800 watts/m2. Uma usina
termossolar utiliza concentradores solares parablicos que chegam a dezenas de quilmetros de extenso. Nesses
coletores solares parablicos, a luz refletida pela superfcie parablica espelhada focalizada em um receptor em
forma de cano e aquece o leo contido em seu interior a 400 C. O calor desse leo transferido para a gua,
vaporizando-a em uma caldeira. O vapor em alta presso movimenta uma turbina acoplada a um gerador de energia
eltrica.


Considerando que a distncia entre a borda inferior e a borda superior da superfcie refletora tenha 6 m de largura e
que focaliza no receptor os 800 watts/m2 de radiao provenientes do Sol, e que o calor especfico da gua 1 cal g1
C1 = 4.200 J kg1 C1, ento o comprimento linear do refletor parablico necessrio para elevar a temperatura de 1
m3 (equivalente a 1 t) de gua de 20 C para 100 C, em uma hora, estar entre
a) 15 m e 21 m.
b) 22 m e 30 m.
c) 105 m e 125 m.
d) 680 m e 710 m.
e) 6.700 m e 7.150 m.

10. (Enem 2007) A mochila que gera eletricidade, ela tem uma
estrutura rgida semelhante usada por alpinistas. Onde um
compartimento de carga suspenso por molas colocadas na
vertical. Durante a caminhada, os quadris sobem e descem em
mdia cinco centmetros. A energia produzida pelo vai-e-vem do
compartimento de peso faz girar um motor. Com o projeto da
mochila pretende-se aproveitar, na gerao de energia eltrica
para acionar dispositivos eletrnicos portteis, parte da energia
desperdiada no ato de caminhar. As transformaes de energia
envolvidas na produo de eletricidade enquanto uma pessoa
caminha com essa mochila podem ser assim esquematizadas conectado ao gerador de eletricidade.
GAs energias I e II, representadas no esquema acima, podem ser identificadas, respectivamente, como:
a) cintica e eltrica.
b)trmica e cintica.
c) trmica e eltrica.
d) sonora e trmica.
e)radiante e eltrica.

Molas


422
11. (Enem-2009) A eficincia de um processo de converso de energia, definida como sendo a razo entre a
quantidade de energia ou trabalho til e a quantidade de energia que entra no processo, e sempre menor que 100%
devido a limitaes impostas por leis fsicas. A tabela a seguir, mostra a eficincia global de vrios processos de
converso.
HINRICHS, R. A.; KLEINBACH, M. Energia e meio ambiente. Sao Paulo: Pioneira Thomson Learning, 2003 (adaptado).
Se essas limitaes no existissem, os sistemas mostrados na tabela, que
mais se beneficiariam de investimentos em pesquisa para terem suas
eficincias aumentadas, seriam aqueles que envolvem as transformaes de
energia:
(A) mecnica em energia eltrica.
(B) nuclear em energia eltrica.
(C) qumica em energia eltrica.
(D) Qumica em energia trmica.
(E) radiante em energia eltrica.




12. (FMJ SP) Para levar um pacote de 100 kg ao alto de uma rampa inclinada em 30, ele foi amarrado a um fio que,
depois de passar por uma polia, preso no eixo de um motor de 250 W de potncia. Quando acionado, o motor dever
pux-lo em linha reta e com velocidade constante.



Considerando o fio e a polia ideais, desprezando todos os atritos e adotando g = 10 m/s2, quando puxado pelo motor,
o pacote subir a rampa com uma velocidade, em m/s, igual a

13.(Vunesp-SP) Numa rampa de 4,0m de altura, uma caixa de massa igual a 15 kg desce a partir do repouso, devido
ao atrito uma parte da energia mecnica inicial perdida, assim, a criana chega no solo com velocidade de apenas 4
m/s. Calcule a diferena entre as energias mecnicas inicial e final e diga quanto de energia foi perdida como calor
devido ao atrito. Sendo 10 m/s2, o mdulo da acelerao da gravidade local.


14.( ITA ) Um homem de 79,0 kg de massa, est sobre uma superfcie sem atrito. Ele tem na mo um revlver cuja
massa de 1,0 kg dispara duas vezes horizontalmente sua frente. Cada projtil tem massa de 10,0 g e velocidade
inicial de 200 m/s. Sua velocidade de recuo aps os dois disparos ser?


15. Um bloco de massa m = 4,00 kg desliza sobre um plano horizontal sem atrito com velocidade de 2m/s e choca-se
com uma mola horizontal de massa desprezvel, e constante elstica k = 100 N/m, presa a uma parede vertical.
Calcule a compresso mxima da mola



16. O bloco b1 de massa igual a 1,0kg e velocidade de 8,0 m.s-1 colide com um bloco Identico B2, inicialmente em
repouso. Aps a coliso ambos os blocos ficam grudados e sobem a rampa at comprimir a mola M de 0,10m.
Desprezando os atritos e considerando g =10 m.s-2, h = 0,50m e =30, pergunta-se qual o valor da constante da
mola.






423
17.(Enem-2006) A Terra e cercada pelo vcuo espacial e, assim, ela s perde energia ao irradi-la para o espao. O
aquecimento global que se verifica hoje decorre de pequeno desequilbrio energtico, de cerca de 0,3%, entre a
energia que a Terra recebe do Sol e a energia irradiada a cada segundo, algo em torno de 1 W/m2. Isso significa que a
Terra acumula, anualmente, cerca de 1,6 1022 J. Considere que a energia necessria para transformar 1 kg de gelo
a 0 C em gua liquida seja igual a 3,2 105 J. Se toda a energia acumulada anualmente fosse usada para derreter o
gelo nos plos (a 0 C), a quantidade de gelo derretida anualmente, em trilhes de toneladas, estaria entre
A 20 e 40.
B 40 e 60.
C 60 e 80.
D 80 e 100.
E 100 e 120.

18.(ITA) Projetado para subir com velocidade mdia constante a uma altura de 32 m em 40 s, um elevador consome a
potncia de 8,5 kW de seu motor. Considere seja de 370 kg a massa do elevador vazio e a acelerao da gravidade g
= 10 m/s2. Nessas condies, o nmero mximo de passageiros, de 70 kg cada um, a ser transportado pelo elevador
?

19.(Enem-2009) Alem de ser capaz de gerar eletricidade, a energia solar e usada para muitas outras finalidades.
Afigura a seguir mostra o uso da energia solar para dessalinizar a gua. Nela, um tanque contendo gua salgada e
coberto por um plstico transparente e tem a sua parte central abaixada pelo peso de uma pedra, sob a qual se col oca
um recipiente (copo). A gua evaporada se condensa no plstico e escorre ate o ponto mais baixo, caindo dentro do
copo.
HINRICHS, R. A.; KLEINBACH, M. Energia e meio ambiente. So Paulo: Pioneira Thomson Learning, 2003 (adaptado).
Nesse processo, a energia solar cedida a agua salgada
(A) fica retida na gua doce que cai no copo, tornando-a, assim, altamente energizada.
(B) fica armazenada na forma de energia potencial gravitacional contida na gua doce.
(C) e usada para provocar a reao qumica que transforma a gua salgada em gua doce.
(D) e cedida ao ambiente externo atravs do plstico, onde ocorre a condensao do vapor.
(E) e reemitida como calor para fora do tanque, no processo de evaporao da gua salgada

20. (UFF RJ) Um projtil de massa m = 10g viaja horizontalmente com a velocidade
m/s 10 x 0 , 1 v
2
=
. Com esta
velocidade, o mesmo atinge um bloco de massa M= 0,99 kg, que est em repouso na beirada de uma mesa cujo
tampo encontra-se a uma altura h = 80 cm do cho, como mostra a figura. O projtil se aloja no bloco e o conjunto cai
da mesa. Considere desprezveis as dimenses do bloco e do projtil quando comparadas com as da mesa. Suponha
2
m/s 10 g =
.

a) Qual a razo entre os mdulos das foras horizontais que atuam sobre o projtil e o bloco durante a coliso?
b) Com que velocidade, em mdulo e direo, o conjunto sai da mesa?
c) Qual o mdulo da velocidade do conjunto ao atingir o solo?
d) A que distncia da base da mesa o conjunto atinge o solo?


21.(UFTM) Na mina de carvo, um vago carregado de minrio se desprende do cabo que o mantinha parado em um
trecho de aclive, descendo aceleradamente. Mais frente, em um trecho horizontal do trilho, choca-se com um
segundo vago identicamente carregado, atrelando-se a este. O conjunto passa a se mover nesse trecho horizontal
com velocidade de 5 m/s. Considere desprezvel qualquer fora dissipativa como o atrito ou a resistncia do ar.

Dados: massa de um desses vages = 1 000 kg
acelerao da gravidade local = 10 m/s2

a) Determine a velocidade que possua o vago desgovernado, momentos antes do choque com o vago estacionado.
b) Calcule a profundidade que deve ter descido o vago desgovernado at o ponto em que realizou a coliso.

22.(UDESC) Uma partcula com massa de 200 g abandonada, a partir do repouso, no ponto A da Figura 1.
Desprezando o atrito e a resistncia do ar, pode-se afirmar que as velocidades nos pontos B e C so,
respectivamente:


424
Figura 1

a) 7,0 m/s e 8,0 m/s
b) 5,0 m/s e 6,0 m/s
c) 6,0 m/s e 7,0 m/s
d) 8,0 m/s e 9,0 m/s
e) 9,0 m/s e 10,0 m/s

23. (FGV) Em festas de aniversrio, um dispositivo bastante simples
arremessa confetes. A engenhoca constituda essencialmente por um tubo de papelo e uma mola helicoidal
comprimida. No interior do tubo esto acondicionados os confetes. Uma pequena toro na base plstica do tubo
destrava a mola que, em seu processo de relaxamento, empurra, por 20 cm, os confetes para fora do dispositivo.



Ao serem lanados com o tubo na posio vertical, os confetes atingem no mximo 4 metros de altura, 20% do que
conseguiriam se no houvesse a resistncia do ar. Considerando que a poro de confetes a ser arremessada tem
massa total de 10 g, e que a acelerao da gravidade seja de 10 m/s2, o valor da constante elstica da mola utilizada
, aproximadamente, em N/m?

24. (UFAL) Um estudante de peso 600 N salta de bungee jumping de uma ponte a uma distncia considervel do solo
(ver figura). Inicialmente, a corda elstica atada aos seus tornozelos est totalmente sem tenso (energia potencial
elstica nula). O estudante cai, a partir do repouso, uma distncia vertical mxima de 40 m, em relao ao seu ponto
de partida. Desprezando-se as variaes de energia cintica e potencial da corda elstica ideal, bem como as perdas
de energia por dissipao, qual a energia potencial elstica armazenada na corda quando o estudante se encontra no
ponto mais baixo da sua trajetria?



25. (UFG GO) Uma das competies dos X-games so as manobras dos esqueitistas em uma rampa em U. Um atleta
parte do repouso do topo da rampa e atravs do movimento do seu corpo, de peso 800 N, consegue ganhar 600 J a
cada ida e vinda na rampa, conforme ilustrao a seguir.



Desprezando as perdas de energia e o peso do skate, o nmero mnimo de idas e vindas que o atleta deve realizar
para atingir uma altura (h) de 3 m acima do topo da rampa ?








425











PROF.:Maxmiliano N. C.




C
C
o
o
n
n
t
t
e
e

d
d
o
o
s
s
:
:

H
H
i
i
d
d
r
r
o
o
s
s
t
t

t
t
i
i
c
c
a
a
e
e
h
h
i
i
d
d
r
r
o
o
d
d
i
i
n
n

m
m
i
i
c
c
a
a
.
.






M
M
i
i
n
n
i
i
s
s
t
t
r
r
a
a
d
d
o
o
s
s
d
d
u
u
r
r
a
a
n
n
t
t
e
e
o
o
m
m

s
s
d
d
e
e
o
o
u
u
t
t
u
u
b
b
r
r
o
o
















D Da at ta a d da a E En nt tr re eg ga a : : _ __ __ __ __ __ __ __ _/ /_ __ __ __ __ __ __ __ __ __ __ _/ /2 20 01 13 3
LISTA 6 FSICA

426


Presso e Lei de Steven
A aplicao de uma fora sobre uma determinada rea gera uma fora a qual chamamos presso

1.(ENEM) Durante uma obra em um clube, um grupo de trabalhadores teve de remover uma escultura de ferro macio
colocada no fundo de uma piscina vazia. Cinco trabalhadores amarraram cordas escultura e tentaram pux-la para
cima, sem sucesso. Se a piscina for preenchida com gua, ficar mais fcil para os trabalhadores removerem a
escultura, pois a

a) escultura flutuar. Dessa forma. os homens no precisaro fazer fora para remover a escultura do fundo.
b) escultura ficar com peso menor, Dessa forma, a intensidade da fora necessria para elevar a escultura ser
menor.
c) gua exercer uma fora na escultura proporcional a sua massa, e para cima. Esta fora se somar fora que os
trabalhadores fazem para anular a ao da fora peso da escultura.
d) gua exercer uma fora na escultura para baixo, e esta passar a receber uma fora ascendente do piso da
piscina. Esta fora ajudar a anular a ao da fora peso na escultura.
e) gua exercer uma fora na escultura proporcional ao seu volume, e para cima. Esta fora se somar fora que os
trabalhadores fazem, podendo resultar em uma fora ascendente maior que o peso da escultura.


2.(UFF RJ) O sifo um instrumento usado para a retirada de gua de lugares de difcil acesso. Como mostra a figura
abaixo, seu funcionamento se baseia no fato de que, quando o tubo que liga os recipientes A e B est cheio, h uma
diferena de presso hidrosttica entre os pontos P e Q, o que provoca um fluxo de gua de A para B.
Essa diferena de presso depende da seguinte caracterstica do nosso planeta:



Essa diferena de presso depende da seguinte caracterstica do nosso planeta:

a) presso atmosfrica.
b) acelerao da gravidade local.
c) temperatura da superfcie.
d) densidade da atmosfera.
e) velocidade de rotao do planeta.

03.(UEG GO) Em 15 de abril de 1875, na Frana, o balo Zenith voou a uma altitude de 8.600 m. Dois dos seus
tripulantes morreram em decorrncia das mudanas funcionais promovidas pela altitude. Sobre esses tipos de
mudanas numa pessoa saudvel e normal, CORRETO afirmar:
a) os efeitos apenas sero sentidos em altitudes superiores a 8000 m, quando a frequncia respiratria aumenta
drasticamente.
b) o que ocasionou a morte dos dois tripulantes foi um efeito conhecido como hipoxia, ou seja, o alto fornecimento de
oxignio.
c) os efeitos se devem essencialmente diminuio da presso atmosfrica, o que consequncia da diminuio da
densidade do ar.
d) j em baixas altitudes, prximas de 1.000 m, surgem uma srie de distrbios, como dificuldade de respirar,
taquicardia, nusea, vmito e insnia.


Caderno de Atividades

Disciplina:
Fsica

Professor(a):
Maximiliano
Aluno:
3 ano
Ensino Mdio
Data de Recebimento:
_____/_____/_____
Lista 06

Data Entrega:
_____/_____/_____

427
04.(UFPR) No dia 20 de abril de 2010, houve uma exploso numa plataforma petrolfera da British Petroleum, no Golfo
do Mxico, provocando o vazamento de petrleo que se espalhou pelo litoral. O poo est localizado a 1500 m abaixo
do nvel do mar, o que dificultou os trabalhos de reparao. Suponha a densidade da gua do mar com valor constante
e igual a 1,02 g/cm3 e considere a presso atmosfrica igual a 1,00 105 Pa. Com base nesses dados, calcule a
presso na profundidade em que se encontra o poo diga quantas vezes essa presso mltipla da presso
atmosfrica.

05.(UFAL) Uma torneira que no foi fechada corretamente pinga gua a uma taxa constante de 10 mL = 105 m3 por
minuto. A gua derramada dentro de uma lata cilndrica, de rea da base 80 cm2 = 8 103 m2, inicialmente vazia.
A torneira pinga durante 40 minutos, quando, ento, fechada totalmente. Aps o fechamento da torneira, qual a
presso na base da lata devido gua derramada? Dados: o volume de um cilindro igual ao produto da rea da base
pela sua altura; densidade da gua = 103 kg/m3; acelerao da gravidade = 10 m/s2.

06. (UERJ) A figura a seguir representa um fio AB de comprimento igual a 100 cm, formado de duas partes
homogneas sucessivas: uma de alumnio e outra, mais densa, de cobre.
Uma argola P que envolve o fio deslocada de A para B.



Durante esse deslocamento, a massa de cada pedao de comprimento
AP
medida. Os resultados esto
representados no grfico abaixo:



A razo entre a densidade do alumnio e a densidade do cobre aproximadamente igual a?


07 .(UFABC) Impacto Ambiental - Hoje, os produtos da moderna tecnologia esto incorporados ao cotidiano das
pessoas, mas a sua fabricao causa impactos nocivos ao meio ambiente. preciso conhecer sua dimenso para
control-los.Para fabricar um nico microchip de 32 megabites de memria (figura 1) usam-se 1,6 kg de combustvel
fssil e 72 gramas de substncias qumicas (Enciclopdia do Estudante, Estado).
necessria ainda toda a gua contida em um prisma reto de base quadrada (figura 2), com sua capacidade total
preenchida.



Sabendo-se que a densidade da gua, ou massa por unidade de volume, de 1g/mL, pode-se concluir que a massa
da gua usada para fabricar esse microchip igual a ?


428
08.(UEL PR) A massa de um corpo de 60 g e seu volume de 100 cm3. Considere que esse corpo esteja flutuando
em equilbrio na gua. Qual a porcentagem de seu volume que ficar acima da superfcie da gua?
Considere a densidade da gua igual a 1 g/cm3.

09.(UNICAMP SP) O vazamento de petrleo no Golfo do Mxico, em abril de 2010, foi considerado o pior da histria
dos EUA. O vazamento causou o aparecimento de uma extensa mancha de leo na superfcie do oceano, ameaando
a fauna e a flora da regio. Estima-se que o vazamento foi da ordem de 800 milhes de litros de petrleo em cerca de
100 dias.

Quando uma reserva submarina de petrleo atingida por uma broca de perfurao, o petrleo tende a escoar para
cima na tubulao como consequncia da diferena de presso, AP, entre a reserva e a superfcie. Para uma reserva
de petrleo que est a uma profundidade de 2000 m e dado g = 10 m/s2, o menor valor de AP para que o petrleo de
densidade = 0,90 g/cm3 forme uma coluna que alcance a superfcie de?


10. (UERJ) A maior profundidade de um determinado lago de gua doce, situado ao nvel do mar, igual a 10,0 m. A
presso da gua, em atmosferas, na parte mais funda desse lago, de cerca de?

11.(UEL PR) Dois recipientes cilndricos idnticos, de paredes termicamente isoladas, com tampas mveis sem atrito e
de pesos desprezveis (mbolos), contm em seus interiores volumes idnticos V0 de gs ideal a mesma presso
atmosfrica P0 e temperatura T0. No tempo inicial t0, um dos recipientes, que se encontrava inicialmente no meio
atmosfrico, colocado na posio p1 e no interior de um tubo de Venturi e o outro na posio p2 como
esquematizado na figura a seguir:



No tubo de Venturi de seco transversal A1 > A2 um lquido com densidade igual dgua escoa laminarmente com
velocidade de mdulo constante v2 = 2v1.
No tempo t1 > t0, os dois cilindros atingem suas configuraes de equilbrio.

Nos esquemas a seguir assinale a alternativa que melhor representa a configurao de equilbrio dos cilindros.

a)

429
b)
c)
d)
e)




430
12. (UFTM) No interior de dois vasos comunicantes, foi colocada gua, assumindo, aps o repouso, os nveis indicados
pelo desenho.



Considerando que um dos vasos aberto para o ar atmosfrico e que a gua se comporte como um fluido ideal,
correta a comparao

a) PA > PB > PC > PD.
b) PB > PA > PC > PD.
c) PB > PA = PD > PC.
d) PB = PC > PA = PD.
e) PD > PC > PB > PA.


13.(UNIFOR CE) A chamada camada pr-sal uma faixa que se estende ao longo de 800 quilmetros entre os
Estados do Esprito Santo e Santa Catarina, abaixo do leito do mar, e engloba trs bacias sedimentares (Esprito
Santo, Campos e Santos). O petrleo encontrado nesta rea est a profundidades que superam os 7 mil metros,
abaixo de uma extensa camada de sal que, segundo gelogos, conservam a qualidade do petrleo. A profundidade do
leito do oceano, nesta regio, est compreendida entre 1 mil e 2 mil metros. Considere que a densidade mdia da gua
do mar de 1024 kg/m3. Trabalhar a esta profundidade envolve um problema muito complexo que devido ao fato de
a presso hidrosttica ser muito grande. Sabendo que 1 atm vale 105 Pa, a presso hidrosttica entre 1 e 2 mil metros
de profundidade, em termos da presso atmosfrica, situa-se aproximadamente entre:
a) 1000 a 2000 atm
b) 100 a 200 atm
c) 50 a 100 atm
d) 10 a 20 atm
e) 5 a 10 atm


14.(UNESP) As barragens em represas so projetadas para suportar grandes massas de gua. Na situao
representada na figura, temos uma barragem de largura 40 m, retendo uma massa de gua de 30 m de profundidade.
Conhecendo-se o comportamento da presso com a altura da coluna de um fluido e levando-se em conta que a
presso atmosfrica age dos dois lados da barragem, possvel determinar a fora horizontal da gua da represa
sobre a barragem.



Considere a presso atmosfrica como 1 atm 1,0 105 Pa, a densidade da gua gua = 1,0 103 kg/m3 e a
acelerao da gravidade g ~ 10 m/s2. Qual das alternativas melhor representa a variao da presso com a altura h da
gua em relao superfcie, e a fora horizontal exercida por essa massa de gua sobre a barragem?


431
a) F ~ 1,0 108 N.

b) F ~ 1,8 108 N.
c) F ~ 2,0 108 N.
d) F ~ 3,6 108 N.
e) F ~ 7,2 108 N.

15.(PUC RJ) Um avio utilizado na ponte area entre Rio e So Paulo capaz de voar horizontalmente com uma
carga mxima de 62.823,0 kg. Sabendo que a rea somada de suas asas de 105,4 m2 correto afirmar que a
diferena de presso nas asas da aeronave, que promove a sustentao durante o voo, de?
(Considere g = 10,0 m/s2)

16.(UNIOESTE PR) Um tubo em U, com dimetro uniforme, contm mercrio, cuja massa especfica igual a 13,6
g/cm3. Despeja-se num dos ramos do tubo um lquido imiscvel com o mercrio at que a altura do lquido atinja 20 cm
acima do nvel do mercrio deste mesmo ramo. O nvel do mercrio no outro ramo sobe 2 cm em relao ao nvel
inicial. A massa especfica do lquido introduzido no tubo vale?

17.(FAMECA SP) A presso arterial de um vestibulando foi medida por seu mdico e resultou 15,2 por 7,6 cmHg.
Esse jovem quis converter essas medidas em valores correspondentes a colunas de gua. So dados: 76 cmHg = 1,0
105 Pa = 1,0 105 N/m2, densidade absoluta da gua 1,0 103 kg/m3, acelerao da gravidade 10 m/s2. Os valores
obtidos corretamente, em metros, foram

a) 2,0 e 1,0.
b) 20 e 10.
c) 50 e 25.
d) 100 e 50.
e) 200 e 100.



432
18 (ENEM) Uma eclusa um canal que, construdo em guas de um rio com grande desnvel, possibilita a
navegabilidade, subida ou descida de embarcaes. No esquema abaixo, est representada a descida de uma
embarcao, pela eclusa do porto Primavera, do nvel mais alto do rio Paran at o nvel da jusante.



A cmara dessa eclusa tem comprimento aproximado de 200m e largura igual a 17m. A vazo aproximada da gua
durante o esvaziamento da cmara de 4.200 m3 por minuto.
Assim, para descer do nvel mais alto at o nvel da jusante, uma embarcao leva cerca de

a) 2 minutos. b) 5 minutos. c) 11 minutos. d) 16 minutos. e) 21 minutos.


19.(UNIOESTE PR) No sistema da figura abaixo, a poro AC contm mercrio, BC contem leo e o tanque aberto
atmosfera contm gua. As alturas indicadas so:
cm 10 h
0
=
,
cm 5 h
1
=
,
cm 20 h
2
=
e as densidades so:
-3 3
Hg
m kg 10 x 6 , 13 =
e
-3 3
leo
m kg 10 x 8 , 0
. O ponto A interno ao recipiente que contm mercrio. Nestas condies,
pode-se afirmar que



a) a presso no nvel B maior que a do nvel C.
b) a presso no nvel C menor que a do ponto A.
c) a presso no ponto A igual presso no nvel O, na superfcie da gua.
d) a presso no ponto A de 7,42 x 104 Pa.
e) a presso no ponto A de 1,286 x 105 Pa.


20.(UNICAMP SP) Acidentes de trnsito causam milhares de mortes todos os anos nas estradas do pas. Pneus
desgastados (carecas), freios em pssimas condies e excesso de velocidade so fatores que contribuem para
elevar o nmero de acidentes de trnsito. Responsvel por 20% dos acidentes, o uso de pneu careca considerado
falta grave e o condutor recebe punio de 5 pontos na carteira de habilitao. A borracha do pneu, entre outros
materiais, constituda por um polmero de isopreno (C5H8) e tem uma densidade igual a 0,92 g cm3. Considere que
o desgaste mdio de um pneu at o momento de sua troca corresponda ao consumo de 31 mols de isopreno e que a
manta que forma a banda de rodagem desse pneu seja um retngulo de 20 cm x 190 cm. Para esse caso especfico, a
espessura gasta do pneu seria de, aproximadamente?

433
Dados de massas molares em g mol1 : C=12 e H =1.


21. Qual a vazo de gua (em litros por segundo) circulando atravs de um tubo de 32 mm de dimetro, considerando
a velocidade da gua como sendo 4 m/s? Lembre-se que 1 m3 = 1000 litros

22. Qual a velocidade da gua que escoa em um duto de 25 mm se a vazo de 2 litros/s?

23. Qual a velocidade da gua atravs de um furo na lateral de um tanque, se o desnvel entre o furo e a superfcie
livre de 2 m?



24. (ENEM) No nova a idia de se extrair energia dos oceanos aproveitando-se a diferena das mars alta e baixa.
Em 1967, os franceses instalaram a primeira usina mar-motriz, construindo uma barragem equipada de 24 turbinas,
aproveitando-se a potncia mxima instalada de 240 MW, suficiente para a demanda de uma cidade com 200 mil
habitantes. Aproximadamente 10% da potncia total instalada so demandados pelo consumo residencial. Nessa
cidade francesa, aos domingos, quando parcela dos setores industrial e comercial para, a demanda diminui 40%.
Assim, a produo de energia correspondente demanda aos domingos ser atingida mantendo-se
I. todas as turbinas em funcionamento, com 60% da capacidade mxima de produo de cada uma delas.
II. a metade das turbinas funcionando em capacidade mxima e o restante, com 20% da capacidade mxima.
III. quatorze turbinas funcionando em capacidade mxima, uma com 40% da capacidade mxima e as demais
desligadas.
Est correta a situao descrita

a) apenas em I. b) apenas em II. c) apenas em I e III. d) apenas em II e III. e) em I, II e III.

25.(ENEM). O carneiro hidrulico ou ariete, dispositivo usado para bombear gua, no requer combustvel ou energia
eltrica para funcionar, visto que usa a energia da vazo de gua de uma fonte. A figura a seguir ilustra uma instalao
tpica de carneiro em um sitio, e a tabela apresenta dados de seu funcionamento.



A eficincia energtica de um carneiro pode ser obtida pela expresso:
f
b
V
V
x
h
H
= c
, cujas variveis esto definidas na
tabela e na figura.


434
Se, na situao apresentada,
h x 5 H=
, ento, mais provvel que, aps 1 hora de funcionamento ininterrupto, o
carneiro hidrulico bombeie para a caixa dgua
a) de 70 a 100 litros de gua. b) de 75 a 210 litros de gua. c) de 80 a 220 litros de gua.
d) de 100 a 175 litros de gua. e) de 110 a 240 litros de gua.

26. (UFBA) Em um recipiente transparente, cuja rea da seco transversal igual a S1, feita uma pequena abertura
A, de rea S2, a uma altura h2, sendo S1 muito maior que S2.
Deve-se encontrar a altura da gua, h1, de modo que, ao escoar pela pequena abertura, o filete de gua atinja um tubo
de ensaio a uma distncia x do recipiente, como mostra a figura.



Sabe-se que a altura h1 pode ser determinada usando-se a equao de Bernoulli
2
V
gh P
2

+ +
constante em qualquer
ponto do fluido, sendo
P a presso esttica externa, no caso a presso atmosfrica;

a densidade do fluido;
V a velocidade em um ponto do fluido;
h a altura no ponto do fluido de velocidade V;
g a acelerao da gravidade local.
Com base nessas informaes, considerando que as velocidades V1 e V2, nos pontos 1 e 2, respectivamente, so tais
que V1 S1 = V2 S2 e desprezando quaisquer foras dissipativas, mostre, utilizando a equao de Bernoulli, que
g 2
V
h h
2
2
2 1
+ ~
e calcule h1, para x=1m e h2=0,5m.

27.(UNESP) Ao sofrer um corte, notamos que o sangue escorre de nosso corpo; isso ocorre pelo fato de que a presso
sangnea maior que a atmosfrica. Assim, comum em hospitais cenas como a representada na figura. Sabendo
que a presso interna total do sangue cerca de 2,5% maior que a do valor da presso atmosfrica, sendo esta, ao
nvel do mar, aproximadamente
Pa 10 x 1,0
5
, e supondo que a densidade da soluo salina a ser injetada no paciente,
bem como a do sangue, sejam iguais a
3 3
kg/m 1,0 x 1,0
, e
m/s 10 g
2
=
, determine a altura h mnima a partir da qual todo
o soro poder entrar na corrente sangnea do paciente.






435












PROF.: Moiss




C
C
o
o
n
n
t
t
e
e

d
d
o
o
s
s
:
:

- Dilatao Trmica;
- Calorimetria.




M
M
i
i
n
n
i
i
s
s
t
t
r
r
a
a
d
d
o
o
s
s
d
d
u
u
r
r
a
a
n
n
t
t
e
e
o
o
m
m

s
s
d
d
e
e
f
f
e
e
v
v
e
e
r
r
e
e
i
i
r
r
o
o















D Da at ta a d da a E En nt tr re eg ga a : : _ __ __ __ __ __ __ __ _/ /_ __ __ __ __ __ __ __ __ __ __ _/ /2 20 01 13 3
LISTA 1 FSICA

436

Questo 01)A figura ao lado ilustra uma esfera macia de dimetro L e uma barra de mesmo material com
comprimento tambm igual a L, ambos a uma mesma temperatura inicial. Quando a temperatura dos dois corpos for
elevada para um mesmo valor final, a razo entre o aumento do dimetro da esfera e o aumento do comprimento da
barra ser:

a) 1/3
b) 1
c) 1/9
d) 9/1
e) 3/1

Questo 02)Certo lquido est quase transbordando de um bquer de vidro, de capacidade 450 cm
3
a 20C.
Aquecendo-se o conjunto at a temperatura atingir 100C, transbordam 9,0cm
3
do lquido. A dilatao real desse
lquido, em cm
3
, :
a) menor que 0,45
b) 0,45
c) 4,5
d) 9,0
e) maior que 9,0

Questo 03)Uma esfera de ao, oca, foi construda de tal forma que, quando completamente mergulhada em leo
diesel temperatura de 25
o
C, permanece em equilbrio, sem afundar nem emergir. Suponha agora que a temperatura
do sistema, formada pela bola e pelo leo diesel, seja lentamente alterada, de forma que seja sempre mantido o
equilbrio trmico. Sabe-se que o coeficiente de dilatao linear do ao o = 11 x 10
6 o
C
1
e que o coeficiente de
dilatao volumtrica do leo diesel 9,5 x 10
4o
C
-1
. Sobre essa situao, INCORRETO afirmar que:
a) antes da variao da temperatura, a razo entre a massa e o volume da esfera igual densidade do leo diesel.

b) se houver elevao da temperatura, a esfera tender a flutuar.

c) se houver elevao da temperatura, tanto o leo diesel quanto a esfera sofrero dilatao.

d) caso haja diminuio da temperatura do sistema, a razo entre a massa e o volume da esfera se tornar menor do
que a densidade do leo diesel.

e) se houver diminuio da temperatura do sistema, tanto o leo diesel quanto a esfera diminuiro de volume.

Questo 04)Pela manh, com temperatura de 10C, Joo encheu completamente o tanque de seu carro com gasolina
e pagou R$ 33,00. Logo aps o abastecimento deixou o carro no mesmo local, s voltando para busca-lo mais tarde,
quando a temperatura atingiu a marca de 30C.
Sabendo-se que o combustvel extravasou, que o tanque dilatou e que a gasolina custou R$ 1,10 o litro, quanto Joo
perdeu em dinheiro?
Dado: Coeficiente de dilatao trmica da gasolina igual a 1,1 x 10
-3
C
-1
.

Questo 05)Atravs de experimentos, bilogos observaram que a taxa de canto
de grilos de uma determinada espcie estava relacionada com a temperatura
ambiente de uma maneira que poderia ser considerada linear. Experincias
mostraram que, a uma temperatura de 21 C, os grilos cantavam, em mdia, 120
vezes por minuto; e, a uma temperatura de 26 C, os grilos cantavam, em mdia,
180 vezes por minuto. Considerando T a temperatura em graus Celsiuse n o
nmero de vezes que os grilos cantavam por minuto, podemos representar a
relao entre T e n pelo grfico abaixo.

Supondo que os grilos estivessem cantando, em mdia, 156 vezes por minuto, de acordo com o modelo sugerido nesta
questo, estima-se que a temperatura deveria ser igual a:
a) 21,5 C .
b) 22 C .
c) 23 C .
d) 24 C .
e) 25,5 C .

Caderno de Atividades

Disciplina:
Fsica

Professor(a):
Moiss
Aluno:
3 ano
Ensino Mdio
Data de Recebimento:
_____/_____/_____
Lista 01

Data Entrega:
_____/_____/_____

437
Questo 06)Quando uma enfermeira coloca um termmetro clnico de mercrio sob a lngua de um paciente, por
exemplo, ela sempre aguarda algum tempo antes de fazer a sua leitura. Esse intervalo de tempo necessrio.
a) para que o termmetro entre em equilbrio trmico com o corpo do paciente.
b) para que o mercrio, que muito pesado, possa subir pelo tubo capilar.
c) para que o mercrio passe pelo estrangulamento do tubo capilar.
d) devido diferena entre os valores do calor especfico do mercrio e do corpo humano.
e) porque o coeficiente de dilatao do vidro diferente do coeficiente de dilatao do mercrio.

Questo 07)Uma bolinha de ao a 120 C colocada sobre um pequeno cubo de gelo a 0 C. Em escala linear, o
grfico que melhor representa a variao, no tempo, das temperaturas da bolinha de ao e do cubo de gelo, at
alcanarem um estado de equilbrio, :
0
120
a.
t(s)
u ( C)
o

0
120
b.
t(s)
u ( C)
o

0
120
c.
t(s)
u ( C)
o

0
120
d.
t(s)
u ( C)
o

Questo 08)Uma massa de 0,50 kg de gua aquecida em um recipiente durante 21 s, e sua temperatura aumenta
20C.Sabendo que o calor especfico da gua 4,2x10 J.kg
-1
.C
-1
, ao calcular a potncia mdia de aquecimento
fornecida massa de gua encontra-se o seguinte valor:
a) 2,0x10 W
b) 5,0x10 W
c) 2,0x10 W
d) 5,0x10W

Questo 09)Em um calormetro, cuja capacidade trmica de 110 cal/
o
C, que se encontra a 20
o
C, h 220 gramas de
gua mesma temperatura. Um cubo de metal de 300 g, inicialmente a 80
o
C, mergulhado na gua. O sistema atinge
o equilbrio trmico a uma temperatura de 30
o
C.
a) Qual dos metais da tabela abaixo foi utilizado neste experimento?
b) Caso se desejasse uma temperatura final de equilbrio mais elevada do que 30
o
C, deveria ser usado um calormetro
de capacidade trmica maior ou menor do que 110 cal/
o
C? Justifique sua resposta.
Metal Calor Especfico
cal/(g.
o
C.
Alumnio 2,2 x 10
-1

Chumbo 3,1 x 10
-2

Cobre 9,2 x 10
-2

Ferro 1,1 x 10
-1

Prata 5,6 x 10
-2

Tungstnio 3,2 x 10
-2


Questo 10)Quatro balas, feitas de chumbo, cobre, ferro e prata, respectivamente, e de mesma massa, so lanadas
contra uma parede fabricada com um material que um bom isolante trmico.
Todas as balas esto em equilbrio trmico com o ar e tm velocidades diferentes
que sero consideradas horizontais no momento do impacto. Todas as balas
apresentam a mesma temperatura no momento em que o movimento cessa dentro
da parede. Isso significa que, antes do impacto,
a) a de chumbo tinha a maior velocidade.
b) a de cobre tinha a maior velocidade.
c) a de ferro tinha a maior velocidade.
d) a de prata tinha a maior velocidade.
e) todas tinham a mesma velocidade.
Dados
Substncia chumbo cobre ferro prata
calor especfico
(cal / g C) 0,03 0,09 0,11 0,05


438
Questo 11)Uma esfera de chumbo de massa de 250 g est imersa em um banho de gua e em equilbrio trmico a
uma temperatura de 0
o
C. A esfera retirada do banho e levada rapidamente para outro recipiente contendo 250 g de
gua a 100
o
C. A temperatura final de equilbrio ser mais prxima de:

Dados
Substncia chumbo cobre ferro prata
calor especfico
(cal / g C) 0,03 0,09 0,11 0,05


calor especfico da gua = 1cal/gC
a) 100 C
b) 75 C
c) 50 C
d) 25 C
e) 0 C

Questo 12)Os metais usuais, como aqueles utilizados em algumas prteses ortopdicas, dilatam-se quando
aquecidos. Em novembro de 2004 foi anunciada, na literatura cientfica, a descoberta de materiais que se contraem
quando aquecidos.
Suponha que a lei de dilatao de um tal material seja idntica dos metais comuns, exceto pela presena de um
coeficiente de dilatao linear negativo, digamos o.
Imagine uma barra slida de comprimento L, com uma frao f de seu comprimento constituda pelo novo material e a
frao restante, por um metal comum de coeficiente linear de dilatao positivo o.

A fim de que a barra no varie de comprimento sob variaes de temperatura, a frao f deve ser dada por:
a) o / (o + o) .
b) o / o.
c) oo.
d) o / (o + o)
e) o / o.

Questo 13)Uma fonte trmica fornece calor com potncia constante. Ela aquece 100g de gua, de 20
o
C at 50
o
C, em
3,0 min. Para aquecer 250g de um metal, de 25
o
C a 40
o
C, ela gasta 45s. Sendo o calor especfico da gua igual a
1,0cal/g
o
C, o do metal, nas mesmas unidades, vale
a) 0,50
b) 0,40
c) 0,30
d) 0,20
e) 0,10

Questo 14)Um corpo cai de uma altura de 10m e fica em repouso ao atingir o solo. A temperatura do corpo
imediatamente antes do impacto 30C e seu calor especfico 100J/KgC. Supondo que toda a energia mecnica do
corpo foi transformada em calor e que no houve mudana de estado, qual a temperatura final do corpo? (Use g =
10m/s
2
)
a) 29C
b) 31C
c) 311C
d) 30C
e) 40C












439











PROF.: Moiss




C
C
o
o
n
n
t
t
e
e

d
d
o
o
s
s
:
:

- Calorimetria;
- Mudanas de Estado de Agregao.




M
M
i
i
n
n
i
i
s
s
t
t
r
r
a
a
d
d
o
o
s
s
d
d
u
u
r
r
a
a
n
n
t
t
e
e
o
o
m
m

s
s
d
d
e
e
m
m
a
a
r
r

o
o















D Da at ta a d da a E En nt tr re eg ga a : : _ __ __ __ __ __ __ __ _/ /_ __ __ __ __ __ __ __ __ __ __ _/ /2 20 01 13 3
LISTA 2 FSICA

440

Q Questo 01)Suponha que em um recipiente metlico de 200g, termicamente isolado do meio externo e inicialmente
a 20 C, colocaram-se 360g de gua a 60 C. Calcule:

a) a temperatura de equilbrio trmico do sistema gua-recipiente, sabendo-se que o calor especfico da gua 1,0
cal/g C e o do metal 0,20 cal/g C.
b) o valor mximo da massa de uma pedra de gelo a 0 C que, colocada no recipiente, permita que haja apenas gua
quando for restabelecido o equilbrio trmico do sistema, sabendo que o calor latente de fuso do gelo 80 cal/g.

Questo 02) Em um calormetro, cuja capacidade trmica de 110 cal/
o
C, que se encontra a 20
o
C, h 220 gramas de
gua mesma temperatura. Um cubo de metal de 300 g, inicialmente a 80
o
C, mergulhado na gua. O sistema atinge
o equilbrio trmico a uma temperatura de 30
o
C.
a) Qual dos metais da tabela abaixo foi utilizado neste experimento?
b) Caso se desejasse uma temperatura final de equilbrio mais elevada do que 30
o
C, deveria ser usado um calormetro
de capacidade trmica maior ou menor do que 110 cal/
o
C? Justifique sua resposta.
Metal Calor Especfico
cal/(g.
o
C.
Alumnio 2,2 x 10
-1

Chumbo 3,1 x 10
-2

Cobre 9,2 x 10
-2

Ferro 1,1 x 10
-1

Prata 5,6 x 10
-2

Tungstnio3,2 x 10
-2


Questo 03)Uma fonte de energia (trmica) , de potncia constante e igual a 5 cal/s, fornece calor a uma massa slida
de 80 g. O grfico abaixo mostra a variao de temperatura em funo do tempo. O calor especfico do corpo no
estado lquido vale em cal/sC:
Dados:

10 20 30 40 50 60 70 t(s)
50
100
150
200
T( C)
o

a) 0,0125;
b) 0,0250;
c) 0,0200;
d) 0,0400;
e) 0,0500.

Questo 04) Um calormetro, de capacidade trmica desprezvel, contm 200g de gua a 50C. Em seu interior
introduzido um bloco de ferro com massa de 200g a 50C. O calor especfico do ferro 0,11cal/gC. Em seguida, um
bloco de gelo de 500g a 0C tambm colocado dentro do calormetro. O calor especfico da gua de 1cal/gC e o
calor latente de fuso do gelo de 80cal/g. No h trocas de calor com o ambiente. Nestas circunstncias, qual a
temperatura de equilbrio deste sistema, em C?
a) 0
b) 10
c) 20
d) 30
e) 40



Caderno de Atividades

Disciplina:
Fsica

Professor(a):
Moiss
Aluno:
3 ano
Ensino Mdio
Data de Recebimento:
_____/_____/_____
Lista 02

Data Entrega:
_____/_____/_____

441
Questo 05) Um calormetro de capacidade trmica 50 cal/C contm 520 g de gelo a 0 C. Injeta-se no calormetro
vapor de gua a 120 C, na quantidade necessria e suficiente para fundir totalmente o gelo. A massa de gua, em
gramas, que se forma no interior do calormetro vale
Dados:
calor especfico da gua = 1,0 cal/gC
calor especfico do vapor = 0,50 cal/gC
calor latente de fuso do gelo = 80 cal/g
calor latente de vaporizao da gua = 540 cal/g
a) 520
b) 584
c) 589
d) 620
e) 700

Questo 06) Suponha que um meteorito de 1,0 10
12
kgcolida frontalmente com a Terra (6,0 10
24
kg) a 36 000 km/h.
A coliso perfeitamente inelstica e libera enorme quantidade de calor.

a) Que frao da energia cintica do meteorito se transforma em calor e que frao se transforma em energia cintica
do conjunto Terra-Meteorito?
b) Sabendo-se que so necessrios 2,5 10
6
J para vaporizar 1,0 litro de gua, que frao da gua dos oceanos (2,0
10
21
litros) ser vaporizada se o meteoro cair no oceano?


Questo 07) Colocase um cubo de gelo de massa 50 g e temperatura 0C dentre de um copo contendo 200 g de
gua a 70C. Considerando a ocorrncia de trocas de energia apenas entre o gelo e gua, determine a temperatura
final de equilbrio trmico, em C.
Dados:
calor de fuso do gelo = 80cal/g
calor especfico da gua= 1,0cal/gC

Questo 08) Em um dia quente, um atleta corre dissipando 750 W durante 30 min. Suponha que ele s transfira esta
energia para o meio externo atravs de evaporao do suor e que todo seu suor seja aproveitado para sua
refrigerao. Adote L = 2.500 J/g para o calor latente de evaporao da gua na temperatura ambiente.
a) Qual a taxa de perda de gua do atleta em kg/min?
b) Quantos litros de gua ele perde nos 30 min de corrida?

Questo 09) Considere o diagrama de estado de uma substncia e
asafirmaes que seguem.



I. No ponto T, representado no diagrama, a substnciapode simultaneamente
apresentar-se nas fases slida,lquida e gasosa.
II. Se a substncia for aquecida, sob presso constantede 60 cm Hg, desde
a temperatura de 40C at chegara 90C, ela sofrer duas mudanas de fase.
III. Somente pode ocorrer a sublimao dessa substnciase t<40C e p<40
cmHg.

Est correto o que se afirma em
a) II, somente.
b) I e II, somente.
c) I e III, somente.
d) II e III, somente.
e) I, II e III.

Questo 10)Um bloco de gelo com 5 kg de massa encontra-se a -20 C.
Dados: calor especfico: gelo ) ) C ( 50 , 0 c (
1
g

=
-1
g cal , chumbo )) ) C ( 031 , 0 c (
1
c

=
-1
g cal
calor latente de fuso: gelo ) 80 L (
g
-1
g cal = , chumbo ) 9 , 5 L (
c
-1
g cal =
temperatura de fuso: gelo C) 0 T (
g
= , chumbo C) 3 , 327 T (
c
=
a) Calcule a quantidade de calor necessrio para derreter completamente o bloco de gelo.
b) Com o calor necessrio para derreter o bloco de gelo calculado no item acima, qual seria a massa de um bloco de
chumbo que poderia ser derretido, se esse bloco de chumbo estivesse inicialmente, tambm a C 20 ?


442








PROF.: Moiss




C
C
o
o
n
n
t
t
e
e

d
d
o
o
s
s
:
:

- Processos de Transmisso de Calor;
- Gases Perfeitos.





M
M
i
i
n
n
i
i
s
s
t
t
r
r
a
a
d
d
o
o
s
s
d
d
u
u
r
r
a
a
n
n
t
t
e
e
o
o
m
m

s
s
d
d
e
e
a
a
b
b
r
r
i
i
l
l















D Da at ta a d da a E En nt tr re eg ga a : : _ __ __ __ __ __ __ __ _/ /_ __ __ __ __ __ __ __ __ __ __ _/ /2 20 01 13 3
LISTA 3 FSICA

443


Questo 01) A transmisso de calor entre os corpos pode ocorrer por trs processos diferentes. Sobre estes
processos, considere:
I. As trocas de calor por irradiao so resultantes da fragmentao de ncleos de tomos instveis num processo
tambm conhecido por radioatividade.
II. A conduo trmica o processo de transferncia de calor de um meio ao outro atravs de ondas
eletromagnticas.
III. No pode haver propagao de calor nem por conduo, nem por conveco, onde no h meio material.
IV. O fenmeno da inverso trmica ocorre mais freqentemente no inverno e acentua a poluio, j que no ocorre
conveco.
correto o contido em apenas
a) I e II.
b) I e III.
c) II e III.
d) II e IV.
e) III e IV.

Questo 02) O efeito estufa, processo natural de aquecimento da atmosfera, essencial para a existncia de vida na
Terra. Em tal processo, uma parcela da radiao solar refletida e da radiao trmica emitida pela superfcie terrestre
interage com determinados gases presentes na atmosfera, aquecendo-a.
O principal mecanismo fsico responsvel pelo aquecimento da atmosfera devido ao do efeito estufa resulta da
a) absoro, por certos gases da atmosfera, de parte da radiao ultravioleta recebida pela Terra.
b) reflexo, por certos gases da atmosfera, da radiao visvel emitida pela Terra.
c) absoro, por certos gases da atmosfera, de parte da radiao infravermelha proveniente da superfcie da Terra.
d) reflexo, por certos gases da atmosfera, de parte da radiao de microondas recebida pela Terra.

Questo 03) De acordo com a Lei de Stefan-Boltzmann, o equilbrio da atmosfera terrestre obtido pelo balano
energtico entre a energia de radiao do Sol absorvida pela Terra e a reemitida pela mesma. Considere que a energia
fornecida por unidade de tempo pela radiao solar dada por P = A e o T
4
, em que o = 5,67 10
8
W m
2
K
4
; A a
rea da superfcie do corpo; T a temperatura absoluta, e o parmetro e a emissividade que representa a razo entre
a taxa de radiao de uma superfcie particular e a taxa de radiao de uma superfcie de um corpo ideal, com a
mesma rea e mesma temperatura. Considere a temperatura mdia da Terra T = 287K e, nesta situao, e = l.
Sabendo que a emisso de gases responsveis pelo aquecimento global reduza a emissividade, faa uma estimativa
de quanto aumentar a temperatura mdia da Terra devido emisso de gases responsveis pelo aquecimento global,
se a emissividade diminuir 8%.
Considere
4
x
1 ) x 1 (
4 / 1
~

Questo 04)A transferncia de calor ocorre por conduo, por conveco e por irradiao. Sobre estes fenmenos,
assinale o que for correto.
01. Na conduo, o fluxo de calor inversamente proporcional diferena de temperatura entre dois pontos.
02. Na conveco, h transporte de matria.
04. Na irradiao, no ocorre transporte de energia.
08. A irradiao ocorre atravs de ondas eletromagnticas e com predominncia dos raios infravermelhos.
16. Na conduo, necessrio um suporte material para que o fenmeno ocorra.

Questo 05) A garrafa trmica um dispositivo feito para conservar a temperatura de um lquido gelado ou quente.
Sobre a garrafa trmica, correto afirmar:
01. O vcuo existente entre as paredes duplas de vidro espelhado reduz as trocas de calor por irradiao.
02. A quantidade de calor que atravessa as paredes duplas da garrafa determinada pela Lei de Fourier.
03. A radiao trmica que incide nas paredes duplas da garrafa constituda do espectro eletromagntico visvel.
04. As paredes duplas de vidro devem ser pintadas de preto para dificultar a propagao de calor por conduo.
05. As faces externas e internas das paredes de vidro so espelhadas para minimizar o fluxo da radiao trmica,
tanto de dentro para fora quanto de fora para dentro.

Caderno de Atividades

Disciplina:
Fsica

Professor(a):
Moiss
Aluno:
3 ano
Ensino Mdio
Data de Recebimento:
_____/_____/_____
Lista 03

Data Entrega:
_____/_____/_____

444
Questo 06)Observe a figura a seguir sobre a formao das brisas marinhas.




As pessoas que vivem nas proximidades do mar conhecem bem as brisas marinhas, ventos suaves que sopram,
durante o dia, do mar para a Terra, noite, da Terra para o mar.
Dentre as alternativas a seguir, indique a que explica corretamente o fenmeno apresentado:
a) Exemplo de conveco trmica e ocorre pelo fato de a gua ter calor especfico maior que a areia, fazendo com
que a temperatura da areia se altere mais rapidamente que a da superfcie do mar.
b) Exemplo de conduo trmica e ocorre pelo fato dos ventos serem originados por diferentes temperaturas entre a
gua e a terra, em virtude da diferena de seus calores especficos.
c) Exemplo de radiao trmica e ocorre porque durante o dia, ao receber radiaes solares, a terra se aquece mais
rapidamente que a gua do mar.
d) Exemplo de conduo trmica e ocorre pelo fato de a gua ter calor especfico menor que a areia, acarretando a
alterao da temperatura da gua mais rapidamente do que a da superfcie da terra.
e) Exemplo de radiao trmica e ocorre pelo fato de a areia e a gua serem bons condutores trmicos, levando o
calor a dissipar-se rapidamente.

Questo 07) Para a construo de prdios termicamente isolados, necessrio o estudo de processos que
envovlemtransfer~encia de calor. A figura abaixo ilustra duas paredes a e b construdas com diferentes materiais.
Do ponto de vista termodinmico, a taxa de transferncia de calor H , em
regime estacionrio, diretamente proporcional diferena de temperatura
AT nas interfaces da parede e inversamente proporcional resistncia
trmica da parede R , de acordo com as equaes abaixo.

A = =
A = =
b
R
1
3 2
L
A k
b
a
R
1
2 1
L
A k
a
T ) T T ( H
T ) T T ( H
b b
b
a a
a


Nessas equaes, A a rea das interfaces de cada parede, k
a
e k
b
so as
suas condutividades trmicas, L
a
e L
b
so as suas respectivas espessuras, e T
1
, T
2
e T
3
so temperaturas das
interfaces entre o interior e a parede a, entre a parede a e a parede b e entre a parede b e o exterior, respectivamente.
Com base nas informaes acima e nas leis da Termodinmica, julgue os seguintes itens.
01. No equilbrio trmico, quando todas as interfaces das paredes estiverem mesma temperatura, as taxas de
transferncia de calor H
a
e H
b
podero ser diferentes de zero.
02. se T
3
> T
2
> T
1
, o calor fluir do interior para o exterior, independentemente do material de que feita cada parede.
03. Maximizar o isolamento trmico das paredes envolve a procura de materiais de maiores valores de condutividade
trmica.
L
a
k
a
T
2
T
1
interior
parede a parede b
L
b
k
b
T
3
exterior

445
04. Fazendo-se uma analogia do sistema de duas paredes apresentando na figura com um circuito eltrico formado
por dois resistores em srie, ento a diferena de temperatura corresponderia diferena de potencial e a taxa de
transferncia de calor corresponderia corrente eltrica.

Questo 08) Com respeito ao fenmeno da transmisso do calor, assinale o que for correto.
01. Para o calor transferido por radiao, a potncia irradiada por uma superfcie dada pela Lei de Stefan-Boltzmann
P = e oAT
4
.
02. A transmisso de calor por conveco ocorre exclusivamente nos meios lquidos.
04. Bons isolantes trmicos apresentam, em geral, alta condutividade trmica.
08. A conduo do calor, que um dos mecanismos de transmisso do calor, descrita pela equaoQ = kA t
L
T
A
A

16. A quantidade total de energia radiante emitida por um corpo, na unidade de tempo, inversamente proporcional
temperatura desse corpo.

Questo 09) A figura abaixo mostra uma montagem experimental, idealizada por Joule, na qual dois blocos de massa
m, em queda simultnea, fazem que vrias ps girem dentro de um recipiente termicamente isolado (calormetro) e
contendo gua. Nesse experimento, a energia mecnica transformada em energia trmica devido ao atrito viscoso
das ps com a gua, que faz os blocos descerem com velocidade constante. Essa montagem foi utilizada para medir o
equivalente mecnico do calor. Com relao a esse assunto e desprezando as massas das polias e da corda e o atrito
nos eixos das polias, julgue os itens seguintes.



01. Durante a queda dos blocos, a variao da energia cintica das ps igual, em qualquer instante, quantidade de
energia trmica cedida gua.
02. O trabalho realizado pela fora da gravidade durante a queda dos blocos igual soma da energia cintica de
translao dos blocos e da energia de rotao das ps que compem o experimento.
03. Conhecendo-se o deslocamento vertical dos blocos, o mdulo de acelerao da gravidade local, a massa dos
blocos e a massa e o calor especfico da gua, pode-se determinar a elevao da temperatura da gua do sistema
durante a queda dos blocos.
04. A energia trmica armazenada na gua pode ser reconvertida integralmente em energia mecnica.

Questo 10) Duas barras metlicas idnticas, com comprimento L, seo transversal A e condutividade trmica K,
quando colocadas em srie entre dois reservatrios trmicos com temperaturas T
1
e T
2
, conduzem calor razo de 22
J/s. Se as barras so colocadas em paralelo entre os mesmos reservatrios, o fluxo de calor, em J/s, ...

Questo 11) A figura ao lado representa duas barras metlicas, 1 e 2, de mesma seo transversal A e de mesmo
comprimento L, isoladas termicamente do ambiente. As barras esto encostadas uma na outra por uma das
extremidades; a outra extremidade da barra 1 est mergulhada numa cuba que contm



gelo fundente, e a da barra 2 est em contato com uma cmara de temperatura u
C
, mantida constante por um
aquecedor eltrico de resistncia eltrica R submetida a uma ddp igual a U.

446
Considere-se que a conduo trmica nas barras se processa no regime estacionrio,as condutividades trmicas dos
materiais que constituem as barras valem K
1
e K
2
e 1 cal = 4,2 J.
Nessas condies, afirma-se:
01. O fluxo de calor na barra 1 diferente do fluxo de calor na barra 2.
02. Na superfcie de contato entre as barras, o fluxo de calor, em cal/s, igual a .
L
A
2 1
2
1
C
K K
K K
+
-
- - u
04. Na superfcie de contato entre as duas barras, a temperatura igual a .
C
2 1
1
K K
K
u
+

08. A potncia dissipada na resistncia, expressa em watts, .
R
U
2 , 4
2

16. A razo entre a potncia dissipada na resistncia, em watts, e o fluxo de calor, em cal/s, igual a 4,2 .

Questo 12) Um mol de gs ideal, presso de 16,6 atm, ocupa uma caixa cbica cujo volume de 0,001 m
3
. Qual a
temperatura do gs e a fora que o gs exerce sobre a tampa quadrada da caixa?
(Considere PA 10 1,0 atm 0 , 1
-5
= , K J/mol 3 , 8 R = )
a) 100 K e 8,310
3
N
b) 100 K e 16,610
3
N
c) 166 K e 8,310
3
N
d) 200 K e 16,610
3
N
e) 200 K e 8,310
3
N

Questo 13) Sejam o recipiente (1), contendo 1 mol de H
2
(massa molecular M = 2) e o recipiente (2) contendo 1 mol
de He (massa atmica M = 4) ocupando o mesmo volume, ambos mantidos a mesma presso. Assinale a alternativa
correta:
a) A temperatura do gs no recipiente 1 menor que a temperatura do gs no recipiente 2.
b) A temperatura do gs no recipiente 1 maior que a temperatura do gs no recipiente 2.
c) A energia cintica mdia por molcula do recipiente 1 maior que a do recipiente 2.
d) O valor mdio da velocidade das molculas no recipiente 1 menor que o valor mdio da velocidade das molculas
no recipiente 2.
e) O valor mdio da velocidade das molculas no recipiente 1 maior que o valor mdio da velocidade das molculas
no recipiente 2.

Questo 14)
A temperatura de um gs diretamente proporcional energia cintica das suas partculas. Portanto, dois gases A e
B, na mesma temperatura, cujas partculas tenham massas na proporo de m
A
/m
B
=4/1, tero as energias cinticas
mdias das suas partculas na proporo Ec
A
/Ec
B
igual a
a) 1/4
b) 1/2
c) 1
d) 2
e) 4

Questo 15) Uma certa amostra de um gs, com uma massa de 64g, ocupa um volume de 16,4 litros sob uma presso
de 3 atm e uma temperatura de 27C. O nmero de Avogrado vale
23
10 x 6,02 tomos/mol e a constante universal dos
gases 0,082 R = atm.l/mol.K. Nessas condies, o nmero de molculas existentes na amostra gasosa
aproximadamente de:
a) 1,2 x 10
24

b) 2,0 x 10
23

c) 6,0 x 10
23

d) 2,4 x 10
23


Questo 16)Num reservatrio de 32,8 l, indilatvel e isento de vazamentos, encontra-se certa quantidade de oxignio
g/mol) 32 (M= . Alterando-se a temperatura do gs, sua presso varia de acordo com o diagrama ao lado. A massa de
oxignio contida nesse reservatrio

Dado:
K mol
atm
082 , 0 R

=


a) 3,84 . 10
2
g
b) 7,68 . 10
2
g
c) 1,15 . 10
3
g
d) 2,14 . 10
3
g
e) 4,27 . 10
3
g

447
Questo 17)A figura representa uma isoterma correspondente transformao de um gs ideal. Os valores dos
volumes V
1
e V
2
so, respectivamente,

a) 4 L e 9 L.
b) 4 L e 8 L.
c) 3 L e 9 L.
d) 3L e 6 L.





Questo 18) Um gs ideal, inicialmente temperatura de 320 K e ocupando um volume de 4 , 22 , sofre expanso em
uma transformao a presso constante. Considerando que a massa do gs permaneceu inalterada e a temperatura
final foi de 480 K, calcule

a) variao do volume do gs.
b) coeficiente de dilatao volumtrica do gs no incio da transformao.

Questo 19) Uma panela aquecida da temperatura ambiente de 25 C at a temperatura de 100 C. Sabendo que a
presso inicial da panela P
o
e que o volume da panela permaneceu constante durante este processo, podemos
afirmar que:
a) o processo isovolumtrico e a presso final aproximadamente 5P
o
/4.
b) o processo isovolumtrico e a presso final da panela aproximadamente P
o
/3.
c) o processo isobrico e o volume da panela permanece constante.
d) o processo isobrico e apenas a temperatura variou.
e) o processo isovolumtrico e a presso final da panela aproximadamente 3P
o
.

Questo 20) Um tanque industrial, cilndrico, com altura total H
0
= 6,0 m, contm em seu interior gua at uma altura
h
0
, a uma temperatura de 27C (300 K).
O tanque possui um pequeno orifcio A e, portanto, est presso atmosfrica P
0
, como esquematizado em I. No
procedimento seguinte, o orifcio fechado, sendo o tanque invertido e aquecido at 87 C (360 K).
Quando o orifcio reaberto, e mantida a temperatura do tanque, parte da gua escoa, at que as presses no orifcio
se equilibrem, restando no interior do tanque uma altura h
1
= 2,0 m de gua, como em II.

Determine
a) a presso P
1
, em N/m
2
, no interior do tanque, na situao II.
b) a altura inicial h
0
da gua no tanque, em metros, na situao I.

NOTE E ADOTE:
P
atmosfrica
= 1 Pa = 1,0 10
5
N/m
2

(gua) = 1,0 10
3
kg/m
3
; g = 10 m/s
2















448









PROF.: Moiss




C
C
o
o
n
n
t
t
e
e

d
d
o
o
s
s
:
:


- Termodinmica.





M
M
i
i
n
n
i
i
s
s
t
t
r
r
a
a
d
d
o
o
s
s
d
d
u
u
r
r
a
a
n
n
t
t
e
e
o
o
m
m

s
s
d
d
e
e
m
m
a
a
i
i
o
o













D Da at ta a d da a E En nt tr re eg ga a : : _ __ __ __ __ __ __ __ _/ /_ __ __ __ __ __ __ __ __ __ __ _/ /2 20 01 13 3

LISTA 4 FSICA

449


Questo 01) Considere um sistema termodinmico e analise as seguintes afirmativas.
I. Para que a entropia decresa quando um gs ideal sofre uma expanso adiabtica livre, indo de um volume v
1
para
um volume v
2
, v
2
deve ser maior que v
1
.
II. No nvel molecular, a temperatura a grandeza que mede a energia cintica mdia de translao das molculas
de um gs monoatmico e a primeira lei da Termodinmica nos permite definir a energia interna U do sistema.
III. Um processo irreversvel, em termos termodinmicos, graas dissipao de sua energia e variao positiva
de sua entropia.
IV. A segunda lei da Termodinmica pode ser enunciada da seguinte forma: a entropia do universo sempre cresce (ou
permanece constante, em um processo reversvel).

Assinale a alternativa que contm todas as afirmativas corretas.
a) I e II.
b) I e III.
c) II e IV.
d) I, III e IV.
e) II, III e IV.

Questo 02) Um condicionador de ar, funcionando no vero, durante certo intervalo de tempo, consome 1.600 cal de
energia eltrica, retira certa quantidade de energia do ambiente que est sendo climatizado e rejeita 2.400 cal para o
exterior. A eficincia desse condicionador de ar
a) 0,33
b) 0,50
c) 0,63
d) 1,50
e) 2,00

Questo 03) Considere uma mquina trmica operando em um ciclo termodinmico. Esta mquina recebe 300J de
uma fonte quente cuja temperatura de 400K e produz um trabalho de 150J. Ao mesmo tempo, rejeita 150J para uma
fonte fria que se encontra a 300K. A anlise termodinmica da mquina trmica descrita revela que o ciclo proposto
um(a):
a) mquina frigorfica na qual tanto a Primeira Lei quanto a Segunda Lei da termodinmica so violadas.
b) mquina frigorfica na qual a Primeira Lei atendida, mas a Segunda Lei violada.
c) motor trmico no qual tanto a Primeira Lei quanto a Segunda Lei da termodinmica so atendidas.
d) motor trmico no qual a Primeira Lei violada, mas a Segunda Lei atendida.
e) motor trmico no qual a Primeira Lei atendida, mas a Segunda Lei violada.

Questo 04) A cada ciclo, uma mquina trmica extrai 45 kJ de calor da sua fonte quente e descarrega 36 kJ de calor
na sua fonte fria. O rendimento mximo que essa mquina pode ter de
a) 20%
b) 25%
c) 75%
d) 80%
e) 100%

Questo 05) Com base na Segunda Lei da Termodinmica e na mquina de Carnot, assinale a alternativa correta.
a) Uma mquina trmica bem projetada pode chegar a uma eficincia de 100%.
b) O calor flui naturalmente da fonte fria para a fonte quente.
c) possvel construir uma mquina trmica que converta totalmente o calor em trabalho.
d) Quanto maior a diferena entre a temperatura da fonte fria e a temperatura da fonte quente mais eficiente uma
mquina trmica.
e) Um refrigerador um exemplo da mquina de Carnot, quando o ciclo segue primeiro o processo isotrmico, depois o
processo adiabtico e assim por diante at completar o ciclo.


Caderno de Atividades

Disciplina:
Fsica

Professor(a):
Moiss
Aluno:
3 ano
Ensino Mdio
Data de Recebimento:
_____/_____/_____
Lista 04

Data Entrega:
_____/_____/_____

450
Questo 06)Definese o rendimento r de uma mquina trmica como sendor =(w/Q
1
), em que, cada ciclo, Q
1
o calor
absorvido e w o trabalho realizado. Considere uma mquina que segue o ciclo descrito pelo diagrama abaixo.
Sabendo que ela absorve 4 x 10
4
J de calor por ciclo, seu rendimento r de:


a) 15%
b) 50%
c) 25%
d) 75%

Questo 07) Um gs, com um volume inicial V, uma energia interna U e uma presso P, expande-se isobaricamente
at um volume final 2V, alcanando uma energia interna 2U. Esta expanso representada no grfico abaixo.

P
r
e
s
s

o
U 2U
V 2V Volume
P


Aps a anlise do grfico, CORRETO afirmar que o calor absorvido pelo gs, nesta expanso, :
a) 2U + 2PV
b) U PV
c) U + 2PV
d) U 2PV
e) U + PV


Questo 08) Certa quantidade de um gs mantida sob presso constante dentro de um cilindro com o auxlio de um
mbolo pesado, que pode deslizar livremente. O peso do mbolo mais o peso da coluna de ar acima dele de 400N.
Uma quantidade de 28J de calor , ento, transferida lentamente para o gs. Neste processo, o mbolo se eleva de
0,02m e a temperatura do gs aumenta de 20C.

400N
400N
0,02m


Nestas condies, determine:
a) o trabalho realizado pelo gs.
b) o calor especfico do gs no processo, sabendo que sua massa 1,4g.

Questo 09) Um dado sistema gasoso encontra-se, inicialmente, a 40C e a uma presso de 8,4.10
4
N/m.
Fornecendo-se uma quantidade de calor de 4.10
3
cal para esse sistema e mantendo-o presso constante, o seu
volume varia de 0.2 m
3
. Atravs da primeira lei da Termodinmica, determine a variao de temperatura sofrida pelo
gs. Dados: 1 cal = 4.2 J.





451
Questo 10)Um gs perfeito descreve o ciclo ABCDA como indica a figura abaixo. Calcule para o ciclo: o trabalho, o
calor e a variao de energia interna.

1,0 2,0 3,0 4,0 (m)
3
V
(N/m)
2
P
A B
C D
1,0
2,0
3,0


Questo 11)Um gs sofre a transformao termodinmica cclica ABCA representada no grfico p x V. No trecho AB a
transformao isotrmica.

Analise as afirmaes:
00. A presso no ponto A 2,5
.
10
5
N/m
2
.
01. No trecho AB o sistema no troca calor com a vizinhana.
02. No trecho BC o trabalho realizado pelo gs e vale 2,0
.
10
4
J.
03. No trecho CA no h realizao de trabalho.
04. Pelo grfico, o trabalho realizado pelo gs no ciclo ABCA maior do que
4,0
.
10
4
J.


Questo 12) Uma amostra de n mols de um gs ideal monoatmico levada do estado de equilbrio termodinmico
inicial de temperatura T
i
at o estado final de equilbrio de temperatura T
f
mediante dois diferentes processos: no
primeiro, o volume da amostra permanece constante e ela absorve uma quantidade de calor Q
V
; no segundo, a
presso da amostra permanece constante e ela absorve uma quantidade de calor Q
P
. Use a Primeira Lei da
Termodinmica, AU = Q W, sendo AU = (3/2)nRAT, para determinar que se Q
P
for igual a 100 J ento o valor de Q
V

ser igual a:
a) 200 J.
b) 160 J.
c) 100 J.
d) 80 J.
e) 60 J

Questo 13) Calcule a variao de entropia quando, num processo presso constante de 1,0 atm, se transforma
integralmente em vapor 3,0 kg de gua que se encontra inicialmente no estado lquido, temperatura de 100 C.
Dado: calor de vaporizao da gua: Lv = 5,4 x 10
5
cal/kg.

Questo 14) Uma mquina trmica, que opera segundo o ciclo de Carnot (representado no diagrama abaixo), usa um
gs ideal como substncia operante. Sejam T
q
e T
f
as temperaturas dos reservatrios quente e frio, respectivamente.

Assinale a alternativa INCORRETA:
a) o rendimento da mquina dado por
q
f q
T
T T
= c .
b) a energia interna do gs no estado 3 maior que sua energia interna no estado 1.
c) observando o diagrama, pode-se afirmar que
f
4 4
q
2 2
T
V P
T
T P
= .
d) a transformao do estado 2 para o estado 3 adiabtica.
e) esta mquina no pode ter um rendimento igual a 100%.


Questo 15) Um gs sofre a transformaocclica ABCA indicada no grfico abaixo:

Determine :
a) a variao de energia interna;
b) o trabalho realizado pelo gs;
c) a quantidade de calor trocada em cada ciclo.



1
2
3
4
Tq
Tf
V 0
P
B
C
A
600
400
200
0,1 0,2 0,3 0,4 0,5
P(N/m )
V(m )
2
3

452
Questo 16) O calor especfico molar de um gs de 5cal/molK.
Supondo que ele sofra variaes termodinmicas isovolumtricas e que sua temperatura aumente de 20 Cpara 50C,
com um nmero de moles igual a 4, qual ser a variao da energia interna do sistema?
a) 30 cal.
b) 150 cal.
c) 600 cal.
d) 1800 cal.
e) 6000 cal.

Questo 17) A equao matemtica que representa a 1 lei da termodinmica dada por t = A Q E , onde E A a
variao da energia interna do sistema, Q a quantidade de calor trocado, e t o trabalho realizado.
A respeito deste assunto, assinale o que for correto.
01. A 1 lei da termodinmica uma afirmao do princpio da conservao da energia.
02. Em uma transformao cclica, a variao da energia interna do sistema nula.
04. Em uma compresso adiabtica, o sistema recebe trabalho sem fornecer calor.
08. A energia interna de um gs perfeito se conserva durante uma transformao isotrmica.

Questo 18) Um mol de um gs ideal levado do estado B para o estado A, de acordo com o grfico da presso P em
funo do volume V, representado ao lado. Considere T
B
e T
A
as temperaturas do gs nos estados B e A,
respectivamente, e U A a variao energia interna do gs entre os mesmos estados.
correto afirmar que

01. T
B
/T
A
= 1
02. T
B
/T
A
= 4
04. 0 U = A
08. T
A
/T
B
= 4
16. T
A
/T
B
= 1/4

Questo 19)
Uma dada massa de gs perfeito, contida em um cilindro munido de mbolo, levada do estado A para outro B,
como mostra o diagrama da presso em funo do volume.

Sabendo que o gs, nessa transformao, recebeu 300 J de calor, a sua
energia interna sofre um acrscimo, em joules, de
a) 600
b) 450
c) 300
d) 150
e) 100

Questo 20)
Analise as afirmaes abaixo.

I. A variao deentropia do fluido operante num ciclo completo de uma mquina trmica de Carnot igual a Q
1
/T
1
.
II. Otrabalho necessrio para efetivar uma certa mudana de estado num sistema independente do
caminho seguido pelo sistema, quando este evolui do estado inicial para o estado final.
III. Deacordo com a segunda Lei da Termodinmica e de observaes relativas aos processos
reversveis e irreversveis, conclui-se que as entropias inicial e final num processo adiabtico reversvel so
iguais e que, se o processo for adiabtico irreversvel, a entropia final ser maior que a inicial.

Com respeito s trs afirmativas, correto afirmar que:
a) apenas I verdadeira.
b) apenas II verdadeira.
c) apenas III verdadeira.
d) apenas I e II so verdadeiras.
e) apenas II e III so verdadeiras.








453










PROF.: Moiss




C
C
o
o
n
n
t
t
e
e

d
d
o
o
s
s
:
:

- Reflexo da Luz Espelhos Planos;
- Espelhos Esfricos;
- Refrao da Luz;
- Lentes Esfricas.




M
M
i
i
n
n
i
i
s
s
t
t
r
r
a
a
d
d
o
o
s
s
d
d
u
u
r
r
a
a
n
n
t
t
e
e
o
o
m
m

s
s
d
d
e
e
s
s
e
e
t
t
e
e
m
m
b
b
r
r
o
o













D Da at ta a d da a E En nt tr re eg ga a : : _ __ __ __ __ __ __ __ _/ /_ __ __ __ __ __ __ __ __ __ __ _/ /2 20 01 13 3
LISTA 5 FSICA

454

Questo 01)
Dois espelhos planos, sendo um deles mantido na horizontal, formam entre si um ngulo . Uma pessoa observa-se
atravs do espelho inclinado, mantendo seu olhar na direo horizontal. Para que ela veja a imagem de seus olhos, e
os raios retornem pela mesma trajetria que incidiram, aps reflexes nos dois espelhos (com apenas uma reflexo no
espelho horizontal), necessrio que o ngulo seja


a) 15
b) 30
c) 45
d) 60
e) 75

Questo 02)
A figura desenhada fora de escala, representa um espelho plano E que pode girar em torno do ponto P, inicialmente
posicionado perpendicularmente a uma rgua. Um raio de luz r incide no espelho no ponto P gerando o raio refletido r
que atinge a rgua na posio indicada na figura.

r
r
25
80 cm
E
P
r
10
(cm)
60 50 40 30 20 10


Considerando as medidas e o sentido em que o espelho deve girar, indicados na figura, se o espelho girar 10 graus,
em torno do ponto P, o novo raio refletido r atingir a rgua na posio
a) 45,0 cm
b) 55,9 cm
c) 74,6 cm
d) 80,0 cm
e) 82,3 cm

Questo 03)Entre dois espelhos planos, perpendiculares entre si, colocase um
objeto P a 1,5 cm de um deles e a 2,0 cm do outro, como representa a figura.

O nmero de imagens que se obtm e a distncia do objeto imagem mais
afastada, em cm, valem, respectivamente,
a) 2 e 3.
b) 2 e 4.
c) 3 e 3.
d) 3 e 4.
e) 3 e 5.


Caderno de Atividades

Disciplina:
Fsica

Professor(a):
Moiss
Aluno:
3 ano
Ensino Mdio
Data de Recebimento:
_____/_____/_____
Lista 05

Data Entrega:
_____/_____/_____

455
Questo 04)Um raio luminoso emitido por um laser de um ponto F incide em um ponto I de um espelho plano. O ponto
F est a uma distncia b do espelho e a uma distncia a da normal N. Uma mosca voa num plano paralelo ao espelho,
a uma distncia b/2 dele, como ilustra a figura.



Em um certo instante, a mosca atingida pelo raio laser refletido em I.
Calcule, nesse instante, a distncia da mosca normal N.

Questo 05) Os espelhos planos podem ser associados, isto , colocados lado a lado em ngulo ou dispostos
paralelamente entre si. H a possibilidade de essas associaes deslocarem ou multiplicarem o nmero de imagens de
um objeto.
Baseado em seus conhecimentos sobre ptica Geomtrica, em relao s imagens produzidas entre dois espelhos
planos em ngulo, correto afirmar que
a) existe a formao de uma nica imagem, para um ngulo de 180, o que, na prtica, significa um nico espelho.
b) no haver formao de imagens, quando o ngulo for de 0, j que os espelhos ficam dispostos paralelamente.
c) a expresso 1 / 360 n o = no apresenta limitaes, fornecendo o nmero de imagens para qualquer ngulo o
entre 0 e 360.
d) haver a formao de 6 imagens, se os espelhos estiverem dispostos perpendicularmente.
e) podem ser produzidas teoricamente infinitas imagens, desde que os espelhos fiquem dispostos paralelamente, ou
seja, 180 = o .

Questo 06)A imagem I de uma fonte puntiforme P foi obtida atravs de um espelho cncavo colocado direita da
fonte, cujo centro de curvatura se encontra no ponto C.
a) Calcule a distncia da fonte ao vrtice do espelho cncavo.
b) Substituindo-se o espelho cncavo por um convexo, cujo vrtice se encontra no
ponto C, e sabendo-se que a distncia entre P e I no se altera, calcule a distncia
da fonte ao centro de curvatura desse espelho convexo.

Questo 07)Um objeto real, retilneo e de altura y, colocado perpendicularmente
sobre o eixo principal de um espelho esfrico cncavo de raio de curvatura R. Sendo p a distncia entre o vrtice do
espelho e o objeto, a altura y da imagem obtida dada por:
|
|
.
|

\
|

=
p 2 R
R
y y . a
,

|
|
.
|

\
|

=
p R
R 2
y b. y
,

|
|
.
|

\
|

=
p R 2
p
y y . c
,

|
|
.
|

\
|
=
p
p 2 R
y y . d
,


Questo 08)Dispe-se de uma calota esfrica de pequena abertura, espelhada por dentro e por fora, que constitui,
simultaneamente, um espelho cncavo de um lado e um espelho convexo do outro. Quando colocamos um pequeno
objeto em frente face cncava, a 125 cm de seu vrtice, sobre o eixo principal do espelho, tem-se uma imagem
conjugada, invertida e de altura h
1
. Quando o objeto colocado em frente face convexa, tambm a 125 cm do vrtice
do espelho, sua imagem conjugada tem altura h
2
. Desprezando a espessura do espelho e sabendo que
3
7
h
h
2
1
= ,
podemos afirmar que o raio de curvatura do espelho mede:
a) 25 cm
b) 50 cm
c) 75 cm
d) 100 cm
e) 200 cm


456
Questo 09)Sobre um espelho cncavo com 670 mm de dimetro, assinale o que for correto.
01. Sua distncia focal igual a 335 mm.
02. As imagens que ele reflete so invertidas.
04. As imagens que ele reflete so reais.
08. Se o rosto de quem o usa est a 105 mm do espelho, a distncia entre o rosto e sua imagem de 386,4 mm.
16. Ele amplia imagens em 2,68 vezes.

Questo 10) Um espelho esfrico cncavo tem distncia focal 3,0m. Um objeto de dimenses desprezveis se
encontra sobre o eixo principal do espelho, a 6,0m deste. O objeto desliza sobre o eixo principal, aproximando-se do
espelho com velocidade constante de 1,0 m/s.
Aps 2,0 segundos, sua imagem
a) ter se aproximado 6,0m do espelho.
b) ter se afastado 6,0m do espelho.
c) ter se aproximado 3,0m do espelho.
d) ter se afastado 3,0m do espelho.
e) ter se aproximado 12,0m do espelho.

Questo 11) Em trs experimentos distintos, um feixe de luz monocromtica atinge a superfcie de separao entre
dois meios, segundo o mesmo ngulo .

Sabendo que o ndice de refrao da luz desse feixe para o ar tem valor 1, e considerando que a reta tracejada a
normal superfcie de separao dos meios no ponto de incidncia, pode-se concluir que:
a) sen = sen
2
.
b) sen = sen
2
.
c) sen = sen sen.
d) sen = sen sen.
e) sen = sen sen.

Questo 12) O ndice de refrao, n, de um vidro de quartzo depende do comprimento de onda da luz, , conforme
indica o grfico abaixo. Calcule o ngulo de refrao u para uma luz com nm 400 = incidindo sobre uma pea de
quartzo, conforme a figura. Considere o ndice de refrao do ar igual a 1,00.


a) arcsen 0,07
b) arcsen 0,13
c) arcsen 0,34
d) arcsen 0,59
e) arcsen 0,73

Questo 13) A figura representa a trajetria de um feixe de luz monocromtica que incide em uma lmina de vidro, de
ndice de refrao igual a 1,7, com ngulo de incidncia de 60 e sofre um deslocamento lateral de 1,0cm.

Considerando-se a lmina imersa no ar, de ndice de refrao igual a 1,0 e 7 , 1 3 = , pode-se afirmar que a
espessura da lmina igual, em cm, a
01. 1,8
02. 1,7
03. 1,6
04. 1,5
05. 1,4





457
Questo 14) A figura adiante representa a trajetria de um raio de luz que incide perpendicularmente a uma das faces
do prisma e emerge paralelamente outra. (Dados: sen 37 = 0,60; sen 53 = 0,80; sen 90 = 1) O ndice de refrao
absoluto desse prisma vale, aproximadamente:

a) 2,0
b) 1,7
c) 1,25
d) 1,0
e) 0,60

Questo 15)A figura mostra um raio de luz propagando-se num meio de ndice de refrao n
1
e transmitido para uma
esfera transparente de raio R e ndice de refrao n
2
. Considere os valores dos ngulos o ,
1
| e
2
| muito pequenos,
tal que cada ngulo seja respectivamente igual sua tangente e ao seu seno. O valor aproximado de
2
| de

a) ) (
n
n
1
2
1
2
o | = |
b) ) (
n
n
1
2
1
2
o + | = |
c) o
|
|
.
|

\
|
+ | = |
2
1
1
2
1
2
n
n
1
n
n

d)
1
2
1
2
n
n
| = |
e) o
|
|
.
|

\
|
+ | = | 1
n
n
n
n
2
1
1
2
1
2


Questo 16) Como ilustrado na figura, a luz colimada de uma fonte F incide no espelho E, no ar, e refletida para a
face maior do prisma reto P. A luz emerge da face horizontal do prisma, formando com ela um ngulo reto. O espelho
E perpendicular face maior do prisma. Sabendo que a luz incide na direo horizontal e que 30 = o , calcule o
ndice de refrao do prisma.
Dado: n
ar
= 1,0



Questo 17)A figura mostra uma placa de vidro com ndice de refrao 2 n
v
= mergulhada no ar, cujo ndice de
refrao igual a 1,0. Para que um feixe de luz monocromtica se propague pelo interior do vidro atravs de
sucessivas reflexes totais, o seno do ngulo de entrada, e
e
senu dever ser menor ou igual a


a) 0,18
b) 0,37
c) 0,50
d) 0,71
e) 0,87

Questo 18)A figura mostra um raio de luz incidindo sobre uma gotcula com um ngulo de incidncia igual a 45 e
emergindo da mesma aps sofrer um desvio angular o .
Supondo que, para esse raio, o ndice de refrao da gotcula seja
2 n = , o desvio angular o vale:
a) 135;
b) 120;
c) 150;
d) 60;
e) 160.

37
ar
vidro
ar

458
Questo 19) Um estreito feixe de luz, propagando-se no ar, incide na superfcie de um meio transparente de ndice de
refrao n, conforme o esquema abaixo.



Calcule o ndice de refrao n do meio.

Questo 20)
O tringulo retngulo ABC mostrado na figura representa a seo transversal de um prisma. Um raio, proveniente do
ar, incide normalmente sobre o lado AB do tringulo, reflete-se no lado BC e emerge novamente no ar tangencialmente
a AC.


Sabendo que o ndice de refrao do ar n
a
r = 1, calcule o ndice de refrao do material de que feito o prisma.


Questo 21)
Duas lentes esfricas, uma plano-convexa e outra planocncava, so justapostas e inseridas no vcuo (ndice de
refrao igual a 1). Os raios de curvatura de ambas as lentes tm o mesmo valor, entretanto, seus ndices de refrao
diferem.

A vergncia do conjunto, resultado da adio das vergncias individuais de ambas as lentes, em di, pode ser
determinada por
a)
R 2
n n
C
2 1
+
=
b) R
n
n
C
2
1
=
c)
R
n n
C
1 2

=
d)
R
n n
C
2 1
+
=
e)
R
n n
C
2 1

=


459

Questo 22) Analise as afirmaes que seguem, tendo como base a conhecida frmula dos fabricantes de lentes:
|
|
.
|

\
|
+
|
|
.
|

\
|
=
2 face 1 face meio
lente
R
1
R
1
1
n
n
f
1


I. Quando o ndice de refrao do meio ptico no qual a lente est inserida menor do que o ndice de refrao do
material do qual a lente feita, uma lente de bordos espessos ser divergente.
II. No caso de lentes biconvexas de vergncia positiva, quanto maior a diferena entre a espessura dos bordos e a
espessura do centro da lente, mais prximo da lente estaro seus focos principais.
III. Pode-se dizer que a vergncia de uma lente fabricada para ser utilizada inserida no meio ar depende
diretamente do ndice de refrao do material do qual a lente feita.

Est correto o contido em
a) I, apenas.
b) III, apenas.
c) I e II, apenas.
d) II e III, apenas.
e) I, II e III.

Questo 23)Na hipermetropia, o olho focaliza os raios paralelos de um objeto distante, formando a imagem depois da
retina; j na miopia, o olho forma a imagem antes da retina. Nesses casos, para corrigir a formao da imagem sobre a
retina, utilizam-se lentes convergentes e divergentes. correto afirmar:
01. Para corrigir a hipermetropia, as lentes devem ser convergentes.
02. Para a hipermetropia, medida que o objeto se aproxima do olho, a formao da imagem se aproxima da retina.
04. Para a miopia, a maior distncia entre a imagem e a retina quando o objeto est no infinito.
08. Para um objeto no infinito, a imagemformada por uma lente convergente est entre o foco e a lente.
16. O fenmeno fsico, que explica as correes na formao de imagens sobre a retina atravs de lentes, chamado
de refrao.


Questo 24) Situa-se um objeto a uma distncia p diante de uma lente convergente de distncia focal f, de modo a
obter uma imagem real a uma distncia p da lente. Considerando a condio de mnima distncia entre imagem e
objeto, ento correto afirmar que:
a) p
3
+ fpp' + p
3
= 5f
3

b) p
3
+ fpp' + p
3
= 10f
3

c) p
3
+ fpp' + p
3
= 20f
3

d) p
3
+ fpp' + p
3
= 25f
3

e) p
3
+ fpp' + p
3
= 30f
3



Questo 25) O olho humano um sofisticado sistema ptico que pode sofrer pequenas variaes na sua estrutura,
ocasionando os defeitos da viso.
Com base em seus conhecimentos, analise as afirmativas abaixo.
I. No olho mope, a imagem ntida se forma atrs da retina, e esse defeito da viso corrigido usando uma lente
divergente.
II. No olho com hipermetropia, a imagem ntida se forma atrs da retina, e esse defeito da viso corrigido usando
uma lente convergente.
III. No olho com astigmatismo, que consiste na perda da focalizao em determinadas direes, a sua correo feita
com lentes cilndricas.
IV. No olho com presbiopia, ocorre uma dificuldade de acomodao do cristalino, e esse defeito da viso corrigido
mediante o uso de uma lente divergente.

Est(o) correta(s) apenas a(s) afirmativa(s)
a) I e II.
b) III.
c) II e IV.
d) II e III.
e) I e IV.






460











PROF.: Moiss




C
C
o
o
n
n
t
t
e
e

d
d
o
o
s
s
:
:

- Movimento Harmnico Simples (MHS);
- Fenmenos Ondulatrios.






M
M
i
i
n
n
i
i
s
s
t
t
r
r
a
a
d
d
o
o
s
s
d
d
u
u
r
r
a
a
n
n
t
t
e
e
o
o
m
m

s
s
d
d
e
e
o
o
u
u
t
t
u
u
b
b
r
r
o
o













D Da at ta a d da a E En nt tr re eg ga a : : _ __ __ __ __ __ __ __ _/ /_ __ __ __ __ __ __ __ __ __ __ _/ /2 20 01 13 3

LISTA 6 FSICA

461


01 - (UEPG PR) A figura abaixo representa um sistema mola-massa. Inicialmente, a massa encontra-se na posio
A x = e a mola, distendida. O sistema liberado, passa a oscilar entre as posies -A x e A x = = e passa pela posio
de equilbrio 0 x = , executando um movimento harmnico simples. Com base nestas informaes, e desprezando a
fora de atrito entre a massa e a superfcie de apoio, assinale o que for correto a respeito deste evento.


01. Nas posies -A x e A x = = , a energia potencial elstica da mola mnima.
02. Quando 0 x = , a energia cintica mnima e a energia potencial elstica mxima.
04. Nos intervalos 0] [A, e 0] , A [ , a energia cintica aumenta e a energia potencial elstica diminui.
08. Em qualquer posio, a energia mecnica total do sistema ser igual soma da energia cintica com a energia
potencial elstica.


02 - (Mackenzie SP)Dois pndulos simples, A e B, oscilam livremente no mesmo local, sendo que o pndulo A tem
perodo 10% maior que o do perodo do pndulo B. O comprimento do pndulo A
a) 10% maior que o comprimento do pndulo B.
b) 16 % maior que o comprimento do pndulo B.
c) 21% maior que o comprimento do pndulo B.
d) 10 % menor que o comprimento do pndulo B.
e) 21% menor que o comprimento do pndulo B.

03 - (UEM PR)
Duas molas idnticas de constante elstica K so conectadas em paralelo (figura 1) e em srie (figura 2) a um bloco de
massa M. Qual a razo entre o perodo de oscilao das molas conectadas em paralelo e o perodo de oscilao das
molas conectadas em srie?


a) 2
b) 4
c) 1/2
d) 1
e) 3/4


04 - (UEPG PR)O Movimento Harmnico Simples (MHS) um movimento peridico oscilatrio no qual uma partcula
est sujeita a uma fora do tipo F = kx, sempre orientada para a posio de equilbrio. Pela definio apresentada,
assinale o que for correto.
01. O movimento peridico de uma partcula pode, sempre, ser expresso em funo de senos e co-senos.
02. No MHS o perodo e a freqncia independem da amplitude do movimento.
04. No MHS, quando o deslocamento mximo, em qualquer sentido, a velocidade nula, o mdulo da acelerao
mximo,a energia cintica nula e a energia potencial mxima.
08. A energia mecnica total de uma partcula em MHS no constante, porm proporcional ao quadrado da
amplitude.
16. Uma partcula executando um MHS denominada oscilador harmnico simples.




Caderno de Atividades

Disciplina:
Fsica

Professor(a):
Moiss
Aluno:
3 ano
Ensino Mdio
Data de Recebimento:
_____/_____/_____
Lista 06

Data Entrega:
_____/_____/_____

462
05 - (UFPR PR) Um tcnico de laboratrio comprou uma mola com determinada constante elstica. Para confirmar o
valor da constante elstica especificada pelo fabricante, ele fez o seguinte teste: fixou a mola verticalmente no teto por
uma de suas extremidades e, na outra extremidade, suspendeu um bloco com massa igual a 10 kg. Imediatamente
aps suspender o bloco, ele observou que este oscilava com freqncia de 2 Hz. Com base nesses dados, o valor da
constante elstica vale:
a) N/m 160
2
t
b) N/m 16
2
t
c) N/m 6 , 1
2
t
d) N/m ) 16 (
2
t
e) N/m 16 , 0
2
t

06 - (UFPE PE) Um bloco de massa m = 100 g oscila ao longo de uma linha reta na horizontal, em movimento
harmnico simples, ligado a uma mola de constante elstica k = 1,6 x 10
2
N/m. Um grfico da posio x do bloco em
funo do tempo t mostrado na figura abaixo.



Determine a acelerao mxima do bloco, em m/s
2
.
a) 10
b) 20
c) 40
d) 60
e) 80


07 - (UFG/GO/2Fase) No sistema representado na figura abaixo, as duas molas so iguais, tm 1 m de comprimento
e esto relaxadas. Quando o fio cortado, a esfera de massa 5,1 desce 1m at parar momentaneamente.


Dados:
41 , 1 2 =
g = 10 m/s
2

Calcule:
a) o valor da constante elstica k das molas;
b) a energia cintica da massa aps ter descido 75 cm.

08 - (UFG GO) Uma mola de constante elstica k = 50 N/m e massa desprezvel tem uma extremidade fixa no teto e a
outra presa a um corpo de massa m = 0,2 kg. O corpo mantido inicialmente numa posio em que a mola est
relaxada e na vertical. Ao ser abandonado, ele passa a realizar um movimento harmnico simples, em que a amplitude
e a energia cintica mxima so, respectivamente,
Dado: g = 10 m/s
2

a) 4 cm e 0,04 J
b) 4 cm e 0,08 J
c) 8 cm e 0,04 J
d) 8 cm e 0,08 J
e) 8 cm e 0,16 J

463
09 - (Mackenzie SP) Um corpo oscila em torno de um ponto com M.H.S. de amplitude 30 cm. O valor absoluto da
elongao do movimento do corpo, no instante em que a energia cintica igual a
4
3
da energia mecnica, :
a) 25 cm
b) 20 cm
c) 18 cm
d) 15 cm
e) 12 cm

10 - (Mackenzie SP) Um corpo apoiado sobre uma superfcie horizontal lisa e preso a uma mola ideal, comprimida
de 20 cm, abandonado como mostra a figura. Esse corpo realiza um m.h.s. de freqncia5 Hz, sendo O o seu ponto
de equilbrio. A velocidade (v) adquirida pelo corpo, no SI, varia com o tempo (t) obedecendo funo:
a) v = -2 sent (10t .t + t )
b) v = + 2tcos(10t. t + t)
c) v = - tsen (10t .t +t /2)
d) v = + tcos (10t .t + t /2)
e) v = -2tsen(10t. t + 2t/3)


11 - (UFC CE) Uma partcula, de massa m, movendo-se num plano horizontal, sem atrito, presa a um sistema de
molas de quatro maneiras distintas, mostradas abaixo.






Com relao s freqncias de oscilao da partcula, assinale a alternativa correta.
a) As freqncias nos casos II e IV so iguais.
b) As freqncias nos casos III e IV so iguais.
c) A maior freqncia acontece no caso II.
d) A maior freqncia acontece no caso I.
e) A menor freqncia acontece no caso IV.

12 - (UFBA BA) A figura ao lado representa a posio ocupada, no instantet, por uma partcula que descreve um
movimento circular uniforme com velocidade angular e= 4trad/ s , numa
circunferncia de raio R =t cm. A figura representa tambm a posio da projeo
da partcula sobre o eixoOX, paralelo ao dimetro OO, contidos ambos os ei-xos no
plano da circunferncia.
Em relao ao movimento da projeosobre o eixo OX, correto afirmar:

01. O movimento harmnico simples, e sua amplitude igual a2t cm.
02. O perodo do movimento igual a 0,5 s.
04. A funo horria da velocidade escalar instantnea ( ) , t 4 sen 4 v
2
t t = sendo a
fase inicial igual a zero.
08. No ponto de inversox =t cm , a acelerao escalar mxima e igual a 16 t
3
cm
/s
2
.
16. Ao se deslocar dex =t cm at x = 0 , a energia cintica da partcula diminui.

13. Duas fontes sonoras A e B emitem, em fase, um sinal senoidal de mesma amplitude A e com o mesmo
comprimento de onda de 10m. Um observador em O, depois de certo tempo, suficiente para que ambos os sinais,
alcancem P, observar um sinal cuja amplitude vale:


a) 2 A
b) A
c) A/2
d) 0
e) A 2

15m 20m
B P A

464













PROF.: Alexandre Borges





C
C
o
o
n
n
t
t
e
e

d
d
o
o
s
s
:
:

I
I
n
n
t
t
r
r
o
o
d
d
u
u

o
o
a
a
o
o
e
e
s
s
t
t
u
u
d
d
o
o
d
d
a
a
s
s
s
s
o
o
l
l
u
u

e
e
s
s



























D
D
a
a
t
t
a
a
d
d
a
a
E
E
n
n
t
t
r
r
e
e
g
g
a
a
:
:
_
_
_
_
_
_
_
_
_
_
_
_
_
_
_
_
/
/
_
_
_
_
_
_
_
_
_
_
_
_
_
_
_
_
_
_
_
_
_
_
/
/
2
2
0
0
1
1
3
3

LISTA 1 QUMICA

465


01 - (Mackenzie/SP) - O grfico abaixo mostra a curva de solubilidade do cloreto
de amnio, em gramas por 100 g de gua. Se a soluo saturada de cloreto de
amnio, que est temperatura de 70C, for resfriada a 30C, a massa de sal que
precipita ser de:


a.100 g.
b.30 g.
c.40 g.
d.60 g.
e.20 g.

02 - (ITA/SP) -So preparadas duas misturas: uma de gua e sabo e a outra de etanol e sabo. Um feixe de luz
visvel incidindo sobre essas duas misturas visualizado somente atravs da mistura de gua e sabo. Com base
nestas informaes, qual das duas misturas pode ser considerada uma soluo? Por qu?

03 - (ITA/SP) Quando submersos em guas profundas, os mergulhadores necessitam voltar lentamente superfcie
para evitar a formao de bolhas de gs no sangue.
I) Explique o motivo da NO formao de bolhas de gs no sangue quando o mergulhador desloca-se de regies
prximas superfcie para as regies de guas profundas.
II) Explique o motivo da NO formao de bolhas de gs no sangue quando o mergulhador desloca-se muito
lentamente de regies de guas profundas para as regies prximas da superfcie.
III) Explique o motivo da FORMAO de bolhas de gs no sangue quando o mergulhador desloca-se muito
rapidamente de regies de guas profundas para as regies prximas da superfcie.


4- Unicamp
Leia a resposta publicada pela Revista Galileu para uma pergunta bastante comum: faz mal beber gua da chuva?
















Revista Galileu, jul 2009.
Caderno de Atividades

Disciplina:
Qumica

Professor(a):
Alexandre
Aluno:
3 ano
Ensino Mdio
Data de Recebimento:
_____/_____/_____
Lista 01

Data Entrega:
_____/_____/_____

466
Considerando a resposta dada pergunta sobre a gua da chuva, assinale a alternativa CORRETA.
A) O pH da gua da chuva em cidades do interior maior que 7.
B) Beber gua da chuva em Cubato faz bem para a sade.
C) O cido presente na gua da chuva o mesmo cido presente na Coca-Cola.
D) A gua de uma chuva em ambiente isento de poluio levemente cida.
E) A gua da chuva seria potvel se o seu pH fosse acima de 7.

05 - (UFMG/MG) -Este grfico apresenta a variao da solubilidade de KNO
3
em gua, em funoda temperatura:
KNO
3
100
80
60
40
20
0
0 20 40 60 80 100
M
a
s
s
a

d
e

s
o
l
u
t
o
/
1
0
0
g

d
e

g
u
a
Temperatura/C

1.INDIQUE a natureza - endotrmica ou exotrmica - da dissoluo de uma certa quantidade de KNO
3
.
JUSTIFIQUE sua indicao.
2. Durante a dissoluo do KNO
3
, ocorrem estes processos:
I - quebra das interaes soluto/soluto e solvente/solvente; e
II - formao das interaes soluto/solvente.
INDIQUE a natureza - endotrmica ou exotrmica - dos processos I e II.
3. Considerando sua resposta aos itens anteriores desta questo, INDIQUE qualdos processos - I ou II - apresenta o
maior valor de AH em mdulo. JUSTIFIQUE sua indicao.

06 - (ITA/SP) - O frasco mostrado na figura a seguir contm uma soluo aquosa saturada em oxignio, em contato
com ar atmosfrico, sob presso de 1atm e temperatura de 25C.
20
entrada
de gs
sada
de gs
soluo aquosa
saturada em
oxignio
///////////////////////

Quando gs borbulhado atravs desta soluo, sendo a presso de entrada do gs maior do que a presso de sada,
de tal forma que a presso do gs em contato com a soluo possa ser considerada constante e igual a 1atm,
ERRADO afirmar que a concentrao de oxignio dissolvido na soluo :
a.permanece inalterada, quando o gs borbulhado, sob temperatura de 25C, ar atmosfrico.
b.permanece inalterada, quando o gs borbulhado, sob temperatura de 25C nitrognio gasoso.
c.aumenta, quando o gs borbulhado, sob temperatura de 15C, ar atmosfrico.
d.aumenta, quando o gs borbulhado, sob temperatura de 25C, oxignio praticamente puro.
e. permanece inalterada, quando o gs borbulhado, sob temperatura de 25C, uma mistura de argnio e oxignio,
sendo a concentrao de oxignio nesta mistura igual existente no ar atmosfrico.

07 - (ITA/SP) - Considere os sistemas apresentados a seguir:
I. Creme de leite.
II. Maionese comercial.
III. leo de soja.
IV. Gasolina.
V. Poliestireno expandido.
Destes, so classificados como sistemas coloidais
a.apenas I e II.
b.apenas I, II e III.
c.apenas II e V.
d.apenas I, II e V.
e. apenas III e IV.

467
08 - (Feevalle/RS) - O grfico representa as curvas de solubilidade das substncias A, B, C e D.
C
D
B
A
Solubilidade g soluto/100g gua
0 20 40 60 80 100 120 temperatura ( C)
20
40
60
80
100
120

Todas afirmativas a respeito do grfico esto corretas, exceto,
a. a solubilidade da substncia A diminui com a elevao da temperatura.
b. o coeficiente de solubilidade da substncia A a20C 60 g de A em 100 g de gua.
c. entre C e D, a substncia C a mais solvel em qualquer temperatura.
d. A e C apresentam a mesma solubilidade a 40C.
e.para temperaturas inferiores a 40C, a solubilidade de A menor que a de C.

09 - (Fuvest/SP) - Entre as figuras abaixo, a que melhor representa a distribuio das partculas de soluto e de
solvente, numa soluo aquosa diluda de cloreto de sdio, :
Legenda
Na

Cl

H O
2
A B
C
D
E


10 - (UFG/1Etapa) - Ao adoar uma xcara de caf (50 mL), um indivduo utilizou 2 colheres de acar (C
12
H
22
O
11
).
Considere que cada colher contm 5g de acar, cuja solubilidade de 33 g/100g de gua.
Sobre essa xcara de caf, julgue os itens.
1-( ) apresenta corpo de cho constitudo por acar.
2-( ) contm 0,029 mol de acar.
3-( ) poderia dissolver mais uma colher de acar.
4-( ) a massa, em gramas, de uma molcula desse acar igual a 342,176 dividido pelo nmero de Avogadro.

11 - (UEPG/PR/Julho) -Sobre o seguinte grfico de solubilidade do nitrato de potssio (KNO
3
), assinale o que for correto.

01. O ponto B corresponde mxima solubilidade do KNO
3

temperatura de ebulio da gua.
02. No ponto D, a massa de KNO
3
dissolvida igual ao seu
coeficiente de solubilidade.
04. No ponto A, o KNO
3
apresenta seu menor grau de saturao,
constituindo uma soluo diluda.
08. No ponto C, a soluo concentrada.
16. Em qualquer ponto da curva de solubilidade, a soluo
saturada.







468
12 - (ITA/SP) - A figura abaixo mostra a curva de solubilidade do brometo de potssio (KBr) em gua:

Baseado nas informaes apresentadas nesta figura ERRADO
afirmar que:
a-a dissoluo do KBr em gua um processo endotrmico.
b-a 30
o
C, a concentrao de uma soluo aquosa saturada em KBr
aproximadamente 6mol/Kg (molal).
c-misturas correspondentes a pontos situados na regio I da figura so
bifsicas.
d-misturas correspondentes a pontos situados na regio II da figura so
monofsicas.
e-misturas correspondentes a pontos situados sobre a curva so
saturadas em KBr.

13 - (UFPR/PR) - A solubilidade do cloreto de sdio de 36,0 g de
cloreto de sdio por 100 g de gua, a 20 C. Considere a situao descrita e ilustrada a seguir.
A figura abaixo representa uma soluo, A, de cloreto de sdio, a 20 C, preparada adicionando-se certa massa M de
sal a 150 g de gua, agitando-se at que todo o slido seja dissolvido.
Figura I







soluo A so adicionados 5,0 g de cloreto de sdio slido. Com auxlio de um basto de vidro, agita-se bastante
todo o seu contedo. Deixando-se repousar, observa-se uma fase lquida B em equilbrio com um resduo slido de
cloreto de sdio.
Figura II









O resduo slido de cloreto de sdio separado da fase lquida, constituda da soluo B. O slido pesado,
encontrando-se a massa de 1,0 g.
Figura III







Com base nas informaes acima, correto afirmar:
01-B representa uma soluo saturada de cloreto de sdio.
02-A massa M de cloreto de sdio utilizada para preparar a soluo A igual a 50 g.
04-100 g de uma soluo aquosa saturada de cloreto de sdio contm 36 g deste soluto.
08-O resduo slido pode ser separado da soluo B pelo processo descrito a seguir:
- com um papel de filtro seco, de massa m1, filtra-se o conjunto da figura II;
- o resduo slido no papel de filtro lavado com excesso de gua destilada para eliminar a soluo B retida no papel;
- o filtro com o resduo secado e pesado, obtendo-se a massa m
2
;
- a massa do resduo determinada pela diferena (m
2
m
1
).
16-A evaporao da fase lquida da figura III resulta em um resduo slido de 55 g.




A
5,0 g de NaCl(s)
B NaCl(s)
1,0 g de
NaCl(s)
B

469
14 - (UFG/2 Etapa) - Os sistemas, a seguir, contm solues aquosas de NaCl em trs diferentes situaes, mantidas
a temperatura constante:
Soluo
saturada
de NaCl
Soluo
diluda
de NaCl
Corpo
de cho
::
:
: :
:
:
: :
:
:
:
:
: :
:
:
: : :
:
:
: :
:
:
: :
:
:
: :
:
:
: :
: :
:
:
:
Sistemas
I
II III

a. Indique qual(is) sistema(s) est(o) em equilbrio. Justifique sua resposta.
b. O que ocorrer, em cada sistema, se for adicionada uma quantidade muito pequena de NaCl slido?

15 - (UFRJ/RJ) - Os frascos a seguir contm solues saturadas de cloreto de potssio (KCl) em duas temperaturas
diferentes. Na elaborao das solues foram adicionados, em cada frasco, 400 mL de gua e 200 g de KCl.
HO Cl K
Frasco-I Frasco-II
T=?
T=20C
2
+
HO Cl K
2
+
Sal depositado
o

O diagrama a seguir representa a solubilidade do KCl em gua, em gramas de soluto / 100 mL de H
2
O, em diferentes
temperaturas.


a.Determine a temperatura da soluo do frasco I.
b.Sabendo que a temperatura do frasco II de 20 C, calcule a quantidade de sal (KCl) depositado no fundo do frasco.

16 - (UFG/1Etapa) - O grfico a seguir representa a solubilidade de vrios sais em funo da temperatura, expressa
em gramas do soluto por 100 gramas de gua. Sobre esse grfico correto afirmar-se que:
180
160
140
120
- - - - - - -
100
80
-
-
-
-
-
-
-
-
-
-
-
60
40
-
-
20
20 40 60 80 100
g
r
a
m
a
s

d
e

s
a
l
/
1
0
0
g

d
e

H

O
2
Temperatura (C)
CsCl
RbCl
LiCl
NaCl
Li SO
2 4
Cl K

01-a solubilidade dos sais aumenta com a elevao da temperatura na ordem: NaCl, KCl, RbCl, CsCl;
02-com exceo do Li
2
SO
4
, a solubilidade de todos os sais aumenta com elevao da temperatura;
04-a entalpia de solubilizao do KCl positiva;
08-a 0 o NaCl menos solvel que o KCl.





470
17- Enem Em muitas residncias bastante comum o uso de desodorizantes sanitrios. Os desodorizantes so
formulaes que tm na sua composio substncias microbioestticas, capazes de controlar os odores desagradveis
advindos do metabolismo microrgnico. No apresentam efeito letal sobre microrganismos, mas inibem o seu
crescimento e multiplicao. A figura a seguir apresenta o rtulo de um determinado produto adquirido em
supermercados.


Fonte:
Arquivo Pessoal Imagem scaneada.
De acordo com as informaes do rtulo deste desodorizante sanitrio, a massa do princpio ativo e do tensoativo
aninico biodegradvel presente na sua formulao
A) 0,42 g de Preventol e 7,50 g de cido sulfnico
B) 0,042 g de Preventol e 0,75 g de cido sulfnico
C) 7,50 g de Preventol e 0,42 g de cido sulfnico
D) 0,042 g de cido sulfnico e 0,75 g de Preventol
E) 1,2 g de Preventol e 21,43 g de cido sulfnico

18- UNESP Um frasco contendo cerca de 1,5 L de soluo de cido sulfrico, H
2
SO
4
, de concentrao 6,0 mol/L foi
quebrado provocando o seu derramamento pelo laboratrio. Para neutralizar essa soluo decidiu-se despejar
bicarbonato de sdio NaHCO
3
, cuja massa molar corresponde a 84,0 g/mol. O bicarbonato um sal de carter
bsico que reage com o cido sulfrico da seguinte maneira:
2 NaHCO
3(s)
+ H
2
SO
4(aq)
Na
2
SO
4(aq)
+ 2 H
2
O
(l)
+ 2 CO
2(g)

Esse sal deve ser adicionado at que a efervescncia causada pela formao do CO
2(g)
pare.
Assinale a alternativa que apresenta a MASSAaproximadado sal necessria para reagir com todo o cido derramado.
A) 378,0 g
B) 1512 g
C) 151,2 g
D) 756,0 g
E) 75,6 g

19- ufce Numa das etapas de produo do fertilizante sulfato de amnio, o sulfato de clcio reage com o carbonato de
amnio de acordo com a seguinte equao balanceada:
(NH
4
)
2
CO
3 (aq)
+ CaSO
4 (aq)
(NH
4
)
2
SO
4 (aq)
+ CaCO
3 (s
Supondo que a reao de sulfato de amnio [ (NH
4
)
2
SO
4
] acontea com 80% de rendimento e que o carbonato de
amnio [ (NH
4
)
2
CO
3
] tenha sido usado com pureza de 60%. A MASSA, em gramas, de carbonato de clcio [CaCO
3
]
produzida quando se adiciona 46 g de carbonato de amnio [ (NH
4
)
2
CO
3
] :
a) 23g
b) 48g
c) 38 g
d) 29g
e) 31g



471
20- ufgPartir-se de uma soluo concentrada para se obter uma soluo diluda um procedimento de rotina em
laboratrio. Na preparao de uma soluo diluda, com base em uma mais concentrada, retira-se um volume de
soluo concentrada de cido sulfrico (H
2
SO
4
) 4,0 mol/L para se preparar 500 mL de uma soluo diluda de 0,2
mol/L.
Se C
1
V
1
=C
2
V
2
, o volume inicial de soluo de H
2
SO
4
4,0 mol/L retirado para se obter a soluo diluda corresponder a
A) 25 mL
B) 50 mL
C) 100 mL
D) 200 mL
E) 250 mL

21- (UFT/2010 adaptado) Indicador cido-base uma substncia que apresenta uma determinada colorao em
meio cido e outra em meio bsico. Muitas espcies de flores, como as hortncias, contm substncias indicadoras.
Suas ptalas podem adquirir a cor rsea ou azul, dependendo do meio onde elas esto sendo cultivadas.
Durante uma aula de qumica, um acadmico preparou um extrato de folhas de hortncia e utilizou como indicador nos
seguintes experimentos:
Misturou-se uma soluo aquosa de HCl 0,01 mol/L com uma soluo aquosa de NaOH 0,01 mol/L. Aps a mistura,
acrescentou-se gotas do indicador preparado, obtendo-se os seguintes resultados:

Considerando os resultados dos testes,
CORRETO afirmar:
A) Se o indicador for colocado em contato com
suco de limo, apresentar cor amarelo-plido.
B) Se o indicador for colocado em contato com suco de limo apresentar cor rsea.
C) Se misturarmos os reagentes do experimento 1 com os do experimento 3 a cor resultante ser azul.
D) Se misturarmos os reagentes do experimento 1 com os do 3 a cor resultante ser amarelo-plido.
E) Se misturarmos os reagentes do experimento 2 com os do experimento 3 a cor resultante ser azul.

22- UFMG Ao contrrio da maioria das empresas de alimentao, as indstrias de balas, bombons, dropes e gomas de
mascar tm sentido muito pouco a crise econmica que afeta o pas. A produo destas guloseimas aumenta a cada
ano e torna-se importante o controle das substncias que so adicionadas a estes alimentos para a sua
comercializao. o caso dos corantes artificiais, por exemplo, que do a cor necessria para sensibilizar e atrair os
consumidores. Sabe-se, hoje, que os estudos sobre a ao dos corantes na nossa sade so insuficientes e por isso,
faz-se necessrio, o desenvolvimento de novas tcnicas que analisem de forma eficiente e segura, a presena destes
aditivos. Porm, em muitos trabalhos de identificao das substncias, comum que as amostras de anlise sejam
diludas durante o processo.
Suponha que numa destas amostras, de volume 20 mL, constatou-se a presena de 2 mg de um certo corante, comum
em gomas de mascar. Se esta amostra for diluda e acrescentar-se mais 80 mL de gua, a concentrao, em g/L, do
corante nesta soluo obtida ser igual a
a) 0,040
b) 0,010
c) 0,020
d) 0,030

23- ueg Na tabela a seguir informada a percentagem em
massa do soluto de alguns produtos:
Considerando que as solues sejam formadas exclusivamente
pelo soluto predominante e que suas densidades sejam 1,0
g/mL.. Um aluno testou as informaes contidas nos rtulos e
encontrou:
I) 9 g de NaCl em 1000mL de soro fisiolgico.
II) 50 g de C
2
H
4
O
2
em 500 mL de vinagre
III) 1 mol de H
2
O
2
em 1000mL de gua oxigenada.
IV) 10 g de NaClO em 500 mL de gua sanitria
Os produtos que realmente informaram CORRETAMENTE seus clientes foram:

A) gua sanitria e gua oxigenada
B) Vinagre e gua sanitria
C) Vinagre e gua oxigenada
D) Soro fisiolgico e vinagre
E) Soro fisiolgico e gua sanitria

472
24- FUVEST O grfico a seguir representa as curvas de solubilidade de vrias substncias.


Com relao ao grfico anterior, assinale a afirmativa INCORRETA:
A) Com o aumento da temperatura a solubilidade do NaCl praticamente no se altera.
B) A 20C o NaNO
3
mais solvel do que o Na
2
SO
4.

C) Uma soluo de 40g em 200 g de gua de NaCl saturada.
D) A aproximadamente 30C a solubilidade do Na
2
SO
4
diminui com o aumento da temperatura.
E) A aproximadamente 28C a solubilidade do Na
2
SO
4
.10H
2
O igual ao do NaCl.

25- UFRJ Misturam-se duas solues aquosas conforme o esquema a seguir.
Dados:Massas molares: HNO
3
= 63g/mol; Ca(OH)
2
= 74g/mol

Aps a reao, observa-se que a SOLUO FINAL :
A) neutra, pois no h reagente em excesso.
B) bsica, devido a um excesso de 0,3g de Ca(OH)
2
.
C) cida, devido a um excesso de 0,6g de HNO
3
.
D) cida, devido a um excesso de 0,3g de HNO
3
.
E) neutra, devido formao de Ca(NO
3
)
2
.














473




PROF.: Alexandre Borges




C
C
o
o
n
n
t
t
e
e

d
d
o
o
s
s
:
:

D
D
i
i
l
l
u
u
i
i

o
o
e
e
m
m
i
i
s
s
t
t
u
u
r
r
a
a
s
s
d
d
e
e
s
s
o
o
l
l
u
u

e
e
s
s




























D Da at ta a d da a E En nt tr re eg ga a : : _ __ __ __ __ __ __ __ _/ /_ __ __ __ __ __ __ __ __ __ __ _/ /2 20 01 13 3

LISTA 2 QUMICA

474


01 - (UEM/PR/Janeiro/2008) - O tanque de combustvel de uma motocicleta tem uma capacidade mxima de 9 litros.
Considerando que esse veculo utiliza como combustvel gasolina contendo 25% de etanol em volume, quantos mols,
aproximadamente, de etanol possuir um tanque cheio?
(Dados: densidade do etanol = 0,8 g /mL)


02 - (Unicamp/SP/2007) Uma receita de biscoitinhos Petit Four de laranja leva os seguintes ingredientes:
Ingredientes Quantida- Densidade
de/gramas aparente/cm
3
Farinha
de trigo
Carbona-
to de
Amnio
Sal
Manteiga
Acar
Ovos
Raspas de
cascas de
laranjas
360
6
1
90
100
3
0,65
1,5
2,0
0,85
0,90
1,05
0,50
100 (2 ovos)

A densidade aparente da massa recm preparada e antes de ser assada de 1,10 g/cm
3
. Entende-se por densidade
aparente a relao entre a massa da massa ou do ingrediente, na forma em que se encontra, e o respectivo volume
ocupado.
a. Qual o volume ocupado pela massa recm preparada, correspondente a uma receita?
b. Como se justifica o fato da densidade aparente da massa ser diferente da mdia ponderada das densidades
aparentes dos constituintes?

03 - (Acafe/SC/Janeiro/2008) A partir da anlise de uma amostra de vinagre (soluo aquosa de cido actico),
umqumico anotou,no rtulo de uma embalagem, aseguinte informao:4% em v.
Esse dado representa que:
a.h 4 mL de cido actico em 1L de gua.
b.para cada 100 unidades volumtricas de gua, 4 unidades correspondem ao cido actico.
c.h 4mg de cido actico em 1L de gua.
d.para cada 100 unidades volumtricas de vinagre, 4 unidades correspondem ao cido actico.
e. o cido actico concentrado.

04 - (FMTM/MG) - O grfico mostra a variao da solubilidade do oxignio com a temperatura a diferentes presses.

Analisando o grfico, pode-se concluir que as condies de presso inferior atmosfrica, conseguem dissolver maior
quantidade de oxignio, so:
a. p = 580 mmHg, t = 0
o
C
b. p = 580 mmHg, t = 30
o
C
c. p = 760 mmHg, t = 0
o
C
d. p = 254 mmHg, t = 0
o
C
e. p = 254 mmHg, t = 30
o
C

Caderno de Atividades

Disciplina:
Qumica

Professor(a):
Alexandre
Aluno:
3 ano
Ensino Mdio
Data de Recebimento:
_____/_____/_____
Lista 02

Data Entrega:
_____/_____/_____

475
05 - (UFPiau/PI/2009) Em regies mais ridas do Nordeste, os pescadores preferem os horrios mais frios do dia
para pescar. De fato, nesses perodos, a pesca mais farta, porque os peixes vo superfcie em busca de oxignio
(O
2
). A maior concentrao de O
2
na superfcie, nos perodos mais frios, explica-se pelo fato da:
a.reduo na temperatura aumentar a solubilidade de gases em lquidos.
b.reduo na temperatura aumentar a constante de dissociao da gua.
c.elevao no nmero de moles de O
2
ocorrer com a reduo da presso.
d.solubilidade de gases em lquidos independer da presso.
e. elevao na temperatura reduzir a energia de ativao da reao de reduo do oxignio.

06 - (UEM/PR/2008) -Qual a massa, em gramas, de hidrxido de sdio necessria para se preparar 500 mL de uma
soluo aquosa 2,0 mol/L?
(Dados: H = 1; Na = 23; O = 16 )

07 - (UFU/MG/1Fase/2006) O rtulo de um frasco de laboratrio traz a seguinte anotao: HCl = 1,0 mol/L. O volume
desta soluo, que deve ser recolhido, para que nele esteja contido 3,65 gramas de HCl , aproximadamente,
a. 200 mL
b. 50 mL
c.0,01 L
d. 100 mL

08 - (Unesp/SP/Exatas/2007) No descarte de embalagens de produtos qumicos, importante que elas contenham o
mnimo possvel de resduos, evitando ou minimizando conseqncias indesejveis. Sabendo que, depois de
utilizadas, em cada embalagem de 1 litro de NaOH slido restam 4 gramas do produto, considere os seguintes
procedimentos:
embalagem I: uma nica lavagem, com 1 L de gua.
embalagem II: duas lavagens, com 0,5 L de gua em cada vez.
Dados: massas molares: Na = 23 g/mol, O = 16 g/mol e H = 1 g/mol.
a. Qual a concentrao de NaOH, em mol/L, na soluo resultante da lavagem da embalagem I?
b. Considerando que, aps cada lavagem, restam 0,005 L de soluo no frasco, determine a concentrao de NaOH,
em mol/L, na soluo resultante da segunda lavagem da embalagem II e responda: qual dos dois procedimentos de
lavagem foi mais eficiente?

09 - (Oswaldo Cruz/2008) - Quevolume de gua devemos adicionar a 10 mL de soluo 2M para torn-la 0,25M?
a. 80 mL
b. 70 mL
c. 40 mL
d. 250 mL
e.depende do soluto

10 - (UFG/2 Etapa/2005) - Solues de hidrxido de sdio so extremamente usadas em indstrias, residncias e
laboratrios.
Para se preparar 50mL de uma soluo 2mol/L em NaOH.
a-qual a massa em NaOH usada para este prepara?
b-durante o preparo dessa soluo, teremos um aquecimento do recipiente que a contm. Se desejarmos auxiliar esse
processo de dissoluo, deveremos aquecer ou resfriar esse recipiente? Por qu?
c-que volume de gua se adiciona soluo para torn-la 5% em massa? Considere que estas solues tm
densidade igual a 1.

11 - (UFOP/MG/2Fase/2008) - A partir do esquema de diluies
representado a seguir, qual ser a concentrao no frasco D, aps
a execuo das operaesindicadas na seqncia de 1 a 5?
a. 0,075M
b. 0,75M
c. 1,00M
d. 0,10M
e. 7,50M



476
12 - (Faap/SP) - Determine a normalidade de uma soluo aquosa de H
2
SO
4
resultante da mistura de 500mL de uma
soluo aquosa de H
2
SO
4
2M com 1500mL de soluo aquosa do mesmo cido e de concentrao 9,8g/L.
Dados: H=1; O=16; S=32

13 - (UEM/PR/2007) - Qual ser o volume, em mililitros (mL), de uma soluo aquosa de hidrxido de sdio 0,10 mol/L
necessrio para neutralizar 25 mL de uma soluo aquosa de cido clordrico 0,30 mol/L?
(Dados: Na = 23; O = 16; H =1; Cl = 35,5)

14 - (UEPG/PR/Janeiro/2008) - So submetidos a titulao 10 mL de uma soluo de H
2
SO
4
com uma soluo de
NaOH0,5 M. Determine a concentrao da soluo de H
2
SO
4
em g/L, sabendo que foram consumidos 20 mL da
soluo de NaOH.(Dados de massas atmicas: H = 1; S = 32; O = 16.

15 - (UFG/1Etapa/2003) O municpio de Catalo rico em rocha fosftica que, ao ser triturada e tratada com cido
sulfrico, produz uma mistura de sulfatos e fosfatos chamada superfosfato, que um fertilizante de grande importncia
comercial. Essa reao representada pela equao:
Ca
3
(PO
4
)
2
(s) + 2H
2
SO
4
(l) 2CaSO
4
(s) + Ca(H
2
PO
4
)
2
(s)
Sobre essa reao, correto afirmar que:
01. 100 mL de H
2
SO
4
2 mol/L reagem completamente com 31 g de Ca
3
(PO
4
)
2
.
02.o fsforo reduzido a P
5+
.
03.980,9 g de rocha fosftica 30% pura produzem 222 g de superfosfato.
04.os produtos esto em soluo aquosa.

16 - (EFEI/SP/2007) Uma titulao cido-base pode ser utilizada para determinar a acidez ou mesmo a alcalinidade
de vrias amostras. Um exemplo de titulao cido-base est mostrado a seguir:

Qual a alternativa que melhor representa um grfico de pH (ordenada) em funo do volume de NaOH adicionado
(abcissa)?
1
A
1
B

1
C
1
D

1
E




477
17 - (UEM/PR/Janeiro/2005) - Considere as figuras a seguir, a 250C, e assinale a(s) alternativa(s) correta(s).

01. A25C, os trs lquidos A, B e C possuem presso de vapor diferentes, portanto possuem volatilidades diferentes.
02. Se forem adicionados 1g de NaCl em C e 1g de sacarose em B, a diferena entre h
B
e h
C
ir aumentar.
04. O lquido B o mais voltil dentre os trs lquidos.
08. O valor da presso de vapor de cada um dos lquidos pode ser obtida atravs do respectivo valor de h, da
densidade do lquido dentro do manmetro e do valor da acelerao da gravidade.
16. O lquido B deve ser menos polar que os lquidos A e C.
32. Caso se duplique o volume do recipiente B, na temperatura de 25C, a presso de vapor do lquido remanescente
cair metade.
64. Se a densidade do lquido dentro dos manmetros diminuir, os valores de h
A
, h
B
e h
C
tambm diminuiro.

18 - (UFG/2 Etapa/2003) A Chapada dos Veadeiros tambm famosa por suas inmeras cachoeiras e enormes
paredes de cristais de quartzo, que so silicatos. No diagrama de fases, a seguir, esto representadas trs fases
slidas de um silicato de alumnio com frmula molecular Al
2
SiO
5
, onde cada fase slida representa um tipo de mineral
diferente: cianita (densidade = 3,63 g.cm
3
), andalusita (densidade = 3,16 g.cm
3
) e silimanita (densidade = 3,24 g.cm

3
).

Nesse diagrama, a linha vertical reflete a mudana das fases, com a presso,
na temperatura fixa de 700 K.
Explique quais as fases predominantes na transformao isotrmica do
ponto A ao ponto C, identificando cada uma delas.

19 - (ITA/SP/2004) - Considere as seguintes afirmaes relativas aos
sistemas descritos abaixo, sob presso de 1atm:
I. A presso de vapor de uma soluo aquosa de glicose 0,1mol/L
menor
II. do que a presso de vapor de uma soluo de cloreto de sdio 0,1mol/L a
25C.
II. A presso de vapor do n-pentano maior do que a presso de vapor do n-hexano a 25C.
III. A presso de vapor de substncias puras como: acetona, ter etlico, etanol e gua, todas em ebulio, tem o
mesmo valor.
IV. Quanto maior for a temperatura, maior ser a presso de vapor de uma substncia.
V. Quanto maior for o volume de um lquido, maior ser a sua presso de vapor.
Destas afirmaes, esto CORRETAS
a.apenas I, II, III e IV.
b.apenas I, II e V.
c.apenas I, IV e V.
d.apenas II, III e IV.
e. apenas III, IV e V.

20 - (UFU/MG/2008) - As substncias que ocorrem na natureza encontram-se normalmente misturadas com outras
substncias formando misturas homogneas ou heterogneas. As misturas homogneas, ao contrrio das
heterogneas, podem ser confundidas, na aparncia, com substncias puras. Uma forma de diferenciar as misturas
homogneas de substncias puras determinar as propriedades fsicas do sistema em questo como ponto de fuso
(PF), ponto de ebulio (PE), densidade e condutividade eltrica. Considerando esse fato, as seguintes afirmativas em
relao gua do mar esto corretas, EXCETO
a. a densidade da gua do mar maior que a densidade da gua pura.
b. a gua do mar tem presso de vapor superior da gua pura.
c. a gua do mar contm compostos inicos e moleculares dissolvidos.
d. a gua do mar congela numa temperatura inferior da gua pura.


10
2
0 700
P
r
e
s

o
/
1
0
0
0
x
a
t
m
I
II
III
Temperatura/K
A
B
C

478
21 - (UFPE/PE/2007) - O grfico abaixo representa a presso de vapor (eixo das ordenadas), em atm, em funo da
temperatura (eixo das abcissas), em C, de trs amostras, I, II e III. Se uma destas amostras for de gua pura e as
outras duas de gua salgada, podemos afirmar que:

a. a amostra I a amostra de gua salgada.
b. a amostra I a mais voltil.
c. a amostra II mais concentrada que a amostra III.
d. a amostra I a menos voltil.
e.na temperatura T
III
e 1 atm a amostra II ainda no entrou em ebulio.



22 - (UFRJ/RJ/2007) - As hemcias apresentam mesmo volume quando esto no sangue ou em soluo aquosa de
NaCl 9g/L (soluo isotnica). No entanto, quando as hemcias so colocadas em soluo aquosa de NaCl mais
diluda (soluo hipotnica) elas incham podendo at arrebentar. Esse processo chama-se hemlise.
O grfico a seguir apresenta curvas da presso de vapor (P
v
), em funo da temperatura (T) para solues aquosas de
diferentes concentraes de NaCl.
Pv
A
B
T
Soluo
Isotnica

a.Qual das curvas representa a soluo de NaCl que pode ser usada para o processo de hemlise? Justifique sua
resposta, utilizando a propriedade coligativa adequada.
b.Com o objetivo de concentrar 2 litros da soluo isotnica, evapora-se cuidadosamente 10% de seu volume.
Determine a concentrao, em g/L, da soluo resultante.

23-(Enem)- A panela de presso permite que os alimentos sejam cozidos em gua muito mais rapidamente do que em
panelas convencionais. Sua tampa possui uma borracha de vedao que no deixa o vapor escapar, a no ser atravs
de um orifcio central sobre o qual assenta um peso que controla a presso. Quando em uso, desenvolve-se uma
presso elevada no seu interior. Para a sua operao segura, necessrio observar a limpeza do orifcio central e a
existncia de uma vlvula de segurana, normalmente situada na tampa. O esquema da panela de presso e um
diagrama de fase da gua so apresentados abaixo.





479

24-(Enem-2002)- A vantagem do uso de panela de presso a rapidez para o cozimento de alimentos e isto se deve
(A) presso no seu interior, que igual presso externa.
(B) temperatura de seu interior, que est acima da temperatura de ebulio da gua no local.
(C) quantidade de calor adicional que transferida panela.
(D) quantidade de vapor que est sendo liberada pela vlvula.
(E) espessura da sua parede, que maior que a das panelas comuns.


25-(Enem)- Se, por economia, abaixarmos o fogo sob uma panela de presso logo que se inicia a sada de vapor pela
vlvula, de forma simplesmente a manter a fervura, o tempo de cozimento
(A) ser maior porque a panela esfria.
(B) ser menor, pois diminui a perda de gua.
(C) ser maior, pois a presso diminui.
(D) ser maior, pois a evaporao diminui.
(E) no ser alterado, pois a temperatura no varia.




















480






PROF.: Alexandre Borges




C
C
o
o
n
n
t
t
e
e

d
d
o
o
s
s
:
:

T
T
e
e
r
r
m
m
o
o
q
q
u
u

m
m
i
i
c
c
a
a
e
e
c
c
i
i
n
n

t
t
i
i
c
c
a
a
q
q
u
u

m
m
i
i
c
c
a
a



























D Da at ta a d da a E En nt tr re eg ga a : : _ __ __ __ __ __ __ __ _/ /_ __ __ __ __ __ __ __ __ __ __ _/ /2 20 01 13 3

LISTA 3 QUIMICA

481


01 - (UEL/PR/2006) A pirolusita um dos mais importantes minrios que contm o dixido de mangans (MnO
2
). Na
indstria metalrgica, o mangans puro pode ser obtido por processo trmico a partir da pirolusita, atravs da reao: 3
MnO
2
(s) + 4 Al (s) 2 Al
2
O
3
(s) + 3 Mn (s)
Entalpias de formao a 25C e 1atm em kJ/mol: MnO
2
(s) = - 521,0; Al
2
O
3
(s)= - 1676,0
Massa molar (g/mol): Mn = 55,0
Com base nessas informaes, correto afirmar que na produo de 11,0 g de mangans puro, a partir das entalpias
de formao das substncias, ocorre:
a. Absoro de 358 kJ de energia.
b. Liberao de 358 kJ de energia.
c. Absoro de 119 kJ de energia.
d. Liberao de 119 kJ de energia.
e. Liberao de 146 kJ de energia.

02 - (UFPiau/PI/2005) Dentre os diversos efeitos nocivos da poluio ambiental, destaca-se o efeito estufa, uma
vez que as conseqncias deste so sentidas em perodos de tempo inferiores a uma gerao (< 50 anos). O aumento
na temperatura mdia global, como resultado do efeito estufa, tem influncia direta, por exemplo, sobre o equilbrio de
dissociao da gua, 2H
2
O
(l)

H
3
O
+
(aq)
+ OH

(aq)
, por causar reduo no pH da gua neutra. Pode-se dizer, portanto,
que a reao de dissociao da gua :
a. exotrmica.
b. nuclear.
c. trimolecular.
d.irreversvel.
e. endotrmica.


03 - (UEM/PR/2008) - O metanol um lquido combustvel que pode ser considerado como um substituto da gasolina.
Ele pode ser sintetizado a partir do gs natural metano, de acordo com a reao abaixo: 2CH
4
(g) + O
2
(g)2CH
3
OH(l).
Considerando as equaes a seguir e as afirmaes acima, assinale o que for correto.
(Dados: H = 1; C = 12; O = 16.
CH
4
(g) + H
2
O(g)CO(g) + 3 H
2
(g) AH
O
= 206,1kJ
2 H
2
(g) + CO(g) CH
3
OH(l) AH
O
= -128,3kJ
2 H
2
(g) + O
2
(g) 2 H
2
O(g) AH
O
= - 483,6kJ
01. Entalpia de combusto de uma substncia o calor liberado na reao de combusto completa de 1 mol dessa
substncia, a 25C e 1 atm.
02. Uma reao exotrmica possui variao de entalpia padro negativa.
04. Fuso e vaporizao so exemplos de processos endotrmicos, enquanto solidificao e liquefao so exemplos
de processos exotrmicos.
08. O calor de formao de 2 mols de metanol a partir do metano e do oxignio a 25C e 1 atm igual a - 328 kJ.
16. Considerando a reao de formao do metanol a partir de metano e de oxignio a 25C e 1atm, ao aquecer o
sistema, favorece-se a produo de metanol, pois essa reao exotrmica.

04 - (Vunesp/SP/2005) Em uma cozinha, esto ocorrendo os seguintes processos:
I. gs queimando em uma das bocas do fogo e
II. gua fervendo em uma panela que se encontra sobre esta boca do fogo.
Com relao a esses processos, pode-se afirmar que:
a. I e II so exotrmicos.
b. I exotrmico e II endotrmico.
c. I endotrmico e II exotrmico.
d. I isotrmico e II exotrmico.
e. I endotrmico e II isotrmico.



Caderno de Atividades

Disciplina:
Qumica

Professor(a):
Alexandre
Aluno:
3 ano
Ensino Mdio
Data de Recebimento:
_____/_____/_____
Lista 03

Data Entrega:
_____/_____/_____

482
05 - (UFSCar/1 Fase/2006) Considere as equaes:
Ca
2+
(g) + 2Cl

(g) CaCl
2
(s) AH = 2 260 kJ/mol
Ca
2+
(g) Ca
2+
(aq) AH = 1 657 kJ/mol
Cl

(g) Cl

(aq) AH = 340 kJ/mol


A entalpia de dissoluo, em kJ/mol, do cloreto de clcio em gua, :
a. + 714.
b. + 263.
c. + 77.
d. 77.
e. 263.

06 - (FEPECS/DF/2006) No dia 20 de dezembro de 2002 o jornal O Globo publicou uma informao cientfica sobre
a transformao de restos mortais em diamantes.
Diamantes sintticos surgiram em meados dos anos 50 quando a GE desenvolveu um processo para criar pedras
desse tipo para uso industrial, a partir do grafite. Quem percebeu a possibilidade de cinzas de restos humanos se
transformarem em diamante foi RustyVanderBiesen, hoje presidente da firma. Como o corpo humano feito de
carbono, matria essencial dos diamantes, ele imaginou que deveria haver uma maneira de produzir pedras preciosas
a partir de restos humanos.
Grafite e diamante so variedades alotrpicas do elemento carbono que se diferenciam entre si pelo arranjo cristalino.
Enquanto o grafite apresenta brilho metlico e pode ser quebrado com pouco esforo, o diamante o material mais
duro da natureza. Uma outra evidncia da diferena entre eles a quantidade de calor trocado na combusto,
conforme as seguintes equaes termoqumicas:
C
(grafite)
+ O
2(g)
CO
2(g)
AH
(25C, 1 atm)
= 393,7 kJ
C
(diamante)
+ O
(2(g)
CO
2(g)
AH
(25C, 1 atm)
= 395,6 kJ
Com essas informaes, est correto afirmar que aenergia, em kJ, necessria para transformar 48g de grafite em
diamante igual a:
a. 1,9
b. 3,8
c. 5,7
d. 7,6
e. 9,5

07 - (UFMS/Exatas/2008) Calcule a entalpia, AH, em kcal/mol, da reao:CO
2
(g) C(grafite slido) + O
2
(g) ,nas
condies ambientes ( 25C e 1 atm ), sabendo-se que:
I. C
2
H
6
(g) + 7/2 O
2
(g) 2 CO
2
(g) + 3 H
2
O(l)AH = - 372,7 kcal/mol
II. 2 C(grafite slido) + 3 H
2
(g) C
2
H
6
(g) AH = - 20,2 kcal/mol
III. H
2
(g) + 1/2O
2
(g) H
2
O(l) AH = - 68,3 kcal/mol

08 - (UEPB/PB/2008) muito importante conhecer a quantidade de calor envolvida numa reao qumica. Dentre
outros aspectos, esta informao til quando se trata de reaes que se processam em escala industrial, pois serve
para prever se o ambiente tem capacidade de resfriamento suficiente, para impedir que reaes exotrmicas
provoquem superaquecimento e danos nas instalaes. Por exemplo, sabe-se que a reao de hidrogenao do etino
(C
2
H
2
) a etano (C
2
H
6
) se processa de acordo com a equao:
C
2
H
2
(g) + 2H
2
(g) C
2
H
6
(g)
Neste contexto,
a.determine a entalpia desta reao, AH
r
, a partir dos seguintes valores de entalpia de combusto:
AH (C
2
H
2
(g)) = 1300 kJ mol
1

AH (H
2
(g)) = 286 kJ mol
1

AH (C
2
H
6
(g)) = 1560 kJ mol
1

b.indique se esta reao ocorre com absoro ou liberao de calor.

09 - (UFSCar/SP/2 Fase/2007) O cultivo da cana-de-acar faz parte da nossa histria, desde o Brasil Colnia. O
acar e o lcool so seus principais produtos. Com a crise mundial do petrleo, o incentivo fabricao de carros a
lcool surgiu, na dcada de 1970, com o Prolcool. Esse Programa Nacional acabou sendo extinto no final da dcada
de 1990. Um dos pontos altos nas discusses em Joanesburgo sobre desenvolvimento sustentvel foi o pacto entre
Brasil e Alemanha para investimento na produo de carros a lcool.
a. Escreva a equao de combusto do etanol, devidamente balanceada. Calcule o calor de combusto de 1 mol de
etanol, a partir das seguintes equaes:
AH
0
f (kJ/mol)
C (s) + O
2
(g) CO
2
(g) 394
H
2
(g) + 1/2 O
2
(g) H
2
O (l) 286
2 C (g) + 3 H
2
(g) +1/2 O
2
(g) C
2
H
5
OH (l) 278
b. A reao de combusto do etanol endotrmica ou exotmica? Justifique.

483
10 - (Unesp/SP/Biolgicas/2007) O perxido de hidrognio, H
2
O
2
, um lquido incolor cujas solues so alvejantes e
anti-spticas. Esta gua oxigenada preparada num processo cuja equao global : H
2
(g) + O
2
(g) H
2
O
2
(l)
Dadas as equaes das semi-reaes:
H
2
O
2
(l) H
2
O(l) + 1/2O
2
(g) AH = 98,0 kJ/mol
2H
2
(g) + O
2
(g) 2H
2
O(l) AH = 572,0 kJ/mol
pergunta-se:
a.Qual o AH da reao do processo global?
b.Esta reao exotrmica ou endotrmica? Justifique sua resposta.


11 - (UFG/2 Etapa/2004) - Determine a entalpia de formao de cido clordrico gasoso, segundo a reao
representada pela equao:
H
2 (g)
+ Cl
2 (g)
2HCl
(g)

Dados: H
2 (g)
2H
(g)

o
= 436 kJ/mol
Cl
2 (g)
2Cl
(g)

o
= 243 kJ/mol

HCl
(g)
H
(g)
+ Cl
(g)

o
= 431 kJ/mol
Indique os clculos.


12 - (FMTM/MG/2006) -O grfico refere-se ao diagrama energtico da reao de decomposio do cido frmico, onde
se vem destacados dois caminhos de reao:
Energia
Coordenada da reao
I
II
H O
+ CO
HCOOH
2

Analisando o grfico, pode-se afirmar que
a. a curva II representa a reao no catalisada.
b. a reao de decomposio do cido frmico exotrmica.
c. a rapidez da reao catalisada, curva I, maior porque apresenta maior energia de ativao.
d. a rapidez da reao catalisada, curva II, maior e ocorre em vrias etapas.
e.a presena de catalisador diminui o AH da reao decomposio do cido frmico.


13 - (UFMT/MT/2007) - Pesquisas demonstram que algumas regies do estado de Mato Grosso esto entre as mais
poludas do pas. Segundo dados da Fundao Estadual do Meio Ambiente (FEMA), a indstria mato-grossense no
contribui para esse quadro, o problema maior so as queimadas. O homem apontado como o maior responsvel, j
que ele quem ateia fogo nos pastos, nas reas agricultveis, nos terrenos baldios e at no quintal da casa. (Adaptado
de SATO, Michele (org.). Cincias: Introduo s cincias naturais/NEAD. Cuiab: EdUFMT, 1999.)
Sobre as queimadas, assinale a afirmativa INCORRETA.

a.So produzidas pela oxidao de material orgnico.

b.O grfico abaixo representa o diagrama energtico das reaes de combusto ocorridas durante as queimadas.

c.Alastram-se rapidamente em pastagens quando est ventando em decorrncia da grande superfcie
de contato e do aumento da concentrao do gs oxignio no local.
d.A combusto completa da glicose, monmero de alguns polmeros consumidos,
ocorre de acordo com a seguinte reao: C
6
H
12
O
6
+ 6O
2
6CO
2
+ 6H
2
O + energia.
e. Alm do dixido de carbono e outros gases, tambm so produzidos monxido de
carbono e dixido de enxofre.





Entalpia
Percurso da reao

484
14 - (UFMG/MG/1Fase/2008) - Na cozinha de uma casa, foram feitos quatro experimentos para descobrir-se em que
condies uma esponja de l de ao se oxidava mais rapidamente. Nesta tabela, esto descritas as condies em que
os experimentos foram realizados e quais deles resultaram em oxidao do ferro metlico:
Experimento Condies Oxidao da
esponja
I esponja seca, em contato no
com o ar seco
II esponja mida, em contato sim
com o ar seco
III esponja parcialmente sim
mergulhada em gua
IV esponja completamente no
mergulhada em gu fervida

A primeira etapa da oxidao do ferro metlico a converso de Fe (s) em Fe (II). Considerando-se os experimentos
descritos e seus resultados, CORRETO afirmar que a equao que, mais provavelmente, representa essa primeira
etapa :
a.Fe (s) + 1/2 O
2
(g) FeO (s)
b.Fe (s) + H
2
O (l) FeO (s) + H
2
(g)
c.Fe (s) + 1/2 O
2
(g) + H
2
O (l) Fe(OH)
2
(s)
d.Fe (s) + 2 H
2
O (l) Fe(OH)
2
(s) + H
2
(g)

15 - (Unesp/SP/2008) - A queima de um combustvel como a gasolina, ou seja, sua reao com o oxignio, bastante
exotrmica e, do ponto de vista termodinmico, espontnea. Entretanto, essa reao inicia-se somente com a
concorrncia de um estmulo externo, como, por exemplo, uma fasca eltrica. Dizemos que o papel deste estmulo
a. fornecer a energia de ativao necessria para a reao ocorrer.
b. deslocar o equilbrio no sentido de formao de produtos.
c. aumentar a velocidade da reao direta e diminuir a velocidade da reao inversa.
d. favorecer a reao no sentido da formao de reagentes.
e.remover o nitrognio do ar, liberando o oxignio para reagir.

16 - (UFG/1Etapa/2005) - Os seguintes grficos representam variveis de uma reao qumica.
0
t
2
1
t
1
2
C
o
n
c
e
n
t
r
a

o
Tempo
0
t
2
3
t
1
4
V
e
l
o
c
i
d
a
d
e
Tempo

Os grficos indicam que
a. no instante t
1
, a velocidade da reao direta igual a da inversa.
b. aps t
2
, no ocorre reao.
c. no instante t
1
, a reao atingiu o equilbrio.
d. a curva 4 corresponde velocidade da reao inversa.
e.no ponto de interseco das curvas 3 e 4, a concentrao de produtos igual de reagentes.

17 - (UFG/1Etapa/2004) - Em aqurios, utilizam-seborbulhadores de ar para oxigenar a gua. Para um mesmo volume
de ar bombeado nesse processo, bolhas pequenas so mais eficientes, porque em bolhas pequenas
a. a rea superficial total maior.
b. a densidade menor.
c. a presso maior.
d. a velocidade de ascenso menor.
e.o volume total menor.

18 - (UFMG/MG/1Fase/2008) - Considerando-se o papel do catalisador numa reao reversvel, CORRETO afirmar
que:
a. a velocidade da reao independente da concentrao do catalisador.
b. o catalisador acelera apenas a reao direta.
c. o catalisador desloca o equilbrio no sentido de formar mais produtos, mesma temperatura.
d. o catalisador consumido e regenerado durante a reao.
e. o catalisador desloca o equilbrio tanto para o formao de produtos quanto de reagentes

485
19 - (Unesp/SP/2007) - Comparando duas panelas, simultaneamente sobre dois queimadores iguais de um mesmo
fogo, observa-se que a presso dos gases sobre a gua fervente na panela de presso fechada maior que aquela
sobre a gua fervente numa panela aberta. Nessa situao, e se elas contm exatamente as mesmas quantidades de
todos os ingredientes, podemos afirmar que, comparando com o que ocorre na panela aberta, o tempo de cozimento
na panela de presso fechada ser
a. menor, pois a temperatura de ebulio ser menor.
b. menor, pois a temperatura de ebulio ser maior.
c. menor, pois a temperatura de ebulio no varia com a presso.
d. igual, pois a temperatura de ebulio independe da presso.
e.maior, pois a presso ser maior.

20 - (Mackenzie/SP/2008) - Considerando o diagrama da reao genrica A + B C + D, fazem-se as afirmaes:

I) a reao exotrmica.
II) o AH da reao direta igual a 15 kcal/mol.
III) a energia de ativao da reao direta igual a + 25 kcal/mol.
IV) a reao direta ocorre com absoro de calor.
So corretas, somente:
a. I, II e III.
b. I e III.
c. III e IV.
d. II e IV.
e. I e II.

21 - (ITA/SP/2005) - A figura ao lado representa o resultado de dois experimentos diferentes (I) e (II) realizados para
uma mesma reao qumica genrica (reagentes produtos). As reas hachuradas sob as curvas representam o
nmero de partculas reagentes com energia cintica igual ou maior que a energia de ativao da reao (E
at
).
Baseado nas informaes apresentadas nesta figura, CORRETO afirmar que
Energia cintica
das partculas
N

m
e
r
o


d
e


p
a
r
t

c
u
l
a
s
I
II
E
at

a. a constante de equilbrio da reao nas condies do experimento I igual da reao nas condies do
experimento II.
b. a velocidade medida para a reao nas condies do experimento I maior que a medida nas condies do
experimento II.
c. a temperatura do experimento I menor que a temperatura do experimento II.

d. a constante de velocidade medida nas condies do experimento I igual medida nas condies do experimento
II.
e.a energia cintica mdia das partculas, medida nas condies do experimento I, maior que a medida nas
condies do experimento II.

(Enem-2007) - Texto para as questes 22 e 23
lcool, crescimento e pobreza
O lavrador de Ribeiro Preto recebe em mdia R$ 2,50 por tonelada de cana cortada. Nos nos 80, esse trabalhador
cortava cinco toneladas de cana por dia.

486
A mecanizao da colheita o obrigou a ser mais produtivo. O corta-cana derruba agora oito toneladas por dia. O
trabalhador deve cortar a cana rente ao cho, encurvado. Usa roupas mal-ajambradas, quentes, que lhe cobrem o
corpo, para que no seja lanhado pelas folhas da
planta. O excesso de trabalho causa a birola: tontura, desmaio, cibra, convulso. A fim de agentar dores e cansao,
esse trabalhador toma drogas e solues de glicose, quando no farinha mesmo. Tem aumentado o nmero de mortes
por exausto nos canaviais. O setor da cana produz hoje uns 3,5% do PIB. Exporta US$ 8 bilhes. Gera toda a energia
eltrica que
consome e ainda vende excedentes. A indstria de So Paulo contrata cientistas engenheiros para desenvolver
mquinas e equipamentos mais eficientespara ausina de lcool. As esquisas, privada e pblica, na rea agrcola (cana,
laranja, eucalipto etc.) desenvolvem a bioqumica e a gentica no pas. Folha de S. Paulo, 11/3/2007 (com
adaptaes).


22-Enem2007-Ao beber uma soluo de glicose (C6H12O6), um corta-cana ingere uma substncia:
a- que ao ser degradada pelo organismo, produz energia que pode ser usada para movimentar o corpo.
b- inflamvel que, queimada pelo organismo, produz gua para manter a hidratao da clula.
c-que eleva a taxa de acar no sangue e armazenada na clula, o que restabelece o teor de oxignio no organismo.
d-insolvel em gua, o que aumenta a reteno de lquidos pelo organismo.
e-de sabor adocicado que, utilizada na respirao celular, fornece CO2 para manter estvel a taxa de carbono na
atmosfera.

23- Enem2007- H diversas maneiras de o ser humano obter energia para seu prprio metabolismo utilizando energia
armazenada na cana-de-acar. O esquema abaixo apresenta quatro alternativas dessa utilizao.
acar
A partir dessas informaes, conclui-se que:
a-a alternativa 1 a que envolve maior diversidade de
atividades econmicas.
b-a alternativa 2 a que provoca maior emisso de gs
carbnico para a atmosfera.
c-as alternativas 3 e 4 so as que requerem menor
conhecimento tecnolgico.
d-todas as alternativas requerem trabalho humano para a
obteno de energia.
e-todas as alternativas ilustram o consumo direto.

24- Enem-2007-Quanto mais desenvolvida uma nao,
mais lixo cada um de seus habitantes produz. Alm de o progresso elevar o volume de lixo, ele tambm modifica a
qualidade
do material despejado. Quando a sociedade progride, ela troca a televiso, o computador, compra mais brinquedos e
aparelhos eletrnicos. Calcula-se que 700 milhes de aparelhos celulares j foram jogados fora em todo o mundo. O
novo lixo contm mais mercrio, chumbo,
alumnio e brio. Abandonado nos lixes, esse material se deteriora e vaza. As substncias liberadas infiltram-se no
solo e podem chegar aos lenis freticos ou a rios prximos, espalhando-se pela gua.
Anurio Gesto Ambiental 2007, p. 47-8 (com adaptaes).
A respeito da produo de lixo e de sua relao com o ambiente, correto afirmar que:
a)as substncias qumicas encontradas no lixo levam, freqentemente, ao aumento da diversidade de espcies e,
portanto, ao aumento da produtividade agrcola do solo.

487
b)o tipo e a quantidade de lixo produzido pela sociedade independem de polticas de educao que proponham
mudanas no padro de consumo.
c)a produo de lixo inversamente proporcional ao nvel de desenvolvimento econmico das sociedades.
d)o desenvolvimento sustentvel requer controle e monitoramento dos efeitos do lixo sobre espcies existentes em
cursos dgua, solo e vegetao.
e) desenvolvimento tecnolgico tem elevado a criao de produtos descartveis, o que evita a gerao de lixo e
resduos qumicos.

25-Enem2007) Um poeta habitante da cidade de Poos de Caldas MG assim externou o que estava acontecendo
em sua cidade:
Hoje, o planalto de Poos de Caldas no serve mais. Minrio acabou.
S mancha, nunclemais.
Mas esto tapando os buracos, trazendo para c Torta II1, aquele lixo do vizinho que voc no gostaria de ver
jogado no quintal da sua casa.
Sentimentos mil: do povo, do poeta e do Brasil.
Hugo Pontes. In: M.E.M. Helene. A radioatividade e o lixo nuclear. So Paulo: Scipione, 2002, p. 4.
1Torta II lixo radioativo de aspecto pastoso.
A indignao que o poeta expressa no verso Sentimentos mil: do povo, do poeta e do Brasil est relacionada com:

a) extino do minrio decorrente das medidas adotadas pela metrpole portuguesa para explorar as riquezas
minerais, especialmente em Minas Gerais.
b)a deciso tomada pelo governo brasileiro de receber o lixo txico oriundo de pases do Cone Sul, que caracteriza
chamado comercio internacional do lixo.
c) atitude de moradores que residem em casas prximas umas das outras, quando um deles joga lixo no quintal do
vizinho.
d)s chamadas operaes tapa-buracos, desencadeadas com o objetivo de resolver problemas de manuteno das
estradas que ligam as cidades mineiras.
e)os problemas ambientais que podem ser causados quando se escolhe um local para enterrar ou depositar lixo txico.

















488




PROF.: Alexandre Borges




C
C
o
o
n
n
t
t
e
e

d
d
o
o
s
s
:
:

E
E
q
q
u
u
i
i
l
l

b
b
r
r
i
i
o
o
q
q
u
u

m
m
i
i
c
c
o
o
,
,
c
c
o
o
n
n
s
s
t
t
a
a
n
n
t
t
e
e
s
s
d
d
e
e
e
e
q
q
u
u
i
i
l
l

b
b
r
r
i
i
o
o
,
,
p
p
H
H
e
e

p
p
O
O
H
H
,
,
l
l
e
e
i
i
d
d
e
e
L
L
e
e
C
C
h
h
a
a
t
t
e
e
l
l
i
i
e
e
r
r
.
.


























D Da at ta a d da a E En nt tr re eg ga a : : _ __ __ __ __ __ __ __ _/ /_ __ __ __ __ __ __ __ __ __ __ _/ /2 20 01 13 3

LISTA 4 QUIMICA

489

1. (UCG/GO/Janeiro/2004) As proposies, a seguir, tratam de contedos variados. Leia-as atentamente para
assinal-las.

01. Para cada 100 mL de um meio de cultura destinado ao cultivo de bactrias do gnero Mycoplasma, tem-se a
adio de 2,0 mL de soluo de glicose a 50% (m/v) e 0,25 mL de soluo de acetato de tlio a 10% (m/v).
Portanto, a quantidade de glicose adicionada de 1,0 g e a de acetato de tlio de 0,25 g.

02. Com relao proposio anterior, tem-se que o tlio adicionado ao meio sob a forma de acetato um metal do
grupo de transio interna, cujo smbolo T e nmero atmico 81.

03. A reao entre um hormnio (H) e seu receptor (R) , formando o complexo HR, reversvel e pode ser
representada pela equao abaixo. Sobre esse sistema, pode-se dizer que K
1
representa a constante de formao
e K
2
, a constante de dissociao do complexo.
H + R HR
K
1
K
2

04. Sobre a equao anterior, a expresso que representa a constante de dissociao (Kd) do complexo HR :
| |
| | | | R H
HR
Kd

=


05. Com relao ao equilbrio, ainda na proposio 03, a determinao de Kd em fragmentos de alguns tecidos til
para se prever a resposta do paciente ao tratamento hormonal de certos tipos de tumores. Isso possvel porque,
observando-se a equao, aps atingido o equilbrio, para cada mol de complexo HR formado, um mol se dissocia em
H e R.

06. Ainda, considerando-se o equilbrio da mesma proposio, tem-se que valores de Kd variando entre 1,0 . 10
10
e
2,0 . 10
9
indicam que a reao favorvel dissociao do complexo HR.

02 - (Fuvest/SP/1 Fase/2007) -A transformao de um composto A em um composto B, at se atingir o equilbrio
(A

B), foi estudada em trs experimentos. De um experimento para o outro, variou-se a concentrao inicial do
reagente A ou a temperatura ou ambas. Registraram-se as concentraes de reagente e produto em funo do tempo.
Com esses dados, afirma-se:

Com esses dados, afirma-se:
I. Os experimentos 1 e 2 foram realizados mesma temperatura, pois as
constantes de equilbrio correspondentes so iguais.
II. O experimento 3 foi realizado numa temperatura mais elevada que o
experimento 1, pois no experimento 3 o equilbrio foi atingido em um
tempo menor.
III. A reao endotrmica no sentido da formao do produto B.
Dessas afirmaes,
a. todas so corretas.
b. apenas I e III so corretas.
c. apenas II e III so corretas.
d. apenas I correta.
e.apenas II correta.

03 - (UEPG/PR/Janeiro/2008) - So misturados 2 mols de H
2
(g) com 3 mols de Cl
2
(g) num recipiente fechado de V
litros de capacidade, a uma determinada
temperatura. Sabendo que 80% do H
2
(g) reagiu na formao do HCl(g) e que a equao desse equilbrio H
2
(g) +
Cl
2
(g)

2HCl(g) ,
assinale o que for correto.
01. Resta no equilbrio 0,4 mol desse gs.
Caderno de Atividades

Disciplina:
Qumica

Professor(a):
Alexandre
Aluno:
3 ano
Ensino Mdio
Data de Recebimento:
_____/_____/_____
Lista 04

Data Entrega:
_____/_____/_____
A
A
A
B
B
B
9
6
3
0
tempo
c
o
n
c
e
n
t
r
a

o

490
02. Participa da reao 1,6 mol de Cl
2
(g) , restando 1,4 mol no equilbrio.
04. No equilbrio existem 3,2 mols de HCl(g) .
08. A constante de equilbrio pode ser representada por
| |
| || |
2 2
2
c
Cl . H
HCl
K =
16. O valor da constante de equilbrio, Kc, considerando um volume V, igual a 18,3.

05 - (Unicamp/SP/2007) - Cerca de 90% da crosta e do manto terrestres so formados por minerais silicticos.
Entender muitos processos geoqumicos significaconhecer bem o comportamento dessas rochas em todos os
ambientes. Um caso particular desse comportamento na crosta a solubilizao da slica (SiO
2
) por gua a alta
temperatura e presso. Esse processo de dissoluo pode ser representado pela equao:
SiO
2
(s) + 2H
2
O(aq) H
4
SiO
4
(aq)
Em determinado pH a 300C e 500 atmosferas, a constante de equilbrio para essa dissoluo, considerando a gua
como solvente, de 0,012.
a. Escreva a expresso da constante de equilbrio para esse processo de dissoluo.
b. Determine a concentrao em g L
1
de H
4
SiO
4
aquoso quando se estabelece o equilbrio de dissoluo nas condies
descritas.

05 - (PUC/SP/2007) O grfico abaixo correlaciona os valores da constante de equilbrio (Kc) em funo da
temperatura para a reao de sntese da amnia:
N
2
(g)+ 3H
2
(g)

2NH
3
(g)
0,30
0,25
0,20
0,15
0,10
0,05
0,00
700 750 800 850 900 950 1000
T (K)

Sobre o comportamento dessa reao, no intervalo de temperatura considerado no experimento, foram feitas algumas
afirmaes:
I. A reao exotrmica no sentido de formao da amnia.
II. Com o aumento da temperatura, a velocidade da reao diminui.
III. Com o aumento da temperatura, o rendimento da reao diminui, formando-se menos amnia na situao de
equilbrio.
SOMENTE est correto o que se afirma em
a. I.
b. II.
c. III.
d. I e II.
e. I e III.

06 - (Fuvest/SP/1 Fase/2008) Em uma experincia, aqueceu-se, a uma determinada temperatura, uma mistura de
0,40 mol de dixido de enxofre e 0,20 mol de oxignio, contidos em um recipiente de 1L e na presena de um
catalisador. A equao qumica, representando a reao reversvel que ocorre entre esses dois reagentes gasosos, :
2SO
2
(g) + O
2
(g) 2SO
3
(g) . As concentraes dos reagentes e do produto foram determinadas em vrios tempos,
aps o incio da reao, obtendo-se o grfico:

Em uma nova experincia, 0,40 mol de trixido de enxofre, contido em um
recipiente de 1L, foi aquecido mesma temperatura da experincia anterior e na
presena do mesmo catalisador. Acompanhando-se a reao ao longo do tempo,
deve-se ter, ao atingir o equilbrio, uma concentrao de SO
3
de aproximadamente:
a. 0,05 mol/L
b. 0,18 mol/L
c. 0,20 mol/L
d. 0,35 mol/L
e.0,40 mol/L

0,10
0,20
0,30
0,40
Concentrao
(mol/L)
Tempo

491
07 - (Unifor/CE/2008) Considere as reaes que ocorrem quando H
2
gasoso posto em contato com os halognios
F
2
e I
2
gasosos. Com flor, a reao explosiva e completa a temperatura ambiente. Com iodo, a reao lenta,
acontece somente a altas temperaturas e incompleta.
Sendo assim, pode-se afirmar que:
I. a reao de flor com hidrognio deve ser muito rpida;
II. a reao de iodo com hidrognio deve atingir o equilbrio com sobra de reagentes;
III. volumes iguais de iodo e hidrognio ao reagirem formam um volume de produto que menor do que a soma dos
volumes reagentes consumidos (volumes gasosos medidos nas mesmas condies de PeT).
correto afirmar, SOMENTE:
a.I
b.II
c.III
d.I e II
e. II e III

08 - (UFRJ/RJ/2008) - Um mtodo de produo de cianeto de hidrognio a nitrogenao do acetileno em fase
gasosa, de acordo com a equao: N
2
(g)+ C
2
H
2
(g)

2HCN(g). O diagrama a seguir indica os valores das


concentraes (em mol / L) dos compostos N
2
, C
2
H
2
e HCN em equilbrio, a vrias temperaturas diferentes e mostra
que a temperaturas distintas correspondem diferentes condies de equilbrio.

a.Determine a constante de reao K
c
da equao de formao de HCN, temperatura de 300
0
C.
b.Explique por que a reao de produo de HCN endotrmica.


09 - (UFG/2 Etapa/2003) - O processo Haber da sntese da amnia pode ser representado pela equao a
seguir:
a-Escreva a equao da constante de equilbrio e fornea sua unidade.
b-Compare e explique os rendimentos da reao a 25C e a 450C, dadas as constantes de equilbrio nessas
temperaturas:
Seo 1.01 Temperatura (C) Constante de Equilbrio
25 7,6x 10
2
450 6,5x 10
-3


10 - (Unificado/RJ/2007) - Os gases provenientes da eletrlise da gua do mar foram recolhidos em um recipiente
fechado de capacidade igual a 5 litros. A mistura recolhida apresentava 7,5 moles de hidrognio e 5 moles de cloro,
que reagiram de acordo com a seguinte equao: H
2
(g) + Cl
2
(g)

2 HCl(g). Sendo assim, a constante de equilbrio,


em termos de concentrao molar (Kc) a uma dada temperatura em que 5 moles de HCl(g) foram obtidos, ser:
a.1,0
b.2,0
c.2,5
d.4,0
e. 5,0

11 - (UFRJ/RJ/2009) - O acetato de etila, usado como essncia artificial de frutas e como solvente para vernizes e
lacas, entre outras aplicaes, preparado a partir do cido actico e do etanol. A reao para sua obteno pode ser
representada pela equao: CH
3
COOH+ C
2
H
5
OH CH
3
COOCH
2
CH
3
+ H
2
O
a.Partindo-se de 3 moles de cido actico e 3 moles de etanol e realizando-se a reao acima em reator fechado, a
100C, verifica-se que o equilbrio alcanado aps reagirem dois moles do cido.
Determine a constante de equilbrio da reao.
b. A reao de obteno do acetato de etila pode ser realizada na aparelhagem mostrada a seguir, que permite retirar
continuamente a gua originada na reao, por meio da destilao azeotrpica.
2 2 3 (g) (g) (g)
N + 3H 2NH

492

O que ocorrer com equilbrio desta reao quando se retirar a gua formada?

12 - (Unificado/RJ/2006) - O grfico a seguir representa a variao das concentraes das substncias X, Y e Z
durante a reao em que elas tomam parte.

A equao que representa a reao :
a. X + Z Y
b. X + Y Z
c. X Y + Z
d. Y X + Z
e. Z X + Y

13 - (FMTM/MG/2008) -O dixido de carbono no organismo humano transportado pelo sangue de trs modos: 6%
como CO
2
dissolvido; 70% como HCO

3
e 24% na hemoglobina (Hb). Em um indivduo normal, o pH do sangue deve
manter entre os valores 7,35 e 7,45. O transporte de gs carbnico pode ser representado, simplificadamente, pelas
seguintes equaes:
H
+
(aq) + Hb

+ CO
2
(aq)

HHbCO
2
(aq)
CO
2
(g)

CO
2
(aq)
H
2
O(l) + CO
2
(g)

H+(aq) + HCO

3
(aq)
Considerando-se que no h reguladores de pH no sangue, pode-se afirmar que, com
a. o aumento da concentrao de CO
2
(g), o pH diminui.
b. a diminuio da concentrao de HCO

3
(aq), o pH aumenta.
c. a diminuio da concentrao de CO
2
(g), o pH no se altera.
d. o aumento da concentrao de HCO

3
(aq), o pH diminui.
e.o aumento da concentrao de HCO

3
(aq), o pH no se altera.


493
14 - (UFMG/MG/1Fase/2007) - A decomposio do carbonato de prata produz xido de prata e dixido de carbono,
conforme indicado nesta equao:
Ag
2
CO
3
(s)

Ag
2
O (s) + CO
2
(g)
Essa reao foi investigada em diferentes temperaturas, partindo-se, sempre, de 1 mol de Ag
2
CO
3
.
Nesta tabela, esto indicadas as quantidades de dixido de carbono presentes no estado de equilbrio nas
temperaturas investigadas:
TemperaturaC77 127 177 227
Quantidadede
CO / mol 0,00014 0,0043 0,031 0,36
2

Considerando-se essas informaes, CORRETO afirmar que a decomposio de Ag
2
CO
3
:
a. endotrmica e um aumento da presso aumentaria a quantidade de CO
2
produzida.
b. endotrmica e um aumento da presso diminuiria a quantidade de CO
2
produzida.
c. exotrmica e um aumento da presso aumentaria a quantidade de CO
2
produzida.
d. exotrmica e um aumento da presso diminuiria a quantidade de CO
2
produzida.

15 - (Mackenzie/SP/2008) -Se, ao equilbrio acima, se adicionar uma soluo de NaOH,
NH
4
1+
(aq) + OH
1-
(aq.)

NH
3
(g) + H
2
O (lq.)
a. a quantidade de amnia liberada aumenta.
b. a concentrao do on amnio aumenta.
c. o pH da soluo em equilbrio diminui.
d. no h qualquer alterao.
e.a quantidade de amnia liberada diminui.


16 - (UFG/1Etapa/2000) - Analisando o sistema em equilbrio a seguir a 25C e 1 atm.
(w) Pb(NO
3
)
2(aq)
+ (x)KI
(aq)
KNO
3(aq) +
(z)PbI
2(s)

(PbI
2
um precipitado amarelo) conclui-se que :
01-a constante de equilbrio dada por;
02-o aumento da concentrao do KI aumenta a formao do precipitado amarelo (PbI
2
);
04-o aumento da concentrao do KNO
3
aumenta a solubilidade do precipitado amarelo (PbI
2
);
08-os coeficientes w, x, y e z da equao so 1, 2, 2 e 1, respectivamente;
16-o aumento da concentrao do Pb(NO
3
)
2
aumenta a velocidade no sentido v
2

32-a velocidade v
1
da reao dada por: v
1
=K
1
[Pb(NO
3
)
2
]
w
[KI]
x
.

17 - (Vunesp/SP/2007) - Para evitar o desenvolvimento de bactrias em alimentos, utiliza-se cido benzico como
conservante. Sabe-se que:
I) Em soluo aquosa, ocorre o equilbrio:
COOH COO
-
-
+ H
+
(BZH)
(BZ)

II) A ao bactericida devida exclusivamente forma no dissociada do cido (BzH).
III) Quando [BzH] =[Bz
-
], o pH da soluo 4,2.
Com base nestas informaes, e considerando a tabela seguinte,
Artigo II. Alimento .........................pH
Refrigerante....................3,0
Picles...............................3,2
Leite................................6,5
pode-se afirmar que possvel utilizar cido benzico como conservante do:
a. refrigerante, apenas.
b. leite, apenas.
c. refrigerante e picles, apenas.
d. refrigerante e leite, apenas.
e.picles e leite, apenas.



3 2
2 3 K = ________________ c
[ ] [ ]
[ ] [ ]
Pb(NO ) KI
KNO PbI
w x
z y

494
18 - (FEPA/2008) - A expresso para a constante de equilbrio da reao: Zn
(s)
+ Cu
2+
(aq)
= Zn
2+
(aq)
+ Cu
(s)
dada por:
a.[Zn
2+
(aq)
] [Cu
(s)
]
[Zn
(s)
] [Cu
2+
(aq)
]
b.[Zn
(s)
]
[Cu
(s)
]
c. [Zn
2+
(aq)
]
2

[Cu
2+
(aq)
]
2

d. [Cu
(s)
] .
[Cu
2+
(aq)
]
e. [Zn
2+
(aq)
]
[Cu
2+
(aq)
]

19 - (Mackenzie/SP/2008) - Sejam os equilbrios aquosos e suas constantes de ionizao a 25C:
+
+ -
- -4
-5
HF H + F K = 10
HA H + A K = 10
1
2
- -
HF + A HA + F
O valor da constante de equilbrio da reao abaixo, vale:
a. 10
-
9

b. 10
-
5

c. 10
d. 10
-
1

e. 10
-
20


20-(Enem)- A chuva em locais no poludos levemente cida. Em locais onde os nveis de poluio so altos, os
valores do pH da chuva podem ficar abaixo de 5,5, recebendo, ento, a denominao de .chuva cida.. Este tipo de
chuva causa prejuzos nas mais diversas reas: construo civil, agricultura, monumentos histricos, entre outras.
A acidez da chuva est relacionada ao pH da seguinte forma: concentrao de ons hidrognio = 10-pH , sendo que o
pH pode assumir valores entre 0 e 14. Ao realizar o monitoramento do pH da chuva em Campinas (SP) nos meses de
maro, abril e maio de 1998, um centro de pesquisa coletou 21 amostras, das quais quatro tm seus valores mostrados
na tabela:

anlise da frmula e da tabela permite afirmar que:
I. da 6 para a 14 amostra ocorreu um aumento de 50% na
acidez.
II. a 18 amostra a menos cida dentre as expostas.
III. a 8 amostra dez vezes mais cida que a 14.
IV. as nicas amostras de chuvas denominadas cidas so a 6 e a 8
So corretas apenas as afirmativas
(A) I e II
(B) II e IV.
(C) I, II e IV.
(D) I, III e IV.
(E) II, III e IV.

21- Enem 2007- Diretores de uma siderurgica, para evitar o desmatamento e adequar a empresa s normas de
proteo ambiental, resolveram mudar o combustvel dos fornos da indstria. O carvo vegetal foi ento substitudo
pelo carvo mineral. Entretanto, foram observadas alteraes ecolgicas graves em um riacho das imediaes, tais
como a morte dos peixes e dos vegetais ribeirinhos. Tal fato pode ser justificado em decorrncia:
(A) da diminuio de resduos orgnicos na gua do riacho, reduzindo a demanda de oxignio na gua.
(B) do aquecimento da gua do riacho devido ao monxido de carbono liberado na queima do carvo.
(C) da formao de cido clordrico no riacho a partir de produtos da combusto na gua, diminuindo o pH.
(D) do acmulo de elementos no riacho, tais como, ferro, derivados do novo combustvel utilizado.
(E) da formao de cido sulfrico no riacho a partir dos xidos de enxofre liberados na combusto.




495
22- (Enem-2004)-Essa questo j foi formulada por sbios da Grcia antiga. Hoje responderamos que:
(A) a evaporao da gua dos oceanos e o deslocamento do vapor e das nuvens compensam as guas dos rios que
desguam no
mar.
(B) a formao de geleiras com gua dos oceanos, nos plos, contrabalana as guas dos rios que desguam no mar.
(C) as guas dos rios provocam as mars, que as transferem para outras regies mais rasas, durante a vazante.
(D) o volume de gua dos rios insignificante para os oceanos e a gua doce diminui de volume ao receber sal
marinho.
(E) as guas dos rios afundam no mar devido a sua maior densidade, onde so comprimidas pela enorme presso
resultante da coluna de gua.

23- (Enem-2004)-H estudos que apontam razes econmicas e ambientais para que o gs natural possa vir a tornar-
se, ao longo deste sculo, a
principal fonte de energia em lugar do petrleo. Justifica-se essa previso, entre outros motivos, porque o gs natural
(A) alm de muito abundante na natureza um combustvel renovvel.
(B) tem novas jazidas sendo exploradas e menos poluente que o petrleo.
(C) vem sendo produzido com sucesso a partir do carvo mineral.
(D) pode ser renovado em escala de tempo muito inferior do petrleo.
(E) no produz CO2 em sua queima, impedindo o efeito estufa.

24- ( Enem-2004)- As previses de que, em poucas dcadas, a produo mundial de petrleo possa vir a cair tm
gerado preocupao, dado seu
carter estratgico. Por essa razo, em especial no setor de transportes, intensificou-se a busca por alternativas para a
substituio do petrleo por combustveis renovveis. Nesse sentido, alm da utilizao de lcool, vem se propondo,
no Brasil,
ainda que de forma experimental,
(A) a mistura de percentuais de gasolina cada vez maiores no lcool.
(B) a extrao de leos de madeira para sua converso em gs natural.
(C) o desenvolvimento de tecnologias para a produo de biodiesel.
(D) a utilizao de veculos com motores movidos a gs do carvo mineral.
(E) a substituio da gasolina e do diesel pelo gs natural.

25- (Enem-2005)- Emsetembro de 1998, cerca de 10.000 toneladas de cido sulfrico (H2SO4) foram derramadas pelo
navio Bahamas no litoral
do Rio Grande do Sul. Para minimizar o impacto ambiental de um desastre desse tipo, preciso neutralizar a acidez
resultante.
Para isso pode-se, por exemplo, lanar calcrio, minrio rico em carbonato de clcio (CaCO3), na regio atingida.
A equao qumica que representa a neutralizao do H2SO4 por CaCO3, com a proporo aproximada entre as
massas
dessas substncias :





Pode-se avaliar o esforo de mobilizao que deveria ser empreendido para enfrentar tal situao, estimando a
quantidade de
caminhes necessria para carregar o material neutralizante. Para transportar certo calcrio que tem 80% de CaCO3,
esse
nmero de caminhes, cada um com carga de 30 toneladas, seria prximo de
(A) 100. (B) 200. (C) 300. (D) 400. (E) 500.











496







PROF.: Alexandre Borges




C
C
o
o
n
n
t
t
e
e

d
d
o
o
s
s
:
:

E
E
q
q
u
u
i
i
l
l

b
b
r
r
i
i
o
o
q
q
u
u

m
m
i
i
c
c
o
o
c
c
o
o
n
n
s
s
t
t
a
a
n
n
t
t
e
e
d
d
o
o
p
p
r
r
o
o
d
d
u
u
t
t
o
o
d
d
e
e

s
s
o
o
l
l
u
u
b
b
i
i
l
l
i
i
d
d
a
a
d
d
e
e
e
e
h
h
i
i
d
d
r
r

l
l
i
i
s
s
e
e
.
.


























D Da at ta a d da a E En nt tr re eg ga a : : _ __ __ __ __ __ __ __ _/ /_ __ __ __ __ __ __ __ __ __ __ _/ /2 20 01 13 3
LISTA 5 QUIMICA

497

01 - (Mackenzie/SP/2008) -Se, ao equilbrio acima, se adicionar uma soluo de NaOH,
NH
4
1+
(aq) + OH
1-
(aq.)

NH
3
(g) + H
2
O (lq.)
a. a quantidade de amnia liberada aumenta.
b. a concentrao do on amnio aumenta.
c. o pH da soluo em equilbrio diminui.
d. no h qualquer alterao.
e.a quantidade de amnia liberada diminui.

02 - (UFPelotas/RS/2Fase/Janeiro/2007) - O cloro um dos mais utilizados agentes sanitizantes nas operaes de
potabilizao da gua. A utilizao de derivados clorados tem contribudo para o controle de doenas de origem
hdrica. As reaes abaixo so exemplos da utilizao do hipoclorito de sdio na desinfeco da gua. NaClO + H
2
O

HClO + NaOH. A partir desses dados, responda s perguntas a seguir.


Na =23; Cl = 35,5; O = 16; H = 1
O que ocorreria com o equilbrio da reao se aumentssemos a concentrao hidrogeninica do meio? Justifique.

03 - (Vunesp/SP/2008) O hipoclorito ClO

pode ser preparado pela reao representada pela seguinte equao:


Cl
2
(aq) +2OH

(aq)

ClO

(aq) + Cl

(aq) + H
2
O(l)

Composto Solubilidade
18C (mol/L)
Cl H
AgNO
AgCl
KNO
Cl K
9,4
8,3
10
2,6
3,9
-5

Considerando, ainda, as informaes constantes na tabela, qual substncia, ao ser adicionada ao sistema, aumentar
o rendimento da reao?
a. HCl.
b. AgNO
3
.
c. AgCl.
d. KNO
3
.
e. KCl.

04 - (UFPiau/PI/2007) Em 1918, Fritz Haber ganhou o prmio Nobel de qumica pela descoberta do mtodo direto de
sntese da amnia (NH
3
) a partir de seus elementos, de acordo com a reao: N
2(g)
+ 3H
2(g)

2NH
3(g)

Considerando que esta uma reao exotrmica, a elevao da temperatura implica em aumento final de produo de
NH
3
. Operar nessas condies, todavia, representa um elevado custo de produo. Para aumentar a produo de NH
3
,
mantendo-se a temperatura fixa e sem afetar a expresso da constante de equilbrio, necessrio:
a.aumentar o fator de freqncia entre as molculas.
b.diminuir a energia de ativao do sistema.
c.aumentar a presso do reator.
d.aumentar a viscosidade do meio reacional.
e. adicionar uma espcie inica qualquer que se dissocie completamente no meio.




Caderno de Atividades

Disciplina:
Qumica

Professor(a):
Alexandre
Aluno:
3 ano
Ensino Mdio
Data de Recebimento:
_____/_____/_____
Lista 05

Data Entrega:
_____/_____/_____

498
05-(UFMT/MT/2008) -Mato Grosso: somos os maiores A importncia da agricultura para o estado de Mato Grosso (MT)
freqentemente noticiada sob o ponto de vista do comrcio e do censo agrcola, por exemplo, o complexo da soja
exportou sozinho mais de 800 milhes de dlares no primeiro semestre de 2003; ou, a safra mato-grossense de
algodo responde por 60% da produo nacional; ou ainda, MT o maior produtor de soja e algodo do Brasil. Uma
leitura crtica dessas manchetes desvela a outra face das moedas e estatsticas da nossa produo agrcola: somos
tambm os maiores consumidores nacionais de biocidas. Segundo levantamentos preliminares feitos por estudantes
de Agronomia da UFMT, na safra 2000/01, somente os municpios de Rondonpolis e Campo Verde consumiram
juntos, aproximadamente, 750 mil litros de herbicidas, 430 mil litros de inseticidas e 85 mil litros de fungicidas. Os
mesmos estudos revelaram que os princpios ativos mais utilizados foram o Glifosato e o 2,4 D (em herbicidas), o
Endosulfan (em inseticidas), e o Carbendazin (em fungicidas).
C
OH
O
CH
2
N
H
CH
2
P
O
OH
OH
Glifosfato
Cl
Cl
Cl
Cl
Cl
O
S O
O
Cl
Endosulfan
Cl Cl
O COOH
2,4 D
N
N
N
H
C
H
O
O
H
3
C
Carbendazin



Esses agroqumicos so biologicamente ativos por desenho e, a despeito do potencial para atacarem o DNA ou
interferirem no sistema de informao biolgico (neste caso, atuam como hackers), transformaram-se em parte
importante de nossa agricultura empresarial. A solubilidade em gua do 2,4 D 45g/dm
3
(25
o
C e seu pKa 2,6. A
solubilidade do Glifosato, por outro lado, 12 g/dm
3
e seus pKas so 0,8 (1
o
grupo fosfnico), 2,3 (grupo carboxlico),
6,0 (2
o
grupo fosfnico) e 11,0 (grupo amino).
A partir desses dados, assinale a afirmativa correta.
a. Solues aquosas de 2,4 D so mais cidas que solues aquosas de Glifosato de mesma concentrao em
mol/dm
3
.
b. Numa mesma temperatura, volumes iguais de solues saturadas de 2,4 D e de Glifosato possuem a mesma
quantidade de matria.
c. A concentrao de ons H
+
de equilbrio numa soluo de Glifosato resultante apenas da ionizao do primeiro
grupo fosfnico da molcula.
d. A solubilidade em gua do 2,4 D e do Glifosato aumenta com o aumento do pH.
e. O pKb do grupo amina do Glifosato 11.

04- UFMT/2008 - Quando a alimentao humana deficiente em vitamina C, pode ocorrer uma sria doena cujos
sintomas so gengivas inchadas e com sangramento fcil, dentes abalados e suscetveis a queda, sangramentos
subcutneos e cicatrizao lenta. Por sculos essa doena foi comum, principalmente no meio dos navegantes, que
no dispunham de frutas ctricas ou verduras frescas em suas viagens. No era incomum perder grande parte de uma
tripulao numa jornada martima. Essa trgica doena descrita por Luiz de Cames em sua obra clssica Os
Lusadas:
E foi que, de doena crua e feia/ A mais que eu nunca vi, desempararam Muito a vida, e em terra estranha e alheia / os
ossos para sempre sepultaram Quem haver que, sem o ver, o creia / que to disformemente ali lhe incharam As
gengivas na boca, que crescia / a carne e juntamente apodrecia?
H 70 anos, foi realizada a primeira sntese em laboratrio da vitamina C, conforme a reao qumica abaixo:
CHO
C HO H
C OH H
C H HO
CH
2
OH
fenilhidrazina
KCN
CN
C H OH
C O
C OH H
C H HO
CH
2
OH
H
2
O
CO
2
H
C H OH
C O
C OH H
C H HO
CH
2
OH
H
2
O
CH
C
C
C
O
O
OH
HO
C
H
HO
CH
2
OH
L-xilose
c. Ascrbico

(vitamina C)

Apesar de presentes no leite e no fgado, as melhores fontes de vitamina C so frutas frescas, batata assada e
verduras. Como exemplo, temos limo (50 mg /100 g), laranja (47 mg/100 g), goiaba (302 mg /100 g), brcolis (109 mg/
100 g), pimento verde (720 mg/100 g).

499
A ingesto diria de cido ascrbico deve ser igual quantidade excretada ou destruda por oxidao. Um adulto sadio
perde 3% a 4% de sua reserva corporal diariamente. Para um adulto no fumante manter uma reserva mnima de
1.500 mg de vitamina C, necessria a absoro de 60 mg ao dia; para o fumante, so necessrios 67% a mais.
(Adaptao do artigo: A importncia da Vitamina C Qumica Nova na Escola Maio/2003 - SBQ/ Diviso de Ensino)
A partir das informaes do texto INCORRETO afirmar que
a. o pH de uma soluo aquosa de vitamina C menor do que 7.
b. a doena crua e feia, citada por Cames, conhecida como escorbuto.
c. uma goiaba de 80 g contm menos do que a quantidade diria de vitamina C requerida por um adulto para manter a
reserva de 1500 mg ou mais de vitamina C.
d. a ingesto de 80 g de brcolis, como nico alimento ingerido que contm vitamina C, no atende a necessidade de
um fumante.
e. o cido ascrbico comumente utilizado como antioxidante para preservar o sabor e a cor natural de muitos
alimentos, como frutas, legumes processados e laticnios.

05 (UEM/PR/2007) -Assinale a(s) alternativa(s) correta(s).
01.Aps atingido o equilbrio qumico, uma soluo aquosa de nitrito de potssio (KNO
2
) uma soluo bsica. (Dado:
Kado HNO
2
= 10
-4
)
02. O pH de uma soluo aquosa de hidrxido de sdio 0,1 mol/L igual a 1.
04. Na reao entre o HNO
2
e o SO
2
4
, formando NO

2
e HSO

4
-, o HNO
2
atua como um cido de Brnsted e o SO
2
4
-,
como uma base de Lewis.
08. A25C, ao se misturarem volumes iguais de uma soluo de AgNO
3
(aq) 0,2 mol/L com uma soluo de KCl(aq) 0,2
mol/L, o AgCl precipitar. (Dado: Kps do AgCl = 1,6 x10
-10
, a25C)
16. Se o pH dos fluidos estomacais humanos cerca de 1,7 e a molaridade de H
+
dos fluidos pancreticos 6 x 10
-
9
mol/L, ento o fluido estomacal mais cido que o fluido pancretico.
32. Soluo-tampo aquela cujo pH praticamente no se altera com a adio de uma base ou de um cido em
quantidade limitada.

06 - (UFPelotas/RS/2Fase/Janeiro/2008) - A gua no poluda, que se precipita na forma de chuva, neblina ou neve
naturalmente cida (com pH mdio de aproximadamente 5,6), devido presena natural do dixido de carbono
atmosfrico nela dissolvido, originando o cido carbnico, que se ioniza segundo o equilbrio: CO
2
(g) + H
2
O(l) H
2
CO
3

(aq)

H
+
+ HCO

3
. Em algumas regies, a gua da chuva pode apresentar valores de pH = 5,0 ou, inferiores,
decorrentes da contribuio de cidos fortes (cido ntrico, cido sulfrico), causadores da chamada chuva cida. Os
cidos inorgnicos tm origem nas emisses de SO
2
, NO e NO
2
, oriundos da queima de combustveis fsseis, em
veculos, indstrias e termeltricas, segundo as reaes, por exemplo para os derivados do enxofre:
SO
2
(g) + 1/2 O
2
(g) SO
3
(g)
SO
3
(g) + H
2
O(l) H
2
SO
4
(aq)

Com base no exposto e em seus conhecimentos, faa o que se pede.
a. Demonstre por que a dissoluo do trixido de enxofre gasoso em gua apresenta reao cida.
b. Ao dissolvermos 1,12 L de trixido de enxofre gasoso em gua, perfazendo 1L de soluo, nas CNTP, considerando
a dissociao doproduto formado em 100%, calcule o pH da soluo aquosa. V
M
= 22,4L
c. A chuva cida corri as estruturas metlicas com ferro. Equacione a reao.

07 - (Unifor/CE/2007 Considere certa quantidade de gua pura, no estado lquido, temperatura de 25C. Nestas
condies, o produto inico da gua (KW) vale 1,0 x 10
14
. Esse valor aumenta com o aumento de temperatura e
diminui com a diminuio da temperatura.
Sendo assim, pode-se afirmar que, na gua lquida pura,
I. a 40 C, o pH maior do que 7.
II. h maior concentrao de ons OH

(aq)
a 0 C do que a 25 C.
III. [H
+
(aq)] = [OH

(aq)], independentemente da temperatura.


correto o que se afirma SOMENTE em:
a.I
b.II
c.III
d.I e II
e. II e III

08 - (UFSCar/1 Fase/2008) Em um bquer, um qumico misturou 100 mL de uma soluo diluda de base forte,
XOH, de pH = 13 com 400 mL de uma soluo diluda de cido forte, HA, de pH = 2.
Dados pH = log [H
+
], pOH = log [OH

], pH + pOH = 14, e considerando os volumes aditivos e os eletrlitos 100%


dissociados, o valor aproximado do pH da soluo final
a. 2.
b. 6.
c. 8.
d. 10.
e.12
.

500

09 - (UFU/MG/1Fase/2008) A amnia (NH
3
) um gs incolor de odor muito irritante, fabricada na indstria, em
enormes quantidades, pelo processo Haber-Bosh. Sua principal aplicao na fabricao de cido ntrico (HNO
3
),
sendo tambm largamente empregada na fabricao de fertilizantes como NH
4
NO
3
e (NH
4
)
2
SO
4
, e na fabricao de
produtos de limpeza domstica, como o amonaco.
Em relao aos compostos citados no texto, considere as seguintes afirmativas.
I Nos compostos HNO
3
e (NH
4
)
2
SO
4
, o N aparece com nmero de oxidao igual a +5 e 3, respectivamente.
II Os compostos NH
4
NO
3
e (NH
4
)
2
SO
4
so compostos inicos.
III As solues aquosas de NH
4
NO
3
e de (NH
4
)
2
SO
4
so ms condutoras de eletricidade.
IV A soluo aquosa de NH
3
deve apresentar pH = 7.
Marque a alternativa que apresenta afirmaes corretas.
a. Apenas I e III.
b. Apenas III e IV.
c. Apenas I, II e IV.
d. Apenas I e II.


10 - (UFG/1Etapa) - Observe o grfico a seguir:
14
12
10
8
6
4
2
0
10
0
10
-2
10
-4
10
-8
10
-10
10
-6
10
-12
10
-14
[H O ]
3
+
.
.
.
.
.
.
Suco gstrico
Suco de limo
Suco de tomate
Sangue
gua do mar
Leite de magnsia
p
H

Sobre as informaes presentes nesse grfico, correto afirmar-se que:
01-o suco de limo 100 vezes mais cido que o suco de tomate;
02-o leite de magnsia possui concentrao de OH
-
igual a 1 x 10
-4
mol/L;
04-a concentrao de hidrognios cidos igual ao pH;
08-a ingesto de gua pura diminui momentaneamente o pH do estmago;
16-o sangue mais cido que o suco gstrico;
32-misturando-se 505 mL, de uma soluo de NaOH 0,01 mol/L a 495 mL de uma soluo de HCl 0,01 mol/L, o pH
final ser igual ao da gua do mar


11 - (UFG/2 Etapa) - O quadro a seguir relaciona diversos materiais com seus respectivos pH, aproximados:
Material pH
Leite de vaca 6,5
Sangue
humano
7,3
Suco de laranja 4
Leite de
magnsia
10,5
Vinagre 3,0

Intervalo de viragem da fenolftalena
- - - -
0 8 9,8 14
(pH)
incolor vermelho

Considerando-se as informaes anteriores, responda:
a-qual a concentrao molar de hidroxilas no vinagre?
b-qual a concentrao hidrogeninica no suco de laranja?
c-qual o material mais bsico? Justifique.
d-utilizandose apenas a fenolftalena como indicador, pode-se afirmar que o suco de laranja cido? Justifique.

501

12- (UFG/2 Etapa/Grupo-I/2004) A presena de dixido de carbono em gua provoca a dissoluo de rochas
calcrias, tornando a gua dura. A dureza dessa gua pode ser reduzida com a adio de hidrxido de clcio.
DADOS:
K
ps

CaCO
3
Ca(OH)
2

3,0 x 10
9
4,0 x 10
6

a. Escreva as equaes qumicas que representem os equilbrios qumicos estabelecidos pela presena de dixido de
carbono em gua.
b. Escreva a equao de dissoluo de carbonato de clcio em gua que contm dixido de carbono dissolvido.
c. Explique como o hidrxido de clcio reduz a dureza da gua.


13 - (UEPG/PR/Janeiro/2008) - A determinao qualitativa do pH pode ser feita por meio de substncias denominadas
indicadores, que tm a propriedade de indicar a colorao de acordo com o meio: cido ou bsico. Com base na tabela
abaixo, que apresenta alguns indicadores comumente utilizados em laboratrio, com suas respectivas coloraes,
assinale o que for correto.

01. Uma amostra de um efluente aquoso de uma indstria, com [OH

] = 10
10
, apresenta colorao vermelha em
presena de fenolftalena.
02. Uma amostra de clara de ovo cujo pOH igual a 6 apresenta colorao azul em presena de verde de
bromocresol.
04. Um sabonete lquido com alto teor alcalino apresenta colorao amarela em presena do alaranjado de metila.
08. Uma amostra de um determinado refrigerante incolor cuja concentrao de ons [H
+
] igual a 10
4
apresenta
colorao violeta quando em presena de vermelho congo.
16. Uma amostra de gua do mar com pH igual a 8 apresenta colorao vermelha em presena de vermelho de metila.

14 - (Acafe/SC/Janeiro/2008) Cremes dentais e sabonetes formam solues aquosas que adquirem cor avermelhada
em presena de fenolftalena. Isso significa que so solues:
a. cidas
b.bsicas
c.neutras
d. gasosas
e. incolores

15 - (Acafe/SC/Julho/2008) O suco resultante das lavagens do repolho roxo apresenta as seguintes coloraes numa
escala de pH.
pH 3,5 6,5 9,0 11 13 13.5
cor vermelho rosa roxo azul verde amarelo
Utilizando este suco como indicador da presena de um cido, as cores que podem ser produzidas so:
a.roxo e azul
b.rosa e amarelo
c.azul e verde
d.verde e amarelo
e. rosa e vermelho

16-(EFEI/SP/2009) Uma substncia A reage com um metal, liberando um gs. Este gs alimenta a combusto. A
substncia A em meio aquoso, conduz corrente eltrica. Com adio de fenolftalena soluo, a cor permanece
inalterada. A provavelmente:
a.um cido.
b.uma base.
c.um sal.
d.um xido.
e. um metal.



502
17 - (ITA/SP/2006) - Considere os dois eletrodos (I e II) seguintes e seus respectivos potenciais na escala do eletrodo
de hidrognio
o
(E ) e nas condies-padro:
2F
-
(aq) 2e
-
(CM) + F
2
(g) E
o
I
= 2,87V
Mn
2+
(aq) +4H
2
O (l) 5e
-
(CM) + 8H
+
(aq) + MnO

4
E
o
II
= 1,51V
A fora eletromotriz de um elemento galvnico construdo com os dois eletrodos acima de
a. 1,81V.
b. 1,13V.
c. 0,68V.
d. 1,36V.
e. 4,38V.


18 - (IME/RJ/2007) Uma pilha de combustvel utiliza uma soluo de KOH e dois eletrodos porosos de carbono, por
onde so admitidos, respectivamente, hidrognio e oxignio. Este processo resulta numa reao global de combusto
que gera eletricidade. Considerando que a pilha opera nas condies padro:
a. calcule a entropia padro de formao da gua lquida;
b. justifique por que a reao da pilha espontnea;
c. avalie a variao da entropia nas vizinhanas do sistema.

19 - (Mackenzie/SP/2006) -Dados os potenciais de reduo das semi-reaes, I e II,
(I) Cu
2+
+ 2e

Cu
0
AE
0
= + 0,34 V
(II) Al
3+
+ 3e

Al
0
AE
0
= 1,66 V ,
o valor da ddp da pilha Al / Al
3+
// Cu / Cu
2+
:
a. + 4,30 V.
b. 2,00 V.
c. + 1,32 V.
d. + 2,00 V.
e. 1,32 V.

20 - (UEPG/PR/Janeiro/2008) - A respeito de uma pilha com eletrodos de nquel e estanho, com base nas semi-
equaes desses dois metais e os respectivos
potenciais de oxidao, conforme as formulaes abaixo, assinale o que for correto.
Ni

Ni
2+
+ 2 e

E
0
oxid
= + 0,25 V
Sn

Sn
2+
+ 2 e

E
0
oxid
= + 0,14 V
01. A reao de oxirreduo nessa pilha no um processo espontneo.
02. No eletrodo de estanho dessa pilha ocorre oxidao.
04. O eletrodo de nquel nessa pilha denominado nodo.
08. A equao global dessa pilha dada por Ni
0
+ Sn
2+
Ni
2+
+ Sn
0
.
16. A diferena de potencial, AE
0
, nessa pilha igual a + 0,11 V.

21 - (UEM/PR/2007) - Considerando as reaes abaixo com seus respectivos potenciais-padro de reduo, assinale
a(s) alternativa(s) correta(s).
I) 2H
+
+ 2 e
-
H
2
E
o
= 0,0 V
II) Ti
2+
+ 2e
-
Ti E
o
= -1,63 V
III) Cr
2+
+ 2e
-
Cr E
o
= 0,56 V
IV) Fe
2+
+ 2e
-
Fe E
o
= 0,44 V
V) Au
+
+e
-
AuE
o
= 1,68 V
VI) O
2
+ 2H
2
O + 4e
-
4OH
-
E
o
= 1,23 V
01. Uma clula eletroqumica montada com os eletrodos das reaes III e IV possui uma reao global espontnea
com AE
o
= 1,0 V.
02. O valor do potencial-padro de oxidao do eletrodo representado pela reao II 1,63 V.
04. Uma pilha montada com os eletrodos das reaes III e V, utilizando-se uma ponte salina, pode ser representada
por Cr/Cr
2+
//Au
+
/Au.
08. A corroso do ferro est baseada na sua oxidao. Sendo assim, dentre os metais acima, o ouro seria o mais
indicado para proteger o ferro contra a corroso.
16. Considerando que a reao global que representa a corroso do ferro pode ser determinada atravs das reaes
IV e VI, para se produzir 2,0 g de ferrugem, Fe(OH)
2
, so necessrios, aproximadamente, 50 L de O
2
, nas CNTP,
considerando o O
2
como um gs ideal.
(Dados: Fe = 56; O = 16; H = 1)



503
22- ( Enem-2004)- Ferramentas de ao podem sofrer corroso e enferrujar. As etapas qumicas que correspondem a
esses processos podem ser
representadas pelas equaes:

Uma forma de tornar mais lento esse processo de corroso e formao de ferrugem engraxar as ferramentas. Isso se
justifica
porque a graxa proporciona
(A) lubrificao, evitando o contato entre as ferramentas.
(B) impermeabilizao, diminuindo seu contato com o ar mido.
(C) isolamento trmico, protegendo-as do calor ambiente.
(D) galvanizao, criando superfcies metlicas imunes.
(E) polimento, evitando ranhuras nas superfcies.

23- Enem-2004 - As previses de que, em poucas dcadas, a produo mundial de petrleo possa vir a cair tm
gerado preocupao, dado seu
carter estratgico. Por essa razo, em especial no setor de transportes, intensificou-se a busca por alternativas para a
substituio do petrleo por combustveis renovveis. Nesse sentido, alm da utilizao de lcool, vem se propondo,
no Brasil,
ainda que de forma experimental,
(A) a mistura de percentuais de gasolina cada vez maiores no lcool.
(B) a extrao de leos de madeira para sua converso em gs natural.
(C) o desenvolvimento de tecnologias para a produo de biodiesel.
(D) a utilizao de veculos com motores movidos a gs do carvo mineral.
(E) a substituio da gasolina e do diesel pelo gs natural.

24- (Enem2007) -Qual das seguintes fontes de produo de energia a
mais recomendvel para a diminuio dos gases
causadores do aquecimento global?
A leo diesel.
B Gasolina.
C Carvo mineral.
D Gs natural.
E Vento.

25- (Enem-2003)- A gua do mar pode ser fonte de materiais utilizados pelo ser humano, como os exemplificados no
esquema abaixo.







504









PROF.: Alexandre Borges






C
C
o
o
n
n
t
t
e
e

d
d
o
o
s
s
:
:

E
E
l
l
e
e
t
t
r
r
o
o
q
q
u
u
i
i
m
m
i
i
c
c
a
a
:
:
p
p
i
i
l
l
h
h
a
a
s
s
e
e
e
e
l
l
e
e
t
t
r
r

l
l
i
i
s
s
e
e
,
,
e
e
e
e
s
s
t
t
u
u
d
d
o
o
d
d
a
a

r
r
a
a
d
d
i
i
o
o
a
a
t
t
i
i
v
v
i
i
d
d
a
a
d
d
e
e

























D Da at ta a d da a E En nt tr re eg ga a : : _ __ __ __ __ __ __ __ _/ /_ __ __ __ __ __ __ __ __ __ __ _/ /2 20 01 13 3
LISTA 6 QUIMICA

505


01-(UFPiau/PI/2008) Pesquisas na rea mdica tm demonstrado que pessoas com elevados nveis de ferro (Fe. no
sangue apresentam maior possibilidade de sofrer derrame cerebral do que aquelas com nveis mais baixos. Essa
concluso atribuda ligao do Fe aos tomos de nitrognio (N) ou enxofre (S) presentes em algumas enzimas.
Essas ligaes danificam as clulas em um processo denominado estresse oxidativo, onde o metal atua como doador
de eltrons. Analise as afirmativas abaixo e marque a opo correta.
a.O Fe sofre reduo nas reaes oxidantes que transportam o oxignio no sangue.
b.As enzimas atuam como catalisadores nas reaes oxidantes e na hemoglobina, que transporta oxignio no sangue.
c.A possibilidade do Fe misturar-se a outros elementos qumicos atribuda sua alta eletronegatividade.
d.O Fe danifica as clulas ao perder eltrons para os outros elementos no processo de estresse oxidativo.
e. O fato do oxignio ser um gs ideal, aumenta a eficincia da ao enzimtica quanto ao seu transporte no sangue.

02 - (UFPiau/PI/2007) Os solos, por mais secos que paream, sempre contm gua, o que os torna excelentes
meios eletrolticos. Para proteger uma tubulao metlica contra o processo de corroso, faz-se uso, freqentemente,
de uma tcnica denominada proteo catdica ou eletrodo de sacrifcio, conforme ilustrao da figura abaixo:

Anlise as afirmativas abaixo.
I. Quanto mais pura a gua do solo, maior a passagem da corrente
eltrica.
II. O eletrodo de sacrifcio tem AG> 0 em relao ao metal da
tubulao.
III. Ao formar a pilha com a tubulao, o eletrodo de sacrifcio o nodo.
Marque a opo correta.
a.Apenas I verdadeira.
b.Apenas II verdadeira.
c.Apenas III verdadeira.
d.Apenas I e II so verdadeiras.
e. Apenas II e III so verdadeiras.

03 - (Vunesp/SP/2008) A equao seguinte indica as reaes que ocorrem em uma pilha:Zn(s) + Cu
2+
(aq)
Zn
2+
(aq) + Cu(s). Podemos afirmar que:
a. o zinco metlico o ctodo.
b. o on cobre sofre oxidao.
c. o zinco metlico sofre aumento de massa.
d. o cobre o agente redutor.
e. os eltrons passam dos tomos de zinco metlico aos ons de cobre.

04 - (UFCE/1 Fase/2006) As clulas a combustvel, capazes de converter energia das reaes de oxidao-reduo
de reagentes qumicos gasosos diretamente em eletricidade, so consideradas tecnologias prontas para substituir
combustveis derivados do petrleo. A clula a combustvel hidrognio-oxignio baseia-se na conhecida reao de
formao de gua, onde os gases so oxidados e reduzidos, em compartimentos de eletrodos separados por soluo
eletroltica
2H
2(g)
+ O
2(g)
2 H
2
O
(l)
+ Energia
Assinale a alternativa correta.
a.Hidrognio reduzido no anodo, segundo a semi-reao H
2(g)
+ 4OH

(aq)
4H
2
O
(l)
+ 4e


b.Oxignio reduzido no catodo, segundo a semi-reao O
2(g)
+ 2H
2
O
(l)
+ 4e

4OH

(aq)

c.Hidrognio oxidado no anodo, segundo a semi-reao 2H
+
(aq)
+ 4OH

(aq)
4H
2
O
(l)
+ 4e


d.Oxignio oxidado no anodo, segundo a semi-reao O
2(g)
+ 2H
2
O
(l)
+ 4e

4OH

(aq)

e. Oxignio reduzido no catodo, segundo a semi-reao 2O
-
(g)
+ 2H
2
O
(l)
+4e

4OH

(aq)







Caderno de Atividades

Disciplina:
Qumica

Professor(a):
Alexandre
Aluno:
3 ano
Ensino Mdio
Data de Recebimento:
_____/_____/_____
Lista 06

Data Entrega:
_____/_____/_____

506
05 - (Fatec/SP/2006) A ilustrao refere-se a um experimento em que lminas metlicas so imersas em solues de
solutos inicos.
prata
soluo
de ZnSO
4
Tubo-1
zinco
soluo
de AgNO
3
Tubo-2
zinco
soluo
de MgSO
Tubo-3
4
cobre
soluo
de AgNO
3
Tubo-4

Analisando-se os valores dos E
o
de reduo:
V 34 , 0 E
o
Cu / Cu
2
+ =
+

V 80 , 0 E
o
Ag / Ag
+ =
+

V 76 , 0 E
o
Zn / Zn
2
=
+

V 37 , 2 E
o
Mg / Mg
2
=
+

pode-se concluir que no sero observados sinais de transformao qumica
a. no tubo 1.
b. nos tubos 2 e 3.
c. no tubo 2.
d. nos tubos 1 e 3.
e. no tubo 4.


06 - (Fuvest/SP/1 Fase/2007) Trs metais foram acrescentados a solues aquosas de nitratos metlicos, de mesma
concentrao, conforme indicado na tabela. O cruzamento de uma linha com uma coluna representa um experimento.
Um retngulo escurecido indica que o experimento no foi realizado; o sinal (-) indica que no ocorreu reao e o sinal
(+) indica que houve dissoluo do metal acrescentado e precipitao do metal que estava na forma de nitrato.
Cd Co Pb
Cd(NO)
Co(NO)
Pb(NO)
2
2
2 3
3
3
+
+ +

Cada um dos metais citados, mergulhado na soluo aquosa de concentrao 0,1 mol/L de seu nitrato, um eletrodo,
representado por Me|Me
2+
, onde Me indica o metal e Me
2+
, o ction de seu nitrato. A associao de dois desses
eletrodos constitui uma pilha. A pilha com maior diferena de potencial eltrico e polaridade correta de seus eletrodos,
determinada
com um voltmetro, a representada por
a| || | . Cd Cd Pb Pb
2+2+
+

b| || | . Pb Pb Cd Cd
2+2+
+

c| || | . Cd Cd Co Co
2+2+
+

d| || | . Co Co Pb Pb
2+2+
+

e| || | . Pb Pb Co Co
2+2+
+

||
Obs:
significa ponte salina
significa plonegativo
significa plopositivo
+



507
07 - (UFV/MG/2008) A figura ao lado representa uma pilha, onde V um voltmetro, C uma chave e S a ponte
salina, que contm soluo saturada de nitrato de potssio (KNO
3
). O eletrodo de zinco est imerso na soluo de
sulfato de zinco (ZnSO
4
), e o eletrodo de prata est imerso na soluo de nitrato de prata (AgNO
3
).
V
C
S
Zn
Ag
Sol. de ZnSO
4
Sol. de AgNO
3
1mol L
-1
1mol L
-1

Ag
+
+ e
-
Ag E
o
= + 0,799
Zn
2+
+ 2e
-
Zn E
o
= - 0,763 v
Considerando esta pilha e os potenciais-padro de reduo acima representados, assinale a afirmativa CORRETA:
a.O eletrodo de prata perder massa.
b.Com o decorrer da reao, a soluo de ZnSO
4
ficar mais concentrada e a de AgNO
3
, mais diluda.
c.O potencial em V ser + 0,799 volts, com a chave C aberta.
d. medida que a reao se processa, os ctions K
+
da ponte salina se dirigem para a soluo de ZnSO
4
.
e. De acordo com os valores dos potenciais-padro, o Zn
2+
mais oxidante que Ag
+
.

08 - (UEPG/PR/Julho/2006) - Uma placa de certo metal M imersa numa soluo aquosa de CuSO
4
. Aps
determinado tempo, observa-se a deposio de cobre metlico sobre ela, e ainda, que a soluo, inicialmente azul, vai
perdendo sua colorao. Sobre este experimento, assinale o que for correto.
01. O metal M perde eltrons e forma ctions,


09 (UFPelotas/RS/2Fase/Julho/2006) - Uma pilha voltaica aproveita a eletricidade de uma reao qumica espontnea para acender
uma lmpada, como mostrado na figura abaixo. As tiras de alumnio e cobre, dentro de solues de sulfato de alumnio e sulfato de
cobre, respectivamente, agem como eletrodos. A ponte salina (nesse caso, cloreto de potssio) permite aos eltrons fluir entre os
copos de Becker sem que se misturem as solues. Quando o circuito entre os dois sistemas se completa, a reao gera uma corrente
eltrica. Observe que o metal da tira de alumnio consumido e a tira desaparece. A tira de cobre cresce medida que os eltrons
adicionais reagem com a soluo de sulfato de cobre para produzir metal adicional.
Enciclopdia Microsoft Encarta. 1993-2001 Microsoft Corporation [adapt.].


Baseado(a) no texto e em seus conhecimentos sobre o assunto,
a.escreva as semireaes que esto ocorrendo nos recipientes A e B.
b.aponte, justificando, o que se est oxidando e o que se est reduzindo.
c.justifique por que o sentido da corrente eltrica de A para B.

10 - (UFMA/MA/2Fase/2007) - Explique, utilizando argumentos termodinmicos e eletroqumicos, por que a reao
abaixo no ocorre espontaneamente.
Na
+
+ Cl
-
Na
o
+ 1/2 Cl
2
Dados:
1/2Cl
2
+ e
-
Cl
-
E
o
= + 1,36 V
Na
+
+ e
-
Na
o
E
o
= 2,71 V




508
11 - (ITA/SP/2006) - Um elemento galvnico constitudo pelos eletrodos abaixo especificados e separados por uma
ponte salina.
ELETRODO I: placa de chumbo metlico mergulhada em uma soluo aquosa 1 mol/L de nitrato de chumbo.
ELETRODO II: sulfato de chumbo slido prensado contra uma peneira de chumbo metlico mergulhada em uma
soluo aquosa 1 mol/L de cido sulfrico. Nas condies-padro, o potencial de cada um destes eletrodos, em
relao ao eletrodo padro de hidrognio,
V 1264 E
0
Pb / Pb
2
=
+
Eletrodo I
V 3546 E
0
SO , PbSO / Pb
2
4 4
=

Eletrodo II
Assinale a opo que contm a afirmao CORRETA sobre as alteraes ocorridas neste elemento galvnico quando
os dois eletrodos so conectados por um fio de baixa resistncia eltrica e circular corrente eltrica no elemento.
a.A massa de sulfato de chumbo slido na superfcie do ELETRODO II aumenta.
b.A concentrao de ons sulfato na soluo aquosa do ELETRODO II aumenta.
c.O ELETRODO I o plo negativo.
d.O ELETRODO I o anodo.
e. A concentrao de ons chumbo na soluo aquosa do ELETRODO I aumenta.

12 - (Acafe/SC/Janeiro/2007) Considere a pilha de Daniel, em que os eletrodos so de cobre e zinco. O zinco mais
eletropositivo que o cobre, logo, o zinco que doa eltrons para o cobre.
Em relao s consideraes acima, correto afirmar que, aps certo tempo:
a.a soluo de sulfato de zinco fica mais diluda.
b.a massa de cobre diminui.
c.a massa de zinco aumenta.
d.a soluo de sulfato de cobre fica mais concentrada.
e. a massa do cobre aumenta.

13 - (UFMS/MS/Conh.Gerais/2008) - Considere as semi-reaes, abaixo indicadas, com seus respectivos potenciais
padro de reduo, em volt (V).
e
-
+ Ag
+
(aq) Ag(s)AE
red
= + 0,80V
2e
-
+ Cu
2+
(aq) Cu(s)AE
red
= + 0,34V
A fora eletromotriz da cela, cujos eletrodos padro so Cu(s) / Cu
2+
(aq)// Ag
+
(aq) / Ag(s), :
a. - 4,6 V.
b. - 0,46 V.
c. + 0,46 V.
d. + 1,14 V.
e. - 1,14 V.

14 - (Unifesp/SP/2007) - Ferro metlico reage espontaneamente com ons Pb
2+
, em soluo aquosa. Esta reao pode
ser representada por:
Fe + Pb
2+

2+
+ Pb.
Na pilha, representada pela figura
soluo aquosa
contendoFe
soluo aquosa
contendoPb
2+
2+
Fe Pb
Ponte salina

em que ocorre aquela reao global,
a.os ctions devem migrar para o eletrodo de ferro.
b.ocorre deposio de chumbo metlico sobre o eletrodo de ferro.
c.ocorre diminuio da massa do eletrodo de ferro.
d.os eltrons migram atravs da ponte salina do ferro para o chumbo.
e. o eletrodo de chumbo atua como nodo.







509
15 - (UFG/1Etapa/2007) - O quadro abaixo contm informaes sobre radioistopos e suas aplicaes.
RADIOIS-
TOPO
EQUAO DE
DECAIMENTO
MEIA-
VIDA
APLICAO
Flor-18
Cobalto-60
F
18
9
O + 2
18
8
110min
tomografia por
emisso de p-
sitron
esterilizao de
alimentos
Co
60
27
Co
60
27
+

5,26anos

Interpretando as informaes do quadro, pode-se afirmar:
I. O consumo de alimentos contaminados com radiao oferece riscos sade, pois o cobalto-60 apresenta meia-
vida longa.
II. O flor-18 utilizado na tomografia de emisso de psitrons porque sua permanncia no organismo breve.
III. O cobalto-60, por ser emissor de radiao , utilizado em tomografia por emisso de psitrons.
So corretas as afirmaes:
a. I, apenas.
b. II, apenas.
c. III, apenas.
d. I e II.
e. II e III.

16 - (Fuvest/SP/1 Fase/2008) -Um contraste radiolgico, suspeito de causar a morte de pelo menos 21 pessoas, tem
como principal impureza txica um sal que, no estmago, reage liberando dixido de carbono e um on txico (Me
2+
).
Me um metal que pertence ao grupo dos alcalinoterrosos, tais como Ca, Ba e Ra, cujos nmeros atmicos so,
respectivamente, 20, 56 e 88. Istopos desse metal Me so produzidos no bombardeio do urnio-235 com nutrons
lentos: n
1
0
+ U
235
92
Me
142
+ Kr
36
+ 3 n
1
0
. Assim sendo, a impureza txica deve ser :
a. cianeto de brio.
b. cianeto de clcio.
c. carbonato de rdio.
d. carbonato de brio.
e.carbonato de clcio.

17 - (UEL/PR/2008) Os elementos radiativos tm muitas aplicaes. A seguir, esto exemplificadas algumas delas.
I. O iodo utilizado no diagnstico de distrbios da glndula tireide, e pode ser obtido pela seguinte reao:
Te
130
52
+ n
1
0
I
131
53
+ X
II. O fsforo utilizado na agricultura como elemento traador para proporcionar a melhoria na produo do milho, e
pode ser obtido pela reao:
Cl
35
17
+ n
1
0
P
32
15
+ Y
Sua reao de decaimento :
P
32
15
S
32
16
+Z
III. O tecncio usado na obteno de imagens do crebro, fgado e rins, e pode ser representado pela reao:
Tc
99
43
Tc
99
43
+Q
Assinale a alternativa que indica, respectivamente, os significados de X, Y, Z e Q nas afirmativas I, II e III:
a. , , ,
b. , , ,
c. , , ,
d. , , ,
e. , , ,

18-(UnB/DF/Janerio/2006) Uma das tcnicas de conservao de alimentos baseia-se no uso de radiao gama
proveniente, por exemplo, do decaimento do cobalto-60, que pode ser representado pelas equaes seguintes.
I
60
27
Co
60
28
Ni* +
0
1
|
II
60
28
Ni*
60
28
Ni +
A radiao gama, ao penetrar nos alimentos, mata os microrganismos que acelera o seu apodrecimento. Com base
nessas informaes, julgue os itens subseqentes.
01. A partir da equao I, correto concluir que o ncleo radioativo aumenta seu nmero atmico aps o decaimento.
02. A partcula emitida na equao I um eltron.
03. Segundo a equao II, a radiao emitida origina-se de transies que envolvem nveis eletrnicos.
04. Para que a radiaogama possa matar os microrganismo, necessrio adicionar uma substncia radioativa aos
alimentos, o que os contamina, podendo causar srios riscos sade humana.


510
19 - (UnB/DF/2005) - A glndula tireide acumula a maior parte do iodo que ingerido por um ser humano. No
organismo, o iodo interage com uma protena denominada tiroglobulina e os anis aromticos dessa protena tornam-
se iodados. Duas molculas de tiroglobulina iodadas interagem, formando uma molcula de tiroxina, ainda ligada
protena, como mostrado na reao I, abaixo. A tiroxina, hormnio tireoidiano, ento liberada pela quebra da cadeia
protica, conforme mostrado na reao II.
C CH
CH
NH
O
I
OH
I
+
I
O H
I
I
I
O
CH
CH
NH
C
O
2
2
C CH
CH
NH
O
I
OH
I
2
REAO - I

I
I
O
CH
CH
NH
C
O
2
OH
I
I
I
I
O
CH
CH
NH
CO H
2
OH
I
I
2
2
tiroxina
REAO - II

A deficincia de iodo no organismo pode ocasionar o desenvolvimento anormal da glndula tireide, o que conhecido
como bcio. Como medida preventiva a esse problema, tem sido recomendada a adio de um composto de iodo ao
sal de cozinha, material conhecido comercialmente como sal iodado. Imagens de tireide para diagnstico de doenas
podem ser geradas, usando-se detectores da radiao emitida por um determinado radioistopo. Para esse fim, o iodo-
131 (
53
I
131
) tem sido largamente utilizado, geralmente introduzido no organismocomo uma soluo aquosa de NaI. Esse
radioistopo, cujo decaimento produz um elemento
54
X
131
, possui meia-vida de oito dias.
Com referncia ao texto, julgue os itens seguintes.
01. Pelo fato de formar cristais brilhantes, a substncia iodo classificada como um metal.
02. A frmula qumica do sal iodado referido no texto NaI.
03. Para formar o elemento
54
X
131
, o iodo
53
I
131
deve emitir uma partcula o.
04. O elemento X um gs nobre.

20 - (UFG/1Etapa/)Observe o grfico, a seguir, que representa a seqncia de decaimento radioativo do urnio ao
chumbo:
Th
Th
Po
Po
Po
Ra
Pa U
U
234
234 234
238
90
90
91 92
92
230
226
88
222
86
Rn
218
84
84
84 Pb
Pb
Pb
214
210
206
82
82
82
Bi
Bi
214
210 210
214
M
a
s
s
a

A
t

m
i
c
a
Nmero Atmico
206
210
214
218
222
226
230
234
238

Nesse grfico,

01.Pb, Bi e Po so istopos de massa 214.02.esto representados dois istopos radioativos do chumbo (Pb).
03.o urnio decai a trio por emisso de radiao .
04.o radnio (Rn), um gs nobre, no radioativo.

511
21- Enem2001 - A tabela mostra a evoluo da frota de veculos leves, e o grfico, a emisso mdia do poluente
monxido de carbono (em
g/km) por veculo da frota, na regio metropolitana de So Paulo, no perodo de 1992 a 2000.



Comparando-se a emisso mdia de monxido de carbono dos veculos a gasolina e a lcool, pode-se afirmar que
I. no transcorrer do perodo 1992-2000, a frota a lcool emitiu menos monxido de carbono.
II. em meados de 1997, o veculo a gasolina passou a poluir menos que o veculo a lcool.
III. o veculo a lcool passou por um aprimoramento tecnolgico.
correto o que se afirma apenas em
(A) I. (B) I e II. (C) II. (D) III. (E) II e III.


22- Enem 2002 - Segundo uma organizao mundial de estudos ambientais, em 2025, .duas de cada trs pessoas
vivero situaes de carncia de
gua, caso no haja mudanas no padro atual de consumo do produto..
Uma alternativa adequada e vivel para prevenir a escassez, considerando-se a disponibilidade global, seria
(A) desenvolver processos de reutilizao da gua.
(B) explorar leitos de gua subterrnea.
(C) ampliar a oferta de gua, captando-a em outros rios.
(D) captar guas pluviais.
(E) importar gua doce de outros estados.

23-Enem-2003 - Segundo matria publicada em um jornal brasileiro, .Todo o lixo (orgnico) produzido pelo Brasil hoje .
cerca de 20 milhes de
toneladas por ano . seria capaz de aumentar em 15% a oferta de energia eltrica. Isso representa a metade da energia
produzida pela hidreltrica de Itaipu. O segredo est na celulignina, combustvel slido gerado a partir de um processo
qumico
a que so submetidos os resduos orgnicos..
O Estado de So Paulo, 01/01/2001.
Independentemente da viabilidade econmica desse processo, ainda em fase de pesquisa, na produo de energia
pela
tcnica citada nessa matria, a celulignina faria o mesmo papel
(A) do gs natural em uma usina termoeltrica.
(B) do vapor d.gua em uma usina termoeltrica.
(C) da queda d.gua em uma usina hidreltrica.
(D) das ps das turbinas em uma usina elica.
(E) do reator nuclear em uma usina termonuclear.

24- Enem-2002 -Em usinas hidreltricas, a queda d.gua move turbinas que
acionam geradores. Em usinas elicas, os geradores so
acionados por hlices movidas pelo vento. Na converso
direta solar-eltrica so clulas fotovoltaicas que produzem
tenso eltrica. Alm de todos produzirem eletricidade, esses
processos tm em comum o fato de
(A) no provocarem impacto ambiental.
(B) independerem de condies climticas.
(C) a energia gerada poder ser armazenada.
(D) utilizarem fontes de energia renovveis.
(E) dependerem das reservas de combustveis fsseis.



512
25 Enem-2006 -O funcionamento de uma usina nucleoeletrica tpica baseia-se na liberacao de energia resultante da
divisao do nucleo de uranio em nucleos de menor massa, processo
conhecido como fissao nuclear. Nesse processo, utiliza-se uma mistura de diferentes atomos de uranio, de forma a
proporcionar uma concentracao de apenas 4% de material fissil. Em bombas atomicas, sao utilizadas concentraes
acima de 20% de uranio fissil, cuja obtencao e trabalhosa, pois, na natureza, predomina o uranio nao-fissil. Em grande
parte do armamento nuclear hoje existente, utiliza-se, entao, como alternativa, o plutonio, material fssil produzido por
reacoes nucleares no interior do reator das usinas nucleoeletricas. Considerando-se essas informacoes, e correto
afirmar que:
A a disponibilidade do uranio na natureza esta ameacada devido a sua utilizacao em armas
nucleares.
B a proibicao de se instalarem novas usinas nucleoeletricasnao causara impacto na oferta mundial de energia.
C a existencia de usinas nucleoeletricas possibilita que um de seus subprodutos seja utilizado como material belico.
D a obtencao de grandes concentracoes de uranio fssil e viabilizada em usinas nucleoeletrica.
E a baixa concentracao de uranio fissil em usinas nucleoeletricas impossibilita odesenvolvimento energtico.







513







PROF.:Bencio Dias Honorio




C
C
o
o
n
n
t
t
e
e

d
d
o
o
s
s
:
:

O estudo da matria
A composio da matria.
Os estados fsicos da matria.
Mudanas de estado de agregao
Diagramas de mudana de estado de agregao.
Misturas e processos de separao de misturas
Equipamentos de Laboratrio.
M
M
i
i
n
n
i
i
s
s
t
t
r
r
a
a
d
d
o
o
s
s
d
d
u
u
r
r
a
a
n
n
t
t
e
e
o
o
m
m

s
s
d
d
e
e
f
f
e
e
v
v
e
e
r
r
e
e
i
i
r
r
o
o















D Da at ta a d da a E En nt tr re eg ga a : : _ __ __ __ __ __ __ __ _/ /_ __ __ __ __ __ __ __ __ __ __ _/ /2 20 01 13 3
LISTA 1 QUMICA

514

01 - (UEG GO/2008) O carbono um elemento qumico slido temperatura ambiente. Dependendo das condies
de formao, pode ser encontrado na natureza em diversas formas alotrpicas, entre as quais, grafite e diamante.
Esse elemento o pilar bsico da qumica orgnica, fazendo parte de todos os seres vivos. As principais propriedades
dos dois altropos mencionados no texto esto organizadas na tabela abaixo. Use-a, juntamente com a figura, para
responder aos itens subseqentes.

presses altas em exceto Infusvel, K 4600 : fuso de Ponto
dos translci Cristais opacos Cristais
cm g 3,5 a igual Densidade cm g 2,2 a igual Densidade
Mohs de escala na dureza Mxima eficincia alta de seco te Lubrifican
Isolante de eletricida de condutor Bom
Diamante Grafite
3 3
~




a) Explique por que grafite e diamante se comportam de forma diferente em relao ao fenmeno de fuso.
b) D a hibridizao dos tomos de carbono no grafite e no diamante e explique o fato de um deles ser isolante e o
outro, um bom condutor de eletricidade.

02 - (UEM PR/2007) Indique, dentre as matrias ar, sulfato de cobre (CuSO
4
), mercrio (Hg), arroz-doce, gasolina,
cristais de iodo, madeira e gs carbnico (CO
2
), dois exemplos de
a) substncias simples;
b) substncias compostas;
c) misturas homogneas;
d) misturas heterogneas.

03 - (UFG GO/2004) A idia proposta por Aristteles, da existncia de quatro elementos (terra, ar, gua e fogo)
constitutivos de toda a matria existente, mostrou-se no s equivocada como significou tambm um entrave para o
desenvolvimento da Qumica, dada influncia desse filsofo na construo do pensamento ocidental.
a) Cite duas substncias que participam da constituio ou produo de cada um dos elementos de Aristteles.
b) Escreva uma equao qumica que represente uma reao que pode ocorrer entre as substncias qumicas citadas
em (a).

04 - (UFRJ) Sobre o elemento qumico situado no sub-grupo 6A do segundo perodo da classificao peridica, um
estudante escreveu:
Forma duas substncias simples importantes: uma diatmica produzida nas reaes de combusto e consumida
pelos vegetais no processo de fotossntese e outra triatmica, presente na camada superior da atmosfera e que
absorve as radiaes ultravioletas.
No texto acima h dois erros conceituais. Reescreva corretamente o texto.

Caderno de Atividades

Disciplina:
Qumica

Professor(a):
Bencio
Aluno:
3 ano
Ensino Mdio
Data de Recebimento:
_____/_____/_____
Lista 01

Data Entrega:
_____/_____/_____

515
05 - (UEG GO/2008) Em um laboratrio de qumica, um estudante separou em frascos semelhantes trs solventes
que utilizaria em seu experimento. Entretanto, esqueceu de rotular esses frascos no momento da coleta e,
posteriormente, no tinha certeza a respeito do componente de cada um deles. Mas, conhecendo a densidade de cada
um dos lquidos, para sanar sua dvida, efetuou o seguinte experimento. Adicionou 3 mL de cada solvente em tubos de
ensaios separados e posteriormente adicionou 1 mL de gua. A anlise dos resultados permitiu a identificao
inequvoca dos componentes presentes em cada frasco. Os resultados observados para cada tubo de ensaio e a
tabela com as respectivas densidades dos lquidos esto mostrados na figura e na tabela abaixo:
5 , 1 HCCl
7 , 0 Gasolina
8 , 0 OH CH CH
0 , 1 O H
) (g.mL
C 25 a Densidade
Lquido
3
2 3
2
1


A partir das informaes acima, responda ao que se pede.
a) Determine as substncias presentes em cada um dos tubos, justificando em seguida o motivo de sua escolha.
b) Comente a validade da seguinte afirmativa: Todos os lquidos indicados na tabela acima so exemplos de
substncias puras.

06 - (UNICAMP SP/2010) Numa entrevista Revista n163, um astrofsico brasileiro conta que props, em um artigo
cientfico, que uma estrela bastante velha e fria (6.000 K), da constelao de Centauro, tem um ncleo quase
totalmente cristalizado. Esse ncleo seria constitudo principalmente de carbono e a estrela estaria a caminho de se
transformar em uma estrela de diamante, com a cristalizao do carbono.

a) O pesquisador relata ter identificado mais 42 estrelas com as mesmas caractersticas e afirma: Enquanto no termina
o processo de cristalizao do ncleo, as estrelas de diamante permanecem com a temperatura constante. No que diz
respeito temperatura, independentemente de seu valor absoluto, ele complementa essa afirmao fazendo uma
analogia entre o processo que ocorre na estrela e a solidificao da gua na Terra. Com base no conhecimento
cientfico, voc concorda com a analogia feita pelo pesquisador? Justifique.

b) Ao final da reportagem afirma-se que: No diamante da estrela, apenas 0,01 separa os ncleos dos tomos do
elemento que o compem. Considerando-se que o raio atmico do carbono no diamante da Terra de 0,77, quanto
valeria a relao numrica entre os volumes atmicos do carbono (Terra/estrela)? Mostre seu raciocnio.

07 - (UFU MG/2008) O grfico a seguir representa a curva de aquecimento de uma substncia presso constante
de 1 atm.

Pede-se.
a) Quais so os estados fsicos dessa substncia indicados pelas letras A, C e E?
b) Explique o fenmeno que ocorre na regio indicada pela letra D.
c) Qual o ponto de ebulio dessa substncia em C?
d) Qual o intervalo de temperatura em C, no qual estar o ponto de fuso dessa substncia?
08 - (UNIFESP SP/2007) Dois experimentos foram realizados em um laboratrio de qumica.

516

Experimento 1: Trs frascos abertos contendo, separadamente, volumes iguais de trs solventes, I, II e III, foram
deixados em uma capela (cmara de exausto). Aps algum tempo, verificou-se que os volumes dos solventes nos
trs frascos estavam diferentes.



Experimento 2: Com os trs solventes, foram preparadas trs misturas binrias. Verificou-se que os trs solventes
eram miscveis e que no reagiam quimicamente entre si. Sabe-se, ainda, que somente a mistura (I + III) uma
mistura azeotrpica.

a) Coloque os solventes em ordem crescente de presso de vapor. Indique um processo fsico adequado para
separao dos solventes na mistura (I + II).
b) Esboce uma curva de aquecimento (temperatura x tempo) para a mistura (II + III), indicando a transio de fases.
Qual a diferena entre as misturas (II + III) e (I + III) durante a ebulio?

09 - (ITA SP/2002) Na tabela abaixo so mostrados os valores de temperatura de fuso de algumas substncias. Em
termos dos tipos de interao presentes em cada substncia, justifique a ordem crescente de temperatura de fuso das
substncias listadas.

Substncia Temp. de Fuso (
o
C)
Bromo -7
gua 0
Sdio 98
Brometo de Sdio 747
Silcio 1414

10 - (UNICAMP SP) As curvas de fuso das substncias A e B esto
representadas na figura abaixo.

a) Quais as temperaturas de fuso de A e B?
b) A e B misturados em certa proporo formam uma soluo slida
(euttico), que funde em temperatura intermediria s de A e B puros. Em
que intervalo estar o ponto de fuso do euttico?

11 - (UFOP MG/2000)
O grfico abaixo representa a variao de temperatura observada ao se
aquecer uma substncia A durante cerca de 80 minutos.

0 20 40 60
10
20
30
40
50
T
e
m
p
e
r
a
t
u
r
a


/


o
C
Tempo / min


a) A faixa de temperatura em que a substncia A permaneca slida ________.
b) A faixa de temperatura em que a substncia A permanece lquida ________.
c) A temperatura de ebulio da substncia A ________.




517
12 - (UFG GO/2006) As tcnicas de separao dos componentes de uma mistura baseiam-se nas propriedades fsico-
qumicas desses componentes. Assim, considerando os sistemas, apresentados abaixo, associe as misturas s figuras
que representam os equipamentos adequados a suas separaes, bem como s propriedades fsico-qumicas
responsveis pela utilizao da tcnica. Justifique suas escolhas.
Sistema
a) gua e sulfato de brio
b) gua e tetracloreto de carbono
c) gua e etanol
Propriedade
1) Temperatura de ebulio
2) Solubilidade
3) Densidade
S ( ) P ( )
S ( ) P ( )
S ( ) P ( )

13 - (UEG GO/2005) Em uma feira de cincias, dois alunos propuseram um mtodo para dessalinizar a gua do mar
a fim de torn-la potvel, ou seja, prpria para o consumo humano, conforme a foto do evento mostrada abaixo:


Disponvel em:<http://www.cefetes.br/fotos/feira_ciencias/68.htm> Acesso em:
24 maio 2005.
a) Cite e explique a funo de quatro instrumentos de laboratrio essenciais para
a construo e o funcionamento do aparelho utilizado no processo de
dessalinizao da gua, mostrado na foto acima.
b) Cite dois motivos tcnicos que justifiquem ser prefervel preservar a gua
potvel do que produzi-la a partir da imensa quantidade de gua que existe no
mar.


14 - (UFG GO/2000) O desenho abaixo representa um armrio de um laboratrio de qumica:



Escolha, entre os objetos do armrio, o(s) equipamento(s) necessrio(s)
para se determinar, utilizando-se a tcnica de titulao, o grau de pureza de
1 tonelada de cido ctrico, que slido na temperatura ambiente. D o(s)
nome(s) e descreva o processo de utilizao desse(s) equipamento(s).












518
15 - (UFG GO/2009) Um alambique uma forma artesanal de realizar uma separao de misturas. O mesmo
procedimento pode ser realizado com vidrarias e equipamentos de laboratrios qumicos.



Considerando as vidrarias e equipamentos representados, responda:
a) Qual o nome da tcnica de separao de misturas, que representa o processo que ocorre no alambique?
b) Utilizando as vidrarias e os equipamentos representados acima, esquematize um aparelho de laboratrio para
realizar o mesmo processo que ocorre no alambique.

16 - (UFABC SP/2009)
O teor de clcio em uma amostra de conchas de massa igual a 5,0g foi determinado da seguinte maneira:
triturao das conchas;
aquecimento do material triturado com HCl;
filtrao;
precipitao de ons Ca
2+
sob forma de oxalato de clcio monoidratado, CaC
2
O
4
H
2
O, utilizando-se oxalato de
amnio, (NH
4
)
2
C
2
O
4
, como reagente;
filtrao e secagem do oxalato de clcio monoidratado;
calcinao em mufla, produzindo-se 2,0g de CaO como nico resduo slido.

a) Para cada uma das filtraes, indique o que retido no filtro e o que constitui o filtrado.
b) Escreva a equao da reao qumica que ocorre quando oxalato de clcio monoidratado aquecido na mufla.
c) Admitindo que todo o clcio presente nas conchas esteja sob a forma de CaCO
3
e que os outros componentes
dessas conchas no interferem nos procedimentos da anlise, calcule a porcentagem em massa de carbonato de
clcio na amostra de conchas analisada. Mostre os clculos.

17 - (UFRN/2008) Atualmente, o Brasil o maior produtor mundial de
etanol (CH
3
CH
2
OH) obtido a partir da cana-de-acar. Usado como
combustvel, em automveis, o etanol menos poluente que os
combustveis fsseis. Os monossacardeos, provenientes da sacarose
(cana-deacar), produzem, em presena de um microorganismo vivo
especfico, uma soluo que apresenta em torno de 8% de etanol.
O sistema abaixo utilizado, no laboratrio de anlise de uma
indstria, para a purificao do etanol.

a) Nomeie o processo de transformao da sacarose em etanol e o de
purificao do etanol (mostrado na figura).
b) Qual a funo da vidraria (V1) indicada nessa figura? Com base nas
interaes intermoleculares, explique por que, na mistura gasosa, a
quantidade do etanol ser maior que a da gua.



519
18 - (UEG GO/2006) Observe os esquemas abaixo, que revelam dois processos de separao de misturas,
conhecidos, respectivamente, como destilao simples e destilao fracionada. Em seguida, responda ao que se pede.

a) Como se explicam as diferenas entre os dois processos no que se refere ao ponto de ebulio dos componentes da
mistura?
b) Para a separao dos componentes de uma mistura gua mais leo pode-se usar um dos processos representados
nos esquemas acima? Explique.

19 - (CEFET PR/2009)
O diagrama a seguir representa as etapas de separao de uma mistura heterognea, em seus componentes finais.


Com base nestas informaes, correto afirmar que:

a) a fase lquida colorida forma uma mistura azeotrpica.
b) a etapa 1 s pode ser realizada por meio de uma destilao simples.
c) a etapa 2 pode ser realizada utilizando-se um funil de separao.
d) o lquido 2 ferve antes do lquido 1.
e) a etapa 2 pode ser realizada por meio de uma cristalizao fracionada.

20 - (Enem MEC/2009) A atmosfera terrestre composta pelos gases nitrognio (N
2
) e oxignio (O
2
), que somam
cerca de 99%, e por gases traos, entre eles o gs carbnico (CO
2
), vapor de gua (H
2
O), metano (CH
4
), oznio (O
3
) e
o xido nitroso (N
2
O), que compem o restante 1% do ar que respiramos. Os gases traos, por serem constitudos por
pelo menos trs tomos, conseguem absorver o calor irradiado pela Terra, aquecendo o planeta. Esse fenmeno, que
acontece h bilhes de anos, chamado de efeito estufa. A partir da Revoluo Industrial (sculo XIX), a concentrao
de gases traos na atmosfera, em particular o CO
2
, tem aumentado significativamente, o que resultou no aumento da
temperatura em escala global. Mais recentemente, outro fator tornou-se diretamente envolvido no aumento da
concentrao de CO
2
na atmosfera: o desmatamento.
BROWN, I. F.; ALECHANDRE, A. S. Conceitos bsicos sobre clima,
carbono, florestas e comunidades. A.G. Moreira & S.
Schwartzman. As mudanas climticas globais e os
ecossistemas brasileiros. Braslia: Instituto de Pesquisa
Ambiental da Amaznia, 2000 (adaptado).

Considerando o texto, uma alternativa vivel para combater o efeito estufa
a) reduzir o calor irradiado pela Terra mediante a substituio da produo primria pela industrializao refrigerada.
b) promover a queima da biomassa vegetal, responsvel pelo aumento do efeito estufa devido produo de CH
4
.
c) reduzir o desmatamento, mantendo-se, assim, o potencial da vegetao em absorver o CO
2
da atmosfera.
d) aumentar a concentrao atmosfrica de H
2
O, molcula capaz de absorver grande quantidade de calor.
e) remover molculas orgnicas polares da atmosfera, diminuindo a capacidade delas de reter calor.

520
21 - (Enem MEC/2009)
Analise a figura.


Disponvel em: http//www.alcoologia.net.
Acesso em: 15 jul. 2009 (adaptado).

Supondo que seja necessrio dar um ttulo para essa figura, a alternativa que melhor traduziria o processo
representado seria:
a) Concentrao mdia de lcool no sangue ao longo do dia.
b) Variao da frequncia da ingesto de lcool ao longo das horas.
c) Concentrao mnima de lcool no sangue a partir de diferentes dosagens.
d) Estimativa de tempo necessrio para metabolizar diferentes quantidades de lcool.
e) Representao grfica da distribuio de frequncia de lcool em determinada hora do dia.

22 - (Enem MEC/2009) O ciclo biogeoqumico do carbono compreende diversos compartimentos, entre os quais a
Terra, a atmosfera e os oceanos, e diversos processos que permitem a transferncia de compostos entre esses
reservatrios. Os estoques de carbono armazenados na forma de recursos no renovveis, por exemplo, o petrleo,
so limitados, sendo de grande relevncia que se perceba a importncia da substituio de combustveis fsseis por
combustveis de fontes renovveis.

A utilizao de combustveis fsseis interfere no ciclo do carbono, pois provoca

a) aumento da porcentagem de carbono contido na Terra.
b) reduo na taxa de fotossntese dos vegetais superiores.
c) aumento da produo de carboidratos de origem vegetal.
d) aumento na quantidade de carbono presente na atmosfera.
e) reduo da quantidade global de carbono armazenado nos oceanos.

TEXTO: 1 - Comum questo: 23

Apresentao da coletnea
A produo agrcola afeta as relaes de trabalho, o uso da terra, o comrcio, a pesquisa tecnolgica, o meio
ambiente. Refletir sobre a agricultura significa colocar em questo o prprio modo de configurao de uma sociedade.

1) O acar
O branco acar que adoar meu caf
nesta manh de Ipanema
no foi produzido por mim
nem surgiu dentro do aucareiro por milagre.
Vejo-o puro
e afvel ao paladar
como beijo de moa, gua
na pele, flor
que se dissolve na boca. Mas este acar
no foi feito por mim.
Este acar veio
da mercearia da esquina e tampouco o fez o Oliveira,
dono da mercearia.

521
Este acar veio
de uma usina de acar em Pernambuco
ou no Estado do Rio
e tampouco o fez o dono da usina.
Este acar era cana
e veio dos canaviais extensos
que no nascem por acaso
no regao do vale.
Em lugares distantes, onde no h hospital
nem escola,
homens que no sabem ler e morrem de fome
aos 27 anos
plantaram e colheram a cana
que viraria acar.
Em usinas escuras,
homens de vida amarga
e dura
produziram este acar
branco e puro
com que adoo meu caf esta manh em
Ipanema.
(Ferreira Gullar, Dentro da noite veloz. Rio de Janeiro: Civilizao Brasileira, 1975, p. 44, 45.)

23 - (UNICAMP SP/2007) O poema apresentado na coletnea faz aluso ao acar da cana. A preocupao do poeta
no com a qumica, embora passagens do poema possam permitir alguma leitura nessa rea.
Nas questes a serem respondidas, sero citadas algumas passagens do poema, que, sugerimos, seja lido no todo
para facilitar as respostas.
a) No incio o poeta fala em branco acar e depois usa vejo-o puro. Justifique, sob um ponto de vista qumico, por
que nem sempre apropriado associar as palavras branco e puro.
b) Mais frente, o poeta usa a construo: flor que dissolve na boca. Se essa frase fosse usada por um qumico,
como ele justificaria, atravs de interaes intermoleculares, o processo mencionado?
c) Quase ao final, o poeta usa a expresso: plantaram e colheram a cana que viraria acar. Se um qumico
estivesse usando essa frase numa explanao sobre o processo de fabricao do acar, muito provavelmente ele
colocaria, aps a palavra cana, uma seqncia de termos tcnicos para descrever o processo de obteno do
acar, e eliminaria as palavras que viraria acar. A seguir so listados os termos que o qumico usaria.
Coloque-os (todos) na seqncia certa que o qumico usaria ao descrever a produo do acar, reescrevendo a
frase completa: secaram-no, cristalizaram o acar, ensacando-o, concentraram o caldo, moeram-na,
centrifugaram-no.

TEXTO: 2 - Comum questo: 24

Eles esto de volta! Omar Mitta, vulgo Rango, e sua esposa Dina Mitta, vulgo Estrondosa, a dupla explosiva que j
resolveu muitos mistrios utilizando o conhecimento qumico (vestibular UNICAMP 2002). Hoje esto se preparando
para celebrar uma data muito especial. Faa uma boa prova e tenha uma boa festa depois dela. Embora esta prova se
apresente como uma narrativa ficcional, os itens a e b em cada questo de 1 a 12 devem, necessariamente, ser
respondidos.

24 - (UNICAMP SP/2008) Depois das 19 horas, os convidados comearam a chegar. Dina os recepcionava no bar,
onde havia dois baldes: um deles com gelo e o outro com gelo seco. Dina bradava aos quatro cantos: Isso faz a festa
tornar-se mais qumica, j que esses slidos sero usados para resfriar as bebidas! Para cada bebida, Estrondosa
escolhia o slido mais apropriado. Curiosamente algum pediu duas doses iguais de usque, uma com gelo e outra
com gelo seco, mas colocou os copos em uma mesa e no consumiu as bebidas. Passado um certo tempo, um colega
de faculdade resolveu verificar se Dina ainda era a sabichona de antigamente, e foi logo perguntando:
a) Esses slidos, quando colocados nas bebidas, sofrem transformaes. Que nomes so dados para essas duas
transformaes? E por que essas transformaes fazem com que as bebidas se resfriem?
b) Dina, veja essas figuras e pense naqueles dois copos de usque que nosso amigo no bebeu. Qual copo, da
situao inicial, corresponde ao copo d da situao final? Em algum dos copos, a concentrao final de lcool ficou
diferente da concentrao inicial? Por qu?
Obs: considerar a figura para responder ao tem b.



522
TEXTO: 3 - Comum questo: 25


Uma festa de aniversrio foi decorada com dois tipos de bales. Diferentes componentes gasosos foram usados para
encher cada tipo de balo. As figuras observadas representam as substncias presentes no interior de cada balo.


25 - (UFRJ/2009)
a) Indique quantos elementos diferentes e quantas substncias simples diferentes existem nos bales.
b) Classifique o tipo de sistema de cada balo quanto homogeneidade.
















































523












PROF.:Bencio Dias Honorio



C
C
o
o
n
n
t
t
e
e

d
d
o
o
s
s
:
:

Transformaes da Matria
- Fenmenos fsicos
- Fenmenos qumicos

- Estrutura do tomo
- Principais caractersticas
nmero de massa, atmico
- Evoluo do modelo atmico

M
M
i
i
n
n
i
i
s
s
t
t
r
r
a
a
d
d
o
o
s
s
d
d
u
u
r
r
a
a
n
n
t
t
e
e
o
o
m
m

s
s
d
d
e
e
m
m
a
a
r
r

o
o













D Da at ta a d da a E En nt tr re eg ga a : : _ __ __ __ __ __ __ __ _/ /_ __ __ __ __ __ __ __ __ __ __ _/ /2 20 01 13 3

LISTA 2 QUMICA

524

01 - (UFSCAR SP/2009) A compreenso e o controle das transformaes qumicas foram muito importantes para
grandes mudanas sociais e econmicas na sociedade, principalmente nos ltimos 200 anos. Atualmente, possvel
diferenciar as transformaes fsicas das transformaes qumicas, sendo que estas ltimas podem ser representadas
na forma de equaes qumicas. Considerando os fenmenos de degelo dos icebergs e a queima de gs metano,
produzido nos aterros sanitrios:

a) Escreva a frmula qumica do composto envolvido no processo que, dentre os fenmenos apontados, representa
transformao fsica.
b) Escreva a equao que representa a transformao qumica, dentre esses dois fenmenos.

02 - (UFG GO/2010) Observe a tabela de converses de energia a seguir.

- Engrenagem Frenagem Gerador Mecnica
o Galvaniza Ventilador
roupa
passar de Ferro
dor Transforma Eltrica
qumicas
Reaes
Msculo
alimentos
de Digesto
pilha ou
Bateria
Qumica
Qumica Mecnica Trmica Eltrica Para De


Considerando a tabela acima, responda:

a) em quais converses h ruptura de ligao qumica?
b) Quais converses so exemplos de fenmenos fsicos e quais so os de fenmenos qumicos? Indique as
converses na folha de respostas, usando a seguinte legenda: (Q) = fenmeno qumico e (F) = fenmeno fsico.

03 - (UFRR/2010) Sempre que tocamos, misturamos ou pesamos alguma coisa, estamos tratando com a matria. O
ouro, um osso ou a gua, so exemplos de matria. Observe com ateno os fenmenos abaixo:

I. o ponto de fuso do sdio metlico 97,8
o
C
II. a soda castica uma base
III. o ferro enferruja em rea de alta umidade
IV. a densidade da gua igual 1g . cm
-3


Considerando a ordem acima, classifique as propriedades como fsica ou qumica da matria e escolha a nica opo
que represente esta informao:

a) I qumica; II qumica; III qumica; IV - fsica.
b) I fsica; II fsica; III qumica; IV - fsica.
c) I fsica; II qumica; III fsica; IV - qumica.
d) I fsica; II fsica; III qumica; IV - qumica.
e) I fsica; II qumica; III - qumica; IV - fsica.

04 - (UFU MG/2008) Considere os processos descritos a seguir.

I. Obteno de gasolina a partir do petrleo.
II. Obteno de zinco a partir da calamina.
III. Obteno de gelo seco a partir de gs carbnico.
IV. Queima de uma vela.
V. Sublimao de iodo.

Marque a alternativa que rene somente transformaes qumicas:
a) II, IV e V.
b) I, II e IV.
c) II e IV.
d) I e II.
Caderno de Atividades

Disciplina:
Qumica

Professor(a):
Bencio
Aluno:
3 ano
Ensino Mdio
Data de Recebimento:
_____/_____/_____
Lista 02

Data Entrega:
_____/_____/_____

525
05 - (UFABC SP/2009) Os fogos de artifcio propiciam espetculos em diferentes eventos. Para que esses
dispositivos funcionem, precisam ter em sua composio uma fonte de oxignio, como o clorato de potssio (KClO
3
),
combustveis, como o enxofre (S
8
) e o carbono (C), alm de agentes de cor como o SrCl
2
(cor vermelha), o CuCl
2
(cor
verde esmeralda) e outros. Podem conter tambm metais pirofricos como Mg que, durante a combusto, emite
intensa luz branca, como a do flash de mquinas fotogrficas.

a) Escreva as equaes qumicas, balanceadas, que representam:
a decomposio do clorato de potssio, produzindo cloreto de potssio e oxignio diatmico;
a combusto do enxofre;
a combusto do magnsio.
b) Considerando o modelo atmico de Rutherford-Bohr, como se explica a emisso de luz colorida pela detonao de
fogos de artifcio?

06 - (UEG GO/2009)
Em 1911, Rutherford e colaboradores realizaram o experimento mostrado na figura abaixo:

i a maioria das partculas o, sem sofrer algum desvio, atravessaram livremente a lmina, produzindo cintilaes na
chapa fluorescente.
ii ocasionalmente, porm, algumas partculas o eram desviadas de sua trajetria, ao atravessarem a lmina,
produzindo cintilaes em pontos afastados da regio de incidncia da grande maioria das demais partculas o.
iii muito raramente, algumas partculas o eram refletidas ao incidir sobre a lmina de ouro.

CARVALHO, Geraldo Camargo de. Qumica moderna. So Paulo: Scipione, 1997 p. 15. (Adaptado).

Sabendo que as partculas o so carregadas positivamente e de acordo com o contexto e as informaes
apresentadas na figura,
a) comente como as observaes colhidas no experimento contriburam para Rutherford propor o seu modelo atmico.
b) comente a falha do modelo de Rutherford, segundo a fsica clssica, e como Bohr aprimorou esse modelo.

07 - (UFOP MG/2008) Com base nas teorias atmicas de Dalton e Bohr, responda s questes apresentadas a seguir.
a) A teoria atmica de John Dalton foi publicada em 1804. Duas idias importantes expressas nessa teoria esto
listadas a seguir. De que maneira o atual entendimento da teoria atmica difere de cada uma dessas duas idias?
a. Toda a matria composta de partculas minsculas e indivsiveis, chamadas tomos.
b. tomos de um mesmo elemento so idnticos em todos os aspectos.
b) Em 1913, o fsico dinamarqus Niels Bohr props uma teoria para explicar o espectro de emisso do hidrognio.
Essa teoria postulava que:
o eltron no tomo de hidrognio girava em torno do ncleo em rbitas fixas.
cada rbita representava um estado fixo de energia.
rbitas mais prximas do ncleo tinham menor energia que as rbitas mais afastadas.
De modo geral, como a atual compreenso da estrutura eletrnica dos tomos difere daquela proposta por Bohr para o
tomo de hidrognio?

08 - (UNESP SP/2006) O sucesso do modelo atmico de Niels Bohr estava na explicao da emisso de luz pelos
tomos. A emisso de luz provocada por uma descarga eltrica atravs do gs sob investigao. Bohr desenvolveu
um modelo do tomo de hidrognio que lhe permitiu explicar esse fenmeno.
a) Descreva o modelo de Bohr.
b) Descreva o que ocorre, segundo o modelo do tomo de Bohr, com o eltron do hidrognio quando submetido
descarga eltrica.






526
09 - (UEG GO/2006) A fabricao de fogos de artifcio requer um controle rigoroso das variaes do processo como,
por exemplo, a proporo dos componentes qumicos utilizados e a temperatura de exploso. A temperatura
necessria para acionar os fogos de artifcio de mdio e grande porte de cerca de 3600 C. a gerao desse calor
que responsvel pela produo de ondas luminosas, pois provoca a emisso atmica, ou seja, a emisso de luz que
ocorre quando o eltron sofre uma transio de um nvel mais energtico para outro de menor energia.
Considerando este assunto, responda aos itens abaixo:
a) A qual modelo atmico esse fenmeno de emisso de luz est ligado?
b) Explique esse fenmeno de emisso de luz em termos de eltrons e nveis de energia.

10 - (UFG GO/2006) Observe o trecho da histria em quadrinhos a seguir, no qual h a representao de um modelo
atmico para o hidrognio.


WATCHMEN. So Paulo: Abril, n. 2, dez. 1988.

Qual o modelo atmico escolhido pelo personagem no ltimo quadrinho? Explique-o.

11 - (UEPG PR/2010)
O conhecimento atmico atual fruto de muitos estudos anteriores. Abaixo esto descritas algumas concluses sobre
os estudos da estrutura atmica.

I. Quando um eltron do tomo recebe energia, salta para um nvel de maior energia e quando retorna ao nvel
anterior, cede energia recebida sob forma de radiao eletromagntica (Bohr).
II. Rutherford, ao fazer incidir partculas radioativas em uma lmina de ouro, observou que a maioria das partculas
atravessava a lmina, algumas se desviavam e poucas se refletiam.
III. tomos do mesmo elemento qumico apresentam a mesma massa (Dalton).

Nesse contexto, assinale o que for correto.

01. A afirmao II permitiu concluir que no centro do tomo existe um ncleo pequeno e denso.
02. Os estudos de Bohr implicaram no modelo de partcula-onda para o eltron.
04. A afirmao II permitiu concluir que no tomo h grandes espaos vazios.
08. A afirmao III est incorreta, o que pode ser comprovado pela existncia dos istopos.
16. Os fogos de artifcio e os letreiros de neon so aplicaes do princpio de Bohr.

12 - (UFG GO/2010) O esquema a seguir representa de modo simplificado o experimento de J. J. Thomson. Um feixe
de partculas sai do ctodo, passa atravs de um orifcio no nodo e sofre a influncia das placas metlicas A e B.

De acordo com esse esquema, o feixe se aproxima de A quando

a) as placas A e B forem negativas.
b) a placa A for negativa e a B, positiva.
c) a placa A for positiva e a B negativa.
d) as placas A e B forem positivas.
e) as placas A e B forem neutras.


527
13 - (UFPR/2010) Considere as seguintes afirmativas sobre o modelo atmico de Rutherford:

1. O modelo atmico de Rutherford tambm conhecido como modelo planetrio do tomo.
2. No modelo atmico, considera-se que eltrons de cargas negativas circundam em rbitas ao redor de um ncleo de
carga positiva.
3. Segundo Rutherford, a eletrosfera, local onde se encontram os eltrons, possui um dimetro menor que o ncleo
atmico.
4. Na proposio do seu modelo atmico, Rutherford se baseou num experimento em que uma lamnula de ouro foi
bombardeada por partculas alfa.

Assinale a alternativa correta.

a) Somente a afirmativa 1 verdadeira.
b) Somente as afirmativas 3 e 4 so verdadeiras.
c) Somente as afirmativas 1, 2 e 3 so verdadeiras.
d) Somente as afirmativas 1, 2 e 4 so verdadeiras.
e) As afirmativas 1, 2, 3 e 4 so verdadeiras.

14 - (UECE/2009) A primitiva noo de tomo surge na Grcia antiga, a partir de Demcrito, Leucipo e Epicuro;
avana at o sculo XX enriquecida com outras idias que ajudaram a desenhar o modelo atmico atual. Na Coluna I,
a seguir, esto listadas algumas contribuies para que se chegasse ao modelo atual de tomo e na Coluna II, os
nomes de seus autores.
Numere a Coluna II de acordo com a Coluna I, associando cada contribuio a seu autor.

Coluna I
1. Descoberta do eltron
2. Descoberta do ncleo e da eletrosfera
3. Descoberta dos nveis de energia dos tomos
4. Princpio da incerteza
5. Regra da mxima multiplicidade

Coluna II
( ) Rutherford
( ) Thomson
( ) Hund
( ) Bohr
( ) Heisenberg

Assinale a opo contendo a sequncia correta, de cima para baixo.

a) 1, 5, 2, 4, 3
b) 2, 4, 1, 3, 5
c) 2, 1, 5, 4, 3
d) 2, 1, 5, 3, 4

15 - (UFG GO) Escreva um perodo coerente, segundo os conceitos da Qumica, com no mximo cinco linhas e utilize,
no mnimo, cinco palavras a seguir:

Eltron Substncia Ncleo Separao
Heterognea Ligao Mistura Prton
Destilao Molcula Eletrosfera Homognea
Fase on Filtrao tomo

16 - (UFU MG/2009) Nas festas de Rveillon, o cu fica embelezado pelas cores emitidas pela queima dos fogos de
artifcio. A esses fogos so adicionadas substncias, cujos tomos emitem radiaes de luminosidades diferentes.
Considerando uma explicao para a observao das cores, na queima dos fogos de artifcio, por meio de modelos
atmicos propostos no incio do sculo XX, marque a alternativa INCORRETA.

a) Na emisso de energia, devido transio de eltrons, encontra-se uma explicao para a observao das cores
dos fogos de artifcios, pois segundo os estudos de Bohr, o eltron pode emitir ou absorver uma quantidade
definida de energia chamada quantum.
b) Os estudos realizados por Thomson, assim como o modelo atmico proposto por ele, reconhecem a natureza
eltrica da matria e explicam a eletrizao por atrito, a corrente eltrica, a formao dos ons e as descargas
eltricas em gases. Contudo, o modelo no explica as cores observadas na queima dos fogos de artifcio.
c) Os estudos realizados por Dalton, assim como o modelo atmico proposto por ele, contriburam para resgatar as
idias sobre o tomo, ao proporem que tomos diferentes possuem diferentes pesos atmicos. No entanto, o peso
atmico no o responsvel pela exibio das cores quando da queima dos fogos de artifcios.
d) De acordo com o modelo de Rutherford-Bohr, as cores produzidas na queima de fogos so as emisses de
energia na forma de luz. Essa emisso de energia ocorre quando os eltrons excitados dos ons metlicos,
presentes nos fogos de artifcios, retornam para os nveis de maior energia.


528
17 - (UFLA MG/2008)
Quanta do latim
Plural de quantum
Quando quase no h
Quantidade que se medir
Qualidade que se expressar [...]
Quantum granulado no mel
Quantum ondulado do sal
Gilberto Gil (Quanta)

A msica de Gilberto Gil fala do tomo, das partculas subatmicas e algumas de suas caractersticas.

Segundo a evoluo dos modelos atmicos e os conceitos de estrutura atmica, assinale a alternativa CORRETA.
a) O eltron possui carga negativa (1,602 x 10
19
C) e sua massa to pequena que no pode ser medida.
b) Segundo Planck, a energia s pode ser emitida ou absorvida pelos tomos em pacotinhos. Cada pacotinho contm
uma certa quantidade de energia.
c) Diferentemente dos eltrons e dos prtons, os nutrons no possuem carga e tm massa cerca de 10.000 vezes
maior que a do prton.
d) De acordo com a fsica moderna, a radiao eletromagntica uma partcula e no uma onda.

18 - (FGV SP/2008) As figuras representam alguns experimentos de raios catdicos realizados no incio do sculo
passado, no estudo da estrutura atmica.

O tubo nas figuras (a) e (b) contm um gs submetido alta tenso. Figura (a): antes de ser evacuado. Figura (b): a
baixas presses.
Quando se reduz a presso h surgimento de uma incandescncia, cuja cor depende do gs no tubo. A figura (c)
apresenta a deflexo dos raios catdicos em um campo eltrico.

Em relao aos experimentos e s teorias atmicas, analise as seguintes afirmaes:
I. Na figura (b), fica evidenciado que os raios catdicos se movimentam numa trajetria linear.
II. Na figura (c), verifica-se que os raios catdicos apresentam carga eltrica negativa.
III. Os raios catdicos so constitudos por partculas alfa.
IV. Esses experimentos so aqueles desenvolvidos por Rutherford para propor a sua teoria atmica, conhecido como
modelo de Rutherford.
As afirmativas corretas so aquelas contidas apenas em
a) I, II e III.
b) II, III e IV.
c) I e II.
d) II e IV.
e) IV.

19 - (UFLA MG/2006) O eltron foi descoberto por Thomson no fim do sculo XIX, o que lhe rendeu o prmio Nobel.
Uma caracterstica do modelo atmico proposto por ele :
a) O tomo indivisvel.
b) Os eltrons ocupam orbitais com energias bem definidas.
c) O tomo sofre decaimento radioativo naturalmente.
d) O tomo macio e poderia ser associado a um pudim de passas.

20 - (UFMG/2006) No fim do sculo XIX, Thomson realizou
experimentos em tubos de vidro que continham gases a baixas
presses, em que aplicava uma grande diferena de potencial.
Isso provocava a emisso de raios catdicos. Esses raios,
produzidos num ctodo metlico, deslocavam-se em direo
extremidade do tubo (E).
(Na figura, essa trajetria representada pela linha tracejada X.)

Nesses experimentos, Thomson observou que

529
I) a razo entre a carga e a massa dos raios catdicos era independente da natureza do metal constituinte do ctodo
ou do gs existente no tubo; e
II) os raios catdicos, ao passarem entre duas placas carregadas, com cargas de sinal contrrio, se desviavam na
direo da placa positiva.
(Na figura, esse desvio representado pela linha tracejada Y.)
Considerando-se essas observaes, CORRETO afirmar que os raios catdicos so constitudos de
a) eltrons.
b) nions.
c) prtons.
d) ctions.

21 - (UEPB) O tomo possui inmeras partculas, tais como msons, neutrinos etc., que no tm interesse significativo
para a Qumica. Do ponto de vista quntico, podemos dizer que os tomos so formados apenas por prtons, eltrons
e nutrons. Com base nesta afirmao, assinale a alternativa que contm o nome do descobridor da cada partcula
atmica, respectivamente.
a) Rutherford, Thomson e Chadwick.
b) Thomson, Goldstein e Stoney.
c) Rutherford, Goldstein e Chadwick.
d) Crookes, Rutherford e Goldstein.
e) Goldstein, Chadwick e Stoney.

22 - (Enem MEC/2009) O processo de industrializao tem gerado srios problemas de ordem ambiental, econmica
e social, entre os quais se pode citar a chuva cida. Os cidos usualmente presentes em maiores propores na gua
da chuva so o H
2
CO
3
, formado pela reao do CO
2
atmosfrico com a gua, o HNO
3
, o HNO
2
, o H
2
SO
4
e o H
2
SO
3
.
Esses quatro ltimos so formados principalmente a partir da reao da gua com os xidos de nitrognio e de enxofre
gerados pela queima de combustveis fsseis.

A formao de chuva mais ou menos cida depende no s da concentrao do cido formado, como tambm do tipo
de cido. Essa pode ser uma informao til na elaborao de estratgias para minimizar esse problema ambiental. Se
consideradas concentraes idnticas, quais dos cidos citados no texto conferem maior acidez s guas das chuvas?

a) HNO
3
e HNO
2
.
b) H
2
SO
4
e H
2
SO
3
.
c) H
2
SO
3
e HNO
2
.
d) H
2
SO
4
e HNO
3
.
e) H
2
CO
3
e H
2
SO
3
.


23 - (Enem MEC/2009) Um novo mtodo para produzir insulina artificial que utiliza tecnologia de DNA recombinante
foi desenvolvido por pesquisadores do Departamento de Biologia Celular da Universidade de Braslia (UnB) em
parceria com a iniciativa privada. Os pesquisadores modificaram geneticamente a bactria Escherichia coli para torn-
la capaz de sintetizar o hormnio. O processo permitiu fabricar insulina em maior quantidade e em apenas 30 dias, um
tero do tempo necessrio para obt-la pelo mtodo tradicional, que consiste na extrao do hormnio a partir do
pncreas de animais abatidos.
Cincia Hoje, 24 abr. 2001. Disponvel em:
http://cienciahoje.uol.com.br (adaptado).

A produo de insulina pela tcnica do DNA recombinante tem, como consequncia,

a) o aperfeioamento do processo de extrao de insulina a partir do pncreas suno.
b) a seleo de microrganismos resistentes a antibiticos.
c) o progresso na tcnica da sntese qumica de hormnios.
d) impacto favorvel na sade de indivduos diabticos.
e) a criao de animais transgnicos.

24 - (Enem MEC/2009) A economia moderna depende da disponibilidade de muita energia em diferentes formas, para
funcionar e crescer. No Brasil, o consumo total de energia pelas indstrias cresceu mais de quatro vezes no perodo
entre 1970 e 2005. Enquanto os investimentos em energias limpas e renovveis, como solar e elica, ainda so
incipientes, ao se avaliar a possibilidade de instalao de usinas geradoras de energia eltrica, diversos fatores devem
ser levados em considerao, tais como os impactos causados ao ambiente e s populaes locais.
RICARDO, B.; CAMPANILI, M. Almanaque Brasil Socioambiental.
So Paulo: Instituto Socioambiental, 2007 (adaptado).

Em uma situao hipottica, optou-se por construir uma usina hidreltrica em regio que abrange diversas quedas

530
dgua em rios cercados por mata, alegando-se que causaria impacto ambiental muito menor que uma usina
termeltrica. Entre os possveis impactos da instalao de uma usina hidreltrica nessa regio, inclui-se

a) a poluio da gua por metais da usina.
b) a destruio do habitat de animais terrestres.
c) o aumento expressivo na liberao de CO
2
para a atmosfera.
d) o consumo no renovvel de toda gua que passa pelas turbinas.
e) o aprofundamento no leito do rio, com a menor deposio de resduos no trecho de rio anterior represa.

TEXTO: 1 - Comum questo: 25

Essa Maria Fumaa devagar quase parada
seu foguista, bota fogo na fogueira
Que essa chaleira tem que estar at sexta-feira
Na estao de Pedro Osrio, sim senhor
Se esse trem no chega a tempo
vou perder meu casamento
Atraca, atraca-lhe carvo nessa lareira
Esse fogo que acelera essa banheira...
(KLEITON e KLEDIR. Maria Fumaa. Disponvel em: <http://letras.terra.com.br>. Acesso em: 15 set. 2009.)



Figura 2: MONET. Le train dans la neige. 1875. (Disponvel em: http://www.railart.co.uk/images/monet.jpg. Acesso: 22
maio 2009.)

25 - (UEL PR/2010) Em relao aos conhecimentos sobre transformaes fsicas e qumicas, correto afirmar:

a) Na vaporizao so rompidas ligaes intermoleculares, e na atomizao so rompidas ligaes intramoleculares.
b) A fogueira, a lareira e o fogo remetem a uma caldeira para gerar vapor atravs da troca qumica entre
combustvel e gua.
c) A chaleira representa o dispositivo da Maria Fumaa que ir transformar gua lquida em vapor por processo
exotrmico.
d) O carvo na lareira sofre reduo e libera os gases metano e oxignio.
e) A energia necessria para vaporizar 1 mol de gua lquida igual energia necessria para transformar a mesma
quantidade de gua em tomos isolados.











531








PROF.:Bencio Dias Honorio



C
C
o
o
n
n
t
t
e
e

d
d
o
o
s
s
:
:

- Distribuio eletrnica
Tabela peridica
- Classificao dos elementos
- Propriedades dos elementos
- Propriedades peridicas e aperidicas


M
M
i
i
n
n
i
i
s
s
t
t
r
r
a
a
d
d
o
o
s
s
d
d
u
u
r
r
a
a
n
n
t
t
e
e
o
o
m
m

s
s
d
d
e
e
a
a
b
b
r
r
i
i
l
l















D Da at ta a d da a E En nt tr re eg ga a : : _ __ __ __ __ __ __ __ _/ /_ __ __ __ __ __ __ __ __ __ __ _/ /2 20 01 13 3

LISTA 3 QUMICA

532


01 - (UFC CE/2001) Clcio metlico reage com hidrognio gasoso, em elevadas temperaturas, para formar o hidreto
de clcio.
a) Descreva a equao qumica balanceada para este processo.
b) Descreva as configuraes eletrnicas dos elementos, tal como encontram-se no produto da reao.

02 - (UFPEL RS/2003) A srie sobre Harry Potter trouxe para as telas do cinema o simptico bruxinho, campeo de
vendas nas livrarias. Criticado por alguns e amado por muitos outros, Harry Potter traz tona temas como bruxaria e
alquimia. Essas duas crenas, ou pseudo-cincias, foram e ainda so ridicularizadas pelos cientistas, mas graas a
bruxos, bruxas e alquimistas que a qumica nasceu e deu os primeiros passos, afirmando-se como cincia. Muitos
conceitos bsicos da qumica, como energia das reaes, isotopia, classificao peridica e modelos atmicos foram
alicerados pelos trabalhos e observaes desses cientistas ou, como queiram, bruxos annimos.
Sobre os conceitos fundamentais da qumica, diferencie os conceitos de isotopia e isotonia.

03 - (UEG GO/2008) Istopos so tomos do mesmo elemento qumico que apresentam as mesmas propriedades
qumicas e diferentes propriedades fsicas. Para a caracterizao de um tomo necessrio conhecer o seu nmero
atmico e o seu nmero de massa. Sobre esse assunto, considere os elementos qumicos hipotticos
(a + 7)
X
(3a)
e
(2a +
2)
Y
(3a + 2)
. Sabendo-se que esses elementos so istopos entre si, responda ao que se pede.
a) Calcule a massa atmica e o nmero atmico para cada um dos elementos qumicos X e Y.
b) Obtenha, em subnveis de energia, a distribuio eletrnica do on X
2+
.
c) O on X
2+
dever apresentar maior ou menor raio atmico do que o elemento X? Explique.

04 - (UEG GO/2006) Os nmeros qunticos so utilizados para caracterizar cada eltron da eletrosfera de um tomo.
Analise o diagrama abaixo e determine os quatro nmeros qunticos dos eltrons A e B. Considere
+ 2 / 1
e
| + 2 / 1
.



05 - (UFPEL RS/2003) A srie sobre Harry Potter trouxe para as telas do cinema o simptico bruxinho, campeo de
vendas nas livrarias. Criticado por alguns e amado por muitos outros, Harry Potter traz tona temas como bruxaria e
alquimia. Essas duas crenas, ou pseudo-cincias, foram e ainda so ridicularizadas pelos cientistas, mas graas a
bruxos, bruxas e alquimistas que a qumica nasceu e deu os primeiros passos, afirmando-se como cincia. Muitos
conceitos bsicos da qumica, como energia das reaes, isotopia, classificao peridica e modelos atmicos foram
alicerados pelos trabalhos e observaes desses cientistas ou, como queiram, bruxos annimos.
Sobre os conceitos fundamentais da qumica, cite os nmeros qunticos principal e secundrio do eltron de
valncia do tomo neutro de enxofre.

06 - (UEPG PR/2009) Com relao configurao eletrnica de um tomo de sdio que apresenta
Na
23
11
,
convencionando-se que para o primeiro eltron de um orbital
2 / 1 S = |
, assinale o que for correto.

01. Um eltron no orbital 2s ter o mesmo valor de energia que um eltron no orbital 2p.
02. A configurao eletrnica no estado fundamental : 1s
2
2s
2
2p
6
3s
1
.
04. A configurao eletrnica 1s
2
2s
2
2p
6
6s
1
representa a configurao de um tomo de Na no estado excitado.
08. O Na apresenta 11 prtons, enquanto Na
+
apresenta 12 prtons.
Caderno de Atividades

Disciplina:
Qumica

Professor(a):
Bencio
Aluno:
3 ano
Ensino Mdio
Data de Recebimento:
_____/_____/_____
Lista 03

Data Entrega:
_____/_____/_____

533
07 - (CEFET PR/2009) Analise as afirmativas e verifique se so verdadeiras (V) ou falsas (F) e assinale a alternativa
que indica a seqncia correta de cima para baixo.

( ) O on
8
O
2
isoeletrnico com o on
16
S
1
.
( ) O on F
2
possui o mesmo nmero de prtons que o on F
1
.
( ) a distribuio eletrnica do on
12
Mg
2+
igual distribuio eletrnica do on
11
Na
1+
.
( ) comparando o tomo de
Ba
137
56
com o
Cs
137
55
, pode-se afirmar que so istonos.
( ) A distribuio eletrnica no nvel de valncia do
18
Ar 3s
2
3p
6
.
( ) Os nmeros qunticos do eltron mais energtico dos ons
9
F
1
e do on
8
O
1
so iguais.

a) F F F V V F
b) V F F V V F
c) V F V V F V
d) F V V F V F
e) F V F V V F

08 - (UESC BA/2009) A cor das pedras preciosas decorre da presena de ons na estrutura qumica dessas gemas. O
rubi tem a cor vermelha em razo da presena de ons Cr
3+
, e as safiras apresentam cores variadas atribudas aos ons
Co
2+
e Fe
2+
, dentre outros. A distribuio eletrnica desses ons obedece ao princpio da energia mnima. Assim, os
eltrons so colocados nos subnveis de menor energia no tomo.

Considerando-se a tendncia energia mnima na distribuio eletrnica, correto afirmar:

01. A distribuio eletrnica do on Cr
3+
representada por [Ar] 4s
1
3d
2
.
02. Os eltrons de maior energia no on Co
2+
apresentam a configurao eletrnica 4s
2
.
03. A configurao eletrnica do on Fe
2+
representada por [Ar] 3d
6
4s
2
.
04. A configurao eletrnica do on Co
2+
representada por [Ar] 3d
7
.
05. A configurao eletrnica da camada de valncia do tomo de ferro representada por 3d
8
.

09 - (UFG GO/2010) As relaes entre os produtos e os reagentes da reao qumica CO + H
2
O

H
2
+ CO
2

podem ser descritas como relaes de conjuntos matemticos. Considere o conjunto domnio CO, formado pelos
istopos
12
C e
13
C, e o conjunto imagem formado pelo produto CO
2
.

Desse modo, com base na relao entre o domnio e a imagem, pode-se concluir que

a) cada elemento do domnio corresponde a um elemento da imagem.
b) cada elemento da imagem corresponde a dois elementos do domnio.
c) cada elemento do domnio corresponde a dois elementos da imagem.
d) cada elemento da imagem corresponde a trs elementos do domnio.
e) cada elemento do domnio corresponde a trs elementos da imagem.

10 - (UFAC/2010) A energia gerada pelas usinas de energia nuclear produzida pela fisso nuclear. O combustvel
nuclear deve ser uma substncia de natureza fssil como o
235
U. Na natureza, o urnio encontrado na composio de
99,284% do istopo
238
U e 0,711% do istopo
235
U. Para ser usado como combustvel nuclear, o urnio submetido a
um processo de enriquecimento, a fim de concentrar na mistura de istopos o teor do istopo fssil.

Com relao aos istopos
235
U e
238
U, incorreto afirmar que os mesmos possuem:
a) o mesmo nmero de prtons.
b) diferentes nmeros de massa.
c) diferentes nmeros de eltrons.
d) o mesmo nmero de eltrons.
e) diferentes nmeros de nutrons.

11 - (UFRR/2010) A configurao eletrnica do tomo de oxignio no estado fundamental 1s
2
2s
2
2p
4
. Considerando
o ltimo subnvel dessa configurao eletrnica assinale a nica alternativa que contm os valores dos nmeros
qunticos principal, secundrio, magntico e de spin para o ltimo eltron deste subnvel.

a) 2, 0, +1 e +1/2.
b) 2, 1, -1 e +1/2.
c) 4, 2, +1 e -1/2
d) 4, 0, -1 e +1/2.
e) 0, -1, +1/2, 2.


534
12 - (UEPG PR/2009)
Os nmeros qunticos n, l, m, denominados, respectivamente, principal, secundrio e magntico, correspondem
descrio ondulatria de um eltron num tomo. A respeito destes nmeros qunticos, assinale o que for correto.

01. Quando n = 2, os valores de l podem ser 0 e 1.
02. Quando l = 2, o subnvel d.
04. Quando l = 1, os valores de m podem ser 1, 0 e +1 e o subnvel p.
08. Quando um subnvel f, so 7 os valores de m e existem 7 orbitais no subnvel.
16. Quando um subnvel s, o valor de l 0 e o valor de m 1.

13 - (UEG GO/2010)
A posio dos elementos qumicos na tabela peridica est associada s suas respectivas distribuies eletrnicas.
Por exemplo, o clcio pertence famlia dos metais alcalinos terrosos e pode gerar um on bivalente.
Considerando essas duas espcies qumicas,

a) Faa a distribuio eletrnica em subnveis de energia do on Ca
2+
;
b) Explique qual delas apresenta o maior raio atmico.

14 - (UFRJ/2008) O livro A Tabela Peridica, de Primo Levi, rene relatos autobiogrficos e contos que tm a
qumica como denominador comum. Cada um de seus 21 captulos recebeu o nome de um dos seguintes elementos
da tabela peridica: Argnio, Hidrognio, Zinco, Ferro, Potssio, Nquel, Chumbo, Mercrio, Fsforo, Ouro, Crio,
Cromo, Enxofre, Titnio, Arsnio, Nitrognio, Estanho, Urnio, Prata, Vandio, Carbono.

Escreva o smbolo do elemento que d nome a um captulo e corresponde a cada uma das seis descries a seguir.
I metal alcalino.
II lquido na temperatura ambiente.
III o de menor potencial de ionizao do grupo 15.
IV radioativo, usado em usinas nucleares.
V Aparece na natureza na forma de gs monoatmico.
VI lantandeo.

15 - (UFRRJ/2007) Um homem de 70 kg poder apresentar, aproximadamente, 2,8 kg de sais minerais em seu
organismo. Abaixo esto alguns minerais e algumas de suas funes no corpo humano.
- Magnsio: ativa as enzimas que participam na sntese das protenas.
- Zinco: componente das enzimas que participam na digesto.
- Cobre: componente das enzimas associadas ao metabolismo do ferro.
- Potssio: transmisso de impulso.
- Clcio: formao dos ossos e dentes.
- Ferro: compe a hemoglobina e as enzimas que atuam no metabolismo energtico.
(Marta Pires, Interatividade Qumica. Volume nico, 2003 FTD)
Utilizando a Tabela Peridica , responda:
a) Faa a distribuio eletrnica da espcie inica ferro III.
b) Comparando os raios do cobre metlico e do on cobre I, qual raio apresenta menor tamanho? Justifique.

16 - (UEG GO/2007) Na tabela abaixo, os elementos qumicos so representados pelas letras A, B, C, D e E. Analise-
a e responda ao que se pede.



a) Escreva as frmulas do xido e do cloreto formados pelo elemento A.
b) Quais elementos presentes na tabela apresentam maior raio atmico e maior potencial de ionizao
respectivamente? Explique.




535
17 - (UFRJ/2007) Considere as espcies qumicas representadas no quadro a seguir.

+ + +
Cl Al Ca Fe Ar S
3 2 3 2


a) Identifique, com o auxlio da Tabela Peridica, as espcies isoeletrnicas, apresentando-as em ordem decrescente
de raio.
b) Identifique, dentre as espcies qumicas cujos elementos pertencem ao terceiro perodo, aquela que apresenta o
menor potencial de ionizao. Justifique sua resposta.

18 - (UFOP MG/2007) Os seguintes grficos mostram como certas propriedades variam atravs de um perodo ou ao
longo de um grupo na tabela peridica.



Dentre os grficos A, B e C, acima, selecione aquele que melhor representa a tendncia em cada uma das seguintes
propriedades:

I) Raio atmico de Li, Na, K, Rb.
II) Nmero de eltrons em F

, Ne, Na
+
, Mg
2+
.
III) Primeira energia de ionizao de Li, Na, K, Rb.
IV) Eletronegatividade de C, N, O, F.
V) Raio inico de S
2
, Cl

, K
+
, Ca
2+
.

19 - (UFF RJ/2010) Existem metais que, na forma de compostos, do coloraes caractersticas chama azul do bico
de Bunsen. Essa propriedade usada em laboratrios no reconhecimento de metais. Com o calor da chama do bico de
Bunsen, os eltrons dos ons metlicos absorvem energia e saltam para nveis mais externos e, ao retornarem para os
nveis internos, emitem radiaes coloridas tpicas de cada metal.

Observe a figura abaixo e assinale a opo correta.


a) Os elementos Li, Na, Cu, Sr, Ca e Ba, nessa ordem, esto colocados em ordem crescente de energia de ionizao.
b) A configurao eletrnica do Cobre : [Ar] 4s
2
3d
10
.
c) Elementos Ba, Sr e Ca pertencem ao grupo dos alcalinos terrosos.
d) O elemento de transio interna o Cu, j que seu subnvel de maior energia o f.
e) Os elementos Li, Cu, Ba, Sr, Na e Ca, nessa ordem, esto colocados em ordem crescente de raio atmico.

20 - (UFC CE/2010) O raio atmico (ou inico) uma propriedade peridica que exerce grande influncia na
reatividade dos tomos (ou dos ons).
a) Explique, em termos de carga nuclear efetiva, a variao apresentada pelo raio atmico (ou inico) dentro de um
mesmo perodo da Tabela Peridica.
b) Considere os seguintes pares de espcies: i) Al
+
e Al
2+
; ii) F e F
-
e iii) Li e Li
+
. Indique, para cada par, a espcie que
apresenta o maior raio.





536
21 - (Enem MEC/2009) Uma pesquisadora deseja reflorestar uma rea de mata ciliar quase que totalmente
desmatada. Essa formao vegetal um tipo de floresta muito comum nas margens de rios dos cerrados no Brasil
central e, em seu clmax, possui vegetao arbrea perene e apresenta dossel fechado, com pouca incidncia
luminosa no solo e nas plntulas. Sabe-se que a incidncia de luz, a disponibilidade de nutrientes e a umidade do solo
so os principais fatores do meio ambiente fsico que influenciam no desenvolvimento da planta. Para testar
unicamente os efeitos da variao de luz, a pesquisadora analisou, em casas de vegetao com condies
controladas, o desenvolvimento de plantas de 10 espcies nativas da regio desmatada sob quatro condies de
luminosidade: uma sob sol pleno e as demais em diferentes nveis de sombreamento. Para cada tratamento
experimental, a pesquisadora relatou se o desenvolvimento da planta foi bom, razovel ou ruim, de acordo com
critrios especficos. Os resultados obtidos foram os seguintes:

Para o reflorestamento da regio desmatada,
a) a espcie 8 mais indicada que a 1, uma vez que aquela possui
melhor adaptao a regies com maior incidncia de luz.
b) recomenda-se a utilizao de espcies pioneiras, isto , aquelas que
suportam alta incidncia de luz, como as espcies 2, 3 e 5.
c) sugere-se o uso de espcies exticas, pois somente essas podem
suportar a alta incidncia luminosa caracterstica de regies desmatadas.
d) espcies de comunidade clmax, como as 4 e 7, so as mais indicadas,
uma vez que possuem boa capacidade de aclimatao a diferentes
ambientes.
e) recomendado o uso de espcies com melhor desenvolvimento
sombra, como as plantas das espcies 4, 6, 7, 9 e 10, pois essa
floresta, mesmo no estgio de degradao referido, possui dossel fechado, o que impede a entrada de luz.

22 - (Enem MEC/2009) As mudanas climticas e da vegetao ocorridas nos trpicos da Amrica do Sul tm sido
bem documentadas por diversos autores, existindo um grande acmulo de evidncias geolgicas ou
paleoclimatolgicas que evidenciam essas mudanas ocorridas durante o Quaternrio nessa regio. Essas mudanas
resultaram em restrio da distribuio das florestas pluviais, com expanses concomitantes de habitats no-florestais
durante perodos ridos (glaciais), seguido da expanso das florestas pluviais e restrio das reas no-florestais
durante perodos midos (interglaciais).
Disponvel em: http://zoo.bio.ufpr.br. Acesso em: 1 maio 2009.
Durante os perodos glaciais,
a) as reas no-florestais ficam restritas a refgios ecolgicos devido baixa adaptabilidade de espcies no-
florestais a ambientes ridos.
b) grande parte da diversidade de espcies vegetais reduzida, uma vez que necessitam de condies semelhantes
a dos perodos interglaciais.
c) a vegetao comum ao cerrado deve ter se limitado a uma pequena regio do centro do Brasil, da qual se
expandiu at atingir a atual distribuio.
d) plantas com adaptaes ao clima rido, como o desenvolvimento de estruturas que reduzem a perda de gua,
devem apresentar maior rea de distribuio.
e) florestas tropicais como a amaznica apresentam distribuio geogrfica mais ampla, uma vez que so densas e
diminuem a ao da radiao solar sobre o solo e reduzem os efeitos da aridez.

23 - (Enem MEC/2009) Os ncleos dos tomos so constitudos de prtons e nutrons, sendo ambos os principais
responsveis pela sua massa. Nota-se que, na maioria dos ncleos, essas partculas no esto presentes na mesma
proporo. O grfico mostra a quantidade de nutrons (N) em funo da quantidade de prtons (Z) para os ncleos
estveis conhecidos.

KAPLAN, I. Fsica Nuclear. Rio de Janeiro:
Guanabara Dois, 1978 (adaptado).

O antimnio um elemento qumico que possui 50
prtons e possui vrios istopos tomos que s se
diferem pelo nmero de nutrons. De acordo com o
grfico, os istopos estveis do antimnio possuem

a) entre 12 e 24 nutrons a menos que o nmero de
prtons.
b) exatamente o mesmo nmero de prtons e nutrons.
c) entre 0 e 12 nutrons a mais que o nmero de prtons.
d) entre 12 e 24 nutrons a mais que o nmero de prtons.
e) entre 0 e 12 nutrons a menos que o nmero de
prtons.

537

TEXTO: 1 - Comum questo: 24

No poema Confidncia do Itabirano de Carlos Drummond de Andrade, possvel identificar a relao que o poeta
estabelece entre seus sentimentos e a propriedade do metal mais produzido no mundo, o ferro.

Alguns anos vivi em Itabira.
Principalmente nasci em Itabira.
Por isso sou triste, orgulhoso: de ferro.
Noventa por cento de ferro nas caladas.
Oitenta por cento de ferro nas almas.
E esse alheamento do que na vida
porosidade e comunicao.
........
De Itabira trouxe prendas diversas que ora
te ofereo;
Este So Benedito do velho santeiro
Alfredo Durval;
Esta pedra de ferro, futuro ao do Brasil;
Este couro de anta, estendido no sof da
sala de visitas;
Este orgulho, esta cabea baixa....

24 - (UFRRJ/2006) Esta pedra de ferro, futuro ao do Brasil. O ao uma liga metlica constituda de Fe, Cr, Ni e
C. Coloque os metais que compem esta liga em ordem crescente de raio atmico.

TEXTO: 2 - Comum questo: 25


O solo torna-se cido principalmente devido presena dos ctions: hidrognio monovalente e alumnio trivalente, que
na presena de gua forma hidrxido de alumnio e on hidrognio positivo. Essa acidez neutralizada com carbonato
de clcio, formando dixido de carbono, ons, clcio livre e gua.

25 - (UDESC SC/2006)
b) Qual maior: o alumnio ou seu on? Justifique sua resposta.
Dado: Nmero atmico do alumnio 13.
.





















538






PROF.:Bencio Dias Honorio


C
C
o
o
n
n
t
t
e
e

d
d
o
o
s
s
:
:

- Ligaes inicas, covalentes e metlica
- Polaridade
-Geometria molecular
- Foras Intermoleculares



M
M
i
i
n
n
i
i
s
s
t
t
r
r
a
a
d
d
o
o
s
s
d
d
u
u
r
r
a
a
n
n
t
t
e
e
o
o
m
m

s
s
d
d
e
e
m
m
a
a
i
i
o
o














D Da at ta a d da a E En nt tr re eg ga a : : _ __ __ __ __ __ __ __ _/ /_ __ __ __ __ __ __ __ __ __ __ _/ /2 20 01 13 3
LISTA 4 QUMICA

539

01 - (UFU MG/2010) Algumas pessoas usam desodorantes para disfarar os odores das axilas, mas, se suam muito,
com certeza necessitam de um desodorante antiperspirante (antitranspirante) para diminuir a produo do suor, que
aumenta quando nos expomos ao calor, esforo fsico, estresse ou nervosismo.
Ingredientes como cera, emoliente lquido e um ingrediente ativo so encontrados nos antiperspirantes. o
ingrediente ativo que faz os antiperspirantes bloquearem o suor. Na maioria dos antiperspirantes, o ingrediente ativo
um composto base de alumnio, e o mais comum o cloridrxido de alumnio, cuja formula emprica
Al
2
(OH)
5
Cl.2H
2
O. O cloreto de alumnio tambm pode ser empregado como princpio ativo.
Os antiperspirantes de venda comercial livre podem ter uma concentrao de ingrediente ativo mxima de 25
% massa/massa e so comercializados, em geral, em embalagens de 50 g. Para as pessoas que apresentam
transpirao excessiva nas axilas, existem produtos vendidos sob prescrio mdica que contm concentraes mais
altas que os antiperspirantes de venda comercial livre.
Considerando as informaes acima, faa o que se pede.

a) Defina o tipo de interao qumica que ocorre entre os tomos da substncia cloreto de alumnio.
b) Para o princpio ativo cloreto de alumnio, apresente a frmula mnima e sua estrutura conforme representao de
Lewis.
c) Quantas gramas de alumnio so encontradas em uma embalagem de um desodorante antiperspirante que contenha
como princpio ativo cloridrato de alumnio, Al
2
(OH)
5
Cl.2H
2
O ?

02 - (UERJ/2010) A anlise da Classificao Peridica dos Elementos permite ao estudante fazer analogias entre
tomos, ons e molculas.
Considere as seguintes espcies qumicas:
+
4
NH
, NH
3
, O
2
, N
2
H
4
, Cl


Dentre essas espcies, identifique os ons isoeletrnicos. Em seguida, apresente a frmula estrutural plana do on
formado por um elemento qumico do terceiro perodo da Classificao Perodica dos Elementos com estrutura idntica
do amnio.

03 - (UEG GO/2007) Dadas as configuraes eletrnicas fundamentais de trs tomos neutros, responda ao que se
pede.

A = 1s
2
2s
2
2p
6

B = 1s
2
2s
2
2p
6
3s
1

C = 1s
2
2s
2
2p
6
3s
2
3p
5


a) Qual apresenta maior energia de ionizao? Explique.
b) Qual a frmula resultante da combinao de A e B? O composto formado de natureza inica ou molecular?
Explique.

04 - (UERJ/2007) Um laboratrio recebe trs amostras para anlise. A tabela abaixo descreve algumas de suas
principais caractersticas.

Trs elementos qumicos fazem parte da
constituio das amostras; no entanto, cada
uma composta por apenas dois deles. Os
tomos desses trs elementos, no estado
fundamental, possuem 2, 3 e 7 eltrons de
valncia situados na terceira camada
eletrnica.

Explique a alta condutividade eltrica da
amostra I, a partir de sua composio
qumica, e indique as frmulas das
substncias presentes nas amostras II e III.


Caderno de Atividades

Disciplina:
Qumica

Professor(a):
Bencio
Aluno:
3 ano
Ensino Mdio
Data de Recebimento:
_____/_____/_____
Lista 04

Data Entrega:
_____/_____/_____
C 1412 C 714 baixa muito branco p III
- C 194 baixa miuto branco p II
- - alta slido I
ebulio
de ponto
fuso
de ponto
ambiente
a temperatur
eltrica
ade condutivid
material
do ecto asp
Amostra

540
05 - (UFOP MG/2006)
Ao se analisar um slido desconhecido, observa-se que ele apresenta as seguintes propriedades:

macio nte relativame IV
eltrica corrente conduz No III
gua em insolvel te Praticamen II
baixa fuso de a Temperatur I


a) Indique o tipo de ligao esperado entre as partculas dessa substncia.
b) Com base no tipo de ligao esperado entre as partculas, explique por que o slido desconhecido apresenta
temperatura de fuso baixa.
c) Explique por que as partculas dessa substncia no conduzem eletricidade.

06 - (UFRJ/2007)
QUANTA (Gilberto Gil)
Fragmento infinitsimo
Quase apenas mental
Quantum granulado no mel
Quantum ondulado do sal
Mel de urnio, sal de rdio
Qualquer coisa quase ideal
Com base na Tabela Peridica fornecida no final da prova, escreva a frmula do sal formado pelo halognio mais
eletronegativo e o metal alcalino terroso citado por Gilberto Gil na letra de Quanta, indicando o tipo de ligao qumica
do sal formado.

07 - (UNESP SP/2006) Uma ligao qumica forma-se entre dois tomos, iguais ou diferentes, quando o arranjo
resultante de seus ncleos e eltrons tem energia mais baixa do que quando os tomos estavam separados. Na
ligao inica, ocorre a transferncia de um ou mais eltrons de um tomo para outro, formando-se ons, que se
atraem e passam a formar um retculo cristalino.
a) Deduza a frmula do composto inico formado entre Ca (Z = 20) e P (Z = 15).
b) Explique, com justificativas, por que as substncias inicas so duras e quebradias e possuem elevados pontos de
fuso.

08 - (UDESC SC/2006) Considere o elemento cloro formando compostos com, respectivamente, hidrognio, carbono,
potssio e alumnio.
a) Com quais desses elementos o cloro forma compostos covalentes? Justifique sua resposta.
b) Indique as frmulas estruturais e moleculares dos compostos covalentes do cloro.

09 - (UNICAMP SP/2009) Fogos de artifcio foram utilizados na abertura e no encerramento da Olimpada de Beijing.
Um dos principais efeitos visuais desses fogos a cor emitida. Freqentemente, a substncia responsvel pela
colorao um slido inico contendo um on de metal alcalino ou alcalino terroso. O sal, a partir da exploso, recebe
energia e sofre vrias transformaes. Inicialmente o sal passa para o estado gasoso, com a posterior separao dos
ons.
Depois, esses ons no estado gasoso se transformam em espcies neutras, sendo as espcies neutras provenientes
dos ctions as responsveis pelo efeito visual.

a) Equacione a seqncia de transformaes que o cloreto de brio sofreria em fogos de artifcio, conforme descrito em
itlico no texto.
b) Observaram-se vrias cores na queima de fogos na abertura dos Jogos Olmpicos, entre elas a alaranjada (mistura
de amarelo e vermelho). Suponha que algum explicasse que essa cor foi obtida pelo uso do composto inico Na
2
Sr.
De acordo com o conhecimento qumico e as informaes dadas, essa explicao seria correta ou no? Justifique.
Dados:





541
10 - (UFCG PB/2009) A capacidade que um tomo tem de atrair eltrons de outro tomo, quando os dois formam uma
ligao qumica, denominada eletronegatividade. Esta uma das propriedades qumicas consideradas no estudo da
polaridade das ligaes.
Considerando os trs compostos: SiCl
4
, AsH
3
e SeH
2
, responda as perguntas abaixo:

a) Demonstre o nmero de ligaes apolares nos trs compostos?
b) Demonstre os pares eletrnicos livres existem nos trs compostos?
c) Quais so os compostos apolares e quais so os polares? Justifique.


11 - (UFRN/2006) A atmosfera terrestre uma mistura de gases. Uns presentes naturalmente nela e outros
resultantes da ao do homem. Alguns gases, tais como N
2
O, SF
6
, CH
4
, CO
2
e H
2
O, absorvem radiao infravermelha,
acentuando o efeito estufa, que tem como conseqncia o aquecimento global do planeta.
Em relao s molculas que formam esses gases do efeito estufa, atenda s seguintes solicitaes.
a) Apresente a estrutura de Lewis dessas molculas e indique a que no segue a regra do octeto.
b) Indique as que so polares. Justifique.

12 - (IME RJ/2001) Analise as afirmativas abaixo e indique se as mesmas so falsas ou verdadeiras, justificando cada
caso.
a) Slidos inicos so bons condutores de eletricidade.
b) Compostos apolares so solveis em gua.
c) Caso no sofresse hibridizao, o boro formaria a molcula BF.
d) A estrutura geomtrica da molcula de hexafluoreto de enxofre tetradrica.

13 - (UEG GO/2010) Considere os ons abaixo e responda ao que se pede.

3
O C
;
+
4
NH
;

SCN

a) Desenhe as suas estruturas de Lewis.
b) Determine as suas geometrias moleculares.

14 - (UFG GO/2009) A teoria da repulso por pares de eltrons da camada de valncia (VSEPR) um modelo para
previso da estrutura tridimensional das molculas. Considere as molculas de NH
3
e de H
2
O.
a) Determine suas geometrias moleculares, considerando os pares de eltrons no-ligantes.
b) Estime os ngulos de ligao dos pares de eltrons ligantes e justifique sua resposta.

15 - (UFRRJ/2008) Conferncia confirma que Pluto deixa de ser planeta....
Publicidade. Folha On-line, agosto, 2006.
Disponvel em http://www1.folha.uol.com.br/folha/ciencia/
ult306u15073.shtml . Acesso em 19/08/2007.

Pluto, descoberto em 1930, foi considerado, durante um longo tempo, como um planeta do Sistema Solar. Entretanto,
a Unio Astronmica Internacional, em sua 26 Assemblia Geral, realizada em Praga, no ano passado, excluiu Pluto
dessa categoria. Considera-se um planeta aquele que tem massa suficiente para ficar isolado em sua rbita, o que no
o caso de Pluto, que possui, em torno da sua rbita, vrios outros corpos.
A atmosfera de Pluto composta por nitrognio, metano e monxido de carbono. Em relao s estruturas
moleculares destes gases, atenda s seguintes solicitaes:
a) Represente a frmula eletrnica (frmula de Lewis) da molcula de maior carter polar.
b) Represente a frmula estrutural plana das molculas apolares, indicando as respectivas geometrias.

16 - (UEG GO/2007) As bexigas de forma ovide, apresentadas na figura abaixo, representam nuvens eletrnicas
associadas a ligaes simples, duplas ou triplas entre tomos. Levando-se em considerao os compostos BeH
2
, H
2
O,
BF
3
, CH
4
, NaCl e BaSO
4
, responda aos itens abaixo:

a) Associe, quando possvel, os compostos s figuras representadas pelas bexigas.
b) Entre as espcies CH
4
e H
2
O, qual apresenta menor ngulo de ligao? Explique.




542
17 - (IME RJ/2006) Um composto de frmula molecular AB
5
constitudo por elementos que pertencem ao mesmo
perodo de um determinado gs nobre. Tal gs nobre apresenta a mesma distribuio eletrnica que um on de um
dado nucldeo X. Sabe-se ainda que o nucldeo X contm 21 prtons, 21 eltrons e 24 nutrons.
O elemento A no-metlico e no pertence ao grupo dos calcognios. Nas CNTP, A encontra-se no estado slido e
B existe como molcula diatmica.
Responda e justifique:
a) a que perodo os elementos A e B pertencem?
b) qual a carga do on do ncldeo X?
c) o composto AB
5
covalente ou inico?
d) os elementos A e B pertencem a quais grupos ou famlias?
e) qual o nome do composto AB
5
?
f) qual a forma geomtrica do composto AB
5
, considerando o modelo de repulso dos pares de eltrons da camada
de valncia?
g) quais so os orbitais hbridos necessrios ao elemento A para acomodar os pares de eltrons no arranjo
geomtrico do item anterior?

18 - (UEG GO/2009) A hidrazina (NH
2
NH
2
), o perxido de hidrognio (H
2
O
2
) e a gua apresentam tenso superficial
excepcionalmente altas em comparao com outras substncias de massas moleculares semelhantes. Nesse
contexto, responda ao que se pede.
a) Desenhe as estruturas de Lewis para os trs compostos.
b) Descreva o motivo do comportamento dessas substncias.

19 - (UEG GO/2007) O eixo y da figura abaixo representa as temperaturas de ebulio de compostos dos elementos
das famlias 14 e 16 da tabela peridica. No eixo x tem-se os valores das massas molares. Levando-se em
considerao o grfico a seguir, responda aos itens abaixo:

a) Explique o comportamento observado para os pontos de ebulio nos compostos da famlia do carbono.
b) Explique por que a gua apresenta ponto de ebulio superior ao dos demais compostos do grupo do oxignio e por
que essa discrepncia no ocorre com os compostos da famlia do carbono.

20 - (UFC CE/2007) As foras intermoleculares so responsveis por vrias propriedades fsicas e qumicas das
molculas, como, por exemplo, a temperatura de fuso. Considere as molculas de F
2
, Cl
2
e Br
2
.
a) Quais as principais foras intermoleculares presentes nessas espcies?
b) Ordene essas espcies em ordem crescente de temperatura de fuso.

21 - (Enem MEC/2009) A fotossntese importante para a vida na Terra.
Nos cloroplastos dos organismos fotossintetizantes, a energia solar convertida em energia qumica que, juntamente
com gua e gs carbnico (CO
2
), utilizada para a sntese de compostos orgnicos (carboidratos). A fotossntese o
nico processo de importncia biolgica capaz de realizar essa converso. Todos os organismos, incluindo os
produtores, aproveitam a energia armazenada nos carboidratos para impulsionar os processos celulares, liberando
CO
2
para a atmosfera e gua para a clula por meio da respirao celular. Alm disso, grande frao dos recursos
energticos do planeta, produzidos tanto no presente (biomassa) como em tempos remotos (combustvel fssil),
resultante da atividade fotossinttica.

As informaes sobre obteno e transformao dos recursos naturais por meio dos processos vitais de fotossntese e
respirao, descritas no texto, permitem concluir que
a) o CO
2
e a gua so molculas de alto teor energtico.
b) os carboidratos convertem energia solar em energia qumica.
c) a vida na Terra depende, em ltima anlise, da energia proveniente do Sol.
d) o processo respiratrio responsvel pela retirada de carbono da atmosfera.
e) a produo de biomassa e de combustvel fssil, por si, responsvel pelo aumento de CO
2
atmosfrico.

543

22 - (Enem MEC/2009) O esquema mostra um diagrama de bloco de uma estao geradora de eletricidade
abastecida por combustvel fssil.


HINRICHS, R. A.; KLEINBACH, M. Energia e meio ambiente.
So Paulo: Pioneira Thomson Learning, 2003 (adaptado).

Se fosse necessrio melhorar o rendimento dessa usina, que forneceria eletricidade para abastecer uma cidade, qual
das seguintes aes poderia resultar em alguma economia de energia, sem afetar a capacidade de gerao da usina?

a) Reduzir a quantidade de combustvel fornecido usina para ser queimado.
b) Reduzir o volume de gua do lago que circula no condensador de vapor.
c) Reduzir o tamanho da bomba usada para devolver a gua lquida caldeira.
d) Melhorar a capacidade dos dutos com vapor conduzirem calor para o ambiente.
e) Usar o calor liberado com os gases pela chamin para mover um outro gerador.

23 - (Enem MEC/2009) Para que todos os rgos do corpo humano funcionem em boas condies, necessrio que
a temperatura do corpo fique sempre entre 36 C e 37 C. Para manter-se dentro dessa faixa, em dias de muito calor
ou durante intensos exerccios fsicos, uma srie de mecanismos fisiolgicos acionada.

Pode-se citar como o principal responsvel pela manuteno da temperatura corporal humana o sistema

a) digestrio, pois produz enzimas que atuam na quebra de alimentos calricos.
b) imunolgico, pois suas clulas agem no sangue, diminuindo a conduo do calor.
c) nervoso, pois promove a sudorese, que permite perda de calor por meio da evaporao da gua.
d) reprodutor, pois secreta hormnios que alteram a temperatura, principalmente durante a menopausa.
e) endcrino, pois fabrica anticorpos que, por sua vez, atuam na variao do dimetro dos vasos perifricos.

24 - (UDESC SC/2006) Muitas bebidas alcolicas, como a cerveja e o vinho, so obtidas atravs da fermentao
(oxidao da glicose em lcool etlico). O lcool etlico acima de concentraes de 0,46g/litro de sangue provoca
alteraes no organismo humano e o risco de acidentes automobilsticos duas vezes maior. Nas estradas, a Polcia
Rodoviria possui o bafmetro para utilizar em motoristas com suspeita de embriaguez. Quando o motorista sopra no
bafmetro, o lcool presente no bafo oxidado a cido actico, conforme mostra a reao no balanceada abaixo.
O H ) SO ( Cr SO K COOH CH
SO H O Cr K OH CH CH
2 3 4 2 4 2 3
4 2 7 2 2 2 3
+ + +
+ +

Em relao a isso:
desenhe a estrutura de Lewis para a molcula CH
3
COOH;

25 - (UFRJ/2009) Uma festa de aniversrio foi decorada com dois tipos de
bales. Diferentes componentes gasosos foram usados para encher cada tipo
de balo. As figuras observadas representam as substncias presentes no
interior de cada balo.

a) O elemento , que aparece no balo II, est localizado no 2 perodo, grupo
14. Um de seus istopos apresenta 8 nutrons. Calcule o nmero de massa
desse istopo.
b) Identifique, no balo II, as molculas que apresentam ligaes do tipo polar e as molculas que apresentam ligaes
do tipo apolar.

544












PROF.:Bencio Dias Honorio


C
C
o
o
n
n
t
t
e
e

d
d
o
o
s
s
:
:

- Dissociao e ionizao
- Conceitos de cido, base e sal pela teoria de Arrhenius
- cidos Bases Sais xidos
Perxidos Superxidos Carbetos Hidretos
caractersticas, nomenclatura e classificao.
- Reaes Inorgnicas



M
M
i
i
n
n
i
i
s
s
t
t
r
r
a
a
d
d
o
o
s
s
d
d
u
u
r
r
a
a
n
n
t
t
e
e
o
o
m
m

s
s
d
d
e
e
s
s
e
e
t
t
e
e
m
m
b
b
r
r
o
o














D Da at ta a d da a E En nt tr re eg ga a : : _ __ __ __ __ __ __ __ _/ /_ __ __ __ __ __ __ __ __ __ __ _/ /2 20 01 13 3
LISTA 5 QUMICA

545

01 - (UFG GO/2009) Para produzir cido sulfrico em um laboratrio foi montada uma aparelhagem, representada
pelo esquema a seguir:


Aps o faiscador ser acionado, ocorre a queima do enxofre no bquer, seguido do aumento do nvel de gua no interior
do bquer invertido.

Considerando o exposto,
a) escreva as equaes balanceadas que representam as reaes qumicas que ocorrem no experimento.
b) explique porque ocorre o aumento do nvel de gua.

02 - (UERJ/2009) O cido ntrico um composto muito empregado em indstrias qumicas, principalmente para a
produo de corantes, fertilizantes, explosivos e nylon. Um processo industrial de obteno do cido ntrico consiste na
seguinte reao:
NaNO
3 (s)
+ H
2
SO
4 (aq)
HNO
3 (aq)
+ NaHSO
4 (aq)

Escreva os nomes dos reagentes empregados nesse processo e apresente a frmula estrutural plana do cido ntrico.

03 - (UFOP MG/2008) Considere a seguinte lista de elementos:

Cl S P Si Al Mg Na
F O N C B Be Li


Utilizando somente os elementos dessa lista, fornea o smbolo ou nome para:
a) o elemento mais eletronegativo.
b) o elemento que forma comumente um on de configurao eletrnica 1s
2
2s
2
2p
6
e carga 2.
c) um elemento que forma um xido anfotrico.
d) um elemento X que forma xidos com frmula XO e XO
2
, com nmero par de eltrons.
e) um elemento encontrado em protenas, mas no em carboidratos.

04 - (UFRJ/2008) A queima do enxofre presente na gasolina e no leo diesel gera dois anidridos que, combinados
com a gua da chuva, formam seus cidos correspondentes.
Escreva a frmula desses cidos e indique o cido mais forte. Justifique sua indicao.

05 - (Unimontes MG/2007) A gua oxigenada (H
2
O
2
) um produto muito utilizado na indstria de tecido e papel. Nas
etapas finais de preparao industrial desse produto, tem-se a reao representada a seguir:
) s ( BaO ) g ( O 2 / 1 ) s ( BaO
2 2
+
Aps a reao, faz-se a reao do BaO
2
com cido sulfrico (H
2
SO
4
) aquoso, para formar a gua oxigenada.
Baseando-se nas informaes fornecidas, responda:
a) O composto BaO tem carter cido ou bsico? Justifique usando uma equao qumica balanceada.
b) Qual a funo qumica do composto BaO
2
?
c) Represente, por equao qumica, a reao do BaO
2
(s) com o H
2
SO
4
(aq).




Caderno de Atividades

Disciplina:
Qumica

Professor(a):
Bencio
Aluno:
3 ano
Ensino Mdio
Data de Recebimento:
_____/_____/_____
Lista 05

Data Entrega:
_____/_____/_____

546
06 - (UFTM MG/2007)
Savanizao da Amaznia: Pesquisadores brasileiros do Inpe (Instituto Nacional de Pesquisas Espaciais) indicam o
tamanho do estrago que o aquecimento global vai fazer na Amaznia. Os resultados dos estudos so alarmantes, pois
at o final deste sculo 18% da rea que hoje mata deve virar uma vegetao rala, semelhante ao cerrado.
(www1.folha.uol.com.br/folha/ciencia/
ult306u16367.shtml 24.04.2007. Adaptado)
Na queima de florestas e combustveis fsseis so liberados gases responsveis pelo efeito estufa e que tambm
contribuem para a formao de chuva cida. Os principais poluentes gerados so: gs carbnico, monxido de
carbono, xidos de enxofre e xidos de nitrognio (NO
x
).
Dentre os poluentes citados, o principal responsvel pelo aquecimento global e um dos poluentes classificados como
xido cido so, respectivamente,
a) CO e NO
2
.
b) CO e SO
2
.
c) CO
2
e NO.
d) CO
2
e NO
2
.
e) SO
2
e NO.

07 - (UFG GO/2007) Superxido de potssio slido, KO
2
(s), comumente empregado em mscaras protetoras
contra gases. Esse superxido remove tanto o vapor dgua exalado quanto o gs carbnico. O vapor dgua
removido aps reagir com o superxido. O gs carbnico removido aps reagir com um dos produtos da reao
anterior. Escreva as equaes qumicas que representam as reaes qumicas envolvidas.

08 - (UFU MG/2006) Uma pesquisa sobre produtos qumicos e suas aplicaes no cotidiano forneceu as seguintes
informaes:

dente. do esmalte o
fortalecer para dental Creme
(II) estanho de Fluoreto
estomacal. acidez de excesso o
combater para Anticido
magnsio de Hidrxido
mascar. de gomas e balas
tes, refrigeran em Acidulante
fosfrico cido
Aplicao Qumico Produto

-


Considerando as informaes acima,
a) escreva, respectivamente, a frmula dos produtos qumicos pesquisados.
b) indique a que funo qumica pertence cada um destes compostos, justificando sua resposta, escrevendo a equao
de reao qumica, segundo a Teoria de Arrhenius.

09 - (UDESC SC/2006) As funes bsicas de nosso organismo necessitam de espcies inicas para o seu adequado
funcionamento. Os ons Na
+
, por exemplo, encontram-se presentes nos fluidos externos das clulas e o on K
+
,
presente no fluido interno das clulas. Juntos so responsveis por manter a presso osmtica adequada e esto
normalmente associados presena do Cl

, que atua para manter a neutralidade das cargas. Outro ction de


importncia fundamental o Ca
2+
, principal integrante dos ossos e dos dentes, que se encontra normalmente na forma
de fosfato
3
4
PO ou carbonato
2
3
CO .
Escreva as frmulas qumicas dos compostos formados pelos pares de ons (ction e nion) associados no texto, e d
os nomes deles.

10 - (UNESP SP/2006) O cloro (grupo 17 da classificao peridica) um gs irritante e sufocante. Misturado gua,
reage produzindo os cidos clordrico e hipocloroso que age como desinfetante, destruindo ou inativando os
microorganismos.
a) Identifique os reagentes e os produtos desta reao e fornea suas frmulas qumicas.
b) A gua de lavadeira uma soluo aquosa de hipoclorito e o cido muritico uma soluo concentrada de cido
clordrico. Ambos podem ser utilizados separadamente na limpeza de alguns tipos de piso. Explique a inconvenincia,
para a pessoa que faz a limpeza, de utilizar uma mistura destes dois produtos.

11 - (UFLA MG/2002) Dados os compostos binrios Na
2
O, CO, N
2
O
5
, MgO e OF
2
, responda as questes a seguir.
a) Quais desses compostos so classificados como xidos? Justifique.
b) Classifique os xidos de acordo com a reatividade em relao gua, cidos e bases.
c) Escreva a equao qumica balanceada da reao entre Na
2
O e N
2
O
5
.





547
12 - (UFRJ/1998) A crosta terrestre composta principalmente por sais e xidos. Nestes compostos, alguns dos
metais mais abundantes so: Na, Mg, Al, K e Ca.
a) Identifique o metal de maior raio atmico, dentre os citados, e escreva a frmula qumica do composto formado pelo
ction desse metal e o nion sulfato.
b) Apresente a frmula qumica do nico xido anftero formado por estes metais.

13 - (CEFET PR/2009) O fosfato de clcio um importante componente dos ossos e dentes do corpo, encontra-se
na forma de minerais. O bicarbonato de sdio um anticido estomacal, neutraliza o excesso de cido clordrico no
suco gstrico. O dihidrogenopirofosfato de sdio misturado com amido e aromatizante no preparo de pudins
instantneos. O tiossulfato de sdio usado para remover o gosto desagradvel da gua potvel fortemente clorada.
Indique a alternativa que contm as frmulas corretas das substncias citadas no texto.

a) Ca
3
(PO
4
)
2
; Na
2
CO
3
; Na
2
P
2
O
7
; Na
2
S
2
O
3

b) Ca(PO
4
); NaHCO
3
; Na
4
P
2
O
7
; Na
2
S
2
O
3

c) Ca
3
(PO
4
)
2
; NaHCO
3
; Na
2
H
2
P
2
O
7
; Na
2
S
2
O
3

d) Ca
3
(PO
4
)
2
; NaHCO
3
; Na
2
H
2
P
2
O
7
; Na
2
S
2
O
4

e) Ca
3
(PO
4
)
2
; NaHCO
3
; Na
4
P
2
O
7
; Na
2
S
2
O
4


14 - (UEM PR/2010) Dadas as reaes abaixo, assinale o que for correto.

Pb(NO
3
)
2
(aq) + 2KI(aq) PbI
2(s)
+ 2KNO
3
(aq) (I)
AgNO
3
(aq) + KCl(aq) AgCl
(s)
+ KNO
3
(aq) (II)
HCl(aq) + NaOH(aq) H
2
O(l) + NaCl(aq) (III)
2Ca
(s)
+ O
2
(g) 2CaO
(s)
(IV)
Zn
(S)
+ 2HBr(aq) ZnBr
2
(aq) + H
2
(g) (V)
2H
2
O
2
(aq) + luz 2H
2
O(l) + O
2
(g) (VI)

01. A reao III uma reao de oxirreduo.
02. As reaes I e II podem ser consideradas reaes de dupla troca.
04. A reao IV uma reao de adio.
08. A reao V uma reao de deslocamento e tambm uma reao de oxirreduo.
16. A reao VI uma reao de dupla troca.

15 - (UDESC SC/2008) Determinadas reaes ocorrem devido ao externa de agentes fsicos, como o calor, a luz
e a eletricidade. Abaixo so dados alguns exemplos desses tipos de reaes.

I. O fermento qumico (bicarbonato de amnio), tambm conhecido como carbonato cido de amnio, quando
misturado massa, para preparao de um bolo, e levado ao forno, decompe-se, produzindo gs dentro da
massa, o que deixa o bolo crescido e fofo.
II. Os vegetais sintetizam os chamados polissacardios, por meio da fotossntese. Essa reao indispensvel
manuteno da vida sobre a Terra. O mecanismo da reao foi esclarecido pelo cientista Melvin Calvin, o que lhe
valeu o Prmio Nobel de Qumica, em 1961.
III. A reao de decomposio do cloreto de sdio slido realizada para se obter sdio metlico e cloro gasoso,
utilizando para isso a eletricidade.
IV. O xido de clcio, tambm denominado cal viva ou cal virgem, pode ser preparado pela decomposio trmica do
carbonato de clcio.

Com base na anlise dos enunciados acima, assinale a alternativa que classifica corretamente as reaes, de acordo
com o agente fsico que provocou tal reao; respectivamente:
a) pirlise; fotlise; pirlise; eletrlise.
b) fotlise; pirlise; eletrlise; fotlise.
c) pirlise; fotlise; eletrlise; pirlise.
d) pirlise; eletrlise; fotlise; pirlise.
e) eletrlise; fotlise; pirlise; fotlise.

16 - (UFV MG/2008) As equaes incompletas mostram algumas das propriedades dos xidos:
I. + (aq) SO H (s) CaO
4 2

II. + (aq) HCl (s) FeO
III. + (s) O Na (aq) SO
2 3

IV. + (aq) KOH (s) O P
5 2





548
Considerando as reaes completas, os produtos principais das equaes so, respectivamente:
a) CaSO
4 (aq)
, FeCl
3 (aq)
, Na
2
S
2
O
7 (aq)
e KP
2
O
6 (aq)
.
b) CaSO
4 (aq)
, FeCl
(aq)
, Na
2
SO
3 (aq)
e KP
2
O
6 (aq)
.
c) CaSO
4 (aq)
, FeCl
2 (aq)
, Na
2
SO
4 (aq)
e K
3
PO
4 (aq)
.
d) CaSO
3 (aq)
, FeCl
2 (aq)
, Na
2
SO
3 (aq)
e K
3
PO
4 (aq)
.

17 - (UFMG/2008) Num laboratrio, foram feitos testes para avaliar a reatividade de trs metais cobre, Cu,
magnsio, Mg, e zinco, Zn.
Para tanto, cada um desses metais foi mergulhado em trs solues diferentes uma de nitrato de cobre, Cu(NO
3
)
2
,
uma de nitrato de magnsio, Mg(NO
3
)
2
, e uma de nitrato de zinco, Zn(NO
3
)
2
.
Neste quadro, esto resumidas as observaes feitas ao longo dos testes:

Considerando-se essas informaes, CORRETO afirmar que a disposio dos trs metais testados, segundo a
ordem crescente de reatividade de cada um deles,

a) Cu / Mg / Zn .
b) Cu / Zn / Mg .
c) Mg / Zn / Cu .
d) Zn / Cu / Mg .

18 - (UEG GO/2007)



A uria e o cido rico podem ser encontrados na urina de alguns animais, como resultado do metabolismo dos
compostos nitrogenados. Esses dois compostos, por causa do alto teor de nitrognio, so empregados,
respectivamente, em nutrio animal e fertilizantes. Suas estruturas esto mostradas na figura a seguir.
N
N
N
NH
O
H
H
O
O
H
H
2
N NH
2
O
Uria
cido rico



Com base no cartum, nas informaes fornecidas pelo texto e nas representaes das estruturas qumicas do cido
rico e da uria, CORRETO afirmar:
a) O cartum sugere que a urina promove a oxidao do ferro e do alumnio. Os dois xidos formados nesse processo,
quando em contato com a gua, levam a um pH alcalino.
b) A frmula molecular do acido rico C
5
H
6
N
4
O
3
.
c) A excreo da uria na urina ocorre por sua alta solubilidade em gua, o que explicado pelas fortes interaes de
van der Walls entre o soluto e o solvente.
d) A uria pode ser obtida pelo aquecimento do hidrxido de amnio.




549
19 - (UFG GO/2008) Um dos processos de obteno do cobre a partir da calcopirita, um sulfeto misto de ferro e
cobre, o pirometalrgico. Numa determinada etapa desse processo, o minrio aquecido ao ar, para produzir dois
mols de sulfeto de cobre (II), juntamente com dois mols de xido de ferro (II) e dois mols de dixido de enxofre gasoso.
Essa reao qumica pode ser representada pela seguinte equao:
a)
(g) 2 (s) (s)
(g) 2 (s) 2
SO 2 FeO 2 Cu
O 3 CuFeS
+ +
+

b)
(g) 2 (s) (s)
(g) 2 (s) (s)
SO 2 FeO 2 CuS
O 3 CuS FeS
+ +
+ +

c)
(g) 2 (s) (s)
(g) 2 (s) 2
2SO FeO 2 CuS
O 3 CuFeS 2
+ +
+

d)
(g) 2 (s) (s)
(g) 2 (s) 2
2SO FeO 2 CuS 2
O 3 CuFeS 2
+ +
+

e)
(g) 2 (s) (s)
(g) 2 (s) (s)
2SO FeO 2 CuS 2
O 3 CuS FeS
+ +
+ +


20 - (Enem MEC/2009) Sabes so sais de cidos carboxlicos de cadeia longa utilizados com a finalidade de
facilitar, durante processos de lavagem, a remoo de substncias de baixa solubilidade em gua, por exemplo, leos
e gorduras. A figura a seguir representa a estrutura de uma molcula de sabo.



Em soluo, os nions do sabo podem hidrolisar a gua e, desse modo, formar o cido carboxlico correspondente.
Por exemplo, para o estearato de sdio, estabelecido o seguinte equilbrio:

+ + OH COOH ) CH ( CH O H COO ) CH ( CH
16 2 3 2 16 2 3


Uma vez que o cido carboxlico formado pouco solvel em gua e menos eficiente na remoo de gorduras, o pH
do meio deve ser controlado de maneira a evitar que o equilbrio acima seja deslocado para a direita.

Com base nas informaes do texto, correto concluir que os sabes atuam de maneira
a) mais eficiente em pH bsico.
b) mais eficiente em pH cido.
c) mais eficiente em pH neutro.
d) eficiente em qualquer faixa de pH.
e) mais eficiente em pH cido ou neutro.

21 - (Enem MEC/2009) A abertura e a pavimentao de rodovias em zonas rurais e regies afastadas dos centros
urbanos, por um lado, possibilita melhor acesso e maior integrao entre as comunidades, contribuindo com o
desenvolvimento social e urbano de populaes isoladas. Por outro lado, a construo de rodovias pode trazer
impactos indesejveis ao meio ambiente, visto que a abertura de estradas pode resultar na fragmentao de habitats,
comprometendo o fluxo gnico e as interaes entre espcies silvestres, alm de prejudicar o fluxo natural de rios e
riachos, possibilitar o ingresso de espcies exticas em ambientes naturais e aumentar a presso antrpica sobre os
ecossistemas nativos.
BARBOSA, N. P. U.; FERNANDES, G. W.
A destruio do jardim. Scientific American Brasil.
Ano 7, nmero 80, dez. 2008 (adaptado).

Nesse contexto, para conciliar os interesses aparentemente contraditrios entre o progresso social e urbano e a
conservao do meio ambiente, seria razovel

a) impedir a abertura e a pavimentao de rodovias em reas rurais e em regies preservadas, pois a qualidade de vida
e as tecnologias encontradas nos centros urbanos so prescindveis s populaes rurais.
b) impedir a abertura e a pavimentao de rodovias em reas rurais e em regies preservadas, promovendo a migrao
das populaes rurais para os centros urbanos, onde a qualidade de vida melhor.
c) permitir a abertura e a pavimentao de rodovias apenas em reas rurais produtivas, haja vista que nas demais
reas o retorno financeiro necessrio para produzir uma melhoria na qualidade de vida da regio no garantido.
d) permitir a abertura e a pavimentao de rodovias, desde que comprovada a sua real necessidade e aps a
realizao de estudos que demonstrem ser possvel contornar ou compensar seus impactos ambientais.
e) permitir a abertura e a pavimentao de rodovias, haja vista que os impactos ao meio ambiente so temporrios e
podem ser facilmente revertidos com as tecnologias existentes para recuperao de reas degradadas.



550
22 - (Enem MEC/2009) Cerca de 1% do lixo urbano constitudo por resduos slidos contendo elementos txicos.
Entre esses elementos esto metais pesados como o cdmio, o chumbo e o mercrio, componentes de pilhas e
baterias, que so perigosos sade humana e ao meio ambiente.
Quando descartadas em lixos comuns, pilhas e baterias vo para aterros sanitrios ou lixes a cu aberto, e o
vazamento de seus componentes contamina o solo, os rios e o lenol fretico, atingindo a flora e a fauna. Por serem
bioacumulativos e no biodegradveis, esses metais chegam de forma acumulada aos seres humanos, por meio da
cadeia alimentar. A legislao vigente (Resoluo CONAMA no 257/1999) regulamenta o destino de pilhas e baterias
aps seu esgotamento energtico e determina aos fabricantes e/ou importadores a quantidade mxima permitida
desses metais em cada tipo de pilha/bateria, porm o problema ainda persiste.
Disponvel em: http://www.mma.gov.br.
Acesso em: 11 jul. 2009 (adaptado).

Uma medida que poderia contribuir para acabar definitivamente com o problema da poluio ambiental por metais
pesados relatado no texto seria

a) deixar de consumir aparelhos eltricos que utilizem pilha ou bateria como fonte de energia.
b) usar apenas pilhas ou baterias recarregveis e de vida til longa e evitar ingerir alimentos contaminados,
especialmente peixes.
c) devolver pilhas e baterias, aps o esgotamento da energia armazenada, rede de assistncia tcnica especializada
para repasse a fabricantes e/ou importadores.
d) criar nas cidades, especialmente naquelas com mais de 100 mil habitantes, pontos estratgicos de coleta de baterias
e pilhas, para posterior repasse a fabricantes e/ou importadores.
e) exigir que fabricantes invistam em pesquisa para a substituio desses metais txicos por substncias menos
nocivas ao homem e ao ambiente, e que no sejam bioacumulativas.

TEXTO: 1 - Comum questo: 23

Muitas bebidas alcolicas, como a cerveja e o vinho, so obtidas atravs da fermentao (oxidao da glicose em
lcool etlico). O lcool etlico acima de concentraes de 0,46g/litro de sangue provoca alteraes no organismo
humano e o risco de acidentes automobilsticos duas vezes maior. Nas estradas, a Polcia Rodoviria possui o
bafmetro para utilizar em motoristas com suspeita de embriaguez. Quando o motorista sopra no bafmetro, o lcool
presente no bafo oxidado a cido actico, conforme mostra a reao no balanceada abaixo.

O H ) SO ( Cr SO K COOH CH
SO H O Cr K OH CH CH
2 3 4 2 4 2 3
4 2 7 2 2 2 3
+ + +
+ +


23 - (UDESC SC/2006)
Em relao a isso:
c) escreva o nome das seguintes molculas: K
2
SO
4
, H
2
SO
4
, K
2
Cr
2
O
7
.

TEXTO: 2 - Comum questo: 24

A produo de energia nas usinas de Angra 1 e Angra 2 baseada na fisso nuclear de tomos de urnio radioativo
238
U. O urnio obtido a partir de jazidas minerais, na regio de Caetit, localizada na Bahia, onde beneficiado at a
obteno de um concentrado bruto de U
3
O
8
, tambm chamado de yellowcake.
O concentrado bruto de urnio processado atravs de uma srie de etapas at chegar ao hexafluoreto de urnio,
composto que ser submetido ao processo final de enriquecimento no istopo radioativo
238
U, conforme o esquema a
seguir.

551
24 - (UFRJ/2007) Com base no esquema:
a) Apresente os nomes do oxicido e da base utilizados no processo.
b) Indique os nmeros de oxidao do tomo de urnio nos compostos U
3
O
8
e (NH
4
)
2
U
2
O
7
.

TEXTO: 3 - Comum questo: 25

O sdio uma substncia extremamente reativa e perigosa, podendo pegar fogo em contato com o ar:
(2) (s) O 2Na (g) O (s) 4Na
2 2
+
e reagir violentamente com a gua:
(3) (g) H (s) 2NaOH (l) O 2H (s) 2Na
2 2
+ +
um elemento qumico considerado essencial vida humana. Quando combinado a outras substncias, utilizado,
por exemplo, na produo de papel, de sabo e no tratamento de guas.

25 - (UFRN/2009) Os compostos Na
2
O e NaOH, obtidos, respectivamente, nas reaes (2) e (3), podem ser
classificados como:
Lewis de fraca base inico e anftero xido
gua em
insolvel e forte base
covalente e bsico xido
gua em
solvel e fraca base
covalente e anftero xido
Arrhenius de forte base inico e bsico xido
NaOH O Na
) d
) c
) b
) a
2









552









PROF.:Bencio Dias Honorio


C
C
o
o
n
n
t
t
e
e

d
d
o
o
s
s
:
:

Relaes de Massa
Estequiometria
- Tipos de frmulas
frmula percentual, mnima e molecular.
Os coeficientes e a quantidade de substncia
-Reagente em excesso e reagente limitante.
- Rendimento de uma reao qumica.


M
M
i
i
n
n
i
i
s
s
t
t
r
r
a
a
d
d
o
o
s
s
d
d
u
u
r
r
a
a
n
n
t
t
e
e
o
o
m
m

s
s
d
d
e
e
o
o
u
u
t
t
u
u
b
b
r
r
o
o















D Da at ta a d da a E En nt tr re eg ga a : : _ __ __ __ __ __ __ __ _/ /_ __ __ __ __ __ __ __ __ __ __ _/ /2 20 01 13 3
LISTA 6 QUMICA

553

01 - (PUC RJ/2006) O elemento boro tem nmero atmico 5, faz parte do terceiro grupo de elementos representativos
e sua massa atmica 10,8u.m.a.. Sendo o boro natural constitudo por dois istopos,
11
B e
10
B:
a) calcule a abundncia relativa dos dois istopos do elemento boro.
b) calcule o nmero de prtons, de nutrons e de eltrons do nucldeo neutro
11
B.
c) calcule a porcentagem em massa do elemento boro no brax, cuja frmula Na
2
B
4
O
7
.10H
2
O.

02 - (UNESP SP/2007) Como o dixido de carbono, o metano exerce tambm um efeito estufa na atmosfera. Uma
das principais fontes desse gs provm do cultivo de arroz irrigado por inundao. Segundo a Embrapa, estima-se que
esse tipo de cultura, no Brasil, seja responsvel pela emisso de cerca de 288 Gg (1Gg = 1 10
9
gramas) de metano
por ano.
Calcule o nmero de molculas de metano correspondente.

Massas molares, g.mol
1
: H=1 e C=12.
Constante de Avogadro = 6,0 10
23
.

03 - (UFMS/2009) Contribuindo para o aumento dos ndices de poluio atmosfrica, os motores do ciclo diesel
lanam no ar que respiramos diversos gases txicos, entre eles o dixido de enxofre e o monxido de carbono. A
anlise de uma amostra dos gases emitidos por um motor a diesel mostrou que ela continha 0,5 mols de dixido de
enxofre e 3,0 10
23
molculas de monxido de carbono. A massa total, em gramas, referente aos gases citados igual
a
(Dados: Massas atmicas em g/mol: C = 12; O = 16; S = 32)

a) 12,8.
b) 14,4.
c) 24,4.
d) 40,4.
e) 46,0.

04 - (UFMS/2009) Sabendo-se que o nmero de Avogadro igual a 6,0210
23
e dadas as massas atmicas dos
elementos qumicos em g/mol: C=12, O=16, H=1, N=14, assinale a(s) proposio(es) correta(s).

01. A cafena um alcalide estimulante do sistema nervoso central, encontrado nos gros de caf, nas folhas de
certos tipos de ch e em refrigerantes base de cola. Sabendo-se que 60mL de caf de coador possui 44,4mg de
cafena cuja frmula molecular C
8
H
10
N
4
O
2
, a quantidade de mols de molculas de cafena presentes em uma
xcara de 100 mL de caf comum ser de 3,810
4
mols.
02. A creatina, de frmula molecular C
4
H
9
N
3
O
2
, uma substncia comumente ingerida pelos freqentadores de
academias de musculao, pois derivada de um aminocido presente nas clulas musculares e serve para repor
energia quando h fadiga muscular. Em 1kg de creatina, haver aproximadamente 7,63mols e 4,5910
24

molculas.
04. Considerando-se que a dose diria recomendada de vitamina C (C
6
H
8
O
6
) de aproximadamente 70mg, quando
uma pessoa ingere essa massa de vitamina C, significa que o nmero de tomos de carbono ingeridos foi de,
aproximadamente, 2,3910
23
tomos de carbono.
08. Feromnios so hormnios sexuais secretados pelas fmeas de muitos insetos. Normalmente, a quantidade
secretada de aproximadamente 110
12
g. O nmero de molculas existentes nessa massa de feromnio de
frmula molecular C
19
H
38
O de cerca de 4,310
15
.
16. Num dado mao de cigarros, consta a informao de que o produto contm milhares de substncias txicas, entre
elas a nicotina (C
10
H
14
N
2
), com 0,65mg dessa substncia por unidade. Sabe-se que a dependncia do cigarro se
deve presena da nicotina, e o teor dessa substncia refere-se fumaa gerada pela queima de um cigarro. A
quantidade em mol de molculas de nicotina presentes na fumaa de um cigarro desse mao de cerca de 410
6

mol.




Caderno de Atividades

Disciplina:
Qumica

Professor(a):
Bencio
Aluno:
3 ano
Ensino Mdio
Data de Recebimento:
_____/_____/_____
Lista 06

Data Entrega:
_____/_____/_____

554
05 - (FEPECS DF/2009) Os sulfitos so conservantes usados em sucos concentrados, frutas secas, bebidas alcolicas
e outros produtos, e podem provocar reaes alrgicas se ingeridos acima da IDA (ingesto diria aceitvel), que de
0,7 mg/kg de peso corpreo.
Adaptado de Cincia Hoje, vol 43, p 54, 2008.
O nmero de milimols de sulfito ingerido por uma criana de 40 kg que alcanou a IDA corresponde a:
a) 0,35;
b) 0,70;
c) 1,40;
d) 2,10;
e) 2,85.


06 - Macro-elementos essenciais, tais como Ca, P, Na, Mg e K, so aqueles que o organismo necessita em
quantidades iguais ou superiores a 100 mg por dia.
A quantidade mnima em mol de tomos de clcio e de fsforo recomendada para um indivduo consumir ,
aproximada e respectivamente,
a) 4,0 e 3,1.
b) 2,5 e 3,2.
c) 0,40 e 0,31
d) 2,5 x 10
3
e 3,2 x 10
3
.
e) 2,5 x 10
3
e 2,6 x 10
3
.

07 - (UESPI/2010) A butadiona (C
4
H
6
O
2
) uma substncia orgnica lquida, amarela, voltil, com cheiro de queijo e
utilizada na fabricao da manteiga. Quando se usa 4,3 g dessa substncia na preparao da manteiga, quantos
tomos de carbono esto sendo adicionados?

Dados: Massas molares em g . mol
1
: C = 12; H = 1; O = 16. Constante de Avogadro = 6 10
23
mol
1
.
a) 1,2 10
23
tomos
b) 12 tomos
c) 12 10
23
tomos
d) 6 10
22
tomos
e) 6 10
23
tomos

08 - (UNESP SP/2005) Um composto orgnico destilado da madeira possui massa molar de 32,4 g.mol
1
e a
composio: 37,5% de carbono, 12,6% de hidrognio e 49,9% de oxignio.
Dados:
Massas atmicas: C = 12,0 u, H = 1,01 u, O = 16,0 u
Nmeros atmicos: C = 6, O = 8 e H = 1,

a) determine a frmula molecular do composto orgnico e deduza o grupo funcional;
b) escreva a estrutura de pontos (estrutura de Lewis) do composto e d o nome da figura geomtrica em torno do
tomo de carbono.

09 - (UFU MG/2005) Um xido de nitrognio foi analisado e apresentou as seguintes porcentagens em massa: 25,9%
de nitrognio e 74,1% de oxignio.

Tendo em vista as informaes apresentadas, faa o que se pede.
D a frmula emprica deste composto, demonstrando os clculos utilizados.
Dados: N = 14; O = 16.

10 - (UFG GO/2003) O Parque Nacional da Emas, localizado no extremo sudoeste do Estado, caracterizado pelo
agrupamento de cupinzeiros, que servem como local de desenvolvimento de larvas de vaga-lumes. Pela emisso de
luz, elas atraem outros insetos que lhes servem de alimento. Esse fenmeno de emisso de luz chamado de
bioluminescncia e ocorre, principalmente, pela oxidao de uma substncia qumica conhecida por luciferina,
representada a seguir:
N
S
S
N
HO
COOH

Determine a frmula molecular, a massa molecular e a composio percentual da luciferina.





555
11 - (UFMS/2008) O alumnio o elemento metlico mais abundante da crosta terrestre. Sua fonte natural comercial
a bauxita (Al
2
O
3
.xH
2
O) e, para a transformao desse minrio no metal (Al
0
), utiliza-se um mtodo eletroltico
denominado processo de Hall, que utiliza como fundente a criolita. A adio de criolita faz com que a temperatura de
fuso baixe de 2050C para 950C, reduzindo significativamente o consumo de energia. A criolita uma substncia
composta por 32,86% de Na, 12,86% de Al e 54,29% de F, sua massa molar igual a 210 g/mol. Qual a sua frmula
molecular?
Dados: MA(Na)=23u;MA(Al)=27u;MA(F)=19u.
a) Na
1
Al
2
F
6
.
b) Na
2
AlF
5
.
c) Na
3
Al
2
F
4
.
d) Na
2
Al
3
F
5
.
e) Na
3
AlF
6
.

12 - (UFMS/2009) O Brasil tem vantagens acentuadas na produo de etanol de primeira gerao, feito a partir da
fermentao da sacarose, mas h vrios desafios que precisamos vencer para melhorar sua produtividade... Tambm
h oportunidades importantes de desenvolvimento tecnolgico do etanol de segunda gerao, produzido a partir da
celulose....
(Revista Pesquisa FAPESP, julho de 2008).

A obteno de etanol, a partir de sacarose (acar) por fermentao, pode ser representada pela seguinte equao
no balanceada:
C
12
H
22
O
11
(s) + H
2
O(l) C
2
H
5
OH(l) + CO
2
(g)

Considerando-se que o processo tenha rendimento de 75% e que o etanol seja anidro (puro), calcule a massa, em
toneladas (t), de acar necessria para produzir um volume de 402,5 m
3
de etanol.
(Dados: Densidade do etanol = 0,8 t/m
3
; Massa molar da sacarose = 342g/mol; Massa molar do etanol = 46 g/mol).

13 - (UFC CE/2009) O cido sulfrico um dos produtos qumicos de maior importncia comercial, sendo utilizado
como matria-prima para diversos produtos, tais como fertilizantes, derivados de petrleo e detergentes. A produo
de cido sulfrico ocorre a partir de trs etapas fundamentais:

I. combusto do enxofre para formar dixido de enxofre;
II. converso do dixido de enxofre em trixido de enxofre a partir da reao com oxignio molecular;
III. reao do trixido de enxofre com gua para formar cido sulfrico.

Com base nessas informaes, responda o que se pede a seguir.
a) Apresente as equaes qumicas balanceadas para as reaes das etapas I, II e III.
b) Determine a quantidade mxima, em gramas, de cido sulfrico que pode ser produzido a partir da combusto
completa de 1.605 g de enxofre.

14 - (UEG GO/2009) A glicose (C
6
H
12
O
6
) sofre combusto completa na presena de oxignio, produzindo CO
2
e H
2
O.
Sabendo-se que nessa reao o sistema era formado por 216 g de cada um dos reagentes, determine

a) o reagente limitante na reao.
b) a massa residual, em gramas, do reagente em excesso.
c) a presso, em atm, exercida pelo gs carbnico, caso seja coletado em um recipiente de 200 mL, a 27 C.
Dado: R = 0,082 atm.L.mol
-1
.K
-1


15 - (UFRRJ/2008) O monxido de carbono (CO) um gs incolor, inodoro e altamente txico. O monxido de
carbono, ao ser inalado, combina-se com a hemoglobina nos glbulos vermelhos do sangue, inutilizando-os para sua
funo essencial, que transportar oxignio (O
2
) pelo corpo. Sem oxignio, morremos rapidamente. No ms de agosto
de 2007, na zona oeste do municpio do Rio de Janeiro, houve um acidente num banheiro de um Apart Hotel que levou
ao bito duas vtimas pela inalao do monxido de carbono, resultante da combusto da mistura de metano (CH
4
) e
etano (C
2
H
6
), em um aquecedor. Com base na equao da combusto incompleta do etano (C
2
H
6
), representada
abaixo:

O H 3 CO 2 O 5/2 H C
2 2 6 2
+ +

Determine:
a) O volume de CO produzido, nas CNTP, a partir de 0,5 mol de etano.
Dado: V(CNTP) = 22,4 L/mol.

b) O nmero de molculas de CO produzido a partir de 40 g de oxignio.
Dados: massa atmica (g/mol). O = 16 e Nmero de
Avogadro = 6.10
23
molculas/mol.

556
16 - (UFU MG/2008) A equao abaixo representa a reao do processo de fermentao para se obter etanol puro, a
partir da sacarose.

2 5 2 2 11 22 12
4CO OH H 4C O H O H C + +

Pede-se:
a) Calcule a massa (em gramas) de sacarose necessria para produzir um volume de 50 litros de etanol. Considere o
rendimento do processo de 100%. Dados: densidade do etanol = 0,8 g cm
3
.
b) Qual o volume de gs carbnico produzido na combusto total de 1 mol de etanol nas Condies Normais de
Temperatura e Presso (CNTP)?

17 - (UFPE/2008) A preocupao com o meio ambiente levou ao desenvolvimento de metodologias verdes (pouco
poluentes), que procuram reduzir a produo de rejeitos e a utilizao de reagentes txicos. Um exemplo de
metodologia verde a sntese descrita abaixo do cido adpico, utilizado na preparao do nilon-66. Considere as
massas molares do cicloexeno e do cido adpico iguais a 82 e 146 gmol
1
, respectivamente.

N a
2
W O
4 .
4 H
2
O
[ C H
3
( n - C
8
H
1 7
) N ] H S O
4
a q u e c i m e n t o
H O
O
O
O H
+

4

H
2
O
c i d o

a d p i c o

Calcule a massa de cido adpico em kg que ser formada a partir de 41 kg de cicloexeno, considerando que o
rendimento da reao 85%. Assinale o nmero inteiro mais prximo.

18 - (UFCG PB/2008) O uso do fogo na agricultura condenado h mais de um sculo pelos manuais de conservao
do solo, pois provoca conseqncias negativas na produtividade da terra. No Brasil, desde o incio da colonizao, as
queimadas foram utilizadas para a preparao de reas para o plantio da cana de acar, do cacau e da monocultura
como um todo, sendo o fogo ateado para a destruio de campos e florestas. O socilogo Gilberto Freyre afirma que "o
canavial desvirginou todo esse mato grosso de modo mais cru pela queimada. A cultura da cana valorizou o canavial e
tornou desprezvel a mata". Para plantar a cana, o ferro e o fogo tornaram-se parceiros do senhor de engenho,
cerrando a floresta e desvirginando a Mata Atlntica. O inconseqente uso do fogo para as prticas agropastoris e para
a abertura de locais de habitao humana, sempre foi uma das identidades do Brasil que queima, que corta, que cerra
e que destri.
Adaptado de FERREIRA, Manoel Eduardo. A queimada da cana e
seu impacto socioambiental. Disponvel em: http://www.adital.com.br. Acesso em 6 jul. 2007

Admitindo que a madeira constituda de aproximadamente 50% em massa de celulose (polmero formado por
unidades repetidas de glicose (C
6
H
12
O
6
)) e admitindo ainda que a queimada de um hectare (10.000 m
2
) da mata
atlntica provoca a combusto de 30 toneladas de madeira, responda as questes abaixo considerando a combusto
completa da glicose.
a) Escreva a equao qumica balanceada da reao de combusto da glicose.
b) Determine a massa de glicose proveniente de uma rea de caatinga equivalente a um campo de futebol de 100m x
50m.
c) Determine o volume, em metros cbicos, de gs carbnico produzido, medido nas condies normais de temperatura
e presso, pela queimada de uma rea de caatinga equivalente a um campo de futebol de 100m x 50m.

19 - (UFC CE/2008) O cido fosfrico, H
3
PO
4
, pode ser produzido a partir da reao entre a fluoroapatita, Ca
5
(PO
4
)
3
F,
e o cido sulfrico, H
2
SO
4
, de acordo com a seguinte equao qumica:
(g) HF (s) CaSO 5 (l) PO H 3
(l) SO H 5 (s) F ) (PO Ca
4 4 3
4 2 3 4 5
+ +
+

Considere a reao completa entre 50,45 g de fluoroapatita com 98,12 g de cido sulfrico.
a) Qual o reagente limitante da reao?
b) Determine a quantidade mxima de cido fosfrico produzida.

20 - (UEM PR/2008) A plvora consiste em uma mistura de substncias que, em condies adequadas, reagem, com
rendimento de 100%, segundo a equao qumica abaixo:
(s) 2 (s) 3 2 2(g) (g) (g) 2
(s) (s) (s) 3
S K CO K N 2 CO 3 CO 3
S C 7 KNO 4
+ + + +
+ +

Sob condies normais de temperatura e presso, e admitindo comportamento ideal para todos os gases, considere a
reao de uma amostra de plvora contendo 1515 g de KNO
3
com 80% de pureza.
Calcule o volume total de gases produzidos na reao. Em seguida, nomeie os sais formados.

557
21 - (UFU MG/2007) O cido sulfrico, importante reagente qumico produzido e consumido pelas indstrias qumicas
em todo o mundo, muitas vezes empregado com indicativo de crescimento dos pases. O processo industrial para
obteno do cido sulfrico envolve reaes de oxi-reduo a partir da matria prima: o enxofre.
Pede-se:
a) escreva as trs reaes que mostram a obteno do cido sulfrico.
b) explique por que esse cido, quando comercializado, no 100% puro.
c) calcule a massa aproximada de enxofre puro que deve ser utilizada para produzir um quilograma do cido.

22 - (Enem MEC/2009) Para que apresente condutividade eltrica adequada a muitas aplicaes, o cobre bruto obtido
por mtodos trmicos purificado eletroliticamente. Nesse processo, o cobre bruto impuro constitui o nodo da clula,
que est imerso em uma soluo de CuSO
4
. medida que o cobre impuro oxidado no nodo, ons Cu
2+
da soluo
so depositados na forma pura no ctodo. Quanto s impurezas metlicas, algumas so oxidadas, passando
soluo, enquanto outras simplesmente se desprendem do nodo e se sedimentam abaixo dele. As impurezas
sedimentadas so posteriormente processadas, e sua comercializao gera receita que ajuda a cobrir os custos do
processo. A srie eletroqumica a seguir lista o cobre e alguns metais presentes como impurezas no
cobre bruto de acordo com suas foras redutoras relativas.

Entre as impurezas metlicas que constam na srie apresentada, as que se sedimentam abaixo do
nodo de cobre so
a) Au, Pt, Ag, Zn, Ni e Pb.
b) Au, Pt e Ag.
c) Zn, Ni e Pb.
d) Au e Zn.
e) Ag e Pb.

23 - (Enem MEC/2009) O lcool hidratado utilizado como combustvel veicular obtido por meio da destilao
fracionada de solues aquosas geradas a partir da fermentao de biomassa. Durante a destilao, o teor de etanol
da mistura aumentado, at o limite de 96% em massa.

Considere que, em uma usina de produo de etanol, 800 kg de uma mistura etanol/gua com concentrao 20% em
massa de etanol foram destilados, sendo obtidos 100 kg de lcool hidratado 96% em massa de etanol. A partir desses
dados, correto concluir que a destilao em questo gerou um resduo com uma concentrao de etanol em massa
a) de 0%.
b) de 8,0%.
c) entre 8,4% e 8,6%.
d) entre 9,0% e 9,2%.
e) entre 13% e 14%.

24 - (Enem MEC/2009) O lixo orgnico de casa constitudo de restos de verduras, frutas, legumes, cascas de ovo,
aparas de grama, entre outros , se for depositado nos lixes, pode contribuir para o aparecimento de animais e de
odores indesejveis.
Entretanto, sua reciclagem gera um excelente adubo orgnico, que pode ser usado no cultivo de hortalias, frutferas e
plantas ornamentais. A produo do adubo ou composto orgnico se d por meio da compostagem, um processo
simples que requer alguns cuidados especiais. O material que acumulado diariamente em recipientes prprios deve
ser revirado com auxlio de ferramentas adequadas, semanalmente, de forma a homogeneiz-lo. preciso tambm
umedec-lo periodicamente. O material de restos de capina pode ser intercalado entre uma camada e outra de lixo da
cozinha. Por meio desse mtodo, o adubo orgnico estar pronto em aproximadamente dois a trs meses.
Como usar o lixo orgnico em casa? Cincia
Hoje, v. 42, jun. 2008 (adaptado).

Suponha que uma pessoa, desejosa de fazer seu prprio adubo orgnico, tenha seguido o procedimento descrito no
texto, exceto no que se refere ao umedecimento peridico do composto. Nessa situao,

a) o processo de compostagem iria produzir intenso mau cheiro.
b) o adubo formado seria pobre em matria orgnica que no foi transformada em composto.
c) a falta de gua no composto vai impedir que microrganismos decomponham a matria orgnica.
d) a falta de gua no composto iria elevar a temperatura da mistura, o que resultaria na perda de nutrientes essenciais.
e) apenas microrganismos que independem de oxignio poderiam agir sobre a matria orgnica e transform-la em
adubo.






558
25 - (UNIFOR CE/2007) Ateno: Considere o AAS (cido acetilsaliclico) cuja frmula estrutural
C
O
OH
O C
O
CH
3
(massa molar 180 g/mol)



Um comprimido de AAS Tamponado contm 0,325g de ingrediente ativo. Os demais constituintes podem ser MgCO
3
,
Mg(OH
2
), Al(OH)
3
e glicinato de alumnio. Uma pessoa, pesando 72 kg, que acaba de ingerir um comprimido desse
AAS, que quantidade em mols de substncia ativa absorver por kg de seu peso?
a) 1,0 x 10
5
mol
b) 1,5 x 10
5
mol
c) 2,0 x 10
5
mol
d) 2,5 x 10
5
mol
e) 3,0 x 10
5
mol


26 - (UNICAMP SP/2008) Eles esto de volta! Omar Mitta, vulgo Rango, e sua esposa Dina Mitta, vulgo Estrondosa, a
dupla explosiva que j resolveu muitos mistrios utilizando o conhecimento qumico (vestibular UNICAMP 2002). Hoje
esto se preparando para celebrar uma data muito especial. Faa uma boa prova e tenha uma boa festa depois dela.
Embora esta prova se apresente como uma narrativa ficcional, os itens a e b em cada questo de 1 a 12 devem,
necessariamente, ser respondidos
Especialmente para as crianas, havia uma sala reservada com muitos brinquedos, guloseimas, um palhao e um
mgico. Como Rango tambm tinha problemas com acar, algumas vezes ele colocava pouco acar nas receitas.
Ao experimentar a pipoca doce, uma das crianas logo berrou: Tio Rango, essa pipoca t com pouco acar! Aquela
observao intrigou Rango, que ficou ali pensando...

a) Coloquei duas xcaras de milho na panela e, depois que ele estourou, juntei trs colheres de acar para derreter e
queimar um pouco. Se cada colher tem mais ou menos 20 gramas de acar, quantas molculas de sacarose
(C
12
H
22
O
11
) eu usei em uma panelada?
b) Eu tambm sei que parte desse acar, aps caramelizar, se decompe em gua e carbono. Se 1% desse acar
se decompe dessa forma, quantos gramas de carbono se formaram em cada panelada?
Dado: Constante de Avogadro = 6,0210
23
mol
1

Das könnte Ihnen auch gefallen